Tuyển tập 42 đề thi học sinh giỏi môn Tiếng Anh lớp 10 (có keys)

487 17.8K 140
Tuyển tập 42 đề thi học sinh giỏi môn Tiếng Anh lớp 10 (có keys)

Đang tải... (xem toàn văn)

Tài liệu hạn chế xem trước, để xem đầy đủ mời bạn chọn Tải xuống

Thông tin tài liệu

... hảI dơng Kì thi chọn học sinh giỏi tỉnh Lớp 10 thpt năm học 2012 - 2013 -Môn thi : Tiếng Anh Thời gian làm bài: 180 phút (Đề thi gồm 05 trang) đề thức Học sinh làm vào tờ giấy thi Phần trắc... THI HC SINH GII LP 10 Môn Tiếng Anh - năm học 2012-2013 A LISTENING: ( 15 points) ( point for each correct answer) Part 1 10 am Lee June 1st University Hall Health for Life Part 2: T F F T 10. .. GIO DC V O TO H TNH K THI CHN HC SINH GII TNH CP THPT NM HC 2012 - 2013 Mụn thi: TING ANH 10 Thi gian lm bi: 180 phỳt ( thi cú 08 trang, gm 11 phn) CHNH THC Lu ý: Thớ sinh khụng s dng bt k

SỞ GIÁO DỤC VÀ ĐÀO TẠO HÀ TĨNH KỲ THI CHỌN HỌC SINH GIỎI TỈNH CẤP THPT NĂM HỌC 2012 - 2013 Môn thi: TIẾNG ANH 10 Thời gian làm bài: 180 phút (Đề thi có 08 trang, gồm 11 phần) ĐỀ CHÍNH THỨC Lưu ý: → → → → Thí sinh không sử dụng bất kể tài liệu nào, kể cả từ điển. Thí sinh làm bài trực tiếp vào đề thi, ghi câu trả lời vào các ô cho sẵn ở cuối các phần. Riêng phần trắc nghiệm thí sinh chỉ ghi đáp án A, B, C hoặc D vào ô cho sẵn. Giám thị không giải thích gì thêm. Điểm của toàn bài thi (Bằng số) (Bằng chữ) Các giám khảo Số phách (Ký và ghi rõ họ tên) (Do Trưởng Ban chấm thi ghi) Giám khảo 1: Giám khảo 2: A. LISTENING PART I. Listen to the dialogue on the phone between and a man and a girl named Juliet and fill in the form. You are allowed to listen TWICE. Give your answers in the numbered spaces. Name: Juliet A. Eastman Age: (1) …………………………………… Hair color: (2) …………………………………… Eye color: (3) …………………………………… Height: (4) …………………………………… Occupation: (5) …………………………………… ………………………………………………………….. Likes: going out and having fun, sports, (6) ……………………………………… and (7) …………………………………………… Wants to meet someone who : (8) …………………………………………………, likes same (9)………………………………………………. and (10) ……………………………………… Part II. You are going to hear an expert talk about sleeping and dreaming. Listen and write True (T) or False (F) for each sentence. You are allowed to listen TWICE. Your answers True (T) 1. Women sleep more than men. 2. A sound sleeper moves less than a light sleeper. 3. Most people need 9 hours of sleep a night. 4. Reading in bed helps you sleep. 1 False (F) 5. Some people don’t dream at all. 6. The average person has about four dreams a night. 7. Not everyone can remember his or her dreams. 8. Eating before bed can give you nightmares. PART III. You are going to listen to a talk about Margaret Mead. Listen and choose the best answer A, B, C or D for each question. You are allowed to listen TWICE. 1. What was Margaret Mead’s job? A. a photographer B. a biologist C. an anthropologist D. a journalist 2. What was Margaret Mead’s main interest? A. taking photographs B. exploring new places C. how children were looked after. D. living in pour areas. 3. When did Margaret Mead go to Samoa? A. in 1901. B. in the 1920s. C. in 1938. D. in 1978. 4. Who did she interview in her first trip to Samoa? A. girls between 9 and 20 years old. B. boys and girls between 9 and 20 years old. C. women over 20 years old. D. men and women over 20 years old. 5. What was the title of Margaret Mead’s book? A. The pacific Islands. B. Teenagers around the World. C. Growing Up in New Guinea. D. Coming of Age in Samoa. 6. What was the main reason why Margaret Mead took photos? A. She liked photography. B. Cameras were not very common at that time. C. Her husband liked photos. D. It was the best way to share what she learned. 7. What is the main topic of the listening passage? A. Margaret Mead went to college in New York. B. Margaret Mead did research on the role of culture. C. Margaret Mead took photographs and wrote books. D. Margaret Mead was born in Philadelphia. Your answers 1. 2. 3. 4. 5. 6. 7. B. LEXICO – GRAMMAR PART IV. Choose the answer A, B, C or D which best fits the space in each of the following sentences. 1. _________ saying was so important that I asked everyone to stop talking and listen. A. What the woman was B. That the woman was C. The woman was D. When was the woman 2. -“Do you mind if I take a seat?” - “_____________.” A. Yes, I don’t mind B. No, do as you please C. No I mind D. Yes, do as you please 3. As the two teams left the football ground, the 100,000 _________ gave them a standing ovation. A. bystanders B. spectators C. viewers D. audiences 4 My parents lent me the money. _________, I couldn’t have afforded the trip. A. However B. Therefore C. Only if D. Otherwise 5. It is interesting to take _________ a new hobby such as collecting stamps or going fishing. A. over B. on C. in D. up 6. Jack made me _________ him next week. A. promise calling B. to promise calling C. to promise to call D. promise to call 2 7. “I passed the TOEFL test, Mom.” - “ _________.” A. All right B. Thank you C. Well done D. Good luck 8. The bad weather caused serious damage to the crop. If only it _______ warmer. A. was B. were C. has been D. had been 9. - “Eric is really upset about losing his job.” - “ Well, ____once myself, I can understand.” A. Having been fired B. Fired C. Having fired D. Being fired 10. ________ you, I’d think twice about that decision. I could be a bad move. A. Were I B. Should I be C. If I am D. If I had been 11. The teacher asked a difficult question, but finally Ted _________ a good answer. A. put up with B. keep pace with C. made way for D. came up with 12. Not only ________ to speak to him, but she also vowed never to see him again. A. she refused B. did she refuse C. she did refuse D. when she refused 13. The judge ________ the pedestrian for the accident. A. accused B. charged C. caught D. blamed 14. She had to borrow her sister’s car because hers was _________. A. out of work B. out of order C. on duty D. off work 15. We should participate in the movement _________ to conserve the natural environment. A. to organize B. organizing C. which organized D. organized 16. His brother refuses to even listen to anyone else’s point of view. He is very_________. A. open-minded B. kind-hearted C. narrow-minded D. absent-minded 17. There is _________ in my bedroom. A. a square wooden old table B. an old square wooden table C. a wooden old square table D. an old wooden square table 18. “I am sorry. I broke the vase”. - “ _________.” A. OK. Go ahead B. Yes, certainly C. Don’t worry. Things break D. I’d rather not. 19. One’s fingerprints are _________ other person. A. different from B. different from any C. differ from any D. different from those of any 20. He is very happy because he passed his exam with __________ colours. A. flying B. failing C. imagining D. changing Your answers: 1. 2. 3. 4. 5. 6. 7. 8. 9. 10. 11. 12. 13. 14. 15. 16. 17. 18. 19. 20 PART V. From four underlined parts, choose the one that needs correction then correct it. For example : The teacher did not allow the students discussing the take-home exam with each other. discussing → to discuss 1. A Geiger counter is an electronic instrument is used to measure the presence and intensity of radiation. 2. A dolphin locates underwater objects in their path by making a series of clicking and whistling sounds. 3. In spite of its small size, Europe had a great impact on world history than other continents. 4. Before she moved here , Alene has been president of the organization for four years. 5. That Marta's been chosen as the most outstanding student on her campus make her parents very happy. 6. My cousin composes not only the music, but also sings the songs for the major Broadway musicals. 7. Our civilization is so commonplace to us that rarely we stop to think about its complexity. 8. Ever since the world began, nations have difficulty in keeping peace with their neighbors. 3 9. Those of us who have a family history of heart disease should do yearly appointments with our doctors. 10. If one had thought about the alternatives, he would not have chosen such difficult a topic for a term paper. Your answers Mistake Correction 1. 2. 3. 4. 5. 6. 7. 8. 9. 10. PART VI. Fill in each space in the following sentences with the most suitable prepositions. 1. I'm afraid Tom's _______ work. But Jack's in. Would you like to speak to him? 2. Have you been to the theatre recently? ~ Yes, I was _______ the Old Vie last night. 3. At first I found the work very tiring, but _______ a few weeks I got used to it. 4. _______ the daytime the streets are crowded but at night they are quite deserted. 5. I saw Tom at the bus stop this morning but couldn't speak to him because we were standing _____ a queue. 6. He is always in a hurry. He drives _______ a tremendous speed. 7. Write ________ ink and put your name on the top of the page. 8. The man with the pipe and red hair is the brother of the girl ________ blue. 9. He sits at his desk all day with his head in his hands. It gets ________ my nerves. 10. The children hastily changed _______ bathing things and jumped into the river with shouts of delight. Your answers: 1. 2. 3. 4. 5. 6. 7. 8. 9. 10. PART VII. Give the correct form of the word in bracket to complete the passage. Your answers: The __1__ (say) “never judge a book by its cover” could not be more true for Ridiculous Rules by Marjorie Allen. The cover is completely blank, whereas 1. ............................................ the book is crammed full of wonderful examples and anecdotes. Allen is an 2. ............................................ __2__ (speak) critic of what is taught to native and non-native speakers of 3. ............................................ English, and has issued a __3__ (declare) of war against textbooks and style books which tell lies. 4............................................. Take the ridiculous and __4__ (mean) rule of never ending a sentence with a preposition. The lovely - if famous – story goes, that Winston Churchill, 5............................................. well-known for his numerous __5__ (write) as well as for being British Prime Minister during the Second World War, received a manuscript back from an 6. ............................................ ignorant __6__ (edit), who had told him rather rudely that he had to __7__ 7. ............................................ 4 (phrase) a sentence which ended with a preposition. Churchill responded by making the simple yet forceful __8__ (state) in the margin: “This is an 8. ............................................ impertinence up with which I will not put.” – the __9__ (imply) being that not 9. ............................................ to end a sentence with a preposition often sounds ridiculous in English, Sadly, 10. ............................................ Allen informs us that the story is probably mere __10__ (hear), and that Churchill may have actually only written “rubbish!” in the margin. C. READING PART VIII. Read the passage and choose the best answer A, B, C, or D to each of the questions Large animals that inhabit the desert have evolved a number of adaptations for reducing the effects of extreme heat. One adaptation is to be light in color, and to reflect rather than absorb the Sun's rays. Desert mammals also depart from the normal mammalian practice of maintaining a constant body temperature. Instead of trying to keep down the body temperature deep inside the body, which would involve the expenditure of water and energy, desert mammals allow their temperatures to rise to what would normally be fever height, and temperatures as high as 46 degrees Celsius have been measured in Grant's gazelles. The overheated body then cools down during the cold desert night, and indeed the temperature may fall unusually low by dawn, as low as 34 degrees Celsius in the camel. This is an advantage since the heat of the first few hours of daylight is absorbed in warming up the body, and an excessive buildup of heat does not begin until well into the day. Another strategy of large desert animals is to tolerate the loss of body water to a point that would be fatal for non-adapted animals. The camel can lose up to 30 percent of its body weight as water without harm to itself, whereas human beings die after losing only 12 to 13 percent of their body weight. An equally important adaptation is the ability to replenish this water loss at one drink. Desert animals can drink prodigious volumes in a short time, and camels have been known to imbibe over 100 liters in a few minutes. A very dehydrated person, on the other hand, cannot drink enough water to dehydrate at one session, because the human stomach is not sufficiently big and because a too rapid dilution of the body fluids causes death from water intoxication. The tolerance of water loss is of obvious advantage in the desert, as animals do not have to remain near a water hole but can obtain food from grazing sparse and far-flung pastures. Desert-adapted mammals have the further ability to feed normally when extremely dehydrated, it is a common experience in people that appetite is lost even under conditions of moderate thirst. 1. What is the main topic of the passage? A. Weather variations in the desert B. Adaptations of desert animals C. Diseased of desert animals D. Human use of desert animals. 2. According to the passage, why is light coloring an advantage to large desert animals? A. It helps them hide from predators. B. It does not absorb sunlight as much as dark colors. C. It helps them see their young at night D. It keeps them cool at night. 3. The word "maintaining" is closest in meaning to ___________. A. measuring B. inheriting C. preserving D. delaying 4. The author uses of Grant's gazelle as an example of ___________. A. an animal with a low average temperature B. an animal that is not as well adapted as the camel C. a desert animal that can withstand high body temperatures D. a desert animal with a constant body temperature 5. When is the internal temperature of a large desert mammal lower? A. Just before sunrise B. In the middle of the day C. Just after sunset D. Just after drinking 5 6. The word "tolerate" is closest in meaning to ___________. A. endure B. replace C. compensate D. reduce 7. What causes water intoxication? A. Drinking too much water very quickly B. Drinking polluted water C. Bacteria in water D. Lack of water. 8. What does the author imply about desert-adapted mammals? A. They do not need to eat much food. B. They can eat large quantities quickly C. They easily lose their appetites. D. They can travel long distances looking for food. 9. Why does the author mention humans in the second paragraph? A. To show how they use camels. B. To contrast them to desert mammals. C. To give instructions about desert survival. D. To show how they have adapted to desert life. 10. Which of the following is NOT mentioned as an adaptation of large desert animals? A. Variation in body temperatures B. Eating while dehydrated C. Drinking water quickly D. Being active at night. Your answers: 1. 2. 3. 4. 5. 6. 7. 8. 9. 10. PART IX. Read the text below and decide which answer A, B, C or D best fits each space. When you read something in a foreign language, you frequently (1)______ across words you do not fully understand. Sometimes you check the meaning in a dictionary and sometimes you (2)______. The strategy you adopt depends very much upon the (3)______ of accuracy you require and the time at your disposal. If you are the sort of person who tends to turn to the dictionary frequently, it is worth remembering that every dictionary has its (4)______. Each definition is only an approximation and one builds up an accurate picture of the meaning of a word only after meeting it in a (5)______ of contexts. It is also important to recognize the special dangers of dictionaries that translate from English into your native language and vice versa. If you must use a dictionary, it is usually far safer to (6)______ an English-English dictionary. In most exams you are not permitted to use a dictionary. (7)______ you are allowed to use one, it is very time-consuming to look up words, and time in exams is usually limited. You are, (8)______ , forced to guess the meaning of unfamiliar words. When you find unknown words in an exam text, it is very easy to panic. However, if you develop efficient techniques for guessing the meaning, you will (9)______ a number of possible problems and help yourself to understand far more of the text than you at first thought likely. Two strategies which may help you guess the meaning of a word are: using contextual clues, both within the sentence and outside, and making use of clues (10)______ from the formation of the word. 1. 2. 3. 4. 5. 6. 7. 8. 9. 10. A. put A. look A. extent A. limitations A. multiple A. survey A. or else A. so A. surpass A. derived B. drop B. guess B. range B. values B. variety B. consult B. Provided B. therefore B. get over B. extracted C. see C. examine C. degree C. advantages C. variation C. refer C. Although C. completely C. go over C. coming D. come D. inspect D. level D. entry D. diversity D. inquire D. Even if D. so that D. overcome D. originated Your answers: 1. 2. 3. 4. 5. 6. 7. 8. 9. 10. 6 D. WRITING PART X. Complete the second sentence so that it has similar meaning to the first one. 1. This will be my student's first performance in Canada. → This will be the first time …………………………………………………………………………... 2. This course will take us six months to complete. → In six months time ………………………………………………………………………………….. 3. The number of people who understand his ideas exceed his expectations. → More people ………………………………………………………………………………………… 4. She'll have to make her presentation at the end of his speech. → The moment he …………………………………………………………………………………….. 5. Sharon will finish her exams. Then she will have more free time. → Once ………………………………………………………………………………………………… 6. Both Mary and Peter prefer jazz to classical music. → Neither……………………………………………………………………………………………… 7. They repaired my car at the garage in town. → I............................................................................................................................................................ 8. This is the last time I will speak to you. → I........................................................................................................................................................... 9. I prefer staying in to going out. → I'd rather.............................................................................................................................................. 10. They passed the driving test because of the easy questions. → If …………………………………………………………………………………………………… Your answers: 1. ................................................................................................................................................................................................... 2. ................................................................................................................................................................................................... 3. ................................................................................................................................................................................................... 4. ................................................................................................................................................................................................... 5. ................................................................................................................................................................................................... 6. ................................................................................................................................................................................................... 7. ................................................................................................................................................................................................... 8. ................................................................................................................................................................................................... 9. .................................................................................................................................................................................................. 10. .................................................................................................................................................................................................. PART XI. ESSAY WRITING Some people say that cell phones have improved modern life. Others believe that cell phones have caused many problems to people. What is your opinion? In about 250 words, write an essay to assert your point of view on this problem. ………………………………………………………………………………………………………… ………………………………………………………………………………………………………… ………………………………………………………………………………………………………… 7 ………………………………………………………………………………………………………… ………………………………………………………………………………………………………… ………………………………………………………………………………………………………… ………………………………………………………………………………………………………… ………………………………………………………………………………………………………… ………………………………………………………………………………………………………… ………………………………………………………………………………………………………… ………………………………………………………………………………………………………… ………………………………………………………………………………………………………… ………………………………………………………………………………………………………… ………………………………………………………………………………………………………… ………………………………………………………………………………………………………… ………………………………………………………………………………………………………… ………………………………………………………………………………………………………… ………………………………………………………………………………………………………… ………………………………………………………………………………………………………… ………………………………………………………………………………………………………… ………………………………………………………………………………………………………… ………………………………………………………………………………………………………… ………………………………………………………………………………………………………… ………………………………………………………………………………………………………… ………………………………………………………………………………………………………… ………………………………………………………………………………………………………… ………………………………………………………………………………………………………… ………………………………………………………………………………………………………… ………………………………………………………………………………………………………… ………………………………………………………………………………………………………… ………………………………………………………………………………………………………… THE END 8 SỞ GIÁO DỤC VÀ ĐÀO TẠO HÀ TĨNH ĐÁP ÁN KỲ THI CHỌN HỌC SINH GIỎI TỈNH LỚP 9 THCS NĂM HỌC 2012 - 2013 Môn thi: TIẾNG ANH Tổng: 20 điểm, cụ thể như sau: A. LISTENING – 3 ĐIỂM PART I: 1,5 điểm = 0,15/ 1 câu đúng 1. twenty-three/ 23 2. blonde / blond 3. blue 4. average 5. computer programmer 6. music 7.( watching ) TV 8. outgoing 9. music/ sports 10. sports/ music PART II: 0,7 điểm = 0,1/ 1 câu đúng 1. F 2. F 3. F 4. T 5. F 6. T 7. T 8.F PART III: 0,8 điểm = 0,1/ 1 câu đúng 1. C 2. C 3. B 4. A 5. C 6. D 7. B B. LEXICO – GRAMMAR – 8 ĐIỂM PART IV: 4 điểm = 0,2 / 1 câu đúng 1. A 2. B 3. B 4. D 5. D 6. D 7. C 8. D 9. A 10.A 11. D 12. B 13. D 14. B 15. D 16. C 17. B 18. C 19. D 20. A PART V: 2 điểm = 0,1 / 1 câu đúng 1. is used → used 2. their path → its path 3. great impact → greater impact 4. has been → had been 5. make → makes/made 6. composes not only → not only composes 7. rarely we stop → rarely do we stop 8. have difficulty → have had difficulty 9. do yearly → make yearly 10. such difficult → so difficult PART VI: 1 điểm = 0,1 / 1 câu đúng 1. at 2. at 3. for 4. in 5. in 6. at 7. in 8. in 9. on 10. into PART VII: 1 điểm = 0,1 / 1 câu đúng 1. saying 2. outspoken 3. declaration 4. meaningless 5. writings 6. editor 7. rephrase 8. statement 9. implication 10. hearsay C. READING – 4 ĐIỂM PART VIII: 2 điểm = 0,2 / 1 câu đúng 9 1. B 2. B 3. C 4. C 5. A 6. A 7. A 8. D 9. B 10. D PART IX: 2 điểm = 0,2 / 1 câu đúng 1. D 2. B 3. C 4. C 5. B 6. B 7. D 8. B 9. D 10. A D. WRITING – 5 ĐIỂM Part IX: 2 điểm = 0,2 / 1 câu đúng my student has performed in Canada. 1. This will be the first time 2. In six months time we will have completed this course. 3. More people understand him than he expected./ has expected/ expects. 4. The moment he finishes she'll have to make her presentation. 5. Once Sharon finishes her exams, she will have more free time 6. Neither Mary nor Peter prefers classical music to jazz. 7. I had my car repaired at the garage in town. 8. I will not/never speak to you (again). 9. I'd rather stay in than go out. 10. If my student has given the performance in Canada. the questions hadn’t been easy, they wouldn’t have passed the driving test. the questions had been (more) difficult, they would have failed the driving test. It hadn’t been for the easy questions, Part X: 3 điểm - Nội dung (content): 1.5 điểm - Từ vựng (vocabulary): 0.5 điểm - Ngữ pháp (grammar): 0.5 điểm - Tính mạch lạc và trôi chảy (coherence and cohesion) + độ dài (length): 0.5 điểm 10 SỞ GDĐT BẠC LIÊU ĐỀ CHÍNH THỨC (Gồm 06 trang) KỲ THI CHỌN HSG LỚP 10, 11 VÒNG TỈNH NĂM HỌC 2011 - 2012 * Môn thi: TIẾNG ANH * Lớp: 10 (Bảng A) * Thời gian: 180 phút (Không kể thời gian giao đề) ĐỀ PART 1. LISTENING (2 points) A. Listen and circle the best answer. 1. This is _________ . A. a survey B. an interview C. a face-to-face conversation D. a chat 2. The people talking __________. A. are co-workers B. are friends C. are colleagues D. do not know each other 3. The passenger’s bags haven’t arrived because _________. A. they have been missed B. they have been postponed C. they have been taken by mistake D. they have been stolen B. Listen again and complete the information on the form. Missing Baggage Details Flight number: (4) _____________ From: (5)____________ Passenger Name: (6) _____________ First name: (7) _____________ Number of bags (8) _____________ Phone : (9) ____________ Length of stay: (10) ____________ at : _Princess Hotel____ PART 2. PHONETICS I. Circle the correct answer (A, B, C or D) that has the underlined letter(s) pronounced differently from the rest (1 point). 1. A. thank B. band C. complain D. insert 2. A. lays B. says C. stays D. plays 3. A. scholarship B. chaos C. cherish D. chorus 4. A. message B. privilege C. college D. collage 5. A. talked B. naked C. asked D. liked II. Circle the correct answer (A, B, C or D) whose main stress is placed differently from the others (1 point) 1. A. individual B. reputation C. experience D. scientific 2. A. carpenter B. revise C. ignore D. traditional 3. A. necessary B. achieve C. poetic D. communicate 4. A. influence B. modern C. consider D. different 5. A. contain B. poisonous C. chemical D. scientist 1 PART 3. LANGUAGE FOCUS I. Circle the best answer (A, B, C or D) to complete each of the sentences below. (1 point). 1.There are not thing special about his clothes ………..from his flowery tie. A. but B. except C. other D. apart 2. I’d ……….you explained to her why we can’t go. A. better B. rather C. want D. need 3. I …………we meet outside the cinema tomorrow at 8.30. A. think B. suggest C. consider D. introduce 4. I walked away as calmly as I could ………..they thought I was the thief. A. or else B. to avoid C. owing to D. in case 5. ………..the step when you go in. A. Consider B. Mind C. Attend D. Look 6. ………..of all of us who are tonight, I would like to thank Mr Jones for his talk. A. On behalf B. On account C. In person D. Instead 7. She refused to eat meat under any …………. A. circumtances B. occasion C. opportunity D. reason 8. There was nothing they could do …………….leave the car at the roadside where it had brokendown. A. than B.unless C. instead of D. but 9. To our ………..his illness proved not to be as serious as we had feared. A. anxiety B. eyes C. belief D. judgement 10. Mary Smith decided to give up her job for the …………of her children. A. sake B. care C. mean D. concern II. Choose the best option (A, B, C or D) to complete the passage below. (1 point) PAINTING Nancy Andrews’ latest exhibition at the Seagull Gallery proves that she is one of our most (1) ................... painters. She has emerged from her (2) ................... phase, in which the influence of Picasso was apparent, and returned to a more (3) ................... style of painting, but without losing her (4) ................... sense of colour and form. She has chosen (5) ................... themes for this show, and her portrait of Bacchus, for example, has a wonderful (6) ................... about it. But the most (7) ...................work is the large canvas called simply “Mars”. It is a (8) ................... study, in various shades of red and gold, and its balanced (9) ...................gives it a pleasing, even restful feel, despite the (10) ................... of its subject matter. 1. A. gifting B. gift C. gifts D. gifted 2. A. cubist B. cubism C. cubical D. cubic 3. A. tradition B. traditional C. traditionalized D. traditionalizing 4. A. delight B. delighted C. delightful D. delighting 5. A. mythology B. mythologic C. mythological D. mythologious 6. A. simplify B. simplicity C. simplification D. simple 7. A. impressive B. impression C. impress D. impressed 8. A. coloured B. colourless C. colour D. colourful 9. A. compose B. composing C. composition D. composion 10. A. violence B. violent C. violency D. violently III. Put the words in brackets into the correct forms. (1 point) Computers have had the ability to play chess for many years now, and their ………………….. (1. PERFORM) in games against the best players in the world has shown steady improvement. However, it will be years before the designers of computer games machines can beat their ………………….. (2. BIG) challenge yet – the ancient board game 2 called “Go”. The playing area is ………………….. (3. CONSIDERABLE) larger than in chess and there are far more pieces, so that the combination of movies is almost endless. The game involves planning so many moves ahead that even the impressive calculations of the fastest modern computers are ………………….. (4. SUFFICIENT) to deal with the problems of the game. In a recent competition for computer “Go” machines, the best machine beat all its rivals, but lost ………………….. (5. HEAVY) to three young schoolchildren, so there is obviously still a lot of work to do. IV. Supply the correct verb tenses of the verbs in brackets. (1 point) 1. She says that she (live) …………….. in the countryside when she (be) ……………. a child. 2. Oil (leak) ……………….. from a petrol tanker onto the motorway, so the police (close) …………….. off the section between Junctions 5 and 6. 3. Foreign currency (convert) …………….. into sterling at a number of points in the city, but the best rate and lowest commisson charges (provide) ………………… at banks. 4. Most of the news on the front page of both daily newspapers (concern) ……………. the progress of the peace conference. 5. Why are you carrying that saw? ∼ I (shorten) ………………. the legs of the dining room table. V. Choose the best option (A, B, C, D) to complete each of the sentences below. (1 point) 1. “Do well and do not ...................” A. look around B. look round C. look up D. look down 2. At this time of the year, we are always...................with applications for training course. A. snowed under B. come under C. kept under D. gone under 3. I was asked to...................with a scheme to reduce the pollution caused by our factory. A. make up B. set up C. come up D. check up 4. We were all too happy when he managed to...................the deal................... A. run/ through B. go/ through C. get/ through D. put/ through 5. I ...................to the man when I saw him collapse, but it was too late. A. stole up B. come up C. faced up D. rushed up VI. Finish each of the following sentences in such a way that it means exactly the same as the sentence printed before it (2 points) 1. In my opinion you are not telling the truth. Æ I suggest that...................................................................................................................... 2. They reported that she was a policewoman. Æ She ..................................................................................................................................... 3. “I’m sorry I didn’t ring you to say I’d be late”, he said. Æ He....................................................................................................................................... 4. Everyone started complaining the moment the announcement was made. Æ No sooner .......................................................................................................................... 5. The only thing that prevented the passing of the bill was the death of the Prime Minister. 3 Æ Had it not .......................................................................................................................... VII. Rewrite the second sentence so that it has a similar meaning to the first one, using the word given. Do not change the word given. (2 points) 1. Your attitude will have to change if you want to succeed. (leaf) ................................................................................................................................................. 2. You couldn’t do anything more stupid than to give up your job now. (height) ................................................................................................................................................. 3. The trouble all came about because our computers crashed. (stemmed) ................................................................................................................................................. 4. Do you have to wear a uniform at school? (compulsory) ................................................................................................................................................. 5. The way many sportsmen behave in public influences their young fans. (impression) ................................................................................................................................................. PART 4. READING I. Read the text below and give the word which best fits each space. (1 point) One of the greatest problems with holidays, (1)……………... from the usual travel complications and accommodation difficulties, (2)…………………..… the expectations people have of them. When we go on holiday we expect to leave all the stresses and strains of our daily lives (3)…………….... us. We imagine we will be able to escape to (4)…………….. a degree that we even tend to believe, consciously or not, that we can change our own personalities and become completely different people. The average business-person, tense, preoccupied, short-tempered (5)……………….. to relax, envisages herself/himself (6)……………... from the moment of locking the office door, a radically different (7)…………..…. of person: carefree, good-humoured, ready to relax and enjoy whatever adventures present (8)……………. In practice, we take ourselves with us (9)…………….. we go, and the personality that is shaped over years of stress and tension is almost impossible to shake off at a moment’s notice. It is no wonder so many holidays are a disappointment, no matter how smoothly they go or how lovely the weather is. In fact, the frequent problems that crop (10)………..….. during the average holiday are probably a welcome distraction from the nagging feeling that we are not enjoying ourselves as much as we should. II. Read the passage below and circle the best answer (A, B, C or D) to each question. (3.0 points) The fertile valleys of the river Nile straddle the hot desert land of Egypt. Rain is relatively scarce, and the summers are scorching hot. Nevertheless the strip of land, known as the Cultivation, on either side of the Nile is reputed to be one of the most fertile places in the world. Its rich black soil is the result of accumulation of silt deposited by the annual flooding of the Nile thousands of years ago. From June to October, the river overflowed its banks. Modern damps were then constructed to control the flooding. The floodwaters left behind a 4 rich sticky black mud which made it suitable for the cultivation of crops. As long as the soil was well irrigated, two or three crops could be grown in one season. Its rich soil led to the growth and rise of the brilliant civilization of the ancient Egyptians on the Nile valley more than 5000 years ago. The earliest Egyptians had acquired the skills to till the land along the banks, drawing water from the Nile for irrigation purposes. As the villagers along the Nile became wealthy, they embarked on projects of digging ditches and constructing dams to control the floods. In about 3100 BC, the whole Egypt was united under the reign of King Menes. He and his descendants made up Egypt’s first ruling family, or dynasty. His rule led to the great development of the arts like writing, painting, architecture, and crafts. Egyptian power and influence were and influence were to last for the next 2000 years. The Egyptian kings had absolute powers. The king possesses all the land, and the peasants had to surrender part of their crops to the king. An army of officials and scribes did the task of collecting the exact amount of due from the individual farmers was united. Crops and livestock were often seen at the storehouses surrounding the royal palaces. In return for their uphill task, the king paid his officials and dishes out funds for huge irrigation projects. Egypt’s trade with the outside world stretches far and wide. In return for gold, copper, gemstones, and building stores, it purchases the goods that it did not have. Timber, resins, oils, silver and slaves came from Lebanon. From Deria and Anatolia came horses, while the blue stone called lapis lazuli was imported from Mesopotamia. Strong forts were constructed to protect overland trade routes. Egypt held Nubia in the south for almost 800 years. It served as Egypt’s most vital source of gold and slaves. 1. The civilization of the ancient Egyptians was brought about by ……….. A. the rich alluvial soil of the Nile B. Egypt’s first ruling family C. the highly-developed writing, painting, architecture and crafts D. Egypt’s trade with the outside world 2. Egyptian slaves came from ………. A. Mesopotamia B. Syria and Anatolia C. Lebanon D. Palestine 3. The king was wealthy because ………. A. huge irrigation projects were set up B. farmer paid him their tribute C. he was in absolute power D. he possessed all the land 4. The writer said that the annual flooding of the Nile ……….. A. helped to develop the arts B. caused an influx of foreign traders C. needed to be controlled D. led to the discovery of new mines 5. According to the passage, the king was wise ……… A. to important the things that Egypt did not have B. to pay for huge irrigation projects C. to control the land D. to own slaves and gold 6. In return for, in the first line, last paragraph, refers to ……….. A. the respect Egypt received B. how fast the profit came C. the protection the Egyptian army provided D. what Egypt exported 7. A suitable title for this passage is ……… A. The Civilization Of The Ancient Egyptians B. The Farming Methods Of The Egyptians C. The Might Of The Egyptian Army D. The Landscape Of The Egypt 5 8. We know the Egyptian Empire was powerful from the ………. A. way its army fought with other invaders B. long period it extended its power and influence C. trade and business done with other nations D. number of crops grown in one season 9. Farm crops would be ……….if there was no water for irrigation. A. harvested B. unprotected C. destroyed D. unavailable 10. From June to October, the flood plains would be ………. A. inundated with floodwaters from the Nile B. open to the building of new dams C. left to the sowing of new crops D. raised to a higher level PART 5. WRITING (3 points) Write a paragragh ( about 150 words) about the following topic: “ How important is music in a modern life?” ................................................................................................................................ ................................................................................................................................ ................................................................................................................................ ................................................................................................................................ ................................................................................................................................ ................................................................................................................................ ................................................................................................................................ ................................................................................................................................ ................................................................................................................................ ................................................................................................................................ ................................................................................................................................ ................................................................................................................................ ................................................................................................................................ ................................................................................................................................ ................................................................................................................................ ................................................................................................................................ ................................................................................................................................ ................................................................................................................................ ................................................................................................................................ --- HẾT --6 SỞ GDĐT BẠC LIÊU ĐỀ CHÍNH THỨC (Gồm 02 trang) KỲ THI CHỌN HSG LỚP 10, 11 VÒNG TỈNH NĂM HỌC 2011 - 2012 * Môn thi: TIẾNG ANH * Lớp: 10 (bảng A) * Thời gian: 180 phút (Không kể thời gian giao đề) HƯỚNG DẪN CHẤM PART 1. LISTENING Each correct answer gets 0.2p 1. C 2. D 3. B 4. UA675 5. Tokyo 6. Johnstone 7. Bill 8. 2 9. 555-432 10. 3 PART 2. PHONETICS I. 1 point. Each correct answer gets 0.2p 1. A 2. B 3. C 4. D 5. B II. 1 point. Each correct answer gets 0.2p 1. C 2. A 3. A 4. C 5. A PART 3. LANGUAGE FOCUS I. 1 point. Each correct answer gets 0.1p 1. D 2. B 3. B 4. D 5. B 6. A 7. A 8. D 9. C 10. A II. 1 point. Each correct answer gets 0.1p 1.D 2.A 3.B 4.C 5.C 6.B 7.A 8.D 9.C 10. A III. 1 point. Each correct answer gets 0.2p 1. performance 2. biggest 3. considerably 4. insufficient 5. heavily IV. 1 point. Each correct answer gets 0.2p 1. lived – was 2. is leaking – are closing 3. can be converted – are provided 4. concerns 5. am going to shorten V. 1 point. Each correct answer gets 0.2p 1.B 2.A 3.C 4.D 5. D VI. 2 points. Each correct answer gets 0.4p 1. I suggest that you be telling the truth. 2. She was reported to be a policewoman. 3. He apologized (to me) for not ringing me to say he’d be late. 4. No sooner had the announcement been made than everyone started complaining. 5. Had it not been the death of the Prime Minister, the bill’d have been passed. VII. 2 points. Each correct answer gets 0.4p 1. You will have to turn over a new leaf if you want to suceed. 2. It would be the height of your stupidity to give up your job now. 3. The trouble all stemmed from our computers crashing. 4. Is it compulsory for you to wear a uniform at school? 5. The way many sportsmen behave in public makes an impression on their young fans. PART 4. READING I. 1 point. Each correct answer gets 0.1p 1. apart 2. is 3. behind 4. such 5. unable 6. as 7. kind 8. themselves 9. wherever 10. up II. 3 points. Each correct answer gets 0.3p 1. A 2. C 3. D 4. C 5. B 6. D 7. A 8. B 9. C 10.A PART 5. WRITING (3 points) From 2.75 to 3 : Excellent 1 Natural Englsh with minimal errors and complete realisation of the task set. From 2 to 2,5 : Good Good vocabulary and structure. Errors non-basic. From 1.5 to 1.75 : pass Simple but accurate realisation of task. Not many errors Form 1 to 1.25 : Weak Reasonably correct with some serious errors. Form 0 to 0.75 : Very poor Incoherent.Errors showing lack of basic knowledge of English. --- HẾT--- 2 SỞ GDĐT BẠC LIÊU ĐỀ CHÍNH THỨC (Gồm 06 trang) KỲ THI CHỌN HSG LỚP 10, 11 VÒNG TỈNH NĂM HỌC 2011 - 2012 * Môn thi: TIẾNG ANH * Lớp: 10 (Bảng B) * Thời gian: 180 phút (Không kể thời gian giao đề) ĐỀ PART 1. LISTENING (2 points) A. Listen and circle the best answer. 1. This is _________ . A. a survey B. an interview C. a face-to-face conversation D. a chat 2. The people talking __________ A. are co-workers B. are friends C. are colleagues D. do not know each other 3. The passenger’s bags haven’t arrived because _________ A. they have been missed B. they have been postponed C. they have been taken by mistake D. they have been stolen B. Listen again and complete the information on the form. Missing Baggage Details Flight number: (4) _____________ From: (5)____________ Passenger Name: (6) _____________ First name: (7) _____________ Number of bags (8) _____________ Phone : (9) ____________ Length of stay: (10) ____________ at : _Princess Hotel____ PART 2. PHONETICS I. Circle the correct answer (A, B, C or D) that has the underlined letter(s) pronounced differently from the rest (1 point). 1. A. thank B. band C. complain D. insert 2. A. lays B. says C. stays D. plays 3. A. scholarship B. chaos C. cherish D. chorus 4. A. message B. privilege C. college D. collage 5. A. talked B. naked C. asked D. liked II. Circle the correct answer (A, B, C or D) whose main stress is placed differently from the others (1 point) 1. A. individual B. reputation C. experience D. scientific 2. A. carpenter B. revise C. ignore D. traditional 3. A. necessary B. achieve C. poetic D. communicate 4. A. influence B. modern C. consider D. different 5. A. contain B. poisonous C. chemical D. scientist PART 3. LANGUAGE FOCUS 1 I. Circle the best answer (A, B, C or D) to complete each of the sentences below. (1 point). 1.There are not thing special about his clothes ………..from his flowery tie. A. but B. except C. other D. apart 2. I’d ……….you explained to her why we can’t go. A. better B. rather C. want D. need 3. I …………we meet outside the cinema tomorrow at 8.30. A. think B. suggest C. consider D. introduce 4. I walked away as calmly as I could ………..they thought I was the thief. A. or else B. to avoid C. owing to D. in case 5. ………..the step when you go in. A. Consider B. Mind C. Attend D. Look 6. ………..of all of us who are tonight, I would like to thank Mr Jones for his talk. A. On behalf B. On account C. In person D. Instead 7. She refused to eat meat under any …………. A. circumtances B. occasion C. opportunity D. reason 8. There was not thing they could do …………….leave the car at the roadside where it had brokendown. A. than B.unless C. instead of D. but 9. To our ………..his illness proved not to be as serious as we had feared. A. anxiety B. eyes C. belief D. judgement 10. Mary Smith decided to give up her job for the …………of her children. A. sake B. care C. mean D. concern II. Choose the best option (A, B, C or D) to complete the passage below. (1 point) NATURE Since the world has become (1) ..................., there has been an increase in the number of animal species that have either become extinct or have neared (2) ................... . Bengal tigers, for instance, which once roamed the jungles in vast numbers, now (3) ...................only about 2,300 and by the year 2025 their population is estimated to be down to zero. What is (4) ................... about the case of the Bengal tiger is that this extinction will have been caused almost entirely by (5) ................... who, according to some sources, are not interested in material gain but in personal (6) ................... This is an example of the (7) ................... that is part of what is causing the problem of extinction. Animals like the Bengal tiger, as well as other (8) ...................species, are a valuable part of the world’s ecosystem. International laws (9) ................... these animals must (10) ................... to ensure their survival, and the survival of our planet. 1. A. industrialized B. industry C. industrial D. industrialize 2. A. extinctation B. extinctive C. extinct D. extinction 3. A. is numbering B. numbered C. number D. is numbered 4. A. alarmist B. alarmed C. alarm D. alarming 5. A. poacher B. poachers C. poachors D. poachor 6. A. gratificating B. gratify C. gratification D. gratifiation 7. A. callous B. callously C. callousness D. callosity 8. A. dangered B. endangered C. endanger D. dangerous 9. A. protecting B. protect C. protected D. is protected 10. A. be acted B. be enacted C. be enacting D. be acting III. Put the words in brackets into the correct forms. (1 point) Air travel has become much cheaper in recent years, and ……………… (1. TOUR) go on foreign holidays in large numbers every year. The huge travel and tourism industry brings money to many of the poorer countries around the world. The ……………… (2. ARRIVE) of large numbers of visitors every year requires very good organization and planning. 2 However, many countries have ……………..…….. (3. DEVELOPMENT) huge tourist center without much thought for ………………. (4. LOCATION) people and ………………. (5. WILD). IV. Supply the correct verb tenses of the verbs in brackets. (1 point) 1. She says that she (live) …………. in the countryside when she (be) ………..….. a child. 2. Oil (leak) ………….… from a petrol tanker onto the motorway, so the police (close) …………... off the section between Junctions 5 and 6. 3. Foreign currency (convert) ……….…… into sterling at a number of points in the city, but the best rate and lowest commisson charges (provide) …………. at banks. 4. Most of the news on the front page of both daily newspapers (concern) …………... the progress of the peace conference. 5. Why are you carrying that saw? ∼ I (shorten) ……….... the legs of the dining room table. V. Choose the best option (A, B, C, D) to complete each of the sentences below. (1 point) 1. The doctor vaccinated the baby...................polio. A. against B. with C. to D. without 2. She threw the stone...................the dog when it started to chase her. A. to B. toward C. at D. against 3. They learned to accept their stepmother...................time. A. on B. at C. all D. in 4. Have you ever lost...................anybody...................a game...................tennis? A. to/ in/ of B. to/ for/ of C. with/ in/ of D. to/ in/ in 5. Would you like to live...................an igloo...................some time? An igloo is a house that is made...................blocks...................snow or ice. It’s so romantic. A. in/ at/ of/ of B. in/ for/ of/ of C. in/ for/ of/ with D. in/ at/ by/ with VI. Finish each of the following sentences in such a way that it means exactly the same as the sentence printed before it (2 points) 1. Small fishing boats continued to sail out though the coastguard had warned them not to do so. Æ Despite ............................................................................................................................... 2. They’re willing to work hard and we are surprised about that. Æ We’re surprised.................................................................................................................. 3. If you have a better job, you get higher pay. Æ The better the job, .............................................................................................................. 4. They took away his passport so that he wouldn’t leave the country. Æ They prevented .................................................................................................................. 5. You will never meet anyone more generous than Mrs. Jones. Æ Mrs. Jones is the most ……………………………………………………………… VII. Rewrite the second sentence so that it has a similar meaning to the first one, using the word given. Do not change the word given. (2 points) 1. Your attitude will have to change if you want to succeed. (leaf) Æ ……………………………………………………………………………………………. 3 2. You couldn’t do anything more stupid than to give up your job now. (height) Æ ……………………………………………………………………………………………. 3. The trouble all came about because our computers crashed. (stemmed) Æ ……………………………………………………………………………………………. 4. Do you have to wear a uniform at school? (compulsory) Æ ……………………………………………………………………………………………. 5. The way many sportsmen behave in public influences their young fans. (impression) Æ ……………………………………………………………………………………………. PART 4. READING I. Read the text below and give the word which best fits each space. (1 point) After living (1) …. the threat of extinction for more than 30 years, the national bird of the United States has been granted an official reprieve, as the bald eagle and twenty- eight other animal and plant species have been earmarked (2) …. removal from America’s list of endangered species. The bald eagle, also known as the white-headed sea eagle, took pride of place at the top of a list of species likely to (3) …. taken off the endangered register in the coming years. The proposed “delistings” are being promoted by the US interior secretary to counter a growing feeling among Republicans that endangered species laws do not work. Charges of ineffectiveness have been (4) ….against these laws before, but more recently it has even been suggested that the situation may actually have been (5) ….. worse by them. The recovery of the bald eagle follows thirty-one years on the critical list. Its numbers had been (6) …. to fewer than five hundred through the use of pesticides that reacted adversely (7) …. its reproductive system. The number of nesting pairs is now estimated at five thousand. The interior secretary claims that the new list was a vindication of the legislation under which the eagle, a national symbol (8) …. originally from the Indians, and more than a thousand other species were protected.(A spokesperson denied that it was simply a publicity stunt timed to coincide with the swearing (9) .… of the president for his second term). Until now, few species have (10)…. been removed from the list. When they were, it was usually because they had become extinct. II. Read the passage below and circle the best answer (A, B, C or D) to each question. (3.0 points) The fertile valleys of the river Nile straddle the hot desert land of Egypt. Rain is relatively scarce, and the summers are scorching hot. Nevertheless the strip of land, known as the Cultivation, on either side of the Nile is reputed to be one of the most fertile places in the world. Its rich black soil is the result of accumulation of silt deposited by the annual flooding of the Nile thousands of years ago. From June to October, the river overflowed its banks. Modern damps were then constructed to control the flooding. The floodwaters left behind a rich sticky black mud which made it suitable for the cultivation of crops. As long as the soil was well irrigated, two or three crops could be grown in one season. 4 Its rich soil led to the growth and rise of the brilliant civilization of the ancient Egyptians on the Nile valley more than 5000 years ago. The earliest Egyptians had acquired the skills to till the land along the banks, drawing water from the Nile for irrigation purposes. As the villagers along the Nile became wealthy, they embarked on projects of digging ditches and constructing dams to control the floods. In about 3100 BC, the whole Egypt was united under the reign of King Menes. He and his descendants made up Egypt’s first ruling family, or dynasty. His rule led to the great development of the arts like writing, painting, architecture, and crafts. Egyptian power and influence were and influence were to last for the next 2000 years. The Egyptian kings had absolute powers. The king possesses all the land, and the peasants had to surrender part of their crops to the king. An army of officials and scribes did the task of collecting the exact amount of due from the individual farmers was united. Crops and livestock were often seen at the storehouses surrounding the royal palaces. In return for their uphill task, the king paid his officials and dishes out funds for huge irrigation projects. Egypt’s trade with the outside world stretches far and wide. In return for gold, copper, gemstones, and building stores, it purchases the goods that it did not have. Timber, resins, oils, silver and slaves came from Lebanon. From Deria and Anatolia came horses, while the blue stone called lapis lazuli was imported from Mesopotamia. Strong forts were constructed to protect overland trade routes. Egypt held Nubia in the south for almost 800 years. It served as Egypt’s most vital source of gold and slaves. 1. The civilization of the ancient Egyptians was brought about by ……….. A. the rich alluvial soil of the Nile B. Egypt’s first ruling family C. the highly-developed writing, painting, architecture and crafts D. Egypt’s trade with the outside world 2. Egyptian slaves came from ………. A. Mesopotamia B. Syria and Anatolia C. Lebanon D. Palestine 3. The king was wealthy because ………. A. huge irrigation projects were set up B. farmer paid him their tribute C. he was in absolute power D. he possessed all the land 4. The writer said that the annual flooding of the Nile ……….. A. helped to develop the arts B. caused an influx of foreign traders C. needed to be controlled D. led to the discovery of new mines 5. According to the passage, the king was wise ……… A. to important the things that Egypt did not have B. to pay for huge irrigation projects C. to control the land D. to own slaves and gold 6. In return for, in the first line, last paragraph, refers to ……….. A. the respect Egypt received B. how fast the profit came C. the protection the Egyptian army provided D. what Egypt exported 7. A suitable title for this passage is ……… A. The Civilization Of The Ancient Egyptians B. The Farming Methods Of The Egyptians C. The Might Of The Egyptian Army D. The Landscape Of The Egypt 8. We know the Egyptian Empire was powerful from the ………. A. way its army fought with other invaders 5 B. long period it extended its power and influence C. trade and business done with other nations D. number of crops grown in one season 9. Farm crops would be ……….if there was no water for irrigation. A. harvested B. unprotected C. destroyed D. unavailable 10. From June to October, the flood plains would be ………. A. inundated with floodwaters from the Nile B. open to the building of new dams C. left to the sowing of new crops D. raised to a higher level PART 5. WRITING (3 points) Write a paragraph (from 150 to 200 words) about the following topic: “ The advantages and disadvantages of living in a big city” ................................................................................................................................ ................................................................................................................................ ................................................................................................................................ ................................................................................................................................ ................................................................................................................................ ................................................................................................................................ ................................................................................................................................ ................................................................................................................................ ................................................................................................................................ ................................................................................................................................ ................................................................................................................................ ................................................................................................................................ ................................................................................................................................ ................................................................................................................................ ................................................................................................................................ ................................................................................................................................ ................................................................................................................................ ................................................................................................................................ ................................................................................................................................ ................................................................................................................................ ................................................................................................................................ --- THE END --6 SỞ GDĐT BẠC LIÊU ĐỀ CHÍNH THỨC (Gồm 02 trang) KỲ THI CHỌN HSG LỚP 10, 11 VÒNG TỈNH NĂM HỌC 2011 - 2012 * Môn thi: TIẾNG ANH * Lớp: 10 (Bảng B) * Thời gian: 180 phút (Không kể thời gian giao đề) HƯỚNG DẪN CHẤM PART 1. LISTENING 2 points. Each correct answer gets 0.2p 1. C 2. D 3. B 4. UA675 5. Tokyo 6. Johnstone 7. Bill 8. 2 9. 555-432 10. 3 PART 2. PHONETICS I. 1 point. Each correct answer gets 0.2p 1. A 2. B 3. C 4. D 5. B II. 1 point. Each correct answer gets 0.2p 1. C 2. A 3. A 4. C 5. A PART 3. LANGUAGE FOCUS I. 1 point. Each correct answer gets 0.1p 1. D 2. B 3. B 4. D 5. B 6. A 7. A 8. D 9. C 10. A II. 1 point. Each correct answer gets 0.1p 1. A 2. D 3. C 4. D 5. B 6. C 7. C 8. B 9. A 10. B III. 1 point. Each correct answer gets 0.2p 1. Tourists 2. arrival 3. developed 4. local 5. wildlife IV. 1 point. Each correct answer gets 0.2p 1. lived – was 2. is leaking – are closing 3. can be converted – are provided 4. concerns 5. am going to shorten V. 1 point. Each correct answer gets 0.2p 1.A 2.C 3.D 4.A 5.B VI. 2 points. Each correct answer gets 0.4p 1. Despite the coastguard’s warning, small fishing boats continued to sail out 2. We’re surprised about their willingness to work hard 3. The better the job, the higher the pay 4. They prevented him from leaving the country by taking away his passport. 5. Mrs. Jones is the most generous person / woman you have ever met. VII. 2 points. Each correct answer gets 0.4p 1. You will have to turn over a new leaf if you want to suceed. 2. It would be the height of your stupidity to give up your job now. 3. The trouble all stemmed from our computers crashing. 4. Is it compulsory for you to wear a uniform at school? 5. The way many sportsmen behave in public makes an impression on their young fans. PART 4. READING I. 1 point. Each correct answer gets 0.1p 6. reduced 1. under 7. with 2. for 8. taken/ borrwed 3. be 9. in 4. levelled/leveled 10. ever 5. made 1 II. 3 points. Each correct answer gets 0.3p 1. A 2. C 3. D 4. C 5. B 6. D 7. A PART 5. WRITING (3 points) 8. B 9. C 10.A From 2.75 to 3 : Excellent Natural Englsh with minimal errors and complete realisation of the task set. From 2 to 2,5 : Good Good vocabulary and structure. Errors non-basic. From 1.5 to 1.75 : pass Simple but accurate realisation of task. Not many errors Form 1 to 1.25 : Weak Reasonably correct with some serious errors. Form 0 to 0.75 : Very poor Incoherent.Errors showing lack of basic knowledge of English. --- HẾT--- 2 SỞ GDĐT BẠC LIÊU ĐỀ CHÍNH THỨC (Gồm 06 trang) KỲ THI CHỌN HSG LỚP 10, 11 VÒNG TỈNH NĂM HỌC 2011 - 2012 * Môn thi: TIẾNG ANH * Lớp: 10 (Bảng B) * Thời gian: 180 phút (Không kể thời gian giao đề) ĐỀ PART 1. LISTENING (2 points) A. Listen and circle the best answer. 1. This is _________ . A. a survey B. an interview C. a face-to-face conversation D. a chat 2. The people talking __________ A. are co-workers B. are friends C. are colleagues D. do not know each other 3. The passenger’s bags haven’t arrived because _________ A. they have been missed B. they have been postponed C. they have been taken by mistake D. they have been stolen B. Listen again and complete the information on the form. Missing Baggage Details Flight number: (4) _____________ From: (5)____________ Passenger Name: (6) _____________ First name: (7) _____________ Number of bags (8) _____________ Phone : (9) ____________ Length of stay: (10) ____________ at : _Princess Hotel____ PART 2. PHONETICS I. Circle the correct answer (A, B, C or D) that has the underlined letter(s) pronounced differently from the rest (1 point). 1. A. thank B. band C. complain D. insert 2. A. lays B. says C. stays D. plays 3. A. scholarship B. chaos C. cherish D. chorus 4. A. message B. privilege C. college D. collage 5. A. talked B. naked C. asked D. liked II. Circle the correct answer (A, B, C or D) whose main stress is placed differently from the others (1 point) 1. A. individual B. reputation C. experience D. scientific 2. A. carpenter B. revise C. ignore D. traditional 3. A. necessary B. achieve C. poetic D. communicate 4. A. influence B. modern C. consider D. different 5. A. contain B. poisonous C. chemical D. scientist PART 3. LANGUAGE FOCUS 1 I. Circle the best answer (A, B, C or D) to complete each of the sentences below. (1 point). 1.There are not thing special about his clothes ………..from his flowery tie. A. but B. except C. other D. apart 2. I’d ……….you explained to her why we can’t go. A. better B. rather C. want D. need 3. I …………we meet outside the cinema tomorrow at 8.30. A. think B. suggest C. consider D. introduce 4. I walked away as calmly as I could ………..they thought I was the thief. A. or else B. to avoid C. owing to D. in case 5. ………..the step when you go in. A. Consider B. Mind C. Attend D. Look 6. ………..of all of us who are tonight, I would like to thank Mr Jones for his talk. A. On behalf B. On account C. In person D. Instead 7. She refused to eat meat under any …………. A. circumtances B. occasion C. opportunity D. reason 8. There was not thing they could do …………….leave the car at the roadside where it had brokendown. A. than B.unless C. instead of D. but 9. To our ………..his illness proved not to be as serious as we had feared. A. anxiety B. eyes C. belief D. judgement 10. Mary Smith decided to give up her job for the …………of her children. A. sake B. care C. mean D. concern II. Choose the best option (A, B, C or D) to complete the passage below. (1 point) NATURE Since the world has become (1) ..................., there has been an increase in the number of animal species that have either become extinct or have neared (2) ................... . Bengal tigers, for instance, which once roamed the jungles in vast numbers, now (3) ...................only about 2,300 and by the year 2025 their population is estimated to be down to zero. What is (4) ................... about the case of the Bengal tiger is that this extinction will have been caused almost entirely by (5) ................... who, according to some sources, are not interested in material gain but in personal (6) ................... This is an example of the (7) ................... that is part of what is causing the problem of extinction. Animals like the Bengal tiger, as well as other (8) ...................species, are a valuable part of the world’s ecosystem. International laws (9) ................... these animals must (10) ................... to ensure their survival, and the survival of our planet. 1. A. industrialized B. industry C. industrial D. industrialize 2. A. extinctation B. extinctive C. extinct D. extinction 3. A. is numbering B. numbered C. number D. is numbered 4. A. alarmist B. alarmed C. alarm D. alarming 5. A. poacher B. poachers C. poachors D. poachor 6. A. gratificating B. gratify C. gratification D. gratifiation 7. A. callous B. callously C. callousness D. callosity 8. A. dangered B. endangered C. endanger D. dangerous 9. A. protecting B. protect C. protected D. is protected 10. A. be acted B. be enacted C. be enacting D. be acting III. Put the words in brackets into the correct forms. (1 point) Air travel has become much cheaper in recent years, and ……………… (1. TOUR) go on foreign holidays in large numbers every year. The huge travel and tourism industry brings money to many of the poorer countries around the world. The ……………… (2. ARRIVE) of large numbers of visitors every year requires very good organization and planning. 2 However, many countries have ……………..…….. (3. DEVELOPMENT) huge tourist center without much thought for ………………. (4. LOCATION) people and ………………. (5. WILD). IV. Supply the correct verb tenses of the verbs in brackets. (1 point) 1. She says that she (live) …………. in the countryside when she (be) ………..….. a child. 2. Oil (leak) ………….… from a petrol tanker onto the motorway, so the police (close) …………... off the section between Junctions 5 and 6. 3. Foreign currency (convert) ……….…… into sterling at a number of points in the city, but the best rate and lowest commisson charges (provide) …………. at banks. 4. Most of the news on the front page of both daily newspapers (concern) …………... the progress of the peace conference. 5. Why are you carrying that saw? ∼ I (shorten) ……….... the legs of the dining room table. V. Choose the best option (A, B, C, D) to complete each of the sentences below. (1 point) 1. The doctor vaccinated the baby...................polio. A. against B. with C. to D. without 2. She threw the stone...................the dog when it started to chase her. A. to B. toward C. at D. against 3. They learned to accept their stepmother...................time. A. on B. at C. all D. in 4. Have you ever lost...................anybody...................a game...................tennis? A. to/ in/ of B. to/ for/ of C. with/ in/ of D. to/ in/ in 5. Would you like to live...................an igloo...................some time? An igloo is a house that is made...................blocks...................snow or ice. It’s so romantic. A. in/ at/ of/ of B. in/ for/ of/ of C. in/ for/ of/ with D. in/ at/ by/ with VI. Finish each of the following sentences in such a way that it means exactly the same as the sentence printed before it (2 points) 1. Small fishing boats continued to sail out though the coastguard had warned them not to do so. Æ Despite ............................................................................................................................... 2. They’re willing to work hard and we are surprised about that. Æ We’re surprised.................................................................................................................. 3. If you have a better job, you get higher pay. Æ The better the job, .............................................................................................................. 4. They took away his passport so that he wouldn’t leave the country. Æ They prevented .................................................................................................................. 5. You will never meet anyone more generous than Mrs. Jones. Æ Mrs. Jones is the most ……………………………………………………………… VII. Rewrite the second sentence so that it has a similar meaning to the first one, using the word given. Do not change the word given. (2 points) 1. Your attitude will have to change if you want to succeed. (leaf) Æ ……………………………………………………………………………………………. 3 2. You couldn’t do anything more stupid than to give up your job now. (height) Æ ……………………………………………………………………………………………. 3. The trouble all came about because our computers crashed. (stemmed) Æ ……………………………………………………………………………………………. 4. Do you have to wear a uniform at school? (compulsory) Æ ……………………………………………………………………………………………. 5. The way many sportsmen behave in public influences their young fans. (impression) Æ ……………………………………………………………………………………………. PART 4. READING I. Read the text below and give the word which best fits each space. (1 point) After living (1) …. the threat of extinction for more than 30 years, the national bird of the United States has been granted an official reprieve, as the bald eagle and twenty- eight other animal and plant species have been earmarked (2) …. removal from America’s list of endangered species. The bald eagle, also known as the white-headed sea eagle, took pride of place at the top of a list of species likely to (3) …. taken off the endangered register in the coming years. The proposed “delistings” are being promoted by the US interior secretary to counter a growing feeling among Republicans that endangered species laws do not work. Charges of ineffectiveness have been (4) ….against these laws before, but more recently it has even been suggested that the situation may actually have been (5) ….. worse by them. The recovery of the bald eagle follows thirty-one years on the critical list. Its numbers had been (6) …. to fewer than five hundred through the use of pesticides that reacted adversely (7) …. its reproductive system. The number of nesting pairs is now estimated at five thousand. The interior secretary claims that the new list was a vindication of the legislation under which the eagle, a national symbol (8) …. originally from the Indians, and more than a thousand other species were protected.(A spokesperson denied that it was simply a publicity stunt timed to coincide with the swearing (9) .… of the president for his second term). Until now, few species have (10)…. been removed from the list. When they were, it was usually because they had become extinct. II. Read the passage below and circle the best answer (A, B, C or D) to each question. (3.0 points) The fertile valleys of the river Nile straddle the hot desert land of Egypt. Rain is relatively scarce, and the summers are scorching hot. Nevertheless the strip of land, known as the Cultivation, on either side of the Nile is reputed to be one of the most fertile places in the world. Its rich black soil is the result of accumulation of silt deposited by the annual flooding of the Nile thousands of years ago. From June to October, the river overflowed its banks. Modern damps were then constructed to control the flooding. The floodwaters left behind a rich sticky black mud which made it suitable for the cultivation of crops. As long as the soil was well irrigated, two or three crops could be grown in one season. 4 Its rich soil led to the growth and rise of the brilliant civilization of the ancient Egyptians on the Nile valley more than 5000 years ago. The earliest Egyptians had acquired the skills to till the land along the banks, drawing water from the Nile for irrigation purposes. As the villagers along the Nile became wealthy, they embarked on projects of digging ditches and constructing dams to control the floods. In about 3100 BC, the whole Egypt was united under the reign of King Menes. He and his descendants made up Egypt’s first ruling family, or dynasty. His rule led to the great development of the arts like writing, painting, architecture, and crafts. Egyptian power and influence were and influence were to last for the next 2000 years. The Egyptian kings had absolute powers. The king possesses all the land, and the peasants had to surrender part of their crops to the king. An army of officials and scribes did the task of collecting the exact amount of due from the individual farmers was united. Crops and livestock were often seen at the storehouses surrounding the royal palaces. In return for their uphill task, the king paid his officials and dishes out funds for huge irrigation projects. Egypt’s trade with the outside world stretches far and wide. In return for gold, copper, gemstones, and building stores, it purchases the goods that it did not have. Timber, resins, oils, silver and slaves came from Lebanon. From Deria and Anatolia came horses, while the blue stone called lapis lazuli was imported from Mesopotamia. Strong forts were constructed to protect overland trade routes. Egypt held Nubia in the south for almost 800 years. It served as Egypt’s most vital source of gold and slaves. 1. The civilization of the ancient Egyptians was brought about by ……….. A. the rich alluvial soil of the Nile B. Egypt’s first ruling family C. the highly-developed writing, painting, architecture and crafts D. Egypt’s trade with the outside world 2. Egyptian slaves came from ………. A. Mesopotamia B. Syria and Anatolia C. Lebanon D. Palestine 3. The king was wealthy because ………. A. huge irrigation projects were set up B. farmer paid him their tribute C. he was in absolute power D. he possessed all the land 4. The writer said that the annual flooding of the Nile ……….. A. helped to develop the arts B. caused an influx of foreign traders C. needed to be controlled D. led to the discovery of new mines 5. According to the passage, the king was wise ……… A. to important the things that Egypt did not have B. to pay for huge irrigation projects C. to control the land D. to own slaves and gold 6. In return for, in the first line, last paragraph, refers to ……….. A. the respect Egypt received B. how fast the profit came C. the protection the Egyptian army provided D. what Egypt exported 7. A suitable title for this passage is ……… A. The Civilization Of The Ancient Egyptians B. The Farming Methods Of The Egyptians C. The Might Of The Egyptian Army D. The Landscape Of The Egypt 8. We know the Egyptian Empire was powerful from the ………. A. way its army fought with other invaders 5 B. long period it extended its power and influence C. trade and business done with other nations D. number of crops grown in one season 9. Farm crops would be ……….if there was no water for irrigation. A. harvested B. unprotected C. destroyed D. unavailable 10. From June to October, the flood plains would be ………. A. inundated with floodwaters from the Nile B. open to the building of new dams C. left to the sowing of new crops D. raised to a higher level PART 5. WRITING (3 points) Write a paragraph (from 150 to 200 words) about the following topic: “ The advantages and disadvantages of living in a big city” ................................................................................................................................ ................................................................................................................................ ................................................................................................................................ ................................................................................................................................ ................................................................................................................................ ................................................................................................................................ ................................................................................................................................ ................................................................................................................................ ................................................................................................................................ ................................................................................................................................ ................................................................................................................................ ................................................................................................................................ ................................................................................................................................ ................................................................................................................................ ................................................................................................................................ ................................................................................................................................ ................................................................................................................................ ................................................................................................................................ ................................................................................................................................ ................................................................................................................................ ................................................................................................................................ --- THE END --6 SỞ GDĐT BẠC LIÊU ĐỀ CHÍNH THỨC (Gồm 02 trang) KỲ THI CHỌN HSG LỚP 10, 11 VÒNG TỈNH NĂM HỌC 2011 - 2012 * Môn thi: TIẾNG ANH * Lớp: 10 (Bảng B) * Thời gian: 180 phút (Không kể thời gian giao đề) HƯỚNG DẪN CHẤM PART 1. LISTENING 2 points. Each correct answer gets 0.2p 1. C 2. D 3. B 4. UA675 5. Tokyo 6. Johnstone 7. Bill 8. 2 9. 555-432 10. 3 PART 2. PHONETICS I. 1 point. Each correct answer gets 0.2p 1. A 2. B 3. C 4. D 5. B II. 1 point. Each correct answer gets 0.2p 1. C 2. A 3. A 4. C 5. A PART 3. LANGUAGE FOCUS I. 1 point. Each correct answer gets 0.1p 1. D 2. B 3. B 4. D 5. B 6. A 7. A 8. D 9. C 10. A II. 1 point. Each correct answer gets 0.1p 1. A 2. D 3. C 4. D 5. B 6. C 7. C 8. B 9. A 10. B III. 1 point. Each correct answer gets 0.2p 1. Tourists 2. arrival 3. developed 4. local 5. wildlife IV. 1 point. Each correct answer gets 0.2p 1. lived – was 2. is leaking – are closing 3. can be converted – are provided 4. concerns 5. am going to shorten V. 1 point. Each correct answer gets 0.2p 1.A 2.C 3.D 4.A 5.B VI. 2 points. Each correct answer gets 0.4p 1. Despite the coastguard’s warning, small fishing boats continued to sail out 2. We’re surprised about their willingness to work hard 3. The better the job, the higher the pay 4. They prevented him from leaving the country by taking away his passport. 5. Mrs. Jones is the most generous person / woman you have ever met. VII. 2 points. Each correct answer gets 0.4p 1. You will have to turn over a new leaf if you want to suceed. 2. It would be the height of your stupidity to give up your job now. 3. The trouble all stemmed from our computers crashing. 4. Is it compulsory for you to wear a uniform at school? 5. The way many sportsmen behave in public makes an impression on their young fans. PART 4. READING I. 1 point. Each correct answer gets 0.1p 6. reduced 1. under 7. with 2. for 8. taken/ borrwed 3. be 9. in 4. levelled/leveled 10. ever 5. made 1 II. 3 points. Each correct answer gets 0.3p 1. A 2. C 3. D 4. C 5. B 6. D 7. A PART 5. WRITING (3 points) 8. B 9. C 10.A From 2.75 to 3 : Excellent Natural Englsh with minimal errors and complete realisation of the task set. From 2 to 2,5 : Good Good vocabulary and structure. Errors non-basic. From 1.5 to 1.75 : pass Simple but accurate realisation of task. Not many errors Form 1 to 1.25 : Weak Reasonably correct with some serious errors. Form 0 to 0.75 : Very poor Incoherent.Errors showing lack of basic knowledge of English. --- HẾT--- 2 Së gi¸o dôc vµ ®µo t¹o h¶I d−¬ng K× thi chän häc sinh giái tØnh Líp 10 thpt n¨m häc 2012 - 2013 -----------------M«n thi : TiÕng Anh Thêi gian lµm bµi: 180 phót (§Ò thi gåm 05 trang) ®Ò chÝnh thøc Häc sinh lµm bµi vµo tê giÊy thi. PhÇn tr¾c nghiÖm: ChØ cÇn viÕt ch÷ c¸i A hoÆc B, C, D... PhÇn tù luËn : ViÕt ®Çy ®ñ theo yªu cÇu cña bµi. (ThÝ sinh kh«ng ®−îc sö dông bÊt cø tµi liÖu g×.) A. LISTENING (15 points) H−íng dÉn phÇn thi Nghe hiÓu: Bµi nghe gåm 2 phÇn, mçi phÇn thÝ sinh ®−îc nghe 2 lÇn. Më ®Çu vµ kÕt thóc mçi phÇn nghe cã tÝn hiÖu. Mäi h−íng dÉn cho thÝ sinh (b»ng tiÕng Anh) cã trong bµi nghe. Part 1. Listen to a conversation between an optometrist and a patient and fill in the form. Write no more than 3 words or numbers for each blank Patient record Time of appointment (1)………………………….. Given name Simon Anthony Family name (2)………………………….. Date of birth (3)………………………….., 1989 Address (4)………………………….., Adam Terrace, Wellington Name of insurance company (5)………………………….. Date of last eye test September 2006 Patient’s observations Problems: seeing the distance Part 2: You are going to hear a travel agent discussing the holiday booking with 2 customers. Listen to their conversation and decide whether the statements are True (T), False (F) or Not given (NG) Statements T F NG 6. They want to book a holiday for July 7. They have decided where to go for the holiday. 8. Both customers are free to travel in the first week. 9. Last year, both of them visited France 10. They would like to go to the mountains for skiing this year 11. They don’t want to go to Italy because the dates don’t suit them. 12. They don’t like to go to Sweden because there are no beaches 13. It would be 385 pounds for them to visit Portugal. 14. The customers prefer to visit Portugal by flight from London. 15. The flight stops at Manchester on the way to Portugal. B. PHONETICS (5 points). Choose the word that has a different stress pattern from the others. 16. A. community B. developing C. conditioner D. interested 17. A. continue B. importance C. different D. directed 18. A. medicines B. opposite C. pollution D. capable 19. A. preservation B. inspiration C. disposable D. popularity 20. A. exhausted B. atmosphere C. suspect D. computer 1 C. GRAMMAR- VOCABULARY-LANGUAGE FUNCTIONS I. Choose the word, phrase or expression which best completes each sentence (15 points). 21. Hoa: “Are you going to buy a new computer or just continue using the old one?” Mary: “_________” A. Yes, I am. B. Yes, I’d like one. Thank you. C. That’s impossible. I can’t afford a new one. D. Neither. I’m going to lease one. 22. Mr. Black: “What a lovely house you have!” Mr John: “_________” A. No problem B. Thank you. Hope you will drop in. C. Of course not, it’s not costly D. I think so. 23. You have never been to Italy, ________? A. have you B. haven’t you C. did you D. had you 24. You can’t tell what someone is like just from their ________. A. character B. looking C. appearance D. personality 25. “How is it going?” - “________” A. By bike B. Not much C. It sounds better D. Mustn’t grumble 26. ________ a dentist, Mike is very concerned about having healthy teeth. A. Because B. He is C. As D. That he is 27. ________, you need to achieve a score of 60% or more. A. To pass this test B. For being passed this test C. In order pass this test D. So that to pass this test 28. As a famous person ________ many children admire, it is important for her to act responsibly. A. whose B. whom C. which D. when 29. The brochure says that the hotel has a great ________ of the sea. A. appearance B. look C. sight D. view 30. Our new coach is popular ________ the whole team. A. with B. to C. by D. for 31. As soon as you ________ that, I’d like you to go to bed. A. have done B. did C. will do D. will have done 32. Margaret was slow at school, but she went on ________ Prime Minister. A. being B. to be C. having been D. to have been 33. In 1870, ________, John D. Rockefeller and others created the Standard Oil Company. A. in spite of oil prices fluctuated B. despite fluctuating oil prices C. but the oil prices fluctuated D. oil prices were fluctuating 34. They’re staying in rented accommodation for the time ________. A. going B. making C. doing D. being 35. “I have an idea. Let’s go for a swim on Sunday afternoon”. - “________” A. OK, what time? B. You’re kidding C. I know D. I’m sure II. Give the correct form of the words in brackets (10 points). 36. We found it ____________ (thrill) to your wonderful news. 37. He left the room without any ____________ (explain). 38. He didn’t feel happy because he worked ____________ (success). 39. Many people expressed ____________ (disagree) with the whole idea. 40. There was a ____________ (wide) dissatisfaction with the government’s policies. 41. Her health has ____________ (bad) considerably since we last saw her. 42. A lot of plants and animals could be used as medicines against cancer, AIDS, heart diseases and other ____________ (sick). 43. He was ____________ (information) of the consequences in advance. 44. I was kept ____________ (wake) last night by the noise from a party in the flat above. 45. This road is so bad that it needs ____________ (surface). III. Choose the underlined word or phrase in each sentence that needs correcting (5 points). 46. Last week unless my mother had had enough money, she would have bought that toy for me. A B C D 47. It often takes me about fifteen minutes to go to work from here by foot. A B C D 48. Those people say that it is such polluted air that they can’t breath, don’t they? A B C D 2 49. It is noisy enough in this room, so I would rather you stop shouting like that. A B C D 50. Dr. Roberts, the first woman to be elected president of the university, is intelligent, capable and A B awareness of the problem to be solved. C D D. READING I. There are 5 blanks in the passage below. From the words given in the box, choose the most suitable for each blank. There are more words than blanks, so you don’t need all of them. (5 points). A. terrible B. size C. hope D. wrong E. loose F. problem G. want Choosing clothes can be difficult. Some people (51)________ to be fashionable, but they don’t want to look exactly like everybody else. Not all clothes are suitable for work or school, perhaps because they are not formal enough, or simply not comfortable. It is easy to buy the (52)_________ size, and find that your trousers are too tight, especially if you are a little bit overweight. Very (53)________ clothes make you feel slim, but when they have shrunk in the washing machine, then you have the same (54)________! If you buy light cotton clothes, then they might not be warm enough for winter. If your shoes are not tight, and if you aren’t dressed for the cold, you might look good, but feel (55)________! II. Read the following passage and decide which option A, B, C or D best fits each space. (10 points) Schools in the United States have not always had a large number of libraries. As (56)________as 1958 about half of the public schools in the United States had no libraries at all. The number of public school libraries increased dramatically (57)_______ the federal government passed the Elementary and Secondary Education Act of 1965, ( 58)_______ provided funds for school districts to improve their education programs and facilities, including their libraries. (59) ________, many educators claim that since the legislation was passed federal spending has not increased sufficiently to meet the rising (60)______ of new library technologies such as computer databases and Internet access. Because the federal government provides only limited funds to schools, individual school districts (61) _______ on funds from local property taxes to meet the vast majority of public school expenses. Therefore, the libraries of the public schools tend to reflect the (62) ______ capabilities of the communities in which they are located. Districts in wealthy suburbs often have fully staffed libraries (63) _______ abundant resources, spacious facilities, and curricular and instructional support. In (64) ______, school districts in many poor areas house their libraries in ordinary classrooms or in small rooms. The libraries in such areas are generally staffed by volunteers, who organize and (65) ______ books that are often out-of-date, irrelevant, or damaged. D. newly B. recently C. frequently 56. A. freshly C. during D. when 57. A. though B. with D. this B. who C. which 58. A. that D. Otherwise C. Consequently B. Therefore 59. A. Nevertheless D. sum C. cost B. fee 60. A. fine D. stay C. rely B. come 61. A. go D. financial C. political B. economical 62. A. educational D. by B. with C. on 63. A. for C. contrast D. conflict 64. A. country B. converse D. maintain C. contain B. obtain 65. A. attain III. Read the passage and choose the correct answer A, B, C or D(10 points). Before the mid-nineteenth century, people in the United States ate most foods only in season. Drying, smoking and salting could preserve meat for a short time, but the availability of fresh meat, like that of fresh milk, was very limited; there was no way to prevent spoilage. But in 1810, a French inventor named Nicolas Appert developed the cooking-and-sealing process of canning. And in the 1850’s an American named Gail Borden developed a means of condensing and preserving milk. Canned goods and condensed milk became more common during the 1860’s, but supplies remained low because cans had to be made by hand. By 1880, however, inventors had fashioned stamping and soldering machines 3 that mass-produced cans from tinplate. Suddenly all kinds of food could be preserved and bought at all times of the year. Other trends and inventions had also helped make it possible for Americans to vary their daily diets. Growing urban population created demand that encouraged fruit and vegetable farmers to raise more produce. Railroad refrigerator cars enabled growers and meat packers to ship perishables great distances and to preserve them for longer periods. Thus, by the 1890’s, northern city dwellers could enjoy southern and western strawberries, grapes, and tomatoes, previously available for a month at most, for up to six months of the year. In addition, increased use of iceboxes enabled families to store perishables. As easy means of producing ice commercially had been invented in the 1870’s, and by 1900 the nation had more than two thousand commercial ice plants, most of which made home deliveries. The icebox became a fixture in most homes and remained so until the mechanized refrigerator replaced it in the 1920’s and 1930’s. Almost everyone now had a more diversified diet. Some people continued to eat mainly foods that were heavily in starches or carbohydrates, and not everyone could afford meat. Nevertheless, many families could take advantage of previously unavailable fruits, vegetables, and dairy products to achieve more varied fare. 66. What does the passage mainly discuss? A. Causes of food spoilage B. Commercial production of ice C. Population movements in the nineteenth century D. Inventions that led to changes in the American diet 67. The phrase “in season” in line 1 refers to A. a particular time of year B. a kind of weather C. an official schedule D. a method of flavoring 68. During the 1860’s, canned food products were A. unavailable in rural areas B. available in limited quantities C. shipped in refrigerator cars D. a staple part of the American diet. 69. It can be inferred that railroad refrigerator cars came into use A. before 1860 B. before 1890 C. after 1900 D. after 1920 70. The word” them” in line 13 refers to A. refrigerator cars B.growers C. perishables D. distances. 71. The word” fixture” in line 18 is closest in meaning to A. commonplace object B. substance C. luxury item D. mechanical device 72.The author implies that in the 1920’s and 1930’s home deliveries of ice A. increased in cost B. occurred only in the summer C. decreased in number D. were on an irregular schedule 73. The word “ Nevertheless” in line 21 is closest meaning to A. occasionally B. however C. therefore D. because 74. Which of the following types of food preservation was NOT mentioned in the passage? A. Drying B. Chemical additives C. Canning D. Cold storage 75. Which of the following statements is supported by the passage? A. Most farmers in the United States raised only fruits and vegetables. B. People who lived in cities demanded home delivery of foods. C. Tin cans and iceboxes helped to make many foods more widely available. D. Commercial ice factories were developed by railroad owners E. Writing I. Finish each of the following sentences in such a way that it means exactly the same as the sentences printed before. (5 points) 76. Having nothing else to do, we decided to go for a walk. - Since............................................................................................................................. 77. She was not only bad-tempered but also very lazy. - As well ........................................................................................................................ 78. “ Nothing will persuade me to apply for that kind of job.” She said. _ She flatly....................................................................................................................... 79. The school I studied at last year was better than this one. - This school isn’t............................................................................................................. 80. You must leave now, or you’ll miss the bus. - You’ll miss...................................................................................................................... 4 II. Use the suggested words and phrases to write complete sentences of a letter. (10 points) Dear Sir or Madam. 81. I/ write/ complain/ hair drier/ buy/ your shop/ last Saturday/ and/ treatment/ I receive/ when/ I try/ return/ a few days later. 82.I buy/ hair drier/ Wednesday, November 22nd. 83. first time/ try/ use/ handle become/ extremely hot/ and within a few minutes/ part/ plastic casing/ begin/ melt. 84. I turn/ off/ immediately/ return/ with/ receipt/ your shop/ Saturday. 85. I explain/ situation/ one/ assistants/ ask/ money back/ but / be told/ speak/ you. 86. Unfortunately you/ not available/ that day/ I/ write instead. 87. I enclose/ hair drier/ copy of/ original receipt. 88. Please send/ full fund/ soon/ possible. Yours faithfully, III. Write a paragraph of about 120 words on the following topic: ( 10 points) What club do you want to be founded in your school? Why? ========== The end ========== Hä vµ tªn thÝ sinh: ................................................... Sè b¸o danh ................ Hä tªn, ch÷ ký gi¸m thÞ 1: .................................. Hä tªn, ch÷ ký gi¸m thÞ 2: ................................. 5 HƯỚNG DẪN CHẤM ĐỀ THI HỌC SINH GIỎI LỚP 10 M«n TiÕng Anh - n¨m häc 2012-2013 A. LISTENING: ( 15 points) ( 1 point for each correct answer) Part 1. 1. 10 am 2. Lee 3. June 1st 4. University Hall 5. Health for Life Part 2: 6. T 7. F 8. F 9. T 10. NG 11. T 12. NG 13. T 14. F 15. NG B. PHONETICS: ( 5 points) ( 1 point for each correct answer) 16. D 17. C 18. C 19. C 20. B C. GRAMMAR – VOCABULARY – LANGUAGE FUNCTIONS. I. Choose the word, phrases or expression which best completes each sentence. ( 15 points) ( 1 point for each correct answer) 21. D 29. D 22. B 30.A 23. A 31. A 24. C 32. B 25. D 33. B 26. A 34. D 27. A 35.A 28.B II. Give the correct form of the words in brackets. (10 points) (1 point for each correct answer) 36. THRILLING 37. EXPLANATION 38. UNSUCCESSFULLY 39. DISAGREEMENTS 40. WIDESPREAD 41. WORSENED 42. SICKNESSES 43. INFORMED 44. AWAKE 45. RESURFACING III. Choose the underlined word or phrases in each sentence that needs correcting. ( 5 points) (1 point for each correct answer) 46.A 47. D 48. C 49. D 50.C D. READING: I. There are 5 blanks in the passage below. From the words given in the box, choose the most suitable for each blank. There are more words than blanks, so you don’t need all of them. ( 5 points) (1 point for each correct answer) 51. G 52.D 53.E 54.F 55.A II. Read the following passage and decide which option A, B, C or D best fits each space. ( 10 points) (1 point for each correct answer) 56. B 57. D 58. C 59. A 60. C 61. C 62. D 63. B 64. C 65. D III. Read the passage and choose the correct answer. ( 10 points) (1 point for each correct answer) 66.D 67. A 68. B 69.B 70.C 71.A 72.C 73.B 74.B 75.C E. WRITING. I. Finish each of the following sentences in such a way that it means exactly the same as the sentence printed before it. ( 5 points). ( 1 point for each correct answer) 76. Since we had nothing else to do, we decided to go for a walk. 6 Or (Since we didn’t have anything else to do, we decided to go for a walk.) 77. As well as being bad-tempered, she’s very lazy. 78. She flatly refused to apply for that kind of job. 79. This school isn’t as/so good as the one ( that/which) I studied at last year. Or (This school isn’t as/so good as the one at which/ where I studied last year.) 80. You’ll miss the bus if you don’t leave now. Or (You’ll miss the bus unless you leave now.) II. Use the words or phrases suggested to write full sentences to make a letter. ( 10 points) Dear Sir or Madam. 81. I am writing to complain about the/a hair drier (0.5)( which/that I) bought in your shop last Saturday(0.5) and the treatment (which/that) I received (0.5)when trying/I tried to return it a few days later.(0.5) 82. I bought the hair drier(0.5) on Wednesday, November 22nd.(0.5) 83. The first time I tried to use it,(0.5) the handle became extremely hot (0.5)and within a few minutes part of the plastic casing began to melt.(0.5) 84. I turned it off immediately (0.5)and returned it with the receipt to your shop on Saturday.(0.5) 85. I explained the situation to one of the/ your assistants (0.5)and asked for my/the money back(0.5) but I was told (that I had) to speak to you.(0.5) 86. Unfortunately, you were not available that day,(0.5) so I am writing instead.(0.5) 87. I enclose the hair drier(0.5) and a/the copy of the original receipt.(0.5) 88. Please send me a full refund (0.5) as soon as possible (0.5) Yours faithfully. III. Write a paragraph of about 120 words on the following topic. ( 10p) Gîi ý phÇn cho ®iÓm chi tiÕt nh− sau: 1. Form: (paragraph) (1 point) (§o¹n v¨n kh«ng xuèng dßng - §ñ sè tõ ) 2. Content: (4 points ) Good topic sentence (0.5 p) and appropriate supporting ideas (3.5 points) 3. Language: (5 pts) + Appropriate vocabulary (1.5p) + Suitable connectors (0.5) + Correct grammar (2.5 p) + Punctuating/ Spelling (0.5p) Chó ý: Tæng sè ®iÓm lµ 100, sau ®ã qui vÒ thang ®iÓm 10, vµ lµm trßn ®Õn sè thËp ph©n thø nhÊt. ______________________ 7 SỞ GD&ĐT HÀ NỘI CỤM TRƯỜNG THPT BA ĐÌNH – TÂY HỒ ĐỀ THI OLYMPIC NĂM HỌC 2011-2012 Môn Tiếng Anh - Lớp 10 Thời gian làm bài: 120 phút, không kể thời gian giao đề. Đề thi gồm 05 trang. Đề thi gồm 05 trang. Thí sinh làm bài vào tờ giấy thi và không được dùng bất cứ loại từ điển nào Question 1→10. Choose and circle the word or phrase that best completes each sentence 1. I was passing their house, so I __________Claire and Michael A dropped in B came up with C got on with D run into 2. I don’t really___________winter sports very much A deal with B face up to C go in for D get round to 3. The attorney told his client that___________ A they had little chance of winning the case B it was nearly impossible to win him the case C the case was of a small chance to win D the case had a minimum chance to be won by him 4. One of the professor’s greatest attributes is_____________ A when he gives lectures B how in the manner that he lectures C the way which give lectures D his ability to lecture 5.____________better working conditions A What’s interesting the workers most is B What’s interested the workers most is C What interest the workers most is D What interests the workers most is 6. The concert was given____________the auspices of the Y.M.C.A. A on B with C through D under 7. Her mother’s illness cast a cloud__________her wedding day A over B for C in D on 8. She did six hours’____________studying a day for her exam A solid B heavy C strong D big 9. The weather was fine and everyone was________the coast A going in for B making for C joining in D seeing about 10. There isn’t any sugar, I’m afraid. You’ll have to_________ A run out B put up with C do without D make for Question 11→20. Replace each italicized verb or phrase with the correct form of a synonymous phrasal verb. back down bring up crop up dwell on fall off get round have on look to make out pick up run across stand up to take to track down turn out 11. One of the boys from my class objects to taking part in the performance and I don’t really know how to persuade him. __________ 12. The question was so delicate and problematic that there was nobody willing to mention it. ________ 13. Your report deals too much with the side effects of the venture. Why not write more about its good points? _______ 14. This automobile manufacture produces about a thousand cars a year, half of which are exported. ____ 15. Even the extensive advertising campaign hasn’t prevented the demand for our printers from decreasing. _________ 16. “She says she’s going to be a medicine student” “Oh, don’t believe her. She’s deceiving you”. ________ 17. I’d had no idea there were jewels hidden in my attic. I just found them completely by chance. _____ 18. Don’t worry, he only pretends that he is such a strict teacher. _____ 19. After two-hour search, the hunters decided to give up following the stags. ______ 20. We were convinced Patrick would get to like the moment he met him.______ Question 21→30. Give the correct form of the words given - A Musical Genius For many people Ludwig van Beethoven (1770-1827) is the most influential figure in the history of western classical music. His (21. ORDINARY)___ talent was already clearly evident as a young man, (22. MERCY)___ surviving a somewhat unconventional (23. BRING)___ during which his eccentric father would often force him to take music lessons in the middle of the night. The young Beethoven's ability won him the admiration of the leading contemporary musical figures. Throughout the 1790s he worked hard to secure the interest of wealthy patrons. Such patronage (24. ABLE)___ him to concentrate on becoming a successful composer. Whatever his aweinspiring musical (25. ACHIEVE)___, however, his personal life was something of a disaster. His day-to-day (26. RELATE) ___ with people invariably turned out to be rather turbulent. Although he apparently fell in love with a 1 number of society women, the identity of the girl who lay closest to his heart remains (27. ELUDE)__ to this day. However, just at the point when Beethoven was beginning to reap the rewards of his early endeavours, he had to come to terms with the crushing (28. REALISE)___that his increasing deafness was (29. CURE)____ . From that point on, his music displayed a (30. STRIKE)___change in style, becoming both heavier in tone and larger in scale. Q31→35. Think of ONE word only which can be used appropriately in all THREE sentences. 31 - You need to____all applications before shutting down your computer - Should the factory____down, 100 people will lose their jobs - Mike’s had three car accidents, plus a few other pretty____shaves 32 - It’s just a ____to them. They don’t care what happens - John gave the____away by laughing when Jim walked in - We’ve been in this____for twenty years 33 - Susan is not the____to get annoyed - Open up the accounts database, and____in the amount of each order - Your hair____requires a specific shampoo 34 - We sat and talked by the____of the fire - They set out at first____for the summit - These discoveries may throw new____on the origins of man. 35 - Could you do me a ____and hold the door open while I bring in the shopping? - All those in____of the proposal please raise your hand. - After being out of fashion for years the painter is now in ____ with the critics. Q 36 → 40. Choose the word in each group that has different stress pattern 36. A graduate B impression C applicant D category 37. A beautify B geology C photograph D enemy 38. A classify B humidify C personify D solidify 39. A academic B fantastic C terrific D historic 40. A sacrifice B compliment C nursery D determine Q 41→45 Choose one word whose underlined part is pronounced differently from the rest. 41. A. brown B. down C. now D. shown 42. A. stagnant B. expand C. eventually D. landlord 43. A. compete B. fetch C. separate D. yell 44. A. minute B. muddy C. fu n n y D. number 45. A. died B. continued C. contented D. followed Q46 →57 Fill in each blank with one suitable word Too Good to be True Some say you can do it in seven days, others promise success in 24 hours, or you may prefer to take your time and do it in 5 weeks. What are we talking about? Believe it (46)__ not, these periods (47) __time refer to language learning courses that promise excellent results in less time (48)__it takes to say 'Bonjour'! However, the advertisements of these companies are not necessarily (49) __best guide and if you don't know what to look for in a good course you could be left with little (50) __than a large bill. A complaint was recently made to a consumer rights group (51) __ WhizzLearn Systems, a language school chain, and the company has been forced to remove its claim that its technique is ten times better than any (52) __method. The Managing Director of WhizLearn Systems said “(53)__ we still believe our claim is true, we are willing to change our advert. But the fact remains, if you spend 3 hours (54) __day for 5 weeks on our language course you will soon be speaking the language. What we (55) __have done was to make (56) __ clearer in the advert. Then we wouldn't (57) __had these annoying complaints" Q58→64. Choose the most suitable sentence from the list A-H for each part (58-64) of the article. There is one extra sentence which you do not need to use. Career success in the arts John Prince, famous dancer and choreographer, gives advice on how to succeed in a career in the arts. I asked John how he got started and what requirements there are. "Well, to be a professional dancer it's useful to have had acting lessons or some background in drama. If you want to succeed in musical theatre you have to have a good singing voice as well. When you approach an agent you should take a portfolio with your CV, your statistics sheet and some good photos and reviews of past performances. You'll need dance clothes, ballet shoes, tap shoes, and even roller skates depending on what kind of show you are going to go for." 58 ____ "Of course, you need to be extremely fit if you want to be a professional dancer. I dance or move about for about six hours a day. There are great health benefits to being a dancer. I can eat a lot of pasta without gaining weight because dancing increases your metabolism so much." 59____ John has a very busy schedule in the next few months. He took time out to speak to me today from the making of a pop video to promote N-ergy's latest record. "I choreographed the dance routine for the boys and they only had 2 days in which to learn it! I am going to be working on a video for another 2 well known band - but that's top secret. Next month I'll be touring Spain in a production of a musical that was written by a friend of mine, Michaela Evans. 60____As for the future, I've come to realise that I would never be content to be just a chorus dancer - I'm too much of an individual for that. Like all artists I'd love to become a household name by writing and choreographing my own musicals." John was born in Jamaica to a Jamaican father and a Scottish mother but the family emigrated to England 20 years ago. "I have a little sister I adore, who is also training to be a dancer." How does it feel to have someone else following in your footsteps? 61____ Has he much more to learn, I wondered. "I've spent an incredible amount of my life training to get where I am. I went to college for two years in England, I trained for six months in Paris and about eight months in America. But you never really stop training or learning your art." 62____ So, would you say it's been plain sailing? "I feel I've been lucky to a degree; many people hit problems breaking into the arts. It can be a vicious circle really. You can't become a member of Equity, which is the actors' and dancers' union, without good contracts and you can't get good contracts without being a member of Equity. My advice to people who want to get into the arts would be to go out into the world, and try everything else first. 63 ____ What has a dance career done for you as a person? "Thanks to dancing, I've visited and performed in 23 countries so far. This has opened my eyes to the world, and I've been able to understand issues like racism and inequality from a wider perspective. 64 ____ "So all in all I'm really happy to be a dancer!" A It's fine, but I try not to give out too much advice as it gets irritating! B And if nothing you like comes out of it, then come back and be an actor or dancer. C Without a strict daily timetable like this you find yourself wasting too much time. D After that it's back to England to start a new term of dance classes. E Hopefully this has enabled me to become a better and more tolerant person as a result. F When it comes to coping with stress, I find that exercise helps me to cope with my problems, so I stay in good shape mentally as well. G Like any profession where you're always travelling, you tend to acquire something new almost every day. H Being fully equipped with all this stuff beforehand makes it easier when you go for auditions. Q65 →79 Choose the appropriate words to complete the following passage WHALES How far would you travel for a good meal? If you were a humpback whale, the answer would be five thousand miles. These large sea animals travel at least that far from their winter home off the (65)__ of Columbia to their summer (66) __ areas off Antarctica. The distance covered by some types of whale is amazing, especially when you (67) __ their enormous size. The blue whale is the largest animal that has ever (68) __ and it can weigh as much as thirty elephants. It (69) __ as no surprise, therefore, to hear that ancient folk legends tell of sailors (70) __ these creatures for islands. The more we (71) __ about whales, the more wonderful they seem. Some species can (72) __ their breath for more than an hour and dive to a (73) __ of over 200 metres. They use a system of sounds (74) __ as echo-location to find the fish they eat and they have further sounds to keep in (75) __ with each other. The noises they (76) __ can travel hundred of miles under water. Some species seem to sing complicated songs which (77) __ of a number of separate themes, sung in a specific order which can (78) __ up to half an hour or more. If you ever get the (79) __ to see one of these great creatures in the wild, you will understand why they have inspired so many legends. 65 A beach B coast C seaside D ground 66 A feeding B dining C eating D chewing 67 A view B think C consider D believe 68 A been B stayed C born D lived 69 A gets B comes C goes D seems 70 A supposing B mixing C confusing D mistaking 71 A find out B look out C show up D turn up 72 A store B keep C hold D save 73 A depth B length C width D breadth 74 A called B known C referred D named 75 A reach B touch C call D range 76 A provide B propose C process D produce 77 A consist B compose C include D involve 78 A long B last C play D give 79 A break B choice C chance D luck Q80 →87. You are going to read three different opinions from leading scientists about the future of fuel. For the questions below, choose from the writers A-C. The writers may be chosen more than once. A. Howard Bloom, Author: Even though most people are convinced that peak oil has already passed, to me, peak oil is just a hypothesis. There is a theory that carbon molecules can be found in interstellar gas clouds, comets and in space ice, and if this is the case, our planet could ooze oil for ever. And even if we stay earthbound, those who 3 say we have raped the planet of all its resources are wrong. There's a huge stock of raw materials we haven't yet learned to use. There are bacteria two miles beneath our feet which can turn solid granite into food. If bacteria can do it, surely we creatures with brains can do it better. As far as the near future of energy is concerned, I believe the most promising alternative fuels are biofuels, such as ethanol. It's an alcohol made from waste products such as the bark of trees, woodchips, and other 'waste materials'. And that's not the only waste that can create energy. My friend in the biomass industry is perfecting an energy-generation plant which can run on human waste. We produce that in vast quantities, and it's already gathered in centralised locations. B. Michael Lardelli, Lecturer in Genetics at The University of Adelaide: Nothing exists on this planet without energy. It enables flowers and people to grow and we need it to mine minerals, extract oil or cut wood and then to process these into finished goods. So the most fundamental definition of money is as a mechanism to allow the exchange and allocation of different forms of energy. Recently, people have been using more energy than ever before. Until 2005 it was possible to expand our energy use to meet this demand. However, since 2005 oil supply has been in decline, and at the same time, and as a direct result of this, the world's economy has been unable to expand, leading to global recession. With the world's energy and the profitability of energy production in decline at the same time, the net energy available to support activities other than energy procurement will decrease. We could increase energy production by diverting a large proportion of our remaining oil energy into building nuclear power stations and investing in renewable forms of energy. However, this is very unlikely to happen in democratic nations, because it would require huge, voluntary reductions in living standards. Consequently, the world economy will continue to contract as oil production declines. With energy in decline, it will be impossible for everyone in the world to become wealthier. One person's increased wealth can only come at the expense of another person's worsened poverty. C. Jeroen van der Veer, chief executive of Royal Dutch Shell: People are understandably worried about a future of growing energy shortages, rising prices and international conflict for supplies. These fears are not without foundation. With continued economic growth, the world's energy needs could increase by 50% in the next 25 years. However, I do not believe that the world is running out of energy. Fossil fuels will be able to meet growing demand for a long time in the future. Taking unconventional resources into account, we are not even close to peak oil. The priority for oil companies is to improve efficiency, by increasing the amount of oil recovered from reservoirs. At present, just over a third is recovered. We can also improve the technology to control reservoir processes and improve oil flow. However, these projects are costly, complex and technically demanding, and they depend on experienced people, so it is essential to encourage young people to take up a technical career in the energy industry. Meanwhile, alternative forms of energy need to be made economically viable. International energy companies have the capability, the experience and the commercial drive to work towards solving the energy problem so they will play a key role. But it is not as simple as merely making scientific advances and developing new tools; the challenge is to deliver the technology to people worldwide. Companies will need to share knowledge and use their ideas effectively. Which writer 80. believes oil will be available for many more years _____ 81. believes that from now on, less oil is available _____ 82. believes there are ways to obtain energy that we have not yet discovered _____ 83. sees a great potential in natural fuels _____ 84. believes the fuel crisis will cause the poor to become poorer _____ 85. sees energy and the economy as intrinsically linked _____ 86. believes that better technology can help to maintain oil production levels _____ 87. thinks that oil companies are responsible for developing other types of energy _____ Q 88→ 94. Complete the following sentences so that the second sentence has a similar meaning to the first sentence. Do not change the word given. You must use between two and five words, including the words given. 88. On business trips, I prefer driving home to staying in a hotel overnight rather On business trips, I’d __________________________ in a hotel overnight 89. Jack found it difficult to control his skis on the steep slope. under Jack found it difficult to_________________________ on the steep slope. 90. My views on this subject are exactly the same as yours. difference There ________________________ my views on this subject and yours. 91. They will be selling tickets for the concert during the lunch hour. sale Tickets for the concert will __________________ during the lunch hour. 92. I regret not contacting Brian when I was in Dublin. touch I wish that I _____________________ Brian when I was in Dublin. 93. In my opinion, these two kinds of music are completely different from each other comparison In my opinion, there is _________________________these two kinds of music. 94. I arrived late because I missed the 10.30 train turned 4 If I’d caught the 10.30 train, I _____________________time Q 95→ 100. Complete the following sentences so that the second sentence has a similar meaning to the first sentence, using the word given. Do not change the word given. 95. Nobody expected her to lose, but she did. against _____________________________________________________ 96. Peter always trusts me with his secrets in Peter_________________________________________________ 97. I thought it was marvellous that Jane could jump so high. at I _____________________________________________________ 98. Danny was asked to leave the school for bad behaviour from Danny was _____________________________________________ 99. They didn’t punish Karen, only gave her a warning got Karen _________________________________________________ 100. Gary is proud of the fact that he is never late. on Gary prides_____________________________________________ ------------------------THE END ------------------- 5 HƯỚNG DẪN CHẤM ĐỀ THI OLYMPIC KHỐI 10 - MÔN THI: Tiếng Anh Tổng số câu toàn bài 100 x 0,2= 20 điểm. Làm tròn đến 0,25 1. A dropped in 44 A minute 2. C go in for 45 C contented 3. A they had little chance of 46 or 4. D his ability to lecture 47 of 5. D What interests the workers most is 48 than 6. D under 49 the 7. A over 50 more 8. A solid 51 against/ about/ concerning 9. B making for 52 other/ similar 10. C do without 53 Although/ While/ Whilst 11. get round 54 a/ per/ every 12. bring it up 55 should/ could 13. dwells on 56 it/ this/ things 14. turns out 57 have 15. falling off 58 H 16. having you on 59 F 17. ran across 60 D 18. makes out 61 A 19. tracking down 62 G 20. take to 63 B 21 extraordinary 64 E 22 mercifully 65 B coast 23 upbringing 66 A feeding 24 enabled 67 C consider 25 achievement (s) 68 D lived 26 relationships 69 B comes 27 elusive 70 D mistaking 28 realization 71 A find out 29 incurable 72 C hold 30 striking 73 A depth 31 close 74 B known 32 game 75 B touch 33 type 76 D produce 34 light 77 A consist 35 favour 78 B last 36 B impression 79 C chance 37 B geology 80 C 38 A classify 81 B 39 A academic 82 A 40 D determine 83 A 41 D shown 84 B 42 C eventually 85 B 43 A compete 86 C 87 C 88 rather drive home than stay 89 keep/ get/ have his skis under control 90 is no difference between/ is not any difference between 91 be/ go on sale 92 had been/ got/ kept in touch with 93 no (possible) comparison (at all) between 94 would have turned up on/ in time 95. Against everybody's expectation, she lost. 96. Peter always confides in me 97. I was marvelled at Jane’s ability to jump so high 98. Danny was expelled from school for bad behaviour 6 99. Karen got off with a warning 100. Gary prides himself on always being early 7 SỞ GD&ĐT HÀ NỘI CỤM TRƯỜNG THPT BA ĐÌNH – TÂY HỒ ĐỀ THI OLYMPIC NĂM HỌC 2011-2012 Môn Tiếng Anh - Lớp 10 Thời gian làm bài: 120 phút, không kể thời gian giao đề. Đề thi gồm 05 trang. Đề thi gồm 05 trang. Thí sinh làm bài vào tờ giấy thi và không được dùng bất cứ loại từ điển nào Question 1→10. Choose and circle the word or phrase that best completes each sentence 1. I was passing their house, so I __________Claire and Michael A dropped in B came up with C got on with D run into 2. I don’t really___________winter sports very much A deal with B face up to C go in for D get round to 3. The attorney told his client that___________ A they had little chance of winning the case B it was nearly impossible to win him the case C the case was of a small chance to win D the case had a minimum chance to be won by him 4. One of the professor’s greatest attributes is_____________ A when he gives lectures B how in the manner that he lectures C the way which give lectures D his ability to lecture 5.____________better working conditions A What’s interesting the workers most is B What’s interested the workers most is C What interest the workers most is D What interests the workers most is 6. The concert was given____________the auspices of the Y.M.C.A. A on B with C through D under 7. Her mother’s illness cast a cloud__________her wedding day A over B for C in D on 8. She did six hours’____________studying a day for her exam A solid B heavy C strong D big 9. The weather was fine and everyone was________the coast A going in for B making for C joining in D seeing about 10. There isn’t any sugar, I’m afraid. You’ll have to_________ A run out B put up with C do without D make for Question 11→20. Replace each italicized verb or phrase with the correct form of a synonymous phrasal verb. back down bring up crop up dwell on fall off get round have on look to make out pick up run across stand up to take to track down turn out 11. One of the boys from my class objects to taking part in the performance and I don’t really know how to persuade him. __________ 12. The question was so delicate and problematic that there was nobody willing to mention it. ________ 13. Your report deals too much with the side effects of the venture. Why not write more about its good points? _______ 14. This automobile manufacture produces about a thousand cars a year, half of which are exported. ____ 15. Even the extensive advertising campaign hasn’t prevented the demand for our printers from decreasing. _________ 16. “She says she’s going to be a medicine student” “Oh, don’t believe her. She’s deceiving you”. ________ 17. I’d had no idea there were jewels hidden in my attic. I just found them completely by chance. _____ 18. Don’t worry, he only pretends that he is such a strict teacher. _____ 19. After two-hour search, the hunters decided to give up following the stags. ______ 20. We were convinced Patrick would get to like the moment he met him.______ Question 21→30. Give the correct form of the words given - A Musical Genius For many people Ludwig van Beethoven (1770-1827) is the most influential figure in the history of western classical music. His (21. ORDINARY)___ talent was already clearly evident as a young man, (22. MERCY)___ surviving a somewhat unconventional (23. BRING)___ during which his eccentric father would often force him to take music lessons in the middle of the night. The young Beethoven's ability won him the admiration of the leading contemporary musical figures. Throughout the 1790s he worked hard to secure the interest of wealthy patrons. Such patronage (24. ABLE)___ him to concentrate on becoming a successful composer. Whatever his aweinspiring musical (25. ACHIEVE)___, however, his personal life was something of a disaster. His day-to-day (26. RELATE) ___ with people invariably turned out to be rather turbulent. Although he apparently fell in love with a 1 number of society women, the identity of the girl who lay closest to his heart remains (27. ELUDE)__ to this day. However, just at the point when Beethoven was beginning to reap the rewards of his early endeavours, he had to come to terms with the crushing (28. REALISE)___that his increasing deafness was (29. CURE)____ . From that point on, his music displayed a (30. STRIKE)___change in style, becoming both heavier in tone and larger in scale. Q31→35. Think of ONE word only which can be used appropriately in all THREE sentences. 31 - You need to____all applications before shutting down your computer - Should the factory____down, 100 people will lose their jobs - Mike’s had three car accidents, plus a few other pretty____shaves 32 - It’s just a ____to them. They don’t care what happens - John gave the____away by laughing when Jim walked in - We’ve been in this____for twenty years 33 - Susan is not the____to get annoyed - Open up the accounts database, and____in the amount of each order - Your hair____requires a specific shampoo 34 - We sat and talked by the____of the fire - They set out at first____for the summit - These discoveries may throw new____on the origins of man. 35 - Could you do me a ____and hold the door open while I bring in the shopping? - All those in____of the proposal please raise your hand. - After being out of fashion for years the painter is now in ____ with the critics. Q 36 → 40. Choose the word in each group that has different stress pattern 36. A graduate B impression C applicant D category 37. A beautify B geology C photograph D enemy 38. A classify B humidify C personify D solidify 39. A academic B fantastic C terrific D historic 40. A sacrifice B compliment C nursery D determine Q 41→45 Choose one word whose underlined part is pronounced differently from the rest. B. down C. now D. shown 41. A. brown B. expand C. eventually D. landlord 42. A. stagnant 43. A. compete B. fetch C. separate D. yell 44. A. minute B. muddy C. funny D. number 45. A. died B. continued C. contented D. followed Q46 →57 Fill in each blank with one suitable word Too Good to be True Some say you can do it in seven days, others promise success in 24 hours, or you may prefer to take your time and do it in 5 weeks. What are we talking about? Believe it (46)__ not, these periods (47) __time refer to language learning courses that promise excellent results in less time (48)__it takes to say 'Bonjour'! However, the advertisements of these companies are not necessarily (49) __best guide and if you don't know what to look for in a good course you could be left with little (50) __than a large bill. A complaint was recently made to a consumer rights group (51) __ WhizzLearn Systems, a language school chain, and the company has been forced to remove its claim that its technique is ten times better than any (52) __method. The Managing Director of WhizLearn Systems said “(53)__ we still believe our claim is true, we are willing to change our advert. But the fact remains, if you spend 3 hours (54) __day for 5 weeks on our language course you will soon be speaking the language. What we (55) __have done was to make (56) __ clearer in the advert. Then we wouldn't (57) __had these annoying complaints" Q58→64. Choose the most suitable sentence from the list A-H for each part (58-64) of the article. There is one extra sentence which you do not need to use. Career success in the arts John Prince, famous dancer and choreographer, gives advice on how to succeed in a career in the arts. I asked John how he got started and what requirements there are. "Well, to be a professional dancer it's useful to have had acting lessons or some background in drama. If you want to succeed in musical theatre you have to have a good singing voice as well. When you approach an agent you should take a portfolio with your CV, your statistics sheet and some good photos and reviews of past performances. You'll need dance clothes, ballet shoes, tap shoes, and even roller skates depending on what kind of show you are going to go for." 58 ____ "Of course, you need to be extremely fit if you want to be a professional dancer. I dance or move about for about six hours a day. There are great health benefits to being a dancer. I can eat a lot of pasta without gaining weight because dancing increases your metabolism so much." 59____ John has a very busy schedule in the next few months. He took time out to speak to me today from the making of a pop video to promote N-ergy's latest record. "I choreographed the dance routine for the boys and they only had 2 days in which to learn it! I am going to be working on a video for another 2 well known band - but that's top secret. Next month I'll be touring Spain in a production of a musical that was written by a friend of mine, Michaela Evans. 60____As for the future, I've come to realise that I would never be content to be just a chorus dancer - I'm too much of an individual for that. Like all artists I'd love to become a household name by writing and choreographing my own musicals." John was born in Jamaica to a Jamaican father and a Scottish mother but the family emigrated to England 20 years ago. "I have a little sister I adore, who is also training to be a dancer." How does it feel to have someone else following in your footsteps? 61____ Has he much more to learn, I wondered. "I've spent an incredible amount of my life training to get where I am. I went to college for two years in England, I trained for six months in Paris and about eight months in America. But you never really stop training or learning your art." 62____ So, would you say it's been plain sailing? "I feel I've been lucky to a degree; many people hit problems breaking into the arts. It can be a vicious circle really. You can't become a member of Equity, which is the actors' and dancers' union, without good contracts and you can't get good contracts without being a member of Equity. My advice to people who want to get into the arts would be to go out into the world, and try everything else first. 63 ____ What has a dance career done for you as a person? "Thanks to dancing, I've visited and performed in 23 countries so far. This has opened my eyes to the world, and I've been able to understand issues like racism and inequality from a wider perspective. 64 ____ "So all in all I'm really happy to be a dancer!" A It's fine, but I try not to give out too much advice as it gets irritating! B And if nothing you like comes out of it, then come back and be an actor or dancer. C Without a strict daily timetable like this you find yourself wasting too much time. D After that it's back to England to start a new term of dance classes. E Hopefully this has enabled me to become a better and more tolerant person as a result. F When it comes to coping with stress, I find that exercise helps me to cope with my problems, so I stay in good shape mentally as well. G Like any profession where you're always travelling, you tend to acquire something new almost every day. H Being fully equipped with all this stuff beforehand makes it easier when you go for auditions. Q65 →79 Choose the appropriate words to complete the following passage WHALES How far would you travel for a good meal? If you were a humpback whale, the answer would be five thousand miles. These large sea animals travel at least that far from their winter home off the (65)__ of Columbia to their summer (66) __ areas off Antarctica. The distance covered by some types of whale is amazing, especially when you (67) __ their enormous size. The blue whale is the largest animal that has ever (68) __ and it can weigh as much as thirty elephants. It (69) __ as no surprise, therefore, to hear that ancient folk legends tell of sailors (70) __ these creatures for islands. The more we (71) __ about whales, the more wonderful they seem. Some species can (72) __ their breath for more than an hour and dive to a (73) __ of over 200 metres. They use a system of sounds (74) __ as echo-location to find the fish they eat and they have further sounds to keep in (75) __ with each other. The noises they (76) __ can travel hundred of miles under water. Some species seem to sing complicated songs which (77) __ of a number of separate themes, sung in a specific order which can (78) __ up to half an hour or more. If you ever get the (79) __ to see one of these great creatures in the wild, you will understand why they have inspired so many legends. 65 A beach B coast C seaside D ground 66 A feeding B dining C eating D chewing 67 A view B think C consider D believe 68 A been B stayed C born D lived 69 A gets B comes C goes D seems 70 A supposing B mixing C confusing D mistaking 71 A find out B look out C show up D turn up 72 A store B keep C hold D save 73 A depth B length C width D breadth 74 A called B known C referred D named 75 A reach B touch C call D range 76 A provide B propose C process D produce 77 A consist B compose C include D involve 78 A long B last C play D give 79 A break B choice C chance D luck Q80 →87. You are going to read three different opinions from leading scientists about the future of fuel. For the questions below, choose from the writers A-C. The writers may be chosen more than once. A. Howard Bloom, Author: Even though most people are convinced that peak oil has already passed, to me, peak oil is just a hypothesis. There is a theory that carbon molecules can be found in interstellar gas clouds, comets and in space ice, and if this is the case, our planet could ooze oil for ever. And even if we stay earthbound, those who 3 say we have raped the planet of all its resources are wrong. There's a huge stock of raw materials we haven't yet learned to use. There are bacteria two miles beneath our feet which can turn solid granite into food. If bacteria can do it, surely we creatures with brains can do it better. As far as the near future of energy is concerned, I believe the most promising alternative fuels are biofuels, such as ethanol. It's an alcohol made from waste products such as the bark of trees, woodchips, and other 'waste materials'. And that's not the only waste that can create energy. My friend in the biomass industry is perfecting an energy-generation plant which can run on human waste. We produce that in vast quantities, and it's already gathered in centralised locations. B. Michael Lardelli, Lecturer in Genetics at The University of Adelaide: Nothing exists on this planet without energy. It enables flowers and people to grow and we need it to mine minerals, extract oil or cut wood and then to process these into finished goods. So the most fundamental definition of money is as a mechanism to allow the exchange and allocation of different forms of energy. Recently, people have been using more energy than ever before. Until 2005 it was possible to expand our energy use to meet this demand. However, since 2005 oil supply has been in decline, and at the same time, and as a direct result of this, the world's economy has been unable to expand, leading to global recession. With the world's energy and the profitability of energy production in decline at the same time, the net energy available to support activities other than energy procurement will decrease. We could increase energy production by diverting a large proportion of our remaining oil energy into building nuclear power stations and investing in renewable forms of energy. However, this is very unlikely to happen in democratic nations, because it would require huge, voluntary reductions in living standards. Consequently, the world economy will continue to contract as oil production declines. With energy in decline, it will be impossible for everyone in the world to become wealthier. One person's increased wealth can only come at the expense of another person's worsened poverty. C. Jeroen van der Veer, chief executive of Royal Dutch Shell: People are understandably worried about a future of growing energy shortages, rising prices and international conflict for supplies. These fears are not without foundation. With continued economic growth, the world's energy needs could increase by 50% in the next 25 years. However, I do not believe that the world is running out of energy. Fossil fuels will be able to meet growing demand for a long time in the future. Taking unconventional resources into account, we are not even close to peak oil. The priority for oil companies is to improve efficiency, by increasing the amount of oil recovered from reservoirs. At present, just over a third is recovered. We can also improve the technology to control reservoir processes and improve oil flow. However, these projects are costly, complex and technically demanding, and they depend on experienced people, so it is essential to encourage young people to take up a technical career in the energy industry. Meanwhile, alternative forms of energy need to be made economically viable. International energy companies have the capability, the experience and the commercial drive to work towards solving the energy problem so they will play a key role. But it is not as simple as merely making scientific advances and developing new tools; the challenge is to deliver the technology to people worldwide. Companies will need to share knowledge and use their ideas effectively. Which writer 80. believes oil will be available for many more years _____ 81. believes that from now on, less oil is available _____ 82. believes there are ways to obtain energy that we have not yet discovered _____ 83. sees a great potential in natural fuels _____ 84. believes the fuel crisis will cause the poor to become poorer _____ 85. sees energy and the economy as intrinsically linked _____ 86. believes that better technology can help to maintain oil production levels _____ 87. thinks that oil companies are responsible for developing other types of energy _____ Q 88→ 94. Complete the following sentences so that the second sentence has a similar meaning to the first sentence. Do not change the word given. You must use between two and five words, including the words given. 88. On business trips, I prefer driving home to staying in a hotel overnight rather On business trips, I’d __________________________ in a hotel overnight 89. Jack found it difficult to control his skis on the steep slope. under Jack found it difficult to_________________________ on the steep slope. 90. My views on this subject are exactly the same as yours. difference There ________________________ my views on this subject and yours. 91. They will be selling tickets for the concert during the lunch hour. sale Tickets for the concert will __________________ during the lunch hour. 92. I regret not contacting Brian when I was in Dublin. touch I wish that I _____________________ Brian when I was in Dublin. 93. In my opinion, these two kinds of music are completely different from each other comparison In my opinion, there is _________________________these two kinds of music. 94. I arrived late because I missed the 10.30 train turned 4 If I’d caught the 10.30 train, I _____________________time Q 95→ 100. Complete the following sentences so that the second sentence has a similar meaning to the first sentence, using the word given. Do not change the word given. 95. Nobody expected her to lose, but she did. against _____________________________________________________ 96. Peter always trusts me with his secrets in Peter_________________________________________________ 97. I thought it was marvellous that Jane could jump so high. at I _____________________________________________________ 98. Danny was asked to leave the school for bad behaviour from Danny was _____________________________________________ 99. They didn’t punish Karen, only gave her a warning got Karen _________________________________________________ 100. Gary is proud of the fact that he is never late. on Gary prides_____________________________________________ ------------------------THE END ------------------- 5 HƯỚNG DẪN CHẤM ĐỀ THI OLYMPIC KHỐI 10 - MÔN THI: Tiếng Anh Tổng số câu toàn bài 100 x 0,2= 20 điểm. Làm tròn đến 0,25 1. A dropped in 44 A minute 2. C go in for 45 C contented 3. A they had little chance of 46 or 4. D his ability to lecture 47 of 5. D What interests the workers most is 48 than 6. D under 49 the 7. A over 50 more 8. A solid 51 against/ about/ concerning 9. B making for 52 other/ similar 10. C do without 53 Although/ While/ Whilst 11. get round 54 a/ per/ every 12. bring it up 55 should/ could 13. dwells on 56 it/ this/ things 14. turns out 57 have 15. falling off 58 H 16. having you on 59 F 17. ran across 60 D 18. makes out 61 A 19. tracking down 62 G 20. take to 63 B 21 extraordinary 64 E 22 mercifully 65 B coast 23 upbringing 66 A feeding 24 enabled 67 C consider 25 achievement (s) 68 D lived 26 relationships 69 B comes 27 elusive 70 D mistaking 28 realization 71 A find out 29 incurable 72 C hold 30 striking 73 A depth 31 close 74 B known 32 game 75 B touch 33 type 76 D produce 34 light 77 A consist 35 favour 78 B last 36 B impression 79 C chance 37 B geology 80 C 38 A classify 81 B 39 A academic 82 A 40 D determine 83 A 41 D shown 84 B 42 C eventually 85 B 43 A compete 86 C 87 C 88 rather drive home than stay 89 keep/ get/ have his skis under control 90 is no difference between/ is not any difference between 91 be/ go on sale 92 had been/ got/ kept in touch with 93 no (possible) comparison (at all) between 94 would have turned up on/ in time 95. Against everybody's expectation, she lost. 96. Peter always confides in me 97. I was marvelled at Jane’s ability to jump so high 98. Danny was expelled from school for bad behaviour 6 99. Karen got off with a warning 100. Gary prides himself on always being early 7 Së gi¸o dôc - ®µo t¹o hµ tÜnh Kú thi CHỌN HỌC SINH GIỎI TỈNH líp 10 n¨m häc 2010-2011 ®Ò chÝnh thøc M«n thi: TiÕng Anh Thêi gian lµm bµi: 150 phót (kh«ng kÓ thêi gian giao ®Ò) (đề thi có 05 trang) Lưu ý: Thí sinh chỉ làm bài vào tờ giấy thi. Không được sử dụng bất kì loại tài liệu nào. I. Choose the word whose underlined part is pronounced differently from that of the others. 1. A. nothing B. purpose C. welcome D. compare 2. A. measles B. tease C. please D. measure 3. A. especially B. weather C. exercise D. bench 4. A. thus B. with C. smooth D. maths 5. A. naked B. practiced C. beloved D. wicked 6. A. encourage B. problem C. excellent D. possible 7. A. curiosity B. cure C. business D. during 8. A. many B. habit C. apple D. happy 9. A. bear B. hear C. ear D. clear 10. A. young B. cousin C. couple D. thorough II. Choose the most suitable word or phrase A, B, C or D to complete each sentence. 1. In the hotel lobby the detective caught __________of the man he had been hired to follow. A. glance B. view C. vision D. sight 2. ‘Go on, finish the food. It needs __________ up because it won’t stay fresh until tomorrow. A. eat B. eating C. to eat D. eaten 3. Before he left on his trip to America, the young man promised his parents he _____them regularly. A. is writing B. will write C. should write D. would write 4. Of course I didn’t break it ____________ purpose- it was an accident! A. by B. with C. from D. on 5. Andrew couldn’t __________ himself laughing at the expression on Maggie’s face. A. help B. stop C. escape D. avoid 6. __________ young, chimpanzees are easily trained. A. When are B. When C. They are D. When they 7. “What’s your proposal?” “I propose that the meeting __________” A. is postponing B. be postponed C. to be postponed D. postpones 8. Most of the courses at the banquet were completely consumed, but there _____ food still remaining. A. were few B. was little C. were a few D. was a little 9. The four-storey house __________ on that hill is still new. A. be built B. building C. built D. being building 10. Nam Cao devoted most of his time __________ A. to having written B. to write C. to writing D. to have written 11. I don’t think he will join us, ___________? A. doesn’t he B. won’t he C. will he D. don’t I 12. The young generation must contribute to ____________ the country. A. building of B. the building C. the building of D. build 13. I cannot help feeling anxious __________ the exam results. A. of B. with C. about D. for 1 14. In some vocational schools, the training quality is worse than ___________ used to be. A. they B. it C. them D. that 15. When Elvis Presley died, her daughter came ___________ a fortune. A. into B. up with C. by D. across III. The following passage contains ten errors. Find and correct them. Things started to go wrongly as soon as we got to the hotel. We were all completely exhausting after our long journey and looking forward to a shower and a rest. However, we found that our room was not ready, which was very annoyed, although the manager was extremely apologising. While we were waiting, we asked about the excursions to places of an interest which we have read about in the brochure. Imagine how we felt when we were told they had all been cancelled! Apparently, the person responsible of organizing them had left suddenly and had not been replaced. Then Sally saw a notice pinning to the door of the restaurant, said it was closed for redecoration, and Peter discovered that the swimming pool was empty. When we eventually got to our room we were horrified to find that it was at the back of the hotel, and we had a view of a car park, that seemed to be used as a rubbish dump. We seriously began to wonder whether or not to stay. IV. Read the text below, using the word given in brackets to form a word that fits in the space. Congratulations to all involved with the school (__1__ produce) of The Woman Next Door. The (__2__ advertise) was carried out by the Art Department, and the posters were very (__3__ imagine). We certainly have some very (__4__ art) students in our school! Many people helped with building and painting the (__5__ scene) and the play was written by the English Department, who managed to create an (__6__ amuse) story, with excellent songs. The music was written by Sue Porter, who also (__7__ company) the singers on the piano. Everyone enjoyed a thoroughly (__8__ entertain) evening, and there was a long round of (__9__ applaud) at the end. Jim Barrett gave a brilliant performance as Sergeant Moss, and Liz Aiken was a (__10__delight) Mrs. Jump. Well-done everyone! V. For each question, write one word which can be used in all three sentences For example: I was so tired that I couldn’t even think ___________. I will come __________ to the point – you are fired. It’s time to set the record __________ about what really happened that night. → STRAIGHT I was just _______ to go home, but that’s okay. 1. It’s _______ time you got home – We’ve been worried sick! We’re definitely going to have to do something _______ increasing our market share. 2. I should get a reply from them any ________ now. It will only take them a ________ to update the website. Now is the __________ of truth! 3. We’re definitely living in the ___________ of information. This particular wine doesn’t really mature with __________. It’s time you started acting your __________, young man! 4. I do hope we’re going to arrive _________ time. How anyone could work in that office for years _________ end I just don’t know. 2 Just bet me that Doug was going to get the sack so I said: “You’re ________” 5. To my ____________, the boss said that I could have an extra day off work. Martin always takes great ___________ in talking to anyone about fishing. When Erin finally started to move downhill on the skis for the first time, she laughed in _____. 6. The local council should start a building program to meet the ______ for leisure activity. The Major said he had no _________ of advice from people who didn’t know the situation. This area is badly in ___________ of financial investment. 7. Oh, tell Carrie that I send my ________ when you see her. When my mum and dad met, my mum says it was ________ at first sight. My _____ for travelling comes from the fact that we moved around a lot. 8. We don’t go out during the week as a ________, but we’ll make an exception tonight. People must be made to respect the _________ of law, or we’ll have anarchy. A good ______ of thumb is that you should allow 15 minutes per exercise in the exam. 9. Did you hear that the guy who lives in the flat upstairs has been _______ with burglary? I couldn’t believe they ________ me three euros for a bottle of water! The children ran out of the school gate and ________ down the hill. 10. The head teacher punished Aaron severely in order to make an _________ of him. Why can’t you follow your brother’s ________ and go to university. Give me one _________ of a place round here where young people can go in the evenings. VI. Read the text below and decide which answer A, B, C, or D, best fits each space. Everyone wants to reduce pollution. But the pollution problem is (1)______ complicated as it is serious. It is complicated (2)______ much pollution is caused by things that benefit people. (3)______, exhaust from automobiles causes a large percentage of air pollution. But the automobile (4)______ transportation for millions of people. Factories discharge much (5)______ the material that pollutes the air and water, but factories give employment to a large number of people. Thus, to end (6)_______ greatly reduce pollution immediately, people would have to ( 7)______ using many things that benefit them. Most of the people do not want to do that, of course. But pollution can be (8)______ reduced in several ways. Scientists and engineers can work to find ways to lessen the (9)______ of pollution that such things as automobiles and factories cause. Governments can pass and enforce laws that (10)______ businesses and traffic to stop, or to cut down on certain polluting activities. 1. A. as B. more C. less D. like 2. A. so B. while C. though D. because 3. A. Specific B. For example C. Such as D. Like 4. A. takes B. affords C. carries D. provides 5. A. about B. for C. of D. with 6. A. or B. and C. as well D. then 7. A. start B. continue C. stop D. go on 8. A. carefully B. unexpectedly C. gradually D. little 9. A. way B. figure C. number D. amount 10. A. forbid B. prevent C. request D. require 3 VII. Read the passage and choose the correct answer A, B, C, or D for each question. Bill Jarvis took over our village news agency at time of life when most of us only want to relax. He just thought he would like something but not too much to do, and the news agency was readymade. The business produced little enough for him, but then Bill was a chap who only wanted the simplicity and order and regularity of the job. He had been a long-serving sailor, and all his life had done everything by the clock. Every day he opened his shop at six a.m to catch the early trade; the papers arrived on his doorstep before that. Many of Bill's customers were city workers, and the shop was convenient for the station. Business was tailing off by ten o'clock, so at eleven sharp Bill closed for lunch. It was hard luck on anybody who wanted a paper or magazine in the afternoon, for most likely Bill would be down on the river bank, fishing, and his nearest competitor was five kilometres away. Some time in the afternoon, the evening papers landed on the door-mat, and at four o'clock Bill reopened. The evening rush lasted till seven, and it was worthwhile. He lived in a flat above the shop, alone. Except in very bad weather, you always knew where to find him in the afternoons, as I have said. Once, on a sunny afternoon, I walked home along the river bank from a shopping trip to the village. By my watch it was three minutes past four, so I was astonished to see Bill sitting there on his little chair with a line in the water. He had no luck, I could see, but he was making no effort to move. "What's wrong, Bill?" I called out from the path. For answer, he put a hand inside his jacket and took out a big, golden object. For a moment I had no idea what it could be, and then it suddenly went off with a noise like a fire engine. Stopping the bell, Bill held the thing up and called back: "Ten to four, you see, and this is dead right." He stood up then and began to wind in his line. I had never known anyone carry a brass alarm clock round with him or her before. 1. Bill Jarvis became a newsagent when ________ A. He needed the money. B. He was quite an old man. C. He decided to take things easy. D. He gave up clock repairing. 2. What does the passage tell us about the news agency? A. It was an easy job with fixed hours. B. It was a very profitable business C. It was opened specially for Bill Jarvis D. It belonged to the railway and was part of the station. 3. Why did Bill open the shop so early in the day? A. He liked to do as much as possible before he went to work. B. Bill was never sure of the time. C. The shop had to be open when the morning papers came D. It was then that he did a lot of business. 4. We understand from the passage that the shop closed for lunch ________ A. At eleven o'clock more or less. B. At exactly eleven o'clock. C. Before eleven o'clock. D. Always after eleven o'clock. 5. You might say "hard luck" to someone who ________ A. has just heard some very good news. B. puts great effort into whatever he or she tries. C. is less fortunate than he or she ought to be. D. fails through his or her own fault entirely. 6. Why was the writer on the riverbank that afternoon? A. He was going to do some shopping in the village. B. He was fishing. 4 C. He was going to get the evening paper. D. He was on his way home from the village 7. Why was the writer surprised when he saw Bill Jarvis? A. He thought it was late for Bill to be still fishing. B. Bill had not caught anything, and that seemed strange. C. He thought Bill was ill, because he was not moving at all D. He was surprised because Bill stayed in his flat in the afternoons. 8. From the information given in the passage, who- or what - do you think was wrong? A. The bell was; it must have gone off at the wrong time. B. The writer's watch was fast. C. Bill was; he had dropped off to sleep D. Bill's clock was wrong; it was very old. 9. All of the following are true about Bill Jarvis EXCEPT __________ A. he lived alone B. He had ever worked as a sailor C. He was a newspaper man D. Fishing was his past time 10. What did Bill Jarvis often bring with him when he went fishing? A. a clock B. a gold C. a newspaper D. a fire engine VIII. Complete the second sentence so that it has a similar meaning to the first sentence. Use the word given and other words to complete each sentence. 1. Mr. Holton only operated because he knew the rumour was malignant → Mr. Holton wouldn't.............................................................................................................. 2. All that stood between John and a gold medal was Jim's greater speed → But for ................................................................................................................................. 3. Immediately after their arrival, things went wrong. → No sooner…………………………………………………………………………………. 4. He was so tired that he fell asleep before the end of the film → He was too............................................................................................................................. 5. Hearing that an earthquake had occurred was a great shock to us. → We were .............................................................................................................................. 6. The decorators have finished our first floor. → We have ............................................................................................................................. 7. Her hobby is one thing that she does not intend to give up. → She has ............................................................................................................................... 8. That is the best meal I have ever eaten. → I have never ............................. ............................... ............................... ........................... 9. Home computers used to be much more expensive. → Home computers aren’t................... ............................... ............................... ................... 10. It was a mistake of mine to park outside the police station. → I shouldn’t................... ...................................................................................................... IX. Writing: Write an essay of about 120 – 150 words to answer the following question: “What makes a good friend? 5 Së gi¸o dôc - ®µo t¹o hµ tÜnh H−íng dÉn chÊm m«n tiÕng anh 10 ®¸p ¸n vµ biÓu ®iÓm Tổng: 20 điểm, cụ thể như sau: CÂU I: (2 điểm) - Mỗi câu đúng được 0,2 điểm: 1. A 2. D 3. A 4. D 5. B 6. A 7. C 8. A 9. A 10. D CÂU II: (3 điểm) - Mỗi câu đúng được 0,2điểm: 1. D 2. B 3. D 4. D 5. B 6. B 7. B 8. D 9. C 10 C 11. C 12. C 13. C 14. B 15. A III: (2 điểm) - Mỗi câu đúng được 0,2 điểm: 1. PRODUCTION 6. AMUSING 2. ADVERTISING 7. ACCOMPANIED 3. IMAGINATIVE 8. ENTERTAINING 4. ARTISTIC 9. APPLAUSE 5. SCENERY 10. delightful CÂU IV: (2 điểm) - Mỗi câu đúng được 0,2 điểm: 1. A 2. D 3. B 4. D 5. C 6. A 7. C 8. C 9. D 10. D CÂU V: (1 điểm) - Mỗi câu đúng được 0,1 điểm: 1. about 2. moment 3. age 4. on 5. delight 6. need 7. love 8. rule 9. charged 10. example CÂU VI: (2 điểm) - Mỗi câu đúng được 0,2 điểm: 1. C 2. A 3. D 4. C 5. B 6. D 7. A 8. C 9. B 10. A 6 CÂU VII: (3 điểm) - Mỗi câu đúng được 0,3 điểm: 1. → Mr. Holton wouldn't have operated if he hadn’t known (that) the rumour was malignant 2. → But for Jim's greater speed, John would have won a gold medal 3. → No sooner had they arrived than things went wrong 4. → He was too tired to see/watch the end of the film 5. → We were (greatly/very) shocked to hear/when we heard that an earthquake had occurred 6. → We have had our first floor decorated (by the decorators) 7. → She has no intention of giving up her hobby. 8. → I have never eaten such a good meal as that/a better meal than that 9. → Home computers aren’t as expensive as they used to 10. → I shouldn’t have parked outside the police station CÂU VIII: (2 điểm) - Mỗi câu đúng được 0,2 điểm: 1. wrongly → wrong 6. have read → had read 2. exhausting → exhausted 7. of → for 3. annoyed → annoying 8. pinning → pinned 4. apologising → apologetic 9. said → saying 5. places of an interest → places of interest 10. that → which CÂU IX: (3 điểm) Yêu cầu: + Viết đúng bố cục của một bài luận. + Trình bày rõ ràng, sạch sẽ. + Không phạm nhiều lỗi về chính tả và ngữ pháp 7 Điểm bằng số Điểm bằng chữ Chữ ký G.khảo Chữ ký G.khảo Số phách Số T.tự bài thi [ SỞ GIÁO DỤC VÀ ĐÀO TẠO NINH THUẬN KỲ THI CHỌN HỌC SINH GIỎI CẤP TỈNH NĂM HỌC 2014 – 2015 Khóa ngày: 09 / 11 / 2014 Môn thi: TIẾNG ANH - Cấp THPT Thời gian làm bài: 180 phút (Không kể thời gian phát đề) (Đề chính thức) (Đề thi gồm 10 trang/ 20 điểm) SECTION I: LISTENING HƯỚNG DẪN PHẦN THI NGHE HIỂU   Bài nghe gồm 3 phần, mỗi phần được nghe 2 lần, mỗi lần cách nhau 15 giây, mở đầu và kết thúc mỗi phần nghe có tín hiệu. Mọi hướng dẫn cho thí sinh (bằng tiếng Anh) đã có trong bài nghe. Part 1: You will hear people talking in eight different situations. Choose the best answer (A, B or C) and write your answers in the corresponding numbered boxes. 1. You hear the beginning of a lecture in a university. What is the lecture going to be about? A. the history of a place B. social problems C. a person's life and work 2. You hear part of a radio programme about an island. Why are there so few trees on the island now? A. because of urban development B. because of the expansion of agriculture C. because of the action of the sea 3. You hear a teenage boy talking on the radio about his family's efforts to earn money. What will the money be used for? A. computer games B. new clothes C. an expensive car 4. You hear an announcement at a railway station. What should you do if you want to go to London? A. await further instructions B. travel from a different platform C. get on the next train to arrive 5. You hear a man talking about newspapers. What does he say about the newspaper he reads? A. It is an essential part of life. B. It is an important source of information. C. It is useful for passing the time. 6. You hear a conversation on the radio. What is the programme about? A. solving traffic problems in cities B. studying nature and wildlife C. finding part of an ancient town 7. You hear two people talking about a music festival they have been to. What do they agree about? A. the quality of the performances B. the fairness of the prices C. the standard of the accommodation 8. You hear a travel writer speaking on a radio programme. What aspect of his travels is he talking about? A. what he does to keep healthy B. how he reduces the risk of accidents C. illnesses from which he has suffered Your answers: 1. 2. 3. 4. 5. 6. 7. 8. 1 Điểm bằng số Điểm bằng chữ Chữ ký G.khảo Chữ ký G.khảo Số phách Số T.tự bài thi [ Part 2: For question 1-6, complete the form below. Write NO MORE THAN TWO WORDS AND / OR A NUMBER for each answer. Moving Company Service Report Phone number: (1) ...................................................................................................................................................... USA Adress: 509 (2) ............................................................................................................................. .............. 1137 (3) .................................................................................................................. in Seatle Packing day: (4)....................................................................................................................................................... Date: 11th March Clean-up by: 5:00p.m. Day: (5) ...................................................................................................................................................... About the Price: Rather expensive Storage time: (6) ................................................................................................................................................. ..... For question 7-10: Where does the speaker decide to put items in? Write the correct letter, A, B or C, next to questions 7-10. A. in emergency pack B. in personal package C. in storage with the future Items: 7. cutlery and dishes ..................................... 8. kettle .................................................................. 9. alarm clock ................................................... 10. CD player..................................................... Part 3: You will hear an interview with a man called Stan Leach who is talking about adventure sports. Choose the best answer (A, B or C) and write your answers in the corresponding numbered boxes. 1. Stan says that the best thing about walking is that you can A. get fit by doing it. B. please yourself how you do it. C. do it on your own. 2. Stan's opinion on scrambling is that A. people doing it may need to be accompanied. B. it is unsuitable for beginners. C. it is more exciting than walking. 3. What did Stan discover when he went climbing? A. It was not enjoyable. B. It was harder than he expected. C. It can be very frightening. 4. What does Stan say about mountain biking? A. Britain is not the best place for it. B. It is more expensive in Britain than elsewhere. C. It is best where there are lots of downhill slopes. 2 Điểm bằng số Điểm bằng chữ Chữ ký G.khảo Chữ ký G.khảo Số phách Số T.tự bài thi [ 5. Stan's advice on scuba diving is that A. most of the courses for it are good. C. you should think carefully before trying it. 6. What is Stan's view of skydiving? A. It is surprisingly popular. C. Only certain types of people like it. 7. What does Stan say about canoeing? A. You can do it in conditions that suit you. C. There are few places in Britain to do it. B. it is easier than it seems. B. It is best when done in teams. B. It is best at certain times of the year. Your answers: 1. 2. 3. 4. 5. 6. 7. SECTION II: USE OF ENGLISH Part 1: Choose the correct answer (A, B, C or D) to each of the following questions and write your answers in the corresponding numbered boxes. 1. Paul wasn’t keen on ______ of the jobs she was offered. A. neither B. both C. none D. either 2. Brian has been working ______ since he was promoted. A. as harder B. Just as hardly C. much harder 3. I wish you wouldn’t show off and ______ your success so much! A. boast about B. full of C. bored by D. more hardly D. congratulate on 4. Tom’s employment ______ would be better if he had a clean driving licence. A. opportunities B. prospects C. odds D. likelihood 5. We stopped watching the game before the end, but I thought we ______. A. had won B. have won C. will have won 6. Remember to wear a helmet ______ your head is protected. D. have been winning A. to B. that C. in order that 7. I was passing their house, so I ______ Claire and Michael. D. so as A. came up with B. got on with C. ran into D. dropped in on 8. He didn’t want to ______ of seeing New York, so he agreed to go on the trip. A. pass up the chance B. pass with chance C. catch the chance D. miss up the chance 9. Everyone was ______ to bits that Joe was so successful in the competition. A. blissful B. thrilled C. exultant D. insatiable 10. The new manager made a useful ______ the discussion of the meeting last week. A. comment to B. statement for C. contribution to D. contribution for Your answers: 1. 2. 3. 4. 5. 6. 7. 8. 9. 10. 3 Điểm bằng số Điểm bằng chữ Chữ ký G.khảo Chữ ký G.khảo Số phách Số T.tự bài thi [ Part 2: Write the correct FORM of each bracketed word in the numbered space provided in each column on the right. (0) has been done as an example. Marie Curie (1867-1934) Marie Sklodowska was born on 7th November 1867. From early (0. CHILD) _____, she was fascinated by science and showed great (1. ENTHUSE) ______ for it, as well as (2. ORDINARY) ______ talent. However, it was her marriage to Pierre Curie in 1895 that marked the start of a partnership that was to achieve results of world (3. SIGNIFY) ______ in particular the discovery of the radioactive metals polonium and radium in 1898. By this time Marie Curie, though quiet and unassuming, was held in great esteem by scientists throughout the world. In 1903 she became the first woman to receive the Nobel Prize for Physics. Although the premature death of her husband in 1906 was a bitter blow to her, it so marked a (4. DRAMA) ______ turning point in her career. From this time on, she was to put all her energy into completing alone the work they had originally (5. TAKE) ______together. 0. childhood 1. __________ 2. __________ 3. __________ 4. __________ 5. __________ Part 3: Read the text below and think of the word which best fits each gap. Use only ONE word in each gap. Write your answers in the corresponding boxes provided below the passage. Dealing with waste plastic Every year people throw away millions of tonnes of plastic bottles, boxes and wrapping. These create huge mountains of waste that are extremely hard to get (1)______of. Now a new recycling process promises to reduce this problem by turning old plastic into new. Scientists have taken (2)______long time to develop their ideas because waste plastic has always been a bigger problem (3)______substances like waste paper. You can bury plastic, but it takes many years to break down. If you burn it, it just becomes another form of pollution. A (4)______products, for example bottles, can be re-used, but it is expensive or difficult to do this (5)______a lot of plastic products. Now a group of companies has developed a new method (6)______recycling that could save almost any plastic waste. Nearly every type of waste plastic can be used: it does (7)______have to be sorted. In addition, labels and ink may be left (8)______the products. Everything is simply mixed together (9)______heated to more than 400 degrees centigrade (10)______that it melts. It is then cooled, producing a waxy substance that can be used to make new plastic products, including computer hardware. Your answers: 1. 2. 3. 4. 5. 6. 7. 8. 9. 10. SECTION III: READING Part 1: Read the passage and choose the answer (A, B, C or D) which you think fits best according to the text. Write your answers in the numbered boxes. Language Acquisition Regardless of culture or language, children around the world tend to go through the same stages when first developing their ability to speak. Initially, children practice communication through "conversations" with those close to them, usually a parent, sibling, or caregiver. In the course of daily 4 Điểm bằng số Điểm bằng chữ Chữ ký G.khảo Chữ ký G.khảo Số phách Số T.tự bài thi [ activities and routines, children first express themselves through two modes of communication: gestures and sounds. As a child continues to develop, the motions and sounds he or she uses become more complex until finally the child is able to express intended messages through speech and gestures. The first sounds and gestures a child makes are used in reference to his or her immediate surroundings. This stage of communication can typically be observed between six and eighteen months of age. Starting with basic gestures, most often pointing, children indicate their intentions or show that they recognize familiar objects. These gestures are later accompanied by sounds, approximations of simple words that the child hears often in routine household interactions. Refined through copious repetition, some of these sounds eventually become recognizable as words. For example, when the family dog enters the room, a child might point and say "doddie," trying to replicate the sounds of "dog" or "doggie," that other family members so often say. Interestingly, at this point, the child does not yet realize that "dog" refers to a type of animal rather than just the family pet. As the child grows older and uses the word more frequently, the more general application of the word becomes clear, and the child will begin to demonstrate appropriate usage of the word. From eighteen to twenty-four months of age, children continue to refine and add to their store of sounds and words. The words used at this stage are mainly nouns, such as the names of people around them, or verbs that refer to their ability to interact in the environment, such as "give," "take," and "go." Their speech is very concrete and focused on phenomena that make a strong sensory impression, as one might expect from children still in what Piaget called the sensorimotor stage of cognitive development. During this stage, children continue to add content and meaning to their speech, particularly by moving from one-word utterances like "doggie," to multi-word strings such as, "doggie go." As the complexity of a child's speech increases, the number of gestures that accompany the speech also increases. In fact, researchers have noted that the lengths of a child's verbal and gestural expressions are similar. By two-and-a-half years of age, children can speak in sentences made up of several words. Furthermore, as children's verbal skills mature, so too do their gestures. These gestures are often used to support their verbal communication, to indicate recognition of an object, or to illustrate an object's function. For example, children at this stage of communicative development may mime actions, such as panting or barking, to show their recognition of "dog." Children's interactions with their conversation partners also change during this period. At this stage, children more often look at the person being spoken to rather than at the object of their gestures.  A) From the age of thirty months, there is an explosion in a child's ability to form sentences. In mere months, the child goes from limited short sentences of only a few words to adult-like complexity in the structure of their speech, such as responding to a question like, "Did you see the dog?" with the sentence, "Yes, I did see doggie."  B) In fact, the child's language develops so rapidly at this stage that it is difficult for researchers to describe the exact pattern of acquisition.  C) It is during this final stage that children acquire the majority of the grammar they need to communicate effectively in their first language. Interestingly, children seem able to systematically use grammar in their speech, as though they somehow realize the inherent rules behind different linguistic elements.  D) To recap, children all over the world go through the same basic stages of language acquisition, going from mere sounds at about six months to complete sentences by the age of three. This process is all the more interesting because it happens naturally, irrespective of any formal education. 5 Điểm bằng số Điểm bằng chữ Chữ ký G.khảo Chữ ký G.khảo Số phách Số T.tự bài thi [ 1. The word those in the passage refers to _______. A. conversations B. people C. children D. words 2. Which of the following best expresses the essential information in the highlighted sentence in the passage? Incorrect choices change the meaning in important ways or leave out essential information. A. The child thinks that "dog" is the name of her family pet. B. The child thinks that "dog" is the word for all pets. C. The child thinks that all family pets are dogs. D. The child thinks that all dogs are her family pet. 3. According to paragraph 2, the first words a child speaks are determined by all of the following EXCEPT _______. A. how simple the word is B. how frequently the word is heard C. how old the child is D. how often a child practices sounds 4. According to paragraph 3, what stage of cognitive development are children still at between eighteen and twenty-four months of age? A. The refinement stage B. The sensorimotor stage C. The gestural stage D. The environmental stage 5. The word ‘utterances’ in the passage is closest in meaning to A. extremes B. choices C. demands D. expressions 6. The author mentions "doggie" and "doggie go" in the passage in order to _______. A. define verbal and gestural expressions B. illustrate phenomena that make a strong sensory impression C. provide examples of single-word and multi-word utterances D. explain nouns and verbs 7. Which of the following can be inferred from paragraph 4 about a child's earlier interactions with her conversation partners? A. The child often does not look at her partners. B. Her partners often object to her use of gestures. C. The child cannot tell the difference between people and objects. D. The child uses gestures more effectively than she uses words. 8. According the passage, which of the following is true about the gestures a child makes? A. They have their own form of grammar. B. They are less important for communication than spoken words. C. They become increasingly complex, just as speech does. D. They develop more slowly than spoken communication. 9. Look at the four squares [] that indicate where the following sentence could be added to the passage: ‘The final stage of language development is a rapid one’. Where would the sentence best fit? Choose the square [] where the sentence should be added to the passage. A. 1st square B. 2nd square C. 3rd square D. 4th square 10. The phrase ‘irrespective of any formal education’ in the passage is closest in meaning to _____. A. showing the value of formal education B. demonstrating that formal education is not necessary C. in conjunction with formal education D. no matter how much formal education one has Your answers: 1. 2. 3. 4. 5. 6. 7. 8. 9. 10. 6 Điểm bằng số Điểm bằng chữ Chữ ký G.khảo Chữ ký G.khảo Số phách Số T.tự bài thi [ Part 2: Read the text and choose the correct heading for sections 1—7 from the list of headings below. There are more extra headings which you do not need to use. Write your answers in the numbered boxes. A. Common objections B. Who's planning what C. This type sells best in the shops D. The figures say it all E. Early trials F. They can't get in without these G. How does it work? H. Fighting fraud I. Systems to avoid J. Accepting the inevitable Paragraph 0: ____F______ Students who want to enter the University of Montreal's Athletic Complex need more than just a conventional ID card — their identities must be authenticated by an electronic hand scanner. In some California housing estates, a key alone is insufficient to get someone in the door; his or her voiceprint must also be verified. And soon, customers at some Japanese banks will have to present their faces for scanning before they can enter the building and withdraw their money. Paragraph 1: __________ All of these are applications of biometrics, a little-known but fast-growing technology that involves the use of physical or biological characteristics to identify individuals. In use for more than a decade at some high-security government institutions in the United States and Canada, biometrics are now rapidly popping up in the everyday world. Already, more than 10,000 facilities, from prisons to daycare centres, monitor people's fingerprints or other physical parts to ensure that they are who they claim to be. Some 60 biometric companies around the world pulled in at least $22 million last year and that grand total is expected to mushroom to at least $50 million by 1999. Paragraph 2: __________ Biometric security systems operate by storing a digitised record of some unique human feature. When an authorised user wishes to enter or use the facility, the system scans the person's corresponding characteristics and attempts to match them against those on record. Systems using fingerprints, hands, voices, irises, retinas and faces are already on the market. Others using typing patterns and even body odours are in various stages of development. Paragraph 3: __________ Fingerprint scanners are currently the most widely deployed type of biometric application, thanks to their growing use over the last 20 years by law-enforcement agencies. Sixteen American states now use biometric fingerprint verification systems to check that people claiming welfare payments are genuine. In June, politicians in Toronto voted to do the same, with a pilot project beginning next year. Paragraph 4: __________ To date, the most widely used commercial biometric system is the handkey, a type of hand scanner which reads the unique shape, size and irregularities of people's hands. Originally developed for nuclear power plants, the handkey received its big break when it was used to control access to the Olympic Village in Atlanta by more than 65,000 athletes, trainers and support staff. Now there are scores of other applications. Paragraph 5: __________ Around the world, the market is growing rapidly. Malaysia, for example, is preparing to equip all of its airports with biometric face scanners to match passengers with luggage. And Japan's largest maker of cash dispensers is developing new machines that incorporate iris scanners. The first commercial biometric, a hand reader used by an American firm to monitor employee attendance, was introduced in 1974. But only in the past few years has the technology improved enough for the prices to drop sufficiently to make them commercially viable. `When we started four years ago, I had to explain to everyone what a biometric is,' says one marketing expert. 'Now, there's much more awareness out there.' 7 Điểm bằng số Điểm bằng chữ Chữ ký G.khảo Chữ ký G.khảo Số phách Số T.tự bài thi [ Paragraph 6: __________ Not surprisingly, biometrics raise thorny questions about privacy and the potential for abuse. Some worry that governments and industry will be tempted to use the technology to monitor individual behaviour. `If someone used your fingerprints to match your health-insurance records with a creditcard record showing you regularly bought lots of cigarettes and fatty foods,' says one policy analyst, 'you would see your insurance payments go through the roof.' In Toronto, critics of the welfare fingerprint plan complained that it would stigmatise recipients by forcing them to submit to a procedure widely identified with criminals. Paragraph 7: __________ Nonetheless, support for biometrics is growing in Toronto as it is in many other communities. In an increasingly crowded and complicated world, biometrics may well be a technology whose time has come. Your answers: 0. F 1. 2. 3. 4. 5. 6. 7. Part 3: Choose ONE suitable word from the box below to complete in each blank. There are two words that you do not need to use. Write your answers in the numbered boxes. turned, expected, keen, ancient, advisable, inevitable, sight, changed, agree, other Markets In practically any country in the world you are sure to find a market somewhere. Markets have been with us since (1) ______ times, and arose wherever people needed to exchange the goods they produced. For example, a farmer might have exchanged a cow for tools. But just as times have (2) ______, so have market practices. So, whereas in early times the main activity associated with markets would have been ‘bartering’ - in (3) ______ words exchanging goods - today most stall-holders wouldn't be too (4) ______ on accepting potatoes as payment, for instance, instead of cash. In contrast, what might be a common (5) ______ in a modern market in some countries is a certain amount of ‘haggling’, where customer and seller eventually (6) ______ on a price, after what can sometimes be quite a heated debate. However, behaviour which is (7)______in a market in one country may not be acceptable in another. Even within one country, there may be some markets where you could haggle quite easily and others where it would be (8) ______ not to try! Your answers: 1. 2. 3. 4. 5. 6. 7. 8. SECTION IV: WRITING Part 1: For questions 1-5, complete the second sentence so that it has a similar meaning to the first sentence, using the word given. DO NOT change the word given. You must use between two and six words, including the word given. 1. Lucy succeeded in passing her driving test, even though she had flu. (MANAGE) Despite …………………………………………………………………………………………………………………..….her driving test. 2. Nick’s cooking soon impressed his friends. (REPUTATION)  Among his friends, Nick soon gained ……………….………………………………………………………….. good cook. 8 Điểm bằng số Điểm bằng chữ Chữ ký G.khảo Chữ ký G.khảo Số phách Số T.tự bài thi [ 3. Do you think Tom is likely to win the competition? (CHANCE) Do you think Tom is in …………………………………………………………………………………………..the competition? 4. The pool closes in ten minutes, so there isn’t enough time to go swimming now. (WORTH)  It …………………………………………………………………………………………….now as the pool closes in ten minutes. 5. We will send your new passport tomorrow, provided your paperwork is in order. (LONG)  Your new passport will……………………………………………………………………….…..your paperwork is in order. Part 2: The graph and the table below give information about water use worldwide and water consumption in two different countries. Summarize the information by selecting and reporting the main features, and make comparison where relevant. Write at least 120 words. . Water consumption in Brazil and Congo in 2000 Country Population Irrigated land Water consumption per person Brazil 176 million 26.500 km2 359 m3 Democratic Republic of Congo 5.2 million 100 km2 8 m3 ......................................................................................................................................................... ......................................................................................................................................................... ......................................................................................................................................................... ......................................................................................................................................................... ......................................................................................................................................................... ......................................................................................................................................................... ......................................................................................................................................................... ......................................................................................................................................................... ......................................................................................................................................................... ......................................................................................................................................................... ......................................................................................................................................................... ......................................................................................................................................................... ......................................................................................................................................................... 9 Điểm bằng số Điểm bằng chữ Chữ ký G.khảo Chữ ký G.khảo Số phách Số T.tự bài thi [ Part 3: Write about the following topic (about 250 words): Some people believe that teachers should be responsible for teaching students the difference between right and wrong. Others say that teachers should only teach academic subjects. Discuss both viewpoints and give your own opinion. Give reasons for your answer and include any relevant examples from your own knowledge or experience. (Do not include your personal information). ......................................................................................................................................................... ......................................................................................................................................................... ......................................................................................................................................................... ......................................................................................................................................................... ......................................................................................................................................................... ......................................................................................................................................................... ......................................................................................................................................................... ......................................................................................................................................................... ......................................................................................................................................................... ......................................................................................................................................................... ......................................................................................................................................................... ......................................................................................................................................................... ......................................................................................................................................................... ......................................................................................................................................................... ......................................................................................................................................................... ......................................................................................................................................................... ......................................................................................................................................................... ......................................................................................................................................................... ......................................................................................................................................................... ......................................................................................................................................................... ......................................................................................................................................................... ......................................................................................................................................................... ......................................................................................................................................................... ......................................................................................................................................................... ......................................................................................................................................................... ......................................................................................................................................................... ......................................................................................................................................................... ......................................................................................................................................................... ......................................................................................................................................................... THE END 10 HỘI CÁC TRƯỜNG THPT CHUYÊN KHU VỰC DH & ĐB BẮC BỘ KÌ THI CHỌN HỌC SINH GIỎI KHU VỰC MỞ RỘNG NĂM HỌC 2011- 2012 MÔN THI: TIẾNG ANH LỚP 10 Ngày thi: 21 tháng 4 năm 2012 ĐỀ CHÍNH THỨC (Thời gian làm bài 180 phút không kể thời gian giao đề) Đề thi gồm 12 trang Part I. LISTENING (10 pts) Question 1. You will hear a telephone conversation between a passenger and an officer at the London Heathrow airport. (5 pts) Listen to the conversation and complete the booking form below. Write ONE WORD OR A NUMBER for each answer. LISTEN ONCE ONLY. AIRPORT SHUTTLE BOOKING FORM To: Milton Date: (1) ..................... No. of passengers: One Bus time: (2) .....................p.m Type of ticket: (3) ..................... Name: Jane Thomson Flight No: AC936 From: London Heathrow Address in Milton: Vacation Motel, 24, (4) ..................... Street Fare: $ 35 Credit Card No: (Visa) (5) ..................... Question 2. Choose the correct letter A, B or C. LISTEN TWICE. (5 pts) 1. These sessions with a counsellor are _____ A. compulsory for all students. B. for science students only. C. available to any students. 2. The counsellor says that new students have to _____ A. get used to working independently. B. spend more time on the college premises. C. work harder than they did at school. 3. John complains that the resource centre _____ A. has limited opening hours. B. has too few resources. C. gets too crowded. Page 1 of 12 4. The counsellor suggests to John that _____ A. most other students can cope. B. he needs to study all the time. C. he should be able to fit in some leisure activities. 5. Before being able to help John the counsellor needs to _____ A. talk with some of his lecturers. B. get more information from him. C. consult his tutor. Your answers: 1. 2. 3. 4. 5. Part II. GRAMMAR AND VOCABULARY (40 pts) Question 1. Choose the best answer A, B, C or D in each sentence to complete it. (10 pts) 1. His answer was so confused that I could hardly make any ______ of it at all. A. interpretation B. meaning C. intelligibility D. sense 2. The thought of living in another country has never ______ my mind. A. crossed B. entered C. occurred to D. come 3. I was passing their house, so I ______ Claire and Michael. A. dropped in B. came up with C. got on with D. run into 4. I don’t really ______ winter sport very much. A. deal with B. face up to C. go in for D. get round to 5. Her mother’s illness cast a cloud ______ her wedding day. A. over B. for C. in D. on 6. She did six hours’ ______ studying a day for her exam. A. solid B. heavy C. strong D. big 7. No one liked the new lecturer, so there was a gradual ______ in the attendance of his lectures. A. fall-through B. fall-out C. fall-off D. fall-in 8. I was talking to my aunt when suddenly my cousin Gorge ______ in our conversation. A. interrupted B. broke C. went D. intervened 9. He has been unable to find a job ______ with his ability as an accountant. A. appropriate B. suitable C. commensurate D. requisite 10. Don't count on him. He's liable to ______ out if things get too difficult. A. let B. be C. opt D. stop 11. In this assignment, we will ______ your work and then give you detailed feedback on how to improve your writing. A. assess B. judge C. measure D. test Page 2 of 12 12. 13. 14. 15. 16. 17. 18. 19. 20. In a seminar or tutorial, everyone should take part rather than allow one person to ______ the discussion. A. overwhelm B. dominate C. oppress D. empower Many people were killed instantly at Hiroshima and Nagasaki, but thousands more died from ______ radiation sickness. A. succeeding B. following C. subsequent D. afterwards I’ll move ______ to achieve my goal. A. heaven and earth B. mountains and seas C. paradise and hell D. milky way and horizon The 5% wage increases they propose are ______. A. all for one B. by and large C. top to borrow D. across the board Sarah: “I want you to give me some money” - Paul: “______” A. Oh you want, do you? B. Oh you’d like, would you? C. Oh, you give, do you? D. Oh you do, do you? She often appears not to care about her work, but appearances can be ______. A. cunning B. deceitful C. deceptive D. insincere As they watched the football match the huge crowd ______ in unison. A. chanted B. intoned C. crowed D. bellowed Nothing you say will make a ______ of difference to my decision. A. fragment B. scrap C. gram D. grain ______, Carol s very punctual, but she was late tonight. A. For a rule B. Like a rule C. For a rule of thumb D. As a rule Your answers: 1. 11. 2. 12. 3. 13. 4. 14. 5. 15. 6. 16. 7. 17. 8. 18. 9. 19. 10. 20. Question 2. Error Correction. The passage below contains 10 mistakes. Identify the mistakes and correct them in the space provided. (5pts) Lines 1 MTV stands for Music Television. It’s a television channel dedicating to pop music. It was 2 born on 1st August 1981 in the United States. Because of MTV’s instant succeed in the US, 3 the company expanded other areas. MTV Europe began operating on 1st August 1978. MTV 4 Europe broadcast 24 hours a day from it London studios. It can be seen in 33 countries and 5 reaches an estimated audiences of 110 million viewers. People of 19 different nationalities 6 work at London headquarters, and they try offering a mixture of music from all over 7 Europe. The channel broadcasts in English but Germany provides the bigger number of 8 viewers. Currently, one five of the music is by German artists. Most of TV output is video Page 3 of 12 9 10 11 12 and concerts, but there is also a programme calling Unplugged, where major artist play live and acoustic in front of a small studio audience. In addition on music, the channel’s programmes deal with news, movie information and comedy. MTV has also broadcast special report on racism, immigrate and unemployed teenagers. Your answers: Lines Mistakes Corrections 1. 2. 3. 4. 5. 6. 7. 8. 9. 10. Question 3: Fill in each blank in the sentences with a correct preposition or particle. (5 pts) 1. 2. 3. 4. 5. Jack didn’t expect to come up ______ such difficulties. Make yourself a drink while I go and slip ______ something more comfortable. The problem stems ______ the government’s lack of action. We put ______ a sum of money each month for our summer holidays. “She says she’s going to become a medicine student” - “Oh, don’t believe her. She’s having you ______. 6. Don’t worry, he only makes ______ that he is such a strict teacher. 7. We were convinced Patrick would take ______ the moment we met him. 8. "I understand Diane lost her job." - "Yes, but she's actually better ______." 9. Cottage is beautifully furnished and ______ close proximity to the beach. 10. You look very terrible! What have you been getting ______ to? Your answers: 1. 2. 3. 4. 5. 6. 7. 8. 9. 10. Page 4 of 12 Question 4: Give the correct form the word provided to fill each gap. (5 pts) 1. The people possess ______ rights. VIOLATE 2. Various ______ by police officers were brought to light by the enquiry. PRACTICE 3. Jim is one of the most ______ members of the committee. SPEAK 4. This is an original, but it’s a good ______. PRODUCE 5. The successful candidate will have superb ______ skills. LEAD 6. Kapo, the gorilla was born and bred in ______. CAPTIVE 7. She may look fierce but the lioness has ______ instincts like any other female animal. MOTHER 8. The fish in the river provide an ______ supply of fish for the young bears. ABOUND 9. Nowadays only a ______of wild crocodiles remain here. HAND 10. Patricia's skill in playing the piano is quite ______. No other child in this group can play the difficult pieces with similar mastery. COMPARE Your answers: 1 …………………………... 6 …………………………... 2 …………………………... 7 …………………………... 3 …………………………... 8 …………………………... 4 …………………………... 9 …………………………... 5 …………………………... 10 …………………………... Question 5: Choose the best word(s) to fill in each blank in the following passage. (10pts) Traffic lights The first traffic signal was invented by a railway signaling engineer. It was installed outside the House of Parliament in 1868. It (1)___ like any railway signal of the time, and was operated by gas. (2)___, it explored and killed a policeman, and the accident (3)___ further development until cars became common. Modern traffic lights are an American invention. Red-green systems were installed in Cleveland in 1914. Three-colour signals, operated (4)___ hand from a tower in the (5)___ of the street, were installed in New York in 1918. The (6)___ lights of this type to appear in Britain were in London, on the junction between St. Jam’s Street and Piccadilly, in 1925. Automatic signals were installed (7)___ year later. In the past, traffic lights were special. In New York, some lights had a statue on top. In Los Angeles the lights did not just (8)___ silently, but would ring bells to (9)___ the sleeping motorists of the 1930s. These are gone and have been (10)___ by standard models which are universally adopted. Page 5 of 12 1. A. resembled B. seemed C. showed D. looked 2. A. Although B. However C. Therefore D. Despite 3. 4. A. forbade A. through B. disappointed B. with C. discouraged C. by D. avoided D. in 5. A. halfway B. heart C. focus D. middle 6. 7. A. original A. the B. first B. in the C. primary C. in a D. early D. a 8. A. vary B. alter C. change D. move 9. A. rise B. wake C. raise D. get up 10. A. reproduced B. replaced C. removed D. remained Your answers: 1. 2. 3. 4. 5. 6. 7. 8. 9. 10. Question 6: Fill in the blank with one suitable word. Write your answers in the numbered blanks provided below. (5 pts) If all countries had the same monetary units, a difficult problem of international trade would be solved. One country’s money is not usually good in (1)___, however, and it is necessary to have a system for (2)___ the currency of the buyer into (3)___ of the seller. Bankers handle this by doing (4)___ is called buying or selling foreign exchange. When an exporter sells his goods to a merchant in a foreign country, he makes (5)___ a bill of exchange for the merchandise. The bill of exchange looks (6)___ a common bank check. The exporter sends this bill to his bank and receives his money. (7)___, the exporter receives payment in his own currency. The exporter’s bank sends the bill of exchange to (8)___ branch bank which notifies the (9)___ who pays the bill in his currency. The branch bank keeps his money and uses it to pay future (10)___ of exchange presented by merchants in that country who have goods to export. Your answers: 1 …………………………... 6 …………………………... 2 …………………………... 7 …………………………... 3 …………………………... 8 …………………………... 4 …………………………... 9 …………………………... 5 …………………………... 10 …………………………... Page 6 of 12 Part III. READING Question 1: Read the passage and choose the correct answer for the following questions. (10pts) History books record that the first film with sound was The Jazz Singer in 1927. But sound films, or “talkies”, did not suddenly appear after years of silent screenings. From the earliest public performance in 1896, films were accompanies by music and sound effects. These were produced by a single pianist, a small band, or a full-scale orchestra; large movie theatres could buy soundeffects machines. Research into sound that was reproduced at exactly the same time as the pictures - called “synchronized sound” - began soon after the very first film were shown. With synchronized sound, characters on the movie screen could sing and speak. As early as 1896, the newly invented gramophone, which played a large disc carrying music and dialogue, was used as a sound system. The biggest disadvantage was that the sound and pictures could become unsynchronized if, for example, the gramophone needle jumped or if the speed of the projector changed. This system was only effective for a single song or dialogue sequence. In the “sound - on - film” system, sounds were recorded as a series of marks on celluloid which could be read by an optical sensor. These signals would be placed on the film alongside the image, guaranteeing synchronization. Short feature films were produced in this way as early as 1922. This system eventually brought us “talking pictures”. 1. 2. 3. 4. 5. 6. The passage is mainly about the ______. A. history of silent movies B. disadvantages of synchronized sound C. development of sound with movies D. research into sound production According to the passage, films using sound effects were screened ______. A. before 1896 B. as early as 1922 C. as early as 1896 D. in 1927 The word “screenings” is closest in meaning to ______. A. revelations B. projections C. demonstrations D. diversions Which of the following is NOT mentioned as producer of sound to accompany movies? A. a gramophone B. a single pianist C. a small band D. a jazz singer It can be inferred that ______. A. gramophones were developed about the same time as moving pictures B. sound-effects machines were not common because they were expensive C. orchestras couldn’t synchronize sound with pictures D. most movie theatres had a pianist According to the passage, gramophone were ineffective because they ______. A. got out of synchronization with the pictures B. were too large for most movie theatres. C. were newly invented and still had imperfection D. changed speeds when the needle jumped Page 7 of 12 7. 8. 9. 10. The word “sequence” is closest in meaning to ______. A. interpretation B. progression C. distribution D. organization The phrase “these signals” refers to ______. A. sounds B. series C. marks D. sensors According to the passage, sound-on-film guaranteed synchronization because the record was ______. A. made during the filming of the pictures B. read by an optical sensor C. inserted beside the image on the film D. marked on the gramophone Short feature films produced as early as 1922 ______. A. were recorded by an optical sensor B. preceded talking pictures C. were only effective for dialogue sequences D. put musicians out of work Your answers: 1. 2. 3. 4. 5. 6. 7. 8. 9. 10. Question2: Read the following passage and choose the most suitable heading from the list A-I for each part (1-7) of the passage. There is one extra heading which you do not need to use. One example has been done for you. (7pts) A. B. C. D. E. F. G. H. I. Indoor climbing is preferred Early imperfections Putting up with nature Useful attachments Something in common The demand for indoor practice The inventor of the wall A lighter construction method Watching the expert GOING UP THE WALL 0 I The crowd holds its breath. High above them on the climbing wall, hanging upside down by the tips of two fingers, is the French climber Francois Lombard. He is competing in the World Cup Climbing Championships at Birmingham’s National Indoor Arena. 1 The National Indoor Arena is more famous for staging the TV show Gladiators, but the television programme and the World Cup Climbing Championships share at least one feature - The Wall. And the fact that either event is possible is the result of a new and rapidly developing technology. Page 8 of 12 2 Until the mid- 1960s, climbers practiced their skills on cliffs in areas where there was a plentiful supply of good climbing angles. During the winter they would either tolerate the cold weather, go walking instead or climb on snow and ice in Scotland. 3 However, as the sport developed it was increasingly important for top climbers to keep fit. With the cliffs unusable for much of the year, they used brick-edges or stone buildings to ‘work out’ on. This allowed them to keep their fingers strong and beat off the boredom of not being able to climb. It wasn’t long before many sports centre started building walls specifically for the task, using bricks with special edges to cling on to. 4 Many of these early walls followed the example set by Don Robison, a teacher of physical education who, during the mid- 1960s, constructed a climbing wall in corridor of his department at Leeds University. Robison developed the idea of setting natural rock in a block of concrete, which could then be included in a wall. 5 Scores of climbing walls of this kind were built in sports halls up and down the country throughout the 1970s but they had obvious design problems. Walls could only be built in a vertical plane, whereas cliffs outside have features like overhangs and angled slabs of rock. There was the added drawback that once the walls were up they couldn’t be altered and climbers would eventually tire of their repetitive nature, despite thinking of every combination of holds possible. 6 In 1985, a Frenchman, Francois Savigny, developed a material which he moulded into shapes like those that climbers would find on the cliffs. These could be fixed onto any existing wall and then taken off when climbers got bored with a particular combination. 7 French manufactures also began to experiment with panels on steel framework. Concrete had proved too heavy to create overhanging walls without major building work, but steel frames could be erected anywhere as free - standing structures. A system of interchangeable fixtures gave climbers an endless supply of new holds. Your answers: 1. 2. 3. 4. 5. 6. 7. Question 3: For questions 1-8, read the following text and then choose from the list A-K the best phrase given below to fill each of the spaces. Write one letter (A-K) in the correct space. Each correct phrase may be used only once. Some of the suggested answers do not fit at all. Page 9 of 12 London: A City for All Tastes On first encountering England's splendid capital, many tourists are overcome by a sense of complete and utter awe. It is the streaming lines of rush-hour traffic, the hurrying crowds on their way to work, and the scurrying shoppers seeking out bargains (1)______. Visitors are often at a loss as to where to go and what to visit first, and often worry about how to best spend the limited time (2)______. Most tourists quickly discover (3)______, however, for no matter where you go in the city (4)______. It is usually only then, after a long, but rewarding day of sightseeing, (5)______. It suddenly dawns on them, weary, thirsty and hungry as they are, (6)______! London is teeming with diverse places to eat and drink. Whether you fancy a formal sitdown meal or a quick take-away kebab, this city has something to offer the most discriminating tastes. Bear in mind, though, a trip to London would simply not be complete without a visit to one of the many traditional tea rooms (7)______.Treat yourself to a high tea of scones with jam and clotted cream. You'll soon realise that this is just one of the many things (8)______. A. B. C. D. E. F. G. H. I. J. K. that visitors make another important discovery about the metropolis that have only recently become so popular that have been around for over a century that tourists complain about the noise that makes sure visitors come back again there is always some museum, church or gallery well worth seeing that there are cafes, restaurants and snack bars on virtually every street corner that make London the diverse place that it is that they have available to them that the dilemma solves itself that the city has far too much traffic Your answers: 1. 2. 3. 4. 5. 6. 7. 8. Part IV. WRITING (25 pts) Question 1: Finish each of the sentences in such a way that it means exactly the same as the sentence printed before it. (5 pts) 1. We cannot see animals in a vast area after the forest fire. There is an ...................................................................................................... 2. We cannot make any comparison with her sacrifice. Nothing ............................................................................................................ 3. He will have to spend seven years in prison. He has been ...................................................................................................... Page 10 of 12 4. I expected the film to be good, but it wasn’t at all. The film didn’t ................................................................................................. 5. It was Sir Walter Barron who introduced potatoes and tobacco into England. The English owe .............................................................................................. Question 2: For each of the sentences below, write a new sentence as similar as possible in meaning to the original sentence using the word given. This word must not be altered in any way. (5 points) 1. Jack found it difficult to control his skis on the steep slope. (UNDER) ………………………..…………………………………………………………………. 2. Nobody expected her to lose, but she did. (AGAINST) …………………………………………………………………………………………… 3. Many people nowadays find it increasingly difficult to exist on the money they earn. (MAKE) …………………………………………………………………………………………… 4. You can borrow my bike if you're in a hurry. (MIND) …………………………………………………………………………………………… 5. I firmly believe him to be the rudest person I know. (WITHOUT) …………………………………………………………………………………………… Question 3: Paragraph writing (15 points) Write a paragraph of about 200 words about the following topic: “How do movies influence teenagers’ lifestyle nowadays?” ………………………………………………………………………………………………………. ………………………………………………………………………………………………………. ………………………………………………………………………………………………………. ………………………………………………………………………………………………………. ………………………………………………………………………………………………………. ………………………………………………………………………………………………………. ………………………………………………………………………………………………………. ………………………………………………………………………………………………………. ………………………………………………………………………………………………………. Page 11 of 12 ………………………………………………………………………………………………………. ………………………………………………………………………………………………………. ………………………………………………………………………………………………………. ………………………………………………………………………………………………………. ………………………………………………………………………………………………………. ………………………………………………………………………………………………………. ………………………………………………………………………………………………………. ………………………………………………………………………………………………………. ………………………………………………………………………………………………………. ………………………………………………………………………………………………………. ………………………………………………………………………………………………………. ………………………………………………………………………………………………………. ………………………………………………………………………………………………………. ………………………………………………………………………………………………………. ………………………………………………………………………………………………………. ………………………………………………………………………………………………………. ………………………………………………………………………………………………………. ………………………………………………………………………………………………………. ………………………………………………………………………………………………………. ………………………………………………………………………………………………………. ………………………………………………………………………………………………………. ………………………………………………………………………………………………………. ………………………………………………………………………………………………………. ………………………………………………………………………………………………………. ………………………………………………………………………………………………………. ………………………………………………………………………………………………………. ………………………………………………………………………………………………………. ………………………………………………………………………………………………………. ………………………………………………………………………………………………………. ………………………………………………………………………………………………………. ………………………………………………………………………………………………………. - the end – Page 12 of 12 TRƯỜNG THPT THUẬN THÀNH SỐ 1 Web: http://bacninh.edu.vn/thptthuanthanh1 Ngày 14/03/2013 (Đề thi gồm 05 trang) ĐỀ THI HSG CẤP TRƯỜNG NĂM HỌC 2012 – 2013 MÔN: TIẾNG ANH LỚP 10 Thời gian: 120 phút (Không kể thời gian giao đề) A. Phonetics Mark the letter A, B, C, or D on your answer sheet to indicate the word whose underlined part is pronounced differently from that of the rest in each of the following questions. 1: A. any B. can C. handbag D. sad 2: A. beat B. leather C. red D. headmaster 3: A. cheat B. Christmas C. chemist D. chaos 4: A. reason B. easy C. plays D. sincerely 5: A. plough B. around C. country D. mouth 6. A. thanks B. with C. through D. healthy 7. A. compare B. dear C. share D. wear 8. A .hoped B. finished C. crooked D. walked 9. A. wonderful B. monthly C. sorrow D. sunset 10. A. Mathematician B. American C. musician D. fashion B. Grammar and vocabulary I) Circle the letter A, B, C, or D on your answer sheet to indicate the correct answer to each of the following sentences. 1: John ___________ to return my book the next Friday. A. suggested B. promised C. proposed D. reminded 2: - "Would you like to join our volunteer group this summer?" - "______" A. Do you think I would? B. I wouldn't. Thank you. C. Yes, you're a good friend. D. Yes, I'd love to. Thanks. 3: I was very tired; ______, I determined to walk on to the next village. A. therefore B. however C. and D. for 4: I remember _____ you. I distinctively gave you $50 last week. A. pay B. to pay C. paid D. paying 5: _____ more carefully, he wouldn’t have been in the hospital now. ư A. Had he driven B. If he drove D. If he drives D. If he hadn’t driven 6: More than 50 films _____ in Hanoi since June. A. show B. were shown C. have been shown D. has been shown 7: You must lend me the money for the trip. _______, I won’t be able to go. A. Consequently B. Nevertheless C. Otherwise D. Although 8: The youths nowadays have many things to do in their ______ time. A. leisure B. entertainment C. fun D. amusement 9: He always ______ the crossword in the newspaper before breakfast. A. writes B. makes C. works D. does 10: Although the exam was difficult, ______ the students passed it. A. most of B. none of C. a few D. a lot 11: She _______the money so that no one would be able to find it. A. disclosed B. conveyed C. let on D. hid 12: John was made ______ out side because he was making a noise. A. go B. going C. to go D. gone 13: My watch had stopped so I had no way of knowing the right_______ . A. hour B. time C. moment D. o’clock 14: How often do you go swimming, Linda? - ……………………………… 1 A. I guess I’m OK B. once a week C. about one hour D. last night 15: “ _________ eating out tonight? ” _ “ That’s a good idea.” A. Would you B. How are you C. Are they D. How about 16: My son dislikes other people criticizing him. A.My son doesn’t like if you criticize him. B.Criticizing him is not my son’s hobby. C.My son is afraid of being criticized. D.My son can’t stand if other people criticize him. 17: The government hopes to _________ its plans for introducing cable TV. A. turn out B. carry out C. carry on D. keep on 18. Anyone caught throwing ____________ here should be fined. A. rubbish B. bag C. money D. flowers 19. Let’s go for a picnic this week, ____________? A. do we B. don’t we C. shall we D. have we 20. I can’t stand the car ____________ Therefore, I hate traveling by car. A. illness B. sickness C. ailment D. disease 21. He decided to buy a computer ____________ he didn’t have much money. A. because B. after C. although D. Despite 22. A person who is more beautiful in photographs is said to be ____________. A. photogenic B. photography C. photograph D. photographic 23. I wouldn’t waste time ___________that book if I were you. A. to reading B. reading C. read D. to be read 24. “How does Ann like her new school?” “Fine, she is doing ____________ in her course.” A. extreme good B. extremely good C. extreme well D. extremely well. 25. The series is watched by millions of ___________ A. viewing B. viewees C. viewers D. views II) Fill in each blank with a suitable preposition.) 1.Our team was very excited ----------------- going to the final match. 2.Bill seems unhappy in his job because he doesn’t get -----------------well with his boss. 3. -----------------my opinion, the third examination question was the most unreasonable. 4. Both Ann and her sister look -----------------her mother. 5. She’s never satisfied -----------------what she has. 6. She was unable to warn her mother that she would be late because the telephone was ----------------of order. 7. As you know, we cannot clean -----------------our polluted rivers and seas overnight. 8. Are you fond-----------------working with them? 9. It is difficult to imagine what our lives would be like -----------------computers. 10. It took John a long time to find a job after he left school. -----------------the end, he found a job as a waiter. III) Fill in each blank with a suitable word. Dolphins are not fish. They are mammals ______ live in water. Dolphins are______ the most intelligent animals on earth. Although they can be found in______ oceans in the world, dolphins prefer coastal waters and bays. The size of dolphins vary greatly. The smallest dolphin is just about 50 kg in _________and 1.2 meters in length while______ largest one can weigh up to 8,200 kg and is 100 meters ________Dolphins eat ______fish. A female dolphin gives birth ______one calf every two years after a ______period of eleven or twelve months. A dolphin can normally live from twenty five to sixty five years and some species of dolphins can even live longer. Dolphin populations are at risk ______to the pollution of their habitat and accidental entrapment in fishing nets. IV)The four underlined words or phrases are marked A, B, C, or D. Identify the one underlined expression that is not correct. Find out and correct it. (1p) 2 1. It hopeless to try and find any privacy in the flat so I go out for a walk. A B C D 2 . It was so an easy question that every one could answer it correctly. A B C D 3. The house it has green shutters is for sale. A B C D 4. She got angry when they started to asking about her private life. A B C D 5. People think computers never do any mistakes, but in fact, they do A B C D V) In this letter each line has a word which should not be there. Write your answers on the numbered blanks. Example – 0: enjoying (1 p) Dear Roberto, Thank you for your letter. I am glad to hear you are enjoying well . When I will have some free time, perhaps I’ll come to London to see you, and then we can go out for a meal together somewhere. I’ve just got come back from a trip to Lisbon. My father has just been opened a big new factory in Portugal, and he had wanted me to help him with marketing. Lisbon is said to be a very pretty beautiful city, but I didn’t have the chance to see much of it! Dad always makes me work hard! We had went to lots of business meetings, and I met all the staff. I would have been some more useful if I had learned Portuguese in stead of French at school! but luckily they are all speak very good English. In fact, Dad’s partner was invited me to lunch, and promised to teach me Portuguese, so maybe I’ll be accept his offer next time. On my way home the plane was delayed for four hours because of fog. I think Samantha was waiting at the airport all that time for me. It must be true love! Keep in touch, Best wishes, Thomas 0 => enjoying ..................................... ..................................... ..................................... ..................................... ..................................... ..................................... ..................................... ..................................... ..................................... ..................................... VI. Give the correct form of the verbs in bracket 1. Alice (take) ____ ____________________to the dentist many times. 2. He (do) ___________ ____________nothing before he saw me. 3. Tom is reading a book. He started two hours ago and he is on page 53. He (read) __ ____________for two hours. 4. What you (do)_______ _______ at this time yesterday? 5. (finish) ___ __________ reading the book , I went to bed. 6. The lamps (make) _ ____________ in China are very cheap. 7. He says as if he (come) _____ ________ back from London. 8. My lawyer advised me (not say) __ ______ anything further about the accident. 9. He wore dark glasses to avoid (recognize)______being recognized________ . 10. We were made ___________ (do) all the cleaning in the house. C. Reading comprehension Read the following passage and mark the letter A, B, C, or D on your answer sheet to indicate the correct answer to each of the questions Bill Jarvis took over our village news agency at time of life when most of us only want to relax. He just thought he would like something but not too much to do, and the news agency was ready-made. The business produced little enough for him, but then Bill was a chap who only wanted the simplicity and order and regularity of the job. He had been a long-serving sailor, and all his life had done everything by the clock. 3 Every day he opened his shop at six a. m to catch the early trade; the papers arrived on his doorstep before that. Many of Bill's customers were city workers, and the shop was convenient for the station. Business was tailing off by ten o'clock, so at eleven sharp Bill closed for lunch. It was hard luck on anybody who wanted a paper or magazine in the afternoon, for most likely Bill would be down on the river bank, fishing, and his nearest competitor was five kilometers away. Some time in the afternoon, the evening papers landed on the door-mat, and at four o'clock Bill reopened. The evening rush lasted till seven, and it was worthwhile. He lived in a flat above the shop, alone. Except in very bad weather, you always knew where to find him in the afternoons, as I have said. Once, on a sunny afternoon, I walked home along the river bank from a shopping trip to the village. By my watch it was three minutes past four, so I was astonished to see Bill sitting there on his little chair with a line in the water. He had no luck, I could see, but he was making no effort to move. "What's wrong, Bill?" I called out from the path. For answer, he put a hand inside his jacket and took out a big, golden object. For a moment I had no idea what it could be, and then it suddenly went off with a noise like a fire engine. Stopping the bell, Bill held the thing up and called back: "Ten to four, you see, and this is dead right." He stood up then and began to wind in his line. I had never known anyone carry a brass alarm clock round with him or her before. 1. Bill Jarvis became a newsagent when ________ A. He needed the money. B. He was quite an old man. C. He decided to take things easy. D. He gave up clock repairing. 2. What does the passage tell us about the news agency? A. It was an easy job with fixed hours. B. It was a very profitable business C. It was opened specially for Bill Jarvis D. It belonged to the railway and was part of the station. 3. Why did Bill open the shop so early in the day? A. He liked to do as much as possible before he went to work. B. Bill was never sure of the time. C. The shop had to be open when the morning papers came D. It was then that he did a lot of business. 4. We understand from the passage that the shop closed for lunch ________ A. At eleven o'clock more or less. B. At exactly eleven o'clock. C. Before eleven o'clock. D. Always after eleven o'clock. 5. You might say "hard luck" to someone who ________ A. has just heard some very good news. B. puts great effort into whatever he or she tries. C. is less fortunate than he or she ought to be. D. fails through his or her own fault entirely. 6. Why was the writer on the riverbank that afternoon? A. He was going to do some shopping in the village. B. He was fishing. C. He was going to get the evening paper. D. He was on his way home from the village 7. Why was the writer surprised when he saw Bill Jarvis? A. He thought it was late for Bill to be still fishing. B. Bill had not caught anything, and that seemed strange. C. He thought Bill was ill, because he was not moving at all D. He was surprised because Bill stayed in his flat in the afternoons. 8. From the information given in the passage, who- or what - do you think was wrong? A. The bell was; it must have gone off at the wrong time. 4 B. The writer's watch was fast. C. Bill was; he had dropped off to sleep D. Bill's clock was wrong; it was very old. 9. All of the following are true about Bill Jarvis EXCEPT __________ A. he lived alone B. He had ever worked as a sailor C. He was a newspaper man D. Fishing was his past time 10. What did Bill Jarvis often bring with him when he went fishing? A. a clock B. a gold C. a newspaper D. a fire engine D. Writing 1. Tom is the most industrious pupil. No other pupil _____________________________________________________________ 2. The last time I played football was in 1971. I ________________________________________________________________________ 3. I only bought the dog because my children wanted a pet. If _______________________________________________________________________ 4. The weather was so beautiful that we went swimming. It was ____________________________________________________________________ 5. I am really sorry I didn’t invite her to the party. I really wish _______________________________________________________________ II. Complete the second sentence so that it has a similar meaning to the first sentence, using the given word. 1. She owns all this land now. (belongs) All _____________________________________________________________________ 2. Organized activities don’t interest Eva very much. ( interested) Eva_____________________________________________________________________ 3. Mary is so young that she can’t join our club. (too) Mary is too young to join our club. 4. Do you like meat more than fish? (prefer) Do______________________________________________________________________? 5. He hasn’t got the intelligence to be a programmer ( intelligent) He ______________________________________________________________________ The end 5 SỞ GIÁO DỤC VÀ ĐÀO TẠO HÀ TĨNH KỲ THI CHỌN HỌC SINH GIỎI TỈNH CẤP THPT NĂM HỌC 2012 - 2013 Môn thi: TIẾNG ANH 10 Thời gian làm bài: 180 phút (Đề thi có 08 trang, gồm 11 phần) ĐỀ CHÍNH THỨC Lưu ý: → → → → Thí sinh không sử dụng bất kể tài liệu nào, kể cả từ điển. Thí sinh làm bài trực tiếp vào đề thi, ghi câu trả lời vào các ô cho sẵn ở cuối các phần. Riêng phần trắc nghiệm thí sinh chỉ ghi đáp án A, B, C hoặc D vào ô cho sẵn. Giám thị không giải thích gì thêm. Điểm của toàn bài thi (Bằng số) (Bằng chữ) Các giám khảo Số phách (Ký và ghi rõ họ tên) (Do Trưởng Ban chấm thi ghi) Giám khảo 1: Giám khảo 2: A. LISTENING PART I. Listen to the dialogue on the phone between and a man and a girl named Juliet and fill in the form. You are allowed to listen TWICE. Give your answers in the numbered spaces. Name: Juliet A. Eastman Age: (1) …………………………………… Hair color: (2) …………………………………… Eye color: (3) …………………………………… Height: (4) …………………………………… Occupation: (5) …………………………………… ………………………………………………………….. Likes: going out and having fun, sports, (6) ……………………………………… and (7) …………………………………………… Wants to meet someone who : (8) …………………………………………………, likes same (9)………………………………………………. and (10) ……………………………………… Part II. You are going to hear an expert talk about sleeping and dreaming. Listen and write True (T) or False (F) for each sentence. You are allowed to listen TWICE. Your answers True (T) 1. Women sleep more than men. 2. A sound sleeper moves less than a light sleeper. 3. Most people need 9 hours of sleep a night. 4. Reading in bed helps you sleep. 1 False (F) 5. Some people don’t dream at all. 6. The average person has about four dreams a night. 7. Not everyone can remember his or her dreams. 8. Eating before bed can give you nightmares. PART III. You are going to listen to a talk about Margaret Mead. Listen and choose the best answer A, B, C or D for each question. You are allowed to listen TWICE. 1. What was Margaret Mead’s job? A. a photographer B. a biologist C. an anthropologist D. a journalist 2. What was Margaret Mead’s main interest? A. taking photographs B. exploring new places C. how children were looked after. D. living in pour areas. 3. When did Margaret Mead go to Samoa? A. in 1901. B. in the 1920s. C. in 1938. D. in 1978. 4. Who did she interview in her first trip to Samoa? A. girls between 9 and 20 years old. B. boys and girls between 9 and 20 years old. C. women over 20 years old. D. men and women over 20 years old. 5. What was the title of Margaret Mead’s book? A. The pacific Islands. B. Teenagers around the World. C. Growing Up in New Guinea. D. Coming of Age in Samoa. 6. What was the main reason why Margaret Mead took photos? A. She liked photography. B. Cameras were not very common at that time. C. Her husband liked photos. D. It was the best way to share what she learned. 7. What is the main topic of the listening passage? A. Margaret Mead went to college in New York. B. Margaret Mead did research on the role of culture. C. Margaret Mead took photographs and wrote books. D. Margaret Mead was born in Philadelphia. Your answers 1. 2. 3. 4. 5. 6. 7. B. LEXICO – GRAMMAR PART IV. Choose the answer A, B, C or D which best fits the space in each of the following sentences. 1. _________ saying was so important that I asked everyone to stop talking and listen. A. What the woman was B. That the woman was C. The woman was D. When was the woman 2. -“Do you mind if I take a seat?” - “_____________.” A. Yes, I don’t mind B. No, do as you please C. No I mind D. Yes, do as you please 3. As the two teams left the football ground, the 100,000 _________ gave them a standing ovation. A. bystanders B. spectators C. viewers D. audiences 4 My parents lent me the money. _________, I couldn’t have afforded the trip. A. However B. Therefore C. Only if D. Otherwise 5. It is interesting to take _________ a new hobby such as collecting stamps or going fishing. A. over B. on C. in D. up 6. Jack made me _________ him next week. A. promise calling B. to promise calling C. to promise to call D. promise to call 2 7. “I passed the TOEFL test, Mom.” - “ _________.” A. All right B. Thank you C. Well done D. Good luck 8. The bad weather caused serious damage to the crop. If only it _______ warmer. A. was B. were C. has been D. had been 9. - “Eric is really upset about losing his job.” - “ Well, ____once myself, I can understand.” A. Having been fired B. Fired C. Having fired D. Being fired 10. ________ you, I’d think twice about that decision. I could be a bad move. A. Were I B. Should I be C. If I am D. If I had been 11. The teacher asked a difficult question, but finally Ted _________ a good answer. A. put up with B. keep pace with C. made way for D. came up with 12. Not only ________ to speak to him, but she also vowed never to see him again. A. she refused B. did she refuse C. she did refuse D. when she refused 13. The judge ________ the pedestrian for the accident. A. accused B. charged C. caught D. blamed 14. She had to borrow her sister’s car because hers was _________. A. out of work B. out of order C. on duty D. off work 15. We should participate in the movement _________ to conserve the natural environment. A. to organize B. organizing C. which organized D. organized 16. His brother refuses to even listen to anyone else’s point of view. He is very_________. A. open-minded B. kind-hearted C. narrow-minded D. absent-minded 17. There is _________ in my bedroom. A. a square wooden old table B. an old square wooden table C. a wooden old square table D. an old wooden square table 18. “I am sorry. I broke the vase”. - “ _________.” A. OK. Go ahead B. Yes, certainly C. Don’t worry. Things break D. I’d rather not. 19. One’s fingerprints are _________ other person. A. different from B. different from any C. differ from any D. different from those of any 20. He is very happy because he passed his exam with __________ colours. A. flying B. failing C. imagining D. changing Your answers: 1. 2. 3. 4. 5. 6. 7. 8. 9. 10. 11. 12. 13. 14. 15. 16. 17. 18. 19. 20 PART V. From four underlined parts, choose the one that needs correction then correct it. For example : The teacher did not allow the students discussing the take-home exam with each other. discussing → to discuss 1. A Geiger counter is an electronic instrument is used to measure the presence and intensity of radiation. 2. A dolphin locates underwater objects in their path by making a series of clicking and whistling sounds. 3. In spite of its small size, Europe had a great impact on world history than other continents. 4. Before she moved here , Alene has been president of the organization for four years. 5. That Marta's been chosen as the most outstanding student on her campus make her parents very happy. 6. My cousin composes not only the music, but also sings the songs for the major Broadway musicals. 7. Our civilization is so commonplace to us that rarely we stop to think about its complexity. 8. Ever since the world began, nations have difficulty in keeping peace with their neighbors. 3 9. Those of us who have a family history of heart disease should do yearly appointments with our doctors. 10. If one had thought about the alternatives, he would not have chosen such difficult a topic for a term paper. Your answers Mistake Correction 1. 2. 3. 4. 5. 6. 7. 8. 9. 10. PART VI. Fill in each space in the following sentences with the most suitable prepositions. 1. I'm afraid Tom's _______ work. But Jack's in. Would you like to speak to him? 2. Have you been to the theatre recently? ~ Yes, I was _______ the Old Vie last night. 3. At first I found the work very tiring, but _______ a few weeks I got used to it. 4. _______ the daytime the streets are crowded but at night they are quite deserted. 5. I saw Tom at the bus stop this morning but couldn't speak to him because we were standing _____ a queue. 6. He is always in a hurry. He drives _______ a tremendous speed. 7. Write ________ ink and put your name on the top of the page. 8. The man with the pipe and red hair is the brother of the girl ________ blue. 9. He sits at his desk all day with his head in his hands. It gets ________ my nerves. 10. The children hastily changed _______ bathing things and jumped into the river with shouts of delight. Your answers: 1. 2. 3. 4. 5. 6. 7. 8. 9. 10. PART VII. Give the correct form of the word in bracket to complete the passage. Your answers: The __1__ (say) “never judge a book by its cover” could not be more true for Ridiculous Rules by Marjorie Allen. The cover is completely blank, whereas 1. ............................................ the book is crammed full of wonderful examples and anecdotes. Allen is an 2. ............................................ __2__ (speak) critic of what is taught to native and non-native speakers of 3. ............................................ English, and has issued a __3__ (declare) of war against textbooks and style books which tell lies. 4............................................. Take the ridiculous and __4__ (mean) rule of never ending a sentence with a preposition. The lovely - if famous – story goes, that Winston Churchill, 5............................................. well-known for his numerous __5__ (write) as well as for being British Prime Minister during the Second World War, received a manuscript back from an 6. ............................................ ignorant __6__ (edit), who had told him rather rudely that he had to __7__ 7. ............................................ 4 (phrase) a sentence which ended with a preposition. Churchill responded by making the simple yet forceful __8__ (state) in the margin: “This is an 8. ............................................ impertinence up with which I will not put.” – the __9__ (imply) being that not 9. ............................................ to end a sentence with a preposition often sounds ridiculous in English, Sadly, 10. ............................................ Allen informs us that the story is probably mere __10__ (hear), and that Churchill may have actually only written “rubbish!” in the margin. C. READING PART VIII. Read the passage and choose the best answer A, B, C, or D to each of the questions Large animals that inhabit the desert have evolved a number of adaptations for reducing the effects of extreme heat. One adaptation is to be light in color, and to reflect rather than absorb the Sun's rays. Desert mammals also depart from the normal mammalian practice of maintaining a constant body temperature. Instead of trying to keep down the body temperature deep inside the body, which would involve the expenditure of water and energy, desert mammals allow their temperatures to rise to what would normally be fever height, and temperatures as high as 46 degrees Celsius have been measured in Grant's gazelles. The overheated body then cools down during the cold desert night, and indeed the temperature may fall unusually low by dawn, as low as 34 degrees Celsius in the camel. This is an advantage since the heat of the first few hours of daylight is absorbed in warming up the body, and an excessive buildup of heat does not begin until well into the day. Another strategy of large desert animals is to tolerate the loss of body water to a point that would be fatal for non-adapted animals. The camel can lose up to 30 percent of its body weight as water without harm to itself, whereas human beings die after losing only 12 to 13 percent of their body weight. An equally important adaptation is the ability to replenish this water loss at one drink. Desert animals can drink prodigious volumes in a short time, and camels have been known to imbibe over 100 liters in a few minutes. A very dehydrated person, on the other hand, cannot drink enough water to dehydrate at one session, because the human stomach is not sufficiently big and because a too rapid dilution of the body fluids causes death from water intoxication. The tolerance of water loss is of obvious advantage in the desert, as animals do not have to remain near a water hole but can obtain food from grazing sparse and far-flung pastures. Desert-adapted mammals have the further ability to feed normally when extremely dehydrated, it is a common experience in people that appetite is lost even under conditions of moderate thirst. 1. What is the main topic of the passage? A. Weather variations in the desert B. Adaptations of desert animals C. Diseased of desert animals D. Human use of desert animals. 2. According to the passage, why is light coloring an advantage to large desert animals? A. It helps them hide from predators. B. It does not absorb sunlight as much as dark colors. C. It helps them see their young at night D. It keeps them cool at night. 3. The word "maintaining" is closest in meaning to ___________. A. measuring B. inheriting C. preserving D. delaying 4. The author uses of Grant's gazelle as an example of ___________. A. an animal with a low average temperature B. an animal that is not as well adapted as the camel C. a desert animal that can withstand high body temperatures D. a desert animal with a constant body temperature 5. When is the internal temperature of a large desert mammal lower? A. Just before sunrise B. In the middle of the day C. Just after sunset D. Just after drinking 5 6. The word "tolerate" is closest in meaning to ___________. A. endure B. replace C. compensate D. reduce 7. What causes water intoxication? A. Drinking too much water very quickly B. Drinking polluted water C. Bacteria in water D. Lack of water. 8. What does the author imply about desert-adapted mammals? A. They do not need to eat much food. B. They can eat large quantities quickly C. They easily lose their appetites. D. They can travel long distances looking for food. 9. Why does the author mention humans in the second paragraph? A. To show how they use camels. B. To contrast them to desert mammals. C. To give instructions about desert survival. D. To show how they have adapted to desert life. 10. Which of the following is NOT mentioned as an adaptation of large desert animals? A. Variation in body temperatures B. Eating while dehydrated C. Drinking water quickly D. Being active at night. Your answers: 1. 2. 3. 4. 5. 6. 7. 8. 9. 10. PART IX. Read the text below and decide which answer A, B, C or D best fits each space. When you read something in a foreign language, you frequently (1)______ across words you do not fully understand. Sometimes you check the meaning in a dictionary and sometimes you (2)______. The strategy you adopt depends very much upon the (3)______ of accuracy you require and the time at your disposal. If you are the sort of person who tends to turn to the dictionary frequently, it is worth remembering that every dictionary has its (4)______. Each definition is only an approximation and one builds up an accurate picture of the meaning of a word only after meeting it in a (5)______ of contexts. It is also important to recognize the special dangers of dictionaries that translate from English into your native language and vice versa. If you must use a dictionary, it is usually far safer to (6)______ an English-English dictionary. In most exams you are not permitted to use a dictionary. (7)______ you are allowed to use one, it is very time-consuming to look up words, and time in exams is usually limited. You are, (8)______ , forced to guess the meaning of unfamiliar words. When you find unknown words in an exam text, it is very easy to panic. However, if you develop efficient techniques for guessing the meaning, you will (9)______ a number of possible problems and help yourself to understand far more of the text than you at first thought likely. Two strategies which may help you guess the meaning of a word are: using contextual clues, both within the sentence and outside, and making use of clues (10)______ from the formation of the word. 1. 2. 3. 4. 5. 6. 7. 8. 9. 10. A. put A. look A. extent A. limitations A. multiple A. survey A. or else A. so A. surpass A. derived B. drop B. guess B. range B. values B. variety B. consult B. Provided B. therefore B. get over B. extracted C. see C. examine C. degree C. advantages C. variation C. refer C. Although C. completely C. go over C. coming D. come D. inspect D. level D. entry D. diversity D. inquire D. Even if D. so that D. overcome D. originated Your answers: 1. 2. 3. 4. 5. 6. 7. 8. 9. 10. 6 D. WRITING PART X. Complete the second sentence so that it has similar meaning to the first one. 1. This will be my student's first performance in Canada. → This will be the first time …………………………………………………………………………... 2. This course will take us six months to complete. → In six months time ………………………………………………………………………………….. 3. The number of people who understand his ideas exceed his expectations. → More people ………………………………………………………………………………………… 4. She'll have to make her presentation at the end of his speech. → The moment he …………………………………………………………………………………….. 5. Sharon will finish her exams. Then she will have more free time. → Once ………………………………………………………………………………………………… 6. Both Mary and Peter prefer jazz to classical music. → Neither……………………………………………………………………………………………… 7. They repaired my car at the garage in town. → I............................................................................................................................................................ 8. This is the last time I will speak to you. → I........................................................................................................................................................... 9. I prefer staying in to going out. → I'd rather.............................................................................................................................................. 10. They passed the driving test because of the easy questions. → If …………………………………………………………………………………………………… Your answers: 1. ................................................................................................................................................................................................... 2. ................................................................................................................................................................................................... 3. ................................................................................................................................................................................................... 4. ................................................................................................................................................................................................... 5. ................................................................................................................................................................................................... 6. ................................................................................................................................................................................................... 7. ................................................................................................................................................................................................... 8. ................................................................................................................................................................................................... 9. .................................................................................................................................................................................................. 10. .................................................................................................................................................................................................. PART XI. ESSAY WRITING Some people say that cell phones have improved modern life. Others believe that cell phones have caused many problems to people. What is your opinion? In about 250 words, write an essay to assert your point of view on this problem. ………………………………………………………………………………………………………… ………………………………………………………………………………………………………… ………………………………………………………………………………………………………… 7 ………………………………………………………………………………………………………… ………………………………………………………………………………………………………… ………………………………………………………………………………………………………… ………………………………………………………………………………………………………… ………………………………………………………………………………………………………… ………………………………………………………………………………………………………… ………………………………………………………………………………………………………… ………………………………………………………………………………………………………… ………………………………………………………………………………………………………… ………………………………………………………………………………………………………… ………………………………………………………………………………………………………… ………………………………………………………………………………………………………… ………………………………………………………………………………………………………… ………………………………………………………………………………………………………… ………………………………………………………………………………………………………… ………………………………………………………………………………………………………… ………………………………………………………………………………………………………… ………………………………………………………………………………………………………… ………………………………………………………………………………………………………… ………………………………………………………………………………………………………… ………………………………………………………………………………………………………… ………………………………………………………………………………………………………… ………………………………………………………………………………………………………… ………………………………………………………………………………………………………… ………………………………………………………………………………………………………… ………………………………………………………………………………………………………… ………………………………………………………………………………………………………… ………………………………………………………………………………………………………… THE END 8 SỞ GIÁO DỤC VÀ ĐÀO TẠO HÀ TĨNH ĐÁP ÁN KỲ THI CHỌN HỌC SINH GIỎI TỈNH LỚP 9 THCS NĂM HỌC 2012 - 2013 Môn thi: TIẾNG ANH Tổng: 20 điểm, cụ thể như sau: A. LISTENING – 3 ĐIỂM PART I: 1,5 điểm = 0,15/ 1 câu đúng 1. twenty-three/ 23 2. blonde / blond 3. blue 4. average 5. computer programmer 6. music 7.( watching ) TV 8. outgoing 9. music/ sports 10. sports/ music PART II: 0,7 điểm = 0,1/ 1 câu đúng 1. F 2. F 3. F 4. T 5. F 6. T 7. T 8.F PART III: 0,8 điểm = 0,1/ 1 câu đúng 1. C 2. C 3. B 4. A 5. C 6. D 7. B B. LEXICO – GRAMMAR – 8 ĐIỂM PART IV: 4 điểm = 0,2 / 1 câu đúng 1. A 2. B 3. B 4. D 5. D 6. D 7. C 8. D 9. A 10.A 11. D 12. B 13. D 14. B 15. D 16. C 17. B 18. C 19. D 20. A PART V: 2 điểm = 0,1 / 1 câu đúng 1. is used → used 2. their path → its path 3. great impact → greater impact 4. has been → had been 5. make → makes/made 6. composes not only → not only composes 7. rarely we stop → rarely do we stop 8. have difficulty → have had difficulty 9. do yearly → make yearly 10. such difficult → so difficult PART VI: 1 điểm = 0,1 / 1 câu đúng 1. at 2. at 3. for 4. in 5. in 6. at 7. in 8. in 9. on 10. into PART VII: 1 điểm = 0,1 / 1 câu đúng 1. saying 2. outspoken 3. declaration 4. meaningless 5. writings 6. editor 7. rephrase 8. statement 9. implication 10. hearsay C. READING – 4 ĐIỂM PART VIII: 2 điểm = 0,2 / 1 câu đúng 9 1. B 2. B 3. C 4. C 5. A 6. A 7. A 8. D 9. B 10. D PART IX: 2 điểm = 0,2 / 1 câu đúng 1. D 2. B 3. C 4. C 5. B 6. B 7. D 8. B 9. D 10. A D. WRITING – 5 ĐIỂM Part IX: 2 điểm = 0,2 / 1 câu đúng my student has performed in Canada. 1. This will be the first time 2. In six months time we will have completed this course. 3. More people understand him than he expected./ has expected/ expects. 4. The moment he finishes she'll have to make her presentation. 5. Once Sharon finishes her exams, she will have more free time 6. Neither Mary nor Peter prefers classical music to jazz. 7. I had my car repaired at the garage in town. 8. I will not/never speak to you (again). 9. I'd rather stay in than go out. 10. If my student has given the performance in Canada. the questions hadn’t been easy, they wouldn’t have passed the driving test. the questions had been (more) difficult, they would have failed the driving test. It hadn’t been for the easy questions, Part X: 3 điểm - Nội dung (content): 1.5 điểm - Từ vựng (vocabulary): 0.5 điểm - Ngữ pháp (grammar): 0.5 điểm - Tính mạch lạc và trôi chảy (coherence and cohesion) + độ dài (length): 0.5 điểm 10 SỞ GD&ĐT VĨNH PHÚC ----------------ĐỀ CHÍNH THỨC (Đề thi gồm 5 trang) KỲ THI CHỌN HSG LỚP 10 THPT NĂM HỌC 2011-2012 ĐỀ THI MÔN: TIẾNG ANH (Dành cho học sinh THP không chuyên) Thời gian thi: 180 phút, không kể thời gian giao đề PART I. LISTENING You are going to hear a talk about security in the UK. Listen to the talk and complete the statements below by writing no more than THREE words in the spaces provide. You will hear the talk TWICE. • Don’t carry more (1)________ than you need for daily expenses. • When you stay at a hotel, ask the (2)_______ to keep your valuables in hotel (3)________. • Don’t keep a note of the serial (4)________ together with your traveler’s cheques. • You should carry wallets and purses in an (5)________ pocket or a handbag. • Your passport, (6) ____________ and other important documents should be taken special care of. • You can leave your (7) _________ luggage in a luggage office at most large stations and pick it up later. • It’s necessary to keep the receipt and check the (8) _________ hours when you leave your luggage at the station. • The (9) _________ Property Office can be found at both (10) __________ and the station. PART II. GRAMMAR AND VOCABULARY I. Complete the following sentences by choosing the correct answer among four options (A, B, C or D). 1. He's really shy _______ girl. A. by B. at C. for D. with 2. The teacher _______ her to improve her drawing. A. insisted B. encouraged C. made D. persisted 3. I couldn't quite ______ what they were doing because they were so far away. A. bear out B. make out C. think out D. try out 4. The meal Mary cooked tastes_______. A. well B. nice C. good D. worse 5. ______ at the party, we saw Ruth standing alone. A. Arriving B. We arrived C. Arrived D. We were arriving 6. The people who______ the survey said that they had examined over 1,000 accidents. A. gave B. proceed C. set D. conducted 7. The judge found him ______ of stealing and sent him to prison. A. evil B. innocent C. guilty D. wicked 8. The house we have rented is______. So we will have to buy some beds, chairs, tables, etc. A. unrestored B. unrepaired C. unfurnished D. undecorated 9. He was turned down for the job because he is ________. A. qualified B. qualifying C. unqualified D. qualification 10. The trouble started only______ the other man came into the room. A. when B. until C. and then D. too soon 11. _______, the disaster would not have happened. A. Had you have obeyed the orders B. You had obeyed the orders C. You obeyed the orders D. Had you obeyed the orders 12. _______ had booked in advance were allowed in. A. Only who B. Only those who C. Only who were those D. Only were those who 13. Traveling alone to a jungle is adventurous, ________. A. if not impossible B. if it not impossible C. when not impossible D. when it not impossible 1 14. I ______ the hot weather in the south. A. use to B. used to 15. The meat looked very _______ to the dog. A. invited B. invite C. am use to D. am used to C. inviting D. invitingly II. Use the correct form of each word on the right to complete the numbered spaces provided in the passage. Write your answers on your answer sheet. The mysteries of the skies Three hundred and fifty years before the first men looked down on the amazingly beautiful surface of the moon from close quarters, Galileo’s newly built telescope (1) _____________ him to look at the edge of the hitherto mysterious sphere. He saw that the apparently (2) _____________ surface was not divinely smooth and round, but bumpy and imperfect. He realized that although the moon might appear (3) _____________, resembling a still life painted by the hand of a cosmic (4) ____________, it was a real world, perhaps not very different from our own. This amounted to a great (5) _____________ hardly to be expected in his day and age, although nowadays his (6) _____________ may appear to some to be trivial and (7) _____________. Not long after Galileo lunar’s observations, the skies which had previously been so (8) _____________ revealed more of their extraordinary mysteries. Casting around for further wonders, Galileo focused his lens on the (9) _____________ planet of Jupiter. Nestling next to it, he saw four little points of light circling the distant planet. Our moon it appeared, perhaps (10) ____________ in the eyes of those fearful of what the discovery might mean, was not alone! 1. ABLE 2. LIVE 3. ACT 4. ART 5. ACHIEVE 6. CONCLUDE 7. SIGNIFY 8. ELUDE 9. STRIKE 10. FORTUNE III. In the following passage, some numbered lines contain a word that shouldn’t be there. Tick (√) the sentences that are correct and write the words that shouldn’t be there in the numbered space. KEEPING YOUR DISTANCE Personal space is a term that refers to the distance we like to keep between ourselves and other people. When someone we do not know well gets too close that we usually begin to feel uncomfortable. If such a business colleague comes closer than 1.2 meters, the most common response is to move away. Some interesting studies have been done in libraries. If strangers will come too close, many people get up and leave the building, others use to different methods such as turning their back on the intruder. Living in cities has made people to develop new skills for dealing with situations where they are very close to strangers. Most people on so crowded trains try not to look at strangers; they avoid skin contract, and apologize if hands touch by a mistake. People use newspapers as a barrier between themselves and other people, and if they do not have one, they stare into the distance, making sure they are not looking into anyone’s eyes. 2 0 ___√___ 00 someone 1________ 2 ________ 3 ________ 4 ________ 5 ________ 6 ________ 7 ________ 8 ________ 9 ________ 10 _______ PART III. READING I. Complete the following passage by choosing A, B, C or D to fill in each blank. In recent years, there has been a remarkable increase into happiness. The researchers have come up a number of factors which contribute to a definition of happiness. First of all, there is, in some people, a moderate genetic predisposition to be happy, in other words, happiness (1)_______ in families. And happiness seems to correlate quite strongly with the main dimensions of personalities: extroverts are generally happier, neurotics are less so. Second, people often report good social relations as a reason for their happiness. In particular, friends are a great (2) ______ of joy, partly because of the agreeable things they do together, partly because of the way friends use positive non-verbal (3) ______ such as caressing and touching, to affirm their friendship. Marriage and similar (4) ______ relationships can also form the basis of lasting happiness. Third, job satisfaction undoubtedly (5) ______ overall satisfaction, and vice versa - perhaps this is why some people are happy in boring jobs: it (6) ______ both ways. Job satisfaction is caused not only by the essential nature of the work, but (7)_____ by social interactions with co-workers. Unemployment, on the contrary, can be a serious cause of unhappiness. Fourth, leisure is important because it is more under individual (8) ______ than most other causes of happiness. Activities (9) _____ sport and music, and participation in voluntary work and social clubs of various kinds, can give great joy. This is partly because of the (10) ______themselves, but also because of the social support of other group members – it is very strong in the case of religious groups. 1. A. runs 2. A. source 3. A. movements 4. A. near 5. A. consists of 6. A. works 7. A. too 8. A. check 9. A. so 10. A. facilities B. arrives B. origin B. signals B. tight B. applies to B. effects B. as well B. power B. such B. activities C. goes C. base C. slogans C. close C. counts on C. makes C. also C. choice C. like C. exercises D. descends D. meaning D. motions D. heavy D. contributes to D. turns D. plus D. control D. thus D. amenities II. Read the passage carefully, then choose the correct option (marked A, B, C or D) to answer the questions. Scientists have established that influenza viruses taken from man can cause disease in animals. In addition, man can catch the disease from animals. In fact, a greater numbers of wild birds seem to carry the virus without showing any evidences of illness. Some scientists conclude that a large family of influenza virus may have evolved in the bird kingdom, a group that has been on earth 100 million years and is able to carry the virus without contracting the disease. There is even convincing evidence to show that virus strain are transmitted from place to place and from continent to continent by migrating birds. It is known that two influenza viruses can recombine when both are present in an animal at the same time. The result of such recombination is a great variety of strains containing different H and N spikes. This raises the possibility that a human influenza virus can recombine with an influenza virus from a lower animal to produce an entirely new spike. Research is underway to determine if that is the way major new strains come into being. Another possibility is that two animal influenza strains may recombine in a pig, for example, to produce a new strain which is transmitted to man. 1. According to the passage, scientists have discovered that influenza viruses ______. A. cause ill health in wild animals B. do not always cause symptoms in birds C. are rarely present in wild birds D. change when transmitted from animals to man 2. What is known about the influenza virus? A. It was first found in a group of very old birds. B. All the different strains can be found in wild birds. C. It existed over 100 million years ago. D. It can survive in many different places. 3 3. According to the passage, a great variety of influenza strains can appear when______. A. H and N spikes are produced B. animal and bird viruses are combined C. dissimilar types of viruses recombine D. two viruses of the same type are contracted 4. New strains of viruses are transmitted to man by_______. A. a type of wild pig B. diseased lower animals C. a group of migrating birds D. a variety of means 5. It can be inferred from the passage that all of the following are ways of producing new strains of influenza EXCEPT_______. A. two influenza viruses in the same animal recombining B. animal viruses recombining with human viruses C. two animal viruses recombining D. two animal viruses recombining in a human III. Read the passage and choose the best answer from the four options marked A, B, C or D in the following questions. Identify your answer by writing the corresponding letter A, B, C or D on your answer sheet. Several hundred million years ago, plants similar to modern ferns covered vast stretches of the land. Some were as large as trees, with giant fronds bunched at the top of trunks as straight as pillars. Others were the size of bushes and formed thickets of undergrowth. Still others lived in the shade of giant club mosses and horsetails along the edges of swampy lagoons where giant amphibians swam. A great number of these plants were true ferns, reproducing themselves without fruits or seeds. Others had only the appearance of ferns. Their leaves had organs of sexual reproduction and produced seeds. Although their “flowers” did not have corollas, these false ferns (today completely extinct) ushered in the era of flowering plants. Traces of these floras of the earliest times have been preserved in the form of fossils. Such traces are most commonly found in shale and sandstone rocks wedged between coal beds. Today only tropical forests bear living proof of the ancient greatness of ferns. The species that grow there are no longer those of the Carboniferous period, but their variety and vast numbers, and the great size of some, remind us of the time when ferns ruled the plant kingdom. 1. What does the passage mainly discuss? A. Plant reproduction B. How to locate fossils C. An ancient form of plant life D. Tropical plant life 2. The word “Others” refers to _________. A. plants B. pillars C. trees D. fronds 3. Which of the following is NOT mentioned as a characteristic of the plants described in the passage? A. They once spread over large areas of land. B. They varied greatly in size. C. They coexisted with amphibians, mosses, and horsetails. D. They clung to tree trunks and bushes for support. 4. The word “true” is closest in meaning to which of the following? A. accurate B. genuine C. straight D. dependable 5. The author states that fossils of early plant life are usually found in rocks located between deposits of _______. A. coal B. shale C. sandstone D. corollas IV. Read the passage carefully then fill in the blank a suitable word. As swimming became a popular recreation in England during the 1860s and 1870s, several (1) ______ sports developed, roughly patterned after land sports. (2) ______ them were water football (or soccer), water rugby, water handball, and water polo, in which players rode on floating barrels, painted to look (3) ______ horses, and struck the ball with a stick. Water rugby became most popular of these sports, but somehow the water polo name became attached to it, and it's been attached (4) ______ since. 4 As played in England, the object of the sport was for a player to touch the ball, with both (5) ______, at the goal end of the pool. The goaltender stood on the pool deck, ready to dive on any opponent who was about to score. Water polo quickly became a very rough sport, filled (6) ______ underwater fights away from the ball, and it wasn't unusual for players to pass out for lack of air. In 1877, the sport was tamed in Scotland by the addiction of goalposts. The Scots also replaced (7) ______ original small, hard rubber ball with a soccer ball and adopted (8) ______ that prohibited taking the ball under the surface or, "tackling" a player unless he had the ball. The Scottish game, which emphasized swimming speed, passing, and (9) ______ work, spread to England during the early 1880s, to Hungary in 1889, to Austria and Germany in 1894, to France in 1895, and (10) ______ Belgium in 1900. Water polo was the first team sport added to the Olympic program, in 1900. PART IV: WRITING I. Write the new sentences using the given word. Do not change the word given in any way. 1. They have discovered some interesting new information. (LIGHT) 2. They suspended Jack for the next two matches. (BANNED) 3. I really want to see her again. (DYING) 4. She was so beautiful that I couldn't stop looking at her. (EYES) 5. We are looking forward to watching the program. (WAIT) II. Rewrite each of the following sentences so that it has a similar meaning to the original one. 1. If you changed your mind, you would be welcomed to join our class. → Were you______________________________________________________ 2. I'd rather not go out this afternoon. → I do not feel____________________________________________________ 3. Adeles tries hard, but she doesn't get anywhere. → However______________________________________________________ 4. It is thought that the boss is considering raising wages. → The boss______________________________________________________ 5. His disabilities did not prevent him from sailing around the world. → Despite the fact_________________________________________________ 6. I didn't arrive in time to see her. → I wasn't_______________________________________________________ 7. I'd prefer you not to smoke. → I'd rather______________________________________________________ 8. The mother smiled happily. She took the baby in her arms. → Smiling________________________________________________________ 9. The noise next door did not stop until after midnight. → It was not ______________________________________________________ 10. You can ring this number whenever there is any difficulty. → Should _________________________________________________________ -------------------------------The end--------------------------------- 5 HƯỚNG DẪN CHẤM THI HSG TIẾNG ANH LỚP 10 THPT – NĂM HỌC 2011 - 2012 PART I. LISTENING (15 pts: 1,5pts/item) 1. cash 4. numbers 7. heavy 10. the airport 2. manager 5. inside 8. opening 3. safe 6. travel tickets 9. Lost PART II. GRAMMAR AND VOCABULARY (35pts) I. (15pts: 1pt/item) 1. D 2. B 3. B 4. C 5. A 6. D 7. C 8. C 9. C 10. A 11. D 12. B 13. A 14. D 15. C II. (10pts: 0.1pt/item) 1. enabled 4. artist 7. insignificant 10. unfortunately 2. lifeless 5. achievement 8. elusive 3. inactive 6. conclusion 9. striking III.(10pts: 1pt/item) 1. that 9. a 5. to 3. √ 7. √ 2. such 4. will 6. to 8. so 10. √ PART III . READING (30pts) I. (10pts: 1pt/item) 1. A 3. B 2. A 4. C II. (5pts: 1pt/item) 1. B 2. D III. (5pts: 1pt/item) 1. C 2. A IV. ( 10pts: 1pt/item) 1. water 2. Among 6. with 7. the 5. D 6. A 7. C 8. D 9. C 10. B 3. C 4. D 5. D 3. D 4. B 5. A 3. like 8. rules 4. ever 9. team 5. hands 10. to PART IV. WRITING (20pts) I. (10pt: 2 pts/item) 1. Some interesting new information has come to light. 2. Jack was banned from playing in the next two matches. 3. I'm dying to see her again. 4. She was so beautiful that I couldn't take my eyes off her. 5. We can't wait to watch the program. II. (10 pts: 1pt/item) 1. Were you to change your mind, you would be welcomed to join our class. 2. I do not feel like going out this afternoon. 3. However hard Adeles tries, she doesn't get anywhere/gets nowhere. 4. The boss is thought to be considering raising wages. 5. Despite the fact that he was disabled, he sailed/managed to sail around the world. 6. I wasn't early enough to see her. 7. I'd rather you didn't smoke. 8. Smiling happily, the mother took the baby in her arms. 9. It was not until after midnight that the noise next door stopped 10. Should there is any difficulty, you can ring this number. 6 SỞ GIÁO DỤC VÀ ĐÀO TẠO HÀ TĨNH ĐỀ CHÍNH THỨC KỲ THI TUYỂN SINH LỚP 10 THPT CHUYÊN HÀ TĨNH NĂM HỌC 2013 - 2014 Môn: TIẾNG ANH (Cho thí sinh chuyên) Thời gian làm bài: 150 phút (Đề thi có 8 trang, gồm 11 phần) Lưu ý: → → → → Thí sinh không sử dụng bất kể tài liệu nào, kể cả từ điển. Thí sinh làm bài trực tiếp vào Đề thi, ghi câu trả lời vào các ô cho sẵn ở cuối các phần. Riêng phần trắc nghiệm thí sinh chỉ ghi đáp án A, B, C hoặc D vào ô cho sẵn. Giám thị không giải thích gì thêm. Điểm của toàn bài thi (Bằng số) (Bằng chữ) Các giám khảo Số phách (Ký và ghi rõ họ tên) (Do Trưởng Ban chấm thi ghi) Giám khảo 1: Giám khảo 2: A. LISTENING Part I: You will hear a person talking about first impressions. Listen and fill in the blanks with the missing words. You are allowed to listen twice. According to ___1__, people form first impressions based on how you look, then on how you sound, and finally on what you say. The way you look makes up 55% of a first impression. This includes facial expressions, body language, and eye ___2___, as well as your clothes and general __3__. A friendly smile seems to be the most important part of this. The way you sound makes up 38% of a first impression and includes how fast or slowly you speak. People listen to your __4__ of voice and decide if you sound friendly or unfriendly, interested or __5__, and happy or sad. What we say - the actual words counts for only 7% of the __6__. People form first impressions within 10 seconds of meeting you. And first impressions don’t change __7__. If someone gets the wrong impression of you, it can take a long time to change his or her mind. One problem is that in diferrent parts of the world, the same __8__ can give people a different impression. In some countries, if you look at the person directly, it shows you are friendly. In other countries, it can be rude, and it’s more polite to look away from the person. Standing close to someone and touching the person’s __9__ is normal in some places, but in others it’s better to keep your __10__. The person may think you are being too …. Your answers: 1. 2. 3. 4. 5. 6. 7. 8. 9. 10. 1 Part II: Four people are talking about their best friends. Listen then complete the table below. You are allowed to listen twice. What they do together When they met Ron 1. …………………………………………….. 2. ………… years ago Sally 3. …………………………………………….. 4. ………… years ago 5. …………………………………………….. Ken 6. ……………………………………………. 7. ……… years ago 8. ……………………………………………. Alice 9. ……………………………………………. 10. ………years ago Part III: You are about to hear a news report. Listen and then choose the best answer A, B or C for each sentence. You are allowed to listen twice. 1. What is the main topic? A. a school in Italy B. a special kind of school C. children with special needs 2. Which of these is NOT true about Montessori schools? A. Children walk around. B. Teachers tell students what to do. C. There are a lot of materials to choose from. 3. Maria Montessori was __________. A. a psychiatrist B. the first woman doctor in Italy C. the mother of a student with special needs 4. Dr. Montessori believed that children __________. A. can teach themselves B. need strict discipline in school C. need a teacher to show them everything 5. How old was Dr. Montessori when she started to teach? A. 18 B. 21 C. 28 6. Which of the following is true about the children Dr. Montessori taught? A. They did better than other children on tests. B. They did as well on tests as other children. C. They studied in another school before she worked with them. 7. Which of the following is NOT true? A. Dr. Montessori’s ideas were not popular. B. Dr. Montessori still works with children. C. Dr. Montessori developed materials for children. 8. How many Montessori programs are there in the world? A. ten B. hundreds C. thousands 9. What do Montessori teachers believe? A. Students will learn on their own. B. Teachers have to tell students what to do. C. Children need to have a schedule for every hour of class. 10. What kinds of children can learn in a Montessori school? A. only children with special needs B. only children from regular schools C. all types of children Your answers: 1. 2. 3. 4. 5. 6. 7. 8. 9. 10. 2 B. LEXICO – GRAMMAR Part IV: Choose the correct answer A, B, C or D to complete each of the following sentences. 1. Michael: “We’ll have to hurry if we want to finish this project on time.” - John: “__________” A. OK. But I’ll call you later. B. I’ll say we will! C. I’m tired. I’ll go home early. D. What a wonderful idea! 2. _______for farming purposes, soil must contain the minerals plants require. A. To be good B. Being good C. Be good D. That's good 3. There was a ____________________ table in the kitchen. A. beautiful large round wooden B. large beautiful round wooden C. beautiful round large wooden D. golden large beautiful round 4. I really can’t __________ the twins apart. They look so alike. A. say B. talk C. take D. tell 5. ______ in astronomy, the discovery of Uranus was by accident. A. Alike many finds B. Many alike finds C. Like many finds D. It was like many finds 6. The young generation must contribute to ___________ the country. A. building of B. the building C. the building of D. build 7. I can not help feeling anxious ___________ the exam results. A. on B. with C. about D. of 8. When Elvis Presley died, her daughter came __________ a fortune. A. into B. up with C. over D. across 9. “Go on, finish the desert. It needs _________ up because it won’t stay fresh until tomorrow.” A. eat B. eating C. to eat D. eaten 10. Fossil fuels like coal, oil, and gas produce carbon dioxide when __________ . A. burned B. are burned C. they burned D. are they burned 11. The teacher suspected the students ___________ cheating in the exam. A. on B. about C. in D. of 12. Alain: “Would you like to have dinner with me?” - Helen: “__________.” A. Yes, it is B. Thank you. I’m very happy C. Yes, so do I D. Yes, I’d love to 13. After her illness, Lam had to work hard to _______ her classmates. A. catch sight of B. keep pace with C. get in touch with D. make allowance for 14. I thought you said she was going away the following Sunday, _______? A. wasn't she B. didn't I C. didn't you D. wasn't it 15. The ________ polluted atmosphere in some industrial regions is called "smog.” A. much B. largely C. fully D. heavily Your answers: 1. 2. 3. 4. 5. 6. 7. 8. 9. 10. 11. 12. 13. 14. 15. Part V: Each line in the passage contains one mistake. Underline the mistakes then and correct them. Your answers: Language is considered a significant means for communication in international 1.................................. relations. People in modern world have not longer been bound to their own 2.................................. countries. Moreover, in keeping inform with speedly scientific developments 3.................................. 3 nowadays, they must widen their cooperative relations with the highly developing 4.................................. nations in other parts of the world. Due to the fact that no one are able to 5.................................. communicate with any natives in the world, that is necessary for people to choose to 6.................................. learn a foreign language as a worldwide common medium of communication. Which 7.................................. is why English has been chosen by almost people to be a foreign language to master. 8.................................. More and more people using English and the number of those who learn English as a 9.................................. foreign language has been increased so rapidly that it is the highest nowadays. 10................................ Part VI: Read the text and then give the correct forms of the words in CAPITALS to complete the gaps. The First amendment to the American Constitution declares __1__ (FREE) of the press to all people. Although this right was not __2__ (OFFICE) adopted until 1791, the famous Zenger trial of 1735 laid the groundwork for the insuring of this precious freedom. John Peter Zenger emigrated as a __3__ (TEEN) from Germany. In 1733, he began publishing the New York Weekly Journal. The following year, he was arrested for writing a story about the crown-appointed __4__ (GOVERNMENT) of New York. While John Peter Zenger was __5__ (PRISON) for nine months, his wife - an American woman__6__ (DUTY) published the newspaper every day, __7__ (BRAVE) telling the truth about the corrupt government __8__ (OFFICE) sent by the king to govern the colonies. Finally Zenger’s long-awaited trial took place. The hostile judge dismissed Zinger's local lawyers, making it necessary for his wife to seek out Andrew Hamilton, a prominent Philadelphia lawyer. Persuaded by Hamilton, the jury returned a not-guilty verdict, defying the judge’s orders for a __9__ (CONVICT). As a result of determination and bravery on the part of the colonists, a __10__ (LAST) victory for freedom of the press was gained by a young immigrant. Your answers: 1. 6. 2. 7. 3. 8. 4. 9. 5. 10. C. READING Part VII: Read the text and then choose the correct answer A, B, C or D for each space. SUMMER IN THE CITY Summer is a wonderful season because it means sun, beaches and having fun. However, summer in the city can make one suffer. A mixture of airborne gases and heat can ___1___ something of a mini-greenhouse effect, which is harmful to our health. This, in turn, brings ___2__ a rise in temperature, making it essential to seek __3__ in an air-conditioned building. Some cities even have the added problem of smog. This, combined with the heat, makes conditions unbearable. Asthmatics, who are considered to be more __4__ during the summer season, are the ones who are __5__ the most by these conditions and their lives are often __6__. It is therefore important that these people __7__ going out when pollution levels are high. 4 According to official records, the increase in temperature coincides with a higher number of deaths. Twenty percent more people die from heart attacks or are killed in car accidents. However, there is hope for city dwellers. Experts say that the situation can be improved if we pay more __8___ to looking after our environment. One step would be to __9___ the number of vehicles and limit heavy industry, which produce toxic emissions. Even ___10__ dark-coloured roofs with lighter-coloured ones can cause a drop in temperature of up to 4 degree Celsius. 1 2 3 4 5 6 7 8 9 10 A. result A. up A. rescue A. sensitive A. affected A. warned A. omit A. care A. demolish A. exchanging B. gain B. about B. guard B. sensational B. attacked B. risked B. avoid B. caution B. lower B. urging C. remain C. round C. shelter C. sensible C. obliged C. damaged C. ban C. interest C. reduce C. advancing D. create D. along D. caution D. influenced D. appealed D. endangered D. prevent D. attention D. destroy D. replacing Your answers: 1. 2. 3. 4. 5. 6. 7. 8. 9. 10. Part VIII: Read the passage carefully and find the missing word to complete each space. Thomas Edison, the inventor of the light bulb, thought that sleep was unnecessary and a society that operated ___1__ sleep would be an ideal one. However, he was wrong. Nowadays, we are living in a society which is sleeping less than ever and this has resulted ___2__ many negative effects. According to police reports, more than twenty five percent of motorway accidents are directly or indirectly attributed to lack of sleep. It is true that people ___3___ to sleep nine hours a night, whereas today they sleep seven or even __4___. This is because our ___5__ of life has changed and we are trying to squeeze activities like watching television, shopping or going to the gym in our 24 hour day. Lack of sleep also has a negative effect ___6___ our health. Research that has been carried out on animals continuously deprived of sleep has shown that they are likely to die. Of course, experiments of this kind are highly unlikely to be carried out on humans ___7___. The longest period of wakefulness which ___8__ the record was eleven days. This experiment showed that after a few days not to sleep, the mind and body were __9__ to function normally. To put it in a nutshell, the vast majority of us ___10__ an adequate amount of sleep in every twenty-fourhour period. Your answers: 1. 2. 3. 4. 5. 6. 7. 8. 9. 10. PART IX: Read the passage and choose the best answer A, B, C, or D to each of the questions. MOBILE PHONES: Are they about to transform our lives? We love them so much that some of us sleep with them under the pillow, yet we are increasingly concerned that we cannot escape their electronic reach. We use them to convey our most intimate secrets, yet we worry that they are a threat to our privacy. We rely on them more than the Internet to cope with modern life, yet many of us don’t believe advertisements saying we need more advanced services. 5 Sweeping aside the doubts that many people feel about the benefits of new third generation phones and fears over the health effects of phone masts, a recent report claims that the long-term effects of new mobile technologies will be entirely positive so long as the public can be convinced to make use of them. Research about users of mobile phones reveals that the mobile has already moved beyond being a mere practical communication tool to become the backbone of modern social life, from love affairs to friendship to work. The close relationship between user and phone is most pronounced among teenagers, the report says, who regard their mobiles as an expression of their identity. This is partly because mobiles are seen as being beyond the control of parents. But the researchers suggest that another reason may be that mobiles, especially text messaging is seen as a way of overcoming shyness. The impact of phones, however, has been local rather than global, supporting existing friendship and networks, rather than opening users to a new broader community. Even the language of texting in one area can be incomprehensible to anybody from another area. Among the most important benefits of using mobile phones, the report claims, will be a vastly improved mobile infrastructure, providing gains throughout the economy, and the provision of a more sophisticated location-based services for users. The report calls on government to put more effort into the delivery of services by mobile phone, with suggestion including public transport and traffic information and doctors’ text messages to remind patients of appointments. There are many possibilities. At a recent trade fair in Sweden, a mobile navigation product was launched. When the user enters a destination, a route is automatically downloaded to their mobile and presented by voiced, pictures and maps as they drive. In future, these devices will also be able to plan around congestion and road works in real time. Third generation phones will also allow for remote monitoring of patients by doctors. In Britain scientists are developing an asthma management solution using mobiles to detect early signs of an attack. Mobile phones can be used in education. A group of teachers in Britain use third generation phones to provide fast internet service to children who live beyond the reach of terrestrial broadband services and can have no access to online information. ‘As the new generation of mobile technologies takes off, the social potential of the vastly increase,’ the report argues. 1. What does the writer suggest in the first paragraph about our attitudes to mobile phones? A. We can’t live without them. B. We are worried about using them so much. C. We have contradictory feelings about them. D. We need them more than anything else to deal with modern life. 2. What is the connection between social life and mobile phones? A. Modern social life relies significantly on the use of mobile phones B. Mobile phones make romantic communication easier. C. Mobile phones encourage people to make friends. D. Mobile phones enable people to communicate while moving around 3. Why do teenagers have such a close relationship with their mobile phones? A. They use text messages more than any other group B. They feel independent when they use them C. They are more inclined to be late than older people D. They tend to feel uncomfortable in many situations 4. How might mobile phones be used in the future? A. To give the address of the nearest doctor’s surgery B. To arrange deliveries C. To show bus and train timetables D. To cure diseases 5. What is the general attitude of the report described here? A. Manufacturers need to produce better equipment. B. The government should take over the mobile phone networks. C. There are problems with mobile phones that cannot be overcome. D. Mobile phones can have a variety of very useful applications. 6 Your answers: 1. 2. 3. 4. 5. D. WRITING PART X. Complete the second sentence so that it has similar meaning to the first one. 1. In order to get to the solution, we had to start again from the beginning. → Only by ……………………………………………………………………………….……….……………….. 2. It’s the sunset that attracts tourists to that place. → If it weren’t ……………..……………………………………………………………..….……………………. 3. I had expected the weather to be worse. → The weather wasn’t ……………………………………………………………………...……….……………. 4. His behavior is beginning to annoy me more and more. → I am beginning ..................................................................................................................................................... 5. You just can not compare the quality of her work with his. → There’s ………..................................................................................................................................................... Complete the second sentence so that it has a similar meaning to the first sentence, using the word given and do not change it. You must use between three and seven words, including the word given. 6. Valerie found it hard to concentrate on her book because of the noise. (difficulty) → Valerie ........................................................................................................... on her book because of the noise. 7. My new MP3 player was twice the price of yours. (expensive) → Your MP3 player was ………………….……………………………………. mine. 8. Pauline doesn’t want to be given a main part in the school play. (rather) → Pauline …………….…………………………………..……………………... a main part in the school play. 9. I go to the cinema every Thursday in winter. → I always …………………………………………………….……………….. in winter. (Thursdays) 10. “Do fast cars interest you?” The dealer asked Sarah. (was) → The dealer asked Sarah ................................................................................... fast cars. PART XI. ESSAY WRITING Nowadays our country is facing some big problems (pollution of environment, traffic accidents, unemployment, overcharging in tourism…) In your opinion, which of these problems is the most serious? Write an essay (about 200 words) to give some causes of that problem and suggest the solutions to stop it. ………………………………………………………………………………………………………………… ………………………………………………………………………………………………………………… ………………………………………………………………………………………………………………… ………………………………………………………………………………………………………………… ………………………………………………………………………………………………………………… …………………………………………………………………………………………………………………. 7 ………………………………………………………………………………………………………………… ………………………………………………………………………………………………………………… ………………………………………………………………………………………………………………… ………………………………………………………………………………………………………………… ………………………………………………………………………………………………………………… …………………………………………………………………………………………………………………. ………………………………………………………………………………………………………………… ………………………………………………………………………………………………………………… ………………………………………………………………………………………………………………… ………………………………………………………………………………………………………………… ………………………………………………………………………………………………………………… …………………………………………………………………………………………………………………. ………………………………………………………………………………………………………………… ………………………………………………………………………………………………………………… ………………………………………………………………………………………………………………… ………………………………………………………………………………………………………………… ………………………………………………………………………………………………………………… …………………………………………………………………………………………………………………. ………………………………………………………………………………………………………………… ………………………………………………………………………………………………………………… ………………………………………………………………………………………………………………… ………………………………………………………………………………………………………………… ………………………………………………………………………………………………………………… …………………………………………………………………………………………………………………. ………………………………………………………………………………………………………………… ………………………………………………………………………………………………………………… …………………………………………………………………………………………………………………. …………………………………………………………………………………………………………………. __The end__ 8 SỞ GIÁO DỤC VÀ ĐÀO TẠO HÀ TĨNH ĐỀ CHÍNH THỨC Lưu ý: → → → → KỲ THI CHỌN HỌC SINH GIỎI TỈNH CẤP THPT NĂM HỌC 2012 - 2013 Môn thi: TIẾNG ANH 10 Thời gian làm bài: 180 phút (Đề thi có 08 trang, gồm 11 phần) Thí sinh không sử dụng bất kể tài liệu nào, kể cả từ điển. Thí sinh làm bài trực tiếp vào đề thi, ghi câu trả lời vào các ô cho sẵn ở cuối các phần. Riêng phần trắc nghiệm thí sinh chỉ ghi đáp án A, B, C hoặc D vào ô cho sẵn. Giám thị không giải thích gì thêm. Điểm của toàn bài thi Các giám khảo Số phách (Bằng số) (Ký và ghi rõ họ tên) (Do Trưởng Ban chấm thi ghi) (Bằng chữ) Giám khảo 1: Giám khảo 2: A. LISTENING PART I. Listen to the dialogue on the phone between and a man and a girl named Juliet and fill in the form. You are allowed to listen TWICE. Give your answers in the numbered spaces. Name: Juliet A. Eastman Age: (1) …………………………………… Hair color: (2) …………………………………… Eye color: (3) …………………………………… Height: (4) …………………………………… Occupation: (5) …………………………………… ………………………………………………………….. Likes: going out and having fun, sports, (6) ……………………………………… and (7) …………………………………………… Wants to meet someone who : (8) …………………………………………………, likes same (9)………………………………………………. and (10) ……………………………………… Part II. You are going to hear an expert talk about sleeping and dreaming. Listen and write True (T) or False (F) for each sentence. You are allowed to listen TWICE. Your answers True (T) 1. Women sleep more than men. 2. A sound sleeper moves less than a light sleeper. 3. Most people need 9 hours of sleep a night. 4. Reading in bed helps you sleep. 1 False (F) 5. Some people don’t dream at all. 6. The average person has about four dreams a night. 7. Not everyone can remember his or her dreams. 8. Eating before bed can give you nightmares. PART III. You are going to listen to a talk about Margaret Mead. Listen and choose the best answer A, B, C or D for each question. You are allowed to listen TWICE. 1. What was Margaret Mead’s job? A. a photographer B. a biologist C. an anthropologist D. a journalist 2. What was Margaret Mead’s main interest? A. taking photographs B. exploring new places C. how children were looked after. D. living in pour areas. 3. When did Margaret Mead go to Samoa? A. in 1901. B. in the 1920s. C. in 1938. D. in 1978. 4. Who did she interview in her first trip to Samoa? A. girls between 9 and 20 years old. B. boys and girls between 9 and 20 years old. C. women over 20 years old. D. men and women over 20 years old. 5. What was the title of Margaret Mead’s book? A. The pacific Islands. B. Teenagers around the World. C. Growing Up in New Guinea. D. Coming of Age in Samoa. 6. What was the main reason why Margaret Mead took photos? A. She liked photography. B. Cameras were not very common at that time. C. Her husband liked photos. D. It was the best way to share what she learned. 7. What is the main topic of the listening passage? A. Margaret Mead went to college in New York. B. Margaret Mead did research on the role of culture. C. Margaret Mead took photographs and wrote books. D. Margaret Mead was born in Philadelphia. Your answers 1. 2. 3. 4. 5. 6. 7. B. LEXICO – GRAMMAR PART IV. Choose the answer A, B, C or D which best fits the space in each of the following sentences. 1. _________ saying was so important that I asked everyone to stop talking and listen. A. What the woman was B. That the woman was C. The woman was D. When was the woman 2. -“Do you mind if I take a seat?” - “_____________.” A. Yes, I don’t mind B. No, do as you please C. No I mind D. Yes, do as you please 3. As the two teams left the football ground, the 100,000 _________ gave them a standing ovation. A. bystanders B. spectators C. viewers D. audiences 4 My parents lent me the money. _________, I couldn’t have afforded the trip. A. However B. Therefore C. Only if D. Otherwise 5. It is interesting to take _________ a new hobby such as collecting stamps or going fishing. A. over B. on C. in D. up 6. Jack made me _________ him next week. A. promise calling B. to promise calling C. to promise to call D. promise to call 2 7. “I passed the TOEFL test, Mom.” - “ _________.” A. All right B. Thank you C. Well done D. Good luck 8. The bad weather caused serious damage to the crop. If only it _______ warmer. A. was B. were C. has been D. had been 9. - “Eric is really upset about losing his job.” - “ Well, ____once myself, I can understand.” A. Having been fired B. Fired C. Having fired D. Being fired 10. ________ you, I’d think twice about that decision. I could be a bad move. A. Were I B. Should I be C. If I am D. If I had been 11. The teacher asked a difficult question, but finally Ted _________ a good answer. A. put up with B. keep pace with C. made way for D. came up with 12. Not only ________ to speak to him, but she also vowed never to see him again. A. she refused B. did she refuse C. she did refuse D. when she refused 13. The judge ________ the pedestrian for the accident. A. accused B. charged C. caught D. blamed 14. She had to borrow her sister’s car because hers was _________. A. out of work B. out of order C. on duty D. off work 15. We should participate in the movement _________ to conserve the natural environment. A. to organize B. organizing C. which organized D. organized 16. His brother refuses to even listen to anyone else’s point of view. He is very_________. A. open-minded B. kind-hearted C. narrow-minded D. absent-minded 17. There is _________ in my bedroom. A. a square wooden old table B. an old square wooden table C. a wooden old square table D. an old wooden square table 18. “I am sorry. I broke the vase”. - “ _________.” A. OK. Go ahead B. Yes, certainly C. Don’t worry. Things break D. I’d rather not. 19. One’s fingerprints are _________ other person. A. different from B. different from any C. differ from any D. different from those of any 20. He is very happy because he passed his exam with __________ colours. A. flying B. failing C. imagining D. changing Your answers: 1. 2. 3. 4. 5. 6. 7. 8. 9. 10. 11. 12. 13. 14. 15. 16. 17. 18. 19. 20 PART V. From four underlined parts, choose the one that needs correction then correct it. For example : The teacher did not allow the students discussing the take-home exam with each other. discussing → to discuss 1. A Geiger counter is an electronic instrument is used to measure the presence and intensity of radiation. 2. A dolphin locates underwater objects in their path by making a series of clicking and whistling sounds. 3. In spite of its small size, Europe had a great impact on world history than other continents. 4. Before she moved here , Alene has been president of the organization for four years. 5. That Marta's been chosen as the most outstanding student on her campus make her parents very happy. 6. My cousin composes not only the music, but also sings the songs for the major Broadway musicals. 7. Our civilization is so commonplace to us that rarely we stop to think about its complexity. 8. Ever since the world began, nations have difficulty in keeping peace with their neighbors. 3 9. Those of us who have a family history of heart disease should do yearly appointments with our doctors. 10. If one had thought about the alternatives, he would not have chosen such difficult a topic for a term paper. Your answers Mistake Correction 1. 2. 3. 4. 5. 6. 7. 8. 9. 10. PART VI. Fill in each space in the following sentences with the most suitable prepositions. 1. I'm afraid Tom's _______ work. But Jack's in. Would you like to speak to him? 2. Have you been to the theatre recently? ~ Yes, I was _______ the Old Vie last night. 3. At first I found the work very tiring, but _______ a few weeks I got used to it. 4. _______ the daytime the streets are crowded but at night they are quite deserted. 5. I saw Tom at the bus stop this morning but couldn't speak to him because we were standing _____ a queue. 6. He is always in a hurry. He drives _______ a tremendous speed. 7. Write ________ ink and put your name on the top of the page. 8. The man with the pipe and red hair is the brother of the girl ________ blue. 9. He sits at his desk all day with his head in his hands. It gets ________ my nerves. 10. The children hastily changed _______ bathing things and jumped into the river with shouts of delight. Your answers: 1. 2. 3. 4. 5. 6. 7. 8. 9. 10. PART VII. Give the correct form of the word in bracket to complete the passage. Your answers: The __1__ (say) “never judge a book by its cover” could not be more true for Ridiculous Rules by Marjorie Allen. The cover is completely blank, whereas 1. ............................................ the book is crammed full of wonderful examples and anecdotes. Allen is an 2. ............................................ __2__ (speak) critic of what is taught to native and non-native speakers of 3. ............................................ English, and has issued a __3__ (declare) of war against textbooks and style books which tell lies. 4............................................. Take the ridiculous and __4__ (mean) rule of never ending a sentence with a preposition. The lovely - if famous – story goes, that Winston Churchill, 5............................................. well-known for his numerous __5__ (write) as well as for being British Prime Minister during the Second World War, received a manuscript back from an 6. ............................................ ignorant __6__ (edit), who had told him rather rudely that he had to __7__ 7. ............................................ 4 (phrase) a sentence which ended with a preposition. Churchill responded by making the simple yet forceful __8__ (state) in the margin: “This is an 8. ............................................ impertinence up with which I will not put.” – the __9__ (imply) being that not 9. ............................................ to end a sentence with a preposition often sounds ridiculous in English, Sadly, 10. ............................................ Allen informs us that the story is probably mere __10__ (hear), and that Churchill may have actually only written “rubbish!” in the margin. C. READING PART VIII. Read the passage and choose the best answer A, B, C, or D to each of the questions Large animals that inhabit the desert have evolved a number of adaptations for reducing the effects of extreme heat. One adaptation is to be light in color, and to reflect rather than absorb the Sun's rays. Desert mammals also depart from the normal mammalian practice of maintaining a constant body temperature. Instead of trying to keep down the body temperature deep inside the body, which would involve the expenditure of water and energy, desert mammals allow their temperatures to rise to what would normally be fever height, and temperatures as high as 46 degrees Celsius have been measured in Grant's gazelles. The overheated body then cools down during the cold desert night, and indeed the temperature may fall unusually low by dawn, as low as 34 degrees Celsius in the camel. This is an advantage since the heat of the first few hours of daylight is absorbed in warming up the body, and an excessive buildup of heat does not begin until well into the day. Another strategy of large desert animals is to tolerate the loss of body water to a point that would be fatal for non-adapted animals. The camel can lose up to 30 percent of its body weight as water without harm to itself, whereas human beings die after losing only 12 to 13 percent of their body weight. An equally important adaptation is the ability to replenish this water loss at one drink. Desert animals can drink prodigious volumes in a short time, and camels have been known to imbibe over 100 liters in a few minutes. A very dehydrated person, on the other hand, cannot drink enough water to dehydrate at one session, because the human stomach is not sufficiently big and because a too rapid dilution of the body fluids causes death from water intoxication. The tolerance of water loss is of obvious advantage in the desert, as animals do not have to remain near a water hole but can obtain food from grazing sparse and far-flung pastures. Desert-adapted mammals have the further ability to feed normally when extremely dehydrated, it is a common experience in people that appetite is lost even under conditions of moderate thirst. 1. What is the main topic of the passage? A. Weather variations in the desert B. Adaptations of desert animals C. Diseased of desert animals D. Human use of desert animals. 2. According to the passage, why is light coloring an advantage to large desert animals? A. It helps them hide from predators. B. It does not absorb sunlight as much as dark colors. C. It helps them see their young at night D. It keeps them cool at night. 3. The word "maintaining" is closest in meaning to ___________. A. measuring B. inheriting C. preserving D. delaying 4. The author uses of Grant's gazelle as an example of ___________. A. an animal with a low average temperature B. an animal that is not as well adapted as the camel C. a desert animal that can withstand high body temperatures D. a desert animal with a constant body temperature 5. When is the internal temperature of a large desert mammal lower? A. Just before sunrise B. In the middle of the day C. Just after sunset D. Just after drinking 5 6. The word "tolerate" is closest in meaning to ___________. A. endure B. replace C. compensate D. reduce 7. What causes water intoxication? A. Drinking too much water very quickly B. Drinking polluted water C. Bacteria in water D. Lack of water. 8. What does the author imply about desert-adapted mammals? A. They do not need to eat much food. B. They can eat large quantities quickly C. They easily lose their appetites. D. They can travel long distances looking for food. 9. Why does the author mention humans in the second paragraph? A. To show how they use camels. B. To contrast them to desert mammals. C. To give instructions about desert survival. D. To show how they have adapted to desert life. 10. Which of the following is NOT mentioned as an adaptation of large desert animals? A. Variation in body temperatures B. Eating while dehydrated C. Drinking water quickly D. Being active at night. Your answers: 1. 2. 3. 4. 5. 6. 7. 8. 9. 10. PART IX. Read the text below and decide which answer A, B, C or D best fits each space. When you read something in a foreign language, you frequently (1)______ across words you do not fully understand. Sometimes you check the meaning in a dictionary and sometimes you (2)______. The strategy you adopt depends very much upon the (3)______ of accuracy you require and the time at your disposal. If you are the sort of person who tends to turn to the dictionary frequently, it is worth remembering that every dictionary has its (4)______. Each definition is only an approximation and one builds up an accurate picture of the meaning of a word only after meeting it in a (5)______ of contexts. It is also important to recognize the special dangers of dictionaries that translate from English into your native language and vice versa. If you must use a dictionary, it is usually far safer to (6)______ an English-English dictionary. In most exams you are not permitted to use a dictionary. (7)______ you are allowed to use one, it is very time-consuming to look up words, and time in exams is usually limited. You are, (8)______ , forced to guess the meaning of unfamiliar words. When you find unknown words in an exam text, it is very easy to panic. However, if you develop efficient techniques for guessing the meaning, you will (9)______ a number of possible problems and help yourself to understand far more of the text than you at first thought likely. Two strategies which may help you guess the meaning of a word are: using contextual clues, both within the sentence and outside, and making use of clues (10)______ from the formation of the word. 1. 2. 3. 4. 5. 6. 7. 8. 9. 10. A. put A. look A. extent A. limitations A. multiple A. survey A. or else A. so A. surpass A. derived B. drop B. guess B. range B. values B. variety B. consult B. Provided B. therefore B. get over B. extracted C. see C. examine C. degree C. advantages C. variation C. refer C. Although C. completely C. go over C. coming D. come D. inspect D. level D. entry D. diversity D. inquire D. Even if D. so that D. overcome D. originated Your answers: 1. 2. 3. 4. 5. 6. 7. 8. 9. 10. 6 D. WRITING PART X. Complete the second sentence so that it has similar meaning to the first one. 1. This will be my student's first performance in Canada. → This will be the first time …………………………………………………………………………... 2. This course will take us six months to complete. → In six months time ………………………………………………………………………………….. 3. The number of people who understand his ideas exceed his expectations. → More people ………………………………………………………………………………………… 4. She'll have to make her presentation at the end of his speech. → The moment he …………………………………………………………………………………….. 5. Sharon will finish her exams. Then she will have more free time. → Once ………………………………………………………………………………………………… 6. Both Mary and Peter prefer jazz to classical music. → Neither……………………………………………………………………………………………… 7. They repaired my car at the garage in town. → I............................................................................................................................................................ 8. This is the last time I will speak to you. → I........................................................................................................................................................... 9. I prefer staying in to going out. → I'd rather.............................................................................................................................................. 10. They passed the driving test because of the easy questions. → If …………………………………………………………………………………………………… Your answers: 1. ................................................................................................................................................................................................... 2. ................................................................................................................................................................................................... 3. ................................................................................................................................................................................................... 4. ................................................................................................................................................................................................... 5. ................................................................................................................................................................................................... 6. ................................................................................................................................................................................................... 7. ................................................................................................................................................................................................... 8. ................................................................................................................................................................................................... 9. .................................................................................................................................................................................................. 10. .................................................................................................................................................................................................. PART XI. ESSAY WRITING Some people say that cell phones have improved modern life. Others believe that cell phones have caused many problems to people. What is your opinion? In about 250 words, write an essay to assert your point of view on this problem. ………………………………………………………………………………………………… ……… 7 ………………………………………………………………………………………………… ……… ………………………………………………………………………………………………… ……… ………………………………………………………………………………………………… ……… ………………………………………………………………………………………………… ……… ………………………………………………………………………………………………… ……… ………………………………………………………………………………………………… ……… ………………………………………………………………………………………………… ……… ………………………………………………………………………………………………… ……… ………………………………………………………………………………………………… ……… ………………………………………………………………………………………………… ……… ………………………………………………………………………………………………… ……… ………………………………………………………………………………………………… ……… ………………………………………………………………………………………………… ……… ………………………………………………………………………………………………… ……… 8 ………………………………………………………………………………………………… ……… ………………………………………………………………………………………………… ……… ………………………………………………………………………………………………… ……… ………………………………………………………………………………………………… ……… ………………………………………………………………………………………………… ……… ………………………………………………………………………………………………… ……… ………………………………………………………………………………………………… ……… ………………………………………………………………………………………………… ……… ………………………………………………………………………………………………… ……… ………………………………………………………………………………………………… ……… ………………………………………………………………………………………………… ……… ………………………………………………………………………………………………… ……… ………………………………………………………………………………………………… ……… ………………………………………………………………………………………………… ………………………………………………………………………………………………… ……………… 9 ………………………………………………………………………………………………… ……… THE END SỞ GIÁO DỤC VÀ ĐÀO TẠO HÀ TĨNH ĐÁP ÁN KỲ THI CHỌN HỌC SINH GIỎI TỈNH LỚP 10 THPT NĂM HỌC 2012 - 2013 Môn thi: TIẾNG ANH Tổng: 20 điểm, cụ thể như sau: A. LISTENING – 3 ĐIỂM PART I: 1,5 điểm = 0,15/ 1 câu đúng 1. twenty-three/ 23 2. blonde / blond 3. blue 4. average 5. computer programmer 6. music 7.( watching ) TV 8. outgoing 9. music/ sports 10. sports/ music PART II: 0,8 điểm = 0,1/ 1 câu đúng 1. F 2. F 3. F 4. T 5. F 6. T 7. T 8.F PART III: 0,7 điểm = 0,1/ 1 câu đúng 1. C 2. C 3. B 4. A 5. C 6. D 7. B B. LEXICO – GRAMMAR – 8 ĐIỂM PART IV: 4 điểm = 0,2 / 1 câu đúng 1. A 2. B 3. B 4. D 5. D 6. D 7. C 8. D 9. A 10.A 11. D 12. B 13. D 14. B 15. D 16. C 17. B 18. C 19. D 20. A PART V: 2 điểm = 0,2 / 1 câu đúng 1. is used → used 2. their path → its path 3. great impact → greater impact 4. has been → had been 5. make → makes/made 6. composes not only → not only composes 7. rarely we stop → rarely do we stop 8. have difficulty → have had difficulty 9. do yearly → make yearly 10. such difficult → so difficult PART VI: 1 điểm = 0,1 / 1 câu đúng 1. at 2. at 3. for 4. in 5. in 6. at 7. in 8. in 9. on 10. into 10 PART VII: 1 điểm = 0,1 / 1 câu đúng 1. saying 2. outspoken 3. declaration 4. meaningless 5. writings 6. editor 7. rephrase 8. statement 9. implication 10. hearsay C. READING – 4 ĐIỂM PART VIII: 2 điểm = 0,2 / 1 câu đúng 1. B 2. B 3. C 4. C 5. A 6. A 7. A 8. D 9. B 10. D PART IX: 2 điểm = 0,2 / 1 câu đúng 1. D 2. B 3. C 4. C 5. B 6. B 7. D 8. B 9. D 10. A D. WRITING – 5 ĐIỂM Part IX: 2 điểm = 0,2 / 1 câu đúng my student has performed in Canada. 1. This will be the first time 2. In six months time we will have completed this course. 3. More people understand him/his ideas than he expected./ has expected/ expects. 4. The moment he finishes (his speech), she'll have to make her presentation. 5. Once Sharon finishes/has finished her exams, she will have more free time 6. Neither Mary nor Peter prefers classical music to jazz. 7. I had my car repaired at the garage in town. 8. I will not/never speak to you (again). 9. I'd rather stay in than go out. 10. If my student has given the performance in Canada. the questions hadn’t been easy, they wouldn’t have passed the driving test. the questions had been (more) difficult, they would have failed the driving test. It hadn’t been for the easy questions, Part X: 3 điểm - Nội dung (content): 1.5 điểm - Từ vựng (vocabulary): 0.5 điểm - Ngữ pháp (grammar): 0.5 điểm - Tính mạch lạc và trôi chảy (coherence and cohesion) + độ dài (length): 0.5 điểm 11 Së gi¸o dôc - ®µo t¹o hµ tÜnh Kú thi CHỌN HỌC SINH GIỎI TỈNH líp 10 n¨m häc 2010-2011 ®Ò chÝnh thøc M«n thi: TiÕng Anh Thêi gian lµm bµi: 150 phót (kh«ng kÓ thêi gian giao ®Ò) (đề thi có 05 trang) Lưu ý: Thí sinh chỉ làm bài vào tờ giấy thi. Không được sử dụng bất kì loại tài liệu nào. I. Choose the word whose underlined part is pronounced differently from that of the others. 1. A. nothing B. purpose C. welcome D. compare 2. A. measles B. tease C. please D. measure 3. A. especially B. weather C. exercise D. bench 4. A. thus B. with C. smooth D. maths 5. A. naked B. practiced C. beloved D. wicked 6. A. encourage B. problem C. excellent D. possible 7. A. curiosity B. cure C. business D. during 8. A. many B. habit C. apple D. happy 9. A. bear B. hear C. ear D. clear 10. A. young B. cousin C. couple D. thorough II. Choose the most suitable word or phrase A, B, C or D to complete each sentence. 1. In the hotel lobby the detective caught __________of the man he had been hired to follow. A. glance B. view C. vision D. sight 2. ‘Go on, finish the food. It needs __________ up because it won’t stay fresh until tomorrow. A. eat B. eating C. to eat D. eaten 3. Before he left on his trip to America, the young man promised his parents he _____them regularly. A. is writing B. will write C. should write D. would write 4. Of course I didn’t break it ____________ purpose- it was an accident! A. by B. with C. from D. on 5. Andrew couldn’t __________ himself laughing at the expression on Maggie’s face. A. help B. stop C. escape D. avoid 6. __________ young, chimpanzees are easily trained. A. When are B. When C. They are D. When they 7. “What’s your proposal?” “I propose that the meeting __________” A. is postponing B. be postponed C. to be postponed D. postpones 8. Most of the courses at the banquet were completely consumed, but there _____ food still remaining. A. were few B. was little C. were a few D. was a little 9. The four-storey house __________ on that hill is still new. A. be built B. building C. built D. being building 10. Nam Cao devoted most of his time __________ A. to having written B. to write C. to writing D. to have written 11. I don’t think he will join us, ___________? A. doesn’t he B. won’t he C. will he D. don’t I 12. The young generation must contribute to ____________ the country. A. building of B. the building C. the building of D. build 13. I cannot help feeling anxious __________ the exam results. A. of B. with C. about D. for 1 14. In some vocational schools, the training quality is worse than ___________ used to be. A. they B. it C. them D. that 15. When Elvis Presley died, her daughter came ___________ a fortune. A. into B. up with C. by D. across III. The following passage contains ten errors. Find and correct them. Things started to go wrongly as soon as we got to the hotel. We were all completely exhausting after our long journey and looking forward to a shower and a rest. However, we found that our room was not ready, which was very annoyed, although the manager was extremely apologising. While we were waiting, we asked about the excursions to places of an interest which we have read about in the brochure. Imagine how we felt when we were told they had all been cancelled! Apparently, the person responsible of organizing them had left suddenly and had not been replaced. Then Sally saw a notice pinning to the door of the restaurant, said it was closed for redecoration, and Peter discovered that the swimming pool was empty. When we eventually got to our room we were horrified to find that it was at the back of the hotel, and we had a view of a car park, that seemed to be used as a rubbish dump. We seriously began to wonder whether or not to stay. IV. Read the text below, using the word given in brackets to form a word that fits in the space. Congratulations to all involved with the school (__1__ produce) of The Woman Next Door. The (__2__ advertise) was carried out by the Art Department, and the posters were very (__3__ imagine). We certainly have some very (__4__ art) students in our school! Many people helped with building and painting the (__5__ scene) and the play was written by the English Department, who managed to create an (__6__ amuse) story, with excellent songs. The music was written by Sue Porter, who also (__7__ company) the singers on the piano. Everyone enjoyed a thoroughly (__8__ entertain) evening, and there was a long round of (__9__ applaud) at the end. Jim Barrett gave a brilliant performance as Sergeant Moss, and Liz Aiken was a (__10__delight) Mrs. Jump. Well-done everyone! V. For each question, write one word which can be used in all three sentences For example: I was so tired that I couldn’t even think ___________. I will come __________ to the point – you are fired. It’s time to set the record __________ about what really happened that night. → STRAIGHT I was just _______ to go home, but that’s okay. 1. It’s _______ time you got home – We’ve been worried sick! We’re definitely going to have to do something _______ increasing our market share. 2. I should get a reply from them any ________ now. It will only take them a ________ to update the website. Now is the __________ of truth! 3. We’re definitely living in the ___________ of information. This particular wine doesn’t really mature with __________. It’s time you started acting your __________, young man! 4. I do hope we’re going to arrive _________ time. How anyone could work in that office for years _________ end I just don’t know. 2 Just bet me that Doug was going to get the sack so I said: “You’re ________” 5. To my ____________, the boss said that I could have an extra day off work. Martin always takes great ___________ in talking to anyone about fishing. When Erin finally started to move downhill on the skis for the first time, she laughed in _____. 6. The local council should start a building program to meet the ______ for leisure activity. The Major said he had no _________ of advice from people who didn’t know the situation. This area is badly in ___________ of financial investment. 7. Oh, tell Carrie that I send my ________ when you see her. When my mum and dad met, my mum says it was ________ at first sight. My _____ for travelling comes from the fact that we moved around a lot. 8. We don’t go out during the week as a ________, but we’ll make an exception tonight. People must be made to respect the _________ of law, or we’ll have anarchy. A good ______ of thumb is that you should allow 15 minutes per exercise in the exam. 9. Did you hear that the guy who lives in the flat upstairs has been _______ with burglary? I couldn’t believe they ________ me three euros for a bottle of water! The children ran out of the school gate and ________ down the hill. 10. The head teacher punished Aaron severely in order to make an _________ of him. Why can’t you follow your brother’s ________ and go to university. Give me one _________ of a place round here where young people can go in the evenings. VI. Read the text below and decide which answer A, B, C, or D, best fits each space. Everyone wants to reduce pollution. But the pollution problem is (1)______ complicated as it is serious. It is complicated (2)______ much pollution is caused by things that benefit people. (3)______, exhaust from automobiles causes a large percentage of air pollution. But the automobile (4)______ transportation for millions of people. Factories discharge much (5)______ the material that pollutes the air and water, but factories give employment to a large number of people. Thus, to end (6)_______ greatly reduce pollution immediately, people would have to ( 7)______ using many things that benefit them. Most of the people do not want to do that, of course. But pollution can be (8)______ reduced in several ways. Scientists and engineers can work to find ways to lessen the (9)______ of pollution that such things as automobiles and factories cause. Governments can pass and enforce laws that (10)______ businesses and traffic to stop, or to cut down on certain polluting activities. 1. A. as B. more C. less D. like 2. A. so B. while C. though D. because 3. A. Specific B. For example C. Such as D. Like 4. A. takes B. affords C. carries D. provides 5. A. about B. for C. of D. with 6. A. or B. and C. as well D. then 7. A. start B. continue C. stop D. go on 8. A. carefully B. unexpectedly C. gradually D. little 9. A. way B. figure C. number D. amount 10. A. forbid B. prevent C. request D. require 3 VII. Read the passage and choose the correct answer A, B, C, or D for each question. Bill Jarvis took over our village news agency at time of life when most of us only want to relax. He just thought he would like something but not too much to do, and the news agency was readymade. The business produced little enough for him, but then Bill was a chap who only wanted the simplicity and order and regularity of the job. He had been a long-serving sailor, and all his life had done everything by the clock. Every day he opened his shop at six a.m to catch the early trade; the papers arrived on his doorstep before that. Many of Bill's customers were city workers, and the shop was convenient for the station. Business was tailing off by ten o'clock, so at eleven sharp Bill closed for lunch. It was hard luck on anybody who wanted a paper or magazine in the afternoon, for most likely Bill would be down on the river bank, fishing, and his nearest competitor was five kilometres away. Some time in the afternoon, the evening papers landed on the door-mat, and at four o'clock Bill reopened. The evening rush lasted till seven, and it was worthwhile. He lived in a flat above the shop, alone. Except in very bad weather, you always knew where to find him in the afternoons, as I have said. Once, on a sunny afternoon, I walked home along the river bank from a shopping trip to the village. By my watch it was three minutes past four, so I was astonished to see Bill sitting there on his little chair with a line in the water. He had no luck, I could see, but he was making no effort to move. "What's wrong, Bill?" I called out from the path. For answer, he put a hand inside his jacket and took out a big, golden object. For a moment I had no idea what it could be, and then it suddenly went off with a noise like a fire engine. Stopping the bell, Bill held the thing up and called back: "Ten to four, you see, and this is dead right." He stood up then and began to wind in his line. I had never known anyone carry a brass alarm clock round with him or her before. 1. Bill Jarvis became a newsagent when ________ A. He needed the money. B. He was quite an old man. C. He decided to take things easy. D. He gave up clock repairing. 2. What does the passage tell us about the news agency? A. It was an easy job with fixed hours. B. It was a very profitable business C. It was opened specially for Bill Jarvis D. It belonged to the railway and was part of the station. 3. Why did Bill open the shop so early in the day? A. He liked to do as much as possible before he went to work. B. Bill was never sure of the time. C. The shop had to be open when the morning papers came D. It was then that he did a lot of business. 4. We understand from the passage that the shop closed for lunch ________ A. At eleven o'clock more or less. B. At exactly eleven o'clock. C. Before eleven o'clock. D. Always after eleven o'clock. 5. You might say "hard luck" to someone who ________ A. has just heard some very good news. B. puts great effort into whatever he or she tries. C. is less fortunate than he or she ought to be. D. fails through his or her own fault entirely. 6. Why was the writer on the riverbank that afternoon? A. He was going to do some shopping in the village. B. He was fishing. 4 C. He was going to get the evening paper. D. He was on his way home from the village 7. Why was the writer surprised when he saw Bill Jarvis? A. He thought it was late for Bill to be still fishing. B. Bill had not caught anything, and that seemed strange. C. He thought Bill was ill, because he was not moving at all D. He was surprised because Bill stayed in his flat in the afternoons. 8. From the information given in the passage, who- or what - do you think was wrong? A. The bell was; it must have gone off at the wrong time. B. The writer's watch was fast. C. Bill was; he had dropped off to sleep D. Bill's clock was wrong; it was very old. 9. All of the following are true about Bill Jarvis EXCEPT __________ A. he lived alone B. He had ever worked as a sailor C. He was a newspaper man D. Fishing was his past time 10. What did Bill Jarvis often bring with him when he went fishing? A. a clock B. a gold C. a newspaper D. a fire engine VIII. Complete the second sentence so that it has a similar meaning to the first sentence. Use the word given and other words to complete each sentence. 1. Mr. Holton only operated because he knew the rumour was malignant → Mr. Holton wouldn't.............................................................................................................. 2. All that stood between John and a gold medal was Jim's greater speed → But for ................................................................................................................................. 3. Immediately after their arrival, things went wrong. → No sooner…………………………………………………………………………………. 4. He was so tired that he fell asleep before the end of the film → He was too............................................................................................................................. 5. Hearing that an earthquake had occurred was a great shock to us. → We were .............................................................................................................................. 6. The decorators have finished our first floor. → We have ............................................................................................................................. 7. Her hobby is one thing that she does not intend to give up. → She has ............................................................................................................................... 8. That is the best meal I have ever eaten. → I have never ............................. ............................... ............................... ........................... 9. Home computers used to be much more expensive. → Home computers aren’t................... ............................... ............................... ................... 10. It was a mistake of mine to park outside the police station. → I shouldn’t................... ...................................................................................................... IX. Writing: Write an essay of about 120 – 150 words to answer the following question: “What makes a good friend? 5 Së gi¸o dôc - ®µo t¹o hµ tÜnh H−íng dÉn chÊm m«n tiÕng anh 10 ®¸p ¸n vµ biÓu ®iÓm Tổng: 20 điểm, cụ thể như sau: CÂU I: (2 điểm) - Mỗi câu đúng được 0,2 điểm: 1. A 2. D 3. A 4. D 5. B 6. A 7. C 8. A 9. A 10. D CÂU II: (3 điểm) - Mỗi câu đúng được 0,2điểm: 1. D 2. B 3. D 4. D 5. B 6. B 7. B 8. D 9. C 10 C 11. C 12. C 13. C 14. B 15. A III: (2 điểm) - Mỗi câu đúng được 0,2 điểm: 1. PRODUCTION 6. AMUSING 2. ADVERTISING 7. ACCOMPANIED 3. IMAGINATIVE 8. ENTERTAINING 4. ARTISTIC 9. APPLAUSE 5. SCENERY 10. delightful CÂU IV: (2 điểm) - Mỗi câu đúng được 0,2 điểm: 1. A 2. D 3. B 4. D 5. C 6. A 7. C 8. C 9. D 10. D CÂU V: (1 điểm) - Mỗi câu đúng được 0,1 điểm: 1. about 2. moment 3. age 4. on 5. delight 6. need 7. love 8. rule 9. charged 10. example CÂU VI: (2 điểm) - Mỗi câu đúng được 0,2 điểm: 1. C 2. A 3. D 4. C 5. B 6. D 7. A 8. C 9. B 10. A 6 CÂU VII: (3 điểm) - Mỗi câu đúng được 0,3 điểm: 1. → Mr. Holton wouldn't have operated if he hadn’t known (that) the rumour was malignant 2. → But for Jim's greater speed, John would have won a gold medal 3. → No sooner had they arrived than things went wrong 4. → He was too tired to see/watch the end of the film 5. → We were (greatly/very) shocked to hear/when we heard that an earthquake had occurred 6. → We have had our first floor decorated (by the decorators) 7. → She has no intention of giving up her hobby. 8. → I have never eaten such a good meal as that/a better meal than that 9. → Home computers aren’t as expensive as they used to 10. → I shouldn’t have parked outside the police station CÂU VIII: (2 điểm) - Mỗi câu đúng được 0,2 điểm: 1. wrongly → wrong 6. have read → had read 2. exhausting → exhausted 7. of → for 3. annoyed → annoying 8. pinning → pinned 4. apologising → apologetic 9. said → saying 5. places of an interest → places of interest 10. that → which CÂU IX: (3 điểm) Yêu cầu: + Viết đúng bố cục của một bài luận. + Trình bày rõ ràng, sạch sẽ. + Không phạm nhiều lỗi về chính tả và ngữ pháp 7 UBND THÀNH PHỐ ĐÀ NẴNG SỞ GIÁO DỤC VÀ ĐÀO TẠO ************************************* ĐỀ CHÍNH THỨC KỲ THI CHỌN HỌC SINH GIỎI LỚP 10 MÔN TIẾNG ANH Thời gian làm bài: 150 phút Họ tên và chữ ký giám khảo Giám khảo 1: Số phách do Chủ tịch Hội đồng chấm thi ghi Điểm Bằng số Bằng chữ Giám khảo 2: Đề thi gồm 06 trang. Học sinh làm bài trên đề thi và ghi câu trả lời vào ô trống tương ứng cho mỗi câu hỏi. SECTION A: USE OF ENGLISH I. Select the best option to complete each of the following sentences. Write your answers (A, B, C or D) in the box provided. 1. 2. 3. 4. 5. 6. 7. 8. 9. 10. 11. 12. 13. 14. 15. 16. 17. I find her ___________ . She never stops talking. A. exhaustion B. exhaust C. exhausting D. exhausted Look at these two pieces of material I have just bought. Which do you like __________ ? A. better B. best C. more than D. most Have this one, __________ ? A. do you B. will you C. don’t you D. haven’t you Half the people in the office have __________ a strange illness. A. gone in for B. gone along with C. gone through with D. gone down with The Police are going to __________ him very carefully. A. check up on B. catch up on C. hold out for D. run away with Many people don’t use their computers to their full ____________. A. future B. expectation C. potential D. hope When our friends have bad fortune, we try to show___________. A. love B. sympathy C. affection D. pity She should have been here but she’s _____________ flu. A. come in for B. gone through with C. gone down with D. come up against The instructor on the course was hopeless __________ explaining things. A. in B. of C. at D. to I’m afraid there are no seat left. Every seat is ____________. A. reserved B. full C. served D. free You have to study harder to ____________ your classmates. A. keep in touch B. keep pace with C. keep out of D. keep up ____________ is the safeguarding and preservation of natural resources so that they can be used and enjoyed. A. conservative B. conservatism C. conservationist D. conservation It is imperative that this letter __________ immediately. A. were sent B. sent C. be sent D. send Two of the boys in the art class were doing self-portraits by looking at ________ in the mirror. A. themselves B. oneself C. each other D. one another The Tories won the election in 1979. A. came into effect B. came into power C. came to power D. came to the power While attempting to reach his home before the storm, ____________. A. Tom had an accident on his bike C. it happened that Tom’s bike broke down B. the storm caught Tom D. the bicycle of Tom broke down After several hours on that road, they became __________ to the fact that they would never reach the hotel by nightfall. Đề thi HSG lớp 10 1 18. 19. 20. A. dejected B. resigned He perceived a change in temperature. A. was afraid of B. was fond of In all __________, he’s already left. A. odds B. probability __________, I like the Rolling Stones A. No need to say B. Don’t need to say C. depressed D. disillusioned C. was interested in D. was aware of C. certainty D. possibilities C. Needless saying D. Needless to say Answers 1. 11. 2. 12. 3. 13. 4. 14. 5. 15. 6. 16. 7. 17. 8. 18. 9. 19. 10. 20. II. Use the right form of the word given in CAPITALS to complete the sentences. Write your answers in the box provided. 1. He has known Tom for a long time and valued his __________ . (FRIEND) 2. After all he has done for us, it would be very ________ of us if we didn’t do this for him now. (HELP) 3. The storm caused terrible _________ along the coast. (DESTROY) 4. __________ is one of the qualities required of a social worker. (FLEXIBLE) 5. The new leisure centre doesn’t quite come up to my __________. (EXPECT) 6. The floor of the attic needs _________ if we’re going to make it into a bedroom. (STRONG) 7. Is it possible to __________ between a hobby and an interest? (DISTINCT) 8. This used to be a rural area but it has become ___________. (INDUSTRY) 9. I’m surprised at what you’ve done. I hope my confidence in you has not been ________. (PLACE) 10. This jewel is not simply valuable. It is __________. (VALUE) Answers 1. 6. 2. 7. 3. 8. 4. 9. 5. 10. III. Read the text and look carefully at each line. Tick (√) the lines which are correct and pick out the unnecessary words. There are two examples at the beginning (0 and 00). Write your answers on the right lines provided. CAMPING 0. You’ve finally arrived at your destination place and you’re looking for somewhere 00. to set up your tent. So what should you be looking for? You need to look for 1. relatively level ground, and, whatever you do, never pitch your tent on a slope 2. or you will find yourself rolling around out of your sleeping bag all night. 3. Don’t forget to check it that there is a water source nearby. Water is essential for 4. camping: you’ll need it for the drinking and cooking. You don’t want to have to 5. walk far with a twenty-litre container, do you? Find a suitable area for cooking. 6. Don’t cook in you tent. Locate a flat area away from some leaves, grass or 7. twigs, which can make fire. Another thing you must do is keep the campsite clean. 8. After meals, wash the whole dishes, and put the rubbish in an appropriate place. 9. Remember to follow the campsite rules, which they were made so that everyone 10. can be enjoy the site. Leave it as you would like to find it yourself. SECTION B: ____place_ _____√___ __________ __________ __________ __________ __________ __________ __________ __________ __________ __________ READING COMPREHENSION I. Read the text below and choose the best word or combination of words to fill in each blank. Write your answers (A, B, C or D) in the box provided. AUDIOBOOKS - BOOKS ON CASSETTE In the modern world, there is a wealth of leisure activities to choose from Entertainment industries (1) __________ for your leisure time. You can watch TV, listen to music, go to an art gallery or concert or, of course, read a book. Sometimes it seems that reading is (2) __________ because, (3) Đề thi HSG lớp 10 2 __________ you’re a fast reader, it can take a (4) __________ amount of time to finish a novel, for example. But in the modern world, time is something that can be in short supply. Book publishers haven’t been (5) __________ to realize this and are now selling a product which needn’t (6) __________ as much of your time but still tells you an excellent story. The new products is the audiobook - cassette recordings of shortened novels, often read by well-known personalities or the authors themselves. Audiobooks are relatively new but people are becoming more aware of them and sales are increasing (7) __________. One of the attractions of audiobooks is that they’re (8) __________ listening to the radio, only better. You can listen to what you want when you want, and you won’t ever miss anything. Much of their appeal (9) ________ in their flexibility. They allow you do to other things while you’re listening, such as driving or (10) __________ the housework. For some people, audio books can be a much more enjoyable way of gaining knowledge than reading. 1. 2. 3. 4. 5. 6. 7. 8. 9. 10. A. chase A. neglected A. in spite A. plentiful A. behind A. take up A. ever since then A. as A. stands A. doing Answers 1. 6. 2. 7. B. B. B. B. B. B. B. B. B. B. compete declined no matter broad slow fill out all the time same belongs running 3. 8. C. oppose C. lessened C regardless of C. considerable C. delayed C. go through C. up until now C. like C. bases C. making D. pursue D. disposed D. even if D. lasting D. overdue D. pass by D. from then on D. both D. lies D. cleaning 4. 9. 5. 10. II. Fill in each of the numbered blanks with ONE suitable word. Write your answers (A, B, C or D) in the box provided. About two hundred years ago man lived in greater harmony with his environment because industry was not much developed. Today the situation is quite (1) _________. People all over the world are worried about what is happening to the environment, because of modern industry and the need for more and more energy. Newspapers and magazines write (2) _________water pollution, air pollution and land pollution. Why is there so much (3) ________ about pollution. After all, people have been polluting the world around them for thousands and thousands of years. But in the past, there were not many people and (4) ________of room in the world so they could move to another place when their settlements became dirty. Now, however, many parts of the world are (5)_________, people live in big cities and much of our waste, especially waste from factories, electric (6) _______ stations, the chemical industry and heavy industry is very dangerous. Fish die in the lakes, rivers and seas; forest trees die (7)_______. Much of this dangerous waste goes into the air and is carried by the wind for great distances. The earth is (8) ________home . We must take care of it for ourselves and for the next generations. This means (9) _________our environment clean. The importance of this task is pointed out by ecologists, the scientists who study the (10)_______ between living things and the environment. However, each of us must do everything possible to keep the land, air and water clean. Answers 1. 6. 2. 7. 3. 8. 4. 9. 5. 10. III. Read the passage carefully and choose the best option for each sentence. Most of us lead unhealthy lives: we spend far too much time sitting down. If, in addition, we are careless about our diets, our bodies soon become flabby and our systems sluggish. Then the Đề thi HSG lớp 10 3 guilt feelings start : “I must go on a diet”, “ I must try to lose weight” “ I must get more fresh air and exercise”, “I must stop smoking”, “I must try to keep fit” . There are some aspects of our unhealthy lives that we cannot avoid. I’m thinking of such features of modern urban life as pollution, noise, rushed meals and stress. But keeping fit is a way to minimize the effects of these evils. The usual suggestion to a person who is looking for a way to keep fit is to take some sports. While it is true that every weekend you will find people playing football and hockey in the local park, they are outnumbered a hundred to one by the people who are simply watching them. It is an illusion to think that you will get fit by going to watch the football match every Saturday, unless you count the effort required to fight your way through the crowds to get to the best seats. For those who do not particularly enjoy competitive sports – and it is especially difficult to do so if you are not good at them – there are such solitary activities as cycling, walking and swimming. What often happens, though, is that you do them in such a leisurely way, so slowly, that is doubtful if you are doing yourself much good, apart from the fact that you have at least managed to get up out of your armchair. Of course you can be very thorough about exercise, even fanatical. Many sports shops now sell frightening pieces of apparatus, chest-expanders and other mysterious gadgets of shiny spring steel, which, according to the advertisements, will bring you up to an Olympic standard of fitness, provided that you follow a rigorous and regular programme of exercises. Such programmes generally involve long periods of time bending these curious bits of metal into improbable shapes. It all strikes me as utterly boring and also time-consuming. Somebody suggested recently that all such effort was pointless anyway because if you spend half an hour every day jogging round the local park, you will add to your life exactly the number of hours that you wasted doing the ‘jogging’ in the first place. The argument is false even if the facts are correct, but there is no doubt that exercise in itself can be boring. Even after you have found a routine for keeping in shape, through sport or gymnastics or isometrics, you are still only half way to good health, because, according to the experts, you must also master the art of complete metal and physical relaxation. Now, this does not mean snoozing in the armchair or going dancing ( which is a good form of exercise in itself ) . It has to do with deep breathing , emptying your mind of all thoughts, meditation, and so on. Yoga, as practiced in the West, is the most widely known and popular of the systems for achieving the necessary state of relaxation. Contrary to popular belief, you do not have to learn a lot of strange words or become a Buddhist in order to benefit from Yoga. It seems ironical, though, that as our lives have improved in a material sense, we have found it increasingly necessary to go back to forms of activity – physical effort on the one hand, and relaxation on the other – which were the natural way of life of our forefathers. 1. Unfitness is the result of ___________. A. lack of fresh air and exercise B. overeating, smoking and living in town C. not eating properly and not getting enough exercise D. not taking part in sports 2. Pollution, noise and stress are examples of ___________. A. causes of unfitness B. bad features of living in towns C. the things we must avoid if we are to stay healthy D. industrial life and work 3. Our reaction to being out of condition is to ___________. A. give up smoking and go on a diet B. start a programme of keep-fit exercises C. make resolutions to lead a healthier life D.take up a sport 4. A lot of people who go to the park at the weekend are ___________. A. football players B. football spectators C. keep-fit enthusiasts D. unfit or unhealthy 5. You cannot really enjoy competitive sports unless you are ___________. A. young enough to play them B. very good at them C. fit enough to play them D. a solitary kind of person 6. The reason we don’t get much out of solitary sports such as walking is that we do not _________. Đề thi HSG lớp 10 4 A. do them often enough B. have the encouragement of others C. take them seriously enough D. do them vigorously enough 7. The writer does not like keep-fit programmes because he thinks ___________. A. they take up too much time B. there are much easier ways of getting fit C. they are a waste of time D. the apparatus is very silly 8. To be healthy we must ___________. A. keep fit and active B. keep fit and learn to relax C. be active and practise Yoga D. be very careful about what we eat and drink 9. Many people believe that in order to practise Yoga ___________. A. you must learn a special vocabulary B. it is better to become a Buddhist C. you must learn to relax completely D. you must wear special clothing 10. Our forefathers were healthy because ___________. A. their way of life involved both exercise and relaxation B. they were careful to get plenty of fresh air and keep fit C. they lived in the country and spent time out of doors D. they had simple work to do and very little to worry about Answers 1. 2. 3. 4. 5. 6. 7. 8. 9. 10. SECTION D: WRITING I. Finish each sentence in such a way that it means exactly the same as the one printed before it. 1. His second attempt on the world record was successful He broke_____________________________________________________________________. 2. They were all arrested because of his incompetence. Had_________________________________________________________________________. 3. Please don’t smoke in the kitchen. I’d rather ____________________________________________________________________. 4. “That’s a lovely new dress, Jean”, said her mother. Jean’s mother complimented _____________________________________________________. 5. I don’t really like her even though I admire her achievements Much________________________________________________________________________. II. For each of the sentences, write a new sentence as similar as possible in meaning to the original sentence, using the word given in CAPITALS. This word must not be altered in any way. 1. I don’t think this record will ever be popular. (CATCH) _________________________________________________________________________. 2. A police car has just stopped outside. (UP) _________________________________________________________________________. 3. They didn’t punish Karen, only gave her a warning. (GOT) _________________________________________________________________________. 4. What are you trying to say? (AT) _________________________________________________________________________. 5. Graham stopped talking to deal with a customer. (OFF) _________________________________________________________________________. Đề thi HSG lớp 10 5 III. The Internet is now getting more and more popular all over the world. So, how important is it to your life? Write a paragraph of about 120 words to answer this question. ....................................................................................................................................................... ....................................................................................................................................................... ....................................................................................................................................................... ....................................................................................................................................................... ....................................................................................................................................................... ....................................................................................................................................................... ....................................................................................................................................................... ....................................................................................................................................................... ....................................................................................................................................................... ....................................................................................................................................................... ....................................................................................................................................................... ....................................................................................................................................................... ....................................................................................................................................................... ....................................................................................................................................................... ....................................................................................................................................................... The End Đề thi HSG lớp 10 6 THPT Chuyên Bắc Ninh MÔN THI: Anh KHỐI 10 Nội dung A. PHONETICS (10p) Question 1: Pick out the word whose underlined part is pronounced differently from those of the other words. Write your answers in the numbered blanks provided below (5p) B. canal C. many D. explanatory 1. A. random 2. A. good B. moon C. food D. balloon 3. A. provide B. product C. production D. procedure 4. A. kite B. bite C. Christian D. Christ 5. A. voltage B. voyage C. massage D. dosage Answers: 1. 2. 3. 4. 5. Question 2: Choose the word whose stress pattern is differently from those of the other words. Write your answers in the numbered blanks provided below (5p) 1. A. argumentative B. psychological C. contributory D. hypersensitive 2. A. atomic B. brilliant C. determined D. ambitious 3. A. trigonometry B. explanatory C. immediately D. democracy 4. A. photograph B. payroll C. accent D. regretful 5. A. majority B. ceremony C. astronomy D. investiture B. GRAMMAR & VOCABULARY (20p) Question 3: Give the correct tense/ form of the verbs in the brackets. Write your answers in the numbered blanks provided below (10p) 1. It’s time we (go). If we (not leave) now, we (miss) the train. 2. I know I (not write) to you before, but I (be) so busy recently that I (not have) time for writing letters. I (telephone) you instead, but I (forget) your number. 3. Why didn’t you tell me you could lend me the money? I (not borrow) it from the bank. 4. He is walking (hurry) to the place where his ex-wife, a famous naturalist now (lie) (bury) among the animals she (love) so much. 5. We will see Alice at the corner, she (wait) for us when we (arrive). 6. He resents (be) (tell) what (do). 7. Her mother (go) abroad last month, so it (not be) her you see at the theater last Sunday. Question 4: The passage below contains 10 errors. IDENTIFY and CORRECT them. (0) has been done as an example. Write your answers in the numbered blanks below (5p) After an absence in thirty years, I decided visiting my old school again. I had expected to find changes, but no a completely different building. As I walked up the school drive, I wondered for a moment if I had come to the right address. The grimy, red brick fortress with their tall windows that had looked up grimly on the playground and playing fields had swept away. In its place stood a bright, modern block risen from the ground on great concrete stilts. A huge expanse of glass extending across the face of the building, and in front, there was a well-kept lawn where previous there had been untidy 1 gravel yard. Answers: 0. in of 1. ................ 2. .................. 3. ................. 4. .................. 5. .................. -->............... --> ................. --> ............... -->................. --> ................ 6. ................ 7. ................. 8. ................ 9. .................. 10. ................ -->............... -->................ -->................ -->.................. -->................. Question 5: Fill in each blank with a suitable PREPOSITION or PARTICLE. Write your answers in the numbered blanks provided below the passage. (5p) 1. We’re all very obliged …………… you 2. When the wound has healed ....... you can remove the plaster. 3. Without a fridge, fresh food will go ...... very quickly. 4. I’m faithful …………… my principle. 5. I haven’t heard ........ Mandy since she wrote in July. 6. This service is free …………… charge. 7. They went ahead contrary ……………… my advice. 8. The weather was fine, and everyone was making ..... .. the coast. 9.We’re going ....... Tom’s car tomorrow. 10. Have a card................your sleeve. C. READING (20p) Question 6: Read the following passage and choose the best answer. Write your answers in the numbered blanks provided below the passage. (10p) In addition to the great ridges and volcanic chains, the oceans conceal another form of undersea mountains: the strange guyot, or flat-topped seamount. No marine geologist even suspected the existence of these isolated mountains until they were discovered by geologist Harry H. Hess in 1946. He was serving at the time as naval officer on a ship equipped with a fathometer. Hess named these truncated peaks for the nineteenth-century Swiss-born geologist Arnold Guyot, who had served on the faculty of Princeton University for thirty years. Since then, hundreds of guyots have been discovered in every ocean but the Arctic. Like offshore canyons, guyots present a challenge to oceanographic theory. They are believed to be extinct volcanoes.Their flat tops indicate that they once stood above or just below the surface, where the action of waves leveled off their peaks. Yet today, by definition, their summits are at least 600 feet below the surface, and some are as deep as 8,200 feet. Most lie between 3,200 feet and 6,500 feet. Their tops are not really flat but slope upward to a low pinnacle at the center. Dredging from the tops of guyots has recovered basalt and coral rubble, and that would be expected from the eroded tops of what were once islands. Some of this material is over 80 million years old. Geologists think the drowning of the guyots involved two processes: The great weight of the volcanic mountains depressed the sea floor beneath them, and the level of the sea rose a number of times, especially when the last Ice Age ended, some 8,000 to 11.000 years ago. 1. What is the author's main purpose in writing this passage? A. To trace the career of Arnold Guyot. B. To describe feature of the undersea world. C. To present the results of recent geologic research. D. To discuss underwater ridges and volcano chains 2. The word “conceal” is closest in meaning to which of the following? A. Contain B. Erode C. Hide D. Create 2 3. The passage implies that guyots were first detected by means of ... A. a fathometer B. computer analysic C. a deep-sea diving expedition D. research submarines 4. The author indicates that Arnold Guyot A. was Harry Hess's instructor B. invented the fathometer C. named the guyed after himself D. taught at Princeton University 5. What does the passage sav about the Arctic Ocean? A. The first guyot was discovered there. B. No guyots have ever been found there. C. There are more guyots there than in any other ocean. D. It is impossible that guyots were ever formed there. 6. The author states that offshore canyons and guyots have which of the following characteristics in common? A. Both are found on the ocean floor near continental shelves. B. Both present oceanographers with a mystery. C. Both were formed by volcanic activity. D. Both were, at one time, above the surface of the sea. 7. According to the passage, most guyots are found at a depth of ... A. less than 600 feet. B. between 600 and 3,200 feet. C. between 3,200 and 6,500 feet D. more than 8,200 feet 8. Which of the following is closest in meaning to the word “rubble”? A. Fragments B. Mixture C. Columns D. Core 9. According to the passage, which of the following two processes were involved in the submersion of guyots? A. Erosion and volcanic activity. B. The sinking of the sea floor and the rising of sea level. C. Mountain building and the action of ocean currents D. High tides and earthquakes 10. According to the passage, when did sea level significantly rise? A. In 1946 B. In the nineteenth century C. From 8,000 to 11,000 years D. 80 million years ago Question 7: Read the following passage and choose the best answer. Write your answers in the numbered blanks provided below the passage. (10p) As Christmas evolved in the United States, new customs were adopted and many old ones were reworked. The legend of Santa Claus, for example, had origins in Europe and was brought by Dutch settlers to New York in the early 18th century. Traditionally, Santa Claus - from the Dutch Sinter Klaas was depicted as a tall, dignified, religious figure riding a white horse through the air. Known as Saint Nicholas in Germany, he was usually accompanied by Black Peter, an elf who punished disobedient children. In North America he eventually developed into a fat, jolly old gentleman who had neither the religious attributes of Saint Nicholas nor the strict disciplinarian character of Black Peter. Santa’s transformation began in 1823, when a New York newspaper published the poem A Visit from Saint Nicholas, which Clement Clark Moore had written to amuse his daughter. The poem introduced many Americans to the story of a kindly saint who flew over housetops in a reindeer-drawn sleigh. Portraits and drawings of Santa Claus by American illustrator Thomas Nast further strengthened the legend during the second half of the 19th century. Living at the North Pole and assisted by elves, the modern Santa produced and delivered toys to all good children. By the late 19th century he had become such a prominent figure of American folklore that in 1897, when Virginia O’Hanlon wrote to the New York Sun newspaper asking if Santa was real, she received a direct answer: “Yes, Virginia, there is a 3 Santa Claus”. 1. Who brought the legend of Santa Claus to the USA according to the passage? A. Sinter Klaas B. Saint Nicholas C. A German D. Dutch settlers 2. Santa Claus was traditionally described as a .......... A. tall man who could walk through the air. B. fat, jolly, old man. C. religious figure. D. fat man riding a white horse. 3. Santa Claus in North America was depicted as ... A. a man with the strict disciplinarian character of Black Peter. B. a good old man with less religious character. C. one with religious attributes of Saint Nicholas. D. a jolly man on horseback. 4. Who was Black Peter? A. an elf accompanying Saint Nicholas. C. one of the disobedient children. B. an elf who rode a white horse. D. a popular traditional figure. 5. What word is closest in meaning to “attributes”? A. symbols of a person C. effects B. natural qualities D. outer appearance 6. Where did the legend of Santa Claus come from? A. the North Pole B. Europe C. North America D. the City of New York 7. 1823 was mentioned as a year when ... A. Clement Clark Moore wrote his first poem. B. Clement Clark Moore’s poem made him popular. C. Saint Nicholas visited New York. D. the image of Santa Claus was transformed. 8. According to Clement Clark Moore’s poem, A. Santa Claus had nothing different in appearance from the traditional one. B. Santa Claus had wings and could fly. C. Santa Claus liked poetry. D. Santa Claus was a kindly saint who flew over housetops in a sleigh. 9. The answer “Yes, Virginia, there is a Santa Claus” is an illustration for the fact that... A. the New York Sun was popular with children. B. Santa Claus was a prominent figure at that time. C. newspapers are unreliable. D. Virginia O’Hanlon was a reader of the New York Sun. 10. Which of the following statements is TRUE? A.Santa Claus was an imaginary old man created by artists based on traditional figures. B. Living in the North Pole, Santa Claus visited children at Christmas. C. Santa Claus was a real figure living in northern America. D. Santa Claus was a story based on Saint Nicholas and Black Peter. D. USE OF ENGLISH (20p) Question 8: Choose the word that best fits each of the blanks in the following passage. Write your 4 answers in the numbered blanks provided below the passage. (0) has been done as an example. (5p) Vietnamese generally shake hands when greeting and parting. Using both hands shows respect as does a (1) _____ bow of the head. In rural areas, elderly people do not extend their hand are greeted with a bow. Women are more (2) _____ to bow the head than to shake hands. Vietnamese names begin with the family name and are (3) _____ by a given name. For example, in the name Nguyen Van Due, Nguyen is the family name. People address (4) _____ by their given names, but add a title that indicates their perceived (5) _______ to the other person. These titles are family related rather than professional. Among colleagues, for example, the (6) _____ of the two might combine the given name with the title of Anh ("Older Brother"). A(n) (7)_____ greeting combined with the given name and title is Xin chao ("Hello."). Classifiers for gender and familiarity are also combined with the greeting. In formal meetings, business cards are sometimes (8) _____ on greeting. Vietnamese people have a strong (9) _____ of hospitality and feel embarrassed if they cannot show their guests full respect by preparing for their arrival. Therefore, it (10) _____ to visit someone without having been invited. Gifts are not required, but are appreciated. Flowers, incense, or tea may be proper gifts for the hosts. Hosts also appreciate a small gift for their children or elderly parents. 0 A. put B. shake C. clap D. applaud 1. A. light B. slight C. lightly D. lighted B. capable C. probable 2. A. possible D. likely B. chased C. followed D. taken 3. A. continued B. one other C. the other one D. one another 4. A. each one B. relationship C. relations D. relatives 5. A. relation B. most young C. youngest D. young 6. A. younger B. basic C. fundamentally 7. A. easy D. elementary B. transferred C. reversed 8. A. changed D. exchanged 9. A. sense B. sensation C. sensitivity D. sensible B. inaccurate C. inappropriate D. inexact 10 A. inactive . Question 9: Fill each blank with ONE suitable word. Write your answers in the numbered blanks provided below the passage. (0) has been done as an example. (10p) Cell Phones Cell phones have been popular in Japan (0. since) the early 1990s, but it was (1.)____ until 1999 that their use really took off. The age of cell phones has emerged, but with it come problems. Cell phones are used on buses and trains, in restaurants, and in all areas of (2.)_____. They cause problems when they (3.)_____ during meetings, concerts, weddings, or even funerals. What's more, people speak loudly in public, and students read and text messages during lessons. (4.) ______ seriously, when a cell phone is used near a person (5.)_____ a pacemaker to fegulate his heartbeat, its radio waves may interfere with the functioning of the pacemaker. Now, something is being done to solve these (6.)_______. In many places, new technology is being used to block cell phone calls. Airline (7.) ______ are requested to stop using cell phones while on board. Concert halls ask their audience to switch their phones to the (8.)______ mode. However, phone users fear that if they do not (9.) ______ their phones, they will lose valuable business opportunities. That's why many do not (10.)_____ off their phones even when they are asked to. Question 10: Write the correct FORM of each bracketed word. Write your answers in the numbered spaces provided below. (5p) 1. Those (theatre) group has never lost its appeal. 2. Dolphins, (mammal) species, sometimes jump above the surface of the water. 3. The situation is so (chaos) in some countries now that it is difficult to see any solution. 5 4. He’s the most (please), ill-mannered person I’ve ever met. 5. Good friends can enrich the quality of our lives (measure). 6. This famous singer had two (enter) in the Guinness Book of World Records. 7. Beauty is in the eye of the (hold). 8. Computers are now considered (dispense) in the business world. 9. Due to (electric) the difference between urban life and rural life is more and more reduced. 10. A doctor may prescribe (biotic) if the patient has an infection. E. WRITING (30p) Question 11: (10p) I. Finish each of the following sentences in such a way that it means the same as the sentence printed before it (5p). 1. Someone stole the old lady’s handbag. The old lady was ........................................................................................ 2. A true story forms the basis of Mary’s new novel. Mary’s new novel ......................................................................................... 3. If you have completed your test, you can go home. Get .............................................................................................................. 4. Absolute secrecy was crucial to the success of the mission. Without ....................................................................................................... 5. Something must be done to solve this problem. Urgent .......................................................................................... ................ II. Finish each of the following sentences in such a way that it is as similar its possible in meaning to the original sentence. Use the word given and other words as necessary. Do not change the form of the given word (5p). 1. The job received over a hundred applications. (APPLIED) ....................................................................................................................... 2. Alan prides himself on his punctuality. (TAKES) ....................................................................................................................... 3. Many people attended this year’s festival. (TURNOUT) ....................................................................................................................... 4. He is becoming quite famous as an interviewer. (NAME) ....................................................................................................................... 5. The whole team was in a happy mood. (SPIRITS) ....................................................................................................................... Question 12: Writing a paragraph (20p) Writing a paragraph about the advantages or disadvantages of hi-tech in our life. You should write at least 120 words. 6 BẮC NINH ĐÁP ÁN MẪU MÔN THI: Anh KHỐI 10 TT câu hỏi 1 2 3 4 5 Nội dung A. PHONETICS (10p) Question 1: Pick out the word whose underlined part is pronounced differently from those of the other words. (5p) 1. C 2.A 3.B 4.C 5.C Question 2: Choose the word whose stress pattern is differently from those of the other words. (5p) 1.C 2.B 3.A 4.D 5.B B. GRAMMAR & VOCABULARY (20p) Question 3: Give the correct tense/ form of the verbs in the brackets (10p) 1. went - don’t leave; will miss 2. shouldn’t/ oughtn’t have written - have been - haven’t had – - should/would have telephoned - have forgot(ten) 3. needn’t have borrowed 4. hurrying – lies - buried - loved 5. will be waiting - arrive 6. being - told - to do 7. went – can’t/ couldn’t have been Question 4: The passage below contains 10 errors. IDENTIFY and CORRECT them. Write your answers in the space. (0) has been done as an example. (5p) 1. visiting -->to visit 2. no --> not 3. right address -->the right address 4. their --> its 5. up --> down 6. had --> had been 7. risen --> raised 8.extending --> extended 9. previous -->prevuosly 10. untidy gravel yard --> an untidy gravel yard Question 5: Fill in each blank with a suitable PREPOSITION or PARTICLE. Write your answers in the numbered blanks below. (5p) 1. to 2. over 3. off 4. to 5. from 6. of 7. to 8. for 9. in 10. up C. READING (20p) Question 6: Read the following passage and choose the best answer. Write your answers in the numbered blanks provided below the passage. (10p) 1.B 2.C 3.A 4.D 5.B 6.B 7.C 8.A 9.B 10.C Question 7: Read the following passage and choose the best answer. Write your answers in the numbered blanks provided below the passage. (10p) 1. D 2.C 3. B 4.A 5. A 6.B 7.D 8.D 9. B 10. A D. USE OF ENGLISH (20p) Question 8: Choose the word that best fits each of the blanks in the following passage. (0) has been done as an example. (5p) 1. B 2.D 3.C 4.D 5.B 6.A 7.B 8.D 9.A 10.C Question 9: Fill each blank with ONE suitable word. Write your answers in the numbered blanks provided below the passage. (10p) 1. not 2. life 3. ring 4. More 5.using 7 Điểm 6. problems 7. passengers 8. silent 9. answer 10. turn/switch Question 10: Write the correct FORM of each bracketed word in the numbered spaces provided in the column on the right. (5p) 1. theatrical 2. mammalian 3. chaotic 4. unpleasant 5. immeasurably 6. entries 7. beholder 8. indispensable 9. electrification 10. antibiotics E. WRITING (30p) Question 11: (10p) I. Finish each of the following sentences in such a way that it means the same as the sentence printed before it (5p). 1. The old lady was robbed of her hand bag. 2. Mary’s new novel is based on a true story. 3. Get your test completed/ finished and you can go home. 4. Without absolute secrecy this mission would not have succeeded/ been successful/ been a success/ would have failed. 5. Urgent action is/ measures are necessary/ essential to solve this problem. II. Finish each of the following sentences in such a way that it is as similar its possible in meaning to the original sentence. Use the word given and other words as necessary. Do not change the form of the given word (5p). 1. Over a hundred people applied for the job. 2. Alan takes (a) pride in his punctuality. 3. This year’s festival attracted a record/ high turnout. 4. He is making quite a name for himself as an interviewer. 5. The whole team was in good spirits. Question 12: Writing a paragraph (20p) Writing a paragraph about the advantages or disadvantages of hi-tech in our life. You should write at least 120 words. 1. Content: 50% of total mark: a provision of all main ideas and details as appropriate 2. Language: 30% of total mark: a variety of vocabulary and structures appropriate to the level of English language gifted uppersecondary school students 3. Presentation: 20% of total mark: coherence, cohesion, and style appropriate to the level of English language gifted upper-secondary school students. 8 SỞ GIÁO DỤC VÀ ĐÀO TẠO HÀ TĨNH KỲ THI CHỌN HỌC SINH GIỎI TỈNH CẤP THPT NĂM HỌC 2012 - 2013 Môn thi: TIẾNG ANH 10 Thời gian làm bài: 180 phút (Đề thi có 08 trang, gồm 11 phần) ĐỀ CHÍNH THỨC Lưu ý: → → → → Thí sinh không sử dụng bất kể tài liệu nào, kể cả từ điển. Thí sinh làm bài trực tiếp vào đề thi, ghi câu trả lời vào các ô cho sẵn ở cuối các phần. Riêng phần trắc nghiệm thí sinh chỉ ghi đáp án A, B, C hoặc D vào ô cho sẵn. Giám thị không giải thích gì thêm. Điểm của toàn bài thi (Bằng số) (Bằng chữ) Các giám khảo Số phách (Ký và ghi rõ họ tên) (Do Trưởng Ban chấm thi ghi) Giám khảo 1: Giám khảo 2: A. LISTENING PART I. Listen to the dialogue on the phone between and a man and a girl named Juliet and fill in the form. You are allowed to listen TWICE. Give your answers in the numbered spaces. Name: Juliet A. Eastman Age: (1) …………………………………… Hair color: (2) …………………………………… Eye color: (3) …………………………………… Height: (4) …………………………………… Occupation: (5) …………………………………… ………………………………………………………….. Likes: going out and having fun, sports, (6) ……………………………………… and (7) …………………………………………… Wants to meet someone who : (8) …………………………………………………, likes same (9)………………………………………………. and (10) ……………………………………… Part II. You are going to hear an expert talk about sleeping and dreaming. Listen and write True (T) or False (F) for each sentence. You are allowed to listen TWICE. Your answers True (T) 1. Women sleep more than men. 2. A sound sleeper moves less than a light sleeper. 3. Most people need 9 hours of sleep a night. 4. Reading in bed helps you sleep. 1 False (F) 5. Some people don’t dream at all. 6. The average person has about four dreams a night. 7. Not everyone can remember his or her dreams. 8. Eating before bed can give you nightmares. PART III. You are going to listen to a talk about Margaret Mead. Listen and choose the best answer A, B, C or D for each question. You are allowed to listen TWICE. 1. What was Margaret Mead’s job? A. a photographer B. a biologist C. an anthropologist D. a journalist 2. What was Margaret Mead’s main interest? A. taking photographs B. exploring new places C. how children were looked after. D. living in pour areas. 3. When did Margaret Mead go to Samoa? A. in 1901. B. in the 1920s. C. in 1938. D. in 1978. 4. Who did she interview in her first trip to Samoa? A. girls between 9 and 20 years old. B. boys and girls between 9 and 20 years old. C. women over 20 years old. D. men and women over 20 years old. 5. What was the title of Margaret Mead’s book? A. The pacific Islands. B. Teenagers around the World. C. Growing Up in New Guinea. D. Coming of Age in Samoa. 6. What was the main reason why Margaret Mead took photos? A. She liked photography. B. Cameras were not very common at that time. C. Her husband liked photos. D. It was the best way to share what she learned. 7. What is the main topic of the listening passage? A. Margaret Mead went to college in New York. B. Margaret Mead did research on the role of culture. C. Margaret Mead took photographs and wrote books. D. Margaret Mead was born in Philadelphia. Your answers 1. 2. 3. 4. 5. 6. 7. B. LEXICO – GRAMMAR PART IV. Choose the answer A, B, C or D which best fits the space in each of the following sentences. 1. _________ saying was so important that I asked everyone to stop talking and listen. A. What the woman was B. That the woman was C. The woman was D. When was the woman 2. -“Do you mind if I take a seat?” - “_____________.” A. Yes, I don’t mind B. No, do as you please C. No I mind D. Yes, do as you please 3. As the two teams left the football ground, the 100,000 _________ gave them a standing ovation. A. bystanders B. spectators C. viewers D. audiences 4 My parents lent me the money. _________, I couldn’t have afforded the trip. A. However B. Therefore C. Only if D. Otherwise 5. It is interesting to take _________ a new hobby such as collecting stamps or going fishing. A. over B. on C. in D. up 6. Jack made me _________ him next week. A. promise calling B. to promise calling C. to promise to call D. promise to call 2 7. “I passed the TOEFL test, Mom.” - “ _________.” A. All right B. Thank you C. Well done D. Good luck 8. The bad weather caused serious damage to the crop. If only it _______ warmer. A. was B. were C. has been D. had been 9. - “Eric is really upset about losing his job.” - “ Well, ____once myself, I can understand.” A. Having been fired B. Fired C. Having fired D. Being fired 10. ________ you, I’d think twice about that decision. I could be a bad move. A. Were I B. Should I be C. If I am D. If I had been 11. The teacher asked a difficult question, but finally Ted _________ a good answer. A. put up with B. keep pace with C. made way for D. came up with 12. Not only ________ to speak to him, but she also vowed never to see him again. A. she refused B. did she refuse C. she did refuse D. when she refused 13. The judge ________ the pedestrian for the accident. A. accused B. charged C. caught D. blamed 14. She had to borrow her sister’s car because hers was _________. A. out of work B. out of order C. on duty D. off work 15. We should participate in the movement _________ to conserve the natural environment. A. to organize B. organizing C. which organized D. organized 16. His brother refuses to even listen to anyone else’s point of view. He is very_________. A. open-minded B. kind-hearted C. narrow-minded D. absent-minded 17. There is _________ in my bedroom. A. a square wooden old table B. an old square wooden table C. a wooden old square table D. an old wooden square table 18. “I am sorry. I broke the vase”. - “ _________.” A. OK. Go ahead B. Yes, certainly C. Don’t worry. Things break D. I’d rather not. 19. One’s fingerprints are _________ other person. A. different from B. different from any C. differ from any D. different from those of any 20. He is very happy because he passed his exam with __________ colours. A. flying B. failing C. imagining D. changing Your answers: 1. 2. 3. 4. 5. 6. 7. 8. 9. 10. 11. 12. 13. 14. 15. 16. 17. 18. 19. 20 PART V. From four underlined parts, choose the one that needs correction then correct it. For example : The teacher did not allow the students discussing the take-home exam with each other. discussing → to discuss 1. A Geiger counter is an electronic instrument is used to measure the presence and intensity of radiation. 2. A dolphin locates underwater objects in their path by making a series of clicking and whistling sounds. 3. In spite of its small size, Europe had a great impact on world history than other continents. 4. Before she moved here , Alene has been president of the organization for four years. 5. That Marta's been chosen as the most outstanding student on her campus make her parents very happy. 6. My cousin composes not only the music, but also sings the songs for the major Broadway musicals. 7. Our civilization is so commonplace to us that rarely we stop to think about its complexity. 8. Ever since the world began, nations have difficulty in keeping peace with their neighbors. 3 9. Those of us who have a family history of heart disease should do yearly appointments with our doctors. 10. If one had thought about the alternatives, he would not have chosen such difficult a topic for a term paper. Your answers Mistake Correction 1. 2. 3. 4. 5. 6. 7. 8. 9. 10. PART VI. Fill in each space in the following sentences with the most suitable prepositions. 1. I'm afraid Tom's _______ work. But Jack's in. Would you like to speak to him? 2. Have you been to the theatre recently? ~ Yes, I was _______ the Old Vie last night. 3. At first I found the work very tiring, but _______ a few weeks I got used to it. 4. _______ the daytime the streets are crowded but at night they are quite deserted. 5. I saw Tom at the bus stop this morning but couldn't speak to him because we were standing _____ a queue. 6. He is always in a hurry. He drives _______ a tremendous speed. 7. Write ________ ink and put your name on the top of the page. 8. The man with the pipe and red hair is the brother of the girl ________ blue. 9. He sits at his desk all day with his head in his hands. It gets ________ my nerves. 10. The children hastily changed _______ bathing things and jumped into the river with shouts of delight. Your answers: 1. 2. 3. 4. 5. 6. 7. 8. 9. 10. PART VII. Give the correct form of the word in bracket to complete the passage. Your answers: The __1__ (say) “never judge a book by its cover” could not be more true for Ridiculous Rules by Marjorie Allen. The cover is completely blank, whereas 1. ............................................ the book is crammed full of wonderful examples and anecdotes. Allen is an 2. ............................................ __2__ (speak) critic of what is taught to native and non-native speakers of 3. ............................................ English, and has issued a __3__ (declare) of war against textbooks and style books which tell lies. 4............................................. Take the ridiculous and __4__ (mean) rule of never ending a sentence with a preposition. The lovely - if famous – story goes, that Winston Churchill, 5............................................. well-known for his numerous __5__ (write) as well as for being British Prime Minister during the Second World War, received a manuscript back from an 6. ............................................ ignorant __6__ (edit), who had told him rather rudely that he had to __7__ 7. ............................................ 4 (phrase) a sentence which ended with a preposition. Churchill responded by making the simple yet forceful __8__ (state) in the margin: “This is an 8. ............................................ impertinence up with which I will not put.” – the __9__ (imply) being that not 9. ............................................ to end a sentence with a preposition often sounds ridiculous in English, Sadly, 10. ............................................ Allen informs us that the story is probably mere __10__ (hear), and that Churchill may have actually only written “rubbish!” in the margin. C. READING PART VIII. Read the passage and choose the best answer A, B, C, or D to each of the questions Large animals that inhabit the desert have evolved a number of adaptations for reducing the effects of extreme heat. One adaptation is to be light in color, and to reflect rather than absorb the Sun's rays. Desert mammals also depart from the normal mammalian practice of maintaining a constant body temperature. Instead of trying to keep down the body temperature deep inside the body, which would involve the expenditure of water and energy, desert mammals allow their temperatures to rise to what would normally be fever height, and temperatures as high as 46 degrees Celsius have been measured in Grant's gazelles. The overheated body then cools down during the cold desert night, and indeed the temperature may fall unusually low by dawn, as low as 34 degrees Celsius in the camel. This is an advantage since the heat of the first few hours of daylight is absorbed in warming up the body, and an excessive buildup of heat does not begin until well into the day. Another strategy of large desert animals is to tolerate the loss of body water to a point that would be fatal for non-adapted animals. The camel can lose up to 30 percent of its body weight as water without harm to itself, whereas human beings die after losing only 12 to 13 percent of their body weight. An equally important adaptation is the ability to replenish this water loss at one drink. Desert animals can drink prodigious volumes in a short time, and camels have been known to imbibe over 100 liters in a few minutes. A very dehydrated person, on the other hand, cannot drink enough water to dehydrate at one session, because the human stomach is not sufficiently big and because a too rapid dilution of the body fluids causes death from water intoxication. The tolerance of water loss is of obvious advantage in the desert, as animals do not have to remain near a water hole but can obtain food from grazing sparse and far-flung pastures. Desert-adapted mammals have the further ability to feed normally when extremely dehydrated, it is a common experience in people that appetite is lost even under conditions of moderate thirst. 1. What is the main topic of the passage? A. Weather variations in the desert B. Adaptations of desert animals C. Diseased of desert animals D. Human use of desert animals. 2. According to the passage, why is light coloring an advantage to large desert animals? A. It helps them hide from predators. B. It does not absorb sunlight as much as dark colors. C. It helps them see their young at night D. It keeps them cool at night. 3. The word "maintaining" is closest in meaning to ___________. A. measuring B. inheriting C. preserving D. delaying 4. The author uses of Grant's gazelle as an example of ___________. A. an animal with a low average temperature B. an animal that is not as well adapted as the camel C. a desert animal that can withstand high body temperatures D. a desert animal with a constant body temperature 5. When is the internal temperature of a large desert mammal lower? A. Just before sunrise B. In the middle of the day C. Just after sunset D. Just after drinking 5 6. The word "tolerate" is closest in meaning to ___________. A. endure B. replace C. compensate D. reduce 7. What causes water intoxication? A. Drinking too much water very quickly B. Drinking polluted water C. Bacteria in water D. Lack of water. 8. What does the author imply about desert-adapted mammals? A. They do not need to eat much food. B. They can eat large quantities quickly C. They easily lose their appetites. D. They can travel long distances looking for food. 9. Why does the author mention humans in the second paragraph? A. To show how they use camels. B. To contrast them to desert mammals. C. To give instructions about desert survival. D. To show how they have adapted to desert life. 10. Which of the following is NOT mentioned as an adaptation of large desert animals? A. Variation in body temperatures B. Eating while dehydrated C. Drinking water quickly D. Being active at night. Your answers: 1. 2. 3. 4. 5. 6. 7. 8. 9. 10. PART IX. Read the text below and decide which answer A, B, C or D best fits each space. When you read something in a foreign language, you frequently (1)______ across words you do not fully understand. Sometimes you check the meaning in a dictionary and sometimes you (2)______. The strategy you adopt depends very much upon the (3)______ of accuracy you require and the time at your disposal. If you are the sort of person who tends to turn to the dictionary frequently, it is worth remembering that every dictionary has its (4)______. Each definition is only an approximation and one builds up an accurate picture of the meaning of a word only after meeting it in a (5)______ of contexts. It is also important to recognize the special dangers of dictionaries that translate from English into your native language and vice versa. If you must use a dictionary, it is usually far safer to (6)______ an English-English dictionary. In most exams you are not permitted to use a dictionary. (7)______ you are allowed to use one, it is very time-consuming to look up words, and time in exams is usually limited. You are, (8)______ , forced to guess the meaning of unfamiliar words. When you find unknown words in an exam text, it is very easy to panic. However, if you develop efficient techniques for guessing the meaning, you will (9)______ a number of possible problems and help yourself to understand far more of the text than you at first thought likely. Two strategies which may help you guess the meaning of a word are: using contextual clues, both within the sentence and outside, and making use of clues (10)______ from the formation of the word. 1. 2. 3. 4. 5. 6. 7. 8. 9. 10. A. put A. look A. extent A. limitations A. multiple A. survey A. or else A. so A. surpass A. derived B. drop B. guess B. range B. values B. variety B. consult B. Provided B. therefore B. get over B. extracted C. see C. examine C. degree C. advantages C. variation C. refer C. Although C. completely C. go over C. coming D. come D. inspect D. level D. entry D. diversity D. inquire D. Even if D. so that D. overcome D. originated Your answers: 1. 2. 3. 4. 5. 6. 7. 8. 9. 10. 6 D. WRITING PART X. Complete the second sentence so that it has similar meaning to the first one. 1. This will be my student's first performance in Canada. → This will be the first time …………………………………………………………………………... 2. This course will take us six months to complete. → In six months time ………………………………………………………………………………….. 3. The number of people who understand his ideas exceed his expectations. → More people ………………………………………………………………………………………… 4. She'll have to make her presentation at the end of his speech. → The moment he …………………………………………………………………………………….. 5. Sharon will finish her exams. Then she will have more free time. → Once ………………………………………………………………………………………………… 6. Both Mary and Peter prefer jazz to classical music. → Neither……………………………………………………………………………………………… 7. They repaired my car at the garage in town. → I............................................................................................................................................................ 8. This is the last time I will speak to you. → I........................................................................................................................................................... 9. I prefer staying in to going out. → I'd rather.............................................................................................................................................. 10. They passed the driving test because of the easy questions. → If …………………………………………………………………………………………………… Your answers: 1. ................................................................................................................................................................................................... 2. ................................................................................................................................................................................................... 3. ................................................................................................................................................................................................... 4. ................................................................................................................................................................................................... 5. ................................................................................................................................................................................................... 6. ................................................................................................................................................................................................... 7. ................................................................................................................................................................................................... 8. ................................................................................................................................................................................................... 9. .................................................................................................................................................................................................. 10. .................................................................................................................................................................................................. PART XI. ESSAY WRITING Some people say that cell phones have improved modern life. Others believe that cell phones have caused many problems to people. What is your opinion? In about 250 words, write an essay to assert your point of view on this problem. ………………………………………………………………………………………………………… ………………………………………………………………………………………………………… ………………………………………………………………………………………………………… 7 ………………………………………………………………………………………………………… ………………………………………………………………………………………………………… ………………………………………………………………………………………………………… ………………………………………………………………………………………………………… ………………………………………………………………………………………………………… ………………………………………………………………………………………………………… ………………………………………………………………………………………………………… ………………………………………………………………………………………………………… ………………………………………………………………………………………………………… ………………………………………………………………………………………………………… ………………………………………………………………………………………………………… ………………………………………………………………………………………………………… ………………………………………………………………………………………………………… ………………………………………………………………………………………………………… ………………………………………………………………………………………………………… ………………………………………………………………………………………………………… ………………………………………………………………………………………………………… ………………………………………………………………………………………………………… ………………………………………………………………………………………………………… ………………………………………………………………………………………………………… ………………………………………………………………………………………………………… ………………………………………………………………………………………………………… ………………………………………………………………………………………………………… ………………………………………………………………………………………………………… ………………………………………………………………………………………………………… ………………………………………………………………………………………………………… ………………………………………………………………………………………………………… ………………………………………………………………………………………………………… THE END 8 SỞ GIÁO DỤC & ĐÀO TẠO TP. HỒ CHÍ MINH Trường THPT Chuyên Lê Hồng Phong KỲ THI OLYMPIC TRUYỀN THỐNG 30/4 LẦN XVI- NĂM 2010 Môn thi: ANH - Khoi :10 Ngày thi: 03/04/2010 Thời gian làm bài : 180 phút Ghi chú: Đề này có ... 7.... Trang. Thí sinh làm bài trên phiếu trả lời riêng ====================================================================== A. MULTIPLE CHOICE I. PHONOLOGY Choose the word whose underlined part is pronounced differently from the other three. 1. A. rhinoceros B. vehicle C. whale D. uninhabitable 2. A. unconcernedly B. ragged C. sacred D. hiccupped 3. A. archaic B. archive C. choir D. archery 4. A. suggestion B. congestion C. digestion D. devotion 5. A. dairy B. lair C. fair D. gait Choose the word whose stress pattern is different from that of the other three. 6. A. credulous B. acropolis C. dialect D. obsolete 7. A. ecotourism B. compromise C. disposal D. neighborhood 8. A. beneficial B. detrimental C. understanding D. magnificent, 9. A. zoology B. cement C. conquest D. duet 10. A. argumentative B. psychological C. contributory D. hypersensitive II. READING COMPREHENSION Read the following passage and choose the best option to complete the blank or answer the question. Since water is the basis of life, composing the greatest part of the tissues of all living things, the crucial problem of desert animals is to survive in a world where sources of flowing water is rare. And since man's inexorable necessity is to absorb large quantities of water at frequent intervals, he can scarcely comprehend that many creatures of the desert pass their entire lives without a single drop. Uncompromising as it is, the desert has not eliminated life but only those forms unable to withstand its desiccating effects. No moist-skinned, water-loving animals can exist there. Few large animals are found: the giants of the North American desert are deer, the coyote, and the bobcat. Since desert country is open, it holds more swift-footed, running, and leaping creatures than the tangled forest. Its population is largely nocturnal, silent, filled with reticence, and ruled by stealth. Yet they are not emaciated. Having adapted to their austere environment, they are as healthy as animals anywhere in the world. The secret of their adjustment lies in a combination of behavior and physiology. None could survive, if, like mad dogs and Englishmen, they went out in the midday sun, many would die in a matter of minutes. So most of them pass the burning hours asleep in cool, humid burrows underneath the ground, emerging to hunt only by night. The surface of the sun-baked desert averages around 150 degrees, but 18 inches down the temperature is only 60 degrees. 11. What is the topic of the passage? A Desert plants B. Desert life C. Animal life D. Forest life 12. The phrase 'those forms' refers to all the following except ... A. water-loving animals B. the bobcat C. moist-skinned animals D. many large animals 13. The word 'inexorable' is closest in meaning to ... A. relentless B. indispensable C. full D. demanding 14. The author mentions all of the following as examples of the behavior of desert animals except ... A. animals sleep during the day C. animals are noisy and aggressive 1 B. animals dig homes underground D. animals are watchful and quiet 15. The word 'them' refers to... A. mad dogs and Englishmen B. desert animals C. behavior and physiology D. minutes 16. The word 'emaciated' is closest in meaning to ... A wild B. cunning C. unmanageable D. unhealthy 17. The author states that one characteristic of animals living in the desert is that they ... A. are smaller and fleeter than forest animals B. are less healthy than animals living in different places C. can hunt in temperature of 150 degrees D. live in an accommodating environment 18. Which of the following generalizations is supported by the passage? A. Water is the basis of life. B. All living things adjust to their environments. C. Desert life is colorful. D. Healthy animals live longer lives. 19. The word 'burrows' is closest in meaning to ... A. underground nests B. underground houses C. underground caves D. underground holes 20. How is the temperature 18 inches underground compared to that on the surface in the desert? A. the same B. much higher C. less than half D. half Read the following passage and choose the best option to complete the blank or answer the question. Even before the turn of the century, movies began to develop in two major directions: the realistic and the formalistic. Realism and formalism are merely general, rather than absolute, terms. When used to suggest a tendency toward either polarity, such labels can be helpful, but in the end they are just labels. Few films are exclusively formalist in style, and fewer yet are completely realist. There is also an important difference realism and reality, although this distinction is often forgotten. Realism is a particular type, whereas physical reality is the source of all the raw materials of film, both realistic and formalistic. Virtually, all movie directors go to the photographable world for their subject matter, but what they do with this material- how they shape and manipulate itdetermines their stylistic emphasis. Generally speaking, realistic films attempt to reproduce the surface of concrete reality with a minimum of distortion. In photographing objects and events, the filmmaker tries to suggest the copiousness of life itself. Both realist and formalist film directors must select (and hence emphasize) certain details from the chaotic sprawl of reality. But the element of selectivity in realistic films is less obvious. Realists, in short, try to preserve the illusion that their film world is unmanipulated, an objective mirror of the actual world. Formalists, on the other hand, make no such pretense. They deliberately stylize and distort their raw materials so that only the very naive would mistake a manipulated image of an object or event for the real thing. We rarely notice the style in a realistic movie; the artist tends to be self-effacing. Some filmmakers are more concerned with what is being shown than how it is manipulated. The camera is used conservatively. It is essentially a recording mechanism that produces the surface of tangible objects with as little commentary as possible. A high premium is placed on simplicity, spontaneity, and directness. This is not to suggest that these movies lack artistry, however, for at its best the realistic cinema specializes in art that conceals art. 21. What does the passage mainly discuss? A. Acting styles C. Styles of filmmaking B. Film plots D. Filmmaking 100 years ago 22. With which of the following statements would the author be most likely to agree? A. Realism and formalism are outdated terms. B. Most films are neither exclusively realistic nor formalistic. C. Realistic films are more popular than formalistic ones. 2 D. Formalistic films are less artistic than realistic ones. 23. The phrase " this distinction" in the first paragraph refers to the difference between A. formalists and realists C. general and absolute B. realism and reality D. physical reality and raw materials 24. Whom does the author say is primarily responsible for a style of a film? A. The director C. The producer B. The actors D. The camera operator 25. The word "it" in the first paragraph refers to A. the photographable world C. this material B. their subject matter D. their stylistic emphasis 26. The word “copiousness" in the second paragraph is closest in meaning to A. abundance C. fullness B. greatness D. variety 27. How can one recognize the formalist style? A. It uses familiar images. C. It obviously manipulated images. B. It is very impersonal. D. It mirrors the actual world. 28. The word " tangible" in the last paragraph is closest in meaning to A. concrete C. various B. complex D. comprehensible 29. The word " self-effacing" in the last paragraph is closest in meaning to A. modest C. egocentric B. shy D. introverted 30. Which of the following films would most likely use a realist style? A. A travel documentary C. A musical drama B. A science fiction film D. An animated cartoon III. GUIDED CLOZE Read the following passage and choose the options that best complete the blanks. Sylvia Earle, a (31)_____botanist and one of the (32)_____deep - sea explorers, has spent over 6,000 hours, more than seven months, under water. From her earliest years, she took her first plunge intothe open sea as a teenager. In the years since then, she has taken part in a(n) (33)______ of landmark underwater projects, from exploratory expeditions around the world to her celebrated "Jim dive" in 1978, which was the deepest solo dive (34)_____made without cable connecting the diver to a support vessel at the surface of the sea. (35)_____in a Jim suit, a futuristic suit of plastic and metal armor, which was secured (36) _____a manned submarine, Sylvia Earle plunged vertically into the Pacific Ocean, at times at the speed of 100 feet per minute. (37)_____reaching the ocean floor, she was released from the submarine and from that point her only connection to the sub was an 18-foot tether. For the next two and a half hours, Earle (38)_____the seabed, taking notes, collecting (39)_____, and painting a U.S. flag. Consumed by a desire to descend deeper still, in I981she became involved in the design and manufacture of deep-sea (40)______ one of which took her to a depth of 3000 feet. This did not end Sylvia Earle's accomplishments. 31. A. marine 32. A. furthest 33. A. amount 34. A. really 35. A. Covered 36. A. to 37. A. In 38. A. walked 39. A. specimens 40. A. subcontractors B. underwater B. foremost B. great deal B. later B. Put B. with B. On B. roamed B. models B. submariners C. undersea C. greatest C. average C. ever C. clothed C. from C. At C. dived C. remains C. submersions D. submarine D. utmost D. number D. mostly D Worn D. against D. For D. strolled D. debris D. submersibles 3 B. WRITTEN TEST I. VERB TENSES / FORMS 1. He (go)_____to the last meeting, but he didn't. 2. By the time you come here again, this palace (build)_____. 3. The yesterday accident is thought (cause)_____by human error. 4. We (cook)_____all day for the party that evening and by 8 o'clock we still weren't ready. 5. It is highly desirable that every effort (make)_____to reduce expenditure. 6. (There ,be)_____any errors, let me know. 7. His (take)_____ill was quite unexpected. 8. In 20 hours' time, I (relax)_____on my yacht. 9. What a boring show! I would rather (not go)_____there. 10. It was a close call. We (kill)_____. In democratic countries, any efforts (11) (restrict) _____the freedom of the press are rightly condemned. However, this freedom (12) (easily, abuse)_____. Stories about people often attract far more public attention than political events. Though we may enjoy reading about the lives of others, it is extremely doubtful whether we would equally enjoy reading about ourselves. (13) (act) _____on the contention that facts are sacred, reporters can cause untold suffering to individuals by publishing details about their private lives. Newspapers exert tremendous influence that they cannot only bring about major changes to the lives of ordinary people but (14) (even, overthrow)_____a government. The story of a poor family that (15) (acquire)_____fame and fortune overnight, dramatically illustrates the power of the press. The family lived in Aberdeen, a small town in South Dakota. As the parents had five children, life was a perpetual struggle against poverty. They (16) (expect)_____their sixth child and (17) (face)_____with even more pressing economic problems. If they had had only one more child, the fact would have passed unnoticed. They (18) (continue) _____ to struggle against economic odds and would have lived in obscurity. But they suddenly became the parents of quintuplets, four girls and a boy, an event which radically changed their lives. The day after the birth of the five children, a plane arrived in Aberdeen (19) (bring)_____sixty reporters and photographers. The news was of national importance, for the poor couple (20) (become)_____the parents of the only quintuplets in America. II. PREPOSITIONS & PHRASAL VERBS Part 1: Complete each of the following sentences with (a) suitable preposition(s) or particle(s) from the box in for up to through down off round under out at over 1. I wish you wouldn't fly_____me like that every time I make a mistake. 2. The piece of equipment is very well made and stands_____the roughest treatment. You won't have any trouble with it. 3. The new office staff are shaking_____well. 4. I think a sip of wine can bring him_____. 5. We are_____no obligation to change goods which were not purchased here. 6. I am not friends with Peter any more. We have fallen_____. 7. I knew I was_____the hill when I started needing glasses to read. 8. Tom's family pulled him_____the difficult period following his wife's death. 9. The police came_____a great deal of criticism. 10. I have gone____computer games. They are not as interesting as before. Part 2: Complete the following passage with prepositions. 4 It is not easy trying to cope with fear. Most people (11) _______ some stage in their lives feel afraid of something; fear of the dark as children, afraid of spiders or flying. For the most part, these fears are normal and do not interfere (12) _______ our ability to get on with our lives. However, some people are afraid of something (13) _______ such an extent that it prevents them from leading a normal life. For example, 'electrophobia' - being afraid of electricity - makes life in today's world extremely difficult. In fact, you can be afraid of anything and there is almost certainly a name for it. You can be afraid of clouds, certain colors, bicycles, rain, mushrooms and even sitting down. Apparently, the list of phobia gets longer everyday, but for people who have a real terror of something, help is (14) _______ hand. Researchers are making enormous progress (15) _______ understanding what a phobia is and what makes it come (16) _______ . It is surprising how many people think they suffer from a phobia when actually all they are really experiencing is a strong dislike or distaste (17) _______ something. You may think you are computer phobic and want to throw your machine out of the nearest window. But that is not the same as being really 'mechanophobic', suffering from a racing heart and being short of breath (18) _______ the mere sight of a computer. Experts say that you cannot take a true phobia (19) _______ anything else as it affects the whole of your nervous system. If you cannot run away from whatever is causing the fear, you feel that is inevitable. On the other hand, it's natural for most people to be afraid if they are aboard an airplane which is flying into a storm. Most psychologists agree that phobias can be described (20) _______ three main ways: social phobias, panic disorders and specific phobias. V. OPEN CLOZE TEST Passage 1 Although noise, commonly (l) _______ as unwanted sound, is a widely recognized form of pollution, it is very difficult to measure because the discomfort (2) _______ by different individuals is highly subjective and, therefore, variable. Exposure to lower levels of noise may be (3) _______ irritating, whereas exposure to higher levels may actually cause (4) _______ loss. Particularly in congested urban areas, the noise produced (5) _______ a by-product of our advancing technology causes physical harm, and detracts from the quality of life for those who are exposed to it. Noise causes effects that the hearer cannot control and to (6) _______ the body never becomes accustomed. Loud noises instinctively signal danger (7) _______ any organism with a hearing mechanism, including human beings. (8) _______ response, heartbeat and respiration accelerate, blood vessels constrict, the skin pales, and muscles tense. In fact, there is a general increase in functioning (9) _______ about by the flow of adrenaline release in response to fear, and some of these responses persist even longer than the noise, occasionally as long as 30 minutes (10) _______ the sound has ceased. Passage 2 Unlikely (11) _______ it may seem, there has now been expert confirmation that wild pumas and lynxes are at (12)_______ in parts of Britain, rather than being the figments of some wild imaginations. Previous sightings of such large cats had been put (13) _______ to exaggeration. After all, the argument went, some people are prone to seeing flying saucers and Loch ness monsters, particularly when under the influence of one drink. Some newspapers were suspected of having made up stories such as (14) _______ of the Beast of Exmoor, an animal that is responsible for the deaths of hundreds of sheep over the past ten years. (15) _______ experts have now come up with proof that such stories were in earnest after all. The animals are in all (16) _______ pets which have escaped from small zoos, or been abandoned by their owners. Because the keeping of such animals is severely restricted under the (17) _______ of the Dangerous Wild Animal Act of 1976, owners of unlicensed animals might not (18) _______ an escape for fear of prosecution. Britain's only surviving native species, the wild cat, is confined to Scotland. After examining hair samples, experts now say that the Best of Exmoor in the south of England is without (19) _______ a puma or lynx, both of which are normally native (20) _______ the Middle East and Asia. III. WORD FORMS Part 1: Complete the sentences with the correct forms of the given words. 5 1. His _______ annoys some members of the committee because he is always expressing his ideas frankly. (speak) 2. George is very disorganized and not very_______ (business) 3. She was fully aware of her own _______ . (short) 4. Low income and little administrative support make teachers _______ _______ with their profession. (heart) 5. A film about the _______ ancestors is available in the library. (reptile) 6. There are people whose _______ begins from the moment of their death. (mortal) 7. It's undeniable that the _______ of the local incompetent healer was responsible for her sudden death. (diagnose) 8. As the sole _______ of his uncle's will, he inherited a huge fortune. (benefit) 9. This _______ faulty washing machine should be returned to the manufacturer. (repair) 10. The first time I tried out my new bike I _______ and fell off. (balance) Part 2: Complete the following passage with the correct forms of the given words. PROFESSOR SALE ACQUISITION STARRY BAND INVADE ORIGIN LONELY QUARTER POPULARITY The Beatles became the most popular grouping rock music history. The (11) _______ of extraordinarily talented musicians generated a frenzy that transcended countries and economic strata. While all of them sang, John Lennon and Paul McCartney wrote the majority of their songs. (12) _______ , Lennon and five others formed a group called Quarrymen in 1956, with McCartney joining them later that year. George Harrison, John Lennon and Paul McCartney, together with Stuart Sutcliffe, who played the bass guitar, and Pete Best on the drums, performed together in several bands for a few years, until they finally settled on the Silver Beatles in 1960. American Rock musicians, such as Chuck Berry and Elvis Presley, influenced Lennon's and McCartney's music, whose first hits consisted of simple tunes and lyrics about young love. The Beatles' U.S. tour propelled them to (13) _______ and led to two movies filmed in 1964 and 1965. The so-called British (14) _______ of the United States was in full swing when they took the top five spots on the singles' charts, followed by the release of their first film. During the 1960s, their music matured and (15) _______ a sense of melody. The lyrics of their song became deeper and gained in both imagination and meaning. Their popularity continued to grow as the Beatles turned their attention to social problems and political issues in "Nowhere Man" and Eleanor Rigby". (16) _______ and nostalgia come through in their ballad "Michelle" and 'Yesterday", which fully displayed the group's (17)_______ development and sophistication. Lennon's sardonic music with lyrics written in the first person, and McCartney's songs that created scenarios with off beat individuals, contributed to the character of the music produced by the group. In addition to their music, the Beatles set a social trend that (18)_______ long hair, Indian music, and moss dress. For a variety of reasons, the musicians began to drift apart, and their last concert took place in San Francisco in 1966. The newspapers and tabloids publicized their quarrels and lawsuits, and the much idolized group finally (19)_______ in 1970. However, their albums had (20) _______ those of any other bands in history. Although all of the Beatles continued to performed solo or form new rock groups alone, none could achieve the recognition and success that they had been able to win together. VI. SENTENCE TRANSFORMATION Rewrite the sentences with the given words or beginning in such a way that their meanings remain unchanged. 1. The only reason the party was a success was that a famous film star attended. → Had it ……………………………………………………………………………...................... 2. Government guidelines really do emphasize the importance of starting education early. → A lot of emphasis …………………………………………………………………………….... 3. The trip was so amazing that we will never forget it. → It's too ……………………………………………………………………………..................... 6 4. Your silly questions distracted me. → You drove ……………………………………………………………………………............... 5. She was so disgusted at the way her friend behaved that she refused to speak to him. → Such ……………………………………………………………………………........................ 6. Mike is never reluctant to make tough decisions as a manager. (SHRINKS) → ……………………………………………………………………………................................ 7. The film is similar to Shakespeare's Hamlet in a number of ways. (RESEMBLANCE) → The film ……………………………………………………………………………................. 8. He is determined to become a doctor. (HEART) → ……………………………………………………………………………................................ 9. Alison bought the big house because she wanted to open a hotel. (VIEW) → Alison bought ….……………………………………………………………………………….... 10. We feel uncomfortable in the house. (FISH) → ………………………………………………………………………………................................ IV. ERROR IDENTIFICATION Identify 10 errors in the following passage and correct them. Between 1977 and 1981, three groups of American women, numbered 27 in all, between the age of 35 and 65, were given month-long tests to determine how they would response to conditions resembling those on the space shuttle. Carefully selected from among many applicants, the women were volunteers and pay was barely above the minimum wage. They weren't allowed to smoke or drink alcohol during the tests, and they were expected to tolerate each other's company at closed quarters for the entire period. Among other things, they had to stand pressure three times of the force of gravity and carry out both physical and mental tasks while exhausted from strenuous physical exercises. At the end often days, they had to spend a further twenty days absolutely confined to bed, during that time they suffered backaches and discomforts, and when they were finally allowed up, the more physically active women were especially subject to pains due to a light calcium loss. Results of the tests suggest that women have significant advantages on men in space. They need less food and less oxygen and they stand up to radiation well. Men's advantages in terms of strength and stamina, meanwhile, are virtually wiped out by the zero-gravity condition in space. THE END 7 TRƯỜNG PTTH A THANH LIÊM KỲ THI CHỌN HỌC SINH GIỎI LỚP 10 NĂM HỌC 2011- 2012 Môn thi: TIẾNG ANH Thời gian: 150 phút (không kể thời gian giao đề) (Đề thi này gồm có 4 trang, thang điểm 20) Họ và tên thí sinh:…………………………..................................Lớp: 10A……... Điểm bài thi Bằng số Bằng chữ Họ tên, chữ ký của giám khảo 1............................................................................. PART I : PHONETIC (1.0 p): I. Find the word which has the underlined part pronounced differently from the others. B. hard C. parent D. Park 1. A.pardon 2. A.rose B. cover C. nose D. over 3. A. increase B. ink C. pink D. thank 4. A. beloved B. naked C. ploughed D. learned PART II : VOCABULARY AND GRAMMAR: I. Circle the correct answers (3,5p): 1. He got an excellent grade in his examination ______ the fact that he had not worked particularly hard . A. on account of B. because C. in spite of D. although 2. There were over 30,000 __________ at the match. A. spectators B. viewers C. witnesses D. watchers 3. ____ they are tropical birds, parrots can live in temperate or even cold climate. A. Despite B. Even though C. Nevertheless D. But 4. I was enjoying my book, but I stopped ___ a program on TV. A. to read to watch B. to read for watching C. reading for watching D. reading to watch 5. The man __________ we met yesterday was the manager of a bicycle factory. A. whose B. which C. whom D. when 6. I don't believe a word she says, I think she ___________. A. is laying B. is telling lie C. is lying D. lied 7. AIDS is a(n) ___________ disease. A. endanger B. dangerous C. danger D. endangered 8. I knew they were talking about me _________ they stopped when I entered the room. A. so that B. therefore C. because D. despite 9. When I came to visit her last night, she __________ a bath. A. was having B. has C. is having D. had 10. English ___________ in many parts of the world. A. is spoken B. is speaking C. was spoken D. speaks 11. The children ___________ to the zoo. A. were enjoyed taking B. were enjoyed taken 1 C. enjoyed taking D. enjoyed being taken 12. My parents first __________ each other at the Olympic Games in 1982. A. meet B. met C. had meet D. have met 13.Their ___________ has lasted for more than 20 years. A. friends B. friendship C. friend D. friendly 14. If I had time, I __________ to the beach with you this weekend. A. would have gone B. would go C. will have gone D. will go II. Supply the correct form of the words and verbs in the parentheses ( 2.0 p) 1. Visitors complained about the ………………..in the old museum. (organize) (care) 2. You must be ....................when you open that door . 3."Time of our lives" was the ........................song of the World Cup 2006. (office) 4. When we took an exam last year, we met a very strict ............................. (exam) 5. We can see many ……………………….. on TV everyday . (advertise) 6. I watch the news everyday because it very ………………………….. (inform) 7. He was very …………………………..of the work he had done . (pride) 8. I don’t find him very _______. (communicate) PART III : READING COMPREHENSION: I. Read the passage, and then choose the correct answer (3.0p): All over the world people listen to classical music. Classical music is difficult to describe. It means different things to different people. Some famous classical composers were Bach, Vivaldi, Haydn, and Mozart. In their music, they did not tell a story or show strong emotion. They wanted to make a beautiful, interesting design. They wanted to write lovely sounds. Then composers started to interpret ideas. They told stories about wars, armies, and soldiers. They wrote about religion. Sometimes they composed music for holidays. They told love stories and showed strong emotion. Some of these composers were Beethoven, Schumann, Chopin, Mendelssohn, Wagner, and Tchaikovsky. Classical music stays with people a long time. Bach wrote about 300 years ago, Beethoven wrote about 200 years ago, and Tchaikovsky wrote over 100 years ago. Sometimes people close their eyes to listen to classical music. When they close their eyes, they can see the design. They can listen to the same classical music many times and enjoy it. Sometimes it is difficult to understand. The listener has to think about it. However, we can all learn to enjoy some classical music. It is very important to people. 1. Classical music is famous ________. A. in the Western B. in Europe C. in the United States D. all over the world 2. The first classical composers wanted to ________. A. tell stories about religion B. write lovely sounds C. show strong emotion D. b and c 3. What kind of stories did some composers not tell in their music? A. love B. wars C. religion D. racial integration 4. Beethoven _____________________. A. composed his music a long time ago B. just wanted to make an interesting design C. did not tell a story. D. wrote some music that as difficult to understand 5. People sometimes close their eyes when they listen to classical music in order to ________. A. understand it clearly B. think about the design C. from the picture of the design in their mind D. hear the same classical music many times 6. Classical music __________________. A. is different from people to people 2 B. is difficult to understand so that listener always has to think about it C. was composed by famous musicians very long time ago D. is necessary for people because it makes life more colorful II. Choose the word or phrase which best fits each gap of the passage (2.5p) Yellowstone National park, the first US national park, was (1)_____ in 1872. It is one of the largest parks. It (2)______ about 3,500 square (3)_____or 9,065 square kilometers (4)______ northwest Wyoming and parts of Idaho and Montana. The park has many wild animals (5)______bears, buffalo, elk, deer, antelope coyotes and lynxes and is famous for fine scenery, hot springs and geysers. there are about seventy geysers in the park. 1. A. started B. introduced C. constructed D. established 2. A. covers B. involves C. spreads D. contains 3. A. lands B. feet C. miles D. distances 4. A. of B. in C. at D. from 5. A. adding B. including C. containing D. consisting PART IV : WRITING: I. There is a mistake in the underlined parts of each sentence. Identify your answer by circling the corresponding letter A, B, C or D: ( 1,5 p) 1. My brother hasn’t played football for last year. A B C D 2. I am fond with jogging every morning . A B C D 3. My brother said me to sit down at the table and do my homework . A B C D 4. My father prefers watching films at home than going to the cinema . A B C D 5. I’d like going to the cinema with my friends. A B C D 6. Jane asked me how I go to school everyday. A B C D II. Finish each of the sentences in such a way that it means exactly the same as the sentence printed before it. ( 3.5 p) 1. I couldn’t do the test because it was difficult. Because of …………………………………………………………………………………… 2. Although he got up early, he was late for the first train. In spite ……………………………………………………………………………………………… 3.Mr. Smith said, “I will refuse their offer.” ………………………………………………………………………………………. 4. Henry said, “I can meet them later.” ………………………………………………………………………………………. 5. I have to work late tomorrow, so I can’t join your party If……………………………………………………………………………………………… 6. When I came she was reading newspapers. (change into passive voice) .............................................................................................................................. 7. Are you writing a letter to her ? (change into passive voice) .............................................................................................................................. 3 III. Write a short paragraph (about 120 words) about the following topic: “What do you like to do in your free time” ……………………………………………………………………………………………………………… ……………………………………………………………………………………………………………… ……………………………………………………………………………………………………………… ……………………………………………………………………………………………………………… ……………………………………………………………………………………………………………… ……………………………………………………………………………………………………………… ……………………………………………………………………………………………………………… ……………………………………………………………………………………………………………… ……………………………………………………………………………………………………………… ……………………………………………………………………………………………………………… ……………………………………………………………………………………………………………… ……………………………………………………………………………………………………………… ……………………………………………………………………………………………………………… ………………………………………………………………………………………………………………. the end 4 SỞ GIÁO DỤC & ĐÀO TẠO TP. HỒ CHÍ MINH KỲ THI OLYMPIC TRUYỀN THỐNG 30/4 LẦN XVI- NĂM 2010 Trường THPT Chuyên Lê Hồng Phong Môn thi: ANH – Khối: 10 Ngày thi: 03/04/2010 Thời gian làm bài: 180 phút Ghi chú: Đề này có ... 7.... Trang. Thí sinh làm bài trên phiếu trả lời riêng A. MULTIPLE CHOICE I. PHONOLOGY Choose the word whose underlined part is pronounced differently from the other three. 1. a. rhinoceros b. vehicle c. whale d. uninhabitable 2. a. unconcernedly b. ragged c. sacred d. hiccuped 3. a. archaic b. archive c. choir d. archery 4. a. suggestion b. congestion c. digestion d. devotion 5. a. dairy b. lair c. fair d. gait Choose the word whose stress pattern is different from that of the other three. 6. a. credulous b. acropolis c. dialect d. obsolete 7. a. ecotourism b. compromise c. disposal d. neighborhood 8. a. beneficial b. detrimental c. understanding d. magnificent, 9. a. zoology b. cement c. conquest d. duet 10. a. argumentative b. psychological c. contributory d. hypersensitive II. READING COMPREHENSION Read the following passage and choose the best option to complete the blank or answer the question. Since water is the basis of life, composing the greatest part of the tissues of all living things, the crucial problem of desert animals is to survive in a world where sources of flowing water is rare. And since man's inexorable necessity is to absorb large quantities of water at frequent intervals, he can scarcely comprehend that many creatures of the desert pass their entire lives without a single drop. Uncompromising as it is, the desert has not eliminated life but only those forms unable to withstand its desiccating effects. Nomoist-skinned, water-loving animals can exist there. Few large animals are found: the giants of the North American desert are deer, the coyote, and the bobcat. Since desert country is open, it holds more swiftfooted, running, and leaping creatures than the tangled forest. Its population is largely nocturnal, silent, filled with reticence, and ruled by stealth. Yet they are not emaciated. Having adapted to their austere environment, they are as healthy as animals anywhere in the world. The secret of their adjustment lies in a combination of behavior and physiology. None could survive, if, like mad dogs and Englishmen, they went out in the midday sun, many would die in a matter of minutes. So most of them pass the burning hours asleep in cool, humid burrows underneath the ground, emerging to hunt only by night. The surface of the sun-baked desert averages around 150 degrees, but 18 inches down the temperature is only 60 degrees. 11. What is the topic of the passage? a Desert plants b. Desert life c. Animal life d. Forest life 12. The phrase 'those forms' refers to all the following except ... a. water-loving animals b. the bobcat c. moist-skinned animals d. many large animals 13. The word 'inexorable' is closest in meaning to ... a. relentless b. indispensable c. full d. demanding 14. The author mentions all of the following as examples of the behavior of desert animals except ... a. animals sleep during the day b. animals dig homes underground c. animals are noisy and aggressive d. animals are watchful and quiet 15. The word 'them' refers to... a. mad dogs and Englishmen b. desert animals c. behavior and physiology d. minutes 16. The word 'emaciated' is closest in meaning to ... a. wild b. cunning c. unmanageable d. unhealthy 17. The author states that one characteristic of animals living in the desert is that they ... a. are smaller and fleeter than forest animals b. are less healthy than animals living in different places c. can hunt in temperature of 150 degrees d. live in an accommodating environment 18. Which of the following generalizations is supported by the passage? a. Water is the basis of life. b. All living things adjust to their environments. c. Desert life is colorful. d. Healthy animals live longer lives. 19. The word 'burrows' is closest in meaning to ... a. underground nests b. underground houses c. underground caves d. underground holes 20. How is the temperature 18 inches underground compared to that on the surface in the desert? a. the same b. much higher c. less than half d. half Read the following passage and choose the best option to complete the blank or answer the question. Even before the turn of the century, movies began to develop in two major directions: the realistic and the formalistic. Realism and formalism are merely general, rather than absolute, terms. When used to suggest a tendency toward either polarity, such labels can be helpful, but in the end they are just labels. Few films are exclusively formalist in style, and fewer yet are completely realist. There is also an important difference realism and reality, although this distinction is often forgotten. Realism is a particular type, whereas physical reality is the source of all the raw materials of film, both realistic and formalistic. Virtually, all movie directors go to the photographable world for their subject matter, but what they do with this material- how they shape and manipulate it- determines their stylistic emphasis. Generally speaking, realistic films attempt to reproduce the surface of concrete reality with a minimum of distortion. In photographing objects and events, the filmmaker tries to suggest the copiousness of life itself. Both realist and formalist film directors must select (and hence emphasize) certain details from the chaotic sprawl of reality. But the element of selectivity in realistic films is less obvious. Realists, in short, try to preserve the illusion that their film world is unmanipulated, an objective mirror of the actual world. Formalists, on the other hand, make no such pretense. They deliberately stylize and distort their raw materials so that only the very naive would mistake a manipulated image of an object or event for the real thing. We rarely notice the style in a realistic movie; the artist tends to be self-effacing. Some filmmakers are more concerned with what is being shown than how it is manipulated. The camera is used conservatively. It is essentially a recording mechanism that produces the surface of tangible objects with as little commentary as possible. A high premium is placed on simplicity, spontaneity, and directness. This is not to suggest that these movies lack artistry, however, for at its best the realistic cinema specializes in art that conceals art. 21. What does the passage mainly discuss? a. Acting styles b. Film plots c. Styles of filmmaking d. Filmmaking 100 years ago 22. With which of the following statements would the author be most likely to agree? a. Realism and formalism are outdated terms. b. Most films are neither exclusively realistic nor formalistic. c. Realistic films are more popular than formalistic ones. d. Formalistic films are less artistic than realistic ones. 23. The phrase " this distinction" in the first paragraph refers to the difference between a. formalists and realists b. realism and reality c. general and absolute d. physical reality and raw materials 24. Whom does the author say is primarily responsible for a style of a film? a. The director b. The actors c. The producer d. The camera operator 25. The word "it" in the first paragraph refers to a. the photographable world b. their subject matter c. this material d. their stylistic emphasis 26. The word “copiousness" in the second paragraph is closest in meaning to a. abundance b. greatness c. fullness d. variety 27. How can one recognize the formalist style? a. It uses familiar images. b. It is very impersonal. c. It obviously manipulated images. d. It mirrors the actual world. 28. The word " tangible" in the last paragraph is closest in meaning to a. concrete b. complex c. various d. comprehensible 29. The word " self-effacing" in the last paragraph is closest in meaning to a. modest b. shy c. egocentric d. introverted 30. Which of the following films would most likely use a realist style? a. A travel documentary b. A science fiction film c. A musical drama d. An animated cartoon III. GUIDED CLOZE Read the following passage and choose the options that best complete the blanks. Sylvia Earle, a (31)_____botanist and one of the (32)_____deep - sea explorers, has spent over 6,000 hours, more than seven months, under water. From her earliest years, she took her first plunge into the open sea as a teenager. In the years since then, she has taken part in a(n) (33)______ of landmark underwater projects, from exploratory expeditions around the world to her celebrated "Jim dive" in 1978, which was the deepest solo dive (34)_____made without cable connecting the diver to a support vessel at the surface of the sea. (35)_____in a Jim suit, a futuristic suit of plastic and metal armor, which was secured (36) _____a manned submarine, Sylvia Earle plunged vertically into the Pacific Ocean, at times at the speed of 100 feet per minute. (37)_____reaching the ocean floor, she was released from the submarine and from that point her only connection to the sub was an 18-foot tether. For the next two and a half hours, Earle (38)_____the seabed, taking notes, collecting (39)_____, and painting a U.S. flag. Consumed by a desire to descend deeper still, in I981she became involved in the design and manufacture of deep-sea (40)______ one of which took her to a depth of 3000 feet. This did not end Sylvia Earle's accomplishments. 31. a. marine b. underwater c. undersea d. submarine 32. a. furthest b. foremostc. greatestd. utmost 33. a. amount b. great dealc. averaged. number 34. a. really b. laterc. ever d. mostly 35. a. Covered b. Put c. Clothedd. Worn 36. a. to b. with c. fromd. against 37. a. In b. Onc. Atd. For 38. a. walked b. roamed c. divedd. strolled 39. a. specimens b. modelsc. remainsd. debris 40. a. subcontractors b. submarinersc. submersionsd. submersibles B. WRITTEN TEST I. VERB TENSES / FORMS 1. He (go)_____to the last meeting, but he didn't. 2. By the time you come here again, this palace (build)_____. 3. The yesterday accident is thought (cause)_____by human error. 4. We (cook)_____all day for the party that evening and by 8 o'clock we still weren't ready. 5. It is highly desirable that every effort (make)_____to reduce expenditure. 6. (There ,be)_____any errors, let me know. 7. His (take)_____ill was quite unexpected. 8. In 20 hours' time, I (relax)_____on my yacht. 9. What a boring show! I would rather (not go)_____there. 10. It was a close call. We (kill)_____. In democratic countries, any efforts (11) (restrict) _____the freedom of the press are rightly condemned. However, this freedom (12) (easily, abuse)_____. Stories about people often attract far more public attention than political events. Though we may enjoy reading about the lives of others, it is extremely doubtful whether we would equally enjoy reading about ourselves. (13) (act) _____on the contention that facts are sacred, reporters can cause untold suffering to individuals by publishing details about their private lives. Newspapers exert tremendous influence that they cannot only bring about major changes to the lives of ordinary people but (14) (even, overthrow)_____a government. The story of a poor family that (15) (acquire)_____fame and fortune overnight, dramatically illustrates the power of the press. The family lived in Aberdeen, a small town in South Dakota. As the parents had five children, life was a perpetual struggle against poverty. They (16) (expect)_____their sixth child and (17) (face)_____with even more pressing economic problems. If they had had only one more child, the fact would have passed unnoticed. They (18) (continue) _____ to struggle against economic odds and would have lived in obscurity. But they suddenly became the parents of quintuplets, four girls and a boy, an event which radically changed their lives. The day after the birth of the five children, a plane arrived in Aberdeen (19) (bring)_____sixty reporters and photographers. The news was of national importance, for the poor couple (20) (become)_____the parents of the only quintuplets in America. II. PREPOSITIONS & PHRASAL VERBS Part 1: Complete each of the following sentences with (a) suitable preposition(s) or particle(s) from the box: in for _______ up to __________ through__________ down________ off round_________ under_______ out _______________at __________over 1. I wish you wouldn't fly_____me like that every time I make a mistake. 2. The piece of equipment is very well made and stands_____the roughest treatment. You won't have any trouble with it. 3. The new office staff are shaking_____well. 4. Ithink a sip of wine can bring him_____. 5. We are_____no obligation to change goods which were not purchased here. 6. Iam not friends with Peter any more. We have fallen_____. 7. I knew I was_____the hill when I started needing glasses to read. 8. Tom's family pulled him_____the difficult period following his wife's death. 9. The police came_____a great deal of criticism. 10. I have gone____computer games. They are not as interesting as before. Part 2: Complete the following passage with prepositions. It is not easy trying to cope with fear. Most people (11) _______ some stage in their lives feel afraid of something; fear of the dark as children, afraid of spiders or flying. For the most part, these fears are normal and do not interfere (12) _______ our ability to get on with our lives. However, some people are afraid of something (13) _______ such an extent that it prevents them from leading a normal life. For example, 'electrophobia' - being afraid of electricity - makes life in today's world extremely difficult. In fact, you can be afraid of anything and there is almost certainly a name for it. You can be afraid of clouds, certain colors, bicycles, rain, mushrooms and even sitting down. Apparently, the list of phobia gets longer everyday, but for people who have a real terror of something, help is (14) _______ hand. Researchers are making enormous progress (15) _______ understanding what a phobia is and what makes it come (16) _______ . It is surprising how many people think they suffer from a phobia when actually all they are really experiencing is a strong dislike or distaste (17) _______ something. You may think you are computer phobic and want to throw your machine out of the nearest window. But that is not the same as being really 'mechanophobic', suffering from a racing heart and being short of breath (18) _______ the mere sight of a computer. Experts say that you cannot take a true phobia (19) _______ anything else as it affects the whole of your nervous system. If you cannot run away from whatever is causing the fear, you feel that is inevitable. On the other hand, it's natural for most people to be afraid if they are aboard an airplane which is flying into a storm. Most psychologists agree that phobias can be described (20) _______ three main ways: social phobias, panic disorders and specific phobias. III. OPEN CLOZE TEST Passage 1 Although noise, commonly (l) _______ as unwanted sound, is a widely recognized form of pollution, it is very difficult to measure because the discomfort (2) _______ by different individuals is highly subjective and, therefore, variable. Exposure to lower levels of noise may be (3) _______ irritating, whereas exposure to higher levels may actually cause (4) _______ loss. Particularly in congested urban areas, the noise produced (5) _______ a by-product of our advancing technology causes physical harm, and detracts from the quality of life for those who are exposed to it. Noise causes effects that the hearer cannot control and to (6) _______ the body never becomes accustomed. Loud noises instinctively signal danger (7) _______ any organism with a hearing mechanism, including human beings. (8) _______ response, heartbeat and respiration accelerate, blood vessels constrict, the skin pales, and muscles tense. In fact, there is a general increase in functioning (9) _______ about by the flow of adrenaline release in response to fear, and some of these responses persist even longer than the noise, occasionally as long as 30 minutes (10) _______ the sound has ceased. Passage 2 Unlikely (11) _______ it may seem, there has now been expert confirmation that wild pumas and lynxes are at (12)_______ in parts of Britain, rather than being the figments of some wild imaginations. Previous sightings of such large cats had been put (13) _______ to exaggeration. After all, the argument went, some people are prone to seeing flying saucers and Loch ness monsters, particularly when under the influence of one drink. Some newspapers were suspected of having made up stories such as (14) _______ of the Beast of Exmoor, an animal that is responsible for the deaths of hundreds of sheep over the past ten years. (15) _______ experts have now come up with proof that such stories were in earnest after all. The animals are in all (16) _______ pets which have escaped from small zoos, or been abandoned by their owners. Because the keeping of such animals is severely restricted under the (17) _______ of the Dangerous Wild Animal Act of 1976, owners of unlicensed animals might not (18) _______ an escape for fear of prosecution. Britain's only surviving native species, the wild cat, is confined to Scotland. After examining hair samples, experts now say that the Best of Exmoor in the south of England is without (19) _______ a puma or lynx, both of which are normally native (20) _______ the Middle East and Asia. IV. WORD FORMS Part 1: Complete the sentences with the correct forms of the given words. 1. His _______ annoys some members of the committee because he is always expressing his ideas frankly. (speak) 2. George is very disorganized and not very_______ (business) 3. She was fully aware of her own _______ . (short) 4. Low income and little administrative support make teachers _______ _______ with their profession. (heart) 5. A film about the _______ ancestors is available in the library. (reptile) 6. There are people whose _______ begins from the moment of their death. (mortal) 7. It's undeniable that the _______ of the local incompetent healer was responsible for her sudden death. (diagnose) 8. As the sole _______ of his uncle's will, he inherited a huge fortune. (benefit) 9. This _______ faulty washing machine should be returned to the manufacturer. (repair) 10. The first time I tried out my new bike I _______ and fell off. (balance) Part 2: Complete the following passage with the correct forms of the given words. PROFESSOR ---------- SALE ---------- ACQUISITION ---------- STARRY ---------BAND INVADE---------- ORIGIN---------- LONELY---------- QUARTER ---------POPULARITY The Beatles became the most popular grouping rock music history. The (11) _______ of extraordinarily talented musicians generated a frenzy that transcended countries and economic strata. While all of them sang, John Lennon and Paul McCartney wrote the majority of their songs. (12) _______ , Lennon and five others formed a group called Quarrymen in 1956, with McCartney joining them later that year. George Harrison, John Lennon and Paul McCartney, together with Stuart Sutcliffe, who played the bass guitar, and Pete Best on the drums, performed together in several bands for a few years, until they finally settled on the Silver Beatles in 1960. American Rock musicians, such as Chuck Berry and Elvis Presley, influenced Lennon's and McCartney's music, whose first hits consisted of simple tunes and lyrics about young love. The Beatles' U.S. tour propelled them to (13) _______ and led to two movies filmed in 1964 and 1965. The so-called British (14) _______ of the United States was in full swing when they took the top five spots on the singles' charts, followed by the release of their first film. During the 1960s, their music matured and (15) _______ a sense of melody. The lyrics of their song became deeper and gained in both imagination and meaning. Their popularity continued to grow as the Beatles turned their attention to social problems and political issues in "Nowhere Man" and Eleanor Rigby". (16) _______ and nostalgia come through in their ballad "Michelle" and 'Yesterday", which fully displayed the group's (17)_______ development and sophistication. Lennon's sardonic music with lyrics written in the first person, and McCartney's songs that created scenarios with off beat individuals, contributed to the character of the music produced by the group. In addition to their music, the Beatles set a social trend that (18)_______ long hair, Indian music, and moss dress. For a variety of reasons, the musicians began to drift apart, and their last concert took place in San Francisco in 1966. The newspapers and tabloids publicized their quarrels and lawsuits, and the much idolized group finally (19)_______ in 1970. However, their albums had (20) _______ those of any other bands in history. Although all of the Beatles continued to performed solo or form new rock groups alone, none could achieve the recognition and success that they had been able to win together. V. SENTENCE TRANSFORMATION Rewrite the sentences with the given words or beginning in such a way that their meanings remain unchanged. 1. The only reason the party was a success was that a famous film star attended. → Had it …………………………………………………………………………………................... ... 2. Government guidelines really do emphasize the importance of starting education early. → A lot of emphasis ………………………………………………………………………………….... 3. The trip was so amazing that we will never forget it. → It's too …………………………………………………………………………………................... .. 4. Your silly questions distracted me. → You drove …………………………………………………………………………………............... 5. She was so disgusted at the way her friend behaved that she refused to speak to him. → Such …………………………………………………………………………………................... ..... 6. Mike is never reluctant to make tough decisions as a manager. (SHRINKS) → …………………………………………………………………………………................... ............. 7. The film is similar to Shakespeare's Hamlet in a number of ways. (RESEMBLANCE) → The film …………………………………………………………………………………................. 8. He is determined to become a doctor. (HEART) → …………………………………………………………………………………................... ............. 9. Alison bought the big house because she wanted to open a hotel. (VIEW) → Alison bought ….………………………………………………………………………………….... 10. We feel uncomfortable in the house. (FISH) → …………………………………………………………………………………................... ............. VI. ERROR IDENTIFICATION Identify 10 errors in the following passage and correct them. Between 1977 and 1981, three groups of American women, numbered 27 in all, between the age of 35 and 65, were given month-long tests to determine how they would response to conditions resembling those on the space shuttle. Carefully selected from among many applicants, the women were volunteers and pay was barely above the minimum wage. They weren't allowed to smoke or drink alcohol during the tests, and they were expected to tolerate each other's company at closed quarters for the entire period. Among other things, they had to stand pressure three times of the force of gravity and carry out both physical and mental tasks while exhausted from strenuous physical exercises. At the end often days, they had to spend a further twenty days absolutely confined to bed, during that time they suffered backaches and discomforts, and when they were finally allowed up, the more physically active women were especially subject to pains due to a light calcium loss. Results of the tests suggest that women have significant advantages on men in space. They need less food and less oxygen and they stand up to radiation well. Men's advantages in terms of strength and stamina, meanwhile, are virtually wiped out by the zero-gravity condition in space. THE END KỲ THI CHỌN HỌC SINH GIỎI LỚP 10 NĂM HỌC 2011- 2012 Môn thi: TIẾNG ANH Thời gian: 150 phút (không kể thời gian giao đề) (Đề thi này gồm có 4 trang, thang điểm 20) Họ và tên thí sinh:…………………………..................................Lớp: 10A……... Điểm bài thi Bằng số Bằng chữ Họ tên, chữ ký của giám khảo 1............................................................................. PART I : PHONETIC (1.0 p): I. Find the word which has the underlined part pronounced differently from the others. B. hard C. parent D. Park 1. A.pardon 2. A.rose B. cover C. nose D. over 3. A. increase B. ink C. pink D. thank 4. A. beloved B. naked C. ploughed D. learned PART II : VOCABULARY AND GRAMMAR: I. Circle the correct answers (3,5p): 1. He got an excellent grade in his examination ______ the fact that he had not worked particularly hard . A. on account of B. because C. in spite of D. although 2. There were over 30,000 __________ at the match. A. spectators B. viewers C. witnesses D. watchers 3. ____ they are tropical birds, parrots can live in temperate or even cold climate. A. Despite B. Even though C. Nevertheless D. But 4. I was enjoying my book, but I stopped ___ a program on TV. A. to read to watch B. to read for watching C. reading for watching D. reading to watch 5. The man __________ we met yesterday was the manager of a bicycle factory. A. whose B. which C. whom D. when 6. I don't believe a word she says, I think she ___________. A. is laying B. is telling lie C. is lying D. lied 7. AIDS is a(n) ___________ disease. A. endanger B. dangerous C. danger D. endangered 8. I knew they were talking about me _________ they stopped when I entered the room. A. so that B. therefore C. because D. despite 9. When I came to visit her last night, she __________ a bath. A. was having B. has C. is having D. had 10. English ___________ in many parts of the world. A. is spoken B. is speaking C. was spoken D. speaks 11. The children ___________ to the zoo. A. were enjoyed taking B. were enjoyed taken 1 C. enjoyed taking D. enjoyed being taken 12. My parents first __________ each other at the Olympic Games in 1982. A. meet B. met C. had meet D. have met 13.Their ___________ has lasted for more than 20 years. A. friends B. friendship C. friend D. friendly 14. If I had time, I __________ to the beach with you this weekend. A. would have gone B. would go C. will have gone D. will go II. Supply the correct form of the words and verbs in the parentheses ( 2.0 p) 1. Visitors complained about the ………………..in the old museum. (organize) (care) 2. You must be ....................when you open that door . 3."Time of our lives" was the ........................song of the World Cup 2006. (office) 4. When we took an exam last year, we met a very strict ............................. (exam) 5. We can see many ……………………….. on TV everyday . (advertise) 6. I watch the news everyday because it very ………………………….. (inform) 7. He was very …………………………..of the work he had done . (pride) 8. I don’t find him very _______. (communicate) PART III : READING COMPREHENSION: I. Read the passage, and then choose the correct answer (3.0p): All over the world people listen to classical music. Classical music is difficult to describe. It means different things to different people. Some famous classical composers were Bach, Vivaldi, Haydn, and Mozart. In their music, they did not tell a story or show strong emotion. They wanted to make a beautiful, interesting design. They wanted to write lovely sounds. Then composers started to interpret ideas. They told stories about wars, armies, and soldiers. They wrote about religion. Sometimes they composed music for holidays. They told love stories and showed strong emotion. Some of these composers were Beethoven, Schumann, Chopin, Mendelssohn, Wagner, and Tchaikovsky. Classical music stays with people a long time. Bach wrote about 300 years ago, Beethoven wrote about 200 years ago, and Tchaikovsky wrote over 100 years ago. Sometimes people close their eyes to listen to classical music. When they close their eyes, they can see the design. They can listen to the same classical music many times and enjoy it. Sometimes it is difficult to understand. The listener has to think about it. However, we can all learn to enjoy some classical music. It is very important to people. 1. Classical music is famous ________. A. in the Western B. in Europe C. in the United States D. all over the world 2. The first classical composers wanted to ________. A. tell stories about religion B. write lovely sounds C. show strong emotion D. b and c 3. What kind of stories did some composers not tell in their music? A. love B. wars C. religion D. racial integration 4. Beethoven _____________________. A. composed his music a long time ago B. just wanted to make an interesting design C. did not tell a story. D. wrote some music that as difficult to understand 5. People sometimes close their eyes when they listen to classical music in order to ________. A. understand it clearly B. think about the design C. from the picture of the design in their mind D. hear the same classical music many times 6. Classical music __________________. A. is different from people to people 2 B. is difficult to understand so that listener always has to think about it C. was composed by famous musicians very long time ago D. is necessary for people because it makes life more colorful II. Choose the word or phrase which best fits each gap of the passage (2.5p) Yellowstone National park, the first US national park, was (1)_____ in 1872. It is one of the largest parks. It (2)______ about 3,500 square (3)_____or 9,065 square kilometers (4)______ northwest Wyoming and parts of Idaho and Montana. The park has many wild animals (5)______bears, buffalo, elk, deer, antelope coyotes and lynxes and is famous for fine scenery, hot springs and geysers. there are about seventy geysers in the park. 1. A. started B. introduced C. constructed D. established 2. A. covers B. involves C. spreads D. contains 3. A. lands B. feet C. miles D. distances 4. A. of B. in C. at D. from 5. A. adding B. including C. containing D. consisting PART IV : WRITING: I. There is a mistake in the underlined parts of each sentence. Identify your answer by circling the corresponding letter A, B, C or D: ( 1,5 p) 1. My brother hasn’t played football for last year. A B C D 2. I am fond with jogging every morning . A B C D 3. My brother said me to sit down at the table and do my homework . A B C D 4. My father prefers watching films at home than going to the cinema . A B C D 5. I’d like going to the cinema with my friends. A B C D 6. Jane asked me how I go to school everyday. A B C D II. Finish each of the sentences in such a way that it means exactly the same as the sentence printed before it. ( 3.5 p) 1. I couldn’t do the test because it was difficult. Because of …………………………………………………………………………………… 2. Although he got up early, he was late for the first train. In spite ……………………………………………………………………………………………… 3.Mr. Smith said, “I will refuse their offer.” ………………………………………………………………………………………. 4. Henry said, “I can meet them later.” ………………………………………………………………………………………. 5. I have to work late tomorrow, so I can’t join your party If……………………………………………………………………………………………… 6. When I came she was reading newspapers. (change into passive voice) .............................................................................................................................. 7. Are you writing a letter to her ? (change into passive voice) .............................................................................................................................. 3 III. Write a short paragraph (about 120 words) about the following topic: “What do you like to do in your free time” ……………………………………………………………………………………………………………… ……………………………………………………………………………………………………………… ……………………………………………………………………………………………………………… ……………………………………………………………………………………………………………… ……………………………………………………………………………………………………………… ……………………………………………………………………………………………………………… ……………………………………………………………………………………………………………… ……………………………………………………………………………………………………………… ……………………………………………………………………………………………………………… ……………………………………………………………………………………………………………… ……………………………………………………………………………………………………………… ……………………………………………………………………………………………………………… ……………………………………………………………………………………………………………… ………………………………………………………………………………………………………………. the end 4 ĐỀ THI THỬ HỌC SINH GIỎI ĐỀ CHÍNH THỨC (Đề thi gồm 4 trang) NĂM HỌC 2013 - 2014 MÔN: Tiếng Anh – Khối 10 Thời gian làm bài: 150 phút, không kể thời gian giao đề I. PHONOLOGY: A. Choose the word whose underlined part is pronounced differently: B. nib C. absorb D. absent 1. A. debt 2. A.ceased B. decreased C. caused D. promised B. voyage C. massage D. dosage 3. A. carriage B. Pick out the word that stressed differntly from the others: 4. A. marketing B. ambitious C. fashionable D. immigrant 5. A. cucumber B. powerful C. strawberry D. appointment II. VOCABULARY AND GRAMMAR: A. Choose the best answer A, B, C or D: 1. “I can’t remember us ever ___________,” replied the stranger. A. having met B. to meet C. being met D. to have met 2. Listen to what I am saying, __________? A. would you B. will you C. don’t you D. do you 3. It’s high time you ___________to look after yourself. A. learned B. had learned C. are learning D. would learn 4. “Please, will you just tidy your room, and stop ______ excuses!” A. doing B. having C. taking D. making 5. We saw many soldiers and tanks _____________were moving to the front. A. that B. which C. who D. whom 6. He is in a much________ mood than usual. A. best B. better C. good D. well 7. His father was working _______ at his desk. A. industry B. industrial C. industrious D. industriously 8. Last Sunday was _________ that we decided to go on a picnic. A. so beautiful a day B. such beautiful a day C. so beautiful day D. such beautiful day 9. Living organisms contain more water _______ substance. A. than do any other B. other than do they any C. than any other D. does than any other 10. - "Our team has just won the last football match." - "______" A. Good idea. Thanks for the news. B. Yes. I guess it's very good. C. Well, that's very surprising! D. Yes, it's our pleasure. 11. - “Is it all right if I use your bike?” - “______” A. Sure, go ahead. B. Oh, forget it. C. I don’t care. D. I accept it. 12. ______________ of the students in my class could solve the problem yesterday. A. Not much B. Either C. None D. Neither 13. Don’t touch that wire or you’ll get an electric ______. A. charge B. fire C. shock D. current 14. You need more exercise - you should ______ jogging. A. try on B. hold up C. carry out D. take up 15. “Don’t worry. I have ______ tire at the back of my car.” A. other B. another C. the other D. others B. Complete the following sentences with the appropriate forms of the words in brackets: 1. As a child I had an ................................ friend called Polly (imagine) 2. Not ........................ on such a hot day, the beach was crowded. (surprise) 3. Street noise is one of the ....................... of living in a big city. (advantage) 4. The end of the film really disappointed me. It was totally .................... (believe) 5. This word is very difficult to say. I always ......................... it. (pronunciation) 6. We like Mary. She’s very nice and ............................ (lady) 7. Solphins, ................, species, sometimes jump above the surface of the water. (mammal) 8. The islands have been ............................ by the growth of tourism. (west) 9. There will be no pay rises in the ......................... future. (see) 10. Good friends can enrich the quality of our lives ........................ (measure) C. Supply the correct form of the verbs in brackets: 1. The building (finish) by the year 2020. 2. The last time he saw in public he (wear) a grey suit. 3. I realize I (not say) that. That all right I have forgiven. 4. It’s a waste of time (recommend) him to start his new life in New York. 5. He (follow) my advice before, he (not be) in dangerous now. 6. The small room (fill) with boxes of different sizes which (deliver) the day before. 7. During the winter I decided that I (grow) tomatoes and carrots when the summer came. 8. No sooner he (arrive) than he had to leave again. II. READING: A. Read the passage and use only one suitable word to fill in each gap: In a village on the east coast of Scotland, people were waiting for news. Two of fishingboats had been caught in the storm which had blown up during the night. In the cottages round the harbor people stood by their doors (1).............. worried to talk. The rest of the fishing fleet had (2)............ the harbor before dark, and the men from these ships waited and watched with the wives and families of the missing men. Some had (3)............ thick blankets and some flasks of hot drinks, knowing that the men (4)............ be cold and tired. When dawn began to break over in the east, a small point of light was (5)........... in the darkness of the water and a few minutes later, (6).......... was a shout. (7)................ long, the two boats were turning in, past the lighthouse, to the inside of the harbor. The men (8)............ helped out of their boats, and (9)............. they were stiff (10)............... cold and tiredness, they were all safe. B. Choose the words or phrases that best fit the blanks to make a complete passage: The Statue of Liberty is a monumental sculpture that (1).........freedom throughout the world. Its formal name is Liberty Enlightening the World. The statue (2).........a woman escaping the chains of tyranny, which lie at her feet. Her right hand (3)..........aloft a burning torch that represents liberty. Her left hand holds a tablet inscribed (4) .......... the date "July 4, 1776", the day the United States declared its independence. She is (5).........flowing robes and the seven rays of her spiked crown symbolize the seven seas and continents. The Statue of Liberty was (6) ........... in 1886. It weighs 225 tons and is 301 feet (7) ........... It was a gift to the United States from the people of France. Over the years France and the United States had (8) ...........special relationship. In 1776 France helped the American colonies (9) ............independence from England. The French wanted to (10).........something special for the U.S. centennial, its 100th birthday. 1. A. symbolizes B. illustrates C. shows D. proves 2. A. makes up B. depicts C. draws D. paints 3. A. contains B. grabs C. seizes D. holds 4. A. onto B. into C. with D. in 5. A. putting on B. donning C. wearing D. dressing 6. A. put up B. put off C. put on D. put out 7. A. tall B. height C. in high D. aloft 8. A. the B. a C. an D. Ø 9. A. win B. gain C. achieve D. accomplish 10. A. form B. create C. make D. do C. Read the following passage and answer the questions. The world's oceans are so vast that they can cope with the present levels of pollution. However, little is known about the long-term effects of such slow poisoning. The most serious problem of modern time is that man is destroying the earth's natural resources and transforming huge areas into waste land. As a result, it is becoming extremely difficult to grow enough to feed the world's rapidly increasing population. A way of protecting all wild life on the earth must also be found as many species are in danger of disappearing completely from the face of the earth. The smoke in the atmosphere, for example, is increasing so much that the amount of sunlight has been reduced in many cities. Man's whole environment is being changed in a serious way. 1. What is the process of making something dirty ? 2. Find a word or phrase from the passage with the same meaning as the air, water and land in which we live? 3. What is the air surrounding the earth called ? 4. What could be best replaced wealth, goods or products people can use ? 5. What is a difficulty which needs attention and thought ? III. WRITING: A. Rewrite each sentence, beginning as shown, so that the meaning stays the same: 1. It is believed that the man escaped in a stolen car. The man is ..................................................................................... 2.The train journey from London to Bristol takes two hours. It is ................................................................................................. 3. I don’t really like her, even though I admire her achievements. Much as .......................................................................................... 4. It’s a long time since I last went to a football match. I haven’t........................................................................................... 5. Mary doesn’t realise how serious her illness is. Little ................................................................................................ 6. Your car might break down on the rough mountain road, so take plenty of spare parts. In case .............................................................................................. 7. Is it true that you haven’t found a job yet? Are you still .....................................................................................? 8. There is absolutely no truth in that rumour. That ..................................................................................................... 9. In this company it’s John who handles orders for new stock. John is responsible .............................................................................. 10. I wish Jake hadn’t spent so much money on a new set of golf-clubs. I’m annoyed ........................................................................................ B. Imagine you were Ba, and your penpal, Nam, has invited you to visit his city in England. Write a letter (about 120-150 words) to him, in your letter you should: - apologize - explain why you can’t go - invite him to visit your hometown. Dear Nam, ………………………………………………………………………………………………… ………………………………………………………………………………………………… Your friends, Ba THE END Lưu ý: Giám thị coi thi không giải thích gì thêm. Thí sinh làm bài vào tờ giấy thi ĐÁP ÁN ĐỀ THI CHỌN HSG CẤP TRƯỜNG NĂM HỌC 2013 - 2014 MÔN TIẾNG ANH LỚP 10 (Đáp án gồm 1 trang) I. PHONOLOGY: A. Choose the word whose underlined part is pronounced differently: 1. A 2. C 3. C B. Pick out the word that stressed differntly from the others: 4. B 5. D II. VOCABULARY AND GRAMMAR: A. Choose the best answer A, B, C or D: B. Complete the following sentences with the appropriate forms of the words in brackets: 1. imaginary 2. surprisingly 3. disadvantages 4. unbelievable 5. mispronounce 6. ladylike 7. mammalian 8. westernized 9. foreseeable 10. immeasurably C. Supply the correct form of the verbs in brackets: 1. will have been built 2. was wearning 3. shouldn’t have said 4. to recommend 5. Had he followed/ wouldn’t be 6. was filled/ had been delivered 7. would grow 8. did he arrive II. READING: A. Read the passage and use only one suitable word to fill in each gap: 1. too 2. reached/entered 3. brought/got 4. would 5. seen/ visible 6. there 7. Before 8. were 9. although/ though 10. with/ from B. Choose the words or phrases that best fit the blanks to make a complete passage: 1. A 2. B 3. D 4. C 5. C 6. A 7. A 8. B 9. B 10. D C. Read the following passage and answer the questions. 1. The process of making something dirty is pollution / hoặc trả lời ngắn: pollution. 2. A word or phrase with the same meaning as the air, water and land we live is environment / hoặc trả lời ngắn: environment. 3. The air surrounding the earth is called atmosphere/ hoặc trả lời ngắn: atmosphere. 4. Resources could be best replaced wealth, goods or products people can use / hoặc trả lời ngắn: Resources. 5. A difficulty which needs attention and thought is problem / hoặc trả lời ngắn: problem. III. WRITING: A. Rewrite each sentence, beginning as shown, so that the meaning stays the same: 1. The man is believed to have been escaped in a stolen car. 2. It is a two-hour train journey from London to Bristol. 3. Much as I admire her achievements, I don’t really like her. 4. I haven’t been to a football match for a long time. 5. Little does Mary realise how serious her illness is. 6. In case your car breaks down on the rough mountain road, take plenty of spare parts. 7. Are you still looking for a job?/ Are you still unemployed? 8. That rumour is absolutely false. 9. John is responsible for handling orders for new stock. 10. I’m annoyed at Jake for spending / having spent so much money on a new set of golf-clubs. B. KỲ THI CHỌN HỌC SINH GIỎI NĂM HỌC 2011- 2012 MÔN THI: TIẾNG ANH lỚP 10 THPT Thời gian 120 phút (không kể thời gian giao nhận đề) Điểm bài thi (bằng số và chữ) Giám khảo (Ký và ghi rõ họ tên) Số báo danh (Thí sinh ghi) Số phách (Hội đồng thi ghi) I. SECTION A – PHONETICS Choose the word whose underlined part is pronounced differently from that of the others. 1. 2. 3. 4. 5. A. adventure A. increase A. coughed A. realize A. treated B. future B. widespread B. weighed B. teacher B. asked C. mature C. death C. laughed C. reason C. sacred D. figure D. residential D. photographed D. feature D. suggested Your Answer: 1. 2. 3. 4. 5. II. Identify the word whose stressed pattern is different from that of the others. 1. 2. 3. 4. 5. A. impossibly A. damage A. recognize A. electrician A. comedy B. especially B. pollute B. enemy B. comfortable B. collection C. naturally C. defense C. yesterday C. manufacture C. comical D. importantly D. erode D. responding D. accidental D. calculate Your Answer: 1. I. 1. 2. 3. 4. 5. 6. 7. 8. 2. 3. 4. 5. SECTION B – VOCABULARY AND GRAMMAR Choose the best answer from A, B, C or D. Hurry up! They’ve only got _______ seats left. A. a little B. a few C. a lot of D. plenty of The children _______ to the zoo. A. were enjoyed taken B. enjoyed being taken C. were enjoyed taking D. enjoyed taking I'd rather you _______ anyone what I said. A. don't tell B. won't tell C. didn't tell D. not to tell We were so late that we ______ had time to catch the train. A. nearly B. almost C. hardly D. simply Before the meeting finished, they had arranged when ______ next. A. they met B. they to meet C. to meet D. should they meet Washing machines, dishwashers and vacuum cleaners are _______. A. working machines B. useless machines C. sewing machines D. labour-saving machines Both she and her husband are _______ work. A. out of B. for C. on D. in Tom is having someone _______ the newspaper to her. 1 A. bring B. to bring C. bringing They were just _______ us about Anna's new boyfriend. A. talking B. saying C. speaking 10. You've got to try to keep _______ as you grow older. A. act B. active C. action Your Answer: D. who brings 9. D. telling D. actor 1. 2. 3. 4. 5. 6. 7. 8. 9. 10. II. There is one mistake in each of the following sentences. Find and correct it. 1. The sick needs to be looked after. So money must be spent on hospitals. 2. The accident looked seriously at first but nobody was injured. 3. All the students are looking forward to spending their free time to enjoy their Tet holiday. 4. My family lived in Hue since 1990 to 1996, but we are now living in Sai Gon. 5. She left her job soon to devote more time into her family. 6. We won't mind your being late. Beside, it's hardly your mistake. 7. Congratulations! You’ve gained the first prize in the competitions! 8. Her well-known film, that won several awards, was about the life of Lenin. 9. Mr. Tam, who has a lot teaching skills at junior level, will be joining our school in August. 10. You've seen her new car, haven't you? What does it like? Your Answer: Mistake Correction Mistake 1. 6. 2. 7. 3. 8. 4. 9. 5. 10. Correction III. Fill in the each gap with the most correct forms of the block word on the right. MYSTERY BROAD DECIDE INVITE ECONOMY PERSON 1. He is behaving in a very _______ way. 2. Marry wants to _______ her knowledge of the subject. 3. His _______ to retire surprised all of the workers. 4. Some people _______ didn't come. 5. It is _______ to turn off the lights when it is bright enough. 6. My _______ opinion is that the students should be doing more work outside the classroom. 7. These workers got a bonus because they did their jobs _______. 8. From 1865 to 1875, a remarkable _______ of inventions was produced. 9. She received a lot of _______ from her mother. 10. He doesn‘t agree with the _______ that there is life on other planets. EFFECT VARIOUS COURAGE BELIEVE Your Answer: 1. 2. 3. 4. 5. 6. 7. 8. 9. 10. SECTION C – READING 2 I. Read the following passage, and fill in the numbered gaps with the correct options (from A, B, C , D and E below) to make a meaningful reading: Animals live everywhere. (1) _______. They burrow in the ground. They swim in the sea. They fly through the air. They creep, they leap, they soar, and they dive. A very few - including corals and barnacles -stay in one place. (2) _______. The biggest animals are whales, which can be 100 feet (30 meters) long. The smallest animals can only be seen through a microscope. Zoologists (scientists who study animals) have found more than 2 million species of animals. (3) _______. Several things make animals different from other living things. (4) _______. Animals eat other living things - plants and other animals - to get energy. Animal bodies are made up of more than one cell, unlike bacteria and other life forms with only one cell. Cells are the building blocks of living things. (5) _______. A. Unlike plants, animals cannot make their own food. B. They think they have discovered only a small portion of all animals on Earth. C. Animals also have senses, such as eyes or ears, that tell them what is going on around them. D. They roam the land. E. Animals come in all sizes. Your Answer: 1. 2. 3. 4. 5. II. Fill in each numbered gap with one suitable word. Conservation, sustainable use and protection of natural resources including plants, animals, mineral deposits, soils, clean water, clean air, and fossil fuels (1) _______ as coal, petroleum, and natural gas. (2)_______ resources are grouped into two categories, renewable and (3) _______. A renewable resource is one that may be replaced over time by natural processes, such as fish populations or natural vegetation, or is inexhaustible, such as solar energy. The goal of renewable resource conservation is to ensure (4) _______ such resources are not consumed faster (5) _______ they are replaced. Nonrenewable resources are those in limited supply that cannot be replaced (6) _______ can be replaced only over extremely long periods of time. Nonrenewable resources (7) _______ fossil fuels and mineral deposits, such as iron ore and gold ore. Conservation activities for nonrenewable resources focus (8) _______ maintaining an adequate supply of these resources well into the future. Natural resources are conserved for their biological, economic, and recreational values, as well as their natural beauty and importance to local cultures. For (9) _______, tropical rain forests are protected for their important role in both global ecology (10) _______ the economic livelihood of the local culture; a coral reef may be protected for its recreational value for scuba divers; and a scenic river may be protected for its natural beauty... Your Answer: 1. 2. 3. 4. 5. 6. 7. 8. 9. 10. SECTION D – WRITING I. Complete the second sentence so that it has similar meaning to the first one. 1. The painters painted his house last week. He ................................................................................................................................................. 2. Gerald is too young to vote. Geral is not .................................................................................................................................... 3. Andrew went to the supermarket despite the heavy rain 3 Although ........................................................................................................................................ 4. They made him wait for two hours. He was .......................................................................................................................................... 5. David went home before I arrived. When I arrived, .............................................................................................................................. 6. The station clock showed half past ten. According …………...................................................................................................................... 7. I’m always nervous when I take exams. Taking ............................................................................................................................................ 8. When did you buy your laptop? How long ......................................................................................................................................? 9. It is not worth asking Ms. Hoa to help. There’s no ...................................................................................................................................... 10. If people drive faster, it is more dangerous. The ................................................................................................................................................ II. Make right sentences using the following suggestions. 1. Mike/ offer/ opportunity/ study abroad/ yesterday. ....................................................................................................................................................... 2. Why/ you/ not/ consider/ visit/ me? ....................................................................................................................................................... 3. Many species/ plants/ animals/ danger/extinction. ....................................................................................................................................................... 4. If we/ not/ protect/ wildlife now/ there/ nothing / left/ future generations. ....................................................................................................................................................... 5. Remember/ call/ home/ as soon/you arrive/ your destination. ....................................................................................................................................................... III. Write about 120 – 150 words on the following topic: Imagine you had a friend living abroad and not having been back to the country for along time. Write to your friend to tell about the changes and invite him/her to come back to to visit the home towm/ home village. (Notice: Your name is Phuong Lan) ……………………………………………………………………………………………………………… ……………………………………………....................................................................................................... ....................................................................................................................................................... ……………………………………………………………………………………………………………… …………………………………………………………………………………………………… The end 4 Sở GD & ĐT Hà Nội Trường THPT Phú Xuyên B ĐỀ THI HỌC SINH GIỎI KHỐI 10 NĂM HỌC 2011 – 2012 Môn thi: Tiếng Anh Thời gian: 150 phút Họ và tên thí sinh:………………………………………. Lớp: …………………… SBD:............................... __________________________________________________________________ I. PHẦN TRẮC NGHIỆM Question 1: Pick out the word whose underlined part is pronounced differently 1. A. technology B. different C. interact D. system 2. A. site B. kilo C. write D. my 3. A. good B. foot C. noon D. flood 4. A. modern B. box C. shop D. computer Question 2: Read the text and do the task that follow Camping is an activity in which live temporarily in the outdoors. Campers (5)………. fishing, hunting, swimming, plant study, wildlife watching, and nature photography. It provides physical benefits when it involves hiking to, from and around campsite, and many people believe that camping (6) …………….. youngsters more confident. People throughout the world enjoy hiking to wilderness campsites. This activity is called backpacking because it involves carrying such essential camping gears as (7)…………. tent, food, clothing, and sleeping bag on the back in a bag called a backpack. Backpacking is suited for (8)……… who are in good physical conditions as it may require walking several miles. 5. A. participate in B. take place C. take part D. enter 6. A. enables B. makes C. allows D. causes 7. A. the B. a C. an D. some 8. A. those B. anyone C. someone D. everyone Question 3: Choose the best answer from the four options to complete the sentences below 9. I don’t think Frank is old ….. to stay at home alone. A. so B. too C. enough D. even 10. …… all our careful plans, a lot of things went wrong. A. Because B. Because of C. Although D. In spite of 11. He was accused of ……. a motorbike. A. steal B. to steal C. stealing D. to stealing 12. He …. for London one year ago. A. left B. has left C. leaves D. had left 13. If you see Tom, ….. you mind ….. him to get in touch with me? A. will/remind B. would/remind C. will/ to remind D. will/ reminding 14. He was made ….. for two hours. A. wait B. to wait C. waited D. waiting 15. We don’t allow people …… in this room. A. smoke B. smoking C. to smoke D. smokes 16. I will introduce you to the girl ….. I live with. A. who B. whom C. which D. when 17. When I was a child, I used to ……. in that house. A. live B. to live C. living D. to living 18. Music ….. an important part in our lives. A. takes B. participates C. plays D. grows 19. The girl and the animals ….. he painted were vivid. A. which B. who C. whom D. that 20. When we arrived, the train ….. already….. A. has/left B. had/left C. have/left D. left II. PHẦN TỰ LUẬN 5 Question 4 : Read the text and fill in the form below Tran Ha is a Vietnamese high school student. He was born on January 12, 1990 in Ha Noi. He studies English because he wants to win a scholarship to study abroad. He is living with his family at 18 Le Loi Street, Hoan Kiem District, Ha Noi. His telephone number is 04-8232043. IGE LANGUAGE CENTER Registration Form 21. Name:…………………………………………. 22. Date of birth:………………………………….. 23. Course to register:…………………………….. 24. Occupation:…………………………………… 25. Address:……………………………………….. Question 5: Arrange the following sentences to make a confirmation letter 26. I’m happy to join your birthday party this Saturday. 27. I’ll buy all the things you need and bring them to your house an hour before the party. 28. Of course. I’ll help you prepare everything for the party. 29. Mai. 30. See you later 31. Dear Lan, Answers:………………………………………………………….. Question 6: Find one mistake in each sentence and correct them 32. I spend most of my free time to read books. 33. If I didn’t liked you, I wouldn’t invite you to my house. 34. I have learned English for 1999. 35. In spite interruption, she was still able to finish her assignment before class. 36. Mary is very good about telling funny stories. 37. It is not until his father came home which he did his homework. 38. The children are exciting about the field trip. 39. She wants becoming a doctor. 40. It are difficult to learn English. STT Mistakes Corrections 32 33 34 35 36 37 38 39 40 Question 7: Put the verbs into correct forms 41. If I (have) ………… your address, I would send you a postcard. 42. Two millions (not/be) ………… a large number of money. 43. They (show) …………………. More than fifty films in Hanoi since June. 44. It’s already 320C. It (be) ………. very hot. 45. I get used to (live)…………… in the big city. 46. I must remind my students that this grammar point needs (revise)…………… 47. We found it very difficult (reach)…………… a decision. 48. He is very (interest)……………… in my story. 49. Do you think the film is (excite)……………………? 50. Would you mind telling me how (get) …………… to the post office? Question 8: Rewrite the following sentences without changing their meanings. Use the suggestion below. 51. Tom spends a half an hour walking to the bookshop. 6 It takes …………………………………………………….. 52. Since Mary was careless, she lost her job. Because of ………………………………………………… 53. Although he made great efforts, he failed to win the race. In spite of ……………………………………………………… 54. Tim will be eighteen next week. It is Tim’s ……………………………………………………… 55. It is cheaper to go by train than by car. Going by car……………………………………………………. Question 9: Read the passage and answer the questions American Sign Language (ASL), which is the first language of many deaf people in North America, uses hand gestures, facial expressions, and other body movements communicate. ASL is one of the four most commonly used languages in the United States. ASL was first introduced to America in 1817. In that year, a French teacher who came to America to teach, brought with him French Sign Language (FSL) and taught it to his deaf students. The students then added their own American singing and changed the FSL into the ASL. Modern ASL and FSL share some elements, including a large amount of vocabulary. However, an ASL user cannot understand a FSL user. No form of sign language universal. For example, British Sign Language (BSL) differs notably from ASL. Different sign languages are used in different countries and regions. 56. What does ASL stand for? …………………………………………………………………. 57. How did people change the FSL into the ASL? ………………………………………………………………….. 58. How many commonly used languages are there in the United States? ……………………………………………………………………… 59. What does it in paragraph 2 refer? …………………………………………………………………… 60. What is the main idea of this passage? ……………………………………………………………………. Question 10: Use the following sets of words and phrases to write a letter from Jenny to Diana Dear Diana, 61. Thank you/ much/ your letter/ arrive/ few days ago …………………………………………………………………. 62. It/ be lovely/ hear/ you ………………………………………………………………….. 63. I/ be sorry/ I not write/ such/ long time/ because/ I/ very busy …………………………………………………………………… 64. As you know/ we buy/ new house/ September ……………………………………………………………………. 65. It/ be very bad condition/ and it need/ a lot/ work …………………………………………………………………… 66. We finish/ most/ it now/ and it look/ very nice …………………………………………………………………… 67. Peter and I/ decide/ give/ house-warming party/ May 3rd ……………………………………………………………………….. 68. you think/ you/ able/ come? ………………………………………………………………………… 69. Please give me/ ring/ let/ know/ you/ make it …………………………………………………………………………. 70. I / really/ look forward/ see you again. …………………………………………………………………………….. Love, Jenny 7 Question 11: Write a short paragraph (about 120 words) about the advantages and disadvantages of the Internet ………………………………………………………………………………………………………… ………………………………………………………………………………………………………… ………………………………………………………………………………………………………… ………………………………………………………………………………………………………… ………………………………………………………………………………………………………… ………………………………………………………………………………………………………… ………………………………………………………………………………………………………… ………………………………………………………………………………………………………… ………………………………………………………………………………………………………… ………………………………………………………………………………………………………… ………………………………………………………………………………………………………… ………………………………………………………………………………………………………… ………………………………………………………………………………………………………… ………………………………………………………………………………………………………… ………………………………………………………………………………………………………… ………………………………………………………………………………………………………… ………………………………………………………………………………………………………… ………………………………………………………………………………………………………… ………………………………………………………………………………………………………… ………………………………………………………………………………………………………… ………………………………………………………………………………………………………… ………………………………………………………………………………………………………… ………………………………………………………………………………………………………… ………………………………………………………………………………………………………… ………………………………………………………………………………………………………… ………………………………………………………………………………………………………… ………………………………………………………………………………………………………… ………………………………………………………………………………………………………… ………………………………………………………………………………………………………… ………………………………………………………………………………………………………… ………………………………………………………………………………………………………… ………………………………………………………………………………………………………… ………………………………………………………………………………………………………… ………………………………………………………………………………………………………… ………………………………………………………………………………………………………… ………………………………………………………………………………………………………… ………………………………………………………………………………………………………… ………………………………………………………………………………………………………… ………………………………………………………………………………………………………… ………………………………………………………………………………………………………… ………………………………………………………………………………………………………… ………………………………………………………………………………………………………… ………………………………………………………………………………………………………… ………………………………………………………………………………………………………… ………………………………………………………………………………………………………… ………………………………………………………………………………………………………… ………………………………………………………………………………………………………… ………………………………………………………………………………………………………… ………………………………………………………………………………………………………… ………………………………………………………………………………………………………… ………………………………………………………………………………………………………… ………………………………………………………………………………………………………… ………………………………………………………………………………………………………… 8 ………………………………………………………………………………………………………… ………………………………………………………………………………………………………… THE END KỲ THI HỌC SINH GIỎI NĂM 2008 ĐỀ CHÍNH THỨC TRƯỜNG THPT LÊ QUÝ ĐÔN KHÓA NGÀY 13 - 4 - 2008 MÔN TIẾNG ANH - LỚP 10 Thời gian: 150 phút (không kể thời gian giao đề) A. LISTENING : ( 2ms) I. Listen to the tape twice and then choose the best option:(1m) The man was (1)………….. when his little daughter came to him. She was crying because she had lost a coin. The man said to her that money wasn’t important. About a week later, the man (3) ………….. in the lottery. They were all very happy and they had a little party. The man put the wallet with the money on the table, and they were all enjoying themselves. The daughter threw all the money out of the window and explained to the father “ But Daddy, you said (4)…………….” 1. a. at home b. at house c. at hall 2. a. lull b. love c. learn 3. a. wet b. won c. warm 4. a. money is importance d. at horse d. life d. gained b. money is important c. money isn’t importance d. money isn’t important II. Listen to the tape three times and then fill in the gaps with suitable words or phrases (1 m) 1. How far ……………………………………………? 2. ……………… to my house for lunch on Saturday ? 3. ………………..those ear-rings ……………………? 4. …………………………………………………… ? B. VOCABULARY AND GRAMMAR: ( 6 ms) I. Choose the best option:( 2ms) 1. He was offered the job ………….his qualifications were poor a. although b. in spite of c. because of d. because 2. It’s no use …………. a language if you don’t try to speak it a. learned b. to learn c. learn d. learning 3. He …………. me to take a lawyer to the court with me a. advised c. suggested b. threatened d. insisted 4. She came in quietly ……………the baby a. in order to not wake b. so as not to wake 9 c. to not wake d. to wake 5. ……………you have no keys, you ‘ll have to get back before I go out a. If b. Although c. Unless d. As 6. I really prefer just about anything …………..watching TV a. from b. or c. than d. to 7. Gold ……………..near San Francisco in 1848 and the gold rush started the following year a. has been discovered b. discovered c. was discovered d. is discovered 8. I regret ……………. you that we can’t approve your application a. inform b. to inform c. informing d. informed 9. If I had gone to the bank this morning, I …………money from you now a. would not borrow b. would not have borrowed c. will not borrow d. will no have borrowed II. Give the correct from of the words in brackets: ( 2ms) 1. He is engrossed in doing ( SCIENCE) …………….. research 2. They entered the areas without ( PERMIT) …………….. 3. He wants (WIDTH) ……………… his knowledge of the subject 4. The giant panda is a(n) (DANGER) ……………..species 5. What makes the computer a (MIRACLE). …………device ? 6. You may be surprised at the large (VARIED) ……………..of animals in national parks 7. The architecture in the downtown area is a successful (COMBINE) ………….of old and new 8. He came first in the poetry (COMPETED) …………….. III. Give the correct tenses of the verbs in brackets (2ms) Mary (1. have) ……………….to go to New York last week, but she almost (2.miss) ……………the plane. She (3. stand) ………………in the queue at the check-in desk when she suddenly (4. realize) ………….. that she (5. leave) …………….her passport at home. Fortunately, she ( 6.not/ live)………….. very far from the airport so she (7. have)…………..time to go home to get the passport. She (8. get) ……………. back to the airport just in time for her flight C. READING COMPREHENSION: (6ms) I. Read the following passage and choose the best answer (1.5ms): For the last few years, my children have been going to a summer camp in northern Greece called Skouras Camp. They always seem to have a good time. , so if you’ re wondering what to do with the kids for three weeks this summer, you could do worse than send them to this beautiful camp on the shores of the Aegean Sea. If your children, like mine, are keen on adventure, sports and good company, the Skouras Camp will keep them busy all day doing the things they most enjoy. Skouras is an international camp with children from all over the world. My children have made friends with children with of their own age from Poland , China, Denmark and the United States. Naturally they get lots of opportunities to practice their English as this is the only language spoken. The camp is located in one of the most beautiful parts of 10 Chalkidiki. It is huge (120,000 square meters) and is just a stone’s throw away from the clear, blue Aegean Sea. It takes the children just five minutes to walk to the golden sandy beach on foot. The programme is packed with exciting activities such as horse riding and table tennis. Other sports include basketball, volleyball and athletics. The Camps end with a sports contest in the last week which all parents are invited to attend. 1. All the children come to the camp have to ……….. a. be at the same age b. be only keen on adventure c. speak English d. practice basketball 2. How many kinds of sports can be played in the Camp? a. 5 b. 2 c. 3 d. 4 3. All the statements are true EXCEPT ……………… a. The children will be busy taking part in the Camp’s programmed activities b. The Camp is quite far from the Aegean Sea c. The parents can attend their children’s sports contest d. The children will take more chances of English practice 4. The tone of the passage could best be described as ……………. a. negative b. disbelieving c. supportive d. humourous 5. What should be the best title for the passage? a. an international summer camp b. Children’s summer activities c. Advice on children caring in summer d. Chalkidiki’s landscape- the Aegean Sea 6. How long does it take the children to walk to the golden sandy beach on foot? a. an hour b. 20 minutes c. 5 minutes d. a day II. Read the following passage and choose the best option to fill in the gap (2.5 ms) The (1) …………. part in Ioannina is the lake front. Ships fetch more people and tourists to the island. Near the front lake (2) …………….an ancient castle. If you go for a (3) …………. in the night to the front lake you’ll see (4) …………. lights that shines in the castle. A second way is to go around on a mountain. There you can see the whole town. In the night with so many lights, it looks to you (5) ………….a sky. Another beautiful part in this town (6) ……………the large clock in the square. Specifically, the clock (7) ……………… in the center of the city. My town also has many museums. Paul Vrellis, who is a professor, put in many different and other important people from the 1st and 2nd world war with wax. In my opinion, it’s (8)…………… museum. About remains, there are two renowned theaters. There’s the theater of Dodoni, which is situated in the (9) ………… of the city. It’s normally out of the city. Every summer the most (10) ………. actors of the capital of Greece come to perform there. 1. a. more beautiful b. as beautiful c. so beautiful d. most beautiful 11 2. a. there is b. it is c. it has d. there has 3. a. picnic b. walk c. jog d. run 4. a. more brilliant b. as brilliant c. much brilliant d. most brilliant 5. a. from b. as c. like d. to 6. a. be b. are c. is d. to be 7. situated b. is situated c. be situated d. are situated 8. a. the most best b. the better c. the best d. the well 9. a. outskirt b. top c. side d. edge 10. a. famous b. fame c. famed d. known III. Read the passage bellow and fill in the blank with ONE suitable word (2ms) Today in China (1) …………….……large scale destruction of forests has occurred, the government has required that every citizen (2)……………..the age of 11 and 60 plant three to five trees (3) ……………...year or do the equivalent amount of work in other forest services. The government claims that at ( 4) ……………..1000 million trees have been planted in China every year (5) ……………..1982. In Western countries, increasing consumer demand for wood products that have been produced cause forest land-owners and forest industries to become increasingly accountable for their forest management and timber harvesting practices. The Arbor Day Foundation’s Rain Forest Rescue programme is a charity that helps to (6) ………………deforestation. The charity uses money to buy up (7) ……………preserve rainforest land before the lumber companies can buy it. The Arbor Day Foundation then (8)……………. the land from deforestation D. WRITING ( 6ms) I. Finish each of the following sentences in such a way that it means exactly the same as the sentence printed before it (2ms) Example: I haven’t enjoyed myself so much for years Answer : It’s years since I enjoyed myself so much 1. There’s a large restaurant in the art gallery → The art gallery ……………………………… 2. I haven’t seen that man here before. → It’s …………………………………………. 3. Although Bob didn’t speak Dutch, he decided to settle in Amsterdam → In spite of ……………………………………………………………… 4. Without this treatment, the patient have died → If the patient hadn’t ……………………………. 5. The keepers feed the lions at 3 p.m everyday. → The lions …………………………………………. 12 6. John finds astronomy very interesting → John is ……………………………….. 7. The cinema didn’t become an industry until 1915 → It was not ………………………………………... 8. “I like your coat , I’m looking for one like that myself” she said to me → She told me …………………………………………………………… II. Read the text below and look carefully at each line. Some of the lines are correct, and some have a word which should not be there. If a line is correct, put a tick (√) by the number in the space provided. If a line has a word which should not be there, write the word in the space provided. There is an example at the beginning (2ms) The Spy Returns is a very interesting film that about a wealthy 0. that………. man who visits Italy. He doesn’t think that Rome is interesting 1..……………. and he is very bored with there. Then suddenly one night he is 2.…………… very surprised that a beautiful girl who runs up to him and gives 3.…………… him a mysterious letter. From that moment his life is no longer 4…………… boring. He does too a lot of dangerous things. For example, 5..…………… he jumped into a lake to save a famous person. The film is 6.…………… a very thrilling indeed. I have watched it twice but I always 7.…………... want to see it again. I think many people do it so 8.……………. III. Write an essay of 120-150 words expressing the advantages and disadvantages of the Internet (2ms) ĐÁP ÁN ĐỀ THI CHỌN HSG MÔN ANH : 10 A. LISTENING (2 điểm) I. 1 điểm ( mỗi lựa chọn đúng 0.25 đ) 1.a 2. b 3. b 4. d II. 1 điểm ( mỗi câu đúng o.25 đ) 1….. is your from the city centre 2. How about coming … 3. How much do ….cost? 4. How much do you spell your family name B. VOCABUALRY & GRAMMAR: (6 điểm) 13 I. 2 điểm ( mỗi lựa chọn đúng 0.25 đ) 1. a 5.a 2.d 6.d 3.a 7.c 4.b 8.b II. 2 điểm (mỗi từ đúng 0.25 đ) 1. scientific 5. miraculous 2. permission 6. variety 3. widen 7. combination 4. endangered 8. competition III. 2 điểm ( mỗi thì chia đúng được 0.25 đ) 1. had 5. had left 2. missed 6. doesn’t live 3. was standing 7. had 4. realized 8. got C. READING ( 6 điểm) I. 1.5 điểm ( mỗi lựa chọn đúng o.25 đ) 1. c 2. a 3. b 4. c 5. a 6. c II. 2.5 điểm ( mỗi lựa chọn đúng 0.25đ) 1. d 6. a 2. a 7.b 3. b 8. c 4. a 9. d 5. c 10. a III. 2 điểm ( mỗi từ đúng 0.25đ) 1. where 2. between 3. per 4. least 5. since 6. prevent 7. and 14 8. protects D. WRITING: ( 6 điểm) I. 2 điểm ( mỗi câu đúng 0.25 đ) 1. The art gallery has a large restaurant 2. It’s the first time ( that) I have seen this man here 3. In spite of not speaking Dutch, Bob decided to settle in Amsterdam 4. If the patient hadn’t been given this treatment, he would have died 5. The lions are fed by the keepers at p.m everyday 6. John is interested in astronomy 7. It was not until 1915 that the cinema became an industry 8. She told me that she liked my coat and (she) was looking fro one like it/that herself II. 2 điểm ( mỗi câu đúng 0.25 đ) 1. √ 2. with 3. who 4. √ 5. too 6.√ 7. a 8. it III. 2 điểm 15 KÌ THI CHỌN HSG LỚP 10 THPT NĂM HỌC 2010-2011 Đề thi môn :Tiếng Anh (Thời gian làm bài:180 phút) PART A: LISTENING You will hear someone talking about Spring School in HoChiMinh city. Listen to the tape 3 times then fill in the missing information to complete the following sentences. 1. Spring School is an........................... school in Ho Chi Minh city. 2. Around.............................................................................. live and study at the school. 3. About ............................from District 1 regularly attend classes. 4. Organization for Educational Development co-operated with Spring School to set up English classes in................................... . 5. The school requires.............................to help organize their fund –raising dinner held annually in........................... PART B:PHONETICS I. Choose the word which has the underlined part pronounced differently from that of the others. Identify your answer by circling the corressponding letter A,B,C or D. B.composer C.answer D.serious 6. A. classical 7. A. humans B.dreams C.concerts D.songs 8. A. depend B.temperate C.dental D.telephone 9. A. spoon B.wool C.noon D.bamboo C.burial D.turtle 10.A. excursion B.further II.Choose one word whose stress pattern is different.Identify your answer by circling the corresponding letter A,B,C.or D. 11.A.musician B.museum C.competent D.computer 12.A.conversation B.isolation C.traditional D.situation 13.A.capital B.activity C.different D.opera 14.A.affect B.effective C.expand D.charity 15.A.family B.probably C.animal D.minority PART C:VOCABULARY AND GRAMMAR I.Choose the best answer from the four options to complete each sentence below. Identify your answer by circling the corresponding letter A,B,C.or D. 16.My mother is busy.......... her housework. A.at B.with C.on D.in 17.It was ........ a difficult question that we couldn’t answer it. A.so B.such C.very D.too 18.I don’t think Frank is old..........to stay at home alone. A.so B.too C.enough D.even 19.Your bicycle goes much faster than........because it’s a lot lighter. A.mine B.him C.us D.their 20. .......all our careful plans, a lot of things went wrong. A.Although B.Because C.Because of D.in spite of 21.We arrived.......... the station just in time to catch the train. A.at B.in C.to D.for 22.I’d like to introduce you to Mrs. Chi,.........husband you used to work with. A.whom B.whose C.which D.who 23.I like to study sea life .Marine biology is a ................subject. A.fascinated B. boring C.bored D.fascinating 24.You said the test was too difficult,..........? A.weren’t you B. wasn’t it C.didn’t it D. didn’t you 25.You should...... your lesson before the exam. A.read B.write C.revise D.check 26.Three people were seriously .........in the road accident yesterday. A.injured B.broken C.damaged D.pained 16 27.Don’t be too hard on him; he’s doing the job.............. . A.he can as best B.as he can best C.as best he can D.best as he can 28.I phoned him ten minutes ago ,but he wasn’t......... . A.out B.in C.on D.away 29.We have just visited HaLong Bay .It has.......................... A.beautiful scenery that is much B.many landscapes C. many beautiful sceneries D.much beautiful scenery 30.Mary’s house is.......... the hair salon.Do you think you can find it? A.along B.near to C.against D.next to 31.John decided ......... golf at weekends. A.to begin B.to commence C.to take up D.to start up 32.It..................to me whether we meet them or not . A.makes no difference B.makes not a difference C.is indifferent D.is not a difference 33.We saw ............wild animals while on vocation. A.a few B.much C.little D.sum 34.They will be........... at the party. A.present B.presents C.presenting D.at present 35. “Where do you live now?” – “I live in Hanoi; my...................................... A.parents too do . B.parents do. C.parents do,too . D.parents also do. 36. “Why are you driving so fast?” – “I’m................................” A.must be hurry. B.in hurry . C .in a hurry . D.hurrying. 37.She burst ........tears. A.out of B.into C.for D.in 38.He was accused .......... stealing a motorbike. A.in B.of C.to D.with 39. The building.... he lives .... is very old. A.which-in B.where-in C.where-on D.that-within 40.I don’t know ........ about English literature. A.many B.much C.a few D.little 41.He ...... to HoChiMinh city last year and I........him since then. A.moved-didn’t see B.moves-haven’t seen C.moved-haven’t seen D.moved-hadn’t seen 42.Neither the animals nor their keeper.......prepared for the storm. A.to be B.were C.was D.be 43.A number of students...........volunteered to the job. A .having B.has C.to have D.have 44.Everybody was ......... that he passed the exam. A.surprised B.surprising C.surprise D.surprises 45.I’ll let you know if I........out what’s happening. A.find B.finds C.found D.had found II.Give the correct form of the given words to complete the following sentences 46.There are many good ...............................programmes on VTV2. DOCUMENT 47.Newspapers and magazines present information.......................... . VISION 48.It is a good book because it is very............................... . INFORM 49.Lasers can be used to treat .......................... DEAF 50.Watching Wildlife World programmes makes us more RESPONSIBLE aware of our global................................. PART D:READING I.Choose the best answer from the four options to complete each of the numbered blanks in the following passage. Identify your answer by circling the corresponding letter A,B,C.or D. Diego Armando Maradona was born on October 30,1960 ,in Villa Fiorito,province of Buenos Aires, Argentina .He was (51)........... in an underprivileged family and spent his adolescence playing football in the street of his(52)........... . 17 He made his debut at the(53)................ of 16 and throughout his professional career he played 692 (54).............. games scoring 352 goals.He played 90 games for the Argentina National Team and scored 33 goals, 8 of (55).............. were in world cups. His first team was called Estrella Roja (Red Star) and was (56)..........by his father for the neighbourhood boys.One of the players was Gregorio “El Goyo” Carrizo,(57)............ played in the lower divisions of Argentinos Juniors, and who helped Diego to be part of the team called “Los Cebollitas” (the small onions).With only 14 years of age ,he(58)...........the public at his first game with the team and gained (59).......... and respect as a player.With “Los Cebollitas” he (60)........ a winning spree of over one hundred games. 51.A.risen B.raised C.grown D.shown 52.A.neighbourhood B.colleagues C.frienship D.populariy 53.A.old B.oldest C.elderly D.age 54.A.office B.officer C.official D.officially 55.A.it B.that C.which D.what 56.A.found B.founded C.set D.fixed 57.A.who B.that C.he D.whom 58.A.enjoyed B.fascinate C.interested D.astonished 59.A.famous B.reputation C.achievement D.strength 60.A.kept B.played C.held D.napped II.Read the passage below and choose the best answer for the quesions from 61 to 65. Identify your answer by circling the corresponding letter A,B,C.or D. An English man living near Sheffield recently had a most unlucky day’s fishing.He was standing on a low bridge when he had the misfortune to knock his tin of bait into the river.Leaning over the side of the bridge, he tried to hook the tin and pull it out of the river.As he did so,his car keys fell out of his pocket and disappeared in the water. Page 3 Feeling thoroughly annoyed with himself and his luck ,the fisherman leant over the bridge to try to see where his keys had gone.As he did so, the bridge collapsed and he fell into the river. This was the last straw .The fisherman crawled out of the river and went back to his car. There he discovered that he had locked the doors and could not get in. His anti-thief had immobilized his car.There was nothing left for him to do but walk sadly home. 61.How did the tin of bait get into the river? A.The passage does not say. B.The fisherman knocked it in accidentally. C.It was probably blown in by the wind D.It fell in when the fisherman fell in. 62.When did the fisherman first lean over the bridge? A.When it broke. B.When his keys fell in. C.When the tin of bait fell in. D.A few minutes after he first started fishing. 63.What was the last straw, according to the passage? A.Having to walk home B.Being unable to get into his car. C.The loss of his car keys . D.The collapse of the bridge. 64.What is the meaning of “misfortune”? A. bad luck B. good luck C. accidentally D. a clumsy act 65.What is the meaning of “thoroughly”? A. slightly B.at last C.through D.extremely PART E:WRITING I.Finish each of the following sentences in such a way that it is as similar as possible in meaning to the original sentence. Use the word given and other words as necessary.Do not change the form of the given word. 66.He is talking to the girl with long hair. (whom) ...................................................................................................................... 67. “You had better go to see the doctor immediately,” Jone said. (advised) ........................................................................................................................ 68.She liked Paris very little and Rome less. (worse) ................................................................................................................................ 69.I called my uncle for some advice. (order) 18 .................................................................................................................................. 70.He used to get up very late. (habit) .................................................................................................................................... II.Use the following sets of words and phrases to write a letter from Jenny to Diana. Dear Diana, 71.Thank you /much/your letter/arrive/few days ago. ........................................................................................................................................................ 72.It/be lovely/hear/you. ........................................................................................................................................................ 73.I/be sorry/I not write/such /long time/but I/be very busy. ........................................................................................................................................................ 74.As you know/we buy/new house/September. ........................................................................................................................................................ 75.It/be/very bad condition/and it need/alot/work. ........................................................................................................................................................ 76.We finish/most/ it now/and it look/very nice. ........................................................................................................................................................ 77.Peter and I/decide /give/house-warming party/May 3rd ........................................................................................................................................................ 78.You think/you/able/come? 79.Please give me/ring/let/ know /you/make it. ........................................................................................................................................................ 80.I/really/look forward/see you again. ........................................................................................................................................ Love, Jenny The end 19 ĐÁP ÁN ĐỀ THI HSG MÔN TIẾNG ANH LỚP 10 THPT(2010-2011) Tổng số điểm :80/80 câu.Mỗi câu đúng được 1điểm. PART A:(5 điểm) 1. informal 2. 30 street children 3. 250 children(with special difficulties) 4. 1998 5. Volunteers - June PART B:(10 điểm) I 6.B 7.C 8A 9.B 10.C II 11.C 12.C 13.B 14.D 15.D PART C (35điểm) I 16.B 17.B 18.C 19.A 20.D 21.A 22.B 23.D 24.D 25.C 26.A 27.D 28.B 29.D 30.D 31.C 32.A 33.A 34.A 35.C 36.C 37.B 38.B 39.A 40.B 41.C 42.C 43.D 44.A 45.A II 46. documentary 47. visually 48. informative 49. deafness 50. responsibilities PARTD:(15 điểm) I 51.B 52.A 53.D 54.C 55.C 56.B 57.A 58.D 59.B 60.A II 61.B 62.C 63.D 64.A 65.D PART E (15 điểm) I 66. The gỉrl whom he is talking to has long hair. 67.John advised me to go to see the doctor immediately. 68.She thought Rome even worse than Paris. 69.I called my uncle in order to get some advice 70.He was in the habit of getting up very late. II 71.Thank you very much for your letter which arrived a few days ago. 72.It was lovely to hear from you. 73.I’m sorry I have not written for such a long time but I’ve been very busy. 74.As you know , we bought a new house in September. 75.It was in very bad condition and it needed a lot of work. 76.We have finished most of it now and it looks very nice. 77.Peter and I have decided to give a house-warming party on May 3rd 78.Do you think you will be able to come? 79.Please give me a ring and let me know if you can make it. 80.I’m really looking forward to seeing you again. Love, Jenny 20 Vinhphuc Education and Trainer Services Nguyen Viet Xuan High School GIFTED STUDENT SELECTING TEST: GRADE 10 TIME: 180 MINUTES I. Chooses the word that has the underlined part pronounced differently: 1. a. warm b. wash c. wall d. walk 2. a. height b. weight c. vein d. eight 3. a. wood b. soon c. good d. foot 4. a. student b. huge c. communicate d. sudden c. spread d. peasant 5. a. seafood b. threaten II. Choose the words that have different stress pattern: 6. a. cartoon b. comedy c. document d. action 7. ambitious b. delicious c. industrious d. glorious 8. a. reply b. communicate c. subtract d. practice 9. a. demonstration b. atmosphere c. documentary d. engineer 10. a. beautiful b. miraculous c. endangered d. extraordinary III. Supply the correct form of the word in capital. 11. She has one of the biggest …………. in Britain. COLLECT 12. They have added three new songs to the show, which……… by LONG about fifteen minutes 13. He was very ………… of the work he had done PRIDE 14. I spend my ………….. in the country. CHILD 15. I think it is very ……………… of him to expect us to work REASON over time every night this week. III. Put the verbs in brackets in the correct forms of the verbs. 16. (Come)……………. into the room he (see)…………….. Mary where he (leave)……………. her. 17. The last time he (see)……………….. in the public he (wear)……………….. a grey suit. 18. That’s a good novel but I (not read)………………………… it before. 19. I couldn’t risk (leave)…………………… him alone. 20. Good, everybody’s ready. If we (leave)…………… now, we (miss)…………. the rush hour traffic. 21. I (sit)…………………… there for about ten minutes, pretending to read, when someone (sit)…………………. down at my table. 21 22. Their marriage only lasted three months. If he (be)…………………. less mean, she (not leave)…………….……. him. 23. Heavy rain kept us from (go)……………………… to the class on time. 24. I (watch)………………………….. the TV when you called last night. 25. I (not see)…………………….. her since we (leave)……………………… school. IV. Choose the best answer, A, B, C or D. 26. My classmate told me to watch movies in English ………. Learn spoken English better. A. so as B. so that C. in order that D. in order to 27. I’d like to contribute …………. the school Red Cross fund. A. on B. to C. with D. for 28. By the time you receive this letter, I ………………. for Ho Chi Minh city. A. will leave B. will be leaving 29. He retired early A. on behalf of C. have left D. will have left C. believe D. imagine ill – health. B. on account of 30. Amy, ……….. car had broken down, was in a very bad mood. A. who B. whom C. whose D. that 31. The librarian gave us a lot of information, ……………… was very useful. A. this B. most of that C. which most of it D. most of which 32. Last summer, we had ………….. holiday in Spain A. a two week’s B. a two – weeks C. two – week D. a two – week 33. I felt sorry about breaking my friend’s glasses. Afterwards, I offered ……….. for them. A. pay B. to pay C. paying D. to paying 34. Nobody could hear her ………… she spoke too quietly. A. so B. however C. although D. because 35. The Icelandic language …………. very little over the past 800 years. A. is changing B. was changing C. has changed D. changes 36. …………., Thomas refused the water. A. Although he was thirsty B. Despite he was thirsty C. Though being thirsty D. Even though his thirsty 37. I’m learning English ………….. English songs. A. singing B. to singing C. to sing D. for sing 38. His old story made me ………….. A. boreB. boring C. bored D. boredom 39. There will be ………… good film at ………… cinema next week. A. a – a B. a – the C. the – a D. the – the 40. I watch an ………..... film on TV last night. A. interest B. interests C. interesting D. interested 22 41. It was ………….. 2005 that he became an actor. A. until that B. not when C. not until D. until when 42. The paper has ………….. good news today. A. a B. an C. the D. some 43. If you ……….. to me, you wouldn’t have got so much trouble. A. listened B. would listen C. had listened D. would have listened 44. Many sea animals are at …………. due to hunting and water pollution. A. loss B. death C. risk D. danger. 45. Not until he was 16 …………….. English. A. that he learned B. when he learned C. did he learn D. would he learn V. Find the mistake in each of the sentences below: 46. Ms Baker, she spent her life working with the health and welfare of the families of A B C D worker, is an successful woman in the world. 47. I remember to be given a toy drum on my fifth birthday. A B C D 48. She can sing the song by both Vietnamese and English. A B C D 49. You can lead a horse to water but you can't make him drinking. A B C D 50. The breaking glass was still on the floor. A B C D VI. Choose the most suitable word or phrase to fill in each of the gaps in the following passage. Earlier this morning when some classmates and I were [51] our school, it suddenly began to rain [52] basketball in the park near . We immediately grabbed our basketball and ran for shelter. At first we stood together under a tree [53] Peter told us that it was [54] to stay under a tree in a rainstorm, [55] lighting, so we ran to a nearby store to wait From [57] then it began to thunder. [56] the rain stopped. we were standing, we could see that the [58] rain had started, there had been people everywhere but now [59] football or doing [60] or sitting on the park [61] when there is was empty. Before the was playing basketball or reading the morning papers or chatting with their friends. We stood in the [62] of the store for nearly twenty minutes before the rain finally stopped and the sun came [63] was [64] again from behind the clouds. Within a few minutes the park of people again but now the air was cooler and big [65] of rainwater could be seen everywhere. 23 51. A. hitting B. kicking C. playing D. throwing 52. A. greatly B. heavily C. severely D. very big 53. A. and B. so C. but D. yet 54. A. danger B. in danger C. a danger D. dangerous 55. A. specially B. especially C. mostly D. chiefly 56. A. for B. until 57. A. here B. where C. there D. place 58. A. park B. place C. rain D. shelter 59. A. nobody B. none C. no people D. somebody 60. A. exercise B. nothing C. breathing. D. jogging 61. A. chairs B. sofas C. stools D. benches 62. A. doorway B. door C. gate D. window 63. A. back B. up C. out D. down 64. A. filled B. full C. crowded D. plenty 65. A. ponds B. puddles C. reservoirs D. springs C. when D. as VI. Read the passage and choose the best answer for each of the following questions. London is a big city, but many of the people who live there regard it as a number of small towns put together. Each district has its own identity and atmosphere and some parts are even described by their inhabitants as “village”. Much of the center of the city consists of shops and businesses and the majority of people live in the suburbs. A great many of them travel to work in the city every day by train, bus, tube or car; this is called commuting. Commuters might spend as much as two hours every morning getting to work and another two hours getting home again. The cost of living in London is higher than in most other parts of Britain, and many people are paid extra money on top of their salaries because of this. Millions of visitors come to London every year from all over the world to see the famous sights, such as Buckingham Palace, where the Queen live, and many other historic buildings. London is famous for its theatres, red buses and black taxis. 66. Some parts of London are regarded as ‘village’ by the: A. visitors B. local residents C. officers D. foreigners 67. Which of the following sentences is NOT TRUE about London? A. Most Londoners live in the city’s center. B. In London, people travel to work by car, bus, train and tube. C. It takes time for Londoners to go to work. D. There are many famous places in London. 68. What does the word Commuters mean? A. people who go to work by bus. B. people who travel to work from their home every day. C. people live in the suburbs. 24 D. those who travel to London. 69. Things in London are: A. as expensive as other parts of Britain B. at the same price as other parts of Britain. C. cheaper than other parts of Britain. D. more expensive than other parts of Britain. 70. London is famous for many things EXCEPT: A. buses B. historical buildings C. Cinema D. taxis. VII. Rewrite the following sentences using the following cues. 71. If she had seen him waving, she would have waved back. She didn’t …………………………………………………………….. 72. “I am leaving tomorrow,” she said, “by 4.30 from New York station” She said …………………………………………………………………. 73. The secretary didn’t know where the meeting was. I talked to her five minutes ago. The secretary who …………………………………………………………………… 74. I’m going horseback riding with Tom this afternoon. I’m afraid of horses. Although ……………………………………………………………………………. 75. It's about two kilometers and a half from her school to the cinema. (make questions with the underlined words) …………………………………………………………………………………? VIII. Complete each sentence of the following letter. Dear Mr. Black, 1. Thank you/ letter/ I receive/ this morning. …………………………………………………………………. 2. Thank you/ invite me/ come/ interview/ Tuesday next week. …………………………………………………………………… 3. Unfortunately/ I / not able/ attend/ time/ suggest. …………………………………………………………………… 4. Be/ possible/ rearrange/ interview/ Wednesday morning/ …………………………………………………………………… 5. I/ be/ office/ 8.30/ morning. …………………………………………………………………… 6. I/ be/ most grateful/ you/ contact/ confirm/ time. …………………………………………………………………… 7. I/ look forward/ hear/ you/ and meet/ week. …………………………………………………………………… 25 Yours sincerely, Thomas Dunn -----The end----- KÌ THI CHỌN HỌC SINH GIỎI TRƯỜNG Năm học 2010-2011 Trường THPT QUỲNH CHÂU ĐỀ CHÍNH THỨC Đề thi môn: Tiếng Anh 10 (Đề gồm 02 trang) Thời gian: 90 phút (Không kể thời gian giao đề) Điểm bằng số:………………… Giám khảo 1: …………………… Điểm bằng chữ:….…………… Giám khảo 2: …………………… Số phách: Question I. Choose the words whose underlined parts match the sounds given. enough cup bus played guitar square peas worked couch toy compete music sits stopped chair town knife oil laugh target radio they 1 /ɑ:/ 2 /ei/ 3 /s/ 4 /f / 5 /k/ 6 / ɑi / 7 /z / 8 /t / 9 /eə / 10 /au/ Question II. Find one mistake in each sentence and correct them. 11. I spend most of my free time to read books 12. It was not until his father came home which he did his homework. 13. In spite interruption, she was still able to finish her assignment before class. 14. If I don’t like you, I wouldn’t invite you to my house. 15. The children are exciting about the field trip. TT 11 12 13 14 15 Mistakes Corrections Question III. Put the words in brackets in to correct forms. 16. More than 50 films (show)…………..………… in Ha Noi since June. 17. If I (have) ………………...your address, I would have sent you a postcard. 18. It’s already 32 C. It (be) ………..………very hot. 19. Tom is very good at telling funny stories. I find his stories very (amuse)…...………. 20. I have decided to hold my son’s birthday party. I (invite)….…………. a lot of people. Question IV. Read the passage and answer the questions All of us have to work to earn a living ourselves and to help our family. However, we work not only for material life but also for many other things. We are working to prove our ability. We feel self26 confident and proud because we are independent. Anyone who works is regarded as a useful member of society. We’re working, that means we’re contributing to our country. Working helps us train our minds because we always try our best to do the job. The better we work, the more money we earn and that encourages us to work harder. Without working, a man’s life will be empty, purposeless and meaningless and it’s easy for a jobless person to do wrongs. 21.Why should we work? ………………………………………………………………………… 22. What is anyone who works regarded? ............................................................................................................... 23. What help us work harder? .............................................................................................................. 24. Without working, how is one’s life? ………………………………………………….…………………… 25. Do we feel self- confident and proud because we are dependent? ............................................................................................................ Question V. Rewrite the sentences so that they have the same meaning with the given ones. 26. Max isn’t at home yet. That worries me. Max.......................................................................................................... 27. Although he had a good salary, he was unhappy in his life. In spite…………………………………………………………….. 28. “ I can’t go to the movie with you tomorrow” Mary said to John............................................................................... 29. People spend a lot of money on advertising every day. A lot of money………………………………………………………………………………………………….. 30. Mary failed her driving test last week because she was so nervous. If Mary …………………………………………………………………………………………………………… …….. Question VI. Read the passage and fill in each blank with ONE suitable word. When I was just three years ..........…(31), I went to the post office with my mother where she was going to buy some stamps. While she was being served, I happened to notice a small stocking which was hanging from the counter. It was there to collect contributions for a charity for ….........(32) blind. While her back was turned I took the stocking and emptied the contents into my coat pocket. Of course I …........(33) too young to know any better. When it was realized what I had done, everybody roared with laughter except, that is, for my mother who was a little embarrassed. She quickly emptied the money back into the stocking including, incidentally, a few pennies of my own. One ........…(34)the clerks was something of an amateur cartoonist and he did a drawing of me robbing an old lady. This cartoon was projected in the post ….............(35) for the next couple of years. Question. VII. -Your friend invites you to his birthday party this weekend. You can’t come to the party. Write a letter of refusal to reply him. …………………………………………………………………………………………………………… …………………………………………………………………………………………………………… …………………………………………………………………………………………………………… …………………………………………………………………………………………………………… …………………………………………………………………………………………………………… …………………………………………………………………………………………………………… …………………………………………………………………………………………………………… …………………………………………………………………………………………………………… …………………………………………………………………………………………………………… …………………………………………………………………………………………………………… 27 …………………………………………………………………………………………………………… …………………………………………………………………………………………………………… ………………………………………………………………………… THE END Sở GD & ĐT Hà Nội Trường THPT Ngọc Tảo ĐỀ THI CHỌN HỌC SINH GIỎI Năm học: 2008-2009 MÔN THI: TIẾNG ANH - LỚP 10 Thời gian: 180 phút Họ và tên: ………………………………………Lớp…………………………………. I. PHONETICS A. Identify the word whose underlined part is pronounced differently from that of the rest. 1. A. machine B. school C. stomach D. chemistry 2. A. description B. question C. attraction D. information 3. A. although B. without C. thank D. these 4. A. food B. good C. tooth D. tool 5. A. heavy B. bear C. measure D. eat B. Find the word that has stressed syllable different from the others. 6. A. cartoon B. open C. paper D. answer 7. A. manage B. shortage C. village D. teacher 8. A. remain B. convenient C. volunteer D. memorable 9. A. maintain B. marine C. challenge D. device 10. A. important B. community C. organize D. diseases II. VOCABULARY - GRAMMAR A. Choose the best answer from the four choices (A, B, C, or D) to complete each sentence 11. I am going to have my eye……..tomorrow. A. to test B. test C. testing D. tested 12. Nobody could hear her……..she spoke too quietly. A. so B. however C. although D. because 13. Keep silent! My father…… in his room. A. is sleeping B. was sleeping C. has slept D. will be slept 14. I’d like to contribute…….the school Red Cross fund. A. on B. to C. with D. for 15. Hoang Anh Tuan …….injury to win the Olympic silver. A. recovered B. suffered C. avoided D. overcame 16. In my father’s ……., it is essential to eat vegetable every day. A. idea B. opinion C. confidence D. principle 17. He showed us the house……….he was born A. which B. where C. in where D. in that 18. Madrid is famous……..its pulls A. as B. with C. for D. of 19. Life here is much easier than it……… A. would be B. used to be C. was D. had better 20. Will you …….after the party. A. call me off B. get me on C. take me up D. pick me up 21. Last summer, we had ……. holiday in Spain A. a two week’s B. a two – weeks C. two – week D. a two – week 22. . Amy, …….car had broken down, was in a very bad mood. A. who B. whom C. whose D. that 23. He…….ever goes to bed before midnight. A. almost B. hardly C. nearly D. scarcely 24. He retired early……..ill – health. 28 A. on behalf of B. on account of C. believe D. imagine 25. You …… better be careful not to miss the train. A. had B. would C. should D. did B. Choose the appropriate sentence from the box below to fill in each gap to complete the following conversation. Ann: Goodbye everyone! (26)………………………………………………….. Jolly: Where are you going? Ann: To Australia. (27)…………………………………………………………. Jolly: When are you coming back? Ann: Well, (28)…………………………………………………………………. Jolly: It sounds great! (29)………………………………………………………. Ann. Thanks. (30)………………………………………………………………. a. b. c. d. e. I’ll send you postcards. We’ve going on a two – week tour of Australia. I’m taking my family to visit the Great Barrier Reef I’m going on holiday Have a good time! C. Complete the passage with the suitable words from the box preserved housed labourers outbuildings village life community fields In most of Europe, farmers’ homes and (31)……………….are generally located within a village, and tools and animals are (32)………………there. Every morning, the farmers and farm (33)……………….leave their (34)……………….to work their land or tend their animals in distant (35)…………… and return to the village at the end of the day. Social (36)……………is thus centripetal; that is, it is focused around the (37)……………center, the village. Only in certain parts of Quebec has this pattern been (38)……………in North America. III. ERROR IDENTIFICATION 39. How did you manage getting here so quickly? A. How B. did C. getting D. so 40. Ms Baker , she spent her life working with the health and welfare of the families of worker, is an successful woman in the world. A. she B. her life C. welfare D. the 41. Because of the Lewis and Clark Expedition in 1972, the United States begin to realize the true value of the Luisiana territory. A. Because B. begin C. the true value D. territory 42. I remember to be given a toy drum on my fifth birthday. A. remember B. to be C. on D. fifth 43. You can lead a horse to water but you can't make him drinking. A. lead B. but C. make D. drinking IV. READING A. Fill in each blank with a suitable word. LOOKING FOR A JOB I finished university six month (44)………….., I’ve got a degree in business administration. I enjoyed the course very much (45)…….…….I realize I should have studied a lot harder! A few of my friends have (46)……………got full-time jobs but most, me, are still waiting (47)………..… something suitable to turn up. Meanwhile, I make sure that I keep myself busy. I look through the job advertisements (48)…………… the newspapers every day and I also ask all the people I know to tell me if they hear of any vacancies (49)……………...they work . What I am looking for is something challenging and I would certainly be happy to move to another city or even work abroad for a while. 29 The (50)………….… is not so important at this stage , provided I earn enough to live on , because I don’t want to continue (51)………………. to depend on my parents , although they are (52)………………..generous to me . At the moment, I’m working in a nearby restaurant two evenings (53)…………….... week, washing up and generally helping out, which brings a little money . The other people working there are very friendly, and many of them are in the same (54)………………as me, so we have lots of to talk (55)………………… B. Choose the words or phrases that best complete this passage. Earlier this morning when some classmates and I were (56) _____ basketball in the park near our school, it suddenly began to rain (57) _____. We immediately grabbed our basketball and ran for shelter. At first we stood together under a tree (58) _____ then it began to thunder. Peter told us that it was (59)_____to stay under a tree in a rainstorm, (60)_____when there is lighting, so we ran to a nearby store to wait (61)_____the rain stopped. From (62)_____we were standing, we could see that the (63) ______ was empty. Before the rain had started, there had been people everywhere but now (64)______was playing basketball or football or doing (65)_____or sitting on the park (66)______reading the morning papers or chatting with their friends. We stood in the (67)_____of the store for nearly twenty minutes before the rain finally stopped and the sun came (68)_____again from behind the clouds. Within a few minutes the park was (69) _____of people again but now the air was cooler and big (70) ____of rainwater could be seen everywhere. 56. A hitting B kicking C playing D throwing 57. A greatly B heavily C severely D very big 58. A and B so C but D yet 59. A danger B in danger C a danger D dangerous 60. A specially B especially C mostly D chiefly 61. A for B until C when D as 62. A here B where C there D place 63. A park B place C rain D shelter 64. A nobody B none ` C no people D somebody 65. A exercise B nothing C breathing D jogging 66. A chairs B sofas C stools D benches 67. A doorway B door C gate D window 68. A back B up C out D down 69. A filled B full C crowded D plenty 70. A ponds B puddles C reservoirs D springs C. Read the passage carefully and choose the correct answers. They called New York “the Big Apple”. Maybe it is not exactly like an apple, it’s certainly very big. There are too many people, that’s the problem. The streets are always full of car and trucks, you can never find a place to park. If you have enough money, you can take a taxi. New York cabs are yellow. They look all the same. But the drivers are very different. Some were born and raised in New York, but many are newcomers to the United States. A few drive slowly, but most go very very fast. Cab driving is difficult job. It can be dangerous too. Thieves often try to steal the drivers’ money. Drivers sometimes get hurt. If you don’t want to take a taxi, you can go by bus or you can take the subway. The subway is quick and it’s cheap, but parts of it are old and dirty. Lights don’t always work and there are often fire on the track. On some subway lines, there are new, clean, silver trains. But you can’t see the color of the old trains easily. There is too much dirt and too much graffiti, inside and out side. 71. What is the problem in New York ? A. it has too many aples B. it is too big C. it looked like an apple D. it is too crowded 72. What does a cab mean ? A. a truck B. a bus C. a taxi D. It is too crowded 30 73. Cab drivers in New York…………….. A. can be dangerous B. look the same C. can be attacked by thieves D. were all born in New York 74. The world “track” can best be replaced by …………. A. roadway B. station C. light bulb D. train 75. Subways in New York …………. A. are quick but dirty B. have no lights C. are clothes D. often cause fires V.WRITING A. Complete the second sentence so that it has a similar meaning to the first 76. When did you last ride a bike? How long…………………………………………… 77. The trip was so interesting that we couldn’t forget it. It……………………………………………………. 78. “ Don’t forget to go to the supermarket after work” He reminded……………………………………….. 79. How many books did they buy yesterday? How many books…………………………………… 80. I'm sorry I didn’t go to University I regret……………………………………………… 81. He prefers plain water to coffee. He’d rather…………………………………………... 82. Although he wasn’t experienced, he got the job. Despite………………………………………………. 83. She didn’t say a word as she left the room. She left the room…………………………………… 84. They are going to repair my car tomorrow. I am going to have……………………………………. 85. You can’t travel on this train if you don’t have a reservation. Unless…………………………………………………….. B. Composition: Write a paragraph to tell about your recent journey (no more than 150 words) …………………………………………………………………………………………………………… …………………………………………………………………………………………………………… …………………………………………………………………………………………………………… ………………………………………………………………… Good luck to you! ANSWER KEY I.PHONETICS (10 points) 1. A 2. B 3.C 6.A 7.A 8. C II. VOCABULARY - GRAMMAR (28 points) A. 11. D 12.D 13.A 16.B 17.B 18.C 21.D 22.C 23.B B. 26d 27c 28b C. 31. outbuildings 32.housed 35.fields 36. life III. ERROR IDENTIFICATION (5 points) 4.B 9.C 5D 10.C 14.B 19.B 24.B 15.D 20.D 25.A 29e 30a 33. labourers 37. community 34. village 38. preserved 31 39. C 40.A 41.B 42.B IV. READING (32 points) A. 44. ago 45. although 46. already 47. for 49. where 50. salary 51 . having 52. very 53. a/per 54. situation 55. about B. 56.C 57B 58C 59c 61B 62B 63A 64A 66D 67A 68C 69B C. 71D 72C 73C 74A 43D 48. in 60B 65A 70B 75A V.WRITING A. (10 points) 76. How long is it since you rode the bike? 77. It was such an interesting trip that we couldn’t forget it. 78. He reminded me to go to supermarket after work 79. How many books were bought yesterday? 80. I regret not going to University. 81. He’d rather plain water than coffee. 82. Despite his inexperience, he got the job 83. She left the room without saying a word. 84. I am going to have my car repaired tomorrow. 85. Unless you have a reservation you can’t travel on this train B. 15 points + Useful language: spelling, vocabulary, connectors (5 points) + Grammar: tense (past tense), structures (5 points) + Content: creative, understandable (5 points) PART ONE : PHONETICS A/.Pick out the word whose underlined part has the pronunciation different from that of the other words. 1 a. steal b. break c. wake d. hate 2a. increase b. independent c. ink d. income 3a. book b. put c. brook d.booth 4a musket b. business c. muscle d. muscular 5a. brother b. mathematics c. think d. dethrone B/. Mark the main stress on these words: 1. perseverance 2. opportunities 3. industry 4 . refugee 5.artificial 6. persuade 7. museum 8. imperialist 9. decade 10. military PART TWO : VOCABULARY Put the word in brakets in their appropriate form: 1. AIDS is the most dangerous…………….. disease of the 20th century. ( infect) 2. The D war zone was the famous military base during the two……………wars. ( resist). 3. There has been a……………. crisis in Asia ( money) 4. When a small bar of a magnet is attached to a pigeon, it is……… to navigate. (able) 5. The Titanic was not only the largest ship that has ever been built but was regarded as ……. (sink) 6. …………………… drugs has theatened the young generation. (legal) 7. UFO stands for an ………………………. Flying object . ( identify) 8. A new economic……………………… zone has been established in our province.( process) 9. Most of the surgical instruments are made of………………steel. (stain) 32 10. The French …………….. were defeated at Dien Bien Phu on 7-5-1954. (colony) 11. PART THREE : GRAMMAR Rewrite the sentences so that they have the exact meaning as the original ones. 1. He speaks more persuasively than his brother. He is a…………………………………………………………………. 2. Mary told the police about the burglary. Mary reported………………………………………….. 3. I never intended to go to the meeting. I never had……………………………………………………. 4. He forgot about the gun until he got home. Not until…………………………………………………… 5.You really should be able to dress yourself by now ! It/ s high time……………………………………………………….. PART FOUR : CLOSE TEST Fill in each number space with one appropriate word. Pop music has (1) ……. Been influenced by ( 2)…….. forms of music: an important (3)……… is folk music. Folk songs are the songs (4)………… and sung by (5)…………. People. The songs may be hundereds of years old, so (6)……………… knows who originally composed them. Modern musis is (7)……………… music (8)…… dancing. In Britain it was traditionally played (9)………… instruments (10)……………………. Flute, accordion. PART FIVE : WRITING Make all the changes and additions necessary to produce complete sentences. Write each sentence in the space provided. Dear Bill, 1. I /expect / you be surprise / get / letter/ me. 2. As /can/see/ address above/ I be/ hospital. 3. Last Wednesday/ I have/ accident/ when I / drive / work. 4. Child / run out/ front/ my car / and I/ HAVE/ STOP/SUDDEN/ that car behind/ crash/ me. 5. Luckily/ I wear/ seat belt/ so /I/ not injure badly/ although/ have/ stay/ here/ next Friday. 6. It/ be/ very boring/ I be pleased/ see/ if you/ have spare time. 7. Visiting hours/ be/ 7.00 to 9.00/evening. 8. I/hope/ able/ come. 9. Give/ my/ regards your family. 10. Look forward/ see you/ again. - The end KEY PART ONE : PHONETICS A/.Pick out the word whose underlined part has the pronunciation different from that of the other words. ( 2m) 1A 2C 3D 4B 5A B/. Mark the main stress on these words: (4m) 1. perse /verance 2. oppor /tunities 3./ industry 4 . refu /gee / / / 5.arti ficial 6. per suade 7. mu seum 8. im /perialist / / 9. decade 10. military PART TWO : VOCABULARY 33 Put the word in brakets in their appropriate form: (4m) 1. infectious 2. resistance 3. monetary 4. unable 5. unsinkable 6. illeglal 7. unidentified 8. processing 9. stainless 10. colonialists. PART THREE : GRAMMAR Rewrite the sentences so that they have the exact meaning as the original ones.(2m) 1. He is a more persuasive SPEAKER/ ORATOR than his brother. 2. Mary reported the burglar to the police.. 3 .I never had any intention of going to the meeting 4. Not unti he got home did he remember about the gun 5. It/ s high time you were able to dress youself. PART FOUR : CLOSE TEST (4m) 1. always 2. other 3. form 4 composed 5.country 6. nobody 7. often 8. for 9.with 10. like PART FIVE : WRITING Make all the changes and additions necessary to produce complete sentences. Write each sentence in the space provided.(4m) Dear Bill, 1. I expect you will be surprised to get a letter from me. 2. As you can see from the address above I am in hospital. 3. Last Wednesday I had an accident when I was driving to work. 4. A child ran outin front of my car , and I Had to STOP so SUDDENly that the car behind crash into me. 5. Luckily I was wearing a seat belt so I wasn/t injured badly although I may have to stay here until next Friday. 6. It is very boring, and I would be pleased to see if you have any spare time. 7. Visiting hours are 7.00 to 9.00 in the evening. 8. I hope you are able to come. 9. Give my regards to your family. 10. Look forward seeing you again. THE END TRƯỜNG THPT TIÊN LỮ ĐỀ THI HỌC SINH GIỎI NĂM HỌC 2009 Môn thi: tiếng anh – khối 10 ( Thời gian làm bài : 150 phút ) Họ và tên : …………………………………...........Lớp : ……………………….SBD : ……………… I.Phonetics A. Choose one word whose stress pattern is different from the others. 1. A. wonderful B. cartoon C. visual D. passive 2. A. joyfulness B. emotion C. favourite D. Beautiful 3. A.event B. hero C. teacher D. Championship 4. A. audience B. tournament C. embarrassed D. violent 5. A. background B. humane C. romantic D. depend B.Choose the word which has the underlined part pronounced differently from the others. 6. A. chorus B. technicolor C. chopstick D. character 7. A. walked B. danced C. laughed D. studied 8. A. pair B. fair C. stair D. Faith 9. A. horses B. tongues C. places D. faces 34 10. A. cheese B. cheer C. peer D. career II. Reading comprehension A.Read the passage and decide the statements are true(T) or false(F). Da Lat lies on Lam Vien plateau, in the central highland province of Lam Dong. Da Lat is a wellknown city attracting all the people who have been there once. DaLat is known as a city of pine trees, waterfalls and flowers. Da Lat is described as a forest of flowers with different colors and various species. Da Lat has the widestrange of orchid varieties in the country. Da Lat has few rivers and canals but it has many picturesque waterfalls. The famous Cam Ly Falls is only three kilometers from the town center. The Prenn Falls is ten kilometers in the South of Da Lat. The water pours down like a white shade. Da Lat people are very proud of it. They always boast to tourists about it in the first place. N0 SENTNECES T F 11 Da Lat is located in the highland province of Lam Dong. 12 There are a lot of rivers and canals in Da Lat. 13 The Cam Ly Falls is about ten kilometers from the town center. 14 In Da Lat, there are various species of flowers with different colors. 15 People know Da Lat as a city of trees, flowers and rivers B.Choose the word or phrase which best fits each gap of the passage. Tropical forests have (16)……………us with many sorts of plants for food, medicine and industry. They also reduce (17)………………..and droughts, keep water clean, and slow down the Greenhouse Effect. However, the tropical forests are being(18)…………….to make rooms for things like farms, ranches, mines and hydroelectric(19)…………………About twenty million hactares(20)…………each year – an area more than twice the size of Australia. 16. A. given B. supplied C. provided D. B and C 17. A. floods B. water C. rain D. pollution 18. A. destroying B. destroyed C. endangering D. endangered 19. A. power B. station C. current D. dams 20. A. is lost B. lost C. are lost D. are losing III. VOCABULARY AND GRAMMAR A. Put the verbs in brackets in the correct forms of the verbs. 21.My cousin and I ( watch)………….. a movie on TV last night when my brother (come)………….. 22. He ( come)………….. a moment ago and (turn)………….. the TV on very loudly . 23. He ( learn)…………. two languages before he (move)…………….. to Canada last month. 24. She always (drink)……………………….. a glass of milk for breakfast, but this morning she (drink)………………. a cup of coffee. 25. Be quiet! The baby ( sleep) in the room. I just( put) him in bed for sleeping. B.Supply the correct form of the words 26. His foolish mistakes ………………………….all of us. (amusement) 27. Ann gave a brilliant ……………………as Juliet in the school play. (perform) 28. She was very …………………at being delayed (anger) 29. Why are you so ………………….? My little sister drove me crazy. (frustrate) 30. Both musically and …………………………..it is very effective. (lyrics) C. Choose the best answer A, B, C, or D 31. …………………..other worker’s constant objection, the director dismissed the worker. A. Because B. Because of C. Although D. Inspite of 32. “ How long have you known him?” “………………..” A. since five years B. for five years C. ever since five years D. along five years 33. …………………is yours, the red one or the green one. A. When B. which C. what D. When 34. I spent a(n)………………….night because of my toothache. A. sleepy B. sleeping C. asleep D. sleepless 35. Whenever my father travels by plane, he buys some …………………goods at the airport. 35 A. free of charge B. duty-free C. fax-free D. free of duty 36. It is an .................. book. I am .................. in it. A. interesting / interested B. interesting / interesting C. interest / interested D. interested / interested 37. It was not until1930………………….he became an actor. A. that B. when C. which D. in which 38. She study hard…………………her parent’sexpectation. A. meeting B. to meet C. meet D. met 39. We would stay at home…………………………………………………… A. If it is snowing. B. If it is going to snow. C. If it would snow. D. If it was snowing 40. The waste from the chemical factory is extremely……………………… A. harmed B. unharmed C. harmful D. harmless 41. A: Does Mr Tan travel to HCM city to work everyday? B: Yes. He takes a …………………bus. A. transportation B. commuter C. passenger D. daily 42. If you’re too ill to come,………………………………………………… A. I’ll come over and see you. B. I wouldn’t have done all this for you. C. I asked someone else. D. I could come over and see you. 43. All the rooms in this small hotel are………………….., and that would make it too expensive to stay in. A. air-condition B. air-conditioning C. air conditioners D. air-conditioned 44. He was …………….the “golden kite” for being the best film director. A. rewarded B. given C.awarded D.enjoyed 45. He couldn't have known what was in the letter ______ he had written it himself. A. until B. if C. unless D. if only 46. Hoang is really .......... about going to Ho Chi Minh City. It will be an ................ experience for him. A. exhaust/ exhausting B. exhausting/ exhausting C. exhausted/ exhausting D. exhausted/ exhausted 47. When I ………………, everyone ………………………. A.arrived/had left B. had arrived/ left C. would arrive/haves D. arrived/ has left. 48. I remember ______ him once in my life. A. to meet B. Meeting C. to have met D. met 49. .................... do you always go to school? - By bus A. What B. What time C. When D. How 50.This house, …………..he bought in 1990, is being repaired at the moment. A.that B.which C.what D.who 51.'No, I didn't steal anything from her,' said Henry. A. Henry confirmed that he had stolen things from her. B. Henry refused to steal things from her. C. Henry denied stealing things from her. D. Henry said no to stealing things from her. 52.John never comes to class on time and………… A.neither does Peter B.neither doesn’t Peter C.so doesn’t Peter D.so does Peter 53. Chinese New Year is .................... important holiday in Taiwan. A. a B. the C. an D. no article 54. “They received this present yesterday” means : A. This present is received yesterday. B. This present was received yesterday. C. This present received yesterday. D. This present has received yesterday. 55. Can I ……………your dictionary, please? I can’t find mine. A. look B. owe C. borrow D. lend 56. How……………….is your house from here? It’s about two hours by taxi. A. far B. many C. long D. much 57. He is ………………to do such work. 36 A. not enough strong B. strong not enough C. not strong enough D. enough strong 58. “ I did my best,” she said, “ so why are you angry………………….me?” A. with B. on C. for D. of 59. They all laughed because the film was very………………….. A. amuse B. amusing C. amused D. amusement 60. Tim ………………go fishing with his father when he was young. A. used to B. has used to C. is used to D. was used to 61. It’s no good…………………your father about your failure. A. to tell B. tell C. telling D. told 62. The boy fell into the water and was …………………. A. missed B. killed C. destroyed D. Drowned 63. The cinema changed completely at ……… end of………. 1920s. A. an- the B. the – x C. x- the D. the- the 64. She was born ……………………….April, 1991. A. in B. on C. at D. since 65. We haven’t seen each other……………………last January. A. for B. on C. since D. In IV. Writing A. Choose the correct sentence with the same meaning as the one in italics. 66. If I knew his telephone number, I would phone him. A. I didn’t know his telephone number, so I couldn’t phone him. B. Because I don’t know his telephone number, I can’t phone him. C. I don’t know his telephone number. That why I can’t phone him. D. I knew his telephone number, and I could phone him. 67. The building has been designed by a famous architect. A. A famous architect designed the building . B. A famous architect has been designed the building C. A famous architect has designed the building . D. A famous architectwas designed the building . 68. “ I think smoking in restaurants should be banned,” he says. A. He doesn’t think smoking in restaurants is permitted. B. He doesn’t like smoking in restaurants. C. He advised everybody not to smoke in restaurants. D. He thinks the government should ban smoking in restaurants. 69. The picnic was cancelled because it rained. A.If it didn’t rain, the picnic wouldn’t be cancelled. B. If it hadn’t rained, the picnic wouldn’t have been cancelled. C.If it didn’t rain, the picnic wouldn’t have been cancelled. D.If it hadn’t rained, the picnic wouldn’t be cancelled 70. It’s ages since I last saw you. A. I haven’t seen you for a long time. B.I didn’t see you since ages. C. I haven’t seen you since a long time. D. I didn’t see you for ages. B. Use the words given to complete a refusal letter. Dear Betty, 71.Thanks/ kind invitation/ daughter’s birthday. ……………………………………………………………………………………….. 72. I/ love/ see/ niece/ but/ unfortunately/ I/ afraid/ won’t/ able/ visit you/ next weekend. …………………………………………………………………………………………… 73. I/ go/ camping trip/ my school students/ occasion/ school anniversary. …………………………………………………………………………………………… 74. What pity! I wish/ give Leslie/ hug/ day. Thank/ anyway/ more. …………………………………………………………………………………………… 75. Remember/ send / photos/ Leslie/ her birthday. ……………………………………………………………………………………………. 37 76. Look/ to/ seeing/ photos/ lovely niece. …………………………………………………………………………………………… Lots of love Marry C.Complete the second sentence so that the meaning is similar to the first one 77.I have no right qualification and I can’t apply for the job. →If…………………………………………………………………………. 78.The prime minister was going to open the new school. →The new school………………………………………………………….. 79. .Silent films didn’t become available until the late 1920s. →It was not until…………………………………………………………… 80.Although he has a very important job, he isn’t particularly well-paid. →In spite of……………………………………………………………….. --The end-Sở GD & ĐT Nghệ An Trường THPT Anh Sơn1 ĐỀ THI CHỌN HỌC SINH GIỎI CẤP TRƯỜNG Năm học: 2008-2009 MÔN THI: TIẾNG ANH - LỚP 10 Thời gian: 120 phút Họ và tên: ………………………………………Lớp…………………………………………. I. PHONETICS A. Identify the word whose underlined part is pronounced differently from that of the rest. B. school C. stomach D. chemistry 1. A. machine 2. A. description B. question C. attraction D. information 3. A. although B. without C. thank D. these 4. A. food B. good C. tooth D. tool 5. A. heavy B. bear C. measure D. eat B. Find the word that has stressed syllable different from the others. 6. A. cartoon B. open C. paper D. answer 7. A. manage B. shortage C. village D. teacher 8. A. remain B. convenient C. volunteer D. memorable 9. A. maintain B. marine C. challenge D. device 10. A. important B. community C. organize D. diseases II. VOCABULARY - GRAMMAR A. Choose the best answer from the four choices (A, B, C, or D) to complete each sentence 11. I am going to have my eye……..tomorrow. A. to test B. test C. testing D. tested 12. Nobody could hear her……..she spoke too quietly. A. so B. however C. although D. because 13. Keep silent! My father…… in his room. A. is sleeping B. was sleeping C. has slept D. will be slept 14. I’d like to contribute…….the school Red Cross fund. A. on B. to C. with D. for 15. Hoang Anh Tuan …….injury to win the Olympic silver. A. recovered B. suffered C. avoided D. overcame 16. In my father’s ……., it is essential to eat vegetable every day. A. idea B. opinion C. confidence D. principle 17. He showed us the house……….he was born A. which B. where C. in where D. in that 18. Madrid is famous……..its pulls A. as B. with C. for D. of 38 19. Life here is much easier than it……… A. would be B. used to be C. was D. had better 20. Will you …….after the party. A. call me off B. get me on C. take me up D. pick me up 21. Last summer, we had ……. holiday in Spain A. a two week’s B. a two – weeks C. two – week D. a two – week 22. . Amy, …….car had broken down, was in a very bad mood. A. who B. whom C. whose D. that 23. He…….ever goes to bed before midnight. A. almost B. hardly C. nearly D. scarcely 24. He retired early……..ill – health. A. on behalf of B. on account of C. believe D. imagine 25. You …… better be careful not to miss the train. A. had B. would C. should D. did B. Choose the appropriate sentence from the box below to fill in each gap to complete the following conversation. Ann: Goodbye everyone! (26)………………………………………………….. Jolly: Where are you going? Ann: To Australia. (27)…………………………………………………………. Jolly: When are you coming back? Ann: Well, (28)…………………………………………………………………. Jolly: It sounds great! (29)………………………………………………………. Ann. Thanks. (30)………………………………………………………………. f. g. h. i. j. I’ll send you postcards. We’ve going on a two – week tour of Australia. I’m taking my family to visit the Great Barrier Reef I’m going on holiday Have a good time! C. Complete the passage with the suitable words from the box preserved housed labourers outbuildings village life community fields In most of Europe, farmers’ homes and (31)……………….are generally located within a village, and tools and animals are (32)………………there. Every morning, the farmers and farm (33)……………….leave their (34)……………….to work their land or tend their animals in distant (35)…………… and return to the village at the end of the day. Social (36)……………is thus centripetal; that is, it is focused around the (37)……………center, the village. Only in certain parts of Quebec has this pattern been (38)……………in North America. III. ERROR IDENTIFICATION 39. How did you manage getting here so quickly? A. How B. did C. getting D. so 40. Ms Baker , she spent her life working with the health and welfare of the families of worker, is an successful woman in the world. A. she B. her life C. welfare D. the 41. Because of the Lewis and Clark Expedition in 1972, the United States begin to realize the true value of the Luisiana territory. A. Because B. begin C. the true value D. territory 42. I remember to be given a toy drum on my fifth birthday. A. remember B. to be C. on D. fifth 43. You can lead a horse to water but you can't make him drinking. A. lead B. but C. make D. drinking 39 IV. READING A. Fill in each blank with a suitable word. LOOKING FOR A JOB I finished university six month (44)………….., I’ve got a degree in business administration. I enjoyed the course very much (45)…….…….I realize I should have studied a lot harder! A few of my friends have (46)……………got full-time jobs but most, me, are still waiting (47)………..… something suitable to turn up. Meanwhile, I make sure that I keep myself busy. I look through the job advertisements (48)…………… the newspapers every day and I also ask all the people I know to tell me if they hear of any vacancies (49)……………...they work . What I am looking for is something challenging and I would certainly be happy to move to another city or even work abroad for a while. The (50)………….… is not so important at this stage , provided I earn enough to live on , because I don’t want to continue (51)………………. to depend on my parents , although they are (52)………………..generous to me . At the moment, I’m working in a nearby restaurant two evenings (53)…………….... week, washing up and generally helping out, which brings a little money . The other people working there are very friendly, and many of them are in the same (54)………………as me, so we have lots of to talk (55)………………… B. Choose the words or phrases that best complete this passage. Earlier this morning when some classmates and I were (56) _____ basketball in the park near our school, it suddenly began to rain (57) _____. We immediately grabbed our basketball and ran for shelter. At first we stood together under a tree (58) _____ then it began to thunder. Peter told us that it was (59)_____to stay under a tree in a rainstorm, (60)_____when there is lighting, so we ran to a nearby store to wait (61)_____the rain stopped. From (62)_____we were standing, we could see that the (63) ______ was empty. Before the rain had started, there had been people everywhere but now (64)______was playing basketball or football or doing (65)_____or sitting on the park (66)______reading the morning papers or chatting with their friends. We stood in the (67)_____of the store for nearly twenty minutes before the rain finally stopped and the sun came (68)_____again from behind the clouds. Within a few minutes the park was (69) _____of people again but now the air was cooler and big (70) ____of rainwater could be seen everywhere. 56. A hitting B kicking C playing D throwing 57. A greatly B heavily C severely D very big 58. A and B so C but D yet 59. A danger B in danger C a danger D dangerous 60. A specially B especially C mostly D chiefly 61. A for B until C when D as 62. A here B where C there D place 63. A park B place C rain D shelter 64. A nobody B none ` C no people D somebody 65. A exercise B nothing C breathing D jogging 66. A chairs B sofas C stools D benches 67. A doorway B door C gate D window 68. A back B up C out D down 69. A filled B full C crowded D plenty 70. A ponds B puddles C reservoirs D springs C. Read the passage carefully and choose the correct answers. They called New York “the Big Apple”. Maybe it is not exactly like an apple, it’s certainly very big. There are too many people, that’s the problem. The streets are always full of car and trucks, you can never find a place to park. If you have enough money, you can take a taxi. New York cabs are yellow. They look all the same. But the drivers are very different. Some were born and raised in New York, but many are newcomers to the United States. A few drive slowly, but most go very very fast. Cab driving is difficult job. It can be dangerous too. Thieves often try to steal the drivers’ money. Drivers sometimes get hurt. 40 If you don’t want to take a taxi, you can go by bus or you can take the subway. The subway is quick and it’s cheap, but parts of it are old and dirty. Lights don’t always work and there are often fire on the track. On some subway lines, there are new, clean, silver trains. But you can’t see the color of the old trains easily. There is too much dirt and too much graffiti, inside and out side. 71. What is the problem in New York ? A. it has too many aples B. it is too big C. it looked like an apple D. it is too crowded 72. What does a cab mean ? A. a truck B. a bus 73. Cab drivers in New York…………….. A. can be dangerous C. a taxi D. It is too crowded B. look the same C. can be attacked by thieves D. were all born in New York 74. The world “track” can best be replaced by …………. A. roadway B. station C. light bulb D. train 75. Subways in New York …………. A. are quick but dirty B. have no lights C. are clothes D. often cause fires V.WRITING A. Complete the second sentence so that it has a similar meaning to the first 76. When did you last ride a bike? How long…………………………………………… 77. The trip was so interesting that we couldn’t forget it. It……………………………………………………. 78. “ Don’t forget to go to the supermarket after work” He reminded……………………………………….. 79. How many books did they buy yesterday? How many books…………………………………… 80. I'm sorry I didn’t go to University I regret……………………………………………… 81. He prefers plain water to coffee. He’d rather…………………………………………... 82. Although he wasn’t experienced, he got the job. Despite………………………………………………. 83. She didn’t say a word as she left the room. She left the room…………………………………… 84. They are going to repair my car tomorrow. I am going to have……………………………………. 85. You can’t travel on this train if you don’t have a reservation. Unless…………………………………………………….. B. Composition: Write a paragraph to tell about your recent journey (no more than 150 words) …………………………………………………………………………………………………………… …………………………………………………………………………………………………………… …………………………………………………………………………………………………………… …………………………………………………………………………………………………………… ……………………………………………………… Good luck to you! ANSWER KEY I.PHONETICS (10 points) 1. A 2. B 3.C 6.A 7.A 8. C II. VOCABULARY - GRAMMAR (28 points) 4.B 9.C 5D 10.C 41 A. 11. D 12.D 13.A 14.B 16.B 17.B 18.C 19.B 21.D 22.C 23.B 24.B B. 26d 27c 28b 29e C. 31. outbuildings 32.housed 33. labourers 35.fields 36. life 37. community III. ERROR IDENTIFICATION (5 points) 39. C 40.A 41.B 42.B IV. READING (32 points) A. 45. ago 45. although 46. already 47. for 49. where 50. salary 51 . having 52. very 53. a/per 54. situation 55. about B. 56.C 57B 58C 59c 61B 62B 63A 64A 66D 67A 68C 69B C. 71D 72C 73C 74A 15.D 20.D 25.A 30a 34. village 38. preserved 43D 48. in 60B 65A 70B 75A V.WRITING A. (10 points) 76. How long is it since you rode the bike? 77. It was such an interesting trip that we couldn’t forget it. 78. He reminded me to go to supermarket after work 79. How many books were bought yesterday? 80. I regret not going to University. 81. He’d rather plain water than coffee. 82. Despite his inexperience, he got the job 83. She left the room without saying a word. 84. I am going to have my car repaired tomorrow. 85. Unless you have a reservation you can’t travel on this train B. 15 points + Useful language: spelling, vocabulary, connectors (5 points) + Grammar: tense (past tense), structures (5 points) + Content: creative, understandable (5 points) ĐỀ THI THỬ ĐẠI HỌC Thoi gian lam bai 150’ I. ON YOUR ANSWER SHEET INDICATE THE LETTER A,B,C OR D AGAINST THE NUMBER OF EACH ITEM: 1. According to the---------of the contract, tenants must give six months’ notice if they intend to leave. A. Laws B. Rules C. Terms D. Details 2. The injured man was taken to hospital and ---------for internal injuries. A. Cured B. Healed C. Operated D. Treated 3. The door hinges had all been oiled to stop them----------A. Squeaking B. Screeching C. Shrieking D. Squealing. 4. Mary attempted to--------herself with her boss by volunteering to take on extra work. A. Gratify B. Please C. Ingratiate D. Commend. 5. You are under no obligation-------to accept this offer. A. Indeed B. Eventually C. Apart D. Whatsoever. 6. The dealer wanted $400, I wanted to pay $ 300, and we finally agreed to ---------the difference. 42 A. Divide B. Split C. Drop D. Decrease. 7. I was ----------in the book, I was reading and didn’t hear the phone. A. Distracted B. Submerged C. Gripped D. Engrossed 8. A washing machine of this type will certainly---------up to normal domestic use. A. Hold B. Stand C. Come D. Take 9. -------------any other politician would have given way to this sort of pressure years ago. A. Really B. Practically C. Actually D. Utterly. 10. If you---------too long, you may miss a wonderful opportunity. A. Loiter B. Doubt C. Hover D. Hesitate 11. The singer’s performance was so exciting that many of his fans were-------enthusiasm. A. Carried away with B. Moved to C. Taken back with D. Stirred up with 12. Passengers are---------not to leave cases and packages here. A. Commanded B. Informed C. Notified D. Advised 13. He has an excellent--------as a criminal lawyer. 14. Hardly had the van turned the corner when one of the back wheels--------A. Broke away B. Turned around C. Came off D. Rolled down. 15. There was no sound to be heard except the-----------of raindrops on the roof. II.ON YOUR ANSWER SHEET INDICATE THE LETTER A,B,C OR D AGAINST THE NUMBER OF EACH ITEM: FIRST PASSAGE Lily Chen always prepared an ‘evening’ snack for her husband to consume on his return at 1.15p.m. This was not strictly necessary since Chen enjoyed at the unusually late hour of 11.45 pm, what the boss boasted was the best employees’ dinner in any restaurant. They sat waiters, boss, and boss’s mother too, at a round table and ate soup, a huge fish, vegetables, shredded pork, and a tureen of steaming rice. Lily still went ahead and prepared broth, golden- yellow with floating oily rings and put it before her husband when he returned. She felt she would have been failing in her wifely duties otherwise. Dutifully, Chan drank the soup he raised to his mouth in the porcelain spoon while Lily watched him closely from the sofa. It was far too rich for him. Lily had the gas fire burning five minutes before her husband’s footfall on the stone stairs and Chan would be perspiring heavily by the time he finished, abandoning the spoon and applying the bowl to his lips to drink the last awkward inches, the beads of moisture on his forehead as salty as the broth. He fancied they fell in and over- seasoned the last of the soup. Four years ago, at the beginning of their marriage, Chen had tried leaving the last spoonful but Lily’s reproachful eyes were intolerable. She was merciless now, watching him with sidelong glances from the sofa, her knees pressed closely together while she paired the baby’s socks from the plastic basket on the floor. ‘Did you enjoy that, husband?’ ‘Was it nice?’ she would enquire brightly. Chen would grunt in his solid way, not wishing to hurt her feelings but also careful not to let himself in for a bigger bowl in the future. Although comfortably full, Chen would have liked a biscuit but Lily was unrelenting here as well. Sweet after salty was dangerous for the system, so she had been taught; it could upset the whole balance of the dualistic or female and male principles, yin and yang. Lily was full of incontrovertible pieces of lore like this which she had picked up from her father who had been a part- time bone-setter and Chinese boxer. For four years, therefore, Chen had been going to bed tortured with the last extremities of thirst but with his dualistic male and female principles in harmony. This was more than could be said for Lily, Chen often thought, who concealed a steely will behind her demure exterior. 16. What was Chen’s job? A. He ran a restaurant B. He washed dishes C. He was a cook D. he worked as a waiter 17. How did Chen feel about his meal at home? A. It was too hot B. He looked forward to it C. It was well-balanced D. He would rather not have it 18. Chen always finished his soup because Lily A. Felt content if he did. B. Would then allow him to drink C. Did not like to throw food away D. Complained bitterly if he didn’t 19. Chen’s homelife was difficult because of his wife’s A. Lack of concern B. Rigid ideas. C. Thoughtlessness D. Unco-operative behavior 20. Which of the following statements correctly describes the relationship between Lily and Chen? 43 A. They were indifferent to each other B. They respected each other C. They made each other suffer. D. They did not get on well. III. USE THE WORDS TO THE RIGHT OF THE TWO TEXTS TO FORM ONE WORD THAT FITS IN THE SAME NUMBERED SPACE IN THE TEXT: Traffic congestion is now a problem in practically major city in the world but nobody has yet found a workable (21)---------to the seemingly inevitable chaos. A metro system is sadly (22)---------in most cities for geographical reasons. Tram systems are (23)-------in old cities where narrow, winding streets make the installation of overhead cables a practical (24)--------many local government the business of coaxing people into buses and (25)-----------them from using their cars easier said than done. And yet it is (26)-------that the situation should be allowed to remain as it is. The arguments are in favor of direct action are now (27)-------if we are ever to prevent (28)------levels of population and economic chaos. It is astonishing how many people set off to climb Mount Olympus in completely (29)-------- clothing. The weather conditions on the mountain are notoriously (30)----------but people are fooled into thinking that just because the bottom is sunny, the summit will be (31)--------warm and bright. Nothing could be further from the truth. Bearing in mind that (32)--------is forearmed’, consult the local climbing club about the likely conditions before setting off. Such local knowledge can be absolutely (33)--------- and you would, to put it mildly, be extremely (34)-------- to ignore it. Whatever the likely weather, a good pair of boots is (35)-------as is some form of waterproof. And it is a steep climb so it goes without saying that a reasonable level of (36)-----------is essential. 21. SOLVE 22. PRACTICE 23. WORK 24. POSSIBLE 25. COURAGE 26. CONCEIVE 27. REPUTE 28. REFUTE 29. PRECEDE 30. SUIT 31. PREDICT 32. SIMILAR 33. WARN 34. VALUE 35. ADVISE 36. DISPENSE IV. COMPLETE THE SECOND SENTENCE SO THAT IT HAS A SIMILAR MEANING TO THE FIRST SENTENCE, USING THE WORD GIVEN. YOU MUST USE BETWEEN THREE AND EIGHT WORDS, INCLUDING THE WORD GIVEN: 37. I hope one day we can agree more favorable terms. REACH I hope one day it------------------on more favorable terms 38. Some supermarket beef can be rather tough TENDENCY. Some supermarket beef---------------------rather tough. 39. Would you kindly inform everyone of the change of procedures? WONDER I-----------------------------as to inform everyone of the change of procedures? 40. May be I could have helped you POSSIBLE May be----------------------------------help you. 41. In the end we were able to communicate with sign language. MESSAGE In the end we succeeded in-------------across with sign language. 42. Don’t you regret reacting in the way you did? DIFFERENTLY Don’t you think--------------------------from the way you did? 43. I bet she felt stupid when she realized DAWNED She must---------------------------------------on her who she has been speaking to. 44. Do you think we should have got her opinion first? SOUNDED Ought-------------------------out about t first? 45. He always refused to behave differently to the senior partners. TREAT He would-----------------------------------respect they were due. 46. It’s possible she didn’t fully appreciate how important this was. GRAVITY She may--------------------------------of the situation. V. FILL EACH OF THE NUMBERED BLANKS IN THE PASSAGE WITH ONE SUITABLE WORD: Many people have proposed that no employee (47)---------be asked to work more than a 35-hour week. This sounds fine in theories though in practice is (48)-------to give rise to all sorts of problems if applied too strictly. On the other hand, the proposal can be seen as a welcome attempt to stem employers’ seemingly (49)-------demands for long hours. For some time, the underlying (50)--------has been for the working week to become longer and even the most unambitious employee has found it hard to fight the (51)--------towards longer working days and work dominated weekends. Anyone who can stop this encroachment of the work place into our leisure hours can boast a (52)-------achievement. 44 47. A. Might B. Should C. Would D. Will 48. A. Liable B. License C. Likewise D. Linked 49. A. Unstopping B. Runaway C. Permanent D. Incessant 50. A. Cause B. Trend C. Current D. Support 51. A. Tendency B. Tenement C. Tenancy D. Tenacity. 52. A. Forthcoming B. Lengthy C. Lasting D. Maintained VI. FINISH EACH OF THE SENTENCE S IN SUCH A WAY THAT IT MEANS EXACTLY THE SAME AS THE SENTENCE PRINTED BEFORE IT. 53. By chance I was there when she revealed the truth. I happened----------------------------------------54. He suddenly thought that he might have misunderstood her. It crossed--------------------------------------------------------55. I didn’t want to buy it, but because he insisted, I bought it. He pushed-----------------------------------------------------56. I met some old friends by chance in the pub I bumped-----------------------------------------------------57. I need an early night I could--------------------------------------------------58. I really need a holiday I’m desperate---------------------------------59. If nothing unfortunate happens, I’ll see you next week. All being-------------------------------------------60. People are persuaded by adverts to spend more than they can afford Adverts tempt----------------------------------------------------------61. She doesn’t think very highly of politicians. She’s got a-----------------------------------------------62. She fully understands that she will have to work hard. She’s well---------------------------------------------------------VII. USING THE CAPITAL WORD TO REWRITE THE SENTENCES 63. The court case has destroyed his reputation completely TATTERS The court case-----------------------------------64. All of the fuss she has made is nothing but a storm in a teacup WATER All of the--------------------------------------------------65. Much to the disappointment of his fans, he went down in defeat. DUST ------------------------------------------------------------------------66. Never mind! He has a reputation for making foolish, unreliable statements. HAT ------------------------------------------------------------------------------67. He was extremely angry as he got stuck in the slow-moving traffic COLLAR -------------------------------------------------------------------------------68. Let’s stop working on the report and take a nap! ICE -------------------------------------------------------------------------69- The criminal eventually died after spending a miserable life in prison BUCKET ----------------------------------------------------------------------------------70. I used to be familiar with every corner of this school. HAND ------------------------------------------------------------------------------------VII. PHONOLOGY A. CHOOSE THE WORD WHOSE UNDERLINED PART IS PRONOUN DIFFERENTLY FROM THE OTHERS: 71. A. child B. wild C. minute D. mild B. dictates C. cost D. pleasure 72. Singular 73. A. epoch B. catch C. match D. teach 74. A. huge B. garage C. ginger D. dangerous 75. A. thunder B. thesis C. theft D. thereafter. B. CHOOSE THE WORD WHOSE STRESS PATTERN IS DIFFERENT FROM THE OTHER WORDS IN EACH GROUP: 45 76. A. answer 77. A. deform 78. A. altitude 79. A. arrangement 80. A. necessary B. active C. abbey D. finance B. attend C. absence D. collect B. recover C. comedy D. library B. adjacent C. expensive D. adjective B. correspond C. considerable D. exceptional ĐÁP ÁN I. 1.C 2. D 3. A 4. C 5. D 6. B 7. D 8.B 9.B 10. D 11. A 12. D 13. D 14. C 15. D II. 16. D 17. D 18. A 19. B 20. B. III. 21. SOLUTION 22.IMPRACTICAL 23. UNWORKABLE 24. IMPOSIBILITY 25. DISCOURAGING 26. INCONCEIVABLE 27. INREFUTABLE 28. UNPRECEDENTED 29. UNSUITABLE 30. UNPREDICTABLE 31. SIMILARLY 32. FOREWARNED 33. INVALUABLE 34. ILL-ADVISED 35. INDISPENSABLE 36. FITNESS IV. 37. Will be possible for us to reach agreement. 42. You should have reacted differently 38. has a tendency to be 43. Have felt stupid when it dawned 39. wonder if you would be so kind 44. We to have sounded her 40. It would have been possible for me to 45. Never / refuse to/ treat the senior partner with the 41. In getting our message 46. Not have appreciated the gravity. V. 47. B 48.A 49. D 50. B 51.A 52. C VI. 53. I happened to be there to be there when she revealed the truth. 54. It crossed his mind that he might have misunderstood her. 55. He pushed me to buy it even though I didn’t like. 56. I bumped into some old friends in the pub 57. I could be satisfied if I had an early night 58. I’m desperate for a holiday 59. All being good keeps going on, I’ll see you next week 60. Adverts tempt people into spending more than they can afford. 61. She has got a low opinion of politicians 62. She’s well aware of the fact that she will have to work hard. 63. The court case has left his reputation in tatters 64. All of the fuss she has made is nothing but water under the bridge. V. 65. His fans bit the DUST when he came down in defeat 66. Never mind! He has a reputation for talking through his HAT 67. He felt hot under the COLLAR as he got stuck in the slow-moving traffic 68. Let the report on ICE and take a nap! 69- The criminal eventually kicked the BUCKET after spending a miserable life in prison 70. I used to know every corner of this school like the back of my HAND VII. PHONOLOGY A. CHOOSE THE WORD WHOSE UNDERLINED PART IS PRONOUN DIFFERENTLY FROM THE OTHERS: 71. C 72. D 73. A 74. B 75. D B. CHOOSE THE WORD WHOSE STRESS PATTERN IS DIFFERENT FROM THE OTHER WORDS IN EACH GROUP: 76. D 77. C 78.B 79.D 80. 46 §Ò thi ®Ò nghÞ m«n tiÕng anh líp 10 (olympic 30/430/4- lÇn thø viiiviii-2007) Tr−êng THPT chuyªn TrÇn H−ng §¹o- B×nh ThuËn Part 1: Phonology I. Choose the word who underlined part s pronounced differently from that of the others in each group. (10pts) 1.A. polio 2.A. chaos 3. A. says 4. A. southern 5. A. disguise 6. A. hour 7. A. sergeant 8. A. promised 9. A. apology 10. A. beloved B. police B. character B. plays B. mathematics B. cruise B. our B. servant B. reduced B. classify B. naked C. polite C. charity C. days C. breath C. bruise C. pour C. service C. released C. testify C. ploughed II. Choose the word with the different stress pattern. (10 pts) 11.A. because B. method C. standard 12. A. American B. industrious C. property 13. A. eventual B. community C. infected 14. A. refuge B. electronic C. education 15. A. assassinate B. equality C. intelligence 16. A. ashtray B. music C. serve 17. A. challenging B. consultant C. purity 18. A. assemble B. glorious C. majestic 19. A. antonym B. consequence C. phenomenon 20.A. destination B. enthusiast C. hallucinate D. pollute D. chord D. raise D. truth D. suit D. sour D. servile D. relieved D. verify D. learned D. wooden D. variation D. militarism D. deficiency D. understanding D. temper D. wholesaler D. together D. ingot D. technology Part 2: Vocabulary Choose the best answer (10 pts) 1.If Harold………….with his piano playing, he could eventually reach concert standard. A. sustains B. perseveres C. maintains D. survives 2. Anna is a very nervous child and she’s very………………….of strangers. A. terrified B. frightened C. petrified D. horrified 3. The government is thinking of bringing………..a law to make it compulsory for cyclists to wear crash helmets. A. on B. up C. in D. round 4. I love the painting of an ald man. He has such a beautiful……………smile. A. childhood B. childish C. childless D. childlike 5. She caught a ……………………of the thief as he made his get- away. A. glance B. sight C. glimpse D. flash 6. It was a service…………..I will be enternally grateful. A. to which B. for which C. to whom D. for whom 7. Frank has a house in the ……………….. A. outskirts B. edges C. limits D. suburds 8. The film lasted every three hours with…………….of 15 minutes between part one and part two. A. a pause B. an interruption C. an interval D. a stop 9. The person who writes symphonies or concertos is a …………….. A. composer B. conductor C. pianist D. playwright 10. Is it far to Stambord? It’s only………………..from here. A. stone’s cut B. bird’s eye view C. short cut D. step in the right direction 47 Part 3: Grammar and stuctures. I.Finish each of the following sentences so that it meaning exactly the sentence above it. (10 pts) 1.Andrew is the most generous person I have ever met. I’ve yet…………………………………………………………………………… 2. Far more people live to entertainment age in Britain than in Philipines. Not………………………………………………………………………………. 3. His father is going to fix the ball for him tomorrow. He is going ………………………………………………………………………… 4. She never seems to succeed eventhough she studies much. Much………………………………………………………………………………… 5. I nevr thought that I would win a prize. It had………………………………………………………………………………… 6. I would love to be rich and famous. If……………………………………………………………………………………... 7. Simon hadn’t expected that he would feel so weak after the operation. The operation left……………………………………………………………………. 8. They secretly declared independence two days ago. A secret……………………………………………………………………………… 9. Juliet simply couldn’t wait for his return. Juliet could…………………………………………………………………………... 10. I don’t intend to apologize to either of them. I have………………………………………………………………………………... II. Fill in each gap with the correct form of the word in brackets.(10 pts) 1. Tax exemption only applies to those with (resident) ……..status. 2. His busy schedule made him completely (access)………………..to his students. 3. He works for UNESCO in a purely (advice)…………………role. 4. The sun and the moon are often (person)…………………in poetry. 5. Now there is every(like)………………………………..that the residents will have a private plane in their garage. 6. They told me the museum was open today but I was obviously (inform)…… 7. Do you think these children are (nourish)………………..? They look very thin. 8. Have you read the latest (reveal)……………….about Madonna’s private life? 9. We try to ensure the (be)…………….of our employees. 10. Tourists forget their (conceive)………ideas as soon as thay visit our country. III.Choose the best answer.(10 pts) 1.I’d rather you………..at my party last night. A. be B. were C. had been D. have been 2. …….will Mr Forbes be able to regain control of the company. A. With hard work B. In spite of his hard work C. Only if he works hardly D. Only with of his hard work 3. Barbara didn’t mention…………about her progress report at work, but I’m sure she is. A. concerning B. to concern C. being concerned D. to be concerned 4. Snow and rain………of nature. A. are phenomenon B. are phenomena C. is phenomena D. is phenomenon 5. Everyone was going away on holiday this week,………………..? A. wasn’t it B. wasn’t be C. weren’t they D. wasn’t she 6…………….my key, I wasn’t able to bet into the room. A. Losing B. I had lost C. I lost D. Having lost 7. It’s no good pretending. You’ve got to…………………..reality. 48 A. come up to B. face up to C. come up with D. get round to 8.I haven’t …………………………..decided where to go on holiday this year. A. still B. yet C. already D. then 9. “Why are you angry at Bob?” - “I don’t like it when he makes jokes………my expense.” A. for B. on C. at D. to 10. A car……………many different pars. A. is made B. is made up of C. is made of D. makes up of Part 4: Reading comprehension A.Choose from the phrasesA-H the one which fits each gap. (10 pts) Scientists estimate that at the beginning of 20th century 100,000 tigers flourished throughout Asia, from estern Russia and Korea through eastern and southern China, Southest Asia, the Indian subcontinient, and into Pakistan. At the start of the 21st century only 5,000 to 7,000 tigers lived (1)………… Cambodia, China, India, Laos, Malaysia, Myanmar, Nepal, North Korea, Russia, Sumatra, Thailand, and Vietnam. Tigers are now extinctin Bali, Java, and around the Caspian Sea, and nearly so in Chinaand North Korea, Less than 20 percent of today’s tiger habiat is located (2)……………………, which means that the majority of the areas where tigers live could be lost to others uses, such as agriculture or urbanization. Tigers are territorial- they live alone (3)…………..jother tigers. The ideal tiger territory is a large forested area with rich vegetation for cover, plentiful water to drink and cool off in, and urbudant deer, swine, and other large mammals to eat. With these three esential, tigers can thrive (4)…………, tropical rain forests in Sumatra and Southest Asia; cool oak and pine forest in the Amur River Valley in far eastern Russia; tall grass jungles (5)………in Bamngladesh; and mountain slopes in Bhutan. A.in no more than a hundred species left B.in the wild in just 14 Asian countries: Bangladesh, Bhutan. C. in will life preserves which are built D. in large areas that they defend from E. in national parks or other protected areas F. in India and Nepal; coastal mangrove forests G. in deverse habiats and climates including H. in the surburbs or on the outskirts of the city II. read the passage and answer the questions. (10 pts) According to airline industry statistics, almost 90 percent of airline accidents are survivable or partially survivable. But passengers can increase their chances of servival by learning and following certain tips. Experts say that you should read and listen before take off and ask questions if you have uncertainties. You should fasten you seat belt low on your hips and as tightly as possible. Of course, you should also know how the realisemechanism of your belt operates. During takeoffs and landings, you are advised to keep your feet flat on the floor. Before takeoff you should locate the nearest exit and a alternative exit and count the rows of seats between you and the exits so that you can find them in the dark if necessary. In the event that you are forewarned of a possible accident, you should put your hands on your ankles and keep your head down until your plane comes to complete stop. If smoke is present in the carbin, you should keep your head low and cover your face with napkins, towels or clothing. If possible, wet these for added protection against smoke inhalation. To evacuate as quickly as posible, follow crew commands and do not take personal belongings with you. Do not jump on escape slices before they are in front of you. When you get to the ground, you should move away from the plane as quickly as possible, and never smoke near the wreckage. 49 1.What is the main topic of the passage? 2. According to experts, when are travelers urged to read and listen to safety instructions? 3. Which exits should an airline passenger locate before takeoff? 4. Find out a word which is synonym with the word “evacuate” in the passage. 5. What does the paragraph following the passage most probably discus? Answerkey Part 1: Phonology I.(10 pts) 1.A 2. C 3. A 4.A 5. A 6. C 7. A 8. B 9. C 10.C II.(10 pts) 11.A 12. D 13. D 14. D 15. D 16. C 17. B 18. B 19. C 20. A Part 2: Vocabulary I.(10 pts) 1.B 2. A 3. C 4. D 5. C 6. B 7. D 8. C 9. A 10.A II. (20 pts) 1.survival 2. distributed 3. exist 4. sources 5. increasing 6. developing 7. discharged 8. agriculture 9. conservation 10. inefficient Part 3: Grammar and structure I.(10 pts) 1. I’ve yet to meet anyone who is more generous than Andrew. 2. Not as many people live to retirement age in the Philipines as in Britain. 3. he is going to have his father fix the ball tomorrow. 4. Much as she studies, she never seems to succeed. 5. It had never crossed my mind that I’d win a prize. 6. If only I were rich and famous. 7. The operation of independence was made 2 days ago. 8. A secret declaration left Simon feeling weaker than he had expected. 9. Juliet could only long for his return. 10. I have no intention of apologizing to either of them. II.(10 pts) 1.C 2. C 3. C 4. B 5. C 6. A 7. B 8. B 9. A 10.B Part 4: Reading comprehension I.(10 pts) 1.B 2. E 3. D 4. G 5. F II. (10 pts) 1. Guidelines for increasing aircraft passenger survival. 2. before takeoff 3. The two closest to the passenger’s seat. 4. vacate 5. How to preceed once you are away from the aircraft. 50 SỞ GIÁO DỤC VÀ ĐÀO TẠO TỈNH BẾN TRE TRƯỜNG : THPT CHUYÊN BẾN TRE KỲ THI OLYMPIC TRUYỀN THỐNG 30 - 4 LẦN THỨ 15 ĐỀ THI ĐỀ NGHỊ MÔN: ANH VĂN; LỚP : 10 A. MULTIPLE CHOICE I. Phonology: (5 pts) a. Select the word that has the underlined part pronounced differently from the rest. 1. A. maple B. staple C. apple D. April 2. A. Leonardo B. leopard C. leonine D. leotard 3. A. isotherm B. worthy C. southern D. hitherto 4. A. move B. womb C. tomb D. wolf 5. A. clamor B. grantor C. sailor D. parlor b. Pick out the word stressed on the different syllable from the others. 1. A. Arabic B. aerobic C. lunatic D. politics 2. A. infamous B. inundate C. internal D. intellect 3. A. safari B. detritus C. cognomen D. partisan 4. A. pianist B. facetious C. retrospect D. redolence 5. A. sunbathe B. rattan C. coiffeur D. charade ANSWER I: PHONOLOGY (5.0 pts) a. Pronunciation (.5 x 5 = 2.5 pts) 1. C 2. B 3. A 4. A 5. B b. Stress (.5 x 5 = 2.5 pts) 1. B 2. C 3. D 4. B 5. A B. MULTIPLE CHOICE – READING COMPREHENSION II. READING COMPREHENSION a. Read the passage and then choose the best answer. William Shakespeare William Shakespeare was born in Stratford-Upon-Avon in Warwickshire, England on April 23, 1564. His mother, Mary Arden, had come from a fairly wealthy family. His father, John, was a glove maker and a leather merchant by trade. He also held the prestigious position of town bailiff in their community. In all, the Shakespeares had eight children, William being the third of these children and the first son. Three of William's brothers and sisters died during childhood. Reportedly, Shakespeare did not have an extensive education. He did attend Stratford Grammar School, a school for the sons of prominent citizens, although it appears his family may not have paid for his education due to his father's status as a high-ranking town official. While it is not certain, it is believed that Shakespeare attended this school from age 7 to 14. The time he spent at this grammar school is assumed to be the only education he received, yet the literary quality of his works suggests a more advanced education. has contributed to the debate concerning the authorship of his works. On November 28, 1582, Shakespeare married Anne Hathaway, a farmer's daughter. ■ A) Anne was twenty-six years old when they married and was pregnant at the time. William was only eighteen. ■ B) Their marriage produced three children. ■ C) Shakespeare apparently abandoned his family and disappeared from 1585 to 1592. ■ D) No records of Shakespeare exist from this period of his life, and they are usually referred to as "the lost years." Some have speculated that he either became a schoolteacher, a butcher's apprentice, or was running from the law. 51 Shakespeare reappeared in London where he arrived with the goal of becoming an actor and playwright. Evidently, Shakespeare envy for his talent early on. He even found a sponsor to help pay for his services. Shakespeare's work in the theaters came to a halt, however, when the theaters of London closed down due to the plague in January of 1593. This closing inspired Shakespeare and his company to move to the Globe Theater in the Bankside district, across the river from London's city limits. Shakespeare's company, originally called "Lord Chamberlain's Men," changed their name to "The King's Men" after King James took over the throne in 1603. Because Shakespeare worked and performed for royalty, his company became the biggest and most famous acting company in the area. Consequently, Shakespeare became quite well-to-do as a director, writer, actor, and stockholder in The King's Men. During his time, Shakespeare published and sold his plays in octavo editions. Also known as "penny copies," these were sold to the more literate members of his audience. It is noted that a playwright had never before enjoyed sufficient acclaim as to see his works published and sold as popular literature in the midst of his career His 37 plays span the genres of tragedy, comedy, and history. While Shakespeare could not be considered wealthy by London standards, his success did allow him to purchase New House and retire in comfort in Stratford in 1611. e made a will on March 25, 1616, and died, as the legend has it, on his birthday, April 23, 516. He was buried at Holy Trinity Church in Stratford on April 25th. Shakespeare wrote his own epitaph to avoid the common tendency at the time of a person's gave being dug up after several years to accommodate another body. "Good Friends, for Jesus' sake forbear, To dig the bones enclosed here! Blest be the man that spares these stones, And curst be he that moves my bones." To this day, no one has disturbed Shakespeare's grave. In 1623, two working companions of Shakespeare from the Lord Chamberlain's Men, John Heminges and Henry Condell, printed the First Folio edition of his Collected Works, half of which were previously unpublished. The First Folio also contained Shakespeare's sonnets. Many argue that William Shakespeare's legacy is a body of work that will never again be paralleled in Western civilization. His words have endured for 400 years and still reach across the centuries as powerfully as ever. 1. Which of the following could best replace the word "garnered" as used in paragraph 4? A. discouraged B. found C. attracted D. prevented 2. What does "this matter" in paragraph 2 refer to? A. Shakespeare going to a school for prominent families B. The surprisingly high quality of Shakespeare's works C. The uncertainty of whether he attended the grammar school D. His father's status as a high ranking town official 3. Why does the author mention that Shakespeare's father held the prestigious position of town bailiff in their community? A. To illustrate that his father was also quite talented and accomplished B. To show Shakespeare was not worried about the getting in trouble with the law C. To explain how his father could have married his mother, who was wealthy D. To explain why, although not wealthy, Shakespeare did enjoy some privilege 4. According to the passage, why did Shakespeare stop performing in London? A. The audience in London did not enjoy his poetic style. B. King James forced him to leave the city in 1595. C. He began selling his own plays to literate audience members. D. All the theaters were closed due to the plague. 52 5. According to the passage, why were the years 1585 - 1592 called "the lost years"? A. No records of Shakespeare's life exist from this time. B. Shakespeare was unable to write during this time due to an illness. C. Shakespeare's plays from this period were burned in the Great Fire. D. Shakespeare refused to speak with his wife during these years. 6. Which of the sentences below best expresses the essential information in the highlighted sentence in the passage? Incorrect choices change the meaning in important ways or leave out essential information. A. The sale of Shakespeare's plays made him the most popular playwright of all time. B. London audiences were buying more plays then than ever before. C. Shakespeare was the first playwright who was famous enough to sell his plays while still producing them. D. Shakespeare's plays played an important role in this rise of literacy in the west. 7. All of the following are true EXCEPT A. Shakespeare spent time as a glove maker from 1585 to 1592. B. Shakespeare had three children with his wife. C. Shakespeare's company came to be called "The King's Men." D. Shakespeare is said to have died on his birthday. 8. What can be inferred from paragraph 2 about Shakespeare's works? A. Shakespeare must have gone to university at some point to write them. B. Some scholars today doubt that Shakespeare actually wrote them. C. They are viewed as the best example of what one person can accomplish. D. They have never been considered as having been written by just one person. 9. Why did Shakespeare write his own epitaph? A. He didn't want his grave to be dug up later. B. He didn't want it to be written by an inferior writer. C. It was the custom at the time to write one's own epitaph. D. To dedicate it to two former actors in Lord Chamberlain's Men. 10. Directions: An introductory sentence for a brief summary of the passage is provided below. Complete the summary by selecting the THREE answer choices that express the most important ideas in the passage. Some sentences do not belong in the summary because they express ideas that are not presented in the passage or are minor ideas in the passage. First sentence: William Shakespeare lived an interesting and accomplished life and is today regarded as the greatest English writer the world has ever known. A. Shakespeare was the first playwright to enjoy such wide acclaim as to see his works published during his career. B. Shakespeare's acting company was originally called Lord Chamberlain's Men but later changed its name to The King's Men. C. Shakespeare's works are timeless and have endured for over 400 years. D. Shakespeare's father was a glove maker and the town bailiff of Stratford-Upon-Avon. E. Although he produced works of high quality, Shakespeare is not believed to have had an extensive education. F. Shakespeare's works have been translated into more languages than any other volume of literature, including the Christian Bible. ANSWER: B. READING COMPREHENSION a. Reading comprehension (1.0 x 10 = 10 pts) 1. C 2. B 3. D 4. D 5. A 6. C 7. A 8. B 9. A 10. A, C, E C. MULTIPLE CHOICE – READING COMPREHENSION 53 b. Read the passage. Decide whether the statements below are true or false (T/F) by referring to the information in the text. Types Of Memory [1] As mentioned previously, one of the most important characteristics of a computer is its capability of storing information in its memory long enough to process it. Not all computers have the same type of memory. In this section, three types of memory will be discussed; core memory, semiconductor memory (or chip), and bubble memory. [2] The memory of the first computers was made up of a kind of grid of fine vertical and horizontal wires. At each intersection where the wires crossed, there was a small ferrite ring called a core (hence the name "core memory") which was capable of being either magnetized or demagnetized. Every intersection had its unique address; consequently, when an electrical current was passed through the wires, the magnetized as well as the unmagnetized cores were identified by their respective addresses. Each core represented a binary digit of either 0 or 1, depending on its state. Early computers had a capacity of around 80,000 bits; whereas now, it is not surprising to hear about computers with a memory capacity of millions of bits. This has been made possible by the advent of transistors and by the advances in the manufacture of miniaturized circuitry. As the result, mainframes have been reduced in both size and cost. Throughout the 19508, 1960s and up to the mid-19708, core memory dominated the market. [3] In the 1970s, there was a further development which revolutionized the computer field. This was the ability to etch thousands of integrated circuits onto a tiny piece (chip) of silicon, which is a non-metallic element with semiconductor characteristics. Chips have thousands of identical circuits, each one capable of storing one bit. Because of the very small size of the chip, and consequently of the circuits etched on it, electrical signals do not have to travel far; hence, they are transmitted faster. Moreover, the size of the components containing the circuitry can be considerably reduced, a step which has led to the introduction of both minis and micros. As a result, computers have become smaller, faster, and cheaper. There is one problem with semiconductor memory, however, when power is removed, information in the memory is lost unlike core memory, which is capable of retaining information during a power failure. [4] Another development in the field of computer memories is bubble memory. The concept consists of creating a thin film of metallic alloys over the memory board. When this film is magnetized, it produces magnetic bubbles, the presence, or absence of which represents one bit of information. These bubbles are extremely tiny, about 0.1 micrometer in diameter. Therefore, a magnetic bubble memory can store information at a greater density than existing memories, which makes it suitable for micros. Bubble memories are not expensive, consume little power, are small in size, and are highly reliable. There is probably a lot more to learn about them, and research in this field continues. 1. The most important function of a computer is to hold information in its memory in order to process it. 2. Minicomputers, microcomputers, and mainframes all have the same kind of memory. 3. Semiconductor memory was developed before core memory and after bubble memory. 4. Core memory uses small metal rings which can be magnetized or unmagnified. 5. The state of the core can be represented by either 0 or 1. 6. Early computer memories had less storage capacity than newer ones. 7. A transistor and a chip are the same kind of device. 8. The development of chips made it possible (or minicomputers and microcomputers) to be invented. 9. Bubble memory is smaller than a chip. 10. Bubble memory doesn't have very many advantages. B. READING COMPREHENSION b. Reading comprehension (1.0 x 10 = 10 pts) 1. T 2. T 3. F 4. T 5. T 6. T 7. F 8. T 9. T 10. F 54 A. MULTIPLE CHOICE – READING COMPREHENSION III. Guided cloze test: Read the text and then choose the correct answer to fill in the gap. Sir, Recent letters in your columns about insurance companies prompt me to cite this example. Some months ago a car (1) …………to my daughter was (2) …………from outside her house in Birmingham. As a doctor in daily need of her car, she was surprised when her insurers said she would have to wait six weeks for (3) ………… . To her annoyance they managed, by all kinds of procrastination, to drag this out to three months; and then only after a (4) …………of lengthy, rather unpleasant phone calls would they pay a penny. My daughter was then (5) …………to find that there would be no refund, or no extension of the period of insurance, to cover the three months lost. A full premium (no (6) …………sum for (7) …………cover in Birmingham), had to be paid for a quarter of a year, when no cover was (8) …………at all. Surely this is another example of the way insurance companies (9) …………money, perhaps legally but to my mind immorally, which gives them such a bad image. If any of your readers know of a way (10) ………… this difficulty, I would welcome their advice. Yours faithfully, R.R.E. Potter 1. A. owning B. pertaining C. trusted D. belonging 2. A. stolen B. robbed C. nicked D. broken 3. A. retribution B. premium C. compensation D. payment 4. A. number B. range C. lots D. heap 5. A. appalled B. intimidated C. shocking D. impressed 6. A. little B. bad C. lesser D. mean 7. A. complete B. integral C. comprehensible D. comprehensive 8. A. done B. made C. provided D. affected 9. A. obtain B. extort C. earn D. win 10. A. with B. round C. about D. avoiding ANSWER III. (1.0 X 10 = 10 pts) 1. D 2. A 3. C 4. A 5. A 6. D 7. D 8. C 9. A 10. B B. WRITTEN TEST I. Put the verbs in parentheses in the correct tenses. Add a suitable modal verb or an equivalent where necessary. I (1. become) ………… Managing Director five years ago when I (2. work) ………… as an assistant to the predecessor. I (3. ever, be) ………… Personnel Manager for three years and (4. join) ………… the firm in 1970, so I (5. be) ………… here for fifteen years. I (6. work) ………… in an office on the top floor where I (7. just, meet) ………… an important customer. I (8. retire) ………… in five years. I think I (9. go) ………… and live in the country. I (10. be) ………… with the firm for twenty years by then. I (11. be) ………… still working at eight o' clock yesterday evening. I (12. work) ………… since early morning. We (13. work) ………… very hard at the office lately as we (14. negotiate) ………… an important contract. Tomorrow I (15. fly) ………… to Milan. It (16. still, rain) ………… like this when I (17. get) ………… back, I (18. wonder) …………? I hope not, because I (19. take) ………… a few days off as soon as I can. I (20. work) ………… non-stop for over three weeks. ANSWER I. Tenses & forms (.5 x 20 = 10 pts) 1. became 2. was working 3. had ever been 5. have been 6. work 7. have just met 9. will/shall go 10. will/shall have been 11. was 13. have been working 14. are negotiating 15. am flying 4. joined/had joined 8. retire 12. had been working 16. will still be raining 55 17. get 18. wonder 19. am going to take 20. will have been working B. WRITTEN TEST II. Fill in the blank with a suitable preposition or an adverbial particle. Sir James succeeded (1) …………the hereditary title in 1969. His father died (2) …………injuries received in a road accident. Although Sir James takes pride (3) …………his ancestry, he's no snob. Sheila's very strict (4) …………the children in her class. She's particularly strict (5) …………punctuality. However, her relationship (6) …………the children is a happy one. Hello, John. Good to see you. Come in for a few minutes and sit down. I suppose you want to see Jane. She’s upstairs but she will soon be (7) ………… in a few minutes. In fact you are lucky to find her (8) ………… as she’s just dressing herself (9) …………to go out with her friend. Anyway, I am glad you called (10) ………… because I’d like to talk to you. 11. My French isn’t very good but it’s enough to get ………… . 12. Business has fallen …………a lot lately: turnover is down 15% last year. 13. The old man told me to give …………from his daughter, but I just had to see her again. 14. Why is that man hanging …………outside our house? Do you think he’s a burglar? 15. It was not going to be easy to explain …………the lipstick which his wife had found on his collar. 16. He is now …………the head of his class. 17. Don’t lean …………the stove. 18. You will have to wait here Thursday. 19. His face is familiar …………me. 20. This soup tastes …………iodine. ANSWER II. Preposition & adverbial particle(1.0 x 10 = 10 pts) 1. to 2. from 3. in 4. with 5. about 6. with 7. down 8. in 9. up 11. by 12. off 13. in 14. about 15. away 16. at 17. against 18. till/until 10. in 19. to 20. of B. WRITTEN TEST I. WORD FORM: Fill in the blank with the correct form of the word at the end of the sentence. 1. Eddie just can't stop working and relax any more, he's turning into a real ………………. WORK 2. ………………. people are often critical and sarcastic. CONTENT 3. Tests have shown that people's blood pressure goes up when they are in red colored ……………….SURROUND 4. Insecure people often do things just to avoid other people's ……………….APPROVE 5. Most of us think we have a rational and well-balanced ………………. on life. LOOK 6. There has been a ………………. increase in recovery from leukemia in the last 30 years. MIRACLE 7. Faulty electrical ………………. are a relatively common cause of fires in offices, factories and hotels. APPLY 8. Without color dyes people would find a lot of processed food rather ………………. APPETITE 9. Because he said so little in meetings, his colleagues thought he was ………………. to his work, but actually he was just timid. COMMIT 10. Some hardwood is produced on ………………. so that natural forests are not destroyed. PLANT ANSWER I. Word form (1.0 x 10 = 10.0 pts) 1. workaholic 2. discontented 3. surroundings 4. disapproval 5. Outlook 6. miraculous 7. appliances 8. unappetizing 56 9. uncommitted 10. plantations B. WRITTEN TEST II. Read the text and then fill in the blank with the correct form of the word chosen from the box. able ignore accuracy know believe loaf confide three express understand Seven-year-olds fail test A study by the National Foundation for Educational Research of 3400 seven-year-old pupils shows an (1) ………….of basic math and English which is simply (2) …………. . The results, which were published recently, show that one in seven children lack basic (3) ………….of math and cannot even do simple multiplication, such as 5 x 5. The results also show that a (4) ………….of the children in this age group cannot count up to 100 and do not know what (5) ………….such as half and a quarter, refer to. Moreover, only half of the children had any (6) ………….of the decimal system for money and only one in thirty could (7) ………….read the temperature on a thermometer. Finally, only one in seven could say what the cost of three 50p (8) ………….is. As regards English, more than a quarter have not learnt to read with any (9) ………….and have problems with the alphabet; meanwhile another 25% are (10) ………….to spell easy words, like can, man and hot. ANSWER II. Word form (1.0 x 10 = 10.0 pts) 1. ignorance 2. unbelievable 3. knowledge 6. understanding 7. accurately 8. loaves 4. third 5. expressions 9. confidence 10. unable B. WRITTEN TEST IV. There are 10 errors in the paragraph. Find and correct them. Some lines have more than one mistake. BODY LANGUAGE Body language imparts meaning with the use of words. It is the type of non-verbal communication. There are certain recognized distinctions like types of body language: voluntary/involuntary and universal/cultural. The first distinction is often fuzzy in instance, a smile can be voluntary or involuntary. However, with the second distinction, smiles aren’t universal. They are interpreting the same across all cultures. Nodding and head shaking, however, are cultural. In Turkey, the former is replaced by raising the eyebrows. It was thought that body language has its roots in animal communication. However, great apes raised in captivity are quite proficient for reading human body language. ANSWER IV. Error correction (1.0 x 10 = 10 pts) L1: 1. with (the) → without 2. the (type) →a L2: 3. like (types) → between L3: 4. in (stance) → for L4: 5. with (the) → by 6. aren’t (universal) → are L5: 7. interpreting (the) → interpreted L6: 8. was (thought) → is L7: 9. However → Indeed 10. for (reading) → at 57 B. WRITTEN TEST V. Fill in the blank with ONE suitable word. A. Communication Throughout our lives, right from the moment when (1) ………….infants we cry to express hunger, we are engaging in social interaction of one form or another. Each and every time we encounter (2) ………….human beings, some kind of social interaction will take place, (3) ………….it's getting on a bus and paying the fare for the journey, or socializing with friends. It goes without saying, therefore, that we need the ability to communicate. Without some method of transmitting intentions, we would be (4) ………….a complete loss when it came to interacting socially. Communication involves the exchange of (5) …………., which can be anything from a gesture to a friend (6) ………….boredom to the presentation of a university thesis which may (7) ………….ever be read by a handful of others, or it could be something in (8) ………….the two. Our highly developed languages set us apart from animals. (9) ………….for these languages, we could not communicate sophisticated or abstract ideas. Nor could we talk or write about people or objects not immediately present. (10) ………….we restricted to discussing objects already present, we would be unable to make abstract generalizations about the world. ANSWER V. (0.5 x 20 = 10.0 pts) A. 1. as 2. fellow 6. signaling 7. only 3. whether 8. between 4. at 9. but 5. information 10. Were B. WRITTEN TEST V. Fill in the blank with ONE suitable word. B Scientists are still curious about the effects of the oceanographic phenomenon called El Nino, which (1) …………. in the south-west Pacific Ocean. There is no doubt, however, about the reason for its occurrence. Every few years, a rise in surface temperatures (2) …………. to a temporary decrease in trade winds makes sea levels rise, and waves head north-east up the coast of South America. El Nino is noted (3) ………….its effect on local climates, but recent research has shown that it is also (4) ………….for wider atmospheric changes, resulting, for example, (5) ………….temporary drought in the Sahel or failure of the maize harvest in Zimbabwe. Now scientists involved in oceanographic research are working on a new theory, that El Nino can produce changes (6) ………….ocean patterns that may last for decades. Computers at the Naval Research Laboratory in Mississippi, fed with the latest satellite (7) ………….on sea temperatures and wave heights, show that the giant waves that roll across the Pacific may be the result of a previous El Nino, which will (8) …………. to affect the circulation of the North Pacific for years to come. The 1982 El Nino, the strongest of the 20th century, is (9) ………….of causing the flooding of the Mississippi basin in 1993, with the loss of life and serious (10) ………….to crops and property that resulted. ANSWER B. (0.5 x 20 = 10.0 pts) 1. occurs 2. due 3. for 6. in 7. information 8. continue 4. responsible 5. in 9. suspected 10. damage B. WRITTEN TEST VI. Finish the second sentence without changing the meaning by using the words given at the beginning or one word at the end of the sentence. 1. Just after solving one problem, I was faced with another. Hardly ……………………………………………………… 58 2. Getting specialized qualification will benefit you. It will be in ………………………………………………… 3. We have to repay the loan, now that we have promised to. We are.……………………………………………………… 4. ‘Oh, just a minute, was it Leonardo or Michelangelo who painted the Mona Lisa?’ He couldn’t remember ……………………………………….. 5. The company will well make a profit next year. I wouldn’t be surprised ………………………………………… 6. Don’t forget to contact me if you come here. Don’t forget to get ..……………………………………………. 7. The defense contributed enormously to the team's success. MADE ………… ………………………………………………………… 8. I don’t agree with prohibiting smoking in bars. FAVOR ………………………………………………………………….. 9. Chess is much more skilful game than backgammon. DEAL ………………………………………………………………….. 10. I was just about to leave the house when I heard the phone ring. VERGE ………………………………………………………………………….. ANSWER VI. Sentence transformation (2.0 x 10 = 20.0 pts) 1. Hardly had I solved one problem than I was faced with another. 2. It is/will be in your interests to get/that you get a specialized qualification. 3. We are obliged to repay the loan. / We are committed to repaying the loan. 4. He couldn’t remember if it was Leonardo or Michelangelo who (had) painted the Mona Lisa? 5. I wouldn’t be surprised if the company made a profit next year. 6. Don’t forget to get in touch with me if you come here. 7. The defense made an enormous contribution to the team's success. 8. I’m not in favor of prohibiting smoking in bars. 9. Playing chess requires a good/great deal more skill than playing chess. 10. I was on the verge of leaving/going out of the house when I heard the phone ring. 59 Së GD - §T B¾c Giang Tr−êng THPT T©n Yªn 2 §Ò thi häc sinh giái cÊp tr−êng n¨m häc 20072007-2008 M«n: TiÕng AnhAnh-Líp 10 Thêi gian lµm bµi: 150 phót (kh«ng kÓ thêi gian giao ®Ò) Chó ý: ý - Häc sinh lµm bµi trùc tiÕp vµo ®Ò thi. - §Ò thi gåm 05 trang. Häc sinh kh«ng ®−îc sö dông bÊt cø tµi liÖu nµo kÓ c¶ tõ ®iÓn hay b¶ng ®éng tõ bÊt bÊt quy t¾c. Hä vµ tªn : .................................................. Líp :.................... : Sè b¸o danh:................................................... danh: §iÓm: ................... PhÇn I: I: Phonetics Chän mét tõ mµ phÇn phÇn g¹ch ch©n cã c¸ch ph¸t ©m kh¸c nh÷ng tõ cßn l¹i. X¸c ®Þnh c©u tr¶ lêi b»ng c¸ch khoanh trßn ®¸p ¸n A, B, C hay D (10®) 1. A. says B. decay C. clay D. day 2. A. sport B. doctor C. wrong D. boss 3. A. four B. sport C. sock D. walking 4. A. school B. noon C. cook D. food B. tiny C. hidden D. forbid 5. A. pity 6. A. sugar B. school C. sunny D. seafood 7. A. brochure B. chemist C. Christmas D. character 8. A. talked B. arrived C. missed D. watched B. daughter C. subtract D. walk 9. A. talk 10. A. plays B. cleans C. speaks D. understands PhÇn II: II: vocabulary and grammar Bµi Bµi 1: Chän mét ®¸p ¸n ®óng nhÊt ®Ó hoµn thµnh mçi c©u sau. X¸c ®Þnh c©u tr¶ lêi b»ng c¸ch khoanh trßn ®¸p ¸n A, B, C hay D (30®) 11. The ......... need help from the rich A. rich B. young C. poor D. injured 12. She didn’t ......... to cry when she was small. A. use B. used C. uses D. using 13. The .............. can’t talk with people. A. deaf B. blind C. injured D. dumb 14. This is the book, .......... is very interesting. A. which B. who C. whom D. when 15. Tomorrow Mai and I ............ to Ha noi. A. are going B. am going C. is going D. will go 16. She looks different. Has she ......... her hair cut? A. have B. has C. had D. having 17. My mother is a doctor .......... works in Tan Yen hospital. A. who B. which C. whom D. when 18. His homework has been ............ A. do B. did C. doing D. done 19. Manh has passed his exams, ........... is good news. A. who B. where C. why D. which 20. Next week my class is going for a ............ in Ha Long Bay. A. tour B. trip C. excursion D. travel 21. My class decided ............. the zoo. A. visit B. to visit C. visited D. visiting 22. Hung practises .......... English everyday. A. speaking B. speak C. spoke D. speaks 60 23. I don’t like......................water. I prefer fruit juice. A. the B. a C. any D. some 24. ..........................you swim? – Yes, I learned how to swim when I was seven. A. can B. able to C. must D. when 25. If he .........................her, she could have had an accident. A. didn’t warn B. hadn’t warned C. warned D. doesn’t warn 26. Julia had a great holiday. She enjoyed.......................... A. oneself B. himself C. myself D. herself 27. We tried a lot of hotels, but ..................of them had any free rooms. A. neither B. none C. no D. not 28. Ann is very busy these days. She has ..................free time. A. a little B. little C. a few D. few 29. If I ...................you, I wouldn’t talk to him any more. A. be B. am C. was D. were 30. He was unwilling to explain the reason ..................his absence. A. for B. why C. of D. that 31. Give me a word.........................with “S”. A. begins B. began C. begin D. beginning 32. It’s not worth ......................out now. It is much too late. A. to go B. it go C. going D. it going 33. It’s at least a month since ...................................John. A. I last seeing B. I have last been C. I last see D. I last saw 34. They invited ........................dinner with them. A. us for having B. us to have C. that we have D. our having 35. He dislikes to .....................the crossword puzzle in the newspaper every day. A. fill B. make C. do D. answer 36. ........................me to phone them before I go out. A. Mention B. Make C. Remember D. Remind 37. Music can express one's anger, love, hate and ................ A. friendness B. friendly C. friendship D. friends 38. How long ago ......... Minh? A. did you see B. see C. have you seen D. you saw 39. Let's get her ............. us some water. A. bring B. brings C. to bring D. bringing 40. I have just bought ............... interesting book. A. some B. the C. a D. an Bµi 2: Chia ®éng tõ trong ngoÆc ë thêi vµ thÓ thÝch hîp råi ®iÒn vµo chç trèng ë trong mçi c©u d−íi ®©y (10 ®iÓm). 41. Water .............................at 100o C. (boil) 42. Let’s go for a walk. It ................................raining 2 hours ago. (stop) 43. Normally my sister ............................work at 5 p.m. (finish) 44. ..............................you..............................any good film lately? (see) 45. I .............................Tom and Nancy a few days ago. (meet) 46. Jane .........................for me when I arrived. (wait) 47. Grandpa.........................to the radio in the living-room now. (listen) 48. At 8 o’clock yesterday evening, I .......................dinner with (have) some of my old friends. 49. This is a big factory. Five hundred people .........................here. (employ) 50.The letter.........................a week ago and it.....................yesterday (post, arrive) Bµi 3: Cho d¹ng ®óng cña nh÷ng tõ ë cuèi mçi c©u (10 ®iÓm). 51. What is the ......................................of this tower, please? (high) 52. The whole country is trying to get rid of ....................... (poor) 53. They are looking for the ..................................to the problem. (solve) 61 54. We should .................................this road for better use. 55. During the past years computers have ................................our life. 56. It is much more ............................to buy that car. 57. A true ............................lasts throughout life. 58. The house is big but it is very ..................................to live in. 59. He is ........... because he misses his parents. 60. His ................................has been considered. (wide) (revolution) (economy) (friend) (comfortable) (happy) (explain) PhÇn iII: iII: reading §äc ®o¹n v¨n sau råi chän ®¸p ¸n ®óng ®Ó ®iÒn vµo mçi kho¶ng trèng (10 ®) The 1990s .. (61) .. great changes in the way people communicate. People could …(62)… mail without going to the post office, and go ….(63)… without leaving home. The cause of this great change was the Internet. The idea for the Internet … (64) ….in the early 1960s in the United States. The Department of Defence wanted …. (65) …., their computers together to share private information. In 1969, the ARPA net, ….(66)….. was an early form of the Internet, first connected computers at 4 American Universities. In 1972, scientists ….(67)…. ARPA net with the world. They created a way to send person – to person messages using ARPA net. This was the beginning of email. In the 1990s, two important things happened: the birth of the World Wide Web (WWW) in 1991 and the creation of the first web browser … (68) …. . 1993. The web made it easier to find … (69) …. on the Internet, and to move from place to place using links. Nowadays, computers are an important part of our lives and are changing how we learn, work, shop and …..(70)…... 61. 62. 63. 64. 65. 66. 67. 68. 69. 70. A. A. A. A. A. A. A. A. A. A. sees send shopping begins to connecting whom shared during information communicating B. B. B. B. B. B. B. B. B. B. saw to send to shopping beginning connect who was sharing on inform to communicating C. C. C. C. C. C. C. C. C. C. has seen sending shop began connecting which had shared at informative communicate D. D. D. D. D. D. D. D. D. D. was seeing sent shopped had begun to connect that have shared in informal to communicate PhÇn iv: iv: writing Bµi 1: 1: T×m lçi trong nh÷ng c©u sau b»ng c¸ch khoanh trßn A, B, C hay D (10®) 71. I am very please that we should meet again soon. A B C D 72. Peter introduced Mai with some of his friends in London. A B C D 73. The police stopped everybody enter the house. A B C D 74. The pilot and the crew divided the life preserves between twenty frantic passengers. A B C D 75. Standing among so many strangers, the frightened child began to sob uncontrollable. A B C D 76. Tom keeps to study hard because he intends to go to dental school. A B C D 62 77. When the bell rang, the students have left the class. A B C D 78. The government has decided voting on the resolution now rather than next month. A B C D 79. I don’t recommend to eat in that restaurant. Its food is awful. A B C D 80. One of the primary cause of road accidents is driving after drinking. A B C D Bµi 2: H·y viÕt l¹i nh÷ng c©u sau ®©y mµ kh«ng thay ®æi nghÜa cña c©u, b¾t ®Çu b»ng côm tõ cho s½n (10 ®iÓm). 81. I started learning English 5 years ago. => I have ............................................................................................................. 82. This is the first time I’ve ever been to Hanoi. => I have never ...................................................................................................... 83. The last time I saw him was in 1995. => I haven’t ........................................................................................................... 84. Harry left before we reached the hotel. => By the time ....................................................................................................... 85. We haven’t seen Ann for years. => It’s years ........................................................................................................... 86. School uniform must be worn at all times by young pupils. => Young pupils ................................................................................................... 87. That school has four hundred pupils. => There ............................................................................................................... 88. The map was so old that I could not read it. => It was such ...................................................................................................... 89. It is against the rules to park your car here. => You are.............................................................................................................. 90. He didn’t hurry, so he missed the train. => If ...................................................................................................................... Bµi 3: §æi nh÷ng c©u sau ®©y sang d¹ng bÞ ®éng (10 ®iÓm). 91. My grandfather built this house in 1950. => ............................................................................................................................... 92. You should clean the room every day. => ........................................................................................................................... 93. People advised us not to go out alone. => ............................................................................................................................... 94. Tom gave me an apple. => ............................................................................................................................... 95. I will never read this book. => .............................................................................................................................. 96. They are fixing the drain in the bathroom. => ................................................................................................................................ 97. People speak English all over the world. => ............................................................................................................................. 98. What have they done? => ............................................................................................................................. 99. You must answer all the questions on the paper. => ............................................................................................................................... 100. He still remembers that his father took him to the zoo every Sunday. => ............................................................................................................................ Good luck to all of you! 63 KEY Phần I: Phonetics 1 2 3 4 5 6 7 8 9 A A C C B A A B C Phần II: vocabulary and grammar Bài 1: 11 C 21 B 31 D 12 A 22 A 32 C 13 D 23 C 33 D 14 A 24 A 34 B 15 A 25 B 35 C 16 C 26 D 36 D 17 A 27 B 37 C 18 D 28 B 38 A 19 D 29 D 39 C 20 B 30 A 40 D 2: 41. boils 46. was waiting 42. stopped 47. is listening 43. inishes 48. was having 44. have.............seen 4 9. are employed 45. met 50. was posted, arrived 3: 51. height 56. economical 52. poverty 57. friendship 53. solution 58. uncomfortable 54. widen 59. hardship 55. revolutionized 60. explanation III: reading 61 62 63 64 65 6 67 68 69 B A A C D C A D A IV: writing 1: 71 72 73 74 75 76 77 78 79 B B D C D A C B B 2: 81. I have learned/ have been learning English for five years (now). 8 2. I have never been to Hanoi before. 8 3. I haven’t seen him since 1995. 84. By the time we reached the hotel Harry had left. 85. It’s years since we last saw Ann. 86. Young pupils must wear school uniform all the time/at all times. 87. There are four hundred pupils in that school. 88. It was such an old map that I couldn’t read it. 89. You are parking your car against the rules/ you are not allowed to park your car here. 90. If he had hurried, he wouldn’t have missed the train. Bµi 3: 91. This house was built (by grandfather)in 1950. 92. Your room should be cleaned every day. 93. We were advised not to go out alone. 94. I was given an apple/ an apple was given to me. 95. This book will never be read. 96. The drain in the bathroom is being fixed. 97. English is spoken all over the world. 98. What has been done? 99. All the questions must be answered on the paper. 100. He still remembers that he was taken to the zoo every Sunday (by his father). 10 C 70 C 80 B 64 SỞ GIÁO DỤC VÀ ĐÀO TẠO QUẢNG NAM TRƯỜNG THPT CHUYÊN NGUYỄN BỈNH KHIÊM KỲ THI OLYMPIC TRUYỀN THỐNG 30 - 4 LẦN THỨ 15 ĐỀ THI ĐỀ NGHỊ MÔN: TIẾNG ANH LỚP 10 MULTIPLE CHOICE QUESTIONS I. Phonology (5ms) A. Choose the word whose underlined part is pronounced differently from the others 1. A. parachute B. champagne C. chivalry D. churchgoer 2. A. inflexible B. elegant C. experiment D. recollection 3. A. breakfast B. already C. feasible D. heather 4. A. resign B. resound C. resonant D. resource 5. A. cleared B.hatred C. sacred D. wicked B. Choose the word whose stress pattern is different from that of the others in each group. Circle A, B, C or D to indicate the correct answer. 6. A. development B. competitive C. immediate D. sympathetic 7. A. vocational B. employer C. minority D. reasonable 8. A. analysis B. destination C. presidentia D. apprehension 9. A. exploration B. facilities C. scientific D. educational 10. A. temperature B. acquaintance C. photography D. distinctive KEY: 1 2 D D 3 C 4 D 5 A 6 D 7 D 8 A 9 B 10 A II. Reading comprehension (20ms) Read the passages and choose the best answer Passage A As Christmas evolved in the United States, new customs were adopted and many old ones were reworked. The legend of Santa Claus, for example, had origins in Europe and was brought by Dutch settlers to New York in the early 18th century. Traditionally, Santa Claus – from the Dutch Sinter Klaas – was depicted as a tall, dignified, religious figure riding a white horse through the air. Known as Saint Nicholas in Germany, he was usually accompanied by Black Peter, an elf who punished disobedient children. In North America he eventually developed into a fat, jolly old gentleman who had neither the religious attributes of Saint Nicholas nor the strict disciplinarian character of Black Peter. Santa’s transformation began in 1823, when a New York newspaper published the poem A Visit from Saint Nicholas, which Clement Clark Moore had written to amuse his daughter. The poem introduced many Americans to the story of a kindly saint who flew over housetops in a reindeer-drawn sleigh. Portraits and drawings of Santa Claus by American illustrator Thomas Nast further strengthened the legend during the second half of the 19th century. Living at the North Pole and assisted by elves, the modern Santa produced and delivered toys to all good children. By the late 19th century he had become such a prominent figure of American folklore that in 1897, when Virginia O’Hanlon wrote to the New York Sun newspaper asking if Santa was real, she received a direct answer: “Yes, Virginia, there is a Santa Claus”. 1. Who brought the legend of Santa Claus to the USA according to the passage? A. Sinter Klaas B. Saint Nicholas C. A German D. Dutch settlers 2. Santa Claus was traditionally described as a A. tall man who could walk through the air 65 3. 4. 5. 6. 7. 8. 9. B. fat, jolly, old man C. religious figure D. fat man riding a white horse Santa Claus in North America was depicted as A. a man with the strict disciplinarian character of Black Peter B. a good old man with less religious character C. one with religious attributes of Saint Nicholas D. a jolly man on horseback Who was Black Peter? A. an elf accompanying Saint Nicholas B. an elf who rode a white horse C. one of the disobedient children D. a popular traditional figure What word is closest in meaning to attributes? A. symbols of a person B. natural qualities C. effects D. outer appearance Where did the legend of Santa Claus come from? A. the North Pole B. Europe C. North America D. the City of New York 1823 was mentioned as a year when A. Clement Clark Moore wrote his first poem B. Clement Clark Moore’s poem made him popular C. Saint Nicholas visited New York D. the image of Santa Claus was transformed According to Clement Clark Moore’s poem A. Santa Claus had nothing different in appearance from the traditional one B. Santa Claus had wings and could fly C. Santa Claus liked poetry D. Santa Claus was a kindly saint who flew over housetops in a sleigh The answer Yes, Virginia, there is a Santa Claus is an illustration for the fact that A. the New York Sun was popular with children B. Santa Claus was a prominent figure at that time C. newspapers are unreliable D. Virginia O’Hanlon was a reader of the New York Sun 10. Which of the following statements is TRUE? A. Santa Claus was an imaginary old man created by artists based on traditional figures B. Living in the North Pole, Santa Claus visited children at Christmas C. Santa Claus was a real figure living in northern America D. Santa Claus was a story based on Saint Nicholas and Black Peter KEY: 1. D 2.C 3. B 4. A 5. B 6. B 7D 8D 9. B 10. A Passage B Alfred Bernhard Nobel, a Swedish inventor and philanthropist, bequeathed most of his vast fortune in trust as a fund from which annual prizes could be awarded to individuals and organizations who had achieved the greatest benefit to humanity in a particular year. Originally, there were six classifications for outstanding contributions designated in Nobel’s will, including chemistry, physics, physiology or medicine, literature and international peace. The prizes are administered by the Nobel Foundation in Stockholm. In 1969, a prize for economics endowed by the Central Bank of Sweden was added. Candidates for the prizes must be nominated in writing a qualified authority in the filed of competition. Recipients in physics, chemistry, and economics are selected by the Royal Swedish Academy of Sciences; and physiology or medicine by the Caroline Institute; in literature by the Swedish Academy; and in peace by the Norwegian Nobel 66 Committee appointed by Norway’s parliament. The prizes are usually in Stockholm on December 10, with the King of Sweden officiating, an appropriate tribute Alfred Nobel on the anniversary of his death. Each one includes a gold medal, a diploma, and a cash award of about one million dollars. 1: What does this passage mainly discuss? A. The Nobel prizes B. Alfred Bernhard Nobel C. Swedish philanthropy D. Great contributions to mankind 2: Why were the prizes named for Alfred Bernhard Nobel? A. He won the first Nobel prize for his work in philanthropy B. He is now living in Sweden C. He left money in his will to establish a fund for the prizes D. He serves as chairman of the committee to choose the recipients 3: How often are the Nobel prizes awarded? A. Five times a year B. Once every two years C. Once a year D. Twice a year 4: The word “outstanding” in line 3 most closely means _______. A. recent B. exceptional C. unusual D. established 5: The word “will” in line 4 could best be replaced by _____. A. Nobel’s wishes B. a legal document C. a future intention D. a free choice 6: A Nobel prize would NOT be given to _______. A. an author who wrote a novel B. a doctor who discovered a vaccine C. a diplomat who negotiated a peace D. a composer who wrote a symphony 7: The word “one” in paragraph 2 refers to _____. A. tribute B. anniversary C. candidate D. prize 8: The word “appropriate” in the passage most closely means _______. A. prestigious B. suitable C. customary D. transitory 9: Which individual or organization serves as administrator for the trust? A. The King of Sweden B. The Nobel Foundation C. The Central Bank of Sweden D. Swedish and Norwegian academies and institutes 10: Why are the awards presented on December 10? A. It is a tribute to the King of Sweden. B. Alfred Bernhard Nobel died on that day. C. That date was established in Alfred Nobel’s will. D. The Central Bank of Sweden administers the trust KEY: 1 2 A C 3 C 4 B 5 B 6 D 7 D 8 B 9 B 10 B III. Guided cloze test Choose the best word that fits each gap of the passage. (10ms) National Health Service (NHS) (1) ________ ambulances are available and free for cases of sudden (2) ________ or collapse, for accidents and for doctors’ urgent call. Special (3) _________ services, such as free dental treatment and (4) _______ and immunization of children against certain (5) ________ diseases, are provided under the NHS to safeguard the health of (6) _______ women and young children. Free family planning advice and (7) ________ is available from general practices (GPs) and family planning clinics. Nation-wide screening programmes for breast canner and cervical cancer are available to the most (8) _______ age groups among women. Improvements in the provision and (9) 67 ________ of cancer services are a major priority. The blood (10) _______ services collect over 2.5 million donations a year from voluntary unpaid donors. 1. 2. 3. 4. 5. 6. 7. 8. 9. 10. KEY: 1. B A. emergent A. illness A. prevent A. vaccine A. infect A. pregnan A. treat A. unsuspecting A. avail A. transfuse 2. A 3. C B. emergency C. urgent D. urgency B. disease C. pain D. sore B. preventing C. preventive D. prevention B. vaccinate C. vaccinating D. vaccination B. infecting C. infection D. infectious B. pregnancy C. expecting D. expectant B. treatment C. treating D. maltreatment B. susceptible C. susceptibility D. susceptibly B. available C. availability D. unavailable B. transfused C. transfusing D. transfusion 4. D 5. D 6. A 7. B 8. B 9. C 10. D WRITTEN TESTS I. Verb tenses Choose the correct verb tense for each gap of the passage. (10ms) This time last year I (1) ______ in the rain along a country road in France with a friend of mine. We (2) _____ to go on a cycling holiday in Normandy. Neither of us (3) ______ to France before, but we (4) _____ some French from our time at school and we (5) _____ to brush up on the basics. Now we (6) _____ if we (7) _____ the right decision. We (8) _____ our route carefully in advance, but we (9) ______ one important thing, the weather. It (10) _____ solidly since our arrival and that night we (11) ______ sleeping in the waiting room at a railway station. Then the next morning as we (12) ______ down a steep hill my bike (13) _____ on the wet road and I (14) ______. I (15) ______ immediately that I (16) ______ my arm, and after a visit to the local hospital I (17) ______ the next train to Calais for the ferry home. Unfortunately my parents (18) ______ me home for a fortnight, and (19) _____ away on holiday. So I (20) _____ a miserable couple of weeks alone, reading Teach Yourself French. 1. A. cycled B. was cycling 2. A. decided B. have decided 3. A. have been B. had been 4. A. know B. knew 5. A. managed B. have managed 6. A. wonder B. are wondering 7. A. make B. made 8. A. planned B. have planned 9. A. forgot B. had forgotten 10. A. has rained B. has been raining 11. A. ended up B. had ended up 12. A. rode B. were riding 13. A. skidded B. was skidded 14. A. fell off B. was falling off 15. A. realize B. have realized 16. A. broke B. have broken 17. A. caught B. have caught 18. A. didn’t expect C. haven’t expected 19. A. went B. had gone 20. A. spent B was spending C. had cycled C. had decided C. went C. have known C. would manage C. will wonder C. have made C. had planned C. have forgotten C. was raining C. have ended up C. had ridden C. was skidding C. was fallen off C. realized C. had broken C. had caught B. weren’t expecting D. hadn’t expected C. were going C. have spent D. had been cycling D. had been deciding D. was going D. had known D. had managed D. were wondering D. had made D. had been planning D. were forgetting D. had rained D. would end up D. had been riding D. was being skidded D. had fallen off D. had realized D. had been breaking D. would catch D. have gone D. have been spending KEY: 68 1. B 11. A II. 2. C 3. B 4. B 5. D 6. D 7. D 8. C 9. B 10. D 12. B 13. A 14. A 15. C 16. C 17. A 18. B 19. B 20. A Prepositions and phrasal verbs (10ms) Choose the correct preposition/ particle for each gap of the passage. The Moscow metro system is over sixty years old. Sometimes called “the people’s underground palace”, it has stood (1) ____ well to the passing of time. It carries nine million passengers a day at an average speed of 42 kilometers per hour, and now, even though it has run (2) ____ of money, it still offers impressive examples of past splendor. Tourists from all over the world come to look at the stations lined with granite and marble and decorated with mosaics and chandeliers. Some of the names have changed: for example, Marx Prospekt has turned (3) ____ Hunter’s Row. Some Muscovites find it hard to take (4) ___ the changes, but they are justly proud of their underground and want to look (5) ____ it. Recently a strike was threatened, to force the government to invest in the system. The plan came (6) ____, and so a new line and new stations will be built. The Moscow metro stands (7) ____ among its international competitors for being almost completely free of advertising. Although that will change soon, transport managers have promised to see (8) ____ it that any advertising will be done tastefully. Wherever you go in the world, you won’t come (9) ____ a grander or more impressive underground system. Whether you’re a local, seeing your friends (10) ____ or traveling to work, or a tourist, visiting Moscow for the first time, spare a moment to appreciate the works of art all around you, before going on with your journey. 1. A. up 2. A. away 3. A. back 4. A. up 5. A. round 6. A. off 7. A. over 8. A. through 9. A. up 10. A. forward B. out B. down B. into B. down B. into B. out C. up B. out B. to B. out B. off C. by D. in C. up C. out D out D. against C. in D. on C. for D. after D. in C. up D. for C. off D. round C. across D. to C. out D. away KEY: 1.C; 2.A; 3.A; 4.D; 5.B; 6.A; 7.B; 8.D; 9.C; 10.C Last week I went to visit my friend Fred. Fred is a great guy but at times he can really go (1)………… about things. We were speaking about some of our friends and he came (2)………… this incredible story about Jane. It seems she had butted (3)………… while he was harping (4)………… his favorite complaint: Service in restaurants. Apparently, he had been running (5)………… for quite a while putting (6)………… almost every restaurant he had been to by rattling (7)………… a list of his visits to different restaurants in town. I guess Jane felt that he was talking at her and was fed up with it. She went off about what a rude person he was which shut him (8)………… pretty quickly! I thought about blurting (9)………… that maybe she was right, but decided to clam (10)………… in order to not upset him. 1. A. at 2. A. out with 3. A. in 4. A. around 5. A. to 6. A. down 7. A. of B. in B. down B. of B. up B. on B. about B. off C. on C. about C. off C. into C. out with C. after C. on D. up D. under D. out D. on D. in D. up D. outside 69 8. A. out of 9. A. around 10. A. at KEY: 1. A III. B. on B. up B. on 2. D 3. B 4. C C. into C. out C. up 5. D 6. A 7. B 8. B D. up D. at D. in 9. C 10. B Word forms (20 ms) Supply the correct form of the word in capital letter. (10 points) 1. He suffered from constant ............ . SLEEP 2. The boy was very violent and his parents found him ............. . MANAGE 3. People in the village no longer use water in that well because of its…....... . PURE 4. “Would you like some ………?” - Lan asked. You must be tired after that long walk. REFRESH 5. We have to take care of our own …… for nobody else will do this for us. POSSESS. 6. The city has over million ………….. INHABIT 7. Be careful ! That’s a .............. poison. DEAD 8. Street noise is one of the ………… of living in a big city. ADVANTAGE 9. If we could find out the ……… for cancer, thousands of people would be saved every year. PREVENT 10. The talks were totally .................. . We didn’t reach agreement on anything. PRODUCE KEY: sleeplessness 2. unmanageable 3. impurity 4. refreshments 5. possessions 6.inhabitants 7.deadly 8. disadvantages 9. prevention 10. unproductive Use the correct form of the word given in capitals to complete the numbered spaces in the passage below. 1. synonym 3. hand 5. mathematics 7. own 9. stand 2. rely 4. front 6. position 8. reside 10. likely King of the Watchmakers For a period of its history, the city of Coventry had a considerable reputation as the main center of clock and watch-making in Britain, and Coventry timepieces made then were (1) ............ with both quality and (2) .......... . Few people in the city today will have heard of Samuel Watson, but he almost (3) ............. paved the way for Coventry’s involvement in the clock and watch business. He was at the (4) ................ of the watch-making revolution in the 1680s, and although it is not known how Watson became involved in the trade, he was a trailblazer for others. Watson made his name in 1682 when he sold a clock to King Charles II and was invited to be the King’s (5) ......................... . The following he began work on an astronomical clock for the King, complete with planets and signs of the zodiac, which took seven years to build. It not only told the time of day but also the (6) ......................... changes of the planets. Queen Mary acquired it in 1691 and it is still in the (7) ........... of the Royal Family. He built several other clocks, and by 1690 the clamor for Watson’s clocks was such that he left Coventry and took up (8) ........... in London. He became Master of the London Clockmakers’ Company in 1692, which is testament to his (9) ............. in the growing industry. In 1712, Samuel Watson’s name disappears from the records of the London Clockmakers’ Company, and the (10) ................ is that he died in that year. KEY: 1. synonymous 3. single-handedly 5. mathematician 7. ownership 9. standing 2. reliability 4. forefront 6. positional 8. residence 10. likelihood IV. Error identification (10 ms) 70 The passage below contains 10 errors. Underline and correct them. (00) has been done as an example. Leonardo DiCaprio is one of the hotter young film stars around at the moment. His face has been on the covers of all the top movies and young magazines over the last few months and he has been the subject of countless articles, rumours and showbiz gossip. Leonard doesn’t like reading about him because “I read things about me that I’ve never said in my life and never did" . 00 hottest...... ........... . Leonardo DiCaprio was born in Los Angeles on 11 November, 1974. He’s a Scorpio. His full name is Leonardo Wilhelm DiCaprio. His mother is Germany and his father Italian-American. They called him Leonardo because when his mother was still pregnant, he started kicking while she was stood in front of a painting by Leonardo Da Vinci. His friends call him Leo. He has a scar from when he was stinging by a Portuguese man-of-war. His parents separated before he was born, so his mother moved to a poor neighbourhood of Hollywood there Leo grew up . At school he was very good at imitating people, especially Michael Jackson. This made him very popularly. His childhood hero was Poseidon, the Greek god of the sea. After appearance in TV commercials and episodes of Roseanne, he played the cast of Roseanne, the TV sitcom starring Kirk Cameron. Leonard played the part of Luke, a homeless boy. Lately, he played the part of Jim Carroll in The Basketball Diaries. But he has really become famous since he acted in the film Titanic KEY: 1. young --> youth 2. him --> himself 4.Germany--> German 5. stood standing 7. there --> where 8. popularly -->popular 9. appearance --> appearing 10. Lately --> Later 3. did--> done 6. stinging--> stung V.Open cloze tests Fill in each numbered space with one appropriate word Albert Einstein Best known (1) …… his Theory of Relativity, Albert Einstein has been acknowledged (2) …… one of the greatest scientists in human society. Born in Germany in 1879, Einstein did (3) …… in all his school subjects, with the (4) …… of mathematics, which was his (5) …… subject. While (6) …… as a clerk in Switzerland, Einstein produced the Special Theory of Relativity. This theory states that mass, length and time change depending on (7) …… a person looks at them. In 1915, Einstein published his General Theory of Relativity. This new theory created a controversy (8) …… many scientists did not agree with him. In the end, simple experiments (9) …… Einstein to be correct. Einstein was (10) ……the Nobel Prize for his work in physics. KEY: 1. for 6. working 2. as 7. how 3. poorly 8. because 4. exception 9. proved 5. favourite/ favorite 10. awarded Read the following passage and use ONE word to fill in each gap. The Great (1)………………of Giza was built as a (2)………..of Pharaoh Cheops in 2720 BC. (3)……………..it was built a long time ago, its (4)………………….makes it one of the true (5)…………..of the world. The four (6)…………….of the pyramid are put almost exactly on true north, south, east and 71 west. Explorations and detailed examinations of the base of the structure reveal many interesting lines. Further (7)……….. study indicates that (8)…………..represent a type of line of events-past, present and future. Many of the (9)………….have been interpreted and found to coincide with known facts of the past. Was this super structure made by ordinary beings or one built by a race (10)…………superior to any known today? KEY: 1. pyramid 2. tomb 3. Although 4. construction 5. wonders 6. sides 7. scientific 8. these 9. events 10. far VI. Sentence transformation (20 ms) Finish each of the following sentences in such a way that it is as similar as possible in meaning to the original sentence. Use the word given and other words necessary. Do not change the form of the given word. 1. The Yeti has very rarely been seen at this attitude. There have____________________________________________ 2. It is not certain that Jones will get the job. It is open ____________________________________________ 3. Everyone started complaining the moment the announcement was made. No sooner ____________________________________________ 4. As I get older, I want to travel less. The older _____________________________________________ 5. After six-year relationship, Martha and Billy have decided to get married. (KNOT) _____________________________________________________ 6. At this very moment my wife may be arriving back home. (PROBABLY) _____________________________________________________ 7. Noise is something that it is not possible for me to tolerate. (PUT) _____________________________________________________ 8. He is becoming quite famous as an interviewer. (NAME) _____________________________________________________ 9. He makes sure that he isn't associated with policies he disagrees with. (DISTANCES) _____________________________________________________ 10.I really don't know what you're talking about. (FAINTEST) KEY: 1. There have been very few sightings of the Yeti having been seen at this attitude. 2. It is open to doubt whether Jones will get the job. 3. No sooner had the announcement been made than everyone started complaining. 4. The older I get, the less I want to travel. 5. After six-year relationship, Martha and Billy have decided to tie the knot. 6. At this very moment my wife is probably arriving back home. 7. I find it impossible to put up with noise. 8. He is making quite a name for himself as an interviewer. 9. He makes sure (that) he distances himself from policies he disagrees with. 10. I don't have the faintest idea what you're talking about The end 72 §Ò thi m«n tiÕnganh (§Ò 1) 1: Choose the best answer to complete the following sentence : Lan is the........of the two girls. A. prettiest B. prettier C. pretty D. most pretty 2: Choose the best answer to complete the following sentence : To lack means to................ A. be without B. keep secret C. be with D. be weak Read the passage below and choose the best answer for each space ( from number 03 to number 15): A year with overseas volunteers I was with Overseas Volunteers (OV) for a year after leaving university, and I was sent to an isolated village in Chad, about 500 kilometers, from the capital. Coming from a rich country, I got quite a shock as conditions were much harder than I had expected. But after a few days I soon got used to..(3)… there. The people were always very friendly and helpful, and I soon began to appreciate how…(4)… the countryside was. One of my job was to supply the village with water. The well was a long walk away, and the women used to…(5) a long time every day..(6)..heavy pots backwards and forwards. So I contacted the organization and arranged to…(7)..some pipes delivered. We built a simple pineline and a pump, and it worked first time. It wasn’t perfect- there were a few..(8), but it made a great difference to the villagers, …(9) had never had running water before. And not …(10)did we have running water, but in the evening it was hot, because the pipe had been…..(11) in the sun all day. All in all, I think my time with OV was a good experience. Although it was not well-paid, it was well…(12)doing, and I would recommend it to anyone who was…(13) working for a charity. Finally, there’s one more reason why I will never….(14) working for OV. A few months before I left, I met and fell in love…(15)another volunteer, and we got married when we returned to England. C©u 3. A. live B. lived C. living D. lives C©u 4. A. beautiful B. good-looking C. handsome D. sweet C©u 5. A. spend B. spent C. spends D. spending C©u 6. A. fetching B. wearing C. carrying D. holding C©u 7. A. make B. let C. have D. allow C©u 8. A. breaks B. leaks C. splits D. punctures C©u 9. A. which B. that C. they D. who C©u 10. A. hardly B. scarcely C. only D. also C©u 11. A. lied B. lay C. laying D. lying C©u 12. A. worth B. value C. cost D. price C©u 13. A. considering B. thinking C. going D. planning C©u 14. A. regret B. feel sorry C. miss D. lose C©u 15. A. at B. to C. for D. with C©u 16: Choose the best answer to complete the following sentence : Mr. Brown.......around the country if he saved enough money. B. would travel C. has traveled D. traveled A. will travel C©u 17: Choose the word with the same meaning as the underlined part : I have’t seen your brother recently. A. suddenly B. immediately C. quietly D. lately C©u 18: Find a word which is pronounced differently in the part underlined A. superb B. suppose C. Support D. Supply C©u 19: Choose the word with the same meaning as the underlined part : We are very anxious about the result of the exam. A. careful B. careless C. excited D. worried C©u 20: Choose the best answer to complete the following sentence : Can you give me a.......for a fruit cake. A. prescription B. receipt C. recipe D. description C©u 21: Choose the best answer to complete the following sentence : 73 Don’t touch that wire or you may get an electric......... . A. current B. shock C. attack D. feeling C©u 22: There is a mistake in the four underlined part of the sentence. Find the mistakes ( A,B, C, or D) : If I (A) find some money (B) on the street, I’d (C) take it to the (D) police station. C©u23: Choose a word that has different stress pattern A. government B. annoyance C. experiment D. participate C©u 24: Read the following list of four words and choose one word that does not belong in the list : A. Listen B. Talk C. Spell D. Pronounce Choose the best answer to complete the following sentence : C©u 25: The teacher says, ‘The time is up’ .This means that the period of time has......... A. begun B. ended C. been interesting D. seemed long C©u26: Choose the best answer to complete the following sentence : When I was turning out my cupboard I........this photograph of Uncle Ba A. came round B. came across C. came over D. came into C©u 27: Choose the best answer to complete the following sentence : Our flat is very small. We wish we.......another bedroom. A. have B. had C. have had D. will have C©u 28: Choose one sentence that has different meaning to the root one : The picture is so high that she can’t reach it. A. The picture is too high for her to reach. B. She is not tall enough to reach the picture. D. She is such a tall girl that she can reach the picture. C. She is too short to reach the picture. C©u 29: Choose the word that has the underlined part pronounced differently from the rest : A. studious B. adult C. public D. thunder C©u 30: Choose the best answer to complete the following sentence : What are you cooking in that saucepan ? It........good. A. makes B. smells C. feels D. flavors C©u 31: Choose one sentence that has different meaning to the root one : The square clock costs $5. The round clock costs $7. A. The round clock is more expensive than the B. The square clock is cheaper than the round square clock. clock. C. The square clock doesn’t cost as much as the D. The round clock costs less than the square round clock. clock. C©u 32: Choose one sentence that has different meaning to the root one : You remembered to post the letter for me, didn’t you ? A. Did you remember to post the letter for me? B. You didn’t forget to post the letter for me, did you? C. Didn’t you forget to post the letter for me? D. You forgot to post the letter for me, didn’t you? C©u33: Choose one sentence that has the same meaning as the root one : If he doesn’t promise to try harder, I won’t help him. A. If he promises to try harder, I will never help B. If he promised to try harder, I wouldn’t help him. him. C. Unless he promises to try harder, I will never D. Unless he doesn’t promise to try harder, I help him. will help him. C©u34: Choose the best answer to complete the following sentence : The High Street is so narrow that the Council have to.......it. A. increase B. widen C. extend D. lengthen C©u35: Choose the word with the same meaning as the underlined part : The Vietnamese people workship their ancestors. A. heirs B. elders C. fossils D. forefathers C©u 36: Choose the word with the same meaning as the underlined part : After the alien spacecraft hovered over the park for a short while, it vanished. A. landed B. rose C. attacked D. disappeared 74 C©u 37 : A. C©u 38: A. C©u 39: A. C. C©u 40 : A. C. C©u 41: A. C©u 42: A. C©u 43: A. C©u 44: C©u 45: A. C©u 46: A. C. C©u 47: A. C©u 48: A. C©u 49: A. C. C©u 50: C©u 51: A. C©u 52: Choose the best answer to complete the following sentence : Another word for baggage is............. luggage B. equipment C. carriage D. furniture Find a word which is pronounced differently in the part underlined: stage B. village C. manage D. baggage Choose the best answer to complete the following sentence : Phong has worked in London for three years and he speaks English......... very by well now B. by well very now very well by now D. by very well now Choose one sentence that has the same meaning as the root one : I don’t normally have coffee for breakfast. I used to have breakfast with coffee. B. I get used to having breakfast with coffee. I am not used to having coffee for breakfast D. I didn’t use to have coffee for breakfast Choose a word that has different stress pattern deforestation B. documentary C. representiative D. satisfactory Choose the word with the same meaning as the underlined part : I think his story is completely incredible. insufficient B. incomparable C. unimportant D. unbelievable Read the following list of four words and choose one word that does not belong in the list : gloves B. boots C. sandals D. shoes There is a mistake in the four underlined part of the sentence. Find the mistakes ( A, B, C, or D) : We (A) reached the (B) nearest village after (C) walk for (D) five hours. Choose the word that has the underlined part pronounced differently from the rest : sugar B. symbol C. sample D. solar Choose one sentence that has the same meaning as the root one : Both Jane and her sister speak Vietnamese fluently. Jane speaks Vietnamese more fluently than B. Jane doesn’t speak Vietnamese as her sister does. fluently as her sister. Jane speaks Vietnamese fluently but her sister D. Jane speaks Vietnamese fluently and doesn’t. so does her sister. Choose the word with the same meaning as the underlined part : The company asked for additional information. necessary B. emphatic C. certain D. further Choose the best answer to complete the following sentence : If Mai comes to England, it will be a good........for her to improve her English. possibility B. opportunity C. advantage D. experience Choose one sentence that has the same meaning as the root one : Despite meeting a lot of difficulties, they finally went through the forest. In spite of the fact that meeting a lot of B. Even though meeting a lot difficulties, difficulties, they finally went through the they finally went through the forest. forest Although they met a lot of difficulties, they D. Though they met a lot of difficulties, but finally went through the forest. they finally went through the forest. There is a mistake in the four underlined part of the sentence. Find the mistakes ( A, B, C, or D) : I (A) asked her (B) whether (C) was there a fridge (D) in the kitchen. Choose the best answer to complete the following sentence : Mary has gone to the baker’s to buy a.........of bread. loaf B. pound C. packet D. bit Choose the best answer to complete the following sentence : The man......we met on tthe train was the headmaster. 75 A. whose B. whom C. where D. which C©u 53: Choose the best answer to complete the following sentence : Some authors are too.....to criticism. B. sensitive C. sensational D. senseless A. sensible C©u 54: There is a mistake in the four underlined part of the sentence. Find the mistakes ( A, B, C, or D) : (A) It’s going to be (B) rainy in Tokyo, (C) that is the (D) capital city of Japan. C©u 55: There is a mistake in the four underlined part of the sentence. Find the mistakes ( A, B, C, or D) : (A) My father’s motorbike is (B) using (C) too many gas (D) these days. Read the passage below and choose the best answer for each question (from number 56 to 60): Last year we had a nice holiday. My friend and I went to the seaside for a month. I had been to the seaside several times before, but this was the first time for my friend. Naturally, it was a great event for him. Finally, the days came. It was a fine morning. We got up very early because we wanted to leave home after breakfast. We made the journey by car. We reached the seaside at noon. We spent many hours on the beach. We enjoyed making castles and channels in the sand. People said we ought to spend at least a few weeks at the seaside. If we could stay longer, so much the better. C©u 56:.How long did the writer and his friend spend at the seaside? A. a week B. a few weeks C. a month D. a few months C©u 57:. Who went to the seaside the first time? A. The writer B. The writer’s friend C. The writer’s family D. The writer and his friend C©u 58:. What was the weather like on the day they started their journey to the seaside? A. it was bad B. It was rainy C. it was snowy D. It was good C©u 59:. How did they travel to the seaside? A. by car B. By train C. by bus D. By air C©u 60:. When did they reach the seaside? A. at 9 a.m B. at 12 o’clock C. at 4 p. m D. at 8 p. m §Ò thi m«n tienganh (§Ò 2) C©u 1 : Choose the best answer to complete the following sentence : The teacher says, ‘The time is up’ .This means that the period of time has......... A. begun B. ended C. been interesting D. seemed long C©u 2 : Choose the best answer to complete the following sentence : Don’t touch that wire or you may get an electric......... . A. shock B. attack C. current D. feeling C©u 3 : Choose the word with the same meaning as the underlined part : I have’t seen your brother recently. A. lately B. immediately C. suddenly D. quietly C©u 4 : Choose the best answer to complete the following sentence : What are you cooking in that saucepan ? It........good. A. flavors B. smells C. feels D. makes C©u 5 : Choose the word that has the underlined part pronounced differently from the rest : A. public B. studious C. adult D. thunder C©u 6 : Choose the word with the same meaning as the underlined part : I think his story is completely incredible. A. insufficient B. incomparable C. unbelievable D. unimportant C©u 7 : Choose one sentence that has the same meaning as the root one : Both Jane and her sister speak Vietnamese fluently. A. Jane speaks Vietnamese more fluently than her B. Jane doesn’t speak Vietnamese as fluently sister does. as her sister. C. Jane speaks Vietnamese fluently but her sister D. Jane speaks Vietnamese fluently and so doesn’t. does her sister. 76 C©u 8 : There is a mistake in the four underlined part of the sentence. Find the mistakes ( A, B, C, or D) : If I (A) find some money (B) on the street, I’d (C) take it to the (D) police station. C©u 9 : Find a word which is pronounced differently in the part underlined A. superb B. suppose C. Support D. Supply C©u 10 : Choose the best answer to complete the following sentence : Can you give me a.......for a fruit cake. A. recipe B. receipt C. prescription D. description C©u 11 : There is a mistake in the four underlined part of the sentence. Find the mistakes ( A, B, C, or D) : (A) It’s going to be (B) rainy in Tokyo, (C) that is the (D) capital city of Japan. C©u 12 : Choose the best answer to complete the following sentence : The High Street is so narrow that the Council have to.......it. B. widen C. extend D. lengthen A. increase C©u 13 : Choose the best answer to complete the following sentence : Lan is the........of the two girls. A. prettier B. pretty C. most pretty D. prettiest C©u 14 : Choose the best answer to complete the following sentence : Mary has gone to the baker’s to buy a.........of bread. A. bit B. pound C. packet D. loaf C©u 15 : Choose the word that has the underlined part pronounced differently from the rest : A. sample B. symbol C. sugar D. solar C©u 16 : Choose a word that has different stress pattern A. government B. experiment C. annoyance D. participate C©u 17 : There is a mistake in the four underlined part of the sentence. Find the mistakes ( A, B, C, or D) : We (A) reached the (B) nearest village after (C) walk for (D) five hours. C©u 18 : Choose the best answer to complete the following sentence : Another word for baggage is............. A. carriage B. equipment C. luggage D. furniture C©u 19 There is a mistake in the four underlined part of the sentence. Find the mistakes ( A, B, C, : or D) : I (A) asked her (B) whether (C) was there a fridge (D) in the kitchen. C©u 20 : Choose the best answer to complete the following sentence : Our flat is very small. We wish we.......another bedroom. A. have had B. had C. will have D. have C©u 21 : There is a mistake in the four underlined part of the sentence. Find the mistakes ( A, B, C, or D) : (A) My father’s motorbike is (B) using (C) too many gas (D) these days. C©u 22 : Choose one sentence that has the same meaning as the root one : I don’t normally have coffee for breakfast. B. I used to have breakfast with coffee. A. I get used to having breakfast with coffee. C. I am not used to having coffee for breakfast D. I didn’t use to have coffee for breakfast C©u 23 : Choose the best answer to complete the following sentence : If Mai comes to England, it will be a good........for her to improve her English. A. experience B. possibility C. advantage D. opportunity C©u 24 : Find a word which is pronounced differently in the part underlined: A. village B. manage C. stage D. baggage C©u 25 : Choose the best answer to complete the following sentence : The man......we met on tthe train was the headmaster. A. whose B. where C. which D. whom Choose one sentence that has different meaning to the root one : C©u You remembered to post the letter for me, didn’t you ? 26: A. Did you remember to post the letter for me? B. You forgot to post the letter for me, didn’t you? C.You didn’t forget to post the letter for me, did you? D. Didn’t you forget to post the letter for 77 me? Read the passage below and choose the best answer for each question ( from number 27 to numberm31): Last year we had a nice holiday. My friend and I went to the seaside for a month. I had been to the seaside several times before, but this was the first time for my friend. Naturally, it was a great event for him. Finally, the days came. It was a fine morning. We got up very early because we wanted to leave home after breakfast. We made the journey by car. We reached the seaside at noon. We spent many hours on the beach. We enjoyed making castles and channels in the sand. People said we ought to spend at least a few weeks at the seaside. If we could stay longer, so much the better. C©u 27.How long did the writer and his friend spend at the seaside? A. a week B. a few weeks C. a month D. a few months C©u 28. Who went to the seaside the first time? A. The writer B. The writer’s friend C. The writer’s family D. The writer and his friend C©u 29. What was the weather like on the day they started their journey to the seaside? A. it was bad B. It was rainy C. it was snowy D. It was good C©u 30. How did they travel to the seaside? A. by car B. By train C. by bus D. By air C©u 31. When did they reach the seaside? A. at 9 a.m B. at 12 o’clock C. at 4 p. m D. at 8 p. m C©u 32 : Read the following list of four words and choose one word that does not belong in the list : A. sandals B. boots C. gloves D. shoes C©u 33 : Choose the word with the same meaning as the underlined part : The Vietnamese people workship their ancestors. B. forefathers C. elders D. heirs A. fossils C©u 34 : Choose one sentence that has different meaning to the root one : The square clock costs $5. The round clock costs $7. B. The round clock costs less than the square A. The round clock is more expensive than the square clock. clock. C. The square clock is cheaper than the round D. The square clock doesn’t cost as much as the clock. round clock. C©u 35 : Choose one sentence that has the same meaning as the root one : Despite meeting a lot of difficulties, they finally went through the forest. A. Even though meeting a lot difficulties, they B. In spite of the fact that meeting a lot of finally went through the forest. difficulties, they finally went through the forest C. Though they met a lot of difficulties, but they D. Although they met a lot of difficulties, they finally went through the forest. finally went through the forest. C©u 36 : Choose the best answer to complete the following sentence : To lack means to................ A. be without B. be weak C. keep secret D. be with C©u 37 : Choose the best answer to complete the following sentence : When I was turning out my cupboard I........this photograph of Uncle Ba B. came across C. came over D. came into A. came round C©u 38 : Choose the best answer to complete the following sentence : Phong has worked in London for three years and he speaks English......... A. by well very now B. by very well now C. very well by now D. very by well now C©u 39 : Choose the word with the same meaning as the underlined part : We are very anxious about the result of the exam. A. worried B. careful C. excited D. careless C©u 40 : Choose one sentence that has the same meaning as the root one : If he doesn’t promise to try harder, I won’t help him. A. If he promises to try harder, I will never help B. Unless he promises to try harder, I will him. never help him. C. If he promised to try harder, I wouldn’t help D. Unless he doesn’t promise to try harder, I him. will help him. 78 C©u 41 : Choose the word with the same meaning as the underlined part : After the alien spacecraft hovered over the park for a short while, it vanished. A. landed B. attacked C. disappeared D. rose C©u 42 : Choose the word with the same meaning as the underlined part : The company asked for additional information. A. certain B. emphatic C. necessary D. further Read the passage below and choose the best answer for each space( from number 43 to number 55): A year with overseas volunteers I was with Overseas Volunteers (OV) for a year after leaving university, and I was sent to an isolated village in Chad, about 500 kilometers, from the capital. Coming from a rich country, I got quite a shock as conditions were much harder than I had expected. But after a few days I soon got used to..(3)… there. The people were always very friendly and helpful, and I soon began to appreciate how…(4)… the countryside was. One of my job was to supply the village with water. The well was a long walk away, and the women used to…(5) a long time every day..(6)..heavy pots backwards and forwards. So I contacted the organization and arranged to…(7)..some pipes delivered. We built a simple pineline and a pump, and it worked first time. It wasn’t perfect- there were a few..(8), but it made a great difference to the villagers, …(9) had never had running water before. And not …(10)did we have running water, but in the evening it was hot, because the pipe had been…..(11) in the sun all day. All in all, I think my time with OV was a good experience. Although it was not well-paid, it was well…(12)doing, and I would recommend it to anyone who was…(13) working for a charity. Finally, there’s one more reason why I will never….(14) working for OV. A few months before I left, I met and fell in love…(15)another volunteer, and we got married when we returned to England. C©u 43. A. live B. lived C. living D. lives C©u 44. A. beautiful B. good-looking C. handsome D. sweet C©u 45. A. spend B. spent C. spends D. spending C©u 46. A. fetching B. wearing C. carrying D. holding C©u 47. A. make B. let C. have D. allow C©u 48. A. breaks B. leaks C. splits D. punctures C©u 49. A. which B. that C. they D. who C©u 50. A. hardly B. scarcely C. only D. also C©u 51. A. lied B. lay C. laying D. lying C©u52. A. worth B. value C. cost D. price C©u 53. A. considering B. thinking C. going D. planning C©u 54. A. regret B. feel sorry C. miss D. lose C©u 55. A. at B. to C. for D. with C©u 56 : Choose the best answer to complete the following sentence : Mr. Brown.......around the country if he saved enough money. A. would travel B. has traveled C. will travel D. traveled C©u 57 : Choose a word that has different stress pattern A. representiative B. satisfactory C. deforestation D. documentary C©u 58 : Choose the best answer to complete the following sentence : Some authors are too.....to criticism. A. sensible B. sensational C. senseless D. sensitive C©u 59 : Choose one sentence that has different meaning to the root one : The picture is so high that she can’t reach it. A. She is too short to reach the picture. B. She is such a tall girl that she can reach the picture. C. She is not tall enough to reach the picture. D. The picture is too high for her to reach. C©u 60 : Read the following list of four words and choose one word that does not belong in the list : A. Listen B. Spell C. Talk D. Pronounce 79 §Ò thi m«n tiÕnganh (§Ò 3) C©u 1 : Choose the best answer to complete the following sentence : Another word for baggage is............. A. carriage B. luggage C. furniture D. equipment C©u 2 : Choose the best answer to complete the following sentence : The High Street is so narrow that the Council have to.......it. A. increase B. widen C. extend D. lengthen C©u 3 : Choose one sentence that has the same meaning as the root one : I don’t normally have coffee for breakfast. A. I used to have breakfast with coffee. B. I am not used to having coffee for breakfast C. I get used to having breakfast with coffee. D. I didn’t use to have coffee for breakfast C©u 4 : Choose the word with the same meaning as the underlined part : After the alien spacecraft hovered over the park for a short while, it vanished. A. rose B. attacked C. landed D. disappeared C©u 5 : Choose the best answer to complete the following sentence : If Mai comes to England, it will be a good........for her to improve her English. A. experience B. possibility C. opportunity D. advantage C©u 6 : Choose one sentence that has different meaning to the root one : The picture is so high that she can’t reach it. A. The picture is too high for her to reach. B. She is too short to reach the picture. D. She is such a tall girl that she can reach the C. She is not tall enough to reach the picture. picture. C©u 7 : There is a mistake in the four underlined part of the sentence. Find the mistakes(A,B,C or D) : If I (A) find some money (B) on the street, I’d (C) take it to the (D) police station. C©u 8 : Choose the best answer to complete the following sentence : To lack means to................ A. keep secret B. be weak C. be without D. be with C©u 9 : Choose the best answer to complete the following sentence : Some authors are too.....to criticism. A. sensible B. sensational C. senseless D. sensitive C©u 10 Read the following list of four words and choose one word that does not belong in the : list : A. Spell B. Pronounce C. Talk D. Listen C©u 11 Choose the best answer to complete the following sentence : : Don’t touch that wire or you may get an electric......... . B. feeling C. attack D. current A. shock C©u 12 Choose one sentence that has the same meaning as the root one : : Both Jane and her sister speak Vietnamese fluently. A. Jane speaks Vietnamese fluently and so does B. Jane doesn’t speak Vietnamese as her sister. fluently as her sister. C. Jane speaks Vietnamese more fluently than her D. Jane speaks Vietnamese fluently but sister does. her sister doesn’t. C©u 13 Find a word which is pronounced differently in the part underlined: : A. village B. manage C. stage D. baggage C©u 14 There is a mistake in the four underlined part of the sentence. Find the mistakes (A, B, C, or : D) : (A) My father’s motorbike is (B) using (C) too many gas (D) these days. C©u 15 There is a mistake in the four underlined part of the sentence. Find the mistakes ( A, B, C, or : D) : We (A) reached the (B) nearest village after (C) walk for (D) five hours. C©u 16 Choose the word with the same meaning as the underlined part : : The company asked for additional information. 80 A. C©u 17 : A. certain B. emphatic C. necessary Choose the word with the same meaning as the underlined part : I have’t seen your brother recently. lately B. immediately C. suddenly D. further D. quietly Read the passage below and choose the best answer for each space(from number 18 to 30): A year with overseas volunteers I was with Overseas Volunteers (OV) for a year after leaving university, and I was sent to an isolated village in Chad, about 500 kilometers, from the capital. Coming from a rich country, I got quite a shock as conditions were much harder than I had expected. But after a few days I soon got used to..(3)… there. The people were always very friendly and helpful, and I soon began to appreciate how…(4)… the countryside was. One of my job was to supply the village with water. The well was a long walk away, and the women used to…(5) a long time every day..(6)..heavy pots backwards and forwards. So I contacted the organization and arranged to…(7)..some pipes delivered. We built a simple pineline and a pump, and it worked first time. It wasn’t perfect- there were a few..(8), but it made a great difference to the villagers, …(9) had never had running water before. And not …(10)did we have running water, but in the evening it was hot, because the pipe had been…..(11) in the sun all day. All in all, I think my time with OV was a good experience. Although it was not well-paid, it was well…(12)doing, and I would recommend it to anyone who was…(13) working for a charity. Finally, there’s one more reason why I will never….(14) working for OV. A few months before I left, I met and fell in love…(15)another volunteer, and we got married when we returned to England. C©u 18. A. live B. lived C. living D. lives C©u 19. A. beautiful B. good-looking C. handsome D. sweet C©u 20. A. spend B. spent C. spends D. spending C©u 21. A. fetching B. wearing C. carrying D. holding C©u 22. A. make B. let C. have D. allow C©u 23. A. breaks B. leaks C. splits D. punctures C©u 24. A. which B. that C. they D. who C©u 25. A. hardly B. scarcely C. only D. also C©u 26. A. lied B. lay C. laying D. lying C©u 27. A. worth B. value C. cost D. price C©u 28. A. considering B. thinking C. going D. planning C©u 29. A. regret B. feel sorry C. miss D. lose C©u 30. A. at B. to C. for D. with C©u 31 : Choose one sentence that has the same meaning as the root one : If he doesn’t promise to try harder, I won’t help him. B. If he promised to try harder, I wouldn’t help A. If he promises to try harder, I will never help him. him. C. Unless he doesn’t promise to try harder, I will D. Unless he promises to try harder, I will help him. never help him. C©u 32 Choose the word that has the underlined part pronounced differently from the rest : : A. sample B. sugar C. solar D. symbol C©u 33 Choose the best answer to complete the following sentence : : Phong has worked in London for three years and he speaks English......... A. by well very now B. by very well now C. very by well now D. very well by now C©u 34 Choose one sentence that has the same meaning as the root one : : Despite meeting a lot of difficulties, they finally went through the forest. A. Even though meeting a lot difficulties, they B. In spite of the fact that meeting a lot of finally went through the forest. difficulties, they finally went through the forest C. Although they met a lot of difficulties, they D. Though they met a lot of difficulties, but finally went through the forest. they finally went through the forest. 81 C©u 35 : A. C©u 36 : A. C©u 37: A. C©u 38 : A. C©u 39 : A. C©u 40 : Choose the best answer to complete the following sentence : Can you give me a.......for a fruit cake. recipe B. prescription C. description Choose the best answer to complete the following sentence : When I was turning out my cupboard I........this photograph of Uncle Ba came over B. came round C. came across Find a word which is pronounced differently in the part underlined D. receipt D. came into superb B. suppose C. Support D. Supply Choose the best answer to complete the following sentence : The man......we met on tthe train was the headmaster. which B. where C. whom D. whose Choose the best answer to complete the following sentence : Our flat is very small. We wish we.......another bedroom. have had B. had C. will have D. have There is a mistake in the four underlined part of the sentence. Find the mistakes ( A, B, C, or D) : (A) It’s going to be (B) rainy in Tokyo, (C) that is the (D) capital city of Japan. Read the passage below and choose the best answer for each question( from number 41 to 45): Last year we had a nice holiday. My friend and I went to the seaside for a month. I had been to the seaside several times before, but this was the first time for my friend. Naturally, it was a great event for him. Finally, the days came. It was a fine morning. We got up very early because we wanted to leave home after breakfast. We made the journey by car. We reached the seaside at noon. We spent many hours on the beach. We enjoyed making castles and channels in the sand. People said we ought to spend at least a few weeks at the seaside. If we could stay longer, so much the better. C©u 41.How long did the writer and his friend spend at the seaside? A. a week B. a few weeks C. a month D. a few months C©u 42. Who went to the seaside the first time? A. The writer B. The writer’s friend C. The writer’s family D. The writer and his friend C©u 43. What was the weather like on the day they started their journey to the seaside? A. it was bad B. It was rainy C. it was snowy D. It was good C©u 44. How did they travel to the seaside? A. by car B. By train C. by bus D. By air C©u 45. When did they reach the seaside? A. at 9 a.m B. at 12 o’clock C. at 4 p. m D. at 8 p. m C©u 46 : Read the following list of four words and choose one word that does not belong in the list : A. sandals B. boots C. gloves D. shoes C©u 47 Choose the word that has the underlined part pronounced differently from the rest : : B. thunder C. public D. adult A. studious C©u 48 Choose a word that has different stress pattern : A. experiment B. government C. annoyance D. participate C©u 49 Choose the best answer to complete the following sentence : : Mary has gone to the baker’s to buy a.........of bread. A. pound B. loaf C. packet D. bit C©u 50 Choose one sentence that has different meaning to the root one : : The square clock costs $5. The round clock costs $7. A. The round clock is more expensive than the B. The square clock is cheaper than the square clock. round clock. C. The round clock costs less than the square D. The square clock doesn’t cost as much clock. as the round clock. C©u 51 Choose the best answer to complete the following sentence : : Mr. Brown.......around the country if he saved enough money. 82 A. C©u52 : A. C©u 53 : C©u 54 : A. C©u 55 : A. C. C©u 56 : A. C©u 57 : A. C©u 58 : A. C©u 59 : A. C©u 60 : A. will travel B. has traveled C. traveled D. would travel Choose the word with the same meaning as the underlined part : The Vietnamese people workship their ancestors. fossils B. elders C. forefathers D. heirs There is a mistake in the four underlined part of the sentence. Find the mistakes ( A, B, C, or D) : I (A) asked her (B) whether (C) was there a fridge (D) in the kitchen. Choose the word with the same meaning as the underlined part : We are very anxious about the result of the exam. careful B. careless C. excited D. worried Choose one sentence that has different meaning to the root one : You remembered to post the letter for me, didn’t you ? Did you remember to post the letter for me? B. You didn’t forget to post the letter for me, did you? You forgot to post the letter for me, didn’t you? D. Didn’t you forget to post the letter for me? Choose the best answer to complete the following sentence : Lan is the........of the two girls. most pretty B. prettier C. pretty D. prettiest Choose the best answer to complete the following sentence : The teacher says, ‘The time is up’ .This means that the period of time has......... ended B. seemed long C. been interesting D. begun Choose the best answer to complete the following sentence : What are you cooking in that saucepan ? It........good. smells B. feels C. makes D. flavors Choose a word that has different stress pattern representiative B. documentary C. satisfactory Choose the word with the same meaning as the underlined part : I think his story is completely incredible. unbelievable B. insufficient C. unimportant D. deforestation D. incomparable §Ò thi m«n tiÕng anh (§Ò 4) C©u 1: A. C©u 2: A. C©u 3: C©u 4: A. C©u 5: A. C. C©u 6: A. C©u 7: Choose the best answer to complete the following sentence : Lan is the........of the two girls. most pretty B. pretty C. prettiest D. prettier Choose the best answer to complete the following sentence : Mary has gone to the baker’s to buy a.........of bread. bit B. packet C. loaf D. pound There is a mistake in the four underlined part of the sentence. Find the mistakes ( A,B,C, orD) : (A) It’s going to be (B) rainy in Tokyo, (C) that is the (D) capital city of Japan. Choose the word that has the underlined part pronounced differently from the rest : thunder B. public C. studious D. adult Choose the best answer to complete the following sentence : Phong has worked in London for three years and he speaks English......... by well very now B. very well by now by very well now D. very by well now Choose the best answer to complete the following sentence : Our flat is very small. We wish we.......another bedroom. have had B. have C. had D. will have Choose one sentence that has different meaning to the root one : 83 You remembered to post the letter for me, didn’t you ? A. Did you remember to post the letter for me? B. Didn’t you forget to post the letter for me? C. You didn’t forget to post the letter for me, did D. You forgot to post the letter for me, didn’t you? you? Read the passage below and choose the best answer for each space (from number 08 to 20): A year with overseas volunteers I was with Overseas Volunteers (OV) for a year after leaving university, and I was sent to an isolated village in Chad, about 500 kilometers, from the capital. Coming from a rich country, I got quite a shock as conditions were much harder than I had expected. But after a few days I soon got used to..(3)… there. The people were always very friendly and helpful, and I soon began to appreciate how…(4)… the countryside was. One of my job was to supply the village with water. The well was a long walk away, and the women used to…(5) a long time every day..(6)..heavy pots backwards and forwards. So I contacted the organization and arranged to…(7)..some pipes delivered. We built a simple pineline and a pump, and it worked first time. It wasn’t perfect- there were a few..(8), but it made a great difference to the villagers, …(9) had never had running water before. And not …(10)did we have running water, but in the evening it was hot, because the pipe had been…..(11) in the sun all day. All in all, I think my time with OV was a good experience. Although it was not well-paid, it was well…(12)doing, and I would recommend it to anyone who was…(13) working for a charity. Finally, there’s one more reason why I will never….(14) working for OV. A few months before I left, I met and fell in love…(15)another volunteer, and we got married when we returned to England. C©u 8. A. live B. lived C. living D. lives C©u 9. A. beautiful B. good-looking C. handsome D. sweet C©u 10. A. spend B. spent C. spends D. spending C©u 11. A. fetching B. wearing C. carrying D. holding C©u 12. A. make B. let C. have D. allow C©u 13. A. breaks B. leaks C. splits D. punctures C©u 14. A. which B. that C. they D. who C©u15. A. hardly B. scarcely C. only D. also C©u 16. A. lied B. lay C. laying D. lying C©u 17. A. worth B. value C. cost D. price C©u 18. A. considering B. thinking C. going D. planning C©u 19. A. regret B. feel sorry C. miss D. lose C©u 20. A. at B. to C. for D. with Read the passage below and choose the best answer for each question( from 21 to 25) : Last year we had a nice holiday. My friend and I went to the seaside for a month. I had been to the seaside several times before, but this was the first time for my friend. Naturally, it was a great event for him. Finally, the days came. It was a fine morning. We got up very early because we wanted to leave home after breakfast. We made the journey by car. We reached the seaside at noon. We spent many hours on the beach. We enjoyed making castles and channels in the sand. People said we ought to spend at least a few weeks at the seaside. If we could stay longer, so much the better. C©u 21.How long did the writer and his friend spend at the seaside? A. a week B. a few weeks C. a month D. a few months C©u 22. Who went to the seaside the first time? A. The writer B. The writer’s friend C. The writer’s family D. The writer and his friend C©u 23. What was the weather like on the day they started their journey to the seaside? A. it was bad B. It was rainy C. it was snowy D. It was good C©u 24. How did they travel to the seaside? A. by car B. By train C. by bus D. By air C©u 25. When did they reach the seaside? A. at 9 a.m B. at 12 o’clock C. at 4 p. m D. at 8 p. m C©u 26: Choose the best answer to complete the following sentence : The teacher says, ‘The time is up’ .This means that the period of time has......... A. been interesting B. seemed long C. begun D. ended C©u Find a word which is pronounced differently in the part underlined: 27: 84 A. baggage B. stage C. manage D. village There is a mistake in the four underlined part of the sentence. Find the mistakes (A,B, C, orD) : We (A) reached the (B) nearest village after (C) walk for (D) five hours. C©u Choose one sentence that has the same meaning as the root one : 29: If he doesn’t promise to try harder, I won’t help him. A. If he promises to try harder, I will never help B. Unless he promises to try harder, I will him. never help him. D. If he promised to try harder, I wouldn’t C. Unless he doesn’t promise to try harder, I will help him. help him. C©u30 Choose the word with the same meaning as the underlined part : : After the alien spacecraft hovered over the park for a short while, it vanished. A. disappeared B. rose C. landed D. attacked C©u Choose the best answer to complete the following sentence : 31: Don’t touch that wire or you may get an electric......... . A. shock B. current C. attack D. feeling C©u Choose one sentence that has the same meaning as the root one : 32: Both Jane and her sister speak Vietnamese fluently. B. Jane speaks Vietnamese fluently and A. Jane doesn’t speak Vietnamese as fluently as her sister. so does her sister. C. Jane speaks Vietnamese more fluently than her D. Jane speaks Vietnamese fluently but sister does. her sister doesn’t. Choose the best answer to complete the following sentence : C©u To lack means to................ 33: A. be without B. be weak C. keep secret D. be with C©u Find a word which is pronounced differently in the part underlined 34: A. suppose B. Supply C. superb D. support C©u Choose the word with the same meaning as the underlined part : 35: The Vietnamese people workship their ancestors. B. elders C. heirs D. forefathers A. fossils C©u Choose the best answer to complete the following sentence : 36: If Mai comes to England, it will be a good........for her to improve her English. A. opportunity B. experience C. possibility D. advantage C©u Choose one sentence that has the same meaning as the root one : 37: I don’t normally have coffee for breakfast. A. I am not used to having coffee for breakfast B. I used to have breakfast with coffee. C. I get used to having breakfast with coffee. D. I didn’t use to have coffee for breakfast Choose the best answer to complete the following sentence : C©u What are you cooking in that saucepan ? It........good. 38: B. flavors C. feels D. smells A. makes C©u There is a mistake in the four underlined part of the sentence. Find the mistakes ( A,B,C, 39: orD) : (A) My father’s motorbike is (B) using (C) too many gas (D) these days. C©u Choose the word with the same meaning as the underlined part : 40: We are very anxious about the result of the exam. A. careful B. careless C. excited D. worried C©u There is a mistake in the four underlined part of the sentence. Find the mistakes (A,B, C, 41: orD) : If I (A) find some money (B) on the street, I’d (C) take it to the (D) police station. C©u There is a mistake in the four underlined part of the sentence. Find the mistakes (A,B, C, 42: orD) : I (A) asked her (B) whether (C) was there a fridge (D) in the kitchen. C©u Choose the best answer to complete the following sentence : C©u 28: 85 43: A. C©u 44: A. C©u 45: A. C. C©u 46: A. C©u 47: A. C©u 48: A. C. C©u 49: A. C©u 50: A. C©u 51: A. C©u 52: A. C©u 53: A. C©u 54: A. C©u 55: A. C©u 56: A. Mr. Brown.......around the country if he saved enough money. will travel B. traveled C. has traveled D. would travel Choose the word with the same meaning as the underlined part : I think his story is completely incredible. unimportant B. incomparable C. insufficient D. unbelievable Choose one sentence that has different meaning to the root one : The picture is so high that she can’t reach it. She is such a tall girl that she can reach the picture. B. She is not tall enough to reach the picture. She is too short to reach the picture. D. The picture is too high for her to reach. Choose the word that has the underlined part pronounced differently from the rest : sample B. sugar C. solar D. symbol Choose the best answer to complete the following sentence : Another word for baggage is............. luggage B. carriage C. equipment D. furniture Choose one sentence that has different meaning to the root one : The square clock costs $5. The round clock costs $7. The round clock is more expensive than the B. The square clock is cheaper than the square clock. round clock. The round clock costs less than the square D. The square clock doesn’t cost as much clock. as the round clock. Choose the best answer to complete the following sentence : Can you give me a.......for a fruit cake. prescription B. recipe C. description D. receipt Choose the best answer to complete the following sentence : The High Street is so narrow that the Council have to.......it. widen B. extend C. increase D. lengthen Choose a word that has different stress pattern experiment B. annoyance C. government D. participate Read the following list of four words and choose one word that does not belong in the list : boots B. shoes C. gloves D. sandals Choose the best answer to complete the following sentence : The man......we met on tthe train was the headmaster. where B. which C. whom D. whose Read the following list of four words and choose one word that does not belong in the list : Listen B. Spell C. Talk D. Pronounce Choose the best answer to complete the following sentence : Some authors are too.....to criticism. sensitive B. sensational C. senseless D. sensible Choose one sentence that has the same meaning as the root one : Despite meeting a lot of difficulties, they finally went through the forest. In spite of the fact that meeting a lot of B. Although they met a lot of difficulties, difficulties, they finally went through the forest they finally went through the forest. C. Even though meeting a lot difficulties, they D. Though they met a lot of difficulties, finally went through the forest. but they finally went through the forest. C©u Choose a word that has different stress pattern 57: A. documentary B. satisfactory C. representiative D. deforestation C©u Choose the word with the same meaning as the underlined part : 58: The company asked for additional information. A. emphatic B. further C. necessary D. certain C©u Choose the word with the same meaning as the underlined part : 86 59: A. C©u 60: A. I have’t seen your brother recently. immediately B. lately C. suddenly Choose the best answer to complete the following sentence : When I was turning out my cupboard I........this photograph of Uncle Ba came round B. came over C. came across D. quietly D. came into ®¸p ¸n (Dµnh cho gi¸m kh¶o) M«n : tienganh §Ò sè : 4 01 02 03 04 05 06 07 08 09 10 11 12 13 14 15 16 17 18 19 20 21 22 23 24 25 26 27 28 29 30 31 32 33 34 35 36 37 38 39 40 41 42 43 44 45 46 47 48 49 50 51 52 53 54 55 56 57 58 59 60 87 ®¸p ¸n (Dµnh cho gi¸m kh¶o) M«n : tienganh §Ò sè : 3 01 02 03 04 05 06 07 08 09 10 11 12 13 14 15 16 17 18 19 20 21 22 23 24 25 26 27 28 29 30 31 32 33 34 35 36 37 38 39 40 41 42 43 44 45 46 47 48 49 50 51 52 53 54 55 56 57 58 59 60 88 ®¸p ¸n (Dµnh cho gi¸m kh¶o) M«n : tienganh §Ò sè : 2 01 02 03 04 05 06 07 08 09 10 11 12 13 14 15 16 17 18 19 20 21 22 23 24 25 26 27 28 29 30 31 32 33 34 35 36 37 38 39 40 41 42 43 44 45 46 47 48 49 50 51 52 53 54 55 56 57 58 59 60 89 ®¸p ¸n (Dµnh cho gi¸m kh¶o) M«n : tienganh §Ò sè : 1 01 02 03 04 05 06 07 08 09 10 11 12 13 14 15 16 17 18 19 20 21 22 23 24 25 26 27 28 29 30 31 32 33 34 35 36 37 38 39 40 41 42 43 44 45 46 47 48 49 50 51 52 53 54 55 56 57 58 59 60 90 Së gd & ®t b¾c ®Ò thi chän häc sinh giái cÊp tr−êng giang M«n: TiÕng TiÕng Anh 10 Tr−êng THPT Lôc (Thêi gian lµm bµi: 120 phót) Ng¹n 4 Full name:……………….. Mark:……………….. Class ……………….. I. Chän tõ cã phÇn g¹ch ch©n ph¸t ©m kh¸c so víi c¸c tõ cßn l¹i b»ng c¸ch khoanh trßn A, B, C or D 1. A. study 2. A. feat 3. A. foot 4. A. landed 5. A. wife B. love B. seat B. cook B. needed B. live C. cousin C. beat C. school C. started C. file D. far D. great D. look D. opened D. nice II. Chän tõ cã träng ©m chÝnh nhÊn vµo ©m tiÕt cã vÞ trÝ kh¸c so víi c¸c tõ cßn l¹i b»ng c¸ch c¸ch khoanh trßn A, B, C or D. 6. A. never 7. A. science 8. A. sytem 9. A. accident 10. A. attend B. concern B. training B. keyboard B. direction B. public C. sooner C. atom C. speaker C. factory C. damage D. wonder D. believe D. device D. hospital D. practice III. Chän ®¸p ¸n ®óng b»ng c¸ch khoanh trßn A, B, C or D. 11. Can we… .at the house and go to the party together? A. see B. meet C. find D. come 12. She occasionally… ..a book in the evening. A. reading B. read C. reads D. is reading 13. When he was small, he used to… . swimming with his father? A. go B. goes C. going D. is going 14. Local news… ..on TV every afternoon at 5 p.m A. were B. was C. is D. are 15. We started our trip… ..foot last week A. at B. on ` C. from D. by 16. Can you… .. me five pounds? A. borrow B.rent C. lend D. do 17. The children enjoy… . Football after school A. played B. playing C. to play D. play 18. For the past two years I… . a lot of things. A. doing B. do C. have done D. did 19. Do you know the lady… .. saved the boy? A. which B. what C. whose D. who 20. Last night, he didn’t… ..to the cinema. A. go B. goes C. went D. going IV. Chia c¸c ®éng tõ trong ngoÆc sau. 21. It was a nice day, so we decided (go) for a walk. 22. He insisted on (do) the job himself. 23. Lan (sweep) the floor once aday 24. He (send) me a postcard two days ago. 25. More than 30 plays (perform) in New York since May. 26. By the time he arrived, all his classmates (have). 27. Look at the black clouds. It (rain). 28. What time you (go) to bed yesterday? ……………….... ………………... ……………….... ……………….... ……………….... .………………... ……………….... ……………….... 91 29. She (swim) in the river at the moment. 30. Peter (just, come) here. ………………..... ………………... V. Cho d¹ng ®óng cña tõ trong ngoÆc. 31. There are a lot of (different) between the North and the South of VN. ………………………………………………….. 32. She was so (beauty) that everyone likes looking at her. ………………………………………………….. 33. It is (possible) for the children under 18 years old to ride a motorbike. ………………………………………………….. 34. A computer can speed up a lot of (calculate) such as add, subtract, multiply …. …………………………………………………. 35. He is ( interest) in collecting stamps ………………………………………………….. VI. T×m vµ söa lçi sai trong c¸c c©u sau. Mçi c©u cã 1 lçi. 36. Much people agree that we should protect the environment ………………………………………………….. 37. Nam has passed his examinations with high marks, that makes us surprised. ………………………………………………….. 38. I rarely eat ice-cream now, but I use to eat when I was small. ………………………………………………….. 39. The disableds should be cared for by others ………………………………………………….. 40. Why did Peter got angry with Mary lastnight ? ………………………………………………….. VII. §iÒn 1 tõ thÝch hîp vµo chç trèng b»ng c¸ch chän c¸c ®¸p ¸n sau: Computers Night To information From Learn Perfect Second Is Same Computers are helpful in many ways. First, they are fast. They can work with information much more quickly than a person. (41)… .computers can work with lots of information at the (42)… ..time. Third, they can keep (43)… .for a long time. They do not forget things the way (44)… .do. Also, computers are almost always correct. They are not (45)… .of course, but they don’t usually make mistakes. These days, it (46)… important to know something about computers. There are a number of ways to (47)… ..Some companies have classes at work. Also most universities offer day and (48)… ..courses in computer science. Another way to learn is (49)… . A book. There are many books about computers in bookstores and libraries. Or you can learn from a friend. After a few hours of practice you can work with (50)… . You may not be an expert, but you can have fun. 41……………….. 45………………... 48………………... 42……………….. 46………………... 49………………... 43………………... 47………………... 50………………... 44……………….. VIII. §äc kü ®o¹n v¨n sau vµ x¸c ®Þnh c¸c c©u bªn d−íi lµ ®óng (T) hay sai (F) When I was younger, I hated being the oldest child and the only girl in my family. But now that I am older, I realize that being “Big Sister” actually has its advantages. First of all, I get special treatment from my parents and brothers. I get my own room, and brothers have to do all the heavy work around the house. Another benefit is that, being the oldest, I have learned to be responsible and 92 dependable. For example, my parents often leave my brothers in my care when they go out. I also try my best in whatever I do in order to be a good example for my brothers. The experience I’ve had in taking care of my brothers has prepared me for my own family in the future. So, even though I didn’t choose to be the oldest child and the only girl in my family, I have succeeded in making this situation work to my best advantages T or F statements? 51/ … When the writer was small, she disliked being the youngest child in her family 52/… . “ Big Sister” means the oldest sister. 53/… Being “Big Sister” helped the writer be responsible and dependable. 54/… The writer often looks after her brothers when her parents go out. 55/… When the writer grows up, she realises that being “ Big Sister” is enjoyable. IX. Dïng tõ gîi ý viÕt c©u hoµn chØnh. 56. Last year/ Lam/ send/ me / 3 postcards. ………………………………………………….. 57. My/ father/ always/ have/ a cup of tea/ breakfast. ………………………………………………….. 58. My cousin/ interested in/ join/ the English club. ………………………………………………….. 59. Albert Einstein/ born/ March 14th 1879/ the Germany. ………………………………………………….. 60. Before/ I/ go to bed/ I/ turn off/ all the lights/ last night. ………………………………………………….. X. §Æt c©u hái cho cho c¸c tõ g¹ch ch©n sau: 61. Mary often gets up at 6.30 p.m ………………………………………………….. 62. He used to go fishing with his friends when he was younger. ………………………………………………….. 63. She went to the cinema lastnight. ………………………………………………….. 64. Now Mai’s father is over fifty years old. ………………………………………………….. .. 65. Lan is writing a letter to her boyfriend at the moment. ………………………………………………….. . = = = The end = = = Good luck to my students 93 SỞ GIÁO DỤC ĐÀO TẠO HẢI PHÒNG TRƯỜNG THPT TOÀN THẮNG CỘNG HÒA Xà HỘI CHỦ NGHĨA VIỆT NAM Độc lập - Tự do - Hạnh phúc ĐỀ THI CHỌN HỌC SINH GIỎI LỚP 10 NĂM HỌC 2009 – 2010 Thời gian 90 phut không kể thời gian giao đề Họ tên học sinh Lớp Người chấm thi Điểm của bài thi Số phách I. Circle your choice A, B, C or D before the word whose underlined part is pronounced differently from the rest. (10 points) 1 A season B feature C increase D threaten 2 A necessary B approach C language D American 3 A nearby B difficulty C healthy D documentary 4 A university B use C understand D student 5 A smoke B home C photo D fog 6 A accept B accurate C success D accident 7 A further B furniture C purity D surgery 8 A importance B difficulty C desire D determination 9 A question B expectation C investigation D occupation B receive C expect D end 10 A prepare II. Circle your choice A, B, C or D before the best answer to complete each sentence. (20 points) 1. He showed us the house....................he was born. A. which B. where C. in where D. in that 2. Mr. Smith was....................in a road accident. A. dropped B. wounded C. injured D. damaged 3. You can’t leave the country....................a passport. A. no B. without C. if not D. unless 4. New York is famous....................it skyscrapers. A. as B. with C. for D. of 5. The cost of the hotel room doesn’t usually....................the price of breakfast. A. include B. add C. contain D. share 6. If you want to have a pet, you must be ready to....................care of it for several years. A. have B. take C. look D. need 7. If I were you, ....................buy this house. A. I’d B. I’ll C. I’d have D. I 8. The villa is conveniently .................... about a mile from the shopping center. A. stationed B. placed C. located D. put 9. The first National Park was....................two years ago. A. set B. established C. located D. stationed 10. If I hadn’t drunk so much coffee, I....................better. 94 A. would have slept B. slept C. will sleep D. sleep 11. The park has changed so much that I cannot.................... A. realize B. recognise C. identify D. notice 12. We enjoy....................to classical music. A. to listen B. listening C. listen D. listened 13. The Eiffel Tower is still ...............man-made structures in the world. A. a very high one B. highest one C. the higher one D. one of the highest 14. The park...............hundred of species of endangered animals. A. contains B. consists C. composes D. holds 15. Is modern English different ....................old English? A. from B. of C. in D. on 16. Do you know who ................... the washing machine? A. discovered B. invented C. founded D. realized 17. What books ................................Mark Twain's best works? A. considered B. consider C. are considering D. are considered 18. It’s essential to....................the environment. A. care B. protect C. prevent D. restrict 19. Trees, grasses and other plant life......... an important part in the natural circulation of water and thus help conserve it. A. make B. play C. keep D. have 20. He asked me ......................... the book I borrowed from the library . A. if I found B. if I had found . C. whether I have found D. whether I found III. Choose the underlined part (A, B, C or D) that is incorrect. (10 points) 1 He invited me to having dinner with him and his family. A B C D 2. This is the school at which Uncle Ho used to studying when he was young. A B C D 3. Alike her elder sister, Jane prefers working as a model. A B C D 4. Tom apologized to the postman for being unpolite to him. AB C D 5. If the question were not so difficult I will be able to answer it. A B C D 6. He didn't get the job despite of his experience in the field. A B C D 7. He practises speaking English every day in order to he can speak it fluently A B C D 8. If I were you, I didn't buy that old building A B C D 9. His father was quite satisfied about his success. A B C D 10. The teacher asked her weather she knew how to solve the problem. A B C D 95 IV. Read the following passages and circle the best answer A, B, C or D. ( 20 points) Passage A: English has for more than a century and a half (1) ________ called a world language. The number of people who speak it as their mother tongue, has been estimated at between three hundred million (2) ________ four hundred million. It is recognised as an (3) ________ language in countries (4) ________ 1.5....... billion people live. In China, the importance (5) ________ to learning English is such that a televised teaching course drew audiences of (6) ________ to 100 million. But this spread of English throughout the world is relatively recent. In the late sixteenth century English was spoken by just (7) ________ five million people. The arrival of English in North America was the key step in its (8) ________ expansion. The United States is a huge commercial market and this has tended to promote the English language in many (9) ________ nations. About eighty per cent of the data stored on the world ‘s computers is believed (10) ________ in English and nowadays insufficient knowledge of English can be a problem in businesses. 1 A being B been C is D be 2 A to B or C and D with 3 A official B office C original D main 4 A which B who C that D where 5 A attached B attaching C attach D which attach 6 A having B on C up D down 7 A below B beneath C less D under 8 A nation– wide B world – wide C country – wide D world– through 9 A others B another C other D various 10 A is B to be C being D that Passage B: Revising for exams is not as easy as it looks. You will need to work out which routine suits you best, and (1) _______ stick to it. Some people like studying at night when it's quiet, whereas others find the early morning is a (2) _______ time to get things done. You might enjoy (3) _______ to music while you revise, but this can be unhelpful. Can you really concentrate (4) _______two things at once? So think (5) _______you turn your radio on! . Your diet is also important while you are revising. This may be a more than usually (6)_______period of your life, when you should take extra care to eat properly. No missed meals, no junk food, or endless cups of coffee! Get plenty of exercise as well. If you've got fed up with (7) _______ you're doing, or find it hard to concentrate, go for a walk to clear your head. (8) _______exercise will help to keep your body fit and your brain working (9) _______ Finally, you also (10) _______to take time off. Go out occasionally, see your friends, make time to relax. Then you will return to your studies fresh and full of enthusiasm! 1 A then B than C after D often 2 A best B good C worse D well 3 A to listen B having listened C listen D listening 4 A on B in C of D for 5 A unless B if C before D after 6 A stressy B stressed C stressing D stressful 7 A which B what C that D who 8 A Strongly B Every C Regular D Always 96 9 A well B energetic C good D correct 10 A must B should C need D may V. Circle the sentence A, B, C or D nearest in meaning to the original sentence. (10 points) 1. She can't possibly stay up to finish her homework tonight. A. She can't do her homework tonight. B. It is absolutely possible of her to stay up to finish her homework tonight. C. She can't possibly get up to finish her assignment tonight. D. It is impossible for her to stay up to finish her homework tonight. 2. Can you tell me her address? . A. Do you know what is her address? B. You know what is her address? C. Do you know what her address is? D. Can you know her address? 3. People believe that there will be severe earthquakes and tidal waves next year. A. It is believed that severe earthquakes and tidal waves will have happened next year. B. It is our belief that their will be severe earthquakes and tidal waves next year. C. It is believed that there will be severe earthquakes and tidal waves next year. D. Our belief is that there will be severe earthquakes and tidal waves next year. . 4. Don't forget to turn off the light before you go to bed. A. Don't forget to make the light off before you go to bed. B. Remember making the light off before bedtime. C. Don't forget to switch the light before bedtime. D. Remember to switch off the light before you go to bed. 5. ''You stole my bicycle!" his neighbour said. A. His neighbour told him to steal her bicycle. B. His neighbour thought that he stole her bicycle. C. His neighbour accused him of having stolen her bicycle. D. His neighbour accused him to have stolen her bicycle. VI. Complete each of the unfinished sentences in such a way that it has the same meaning as the sentence beforw it. (30 points) 1. He can do it better than his friends. His friends ................................................................................................................................................ 2. She didn't say a word as she left the room. She left the room ...................................................................................................................................... 3. That is the most shocking story I have ever heard. I have ....................................................................................................................................................... 4. He got to the station late and missed the train. By the time .............................................................................................................................................. 5. I'm trying to eat fewer fatty foods. I'm trying to cut ........................................................................................................................................ 6. The school was founded ten years ago. It is ten ...................................................................................................................................................... 7. So far I have written ten pages of my letter. I am ......................................................................................................................................................... 8. " Let 's go for a walk in the park," said Andrew Andrew suggested that ........................................................................................................................ 97 9. His second attempt on the world record was successful. He broke ................................................................................................................................................ 10. I 'm sure he doesn't know that his brother is seriously ill. He couldn't ............................................................................................................................... THE END 98 ĐÁP ÁN ĐỀ THI CHỌN HỌC SINH GIỎI LỚP 10 NĂM HỌC 2009 – 2010 I. Circle your choice A, B, C or D before the word whose underlined part is pronounced differently from the rest. (10 points) 1D 2C 3A 4C 5D 6B 7C 8C 9A 10D II. Circle your choice A, B, C or D before the best answer to complete each sentence. (20 points) 1B 2C 3B 4C 5A 6B 7A 8C 9B 10A 11B 12B 13D 14A 15A 16B 17D 18B 19B 20B III. Choose the underlined part (A, B, C or D) that is incorrect. (10 points) 1B 2C 3A 4D 5C 6B 7B 8B 9C 10B IV. Read the 3 following passages and circle the best answer A, B, C or D. ( 20 points) Passage A: 1. B 3. A 5. A 7. D 9. C 2. C 4. D 6. C 8. B 10. B 1. A 3. D 5. C 7. B 9. A 2. B 4. A 6. D 8. C 10. C Passage B: V. Circle the sentence A, B, C or D nearest in meaning to the original sentence. (10 points) 1D 2C 3C 4D 5C VI. Complete each of the unfinished sentences in such a way that it has the same meaning as the sentence beforw it. (30 points) 1. His friends can't do it as well as he. 2. She left the room without saying a word. 3. I have never heard such a shocking story. 4. By the time I got to the station, the train had left. 5. I'm trying to cut down on fatty foods. 6.. It is ten years since the school was founded. 7. I am on the tenth page of the letter I’m writing 8. Andrew suggested that they/we should go for a walk in the park. 9. He broke the world record at / on his second attempt. 10. He can't have known that his brother is seriously ill. 99 SỞ GD & ĐT THÁI BÌNH TRƯỜNG THPT TÂY THỤY ANH §Ò thi kh¶o s¸t häc sinh giái tr-êng THỜI GIAN : 60’ Mã đề thi 452 Họ, tên học sinh:.......................................................................... Số báo danh:............................................ Part 1: Phonetics Choose the word which has the underlined part pronounced differently from the rest. 1. A. species B. polute C. accept D. receipt 2. A. sky B. fly C. cry D. hydroelectric 3. A. found B. house C. lound D. fought 4. A. wrong B. boss C. doctor D. sport 5. A. lamb B. club C. daubt D. bomber Part 2: Vocabulary Choose the best opotion. Mark your choice on the answer sheet. 6. Whales and dolphins both make ........ which in some ways are similar to a language. A. sighs B. symbols C. sounds D. signals 7. There is so .......... on the road nowadays. A. many cars B. much traffic C. much motorbikes D. many bikes 8. Let's go for a walk, .....................? A. do you B. don't you C. shall we D. do we 9. He failed the exams ........... his lazinees. A. because of B. although C. because D. though 10. Martin Lether King ...... a famous speech " I have a dream" in 1963. A. did B. made C. had made D. had done 11. The government is responsible for the ........ of health care. A. provision B. providedly C. provided D. providing 12. Hanoi National University was ......... one hundred years ago. A. established B. begun C. organized D. appeared 13. We are now facing with the problem of ........... . A. destruction B. deforestation C. ferestation D. deforest 14. Many people were killed in the plane crash. The bodies of ......... were taken away; .......were taken to hospital. A. the die - the injured B. the dead - the injured C. the dead - the injury D. the death the injury 15. ....... fertilizer is used very commonly in growing crops. A. Chemic B. Chemist C. Chemistry D. Chemical Part 3: Grammar Choose the best opotion. Mark your choice on the answer sheet. 16. This book was written by a well-known writer. It is worth ............ A. have read B. reading C. read D. to read 17. The police ....... for two men who ........ getting into a black car near the bank at about 1 o'clock last night. A. are looking - were seen B. had looked - were seen C. are looking - saw D. have looked - saw 18. A: Would you like me to collect you? - B: .............................................................. . A. I'd love to B. Not at all C. No, I wouldn't D. Yes, I like 19. You ......... whisper. Nobody can hear us. A. needn't to B. need to C. musn't D. don't have to 20. .......... you, I would think twice about that decision. It could be bad move. A. If I am B. Were I C. If I had been D. Should I be 100 21. Whalses and sharks are carnivores, ....... means they eat meat. A. this B. it C. that D. which 22. Only in this house .......... safe and secure. A. feed I B. I feed C. I do feel D. do I feed 23. " This phone doesn't work". - " ..................................". A. Neither does this one. B. So does this one. C. This one does, either. D c. This one does, too. 24. She said she was afraid of .......... by the strong wind. A. being taken away B. taking away C. took away D. being take away 25. We had a French architect ......... our villa. A. designing B. designed C. to design D. design 26. Two weeks ......... too long for me to wait. A. were B. have C. are D. is 27. Hurry! The next bus ......... at 7.15. A. leaves B. leave C. will leave D. is leaving 28. Jim wanted to know ....... call and tell him where to meet you. A. if you could B. you could C. when you could D. whether you can 29. My friends are talking about the wring competition ........ I took part last week. A. in which B. by which C. on which D. of which 30. He is beleived ............ by terrorists two months ago. A. had been killed B. to be killed C. to have been killed D. was kiied Part 4: Reading A. Complete the blank with suitable words among a,b,c, or d. Forest has always been important to people. In the past, many people got food by ......(31) forest animals and gathering with wild plants. With the ........... (32) of civilization, fewer people now live ......... (33) forests, but forests are ........... (34) valuable than ever. Forests have an important economic value .............. (35) they provide us with resource ............(36) timber. Forests are vital to the enviroment ............ (37) they clean the air we breathe. Forests are also treasured ............ (38) their beauty. 31. A. hunting B. looking C. taking D. having 32. A. advance B. advantage C. hope D. light 33. A. with B. at C. on D. in 34. A. fewer B. few C. more D. less 35. A. because B. but C. though D. even 36. A. such as B. within C. with D. such 37. A. but B. despite C. inspite D. because 38. A. for B. with C. in D. at B. Read the passage carefully, then choose the correct answers. We are all slowly destroying the earth. The seas and rivers are too dirty to swim in. There is so much smoke in the air that it is unhealthy to live in many of the world’s cities. In one well-known city, for example, poisonous gases from cars pollute the air so much that traffic policemen have to wear oxygen masks. We have cut down so many trees that there are now vast areas of waste-land all over the world. As a result, farmers in parts of Africa cannot grow enough to eat. In certain countries in Asia there is too little rice. Moreover, we do not take enough care of the countryside. Wild animals are quickly disappearing. For instant, tigers are rare in India now because we have killed too many for them to survive. However, it isn’t enough simply to talk about the problem. We must act now before it is too late to do anything about it. Join us now. Save the Earth. This is too important to ignore. 39. In one well-known city, traffic policemen have to wear oxygen masks _____. A. in order to protect themselves from being injured B. because there are so many cars on the streets 101 C. because of air pollution D. in spite of poisonous gases 40. Smoke _____. A. causes an healthy life B. makes life in big cities difficult C. is harmful to health D. is full of the big cities 41. Why do farmers in parts of Africa and Asia not grow enough to eat ? A. Because people cut down many trees B. Because there are large areas of land that cannot be used C. Because many trees have been planted D. Because there is too little rice 42. The seas and rivers nowadays _____. A. are dirty enough to swim in B. are contaminated C. are less dirty than they used to be D. cannot be swum in 43. Wild animals are _____. A. killed so many that they cannot live in the forests B. in danger of extinction C. being protected from natural environment D. so rare that they cannot survive 44. What’s the best title for the passage ? A. The Environment B. Conservation C. Protect the Nature! D. Save the Earth! Part 5: Writing Choose the best opotion. Mark your choice on the answer sheet. 45. We couldn't drive because of the fog. A. Because the fog we couldn't go on driving. B. The fog prevented us out of driving. C. The fog stopped us into driving. D. The fog prevented us from driving. 46. remember/ lock/ door/ go out. A. Remember locking the door before going out. B. Remember to lock the door before going out. C. You remember to lock the doot before you go out. D. Remember to lock the door before you going out. 47. Why don't you ask Jack for his advice? A. He asked Jack for his advice. B. Jack was asked for his advice. C. Jack shouldn't ask Jack for his advice. D. You should ake Jack for his advice. 48. People are not allowed to enter the park after midnight because lack of security. A. because B. to enter C. after D. of 49. I'd rather to stay at home than go out at weekends. A. I'd B. go out C. to stay D. weekends 50 . John's mother thought that chocolates were bad to him. A. him B. to C. chocolates D. John's SỞ GIÁO DỤC VÀ ĐÀO TẠO HẬU GIANG TRƯỜNG THPT CHUYÊN VỊ THANH KỲ THI OLYMPIC TRUYỀN THỐNG 30 - 4 LẦN THỨ XVI ĐỀ THI ĐỀ NGHỊ MÔN:TIẾNG ANH; LỚP :10 102 QUESTION I.: (10 points) A. Identify the word that has the underlined part pronounced differently from that of the other words in the group. 1. A. experience B. expensive C. exhausted D. exclusive 2. A. vision B. measure C .usually D. pleasant 3. A. clean B. death C. head D. heavy 4. A. dinosaur B. launch C. nausea D. laurel 5. A. equation B. education C. contribution D. nomination B. Identify the word whose stress pattern is different from that of the other words in the same group. 1. A. trigonometry B. explanatory 2. A. legislature B. repository 3. A. argumentativeB. psychological 4. A. photograph B. payroll 5. A. majority B. ceremony QUESTION1 A. PHONETIC 1.C 2.A 3.A 4.B 5.A B. STRESS 1. A 2. A 3. C 4. D C. immediately C. magnificent C. contributory C. accent C. astronomy D. democracy D. mistake D. hypersensitive D. regretful D. investiture 5. B 103 QUESTION II. :Reading ( 40 points) A. Read the passage and choose the correct answer for the following questions: On the tiny island of Flores, east of Bali and midway between Asia and Australia, the scientists have discovered the remain of a small, hobbit-like species of humans. These people grew no larger than the modern three-year-old child. They lived about 18,000 years ago and are completely different species of human. [1] This remarkable discovery shows that the human species is more varied and flexible in its ability to adapt than previously thought. These hobbit-like people join a short list of other type of human.[2] [3] The researchers believe that these hobbits evolved from a normal size, human population that reached Flores around 840,000 years ago. [4] One likely explanation is that, over thousands of years, the species became smaller because the environmental conditions favored a smaller body size. The dwarfing of mammals on islands occurs frequently. Islands limit food supply and predators and species compete for the same environmental space. Survival would depend on minimizing energy requirements. 1. Which is the main topic of this passage? A. the discoveries of a hobbit-like species of human and its significance. B. the reason for the dwarfing of mammals C. detailed information about the features of a different species of human D. a fictitious character in world history 2. What makes this discovery so significant? A. It shows that hobbits may have existed. B. It shows that human can change a great deal according to the demands of the environment. C. It shows that humans do not need a lot of different foods. D. This shows that small islands are good place for smaller species 3. The word “remain” in the passage is closest in meaning to A. fossils B. pictures C. descendants D. records 4. Which of the following best favors a smaller body size? A. s small, isolated area where there is a limited food supply B. a mainland where there is a little competition for survival C. an environment where there are many predators D. an island that has a large and varied food supply 5. According to the passage, all of the following can dwarf a species of animal or human except A. limited land B. little food C. few predators D. a deeply forested area with little sunshine 6. Which of the following is true of the newly discovered species? A. They are the only human species to live with modern man. B. They were only as intelligent as a three-year-old child. C. They moved from island to island. D. They needed less food than modern human. 7. We learn from the passage that dwarfing has occurred A. on every continent B. on other islands as well C. only on the island discussed in the passage D. none of the above 8. According to the passage, why does a smaller size help species survive under certain condition? A. It allows them to consume less food. 104 B. It makes them more difficult for predators to see. C. It allows them to move more quickly. D. All of the above 9. In the passage, the term “the same environmental space” means A. the island B. Asia C. The mountains D. Australia 10. Which of the best place for the following sentence? “ This discovery has taught scientists a lot about the human species” A. [1] B. [2] C. [3] D. [4] B. Read the passage and then decide which word (A, B, C, or D) best fits each space. Modern cinema audiences expect to see plenty of thrilling scenes in action film. These scenes, which are (1)_______as stunts, are usually given by stuntmen who are specially trained to do dangerous things safely. (2)________ can crash a car, but if you’re shooting a film, you have to extremely (3)______sometimes stopping right in front of the camera and the film crew. At an early (4)________ in the production, an expert stuntman is (5)______in to work out the action scenes and form a team. He is the only person who can go against the wishes of the director, (6)_______ he will usually only do this in the regards of safety. Many famous actor like to do the dangerous part by themselves, which produces better shots, since stuntmen don’t have to (7)______ in for the actors. Actors like to become (8)______in all the important aspects of the character they are playing, but without the recent process in safety equipment, insurance companies would never (9) _______them take the risk. To do their own stunts, actors need to be good athletes, but they must also be sensible and know their (10)_______. If they were to be hurt, the film would come to a sudden halt. 1. A. remarkedB. known C. referred D. named 2. A. Everyone B. Someone C. Anyone D. No-one 3. A. detailed B. plain C. straight D. precise 4. A. period B. minute C. part D. stage 5. A. led B. taken C. drawn D. called 6. A. despite B. so C. although D. otherwise 7. A. work B. get C. put D. stand 8. A. connected B. arranged C. involved D. affected 9. A. allow B. let C. permit D. admit 10. A. limits B. ends C. frontiers D. borders C. Read the passage and fill in the gaps, using one word in each: Whenever you are football crazy or keen on tennis, working out to music can improve the coordination of your mind and body. The suggestion that rock or pop music that may ever (1) ________a part in sports training would have been (2)________ as a joke not so long ago. But today modern music is increasingly filling the gym as well as the front room. The (3)_______of exercise to music is not new. For years, especially in eastern Europe, the benefits of sportsmen and sportswomen receiving instruction in ballet and classical dance, with their stress (4)________total body control and balance, have been (5)________ recognized. Figure- skating and ice-dance are (6)__________to music and can be said to be specialized forms of this type of exercise. But ballet and classical dance can be applied to other sports than are also (7)_________ to the eye, such as gymnastics and skiing, both of (8) ________ demand high standard of balance, coordination and suppleness. In western Europe and North American, much more interest has been shown in working out to classical music. Even sports which seem to (9)_______ muscular strength more than other physical 105 requirement have taken (10)__________ exercise to music as a valuable addition to (11) ______ own specialized training schemes. Devotees of soccer, rugby and rowing now regularly train to music: even those who take part in weightlifting, which demands enormous physical strength, and (12)_____in athletics field events, find that exercise to music is beneficial and (13)______ their movements more fluid. Sport is benefiting form the keep-fit boom of recent years. Since the early 1980s, the advantages aerobics, (14)________ particular, have been brought home to (15)________ mass audience by television, tapes and books. D. Read the article. Choose the most suitable heading from the list A - I for each part (1 - 7) of the article. There is one extra heading that you do not need. There is an example at the beginning (0). 0I Larry Walters was a lorry driver, but he had always wanted to fly. After leaving school, he wanted to become an Air Force pilot, but unfortunately, he was turned down because of his poor eyesight. So he had to do with watching others fly the fighter jets that criss - crossed the skies over his backyard. As he sat there in his garden chair, he dreamed about the magic of flying. 1 Then one day, Larry came across an advertisement in the local paper and realized there was a way of making his dreams come true. He went to a specialist store and bought forty - five weather balloons and several tanks of helium. These were not brightly - coloured party balloons, but large spheres measuring more than one metre when fully inflated. His plan was to float lazily into the sky, and spend the afternoon running himself 10m above his girlfriend's garden before eventually coming back down to earth. 2 When he returned home, he attached the balloons to his garden chair, tied the chair to his car, and filled the balloons with helium. Then he packed a few sandwiches and drinks and took his air gun so that he could burst a few balloons when it was time to return to earth. 3 When his preparations were complete, Larry sat in his chair and cut the cord. But he made a mistake in his calculations and things did not turn out quite as he had planned. He did not float up as gently as he had expected: within seconds, he passed the 10m altitude that he had hoped to reach, rising quickly to 30m and then 300m. he climbed and went on climbing until he finally leveled at 3,000m. 4 At that height, he did not want to risk shooting any of the balloons because he was afraid it might unbalance his aircraft and send him, crashing to the ground. So he stayed up there among the clouds, sailing around for fourteen hours desperately trying to come up with a solution to the problem of how to get back to earth. 5 Eventually, many hours later, he drifted into the main approach corridor for Los Angeles International Airport. Fortunately, a Pan Am flight passed him and air traffic control was alerted. The pilot explained that he had just seen an armed man floating in a garden chair at 3,000m just outside the plane. Understandably, the air traffic controller found this difficult to believe, but a few minutes later a Delta Airlines pilot called with the same message. Radar confirmed the existence of an unidentified flying object above the airport and the authorities sent for a Navy helicopter to investigate. 6 As night began to fall, offshore breezes began to blow Larry out to sea, and when the helicopter arrives, the wind from the propeller kept pushing his home - made aircraft further away. Eventually, they hovered several hundred metres above him and managed to drop down a line, with which they were able to pull him gradually back to safety. 7 As soon as Larry hit the ground, he was taken away by the police charged with invading Los Angeles 'International Airport airspace. But as he was being led away in handcuffs, a television reported called out,” Why did you do it?" Larry stopped, looked at the man and explained. "I've been dreaming of flying for years. I just got tired of waiting". A. No way down F. A bright idea 106 B. An incredible report C. A difficult rescue D. Free as a bird E. Under arrest ANSWER KEYS : QUESTION 2 A 1.B 2.C 3.D B. 1. A 2.B 3.A C. 1. play 3. idea / concept 5. widely 7.pleasing / satisfying 9.demand 11.their 13.makes / keeps 15.a D. 1. F 2. H 3. G G. Heading for the clouds H. Getting ready for take - off I. A lifelong ambition 4.C 4.A 5.D 5.D 6.C 6.D 7.D 7.B 8.C 8. A 9.B 9.A 6.C 7.E 10.A 10.A 2. considered / regarded 4. on 6. performed 8. which 10. up 12. competitors 14. in 4. A 5.B 107 QUESTION III: Grammar ( 20 points) A. Put each verb given in brackets into an appropriate tense or form.( 1. "You have just missed the last train." "Never mind, I (walk)……………..." 2. "When we (take)………………. our exams, we (have) ……………..a holiday." 3. "You (know)……………… the way to the Global Theatre?" "No." "Then I (show) ………………….you." `4. Thousands of people (see)……………… this exhibition by the end of the month. 5. By the time you (finish)……………….. cooking they (do)……………. their homework. 6. I don't think he (change)……………………. in the thirty years I (know)………………….. him. 7. Then about a year ago he (disappear)……………………. and I (never hear) ………………………from him since. 8. My father (fight)……………………… for four years in the last war. 9. I'm taking my daughter out tonight. She (not have)………………. fun for a long time. 10. You're just in time to hear a nice bit of news. Our neighbor is engaged to get married. He (bring)………………….. me the news himself this morning. B. Complete the following sentences using the verbs given in the box. You have to use the correct tenses of the verbs and the appropriate particles. GIVE PUT DRAW SHOUT BRING NOTE GET ADD COME DRIFT 1. The lady ............................. the bus, walked toward the shop and entered it. 2. The victim didn't have the least idea how he ............................. the plane crash. 3. I'll tell you what, she will soon ............................. the secret ............................. 4. The serious face of teacher ............................. the lazy student ............................. cheating in the exam room. 5. The goalkeeper asked something and the captain ............................. a reply. 6. I think a sip of wine can ............................. him. 7. His comment only ............................. her resentment toward him. 8. At the moment they ............................. a plan for their next picnic. 9. After the argument, they were ............................. from each other. 10. The students were trying to ............................. the explanation given by the famous lecturer. C. Fill in each of the blanks with an appropriate preposition. 1. She nodded her head ............................. assent. 2. These new TV sets are ............................. great demand. 3. I've been away so long I feel ............................. touch with things. 4. The favourite will win, he is ............................. very good form. 5. He bought the property ............................. a view to building a factory on it. 6. It's unbelievable. The under - dog is ............................. the lead. 7. They are ............................. league ............................. our enemies. 8. I disagreed with his ideas ............................. principle. 9. In those days people concealed their beliefs ............................. pain of death. 10. I am not ............................. the least interested. ANSWER KEYS QUESTION 3 A. 1. will walk 2. have taken – shall show 3. Do …… know – will show 4. will have seen 5. finish – shall have done 108 6. has changed – have known 7. disappeared – have never heard 8. fought 9. hasn’t had 10. brought B. 1. got off 6. bring – round 2. came through 7. adds // added to 3. give - away 8. drawing up 4. put - off 9. drifting apart 5. shouted back 10. note down C. 1. in 6. in 2. in 7. in – with 3. out of 8. in 4. in 9. on // under 5. with 10. in 109 QUESTION IV: word form(20 points) A. Choose the right verbs provided in the box, then use the most suitable forms of the verbs to fill in the numbered blanks. ENVIRONMENT DEEP EASY LONG ABLE EXPLORE BEGIN INDUSTRY FAR WORRY FREEZE If the worst fear of Russian and Western(0) environmentalists turn out to be true, as seem likely, the(1)…………………………………..of a single major (2)…………………………………plant to poison a take containing one-fifth of the planet’s fresh water will be (3)………………………new evidence of just how(4)………………………….the Earth’s life support system can be harmed. Lake Baikal,400 miles in(5)……………………………and the deepest in the world, is one of the last few(6)………………………places on Earth.(7)…………………….over in winter. it curves through a part of Siberia where Asia is spiltting apart ,the(8)…………………..of a future ocean. More than 5,000 feet in (9)…………………………..with another four-mile-thick layer of mud(10)……………………….down ,the lake’s cold, oxygen-rich water are full of strange life forms. B. Complete these sentences, using the suitable form of the words in brackets : 1. There will be a special ………….for these patients of bird flu.( separated ) 2. I think you should write something more for your instruction. It is ………. ( complete ) 3. ……….is very important in scientific experiments .( accurate ) 4. ……….is a person who follows or attends a course for doing something.( train ) 5. I don’t want to go at first, but later I have to because of her………..( persuade ) 6. The ………. problem should be solved before the end of this month.( theory ) The forest fire seemed to be………..by the rain yesterday.( weak ) 7. They worked …………after their demand had been refused.( patient ) 8. The ABC company has had a very …………………………year.(success) 9. The new schedules are not ….............with the night shift workers in our factory.( popular) Answer keys QUESTION 4 :WORD FORM A. 1. ability 2. industrial 3. worrying 4. easily 5. length 6. unexplored 7. frozen 8. beginning 9. depth 10. further B. 1. separation 3. accuracy 5. persuasion 7. weakened 9. successful 2. 4. 6. 8. uncompleted trainee theoretical impatiently 10. popularized 110 QUESTION V: Error Correction( 10 points) Most of the lines in this text contain an unnecessary word. A few of the lines are correct. Read the text carefully, find the extra words and mark them. Tick any lines that are correct. Two examples are given. ............the....... 0 It’s quite rare to meet teenagers who don’t like the sports. ................... 00 When you are young, you know how important it is to do ...................... 1 physical exercise if only you want to be healthy and strong, ...................... 2 and for there reason you often concentrate on just one sport ...................... 3 with so much of enthusiasm that in the end you can’t live ...................... 4 without it. The problem is, even though, that is you gow ...................... 5 up you have less and less spare time. At our age when you ...................... 6 have to study harder if you want to get good marks for to ...................... 7 go to university , with perhaps only one afternoon a week ...................... 8 to do any sport. This happens just when you are being at ...................... 9 the best age for many sports, such as gymnastics and ..................... 10 swimming .by the time you will finish all your studies you ..................... 11 will probably be too much old to be really good at sports like ..................... 12 those ,but if you have spend enough time on training while ..................... 13 you are young ,then one day you will find it that you are ..................... 14 very good at your sport but too old to study ,and what you ..................... 15 will find it impossible to get good job. Somehow , it doesn’t seem fair. QUESTION 5: ERROR CORRECTION 1. only 2. V 3. of 4. even 5. when 6. for 7. V 8. being 9. V 10. will 11. much 12. have 13. it 14. what 15. V 111 QUESTION VI: Transformation( 10 points) 1.Is this the only way to reach the city center? Isn’t……………………………………………………………………………………? 2.I have never seen such a mess in my life! Never…………………………………………………………………………………… 3.The weather is probably not going to change. …………………………………………………………………………………………… 4.I’d made up my mind, but at the last minute I lost my confidence. (FEET) …………………………………………………………………………………………… 5.I couldn’t help smiling when he told me of his plan. (A STRAIGHT FACE) …………………………………………………………………………………………… 6.He seemed to be worried about something. (IMPRESSION) …………………………………………………………………………………………… 7.Just thinking about his face at that moment makes me laugh. The very……………………………………………………………………………… 8.Their chances of success are small It’ snot…………………………………………………………………………………… 9. I really want to see her again. (DYING) ……………………………………………………………………………………………… 10.This house is very different from the little flat we used to live in. (CRY) …………………………………………………………………………………………… Answer keys QUESTION 6: 1. 2. 3. 4. 5. 6. 7. 8. 9. 10. Isn’t there any another way to reach the city center? Never in my life have I seen such a mess! There will probably be no change in the weather. I’d made up my mind, but at the last minutes I got/had cold feet. I couldn’t keep a straight face when he told me of his plan. I had/got the impression that he was worried about something. The very thought of his face at that moment makes me laugh. It’s not likely that they will succeed. I am dying to see her again. This house is a far cry from the little flat we used to live in. 112 Së Gi¸o dôc - ®µo t¹o hµ néi TRƯỜNG NG TR tr−êng thpt øng hoµ a m«n thi : tiÕng anh 10 k× thi chän chän häc sinh giái cÊp N¨m häc 20092009-2010 Thêi gian : 120 phót §Ò thi gåm 04 trang ) ( Hä vµ tªn thÝ sinh:…………………………………………….Sè b¸o danh……….. -------------------------------------------------------------------------------------------------------------------------------------------A - PhÇn tr¾c nghiÖm: I. Choose the word that has the underlined part pronounced differently from the rest: 1. A. watched B. stopped C. located D. washed 2. A. food B. school C. foot D. afternoon 3. A. sense B. music C. discuss D. serious 4. A. delights B. lives C. business D. roles 5. A. human B. woman C. brilliant D. humane II. Choose A, B, C or D to complete the following sentences : 6. This river is ……………..with chemical waste from those factories. A. polluted B. polluting C. pollutant D. pollution 7. If Americans ate fewer foods with sugar and salt, their general health ……….better. A. be B. will be C. is D. would be 8. You won’t get well…………………….you take your medicine. A. unless B. if C. provided D. in case 9. I saw Jack yesterday morning while I………………….home from work. A. walked B. had been walking C. was walking D. am walking 10. My brother is interested in doing………………….research. A. scientist B. science C. scientific D. scientifically 11. I’ll let you know as soon as I……………….the results. A. will get B. get C. got D. am getting 12. Mary always ………………………….careful notes of what her teachers say. A. makes B. writes C. takes D. draws 13. It takes a long time……………………………….a foreign language. A. to learning B. to learn C. learning D. for us learn 14. Many species of animals and plants today are………………… A. endangered B. in risk C. risky D. under danger 15. She had changed so much that……………………..anyone recognized her. A. almost B. hardly C. not D. nearly 16. He filled in the necessary forms and …………………….for a job. A. appealed B. asked C. requested D. applied 17. I’m very busy at the moment so it may take a…………time to answer your letters. A. little B. few C. small D. some III. Choose A, B, C or D that needs correcting : 18. Jogging is the more popular form of exercise among the under-40s. A B C D 19. It’s nearly two years since I last have gone to Da Lat. A B C D 20. This time tomorrow, I’ll be relax by my swimming pool. A B C D 21. I like the present whom you gave me on my birthday. A B C D 113 22. He didn’t get the job despite of his experience in the field. A B C D 23. Ones of the most interesting things for me to do is listening to music. A B C D 24. The young has the future in their hands. A B C D 25. The man about that I told you yesterday is a famous scientist. A B C D IV. Choose one word or phrase which best completes each gap of the passage: Computers are helpful (26)…………….many ways. First, they are fast. They can work with information (27)……………more quickly than a person. Second, computers can work with (28)………..information at the same time. Third, they can (29)…………. information for a long time. They do not forget things the common people do. Also, computers are (30)………… always correct. They are not perfect, of course, but they usually do not (31)………..mistakes. These day, (32)…………..is important to know about computers. There are a number of things to learn. Some companies have classes (33)…………work. Also, most universities offer day and night courses in computer science. (34)………way to learn is from a book, or from a friend. (35)…………….a few hours of practise, you can work with computers. You may not be an expert, but you can have fun. 26. A. in B. by C. through D. on 27. A. hardly B. even C. wholly D. entirely 28. A. a lot B. a lot of C. plenty D. much of 29. A. stay B. remain C. hold D. keep 30. A. most B. mostly C. almost D. hardly 31. A. do B. take C. make D. have 32. A. this B. that C. they D. it 33. A. at B. in C. for D. with 34. A. Another B. Other C. Others D. The other 35. A. Within B. After C. For D. During V. Choose the correct sentence among A, B, C or D which has the same meaning as the given one. 36. I’m looking for a job as a secretary. A. I’m looking for someone who works as a secretary. B. I was offer a job as a secretary. C. I’m trying to find a job as a secretary. D. A secretary is looking for a job as my job. 37. The thick fog made it impossible for me to drive to work. A. I couldn’t drive. B. I couldn’t control the car because the fog was thick. C. I couldn’t control the car because of the thick fog. D. The thick fog prevented me from driving to work. 38. I rang the doorbell, but nobody answered. A. Although we rang the doorbell, there was no sound. B. Although we rang the doorbell, nobody answered. C. The doorbell rang, but nobody appeared. D. Nobody was in, so the doorbell continued to ring. 39. Unless you can swim, you aren’t allowed to sail this boat. A. Anyone who wants to sail this boat must be able to swim. B. If you can’t swim, you will have to go in this boat. C. You may sail this boat whether you can swim or not. D. The only person allowed to swim are those in this boat. 40. They will have given up the research by now. A. The research will been have given up by now. B. They aren’t doing the research now. 114 C. The research will have been given up by now. D. The research will be continued by now. B - PhÇn tù luËn: I. READING : Read the text carefully and answer the questions below: There is no doubt that the environment is in trouble. Factories burn fossil fuels which produce acid rain, and this kills trees. At the same time, greenhouse gases rise into the air and contribute to global warming, which threatens to melt the polar ice cap. Meanwhile farmers clear huge areas of rain forest in places such as the Amazon to produce feeding land for cattle or produce wood for building. Rivers and oceans are so heavily contaminated by industrial waste that it is no longer safe to go swimming. Cars pump out poisonous gases which we all have to breathe in. Poaching and overfishing are killing off millions of animals, including whale, elephants and other endangered species. In fact, all around us, all living things large and small which comprise our finely balanced ecosystem are being systematically destroyed by human greed and thoughtlessness. There is a lot we can do, however, to prevent this. The easiest thing, of course, is to recycle waste material such as paper and glass so that we can use it again. If you are truly committed to protecting the environment , of course, you should only buy organic fruit and vegetables, safe in the knowledge that they have been naturally cultivated. Finally, we should buy a small car that uses unleaded petrol which is less harmful to the environment or, even better, make more use of public transport. 1. According to the text, what kills trees ? → ………………………………………………………………………………………… 2. Why are rivers and oceans no longer safe enough to swim in ? ………………………………………………………………………………………….. 3. According to the text, which animals are considered endangered ? → ………………………………………………………………………………………… 4. In general, what is destroying our ecosystem ? →…………………………………………………………………………………………… 5. According to the text, what can we do to protect the environment? →…………………………………………………………………………………………… II. Complete each sentence with a word formed from the word in CAPITALS. 1. You should always pay………… to what the teacher says. ATTEND 2. Tony is…………..to win the designing contest this time. DETERMINATION 3. Some students find it hard to learn English……………. . PRONOUNCE 4. The government is trying hard to deal with the problem of…… . EMPLOY 5. Television is one of the cheapest forms of…………….. . ENTERTAIN 6. Our company has just undergone some major……. changes. TECHNOLOGY 7. Her parents rarely……………….. her to go out with her friends, especially at night. PERMISSION 8. The game has been cancelled because the number of…. …………. is too small. PARTICIPATE 1…………………………. 5…………………………………… 2………………………… 6…………………………………… 3………………………… 7…………………………………… 4………………………… 8…………………………………… III. Finish the second sentence so that it has a similar meaning to the first one: 1.The children couldn’t go swimming because the sea was very rough. → The sea was too……………………………………………………………… 2.We didn’t have enough money, so we didn’t go on holiday. → If we…………………………………………………………………………. 3 .Phone now or you will be late. → You will……………………………………………………………………… 4. I advise you to put your money in a bank. → You’d ………………………………………………………………………. 115 5. We haven’t been to a concert for over a year. → The last time………………………………………………………………… 6 .I haven’t eaten this kind of food before. → This is the first………………………………………………………………. 7. They let their children go to the zoo last Sunday. → Their children………………………………………………………………. IV. Build sentences , using the given words to make meaningful sentences. 1. Tourists / come / all over / world / want / visit / historic buildings / London. →…………………………………………………………………………………………………… …………………………………………………………………………… 2. 75% / earth surface / covered / seas / oceans. →…………………………………………………………………………………… 3. Scientists / send / submarines / devices / investigate / ocean depths. →…………………………………………………………………………………………………… ……………………………………………………………………….. 4. Young whales / now / danger / because of / increase / population / other sea animals. →…………………………………………………………………………………………………… ……………………………………………………………………………….. A - I. 1.C II. 6.A 12.C III. 18.B 24.B IV. 26.A 31.C V. 36.C 2.C 7.D 13.B 19.D 25.A 27.B 32.D 37.D THE END – ANSWERS 3.B 8.A 14.A 20.C 4.A 9.C 15.B 21.B 5.D 10.C 16.D 22.B 28.B 33.A 38.B 29.D 34.A 39.A 30.C 35.B 40.C 11.B 17.A 23.A B - Reading : 2,5 ®iÓm 1. Acid rain kills trees. 2. Because they are heavily contaminated by industrial waste. 3. Whales, elephants and some others. 4. Human greed and thoughtlessness are destroying our ecosystem. 5. To protect the environment , we can recycle waste materials, buy only organic fruit and vegetables , use a small economical car , or use public transport. I. 4 ®iÓm 1. attention 5. entertainment 2. determined 6. technological 3. pronunciation 7. permit / permitted 4. unemployment 8. participants II. 3,5 ®iÓm 1. The sea was too rough for the children to go swimming. 2. If we had had enough money, we could have gone on a holiday. 3. You will be late if you don’t phone now. 4. You’d better put your money in a bank. 5. The last time I went to a concert was over a year ago. 6. This is the first time I have eaten this kind of food. 7. Their children were allowed to go to the zoo last Sunday. III. 2 ®iÓm 116 1. Tourists who come from all over the world want to visit historic buildings in London. 2. 75% of the earth surface is covered by seas and oceans. 3. Scientists can send submarines and devices to investigate the ocean depths. 4. Young whales are now in danger because of the increase in population of other sea animals. SỞ GD-ĐT H À NỘI TRƯỜNG THPT PHÚ XUYÊN A ĐĐỀ THI CHỌN HỌC SINH GIỎI NĂM HỌC 2009-2010 MÔN: TIẾNG ANH - KHỐI 10 (Thời gian làm bài: 120 phút) Full name:……………………………………………………………SBD:…………… --------------------------------------------------------------------------------------------------------I. Put the verb in the correct form. A. Marry had to go to New York last week, but she almost(1)…………..(miss) the plane. She(2)…………….(stand) in the queue at the check-in desk when she suddenly(3)…………… (realise) that she(4)……………..(leave) her passport at home. Fortunately, she doesn’t live very far from the airport, so she(5)………………(have) time to take a taxi home to get it. She(6)………….(get) back to the airport just in time for her flight. B. Last night I(7)……………..(just/go) to bed and (8)……………...(read) a book when suddenly I(9)…………….(hear) a noise. I (10)……………(get) up to see what it was but I (11)………………(not/see) anything, so I (12)…………..(go) back to bed. 1…………………. 2………………… 3………………… 4…………………. 5………………….6………………… 7………………… 8…………………. 9………………….10………………….11……………….12……………….. II. Rewrite each of the following sentences in such a way that it has the same meaning as the printed above sentence. 1. I had only just put the phone down when the Boss rang back. -> Hardly………………………………………………………………………………… 2. While I strongly disapprove of your behaviour, I will help you this time. -> Despite my …………………………………………………………………………… 3. Her hobby is one thing that she doesn’t intend to give up. -> She has ……………………………………………………………………………….. 4. They were seasoned travelers, which we hadn’t expected them to be! -> Contrary ……………………………………………………………………………… 5. She never seems to succeed, even though she works hard. -> However …………………………………………………………………………… 6. My mother was the most warm-hearted person I’ve ever known. -> I’ve …………………………………………………………………………………… 7. The noise next door did not stop until after midnight. -> It was not …………………………………………………………………………….. 8. He didn’t return to his native village until the war ended. -> Not until ……………………………………………………………………………… 9. “ Don’t do that! Are you mad?” She said to me. -> She told………………………………… and……………………………………… 10. Someone is going to repair my TV tomorrow. -> I’m ………………………………………………………………………………….. 11. People say that he beats his wife. 117 -> He………………………………………………………………………………… 12. I would love to live in Paris for a year. -> If only………………………………………………………………………………. 13. “ You stole the jewels!” the Inspector said to him. -> The inspector accused………………………………………………………………… 14. You can try to get Tim to lend you his car but you won’t succeed. -> There’s no point………………………………………………………………………. 15. “ Why hasn’t Peter phoned?” She wondered. -> She wondered………………………………………………………………………. III: Read the text below and think of the word which best fits each space. Use only one word in each space. After each space you are given a clue to the kind of word that is missing. Regrets Most of us (0) are always forgetting important dates, apart (1)....................(preposition) the lucky few (2)………………….(relative pronoun) are blessed with a good memory or the ability to organize (3) .................(reflexive pronoun) so they don't forget important obligations. How many times (4) ................... (auxiliary verb) we all said, "I wish I had remembered!" How (5) ............... (adverb of frequency) have we offended people by failing to remember (6)………….(pronoun) birthdays or name days?(7)………….(conjunction) they say it doesn't matter, we know, deep down, that we (8)…………(auxiliary verb) hurt their feelings. We can always try to make it (9) .............. (preposition) to them next time but unfortunately the damage has (10) .............. (auxiliary verb) done and our relationship with that person (11) ............ (modal verb, future) never quite be the same again. On the other hand, we sometimes do (12) .............. (very or too?) much for someone else because we want to please them and then feel we have damaged (13) .................. (pronoun) own interests in doing so. When friends (14) .................. ( auxiliary verb) involved we may find it difficult to say "no" when they ask us to (15)…………..( do or make?) them a favour, but true friendship should mean that we can say "no" without risk to relationship. 1……………… 2……………….3……………….4………………5……………… 6……………… 7………………. 8……………….9……………10…………….. 11…………….. 12………………13………………14……………..15…………….. IV. Read the text below use the words given in capitals at the end of each line to form a word that fits in the space in the same line. occupy One thing i know is that I wouldn’t like to have an occupation that has anything to do with physics, (1)………….. or maths; chemist I am not the (2) …………… type at all. In fact at school, I was a science Complete (3) ……………. In these subjects. Neither am I very good fall at dealing with people, nor am I (4) ………….. , so jobs in ambition business, administration and (5) …………. don’t really interest Manage me either, Morever I find it (6) ……………. to be surrounded by irritate a lot of people; I would much rather have a job involving creative work or (7) ……………..skills of some sort. I would like to have art the chance of work outdoors (8) ………….. and perhaps do a bit of occasion traveling too, I am not (9) …………….concerned about becoming paticular rich but I would like to have a (10) ……………inome enough reason to live completely. 1) ……………… 2) ………………. 3) ……………..4)……………… 5)………………. 6)……………….. 7)………………8)………………. 9)………………. 10)………………. V. There is one mistake in each of the following sentences. Find and correct it. 1. He seldom goes to the market, doesn’t he? 2. One of the girls worked in that company was badly injured in the fire. 3. They are doing plans for their picnic in the countryside. 4. It was so good weather that we decided to go to the beach. 118 5. Thomas asked that Mary should go to see the doctor. 6. Nobody watched, so the boy took the packet of sweets from the shelf and putting it in his pocket. 7. One of the reasons why he was late for the meeting was because the bad weather. 8. My brother is falling in love on the girl he met at university. 9. He drove enough slowly to stop just in front of the accident on the way home. 10. What did you have for lunch? A few rice and some oranges. 1)…………………… 2)………………….3)…………………..4)…………………. 5)…………………… 6)………………….7)…………………..8)…………………. 9)…………………… 10)………………… VI. Make all the changes and additions necessary to produce from the following set for words and phrases, sentences that together make a complete letter or paragraph. Dear Robert, 1. Thank you/letter/ I receive/ when/ get home/ last night. …………………………………………………………………………………………… 2. I/ be pleased/ hear your/ after such/ long time. …………………………………………………………………………………………… 3. I love/ go ballet/ with you/ but I/ not be free/ until 6.30 p.m. …………………………………………………………………………………………… 4. How about come/ my flat/ and have/ something/ eat/ before/ go? …………………………………………………………………………………………… 5. I/ not want/ take my car/ because it be/ difficult/ find somewhere/ park. …………………………………………………………………………………………… 6. Why/ we not go/ taxi? …………………………………………………………………………………………… 7. Let hope/ dancing be/ good as/ reviews say. …………………………………………………………………………………………… 8. I look forward/ see you/ tomorrow night. …………………………………………………………………………………………… Love, Jackie VII. Fill each of the numbered blanks in the following passage. Use only one word in each space. Computers are helpful in many ways: First, they are fast. They can work with information much more quickly than a person. (1)……………, computers can work with lots of information at the(2)……………time. Third, they can keep(3)………….. for a long time. They do not forget things the way(4)……………do.Also, computers are almost always correct. They (5)………… not perfect, of course, but they don’t usually make mistakes. These days, (6)……………is important to know something about computers. There are a number of ways to(7)…………… Some companies have classes at work. Also, most universities offer day and(8)…………….. Courses in computer science. Another way to learn is (9)……………..a book. There are many books about computers in bookstores and libraries. Or you can learn from a friend. After a few hours of practice you can work with (10)…………… you may not be an expert, but you can have fun. 1……………… 2……………… 3……………… 4……………….. 5……………… 6……………… 7……………… 8……………… 9……………….. 10…………….. ĐÁP ÁN ĐỀ THI CHỌN HỌC SINH GIỎI NĂM HỌC 2009- 2010 MÔN: TIẾNG ANH- KHỐI 10 I. Put the verb in the correct form. A. 1. missed 2. was standing 3. realised 4. had left 5. had 6. got B. 7. had just gone 8. was reading 9. heard 10. got 11. didn’t see 12. went II. Rewrite each of the following sentences in such a way that it has the same meaning as the printed above sentence. 119 1.Hardly had i put the phone down when the boss rang. 2. Despite my strong disapprove of your behaviour, I will help you this time. 3. She has no intention of giving up her hoppy. 4. Contrary to what we had expected(them to be)/ our expectation, they were seasoned travelrs. 5. However hard she works she never seems to succeed. 6. I’ve never known a more warm- hearted person than my mother(was). 7. It was not until after midnight that the noise next door stopped. 8. Not until the war ended did he return to his native village. 9. She told me not to do that and asked (me) if I was mad. 10. I’m going to have my TV repaired tomorrow. 11. He is said to beat his wife. 12. If only I could live in Paris for a year. 13. The inspector accused him of stealing jewels. 14.There’s no point in your trying to get him to lend you his car. 15. She wondered why Peter hadn’t phoned. III. Read the text below and think of the word which best fits each space. Use only one word in each space. After each space you are given a clue to the kind of word that is missing. 1. from 2. who 3. themselves 4. have 5. often 6. their 7. although 8.have 9. up 10. been 11. can 12. too 13. our 14. are 15.do IV. Read the text below use the words given in capitals at the end of each line to form a word that fits in the space in the same line. 1. chemistry 2. scientific 3. failure 4. ambitious 5. management 6. irritating 7. artistic 8. occassionally 9. particularly 10. reasonable V. There is one mistake in each of the following sentences. Find and correct it. 1. doesn’t –does 2. worked- working/who worked 3. doing- making 4. so- such 5. asked- suggested 6. putting- put 7. because- because of 8. on – with 9. enough slowly- slowly enough 10. few- little VI. Make all the changes and additions necessary to produce from the following set for words and phrases, sentences that together make a complete letter or paragraph. 1. Thank you for the letter that I received when I got home last night. 2. I was pleased to hear from you after such a long time. 3. I’d love to go to the ballet with you but I won’t free until 6.30 p.m. 4. How about coming to my flat and have something to eat before we go? 5. I don’t want to take my car because it will be difficulty to find somewhere to park. 6. Why don’t we go by taxi? 7. Let’s hope the dancing will be good as the reviews say. 8. I’m looking forward to seeing you tomorrow night. VII. Fill each of the numbered blanks in the following passage. Use only one word in each space. 1. second 2. same 3. information 4. to 5. are 6. it 7. learn8. night 9. from 10. computers 120 KỲ THI CHỌN HỌC SINH GIỎI LỚP 10 THPT NĂM HỌC 2009 - 2010 -----------------------------------------------------ĐỀ CHÍNH THỨC Môn: TIẾNG ANH Thời gian: 180 phút (không kể thời gian giao đề) ĐIỂM ........................ NHẬN XÉT ………………………………………………………… ………………………………………………………… ………………………… PHÁCH ........................... ----------------------------------------------------------------------------------------------------------------I. PHONETICS: ( 10 points) Part 1: Choose the word that has the underlined part pronounced differently from the others. 1 2. 3. 4. 5 a. camera a. gulf a. ocean a. pollute a. club b. famous b surface b. undersea b. species b. lamb c. glad c. submarine c. beneath c. accept c. doubt d. family d. current d. deep d. receipt d. bomber Part 2: Choose the word whose main stress is placed differently from the others, 1. a. tobacco 2. a. scientific 3. a. entertain 4. a. computer 5. a. television b. contented b. atomic b. communicate b. important b. community c. buffalo c. impossible c. receive c. villagers c. atmosphere d. transplanting d. professor d. complain d. condition d. damage II. VOCABULARY & GRAMMAR (20 points) Part 1: Choose the word or phrase which best completes each sentence. Circle the letter A, B, C or D next to the correct word or phrase. 1. We don't allow ................. in the classrooms. A. people smoke B. smoke C. people to smoke D. to smoking 2. What a dangerous thing to do! You .................. have been killed! A. may B. can C. must D. might 3. By the time I applied, all the hoilday vouchers ................. used up. A. had been B have been C. were D. are 4. I felt as if I ................... a confidence. A. have betrayed B. would betray C. had betrayed D. am betraying 5. His job was to………….out the good apples from the bad ones. A. sort B. find C. choose D. share 6. How could you have .......................... him for your brother? A. confused B. considered C. thought D. mistaken 7. The book …………consists of 4 parts, has been translated in French. A. which B. in which C. to which D. on which 8. If we want to ……….up with them we’d better be hurry. A. come B. arrive C. approach D. catch 9. The grocer's shop ................... since the supermarket opened. A. closed B. has closed C. has been closing D. is closed 10. In ten year's time the population of the world...................... to 500 million. 121 A. grows B. is growing C. has grown D. will have grown Part 2: Read the text below. Use the words given in capitals at the end word that fits in the space in the same line. My ideal job One thing I know is that I wouldn't like to have an occupation that has anything to do with physics, (1) .................................. or maths; I am not the (2) .................................. type at all. In fact at school, I was a complete (3) .................................. in these subjects. Neither am I very good at dealing with people, nor am I (4) .................................., so jobs in business, administration and (5) .................................. don't really interest me either. Moreover I find it (6) .................................. to be surrounded by a lot of people; I would much rather have a job involving creative work or (7) .................................. skills of some sort. I'd like to have the chance to work outdoors (8) .................................. and perhaps do a bit of travelling too. I am not (9) ............................ concerned about becoming rich but I would like to have a (10) .................................. income - enough to live comfortably. 1. 2. 3. 4. 6. 7. 8. 9. of each line to form a OCCUPY CHEMIST SCIENCE FAIL AMBITION MANAGE IRRITATE ART OCCASION PARTICULAR REASON 5. 10. III. READING (35 points) Part 1: Read the text about booking a flight and put the following sentences in the correct order. There is one sentence that you don't need to use. Write the letters A, B, C, ... next to the numbers. Number 1 has been done for you as an example. When you want to book a flight to a certain place, visit your nearest travel agent. Tell the travel agent the date when you want to travel and the destination to which you want to go. The travel agent will then type the information into a small computer. After checking everything on the computer screen, he/she will send the information to a central computer. The central computer contains all information about bookings and destinations, and sends a reply, showing which are the most suitable flights. It also shows whether the flights are fully booked or not. The information which is now shown on the small computer screen is continually changing as other bookings are made in other parts of the world. The travel agent now types in your booking, then the computer will ask for your name and address as well as for certain other information. It will also ask how you will pay for your ticket (by cheque or by credit card). Next the computer confirms the booking and makes a request for payment. When you have paid for your ticket, the travel agent types the information into the computer as well. Finally, some computers print out a ticket before you leave the travel agent’s. A. Details about the flights and seats available are sent back. B. The computer then wants information about the method of payment. C. The tickets are issued either at that time or later. D. The computer asks for personal details. E. Make your request to the travel agent and give him/her all the details. F. Your request is then checked and sent to the main computer. G. Go to a travel agent. H. The computer asks for payment. I. The computer asks for the flight number. J. The travel agent fills your initial request into a small computer. K. The booking is typed into the small computer. L. The booking is confirmed. 122 1. G 7......... 2........ 3......... 8......... 4......... 9......... 5......... 10......... 6......... 11......... Part 2: Read the passage and decide whether the following statement s are true or false. Write the correct answer in each blank. (10 points) Important information for passengers Arrival When the plane lands, you are requested to keep the seat belt fastened until the light goes off and remain in your seat until the plane stops moving. This is for your own comfort and safety. Leaving the airport If you are ending your journey at this airport, you should go up to the escalator to passport control and custom. If you are travelling further by rail of coach, you will find an information desk outside the custom hall, as well as a bank where you can change money. Catching another flight If you are planning to change planes, you should follow the signs to the waiting lounge, where you should check in and wait to board your connecting flight. There will be announcement when your flight is due to take off. Our ground staff will be happy to help you. Leaving the plane All passengers leaving the plane should make sure that they take everything with them. If you have checked in any luggage, you should remember to collect it from the baggage area inside the airport. Continuing on this flight We regret that passengers who are continuing their journey on this flight may not get off the plane. You may not smoke while the plane is on the ground. Thank you for flying with us. We hope to be able to welcome you on board again soon. True or false? 1. All passengers must leave the plane . 2. You must wait until the light goes out before undoing your seatbelt. 3. Passengers who are leaving the airport must first go to the passport control. 4. Passengers continuing by another plane must go through custom. 5. You can change money before you go to passport control. 6. You must go up to the escalator to the waiting lounge if you are changing planes. 7. Passengers in the waiting lounge will hear an announcement when their plane is ready to leave. 8. No one is allowed to smoke in the airport. 9. The plane is going on to another place after some of passengers get off. 10. Passengers who are leaving the plane should check that they have not left anything behind. 1. 2. 3. 4. 5. 6. 7. 8. 9. 10. 123 Part 3: Fill in each numbered space with ONE suitable word from the list below, (there are more words than needed): out, however,me, or, from, on, when, over, a, is, kind, this, up, to, real, unless, of, them, go, but. The world of dreams Most people feel that when they dream, they are carried off to another world. (1)...........................the contrary, dreams are often connected (2) ................... our daily lives. (3) ......................... our whole mind (4)........................ filled with something, when we are either very upset (5)..................... when we are in good spirit, a dream will represent (6)...................... reality in symbols. It is often said that we benefit (7) ...................... dreams because they help the spirit to heal itself. when things (8)...............................wrong. Dreams are therefore (9) ....................... kind of escape, almost a holiday from (10) ...................... life, with its fears and responsibilities . It is, (11) ....................., a strange (12) ............................ of holiday because whether we have a wonderful time or whether it turns (13) .................... to be a nightmare, we quickly forget it. Most dreams disappear forever, (14) ......................... you are one of those people disciplined enough to write (15) ........................ down as soon as you wake up. 1. 2. 3. 4. 5. 6. 7. 8. 9. 10 11. 12. 13. 14. 15. IV. WRITING (35 points) Part 1: Complete the second sentence so that it has a similar meaning to the first sentence. 1. I won't be able to go on holiday if my parents don't give me the money. Unless................................................................................................................................ 2. As soon as I put the phone down, it rang again! No sooner........................................................................................................................... 3. What a pity these shops close at lunch-time. I wish .............................................................................................................................. 4. How old is your father? What……………………….. .........................................................................................? 5. We didn't go to the theatre. We stayed at home. We stayed …………………………………………………………………….………. 124 6. You don’t need to finish all the work today It’s ………………………………………………………………………………… 7. If I had known that peter was in hospital, I would have visited him Had .............................................................................................................................. 8. Let’s invite the Browns to the party on Sunday He suggested ................................................................................................................ 9. The refugees continued to feel unsafe until they had crossed the border Not until ....................................................................................................................... 10. We like pasta but we don't have it everyday. Although ......................................................................................................................? Part 2: Read the text below and look carefully at each line. Some of the lines are correct, and some have a word which¸is not correct. If a line is correct, put a tick ( ). If a line has a word which is not correct, write the word. Many countries around the world have no caves. This is 1……………… not because there are no hills or mountains in those place. 2……………… Some of the world's biggest mountains has no caves because 3……………… they are not made of the right sort of stone. But the low, limestone 4……………… hills of the northern of England and the south of France are full of 5……………… interest caves. 6……………… For hundreds of years, people having enjoyed finding caves 7……………… and going inside it. This sport is called "caving", and the people 8……………... who do it are called “cavers”. Cave exploration began in Europe and 9……………… even today most cavers come from there. That is which we know so 10…………… much about European caves. And in many other parts of the world 11……………… there are hundreds of caves which few people have ever visited. For 12……………… example, there are many caves in Russian and Afganistan, in Albania 13……………… and China, but not many cavers in this countries and it is often 14……………… very difficult for foreign cavers to get permision to visit. 15…………… Part 3: COMPOSITION (You should spend about 30 minutes on this task.) In an essay of about 200 words, state the reasons why you want to live in the city or in the country ………………………………………………………………………………………………………… ………………………………………………………………………………………………………… …………………………………………………………………………………. ………………………………………………………………………………………………………… ………………………………………………………………………………………………………… …………………………………………………………………………………. ………………………………………………………………………………………………………… ………………………………………………………………………………………………………… …………………………………………………………………………………. ………………………………………………………………………………………………………… ………………………………………………………………………………………………………… …………………………………………………………………………………. 125 ………………………………………………………………………………………………………… ………………………………………………………………………………………………………… …………………………………………………………………………………. ………………………………………………………………………………………………………… ………………………………………………………………………………………………………… …………………………………………………………………………………. …………………………………………………………………………………………………. ………………………………………………………………………………………………………… ………………………………………………………………………………………………………… …………………………………………………………………………………. THE END 126 BÀI T P TI NG ANH HƯỚNG DẪN CHẤM I. Phonetics: Part 1: Q# 1 2 3 4 5 Key B B A D A Q# 1 2 3 4 5 Key C A A C B Part 2: II. VOCABULARY & GRAMMAR: Part 1. 20 points (10 points) one point for each correct answer Q# 1 2 3 4 5 6 7 8 9 10 Key C D A C A D A D C D Part 2. (10 points) 1. chemistry 3. failure 5. management 7. artistic 2. scientific 4. ambitious 6. irritating 8. occasionally III. READING: 35 points Part 1: ( 10 points) one point for each correct answer 2. E 3. J 4. F 5. A 7. D 8. B 9. L 10. H Part 2. ( 10 points) : 1 point for each correct word 1. F 6. F Part 3 2. T 3. T 7. T 8. F ( 15 points) : 1 point for each correct word 4. F 9. T 9. particularly 10. reasonable 6. K 11. C 5.F 10. T 1. on 2. to 3. when 4. is 5. or 6. this 7. from 8. go 9. a 10. real 11. however 12. kind 13 out 14. unless 15. them IV. WRITING (30 points) Part 1 . (15 points) 1. Unless my parents give me the money, I won't be able to go on holiday. 2. No sooner did/had I put the phone down than it rang again. 3. I wish these shops opened at lunch-time.. 4. What is your father’s age? 5. We stayed at home instead of going to the theatre. 6. It’s not necessary (for you) to finish all the work today 7. Had I known that Peter was in hospital, I would have visited him 8. He suggested that we should invite the Browns to the party on Sunday. 9. Not until had they crossed the border did they feel safe. 10. Although we like pasta, we don't have it everyday. Part 2. (20 points) 1. 9. 2.place 10.which 3.has 11. 4. 12. 5. northern 13.Russian 6.interest 14. this 7. having 15.permision 8. 127 SỞ GD & ĐT ...... ĐỀ ĐỀ XUẤT Đề thi gồm 0… trang KÌ THI CHỌN HỌC SINH GIỎI CẤP TỈNH LỚP 10 THPT CHUYÊN NĂM HỌC 2013-2014 Môn thi: TIẾNG ANH - CHUYÊN Thời gian làm bài: 180 phút, không kể thời gian giao đề Ngày thi: 16/4/2014 PART A: LISTENING Section 1: Question 1-10 Listen and complete the notes below. Write NO MORE THAN THREE WORDS AND/OR A NUMBER for each answer. You will listen to the tape TWICE. JOB INQUIRY Example: Work at: a restaurant • Type of work: 1. _____________________________ • Number of hours per week: 12 hours • Would need work permit • Work in the: 2. ____________________________ branch • Nearest bus stop: next to 3. _____________________________ • Pay: 4. ₤ _____________________________ an hour • Extra benefits: - a free dinner - extra pay when you work on 5. _____________________________ - transport home when you work 6. _____________________________ • Qualities required: - 7. _____________________________ - ability to 8. _____________________________ • Interview arranged for: Thursday 9. _____________________________ at 6p.m • Bring the names of two referees • Ask for: Samira 10 _____________________________ Section 2: Question 11-20 Question 11-16: Listen and choose the best answer A, B, or C. 11. PS Camping has been organizing holidays for __________. A. 15 years B. 20 years C. 25 years 12. The company has most camping sites in _________. A. France B. Italy C. Switzerland 13. Which organized activity can children do every day of the week? A. football B. drama C. model making 14. Some areas of the sites have “no noise” rule after _________. A. 9.30 p.m B. 10.00 p.m C. 10.30 p.m 15. The holiday insurance that is offered by PS Camping ____________. A. can be charged on an annual basis. B. included in the price of the holiday C. must be taken out at the time of booking 16. Customers who recommend PS Camping to friends will receive __________. A. a free gift B. an upgrade to a luxury tent C. a discount Question 17-20: Write the correct letter A, B, or C next to question 17-2 What does the speaker say about the following items? A. They are provided in all tents B. They are found in central areas of the campsite C. They are available on request. 17. barbecue 18. toys 19. cool boxes 20. mops and buckets ____________________ ____________________ ____________________ ____________________ PART B - LEXICO-GRAMMAR I. Complete each of the following sentences with the correct answer (A, B, C or D). Identify your answer by writing the corresponding letter A, B, C or D on your answer sheet. 1. The audience couldn’t listen to the lengthy and pointless speech _________. A. far any more B. any longer C. any more far D. any lengthier 2. He missed two most important lectures. He _________very ill. A. had to be B. must be C. was to be D. must have been 3. Allan: “Do you mind if I use your dictionary?” Nick: “_______.” A. I’m afraid not B. Without doubt C. No, feel free D. Straight ahead 4. Oh, I’m always forgetting _______ these medicines. Is that before or after meal, Ron? A. when to take B. what I will take with C. on which I should take D. when I take 5. We rang the doorbell again _______ they hadn’t heard it the first time. A. because B. although C. for fear of D. in case 6. How boring the lecture was! I _______ just in the first half of it. A. dropped off B. fell out C. called off D. came out 7. Will it make any _______ to them if we deliver their equipment tomorrow? A. displeasure B. alteration C. difference D. conflict 8. What time are you _______ duty? Let’s have a coffee after that. A. over B. on C. off D. out of 9. Don’t give up your harboring dream. _______ working harder and harder. A. Try out B. Keep on C. Speed up D. Go off 10. It’s been a good year. I’ve written two books and a couple of articles, and _______ are now in print. A. both of which B. both of them C. all of which D. all of them 11. A: Why didn't you go to work yesterday? B: I was feeling a bit ____________ A. under the weather B. full of hot air C. breeze D. under pressure 12. The closets are filled with things we don’t need. We should have a _______soon. A. garage sale B. rush hour C. housekeeper D. doggy bag 13. If you want to return the suit, you must bring the __________with you as proof of purchase. A. note B. recipe C. prescription D. receipt 14. Nancy and I had both bought exactly the same kind of school bag without knowing it. It was a complete ____________. A. luck B. chance C. fortune D. coincidence 15. I think some modern designs are more like ________ of art. A. works B. makes C. objects D. jobs 16. Time will _________ whether I made a right choice or not. A. see B. say C. know D. tell 17. That kind of dress is the _________ of fashion these days. A. height B. top C. head D. lead 18. You are having problems now but I am sure things will change _________ the better soon. A. on B. to C. by D. for 19. Despite being a very good student, she didn’t fulfill her ________ later in life. A. makings B. potential C. capacity D. aptitude 20. If you _______ any problems when you arrive at the airport, give me a ring. A. come about B. catch on C. run into D. face up II. Supply the correct form of the word in capital letter. Write your answers on your answer sheet. With the development of fast motorway systems in many countries, once remote parts of the countryside are (1. INCREASE) __________ becoming (2. ACCESS) ____________to people who live in towns and cities. In many parts of the world, rising (3. PERSON) ____________incomes have resulted in mass (4. OWN) ____________ of motor cars at a time when people have more leisure time to fill. This means that there is a lot of (5. PRESS) ___________ on the countryside to accept a growing number of visitors. In Britain, for example, the (6. NATION)____________ parks are used by more than one hundred million people annually. In (7. ADD) __________, similar numbers visit areas that do not enjoy the same level of (8. PROTECT) ____________. Some people regard this invasion of the countryside as (9. DESIRE) ____________and they have begun campaigning to try and save some aspects of (10. TRADITION)____________ rural culture. III. In most lines of this text there is one unnecessary word. It is either incorrect grammatically, or does not fit the sense of the text. Write the unnecessary word in the space beside the text. Tick (√) each correct line. There are two examples at the beginning. Transfer your answers to your answer sheet. I am really keen on going to the cinema, so I’ve got lots of favourite films. But 0 …√……… the best one as I’ve seen lately is called “Pressure”. In some ways, I suppose 00 …as……… that you could regard it as a detective film but it’s different from most films of 1 ………… that kind because the characters are they such unusual people. The detective in it, for example, is a computer expert who solves crimes on her computer using information given to her by her assistants, who go out and interview to people. The case in the film concerns about the wife of a millionaire, who has gone 2 …….……… 3 ….……… 4 …………… 5 ……..……… 6 …….……. 7 …….….… missing. Sometimes the plot gets a bit complicated but it isn’t too hard to keep up with it. There are a lot of strange characters in it, such as a man who always wears two hats on, and some of the scenes really made me laugh. Also, there is 8……….…… a big surprise at the end but I won’t say what that is in the case you go to see it. 9. ……….…….. It’s very well acted and I also like the music in it. But what do I really like most 10. ……………. about the film is that it’s so original- I’ve certainly seen another film quite like that. PART C: READING I. Read the following passage and choose the best answer for each question. Identify your answer by writing the corresponding letter A, B, C or D on your answer sheet. Have you ever thought about the names of the months? Why are “January” and “February” not called “Primo” or “Secondo”? Is it because the original names were created in ancient times? Or is it because the originators preferred odd words? Take February, for example. Say it aloud a few minutes and you start to wonder. Most people don’t know who developed these names. However, a little research reveals that the names of the months came mostly from a combination of the names of Roman gods and goddesses, important festivals, and the original numbers of the months. Julius Caesar and Pope Gregory XIII change the calendar to make it more exact. Caesar developed a new calendar of 364 and a quarter days, the time it takes the earth to orbit the sun from one spring season to the next. The Pope’s astronomers refined the calendar regarding leap years; they determined that there should be no leap year in years ending in 00- unless they were divisible by 400; the years 1700, 1800, 1900 and 2100 would not be considered leap years, while the years 1600 and 2000 would be. This new Gregorian calendar was so accurate that today, scientists need only add leap seconds every few years to the clock in order to keep the calendar matching the Earth’s cycles. 1. What is the topic of the passage? A. how the modern calendar was named and developed B. how the months were named C. how the leap year system was developed D. how accurate the modern day is calendar 2. It can be inferred from paragraph 1 that the author think the names of the months are __________. A. odd B. difficult to pronounce C. inappropriate D. none of the answers 3. The word “they” in paragraph 3 refers to ______________. A. calendars B. days C. astronomers D. years 4. The word “accurate” in paragraph 3 is closest in meaning to ____________. A. interesting B. informative C. correct D. simple 5. Which of the following will be a leap year? A. 2300 B. 2400 C. 2200 D. 2500 6. Which of the following is true of the Gregorian calendar? A. It needs major improvements. B. It was so well designed, it needs little adjusting today. C. It copied the Roman calendar’s formula of leap years. D. none of the answers 7. Why is Caesar important in calendar making? A. He changed the length of the year B. He extended summer. C. He has a month named for him D. He altered the number of days in the year. 8. In what order is the information in the passage presented? A. Caesar’s calendar, the Gregorian calendar, the modern calendar B. Roman Gods, important festivals, original numbers of months C. names of months, Caesar’s calendar, the Gregorian Calendar D. none of the answers 9. The word “refined” in paragraph 3 is closest in meaning to ____________. A. studied B. invented C. observed D. improved 10. Why is the number of 364 and a quarter important? A. It is the length of time from the beginning of spring to the end of winter. B. It is the length of a planetary year. C. It is the most accurate number for calendars. D. It was a number randomly chosen by Caesar for his calendar. II. Read the text below and think of the word which best fits each space. Use only ONE word in each space. Write your answers on your answer sheet. The legend of the root Ginseng is one of the great mysteries of the east. Often referred to as the “elixir of life”, its widespread use in oriental medicine has led to many myths and legends building up around this remarkable plant. Ginseng has featured (1) ____ an active ingredient in oriental medical literature for over 5,000 years. Its beneficial effects were, at one time, (2) ____ widely recognized and praised that the root was said to be worth its weight in gold. (3) ____ the long history of ginseng, no one fully knows how it works. The active part of the ____ (4) is the root. Its full name is Panax Ginseng – the word Panax, (5) ____ the word panacea, coming from the Greek for “all healing”. There is growing interest by western scientists in the study of ginseng. It is today believed that this remarkable plant may (6) ____ beneficial effects in the treatment of many diseases which are difficult to treat with synthetic drug. Today, ginseng is (7) ____ longer a myth or a legend. Throughout the world it is becoming widely recognized that this ancient herb holds the answer to relieving the stresses and ailments of modern living. It is widely used for the treatment of various ailments (8) ____ as arthritis, diabetes, insomnia, hepatitis and anemia. However, the truth behind (9) ____ ginseng works still remains a mystery. Yet its widespread effectiveness shows that the remarkable properties are (10) ____ than just a legend. III. Read the following passage and choose the best answer for each question. Identify your answer by writing the corresponding letter A, B, C or D on your answer sheet. Environmental Concerns Earth is the only place we know of on the universe that can support human life. (1) ________ human activities are making the planet less fit to live on. As the western world (2) _______ on consuming twothirds of the world’s resources while half of the world’s population do so (3) ________ to stay alive we are rapidly destroying the (4) _______resource we have by which all people can survive and prosper. Everywhere fertile soil is (5) ___________built on or washed into the sea. Renewable resources are exploited so much that they will never be able to recover (6) ______. We discharge pollutants into the atmosphere without any thought of the consequences. As a result the planet’s ability to support people is being (7) _______ at the very time when rising human numbers and consumption are (8) _______ increasingly heavy demands on it. The Earth’s natural resources are there for us to use. We need food, water, air, energy, medicines, warmth, shelter and minerals to (9) _______ us fed, comfortable, healthy and active. If we are sensible in how we use the resources they will (10) _______ indefinitely. But if we use them wastefully and excessively they will soon run out and everyone will suffer. 1. A. Although B. Still C. Yet D. Despite 2. A. continues B. repeats C. carries D. follows 3. A. already B. just C. for D. entirely 4. A. alone B. individual C. lone D. only 5. A. sooner B. neither C. either D. rather 6. A. quite B. greatly C. utterly D. completely 7. A. stopped B. narrowed C. reduced D. cut 8. A. doing B. having C. taking D. making 9. A. hold B. maintain C. stay D. keep 10. A. last B. stand C. go D. remain PART D: WRITING I. Complete the second sentence so that it has the similar meaning to the first sentence. Write your answers on your answer sheet. 1. Its lack of irregular verbs makes Esperanto a unique language. Unlike……….........................................………………………………………………………….. 2. Mr. Smith knew little about the Internet, so he didn’t invest into any computer companies. Had….................................................................................................................................................... 3. The Board of Directors discussed the business in length, but came to no decision. The Board of Directors had ................................................................................................................. 4. When she got to the party, everyone was dancing and singing. On.......................................................................................................................................................... 5. Barbara runs a successful company and she also manages to look after her five children. Not only............................................................................................................................................ 6. It is my opinion that you should spend more time practicing English. If........................................................................................................................................................... 7. Everyone was surprised that the famous actor had very little money when he died. The famous actor................................................................................................................................ 8. “Get out of my house or I will shoot you”, the woman shouted at the strange man. The woman threatened....................................................................................................................... 9. Because of the traffic jam, Andrew couldn't get to the interview on time. The traffic jam..................................................................................................... 10. This old car probably won’t last for more than three years. It’s unlikely ……………………………………………………….……………. II. Write a passage of about 200 words to show your opinion on the following statement: “Some people believe that children’s leisure activities must be educational, otherwise they are a complete waste of time” --------------------------------------------------------------------------------------------------------------------------------------------------------------------------------------------------------------------------------------------------------------------------------------------------------------------------------------------------------------------------------------------------------------------------------------------------------------------------------------------------------------------------------------------------------------------------------------------------------------------------------------------------------------------------------------------------------------------------------------------------------------------------------------------------------------------------------------------------------------------------------------------------------------------------------------------------------------------------------------------------------------------------------------------------------------------------------------------------------------------------------------------------------------------------------------------------------------------------------------------------------------------------------------------------------------------------------------------------------------------------------------------------------------------------------------------------------THE END -------------- SỞ GD&ĐT VĨNH PHÚC ----------------ĐỀ CHÍNH THỨC (Đề thi gồm 5 trang) KỲ THI CHỌN HSG LỚP 10 THPT NĂM HỌC 2013-2014 ĐỀ THI MÔN: TIẾNG ANH (Dành cho học sinh THP không chuyên) Thời gian thi: 180 phút, không kể thời gian giao đề PART I. LISTENING Section 1. Complete the form below. Write ONE WORD AND/ OR A NUMBER for each answer. Rented properties customer’s requirements Name: Steven Godfrey Example: Answer: No. of bedrooms: Four Preferred location: In the _______________ (1) area of town Maximum monthly rent: £ ________________ (2) Length of let required: __________________ (3) Starting: September 1st Address Oakington Avenue Rooms Monthly rent Problem Living/dining room, £ 550 No ___________(4) separate kitchen Mead Street Large living room and £ 580 The ___________(5) is kitchen, bathroom and a too large cloakroom Hamilton Road Living room, kitchen£ 550 Too ___________(7) diner, and a _______________(6) Devon Close Living room, dining £ _____________(8) None room, small kitchen Which two facilities in the district of Devon Close are open to the public at the moment? ____________ (9) hall and ____________ (10) pool. Section 2. Choose the correct answer. LATIN AMERICAN STUDIES 1. Paul decided to get work experience in South America because he wanted_______. A. to teach English there B. to improve his Spanish C. to learn about Latin American life D. to improve his English 2. What project work did Paul originally intend to get involved in? A. construction B. agriculture C. tourism D. architecture 3. Why did Paul change from one project to another? A his first job was not well organized B he found doing the routine work very boring C the work was too physically demanding D his first job was boring 4. In the village community, he learnt how important it was to________. A. respect family life B. develop trust C. use money wisely D. spend time with neighbors 5. What does Paul say about his project manager? A. he let Paul do most of the work B. his plans were too ambitious C. he was very supportive of Paul D. he was too ambitious PART II. PHONETICS I. Pick out the word whose underlined part is pronounced differently from those of the other words. 1. A. latter B. label C. ladder D. latitude 2. A. pour B. roar C. flour D. soar B. purchase C. bookcase D. suitcase 3. A. chase B. feather C. bread D. bead 4. A. thread 5. A. prepare B. preface C. preparation D. prejudice II. Choose the word whose stress pattern is differently from those of the other words. 1. A. amphibian B. champagne C. cathedral D. creature 2. A. accommodation B. antibiotic C. counterclockwise D. deforestation 3. A. consciousness B. ecotourism C. biosphere D. confirm 4. A. architectural B. cosmopolitan C. appreciative D. archeologist 5. A. consolidate B. context C. conference D. confidence PART III. LEXICO-GRAMMAR I. Read the sentences carefully and choose the best answers to make complete ones. 1. From 1949 onward, the artist Georgia O’keeffe made New Mexico _______. A. her permanent residence was B. where her permanent residence C. permanent residence for her D. her permanent residence 2. Just as remote-controlled satellites can be employed to explore outer space, _______ employed to investigate the deep sea. A. can be robots B. robots can be C. can robots D. can robots that are 3. _______ is not clear to researchers. A. Why dinosaurs having become extinct. B. Why dinosaurs became extinct C. Did dinosaurs become extinct D. Dinosaurs became extinct 4. The first transatlantic telephone cable system was not established _______ 1956. A. while B. until C. on D. when 5. Drinking water _______ excessive amounts of fluorides may leave a stained or mottled effect on the enamel of teeth. A. containing B. in which containing C. contains D. that contain 6. In the 1820’s physical education became _______ of the curriculum of Harvard and Yale Universities. A. to be part B. which was part C. was part D part 7. If you can win his attention ___________ for you. A. the so much better B. the better so much C. so much the better D. so the much better 8. The President resigned; the whiff of scandal remained ____________. A. otherwise B. therefore C. immediately D. nevertheless 9. Which __________ agency do you work for? A. ads B. advertised C. advertising D. advertisement 10. Van Gogh suffered from depression ____________ by overwork and ill-health. A. brought on B. coming about C. taken up D. put through II. Each sentence below contains 1 mistake. IDENTIFY the mistakes and WRITE THEIR CORRECT FORMS. 1. Had the committee members considered the alternatives more carefully, they would have realized that the second was superior than the first. 2. Malnutrition is a major cause of death in those countries where the cultivation of rice have been impeded by recurrent drought. 3. Because the residents had worked so diligent to renovate the old building, the manager had a party. 4. John’s wisdom teeth were troubling him, so he went to a dental surgeon to see about having them pull. 5. Time spends very slowly when you are waiting for a bus to arrive. 6. Judy decided to wait until after she had taken her exams before having her wisdom teeth pull. 7. Hardly the plane had landed when Adam realized that he had left the file that he needed at his office. 8. When she was asked for her opinion on the course, she said it had been a waist of time. 9. I try to remember your name but I am afraid I cannot remember it. 10. I’d prefer to do it on myself because other people make me nervous. III. Fill in each blank with a suitable PREPOSITION or PARTICLE. 1. I know ________ experience that I do my best work early in the morning. 2. I am astonished ________ the way my students can spend all night at the disco and still remember their prepositions next morning. 3. Is it possible to insure yourself ________ nuclear attack? 4. At school today, we had a long discussion _______ the best way to learn a foreign language. 5. “How can I discourage my boyfriend _______ trying to kiss me all the time?” “Eat plenty of garlic.” 6. Raise the gun to your shoulder, aim _______ the target, and try not to kill anyone. 7. Small children should be watched ____________ carefully. 8. Would you give up your country cottage __________ a town flat? 9. He’ll inherit the money when he comes ______________ age. 10.The performance on the first night came ______________ pretty. IV. Write the correct FORM of each bracketed word. 1. (ADAPT)__________ to a new environment is a difficult thing for old people. 2. Police asked (PASS) ______________ if they had seen the accident happen. 3. What a (WORRY) ______________pair of children. They have to be watched every minute of the day. 4. Children are taught from young to be (RESPECT) _____________ to their elders. 5. They frequently (MOBILE) _________________ the traffic as they march through the streets. 6. He shook his head in _____________ (APPROVE) 7. He fought the illness with courage and ________________ (DETERMINE) 8. She seems (REASON) __________happy in her new job. 9. Can we (ARRANGE) _____________the meeting for next Monday at 7? 10. If you weren’t so (ACT)_______________, you wouldn’t be so fat! PART IV. READING I. Read the text & decide which word best fits each space by choosing A, B, C or D. Action scenes in films Modern cinema audiences expect to see plenty of thrilling scenes in action films. These scenes, which are (1) __________ as stunts, are usually performed by stuntmen who are specially trained to do dangerous things safely. (2) ________ can crash a car, but if you are shooting a film, you have to be extremely (3) __________ sometimes stopping right in front of the camera and film crew. At an early (4) ___________ in the production, an expert stuntman is (5) __________ in to work out the action scenes and form a team. He is the only person who can go (6) __________ the wishes of the director, although he will usually only do this in the (7) __________ of safety. Many famous actors like to do the dangerous parts themselves, which produces better shots, since stuntmen don’t have to (8) _________ in for the actors. Actors like to become (9) _________ in all the important aspects of the character they are playing, but without the recent progress in safety equipment, insurance companies would never let them take the risk. To do their own stunts, actors need to be good athletes, but they must also be sensible and know their (10) ___________.If they were to be hurt, the film would come to a sudden halt. 1. A. remarked B. known C. referred D. named 2. A. Everyone B. Someone C. Anyone D. No one 3. A. detailed B. plain C. straight D. precise 4. A. period B. minute C. part D. stage 5. A. led B. taken C. drawn D. called 6. A. over 7. A. interests 8. A. work 9. A. connected 10. A. limits B. against B. needs B. get B. arranged B. ends C. through C. purposes C. put C. involved C. frontiers D. across D. regards D. stand D. affected D. borders II. Fill in each blank space with an appropriate word. THE BIRTH OF THE T-SHIRT The T-shirt, or at least the T-shirt as we know it, was born in the theatre. When Tennessee William's play A Streetcar Named Oesire opened in New York in December 1947, a young actor (1) ____________ Marlon Brando went (2) __________ stage wearing a (3) _________ of blue jeans and a bright, white, capped-sleeve T-shirt. It was the first time the T-shirt had been seen publicly as anything (4) ____________ an item of underwear and it set a fashion trend that was to last through (5) _____________ the end of the century. The idea for the T-shirt came (6) __________ Brando himself. He had worn one at rehearsals for the play. The director was so impressed by the look that was created that he asked Brando to wear the shirt in the play itself. Brando may have seen the shirt being advertised by the American company Sears Roebuck. They had decided to market the shirt (7) ___________ a fashionable garment in its (8) ____________ right, rather than just something to be worn (9) __________ warmth beneath a denim workshirt (10) ___________ an army uniform. It was Brando, however, who popularized it, especially with the release of the film version of Streetcar in 1951. A short leather jacket completed the look that was to be adopted by teenage rebels in many countries for decades afterwards. III. Read the following passage and choose the best answer. Since the world became industrialized, the number of animal species that have either become extinct or have neared extinction has increased. Bengal tigers, for instance, which once roamed the jungles in vast numbers, now number only about 2,300. By the year 2025, it is estimated that they will become extinct. What is alarming about the case of the Bengal tiger is that this extinction will have been caused almost entirely by poachers who, according to some sources, are not always interested in material gain but in personal gratification. This is an example of the callousness that is contributing to the problem of extinction. Animals, such as Bengal tiger, as well as other endangered species, are valuable parts of the world’s ecosystem. International laws protecting these animals must be enacted to ensure their survival and the survival of our planet. Countries around the world have begun to deal with the problem in various ways. Some countries, in an effort to circumvent the problem, have allocated large amounts of land to animal reserves. They then charge admission prices to help defray the costs of maintaining the parks, and they often must also depend on world organizations for support. This money enables them to invest in equipment and patrols to protect the animals. Another response to the increase in animal extinction is an international boycott of products made from endangered species. This has had some effect, but by itself it will not prevent animals from being hunted and killed. 1. What is the main topic of the passage? A. the Bengal tiger B. international boycott C. endangered species D. problems with industrialization 2. Which of the following is closest in meaning to the world “alarming” in the first paragraph? A. dangerous B. serious C. gripping D. distressing 3. The word “poachers” as used in the first paragraph could best be replaced by which of the following? A. illegal hunters B. enterprising researchers C. concerned scientists D. trained hunters 4. The word “callousness” in the first paragraph could best be replaced by which of the following? A. indirectness B. independence C. incompetence D. insensitivity 5. The above passage is divided into two paragraphs in order to contrast_______. A. a problem and a solution B. a statement and an illustration C. a comparison and contrast D. specific and general information 6. What does the word “this” in the first paragraph refers to in the passage? A. Bengal tigers B. Interest in material gain C. Killing animals for personal satisfaction D. The decrease in the Bengal tiger population 7. Which of the following could best replace the word “allocated” in the second paragraph? A. set aside B. combined C. organized D. taken off 8. The word “defray” in the second paragraph is closest in meaning to which of the following? A. lower B. raise C. make a payment on D. make an investment toward 9. What does the term “international boycott” in the second paragraph refer to? A. buying and selling of animal products overseas B. a refusal to buy animal products worldwide C. a global increase in animal survival D. defraying the cost of maintaining national parks 10. Which of the following best describes the author’s attitude? A. forgiving B. concerned C. vindictive D. surprised PART V. WRITING I. Finish each of the sentences in such a way that it means exactly the same as the sentence printed before it. 1. She never seems to succeed even though she works hard. Hard _____________________________________________________. 2. Do all the washing, please! Let ______________________________________________________. 3. I regret not going to the airport to say good bye to him. I wish ___________________________________________________. 4. Mick thought that we were married. Mick was under ____________________________________________. 5. The only thing that kept us out of prison was the way he spoke the local dialect. But for his command _______________________________________. II. Write an essay (about 250 words) about the following topic: Some people say that traffic accidents are caused by the increasing number of motorbikes. Others blame for man’s fault. Which point of view do you agree? State at least three relevant evidences. -----------------THE END----------------Thí sinh không được sử dụng tài liệu. Cán bộ coi thi không giải thích gì thêm. Họ và tên thí sinh:…………………….………..…….…….….….; Số báo danh…………………… SỞ GD&ĐT VĨNH PHÚC ----------------ĐÁP ÁN (đáp án gồm 5 trang) KỲ THI CHỌN HSG LỚP 11 THPT NĂM HỌC 2013-2014 HƯỚNG DẪN CHẤM MÔN: TIẾNG ANH (Dành cho học sinh THPT không chuyên) Thời gian thi: 180 phút, không kể thời gian giao đề Total: 130pts PART I. LISTENING Section 1. (10pts) 1. central 2. 600 6. study 7. noisy 3. 2 year(s) 8. 595 4. garage 9. concert 5. garden 10. swimming Section 2 (10pts) 1. C 3. A 4. B 5. C 2. C PART II. PHONETICS I. Pick out the word whose underlined part is pronounced differently from those of the other words. (5pts) 1. B 2. C 3. B 4. D 5. D II. Choose the word whose stress pattern is differently from those of the other words. (5pts) 1. D 2. C 3. D 4. C 5. D PART III. LEXICO-GRAMMAR I. Read the sentences carefully and choose the best answers to make complete ones. (10pts) 1. D 2. B 3. B 4. B 5. A 6. D 7. C 8. D 9. C 10. A II. Each sentence below contains 1 mistake. IDENTIFY the mistakes and WRITE THEIR CORRECT FORMS. (10pts) 1. to the first 2. has 3. diligently 4. them pulled 5. Time passes 6. pulled 7. had the plane 8. waste of time 9. am trying 10. by myself III. Fill in each blank with a suitable PREPOSITION or PARTICLE. (10pts) 1. from 2. at/by 3. against 4. about 6. at 7. over 8. for 9. of 5. from 10. off IV. Write the correct FORM of each bracketed word. (10pts) 1. Adaptation 2. passers-by 3. worrisome 4. respectful 6. disapproval 7. determination 8. reasonably 9. rearrange 5. immobilize 10. inactive PART IV. READING I. Read the text & decide which word best fits each space by choosing A, B, C or D. (10pts) 1. B 2. C 3. D 4. D 5. D 6. B 7. A 8. D 9. C 10. A II. Fill in each blank space with an appropriate word. (10pts) 1. named/called 2. on 3. pair 4. but/ except 5. to/until 6. from 7. as 8. own 9. for 10. or III. Read the following passage and choose the best answer. (10pts) 1. C 2. D 3. A 4. D 5. A 6. C 7. A 8. C 9. B 10. B PART V. WRITING I. Finish each of the sentences in such a way that it means exactly the same as the sentence printed before it. (10pts) 1. Hard as/ though she works, she never seems to succeed. 2. Let all the washing be done. 3. I wish I had gone to the airport to say good bye to him. 4. Mick was under the impression that we were married 5. But for his command of the local dialect, we could have been put into prison / jail. II. Write an essay (of about 250 words) about the following topic (20pts) Some people say that traffic accidents are caused by the increasing number of motorbikes. Others blame for man’s fault. Which point of view do you agree? State at least three relevant evidences. Marking scheme The impression mark given is based on the following scheme: 1. Content: 50% of total mark: a provision of all main ideas and details as appropriate 2. Language: 30% of total mark: a variety of vocabulary and structures appropriate to the level of English language gifted upper-secondary school students 3. Presentation: 20% of total mark: coherence, cohesion, and style appropriate to the level of English language gifted upper-secondary school students. ------The end ------ SỞ GD&ĐT --- KỲ THI CHỌN HSG LỚP 10 THPT NĂM HỌC 2012-2013 ĐỀ THI MÔN: TIẾNG ANH ĐỀ CHÍNH THỨC (Dành cho học sinh THPT không chuyên) Thời gian làm bài: 180 phút, không kể thời gian phát đề (Đề thi có 4 trang) PART I: LISTENING I. You will hear a man talking to a group of people about a new sports centre. For each of the questions, fill in the missing information in the numbered space. Use NO MORE THAN THREE WORDS for each answer. NEW SPORTS CENTRE Facilities: * swimming pool * (1) _______ * squash courts * tennis courts (opening on (2) _______) Prices: * Monthly: £35 * (3) £ _______ a year) (special discounts for children) * not included in price: equipment and swimming (4) _______ How to become a member: Fill in a membership form - remember to write your age and weight. For more information phone Mrs. (5) _______ on 467 9900. II. You will hear an interview with a woman who works in a museum. As you listen, choose the best answer to each question. 1. When she was at school, Clara wanted to become ______. A. an actress B. a teacher C. a lawyer 2. Why did Clara not apply for the museum job sooner? A. Her mother advised her to wait. B. It seemed a very difficult job. C. She thought it might be boring. 3. What surprised Clara on her first day at work? A. She had to carry some items. B. She was asked to give a speech. C. She had to meet a lot of people. 4. Clara was told to improve her knowledge of ______. A. computers B. languages C. mathematics 5. What part of her job does Clara enjoy the most? A. planning large exhibitions B. writing descriptions of objects C. talking to visitors about the displays PART 2: PHONETICS III. Choose the word whose stress pattern is different from that of the other words in each group. 1. A. historical B. significant C. satisfaction D. collaborate 2. A. individual B. similarity C. professionally D. opportunity 3. A. contemporary B. continental C. revolution D. alcoholic 4. A. generous B. permissive C. permanent D. sensible 5. A. endure B. value C. divert D. exhaust PART 3: GRAMMAR AND VOCABULARY IV. Choose the word or phrase that best fits the space in each sentence. 1. If you have anything important to do, do it straight away. Don't put it ______. A. on B. off C. over D. up 2. The plane from Dallas ______ two hours late, so I missed my connecting flight from Frankfurt to London. A. took off B. took on C. left on D. left out 3. ______, Carol is very punctual, but she was late tonight. A. For a rule B. Like a rule C. As a rule D. For a rule of thumb 4. 'What did Professor Spencer say when you asked him if the final exam could be postponed?' “He said that it was out of ______ because there's no time to reschedule the test”. A. the question B. order C. all proportion D. reach 5. Solar heat penetrates more deeply into water than _____. A. it is penetrating into soil B. it does into soil C. does it into soil D. that it does into soil 6. ______ on barren slopes can help prevent erosion. A. Trees to be planted B. Planting trees C. Trees being planted D. That trees are planted 7. He has been unable to find a job ______ with his ability as an accountant. A. appropriate B. suitable C. requisite D. commensurate 8. As they watched the football match the huge crowd ______ in unison. A. chanted B. intoned C. crowed D. bellowed 9. In this assignment, we will ______ your work and then give you detailed feedback on how to improve your writing. A. assess B. judge C. measure D. test 10. Although it is impossible to give a ______ age, we believe that the woman was between 25 and 30 when she died. A. clear B. certain C. absolute D. definite V. Find out and correct the 10 mistakes in the following passage. Although party invitations no longer frightened to me, I found myself making carefully preparations for this particularly party. I got a haircut, laid out my best suit, selected a special shirt, a colourful tie, and cufflinks. I had recently went on a diet, but because I didn't want I looked too thin, I discontinued it. What should I bring to my hosts? Flowers? wine? What kind of the wine? Port? Sherry? Or possibly even champagne? Meeting new people was still a major undertaking. It was schedule for this Saturday and I decided to take a cab to the suburb where it was being hold. It was early autumn and the weather was mild, but that morning it turned cold and rainy, and as I listened to the steam hissing in my radiator, it already fell like midwinter. From my newspaper, I learned that a hurricane which had already struck another state was in its way to New York, though it might veer out to sea. E.g. frightened to me → frightened me VI. Use the correct forms of the given words 1. One day he would have to accept the _______ of one of his heroes in public life. 2. After having drunk a lot of wine, he came back home _______. 3. _______, he failed the final exam. 4. There is too much _______ in this world. 5. Mr. Smith is a _______person. If he says he will do something, you know that he will do. 6. The sun and the moon are often _______ in poetry. 7. Don’t you think it would be _______ to see a doctor? 8. I’ve known _______ all my life. There have been times when I couldn’t even afford a loaf of bread. 9. She’s got very _______ parents. 10. She is a highly _______ woman. PART 4. Reading resign steady luck greedy depend person sense hard wealth education VII. Read the text and choose the best answer. Bats are not the dirty, bloodthirsty monsters that they are portrayed to be in vampire films. These animals groom themselves carefully like cats and only rarely carry rabies. Of the hundreds of species of bats, only three rely on blood meals. In fact, the majority eat fruit, insects, spiders, or small 4 animals. They consume an enormous number of pests, pollinate many varieties of plant life, and help reforest barren land by excreting millions of undigested seeds. Almost all bats use echolocation to navigate, especially at night. As they fly, they emit a series of high-pitched squeaks at the rate of about fifty per minute. As these signals bounce off objects in their path, an echo is detected by the bats' sensitive ears which informs them of the direction and distance 10 of obstacles so that they can undertake corrective or evasive action. But bats are not blind as widely assumed. In fact, all species of bats can see, probably about as well as human beings. It is also a little-known fact that bats are highly social creatures. Thousands or even millions of individual bats may belong to a colony, hanging upside down in caves or in trees. Within their social 13 systems, bats assume specialized roles. Some may guard the entrance to their caves, others may scout for food, and still others may warn the colony of approaching danger. A nursery colony may be part of a larger colony to provide mother bats with a safe, supportive environment in which to rear their young. 1. What is the author's opinion of bats? A. They are dirty and they carry rabies. B. They are like the monsters in vampire films. C. They are clean, helpful members of the animal world. D. They are not very important in the animal world. 2. According to the passage, how are bats like cats? A. They both carry rabies. B. Cats groom themselves, and so do bats. C. Both cats and bats eat pests. D. Bats use echolocation, and cats do, too. 3. What do most bats eat? A. Blood meals B. Fruit and insects C. Leaves and trees D. Large animals 4. The word “enormous” in line 4 could best be replaced by ______. A. very heavy B. very regular C. very large D. very necessary 5. Which of the following are NOT characteristics of most bats? A. They pollinate plants. B. They have specialized roles in their colony. C. They use echolocation. D. They eat blood. 6. How do bats help reforest the land? A. By eating pests. B. By hanging upside down in trees at night C. By excreting seeds D. By taking evasive action 7. The word “emit” in line 6 is closest in meaning to _______. A. send B. continue C. find D. stop 8. According to the passage, how do bats navigate? A. By responding to the echoes of their signals bouncing off objects B. By warning the colony of approaching danger with high squeaks C. By beating their wings fifty times per minute D. By using their sensitive ears to hear the noises in their environment 9. Where in the passage does the author refer to the visual range of bats? A. Lines 4-5 B. Lines 6-7 C. Lines 7-9 D. Lines 10-11 10. The word “Some” in line 13 refers to ______. A. social systems B. specialized roles C. bats D. colonies VIII. Fill each blank in the following passage with ONE suitable word. Tom Sponson, aged fifty three, was a thoroughly successful (1) _____. He had worked up a first-class business, (2) _____ a charming wife, and built himself a good house (3) _____ the London suburb that was neither so modern as to be pretentious nor so conventional to be dull. He (4) ______ good taste. His son was doing well (5) _____ Oxford. His daughter, who was attending a good school, had no wish to (6) _____ makeup, to wear low frocks, or to flirt. She still saw herself as too young (7) _____ these trifling amusements. Yet she was gay, affectionate, and thoroughly happy with life. All (8) _____same, for same time, Tom had (9) _____ aware that he was working very hard for very little. His wife gave him a little kiss in the morning when he (10) _____ for the office and, if she were not at a party another little kiss in the evening when he came home. IX. Read the passage and choose the best option for each space. Music is universal _ it is produced by all cultures. Some scientists believe that music came before speech and (1) ______ as a development of making calls. In fact, there is (2) ______ theory that the earliest languages were chanted or sung rather than spoken. Indeed, in some cultures, music is a form of (3) ______ history. The Aboriginal Australians, for example, use music as a means to (4) ______ on histories of the land and spirits to the next generation. New evidence suggested that music does not just satisfy the feel-good factor but it is also good for the brain. A study of intellectually (5) ______ children showed that they could recall more (6) ______ after it was given to them in a song than after it was read to them as story. Researchers also report that people score better on a standard intelligence (7) ______ after listening to Mozart. The so-called “Mozart effect” has also been (8) ______ by findings that rats (9) _______ up on Mozart run faster through a complex network of paths or passages, known as a maze. Overall, it seems that in most instances people who suffer from any form of mental (10) ______ benefit from listening to music. 1. A. was B. swelled C. reacted D. arose 2. A. one B. every C. such D. that 3. A. enjoying B. making C. recording D. stating 4. A. move B. pass C. hand D. happen 5. A. disabled B. inactive C. incapable D. disordered 6. A. facts B. knowledge C. memory D. information 7. A. form B. scheme C. examination D. test 8. A. supported B. given C. marked D. remembered 9. A. held B. brought C. stood D. set 10. A. badness B. hurt C. illness D. pain PART 5. WRITING X. Complete the second sentence so that it has a similar meaning to the first sentence, using the word given. Do not change the word given. You must use between two and five words, including the word given. 1. I would love to learn different languages. keen → I ____________________________ different languages. 2. I'm sorry I was late for the meeting. apology → Please ____________________________ late for the meeting. 3. “You borrowed my watch, didn't you?” Tom's sister said. accused → Tom's sister ____________________________ her watch. 4. Most people hope they will have enough money to buy a house or a flat. afford → Most people hope they ____________________________ to buy a house or a flat. 5. I wish I had not lied to my parents. truth → If only ____________________________ to my parents. XI. Finish the second sentence so that it means the same as the first one. 1. My precious life was saved by the lifeguard. → I owe ________________________________________________. 2. To the best of my knowledge, the building was set on fire deliberately. → As __________________________________________________. 3. I wanted to make sure that all my good work wasn't wasted in that way. → I wanted to prevent _____________________________________. 4. I'm always inviting him for a weekend with us but he's always too busy. → No matter _____________________________________________. 5. The only reason the party was a success was that a famous film star attended. → Had it _________________________________________________. XII. Paragraph writing: Write a paragraph of around 200 words on how students can make progress in learning English. _____The end_____ KỲ THI CHỌN HSG LỚP 10 THPT NĂM HỌC 2012-2013 SỞ GD&ĐT VĨNH PHÚC ĐÁP ÁN MÔN: TIẾNG ANH (Đáp án có 2 trang) (Dành cho học sinh THPT không chuyên) PART 1: LISTENING – 10 points I. Gap fill (5 pts – 1pt/ item) 1. gym 2. Saturday (Sat) 3. 215 4. lessons 5. Reysall 3. A 4. C 5. B 3. A 4. B 5. B II. Multiple choice (5 pts – 1pt/item) 1. B 2. C PART 2. PHONETICS – 5 points III. (5 pts – 1pt/item) 1. C 2. C PART 3: GRAMMAR AND VOCABULARY – 30 points IV. Multiple choice (10pts – 1p/item) 1. B 2. A 3. C 4. A 5. B 6. B 7. D 8. A 9. A 10. D V. Mistake correcting (10pts – 1pt/item -Tìm thấy lổi nhưng sửa sai hoặc không sửa cho 0.5 điểm) 1. carefully -> careful 2. particularly -> particular 3. went -> gone 4. I looked -> to look 5. the wine -> wine (bỏ “the”) VI. Word form (10pts – 1p/item) 1. resignation 2. unsteadily 6. personified 7. sensible 6. schedule -> scheduled 7. hold -> held 8. (the) weather was -> (the) weather had been 9. fell -> felt 10. in -> on 3. Unluckily 8. hardship 4. greed 9. wealthy 5. dependable 10. educated PART 4. READING – 30 points VII. Read the text and choose the best answer. (10 pts – 1p/item) 1. C 2. B 3. B 4. C 5. D 6. C VIII. Gap filling. (10 pts – 1p/item) 1. man/ person 2. married/ had 3. in 6. wear 7. for 8. the 7. A 8. A 4. had 9. been 9. D 10. C 5. at 10. left IX. Read the passage and choose the best option for each space . (10 pts – 1p/item) 1. D 2. A 3. C 4. B 5. A 6. D 7. D 8. A PART 5. WRITING – 25 points X. Sentence transformation (5 pts – 1p/item) 1. …….. am keen on learning 2. …….. accept my apology for being 3. ……… accused him of borrowing/ having borrowed 4. ……… will be able to afford/ can afford 5. I had told the truth XI. Sentence transformation. (5 pts – 1p/item) 1. I owe my precious life to the lifeguard. 2. As far as I know (I am concerned), the building was set on fire deliberately. 9. B 10. C 3. I wanted to prevent all my good work from being wasted. 4. No matter how often I invite him for a weekend, he's always busy. 5. Had it not been for a famous film star's attendance/ the attendance of a famous film star, the party wouldn't have been a success/ have been successful/ have succeeded. XII. Writing a paragraph. (15 pts) • Content: accounts for 50% of the total mark. To be given the maximum of 50% for the content, the candidates should provide all main ideas and details as appropriate. • Language: accounts for 30% of the total mark. To be given the maximum of 30% for language, the candidates should use variety of vocabulary and structures appropriate to the level of English language high school students. • Presentation: accounts for 20% of the total mark. To be given the maximum of 20% for presentation, the candidates should write with coherence, cohesion and can use appropriate styles and linking devices appropriate to the level of English language gifted high school students. SỞ GD&ĐT ....... ———————— ĐỀ ĐỀ XUẤT KỲ THI CHỌN HSG LỚP 10 - THPT NĂM HỌC 2013- 2014 ĐỀ THI MÔN: TIẾNG ANH ———————————— Question I: Phonetics (5points ) Part A: Choose a word whose underlined part is pronounced differently from the rest of the group ( 3points ) B. near C. dear D. fear 1. A. bear 2. A. with B. both C. thing D. think 3. A. money B. mother C. donkey D. other Part B : Choose a word whose stress pattern is different from the rest of the group (2points). 1. A. transform B. office C. failure D. holiday 2. A. effort B. effect C. deafness D. speaker Question II : Verb forms (12 points ) Give the correct form of the verb given in brackets. 1. Look! They (put) sugar in their grapefruit before they eat it. 2. The race (start) as planned if the weather clears. 3. Tom was tired because he (study) hard all day. 4. George (fall) off the ladder while he was painting the ceiling. 5. Before you (leave), don’t forget to shut the windows. 6. If I (know) we would have (wait) so long, I (bring) along a book. 7. It is raining now. It (rain) for 2 hours. 8. I knew I (not be) the first (arrive), because I saw smoke (rise) from the chimney. Question III: Word forms (10 points). Fill in each blank with the correct form of the word given in capital letters. 1. Tom is a friend of mine. He is …………. FRIEND 2. How ………..of you to break that cup! CARE 3. His long ………….made us bored and sleepy. SPEAK 4. He never takes his father’s……….. ADVISE 5. Although they are poor, they are living……. HAPPY 6. He draws cartoons for a ………magazine. HUMOUR 7. Tom spoke ……because he was so excited. BREATHE 8. This car is really……….. LUXURY 9. The ……staff consists of ten experienced journalists. EDIT 10. I am bored. I need some…….in my life. EXCITE Question IV: Choose the best option to complete each of the following sentences (15 points). 1. Whenever we meet, we stop …………….. A. talking B. to talking C. to talk D. talk 2. Are your plans for this weekend still ………………..? A. the same B. same C. like D. alike 3. …………. you run quickly, you are likely to be late. A. If B. Unless C. Since C. However 4. You should spend more time …….. speaking English. A. to practise B. practising C. to practising D. practise 5. Before he joined the army, Danny always went out at night. He never …… stay in. A. would B. used to C. use to D. was used to 6. We always spend our summer holidays …………. A. in the sea B. in the beach C. at the country C. at the seaside 7. He says he enjoys job as a baker, but it has taken him over a year to ….. to working at night. A. be use B. get used C. used D. getting used 8. My brother refused ………… me $10. A. lending B. lend C. to lend D. to lending BY ĐỖ BÌNH – THPT LIẾN SƠN, LẬP THẠCH, VĨNH PHÚC www.violet.vn/quocbinh72 9. You should go and get ……….. rest before tonight’s performance. A. lot of B. some C. many D. few 10. The new office block ……….. well with its surroundings. A. blends in B. stands out C. shapes up D. sets off 11. Whole villages have been ………… by the floods. B. mopped up C. called off D. wiped out A. run down 12. ……….of an accident in the High Street, traffic is moving very slowly on the London Road. A. Through B. Owing C. Because D. Since 13. It was ……a simple question that everyone answered it correctly. A. so B. such C. much D. too 14. The other day I came …………a really beautiful old house in the back street of Birmingham. A. across B. over C. by D. down 15. He said he’d come to the meeting ……….he might be a little late. A. although B. even C. whereas D. unless Question V: Reading (15 points ) Part A: Fill in each of the numbered blanks in the following passage with one suitable word (10 points) Have you ever followed the instructions for knitting something or have you read a (1)_____ of music? If you have, you have behaved in a similar way(2)____ a computer. A computer obeys a program to (3)_____ a particular task. Just(4)_____ a knitting pattern is expressed in numbers and a piece of music in lines and dots,(5)_____ a computer program is expressed in the form of programming language.(6)_____ the English language (or any other language), there are rules of grammar, and a program must be correct in every way. There are hundreds of programming languages, which only a few are well known and widely used. The most popular language(7) _____ is used in offices and factories is called Cobol. This language is used for printing payrolls and keeping records of goods. Engineers and scientists use Fortran in order to carry out calculation. (8)_____ , neither of these two programs is very popular with people working in home computers. Basic is(9)_____ popular language for home computers,(10)______ because it is easy to learn and use. 1. A. bit B. slice C. piece D. sheet 2. A. to B. as C. so D. from 3. A. carry on B. carry out C. carry over D. carry through 4. A. alike B. by C. so D. as 5. A. so B. as C. thus D. such 6. A. So B. Like C. Same D. By 7. A. that B. which C. what D. it 8. A. In addition B. Therefore C. Consequently D. However 9. A. the most B. mostly C. most D. most of 10. A. probably B. chiefly C. hardly D. clearly Part B: Read the passage and choose the best answer. (5 points) Of all my relatives, I like Aunt Emily the best. She's my mother's youngest sister. She has never married, and lives alone in a small village near Bath. She is in her late fifties, but she is quite young in spirit. She has a fair complexion, and dark brown eyes. She has fair hair, which she wears in a bun. She has a kind face, and when you meet her, the first thing you notice is her smile. Her face is a little wrinkled now, but I think she is still rather attractive. She is the sort of person you can go to if you have a problem. She likes reading and gardening, and she still goes for long walks over the hills. She's a very active person. Either she's making something, or mending something, or doing something to entertain herself. She's extremely generous, but not very tolerant with people who don't agree with her. I hope I am as contented as she is when I am her age. 1. According to the writer, Aunt Emily is...... A. a kind and generous woman. B. a widow. C. quite young. D. older than his mother. 2. How old is Aunt Emily? A. About 40 B. About 50 C. About 55 D. About 58 3. In the sentence "She has fair hair, which she wears in a bun." We know that she has.... A. short hair B. long hair C. fair hair D. curly hair 4. Which of the following sentences is true? BY ĐỖ BÌNH – THPT LIẾN SƠN, LẬP THẠCH, VĨNH PHÚC www.violet.vn/quocbinh72 A. Aunt Emily is one of the writer's best relatives. B. Despite her age, her face is a little wrinkled. C. Despite her age, she is quite young in spirit. D. She is neither generous nor tolerant. 5. In the second paragraph the writer means that Aunt Emily..... A. enjoys doing nothing. B. always keeps herself busy. C. is not very satisfied with her present life. D. is a selfish woman. Question VI: Writing (20 points) Part A: There is one mistake in each of the following sentences. Find out and correct it (5 points) 1. He has forgotten to tell us when will she be back. 2. It’s pleasant of reading books in spare time. 3. He is so slow as he never gets to class on time. 4. Ann prefers going swimming than playing table tennis. 5. Not only her parents but her brothers and sisters also live in London. Part B: Combine each pair of the following sentences, using the word in brackets. 1. Ann studied every subject very hard. She won her school's Best Student Prize this year. (so) 2. Mai was very good at Vietnamese in grade 8. Her teacher of Vietnamese was very happy. (so) 3. They can help us clean up the beach. They are certain. (that) 4. We go by plane. It will be quicker. (if) 5. The water is highly polluted. We can't swim in this river. (because) 6. My father went to work. It rained very hard. (although) 7. Mai tried her best. She failed the final exam last year. (but) 8. Ba sold his old newspapers. Ba bought textbooks for the new school year. (and) Part C: Finish each of the following sentences in such a way that it means exactly the same as the sentence printed before it. (10 points). 1. He borrowed my book, but he forgot to return it. I …………………………………………………… 2. They made him wait for two hours. He was…………………………………………….. 3. I must leave now, or I will miss the bus. I will miss…………………………………………. 4. I can’t see that far. It is too……………………………………………… 5. She’s a slower driver than I am. She drives…………………………………………… 6. The police started looking for him two months ago. For two months now the …………………………. 7. How old do you think this house is? When do you think………………………………. 8. This holiday’s so expensive. I don’t think I can go. It’s such…………………………………………… 9. It’s the cheapest watch but it’s the nicest. Not only…………………………………………….. 10. I will get home at about seven, and I’ll give you a ring then. I’ll give you a ring………………………………….. -------------------------THE END-----------------------------Hä vµ tªn thÝ sinh:................................................... sè b¸o danh ................................... BY ĐỖ BÌNH – THPT LIẾN SƠN, LẬP THẠCH, VĨNH PHÚC www.violet.vn/quocbinh72 Së gi¸o dôc vµ ®µo t¹o §Ò THAM KH¶O N¨m häc 2008-2009 M«n thi: TiÕng Anh (dµnh cho häc sinh thi vµo líp chuyªn Anh ) §Ò thi chÝnh thøc A: ®¸p ¸n: Question I: Phonetics (5points ) Part A: ( 3points ) 1. A 2. A 3. C Part B : (2points). 1. A 2. B Question II : Verb forms (12 points ) 1. are putting 2. will start/ will be started 3. had studied/ studied 4. fell 5. leave 6. had known; to wait; would have brought 7. has been raining 8. wasn’t; to arrive; rising. Question III: Word forms (10 points). 1. friendly 2. careless 3.speech 4. advice 5. happily 6. humorous 7. breathlessly 8. luxurious 9. editorial 10. excitement Question IV: (15 points). 1. C 2. A 3. B. 4. B 5. B 6. D 7. B. 8.C Question V: Reading (15 points ) Part A:(10 points) 1. C 2. A 3.B 4.D 5.A Part B: ( 5 points) 1. A 3. B 4. C 2.D 9.B 6.B 10.A 11.D 12.C 7.A 8.D 13.B 9.A 14.A 15.A 10.B 5. B. Question VI: Writing (23 points) Part A: (5points) 1. will she she will 2. of reading to read 3. as that 4. than to 5. but her brothers and sisters also but also her brothers and sisters Part B: ( 8 points) 1. Ann studied every subject very hard, so she won her school's Best Student Prize this year. 2. Mai was very good at Vietnamese in grade 8,so her teacher of Vietnamese was very happy. 3. They are certain that they can help us clean up the beach. 4. If we go by plane, it will be quicker. 5. We can not swim in this river because the water is highly polluted. 6. My father went to work although it rained very hard/ Although it rained very hard, my father went to work. 7. Mai tried her best, but she failed the final exam last year. 8. Ba sold his old newspapers and bought textbooks for the new school year. Part C: (10points) 1. I lent him my book, but he forgot to return it. 2. He was made to wait for two hours. 3. I will miss the bus if I don’t leave now/ unless I leave now. 4. It’s too far (for me) to see. 5. She drives more slowly than me/ I do. BY ĐỖ BÌNH – THPT LIẾN SƠN, LẬP THẠCH, VĨNH PHÚC www.violet.vn/quocbinh72 6. For two months now the police have been looking for him. 7. When do you think this house was built? /they built this house? 8. It’s such an expensive holiday that I don’t think I can go. 9. Not only is it the cheapest watch, but it’s also the nicest. 10. I’ll give you a ring when I get home at about seven. B. huíng dÉn chÊm §iÓm cña bµi thi = Tæng sè c©u, tõ hoÆc côm tõ ®óng 8 Thang ®iÓm 10 -------------------------THE END------------------------------- BY ĐỖ BÌNH – THPT LIẾN SƠN, LẬP THẠCH, VĨNH PHÚC www.violet.vn/quocbinh72 SỞ GD&ĐT ....... ———————— ĐỀ ĐỀ XUẤT KỲ THI CHỌN HSG LỚP 10 - THPT NĂM HỌC 2014- 2015 ĐỀ THI MÔN: TIẾNG ANH ———————————— I. VOCABULARY & GRAMMAR Part 1: Choose the word or phrase (A, B, C or D) which best completes each sentence. (10 points) 1. The school was closed for a month because of serious ______ of fever. A. outcome B. outburst C. outset D. outbreak 2. The campaign raised far more than the ______ of $20.000. A. aim B. object C. goal D. target 3. We have a ______ future ahead with little comfort, food or hope. A. cruel B. pessimistic C. grim D. fierce 4. Only thoroughly unpleasant people leave the ______ of their picnics to spoil the appearance of the countryside. A. rest B. remainder C. remains D. rester 5. An almost ______ line of traffic was moving at a snail’s pace through the town. A. continuous B. constant C. continual D. stopping 6. Medieval travelers’ tales of fantastic creatures were often fascinating but not always ______ A. credible B. creditable C. credulous D. imaginable 7. Don’t be late for the interview, ____ people will think you are a disorganized person. A. unless B. otherwise C. if not D. or so 8. My friends have just moved to a new flat in a residential area on the ______ of Paris. A. suburbs B. outside C. outskirts D. side 9. You are being thoroughly ______ in refusing to allow this ceremony to take place. A. unrequited B. unrepresentative C. unreliable D. unreasonable 10. “But so”, I told him, “you are my own ______.” A. heart and heart B. body and soul C. flesh and blood D. skin and bone Part 2: Use the word in capitals at the end of these sentences to form a word that fits in the blank space. (20 points) 1. How much does ______ of this club cost? (MEMBER) 2. She is extremely ______ about the history of art. (KNOW) 3. Traveling in big cities is becoming more and more ______ every day. (TROUBLE) 4. He is completely ______ ! Not only is he lazy but he is dishonest too. (EMPLOY) 5. His boss told him off because he had behaved ______ (RESPONSIBLE) 6. He won the discus event at the Olympic Games but was later ______ when a medical check proved that he had been taking drugs. (QUALIFY) 7. Women who are slimming can never enjoy a meal without being afraid of ______ their diet. (ORGANISE) 8. The trouble with Mr. Brown is that he’s so ______ One minute he goes mad when you come late; the next he says nothing. You never know where you are! (CONSIST) 9. It is forbidden to hunt for that kind of bird. It has been listed as one of the ______ species (DANGER) 10. I didn’t know who it was – with a mask on she was completely ______ (RECONGNISE) Part 3: Complete each sentence with one of the idiomatic phrases given. Use each phrase once only. (8 points) made her blood boil jumped out of her skin had a lump in her throat kept her head lost her head gave me the cold shoulder pulled her legs took her breath away 1. Pauline completely ignored me this morning. She ______ 2. It really ______ to see her friend copy a poem she had written and then present it to her boyfriend as one that she had written for him. 3. Saying goodbye to her son was a very emotional occasion for her and she ___ as she watched him get on the train. 4. The view from the top of the mountain was absolutely fantastic. It really ______ 5. Susan panicked when the fire broke out. She ______ 6. When I dropped that tray behind her, she got a shock. She almost ______ 7. My sister remained very calm. She ______ 8. Sally didn’t believe us, did she? We only joked! We only ______! Part 4: Put each verb given in brackets into an appropriate tense or form. (17 points) 1. Mr. Pike, who is supposed ______ (witness) the accident, ______ (leave) the town. 2. At this time next week they ______ (sit) in the train on their way to Paris. 3. He said that he watered the plants every day. He _____ (water) them. If he _____ (have), they wouldn’t have died. 4. No sooner the announcement ______ (make) than everyone started complaining. BY ĐỖ BÌNH – THPT LIẾN SƠN, LẬP THẠCH, VĨNH PHÚC www.violet.vn/quocbinh72 5. 6. 7. 8. 9. 10. They said they never ______ (make) to do anything they didn’t want to before. We ______ (watch) the play for nearly thirty minutes when he came. The pop star who ___ (take) part in over 25 films always avoids _____ (recognise) by _____ (wear) dark glasses. There’s no point ______ (try) to get Tim ______ (lend) you his car. Not until John ______ (receive) the offer of promotion in writing he ______ (celebrate) He ______ (have) trouble with the second-hand car since he bought it. He wishes he ______ (not / buy) it. II. READING Part 1: Read the text below and choose the correct word for each space. For each question, circle letter A, B, C or D next to the answer you choose. (10 points) If you’re an environmentalist, plastic is a word you tend to say with a sneer or a snarl. It has become a symbol of our wasteful, throw-way society. But there seems little (1)_____ it is here to stay, and the truth is, of course, that plastic has brought enormous (2)_____ even environmental ones. It’s not really the plastics themselves that are the environmental (3)_____ – it’s the way society choose to use and (4)_____ them. Almost all the 50 or so different kinds of modern plastic are made from oil, gas or coal – non-renewable natural (5)_____ We (6)_____ well over three million tones of the stuff in Britain each year and, sooner or later, most of it is thrown away. A high (7)_____ of our annual consumption is in the (8)_____ of packaging, and this (9)_____ about seven per cent by weight of our domestic (10)_____ Almost all of it could be recycled, but very little of it is, though the plastic recycling (11)_____ is growing fast. The plastics themselves are extremely energy-rich – they have a higher calorific (12)_____ than coal and one (13)_____ of ‘recovery’ strongly (14)_____ by the plastic manufacturers is the (15)_____ of waste plastic into a fuel. 1. A. evidence B. concern C. doubt D. likelihood 2. A. pleasures B. benefits C. savings D. profits 3. A. poison B. disaster C. disadvantage D. evil 4. A. dispose B. store C. endanger D. abuse 5. A. resources B. processes C. products D. fuels 6. A. remove B. import C. consign D. consume 7. A. portion B. amount C. proportion D. rate 8. A. way B. kind C form D. type 9. A. takes B. makes C. carries D. constitutes 10. A. refuse B. goods C. requirements D. rubble 11. A. manufacture B. plant C. factory D. industry 12. A. degree B. value C. demand D. effect 13. A. measure B. mechanism C. method D. medium 14. A. desired B. argued C. favored D. presented 15. A. conversion B. melting C. change D. replacement Part 2: Read the following text and then fill in each gap with ONE suitable word. (10 points) I was reading an article last week in (1)_____ the writer described (2) _____ her children has changed as they (3)_____ up. When they were small she had to (4)_____ up with noisy games in the house, or join in interminable games of football in the garden which (5)_____ her out. If the house went quiet, she wondered what the monsters were getting up to, or what crisis she would have to (6)_____ with next. She dreaded the fact that they might (7)_____ after her husband, who admitted having (8)_____ an uncontrollable child who (9)_____ most of the time showing off to his friends by breaking things or getting into fights. What was worse was that (10)_____ else thought he was a sweet child, and he got away with the most terrible things! However, she had experienced an even greater shocked with her children. They had (11)_____ out of all their naughty behavior, and (12)_____ up serious hobbies (13)_____ as chess and playing the piano. They never did anything without (14)_____ it over first, and coming to a serious decision. She had to face up to the fact that they made her feel rather childish as they got (15)____, and that in some ways she preferred them when they were young and noisy. Part 3: The paragraphs of the magazine article are in the wrong order. Number the paragraphs (A, B, C, D, E) (5 points) A. On the ghost of Chaffin’s second visit to his son, he told him that he would find a will in the overcoat pocket. The coat was actually in the possession of the third brother. B. In 1921, a certain James Chaffin died, leaving his entire fortune to his third son, Marshal, in a will which had been written a full fifteen years earlier, in 1905, and signed in front of witnesses. His wife and two other sons were virtually cut off without a penny. Marshal was not inclined to split up the inheritance he had come into any more fairly. C. Once it was found, they came across a note sewn in the lining of one of the pockets saying they should look in an old family Bible. This Bible was found in the keeping of Chaffin’s widow and examined in front of independent witnesses. Sure enough, there in the Bible they discovered a later version of the will, one which divided the property and BY ĐỖ BÌNH – THPT LIẾN SƠN, LẬP THẠCH, VĨNH PHÚC www.violet.vn/quocbinh72 money evenly between the widow and the three sons. The will appeared to be genuine and Marshal was not prepared to challenge it in court. D. Some people believe that the dead still keep in touch with us through our dreams. One of the most famous and extraordinary cases of contact with the dead was the so-called Chaffin Will affair. E. Four years went by and then, strangely, James Chaffin’s ghost started to appear before one of his other two sons. The apparition had on an old overcoat which had often worn in life. III. WRITING Part 1: Finish each of the following sentences in such a way that it means exactly the same as the sentence printed before it. (20 points) 1. Immediately after his appointment to the post, the new editor fell ill. Scarcely ................................................................................................................................................ 2. I left without saying goodbye as I didn’t want to disturb the meeting. Rather ................................................................................................................................................... 3. The value of sterling has fallen considerably in the past week. There has .............................................................................................................................................. 4. The only reason the party was a success was that a famous film star attended. Had it not .............................................................................................................................................. 5. Harriet was upset because she saw Peter with another woman. It was .................................................................................................................................................... 6. We regret to inform you that your application has not been successful. Much to ................................................................................................................................................ 7. People no longer smoke so many cigarettes as they used to. The ....................................................................................................................................................... 8. Their relationship was doomed because of their incompatibility. Had ....................................................................................................................................................... 9. You should admit that you are to blame, not to conceal it. I’d rather ............................................................................................................................................... 10. If she had been less determined she wouldn’t have been able to get better so quickly. It was her .............................................................................................................................................. SỞ GD&ĐT VĨNH PHÚC ———————— ĐỀ CHÍNH THỨC KỲ THI CHỌN HSG LỚP 1…. - THPT NĂM HỌC 201… - 201… (ĐỀ LUYỆN 02) ĐỀ THI MÔN: TIẾNG ANH ———————————— SECTION A: PHONETICS (10 POINTS) I. Choose one word whose underlined part is pronounced differently from the others. (5 pts) 1. A. resume B. statistics C. position D. designer B. initiate C. certificate D. interactive 2. A. criteria B. gesture C. gymnasium D. ghost 3. A. ginger 4. A. teammate B. reading C. seaside D. creating 5. A. crooked B. naked C. masked D. needed II. Choose one word whose stress pattern is different from the others. (5 pts) 1. A. oceanic B. argument C. temperature D. valuable 2. A. incredible B. fortunately C. astronomy D. evaporate 3. A. committee B. expensive C. guarantee D. successful 4. A. vocabulary B. assistance C. develop D. evidence 5. A. applicant B. recognize C. yesterday D. curriculum SECTION B: GRAMMAR & VOCABULARY (40 POINTS) I. Complete the following sentences with the correct form of the verbs in brackets. (10 pts) 1- 2- 3. My father (work) in Canada for the last year, so by the time he (return) the month after next, I (not see) him for fourteen months. 4. We looked out of the window and saw it (rain), so we stayed in. 5. TV chat shows (increase) in number all the time. 6. The camera (disappear) when we came in. 7. The woman (accuse) of the killing said that she was at the cinema at the time. 8 - 9. Not until we (arrive) at his house we (discover) that he was on holiday. 10. After (take) to the hospital, the injured victim felt much better yesterday. BY ĐỖ BÌNH – THPT LIẾN SƠN, LẬP THẠCH, VĨNH PHÚC www.violet.vn/quocbinh72 II. Choose the best answer to complete the sentences. (10 pts) 1. There is a rumour that the National Bank is going to ______ the company I am working for. A. take on B. take out of C. take off D. take over 2. It would be lovely if the children could see and, ______, touch the animals. A. eventually B. if possible C. at last D. finally 3. ______ he failed in the entrance exam caused his family much sadness. A. Which B. That C. The thing D. What 4. Unless you return money immediately, you will risk being charged ______ theft and getting into serous trouble A. on B. with C. of D. for 5. The old woman came in, ______ by a young man. A. accompany B. to be accompanied C. accompanying D. accompanied 6. I’ve applied for the job I saw ______ in the newspaper last month. A. advertised B. advertising C. be advertised D. being advertised 7. No, I didn’t know his number; ______ I’d have phoned him. A. otherwise B. so C. therefore D. unless 8. ______ the two sisters, Mary is _______. A. Of/ the prettier B. Between/ the prettiest C. Of/ prettier D. Between/ the prettier 9. A: Do you think it will rain tonight? B: ______. I am attending the evening class. A. I don’t hope so B. I hope so C. I hope not D. I am not hoping 10. Ninety ______ the maximum length of time allowed for entrance exams to this school. A. minute is B. minutes are C. minute are D. minutes is III. Read the text below. Give the correct form of the words in brackets. (10pts) JUDO Judo is a sport that has achieved great popularity in many parts of the world. It was (1. origin) _____ developed in Japan in the late 19th century based on ancient methods of self-defence. There are two (2. fight) _____. Although they use physical (3. violent) _____ against each other, they are respectful to their (4. oppose) _____ and bow to each other before and after each contest. Judo is an (5. expense) _____ sport to take up because the only equipment you need is the special loose-fitting suit. It is very suitable for (6. young) _____ if they join a club where the (7. instruct) _____ are properly qualified and pay enough attention to safety. Although Judo is a physically (8. demand) _____ sport which requires a lot of (9. strong) _____ practice, and skill, there are many people who find it (10. enjoy) _____ as a means of relaxation in their spare time. IV. Find one mistake in each sentence below by choosing the letter A, B, C or D. (10 pts) 1. Joel giving up smoking has caused him to gain weight and become irritable with his acquaintances. A B C D 2. There are many different ways of comparing the economy of one nation with those of another. A B C D 3. George won’t lend you any money as long as you promise to pay him back. A B C D 4. Having worked hard during the summer, his result was very successful in the entrance examination A B C D 5. Society will be having to change radically to keep pace with the technology available. A B C D 6. It is necessary that he goes to see a doctor as soon as he returns from taking the exam. A B C D 7. The lion has long been a symbol of strength, power, and it is very cruel. A B C D 8. The Nobel Prize winning candidate, together with his wife and children, are staying in Sweden after the presentation. A B C D 9. My brother composes not only the music, but also sings the songs for the major Broadway musicals. A B C D 10. After driving for twenty miles, he suddenly realized that he has been driving in the wrong direction. A B C D SECTION C: READING (30 POINTS) I. Fill in each numbered space with ONE appropriate word. (10 pts) TSUNAMI IN JAPAN BY ĐỖ BÌNH – THPT LIẾN SƠN, LẬP THẠCH, VĨNH PHÚC www.violet.vn/quocbinh72 Japan's most powerful earthquake since records began has struck the north-east coast, triggering a massive tsunami. Cars, ships and buildings were (1) ______ away by a wall of water after the 8.9 - magnitude tremor, which struck about 400 kms (250 miles) north-east of Tokyo. A state of emergency has been declared at a nuclear power plant, (2) ______ pressure has exceeded normal levels. Officials say more than 10,000 people are dead and about 7,000 (3) ______, but it is feared the final death toll will be (4) ______ higher. In one ward alone in Sendai, a port city in Miyagi prefecture, 200 to 300 bodies were found. “The quake has been the fifth-largest in the world (5) ______ 1900 and nearly 8,000 (6) ______ stronger than the one which devastated Christchurch, New Zealand, last month”, said scientists. Thousands of people (7) ______ near the Fukushima nuclear power plant have been ordered to evacuate. Japanese nuclear officials said that pressure inside a boiling water reactor at the plant was running much higher than normal after the cooling system failed. Officials said they might need to deliberately (8) ______ some radioactive steam to relieve pressure, but that there would be no health risk. US Secretary of State Hillary Clinton had earlier said the US Air Force had flown emergency coolant to the site. But US officials later said (9) ______ coolant had been handed over because the Japanese had decided to handle the situation (10) ______. The UN's nuclear agency said four nuclear power plants had been shut down safely. II. Read the passage below and then choose the correct answer A, B, C or D. (10pts) Women nowadays have more (1) ______ than those in the past. For example, our great grandmothers were not allowed to go to school or to work to earn their own living. (2) ______, they had to depend on their husbands financially. Modern women, on the contrary, can get good education, have their own careers, and (3) ______ their interests. They can even take good positions in politics if they are competent (4) ______ it. However, women living in our modern society have their (5) ______ too. Today women work harder than their great grandmothers so that they can gain the (6) ______ between working life and family life. Many people predict that by 2032, most (7) ______ positions at work will be taken by women. Then, it is possible that women will have more (8) ______ life because, (9) ______ in a very modern society, the women can’t (10) ______ their role in the family. 1. A. advances B. advantages C. benefits D. conveniences 2. A. Therefore B. However C. As a result D. Although 3. A. pursue B. support C. promote D. stimulate 4. A. to B. at C. with D. of 5. A. obstacles B. disputes C. profits D. problems 6. A. equality B. stables C. balance D. steadiness 7. A. senior B. junior C. inferior D. superior 8. A. sheltered B. healthy C. strenuous D. active 9. A. though B. even C. ever D. never 10. A. perform B. adopt C. fulfill D. neglect III. Read the passage below and then choose the correct answer A, B, C or D. (10 pts) Walt Disney was born in 1901 in Chicago, but soon moved to a small farm near Marceline, Missouri. From this rural and rather humble beginning, he later became one of the most famous and beloved motion-picture producers in history. Although he died in 1966, his name and artistic legacy continue to influence the lives of millions of people throughout the world. After several years of barely making ends meet as a cartoon artist operating from his Los Angeles garage, Disney had his first success in 1928, with his release of a Mickey Mouse cartoon. Throughout the next decade, he continued to produce a number of cartoons, and develop more of his highly profitable and enduring creations, such as Donald Duck and Pluto. In the late 1930s, he issued a full-length cartoon film. Snow White became an instant commercial and critical success. This was only the first of many films, both animated and not, produced by Disney and his studio. But as renowned as Disney name is for cartoons and movies, it is probably best known for a string of spectacular amusement and theme parks. Starting with California’s Disneyland in 1955 and culminating with the fantastically successful Disney World and EPCOT Center in Florida, Disney became a household name. In recent years, the theme park concept became international, with openings in Tokyo and Paris. With the continuing success of Disney, the creation of future theme park is under discussion. 1. Which of the following is the title for the passage? A. The history of Disney World and Disneyland. B. Walt Disney’s Boyhood Years C. Walt Disney and his Legacy. D. Walt Disney and Animated Cartoons 2. The word “humble” in bold in paragraph 1 is closest in meaning to __________. A. boring B. shy C. simple D. respectful 3 What is the author’s attitude toward the accomplishments of Walt Disney? A. critical B. respectful C. ambivalent D. approving 4. In paragraph 2 “barely making ends meet” in bold is closest in meaning to __________. BY ĐỖ BÌNH – THPT LIẾN SƠN, LẬP THẠCH, VĨNH PHÚC www.violet.vn/quocbinh72 A. meeting personal failure. B. producing only a few cartoons. C. not making much money. D. trying new businesses. 5. The word “enduring” in bold in paragraph 2 is closest in meaning to __________. A. lasting B. suffering C. famous D. difficult 6. The word “renowned” in bold in paragraph 3 is closest in meaning to _________. A. possessed B. talked about C. useful D. well-known 7. It can be inferred from the passage that ________. A. Snow White was Disney’s most successful film. B. Disney created cartoon movies and “non cartoon” movies. C. the Tokyo theme park is in financial difficulty. D. the California theme park is now closed. 8. The word “concept” in bold in paragraph 3 is closest in meaning to __________. A. idea B. location C. birth D. demand 9. According to the passage, which of the following is TRUE? A. Disney’s first achieved success after his death. B. Mickey Mouse was Disney’s only cartoons creation. C. Snow White was the first full-length cartoon film. D. Disney’s first concern was always profitability. 10. In future years it is most likely that ___________. A. the Disney name will stay well–known. B. Disney will produce only cartoons. C. the Paris theme park will become successful D. the remaining theme parks will also close SECTION D: WRITING (20 POINTS) I. Finish the second sentence so that it means the same as the first one. (10 pts) 1. Peter hasn’t had his hair cut for over 3 months. → It is _____ 2. I meant to post that letter before lunch. → What _____ 3. People will only become aware of the problem if you increase the publicity. → Only if _____ 4. Scientists have tried very hard to find a cure for AIDS. → Enormous _____ 5. Success depends on hard work. → The harder _____ 6. She thought it was too difficult for her to finish the job by lunchtime. → She found _____ 7. They thought that the little girl had found out the keys accidentally. → The keys _____ 8. Someone has suggested abolishing income tax. → It _____ 9. It was wrong of you to allow a 4-year-old child to walk home alone. → You should _____ 10. We were late for the meeting because it rained heavily. → But for _____ II. Use the given word to write the second sentence in such a way that it is as similar as possible in meaning to the original sentence. Do not change the form of the given word. (5pts) 1. They decided to build a new school in this region. SHOULD 2. She got a scholarship as soon as she registered for the training course. SCARCELY 3. Flooding in this region was the result of heavy rain RESULTED 4. I don't like him because he boasts a lot. MOUTH 5. The Embassy said it would not be necessary for me to get a visa. NEED III. Complete the following sentences, using the words given. (5 pts) 1. Exercise / take regularly / improves / fitness / our body / mind. 2. Economic / reform / one / most / important / measure / take / promote / development / country 3. Sleep / next room / boy / wakened / sound / breaking glass. 4. Never / put / tomorrow / you / do / today. 5. Many people / say / most / common / way / attract / someone / attention / by / wave. SỞ GD&ĐT VĨNH PHÚC ———————— ĐỀ CHÍNH THỨC KỲ THI CHỌN HSG LỚP 1…. - THPT NĂM HỌC 201… - 201… (ĐỀ LUYỆN 03) ĐỀ THI MÔN: TIẾNG ANH ———————————— SECTION A: PHONETICS (10 POINTS) I. Choose one word whose underlined part is pronounced differently from the others. (5 pts) 1. A. opponent B. compose C. podium D. advocate B. domestic C. optimistic D. nursery 2. A. reserve 3. A. both B. cloth C. ghost D. sold 4. A. examine B. determine C. famine D. dine 5. A. sacred B. decided C. contaminated D. watered II. Choose one word whose stress pattern is different from the others. (5 pts) 1. A. counterpart B. precede C. nursery D. compliment BY ĐỖ BÌNH – THPT LIẾN SƠN, LẬP THẠCH, VĨNH PHÚC www.violet.vn/quocbinh72 2. 3. 4. 5. A. bewilder A. pessimistic A. tsunami A. processor B. audience C. benefit D. dedicate B. university C. epidemic D. particular B. terrorist C. involvement D. disaster B. windsurfing C. semester D. challenger SECTION B: GRAMMAR & VOCABULARY (40 POINTS) I. Choose the best answer to complete the sentences. (10 pts) 1. According to the boss, John is the most ______ for the position of executive secretary. A. supportive B. caring C. suitable D. comfortable 2. The children went _______ with excitement. A. wild B. wildly C. wilderness D. wildlife 3. The sudden resignation of the financial director put the company in a very _______ position. A. weak B. unsteady C. vulnerable D. collapsed 4. David: Would you like fish or meat? Mary: I _______ fish, please. A. would rather B. would prefer C. suppose D. believe 5. Many teenagers show signs of anxiety and _______ when being asked about their future. A. depress B. depression C. depressed D. depressing 6. There was a huge decline _______ the number of tigers. A. in B. for C. of D. out 7. I’d rather you _______ anything about the garden until the weather improves. A. don’t make B. didn’t do C. don’t do D. didn’t make 8. A part – time job gives me the freedom to _______ my own interests. A. pursue B. chase C. seek D. catch 9. The new road currently under _______ will solve the traffic problems in the town. A. design B. progress C. construction D. work 10. - Daisy: “What a lovely house you have!” - Mary: “______.” A. Lovely, I think so B. Thank you. Hope you will drop in C. Of course not, it’s not costly D. No problem II. Complete the following sentences with the correct form of the verbs in brackets. (10 pts) 1. Bi Rain, together with 58 members of the South Korean National Military Symphony Orchestra and 17 traditional musicians, (come) to Vietnam since yesterday. _______________ 2. Up to now, nothing (do) to solve their problem. _______________ 3. He suggested that his son (be) on time for the interview. _______________ 4. Tom will come home as soon as he (finish) his test. _______________ 5. ASEAN (found) in 1967 in Bangkok, Thai land. _______________ 6. In times of war, the Red Cross (dedicate) to reducing the sufferings of wounded _______________ soldiers, civilians, and prisoners of war. 7. Hardly our teacher (enter) the classroom when it started to rain. _______________ 8. In a few minutes' time, when the clock strikes six, I (wait) for you here. _______________ 9-10. Living in a fast-paced and mobile society (create) family stresses that (not _______________ imagine) by our great grandparents. _______________ III. Give the correct form of the words in brackets. (10pts) 1. The main goals of the Association of Southeast Asian Nations are to promote peace and ______ in the region. (STABLE) 2. On my salary, we have to live as ______ as possible. (ECONOMY) 3. Different conservation efforts have been made in order to save ______ species. (DANGER) 4. The security of the earth can be threatened by ______ groups. (TERROR) 5. It is reported that humans are the main reason for most species' declines and habitat ______. (DESTROY) 6. He resigned for a ______ of reasons. (VARIOUS) 7. I don’t care if you had had too much to drink. Your behaviour last night was ______. (DEFEND) 8. Her son is always mischievous and ______ which annoys her very much. (OBEY) 9. The Americans are much more concerned than the Indians and the Chinese with physical (ATTRACT) ______ when choosing a wife or a husband. 10. You can never be sure what my sister is going to do. She is so ______. (PREDICT) IV. Find one mistake in each sentence below by choosing the letter A, B, C or D. (10 pts) 1. Although to some people reading is a favourite way to spend time, but others just do not like reading. A B C D BY ĐỖ BÌNH – THPT LIẾN SƠN, LẬP THẠCH, VĨNH PHÚC www.violet.vn/quocbinh72 2. If a species does not have the natural genetic protection against particular diseases, an introduced disease can A B C have severely effects on that species. D 3. I believe that only very self-confident, knowledge and attentive students will prefer 100% of eye contact time. A B C D 4. It is likely that all people in Hanoi live in skyscrapers by 2050. A B C D 5. In 1961, America’s first manned spacecraft launched. A B C D 6. Do you really think that candidate is qualify to be President? A B C D 7. Of the more than 1,300 volcanoes in the world, only about 600 can classify as active. A B C D 8. It is important that cancer is diagnosed and treated as early as possible in order to assure a successful cure. A B C D 9. It took us quite a long time to get here. It was a three-hours journey. A B C D 10. Married women are twice so likely as married men to be depressed. A B C D SECTION C: READING (30 POINTS) I. Fill in each numbered space with ONE appropriate word. (10 pts) Kim Jong-il (16th February 1941 – 17th December 2011) was the supreme leader of North Korea (1) ____ 1994 to 2011. Kim Jong-il died (2) _______a suspected heart (3) _______on 17th December 2011 while traveling by train to an area outside Pyongyang. He was succeeded by his youngest son Kim Jong-un, (4) _______was considered by the Korean Central News Agency (5) _______the "Great Successor". The Korean Central News Agency reported that during his death, a fierce snowstorm paused and the sky glowed red above the sacred Mount Paektu. The ice on a famous lake also cracked so loud, it seemed to shake the Heavens and the Earth. Kim Jong-il's funeral took (6) _______on December 28th in Pyongyang, with a mourning period lasting until the (7) _______day. South Korea's military was immediately put on alert after the announcement. Asian stock markets fell soon after the announcement, due to similar concerns. (8) _______January 12th, 2012 North Korea called Kim Jong-il the "eternal leader" and announced that his (9) _______will be preserved and displayed at Pyongyang's Kumsusan Memorial Palace. Officials will also install statues, portraits, and "towers to his immortality" across the country. His (10) _______of February 16th has been declared "the greatest auspicious holiday of the nation", and has been named the Day of the Shining Star. II. Read the passage below and then choose the correct answer A, B, C or D. (15pts) Do you ever wish you were more optimistic, someone who always (1) ______ to be successful? Having someone around who always (2) ______ the worst isn’t really a lot of (3) ______ – we all know someone who sees a single cloud on a sunny day and says, ‘It looks (4) ______ rain.’ But if you catch yourself thinking such things, it’s important to do something (5) ______ it. You can change your view of life, (6) ______ to psychologists. It only takes a little effort, and you will find life more rewarding as a (7) ______. Optimism, they say, is partly about self-respect and confidence, but it’s also a more positive way of looking at life and all it has to (8) ______. Optimists are more (9) ______ to start new projects and are generally more prepared to take risks. Upbringing is obviously very important in forming your (10) ______ to the world. Some people are brought up to (11) ______ too much on others and grow up forever blaming other people when anything (12) ______ wrong. Most optimists, on the (13) ______ hand, have been brought up not to (14) ______ failure as the end of the world- they just (15) ______ with their lives. 1. A. counted B. expected C. felt D. waited 2. A. worries B. cares C. fears D. doubts 3. A. amusement B. play C. enjoyment D. fun 4. A. so B. to C. for D. like 5. A. with B. against C. about D. over 6. A. judging B. according C. concerning D. following 7. A. result B. reason C. purpose D. product 8. A. supply B. suggest C. offer D. propose BY ĐỖ BÌNH – THPT LIẾN SƠN, LẬP THẠCH, VĨNH PHÚC www.violet.vn/quocbinh72 9. A. possible B. likely C. hopeful D. welcome 10. A. opinion B. attitude C. view D. position 11. A. trust B. believe C. depend D. hope 12. A. goes B. fails C. comes D. turns 13. A. opposite B. next C. other D. far 14. A. regard B. respect C. suppose D. think 15. A. get up B. get on C. get out D. get over III. Read the passage below and then choose the correct answer A, B, C or D. (5pts) In the second half of each year, many powerful storms are born in the tropical Atlantic and Caribbean seas. Of these, only about half a dozen become the strong, circling winds of 74 miles per hour or more that are called hurricanes, and several usually make their way to the coast. There they cause millions of dollars of damage, and bring death to large numbers of people. The great storms that hit the coast start as soft circling wind hundreds - even thousands - of miles out to sea. As they travel aimlessly over water warmed by the summer sun, they are carried westward by the southeast winds. When conditions are just right, warm moist air flows in at the bottom of such a wind, moves upward through it and comes out at the top. In the process, the moisture in this warm air produces rain, and with it, the heat is changed to energy in the form of strong winds. As the heat increases, the young hurricane begins to move counter clockwise motion. The life of a hurricane is only about nine days, but it contains almost more power than we can imagine. The energy in the heat released by a hurricane’s rainfall in a single day would satisfy the entire electrical needs of the United States for more than six months. Water, not wind, is the main source of death and destruction in a hurricane. A typical hurricane brings 6 to 12 inches downpour, causing sudden floods. Worst of all is the powerful movement of the sea – the mountains of water moving toward the hurricane center. The water level rises as much as 15 feet above normal as it moves toward shore. 1. When is an ordinary tropical storm called a hurricane? A. When it begins in the Atlantic and Caribbean seas. B. When its winds reach 74 miles per hour. C. When it is more than 74 miles wide. D. When it hits the Coastline. 2. What is the worst thing about hurricanes? A. The terrible effects of water. B. The heat they give off. C. That they last about nine days. D. Their strong winds. 3. Here the underlined word “downpour” means______ A. heavy rainfall B. dangerous waves C. the progress of water to the hurricane center D. the increasing heat 4. Which of the following statements about a hurricane is NOT true? A. It travels more than 75 miles per hour. B. It usually stays about nine days. C. It usually causes 6 to 12 inches downpour. D. It sometimes brings the sea water level to the height of 15 feet. 5. Hurricanes often cause ______? A. a lot of damage B. sudden floods C. death to large numbers of people D. All are correct SECTION D: WRITING (20 POINTS) I. Finish the second sentence so that it means the same as the first one. (10 pts) 1. We couldn’t relax until all the guests had gone home. Only ______ 2. House prices have risen sharply this year. There has ______ 3. The only reason the party was a success was that a famous film star attended. Had it not ______ 4. Would you mind not smoking in my house? I’d rather ______ 5. His second attempt on the world record was successful. He broke ______ 6. I write to him almost every day. Hardly ______ 7. Experts think that all dogs evolved from wolves. All dogs are ______ 8. Don’t go to lunch until you have typed all these letters. Make sure you finish ______ 9. The northwest of Britain has more rain each year than the southeast. The annual ______ 10. Henry regretted buying the second-hand car. Henry wishes ______ II. Use the given word to write the second sentence in such a way that it is as similar as possible in meaning to the original sentence. Do not change the form of the given word. (5pts) 1. The two theories appear to be completely different. COMMON 2. His rude behaviour is too much for me. PUT 3. I find his handwriting very hard to read. DIFFICULTY 4. He doesn’t have money to go on holiday. He doesn’t have time, either. NEITHER 5. You’d feel better if you had a quiet holiday. DO BY ĐỖ BÌNH – THPT LIẾN SƠN, LẬP THẠCH, VĨNH PHÚC www.violet.vn/quocbinh72 III. Complete the following sentences, using the words given. (5 pts) 1. Mother/ take/ responsibility/ run/ household. 2. ASEAN / have/ population/ 575.5 million/ account/ 8.7 %/ the world’s population. 3. It/ not/ until/ Einstein/ eight/ he/ can/ speak. 4. Defensive players / not/ allow/ interfere/ opponent’s movements/ unless/ player/ hold/ ball. 5. Never/ stop/ try/ you/ get/ right solution/ problem. THE END SỞ GD&ĐT VĨNH PHÚC ———————— ĐỀ CHÍNH THỨC KỲ THI CHỌN HSG LỚP 1…. - THPT NĂM HỌC 201… - 201… (ĐỀ LUYỆN 04) ĐỀ THI MÔN: TIẾNG ANH ———————————— PART I. GRAMMAR AND VOCABULARY I. Complete the following sentences by choosing the correct answer among four options (A, B, C or D). (15 pts) 1. She looked very______ when I told her the good news. A. happily B. happiness C. happy D. was happy 2. I don't want much sugar in my coffee. Just_______, please. A. few B. a few C. little D. a little 3. He's left his book at home; he's always so______. A. forgetful B. forgettable C. forgotten D. forgetting 4. 'I'm very ______ to you for putting in so much hard work,' the boss said. A. thanking B. grateful C. considerate D. careful 5. Steel_______ with chromium to produce a noncorrosive substance known as stainless steel. A. is combined B. that is combined C. combining D. when combined 6. Joe seemed to be in a good mood, ________ he snapped at me angrily when I asked him to join us. A. yet B. so C. for D. and 7. A fire must have a readily available supply of oxygen. ________, it will stop burning. A. Consequently B. Furthermore C. Otherwise D. However 8. “Who has brothers and sisters?” “Everyone ________Virginia who’s an only child.” A. except to B. with C. from D. but 9. He is the manager of the factory. He’s ________it. A. charged with B. charged C. in charge D. in charge of 10. He was new on the job, but he quickly fit himself into the ________ routine of the office. A. establishing B. established C. establishes D. establish 11. Florida, ________ the Sunshine State, attracts many tourists every year. A. is B. is known as C. known as D. that is known as 12. ________becoming extinct is of great concern to zoologists. A. That giant pandas are B. Giant pandas are C. Are giant pandas D. Giant panda is 13. Her parents gave her everything she asked for. She's a completed _______ child. A. wounded B. spoilt C. damaged D. destroyed 14. _______ my experience, very few people really understand the problem. A. To B. In C. With D. From 15. When she died, she gave _________ all her money to a charity for cats. A. away B. out C. on D. off II. Complete the following passage by supplying the correct form of the word to fill in each blank. (10 pts) The increase in city crime is a global phenomenon. Some people say that a lot of crime in this country is because of (1 - migrate) ____ and the new people arriving from other countries bring different (2 - culture) ____ values and attitudes to the law. I don’t agree with this idea because the most common crimes are (3 - local) ____ produced and not imported from other countries. (4 - Vandal) ____ is one of the biggest crimes in my city with bus shelters and shop windows being popular targets. Another popular crime is (5 - pay) ____ parking fines, which is unlikely to be because of immigrants because most of them do not own cars. The (6 - oppose) ____ point of view is that young local people feel angry when they can’t get a job and in order to (7 - hand) ____ the change in their environment, they strike out at easy targets. This would explain why bicycle (8 - thief) ____ is more common than car crime these days, especially in rich (9 neighbors) ____ where most cars are protected with electronic alarms. Another reason, though, for so many bicycles getting stolen might be that the (10 - punish) ____ is not very severe compared to car stealing which can land you in prison for a number of years. III. Fill in each blank with one suitable preposition. (5 pts) BY ĐỖ BÌNH – THPT LIẾN SƠN, LẬP THẠCH, VĨNH PHÚC www.violet.vn/quocbinh72 1. Most American men earn _______ average about $110 a week. 2. He was walking through the park when a strange dog suddenly went _______ him. 3. My car is not worth much _______ most $ 50. 4. I’d better not drink that milk, Joe. It’s gone _______. 5. Clive was really cut _______ when he failed his proficiency exam. IV. Identify the error in each sentence. Write the corresponding letter A, B, C or D on the answer sheet. (10 pts) 1. The children forgot picking up the note from the office and now they are worried. A B C D 2. Helen has never met such good person who is ready to help others. A B C D 3. If only I have done the test better than other students did. A B C D 4. If you want to borrow my car, promise to drive careful. A B C D 5. Rumors began circulating that the Prime Minister was seriously illness. A B C D 6. Everybody in class has to choose a topic of your own to write an essay of 500 words. A B C D 7. Neither of the boys had ever been out of town before, so they were really exciting. A B C D 8. I hope that I can help you with the historic questions. A B C D 9. My son learned to talk the time before he was 5 years old. A B C D 10. Jim offers us presents as if it had been Xmas. A B C D PART III: READING I. Read the passage carefully, then choose the correct option (marked A, B, C or D) to answer the questions. (10 pts) Millions of people are using cell phones today. In many places, it is actually considered unusual not to use one. In many countries, cell phones are very popular with young people. They find that the phones are means of communication – having a mobile phone shows that they are cool and connected. The explosion in mobile phone use around the world has made some health professionals worried. Some doctors are concerned that in the future many people may suffer health problems from the use of mobile phones. In England, there has been a serious debate about this issue. Mobile phone companies are worried about the negative publicity of such ideas. They say that there is no proof that mobile phones are bad for your health. On the other hand, medical studies have shown changes in the brain cells of some people who use mobile phones. Signs of change in the issues of the brain and head can be detected with modern scanning equipment. In one case, a traveling salesman had to retire at young age because of serious memory loss. He couldn't remember even simple tasks. He would often forget the name of his own son. This man used to talk on his mobile phone for about six hours a day, every day of his working week, for a couple of years. His family doctor blamed his mobile phone use, but his employer's doctor didn't agree. What is it that makes mobile phones potentially harmful? The answer is radiation. High – tech machines can detect very small amounts of radiation from mobile phones. Mobile phone companies agree that there is some radiation, but they say the amount is too small to worry about. As the discussion about their safety continues, it appears that it's best to use mobile phones less often. Use your regular phone of you want to talk for a long time. Use your mobile phone only when you really need it. Mobile phones can be very useful and convenient, especially in emergencies. In the future, mobile phones may have a warning label that says they are bad for your health. So for now, it's wise not to use your mobile phone too often. 1. According to the passage, cell phones are especially popular with young people because ______. A. they are indispensable in everyday communication B. they make them look more stylish C. they keep the users alert all the time D. they cannot be replaced by regular phones 2. The changes possible caused by the cell phones are mainly concerned with______. A. the mobility of the mind and the body B. the smallest units of brain C. the arteries of the brain D. the resident memory 3. The word "means" in the passage most closely means _______. A. meanings B. expression C. method D. transmission 4. The word "potentially" in the passage most closely means _______. BY ĐỖ BÌNH – THPT LIẾN SƠN, LẬP THẠCH, VĨNH PHÚC www.violet.vn/quocbinh72 A. obviously B. possibly C. certainly D. privately 5. "Negative publicity" in the passage most likely means _______. A. information on the lethal effects of cell phones B. widespread opinion about bad effects of cell phones C. the negative public use of cell phones D. poor ideas about the effects of cell phones 6. Doctors have tentatively concluded that cell phones may _______. A. damage their users' emotions B. cause some mental malfunction C. change their users' temperament D. change their users' social behavior 7. The man mentioned in the passage, who used his cell phone too often _______. A. suffered serious loss of mental ability B. could no longer think lucidly C. abandoned his family D. had a problem with memory 8. According to the passage, what makes mobile phones potentially harmful is ______. A. their radiant light B. their power of attraction C. their raiding power D. their invisible rays 9. According to the writer, people should _______. A. only use mobile phones in urgent cases B. only use mobile phones in medical emergencies C. keep off mobile phones regularly D. never used mobile phones in all cases 10.The most suitable title for the passage could be ______. A. "The reasons why mobile phones are regular" B. "Technological Innovation and their price" C. "The way mobile phones work" D. "Mobile phones – a must of our time" II. Complete the following passage by choosing A, B, C or D to fill in each blank. (15 pts) Stamp collecting! What a wonderful hobby! I began when I was only five. I used to (1) ____ for the postman's arrival, always (2) ____ to seize unwanted envelopes and tear off the corner with the stamp stuck on it. Once – I remember it was too clearly – my mother and father were sunning themselves in the garden when the post (3) ____ on the doormat. I heard the clatter of the letter flap and hurriedly went to (4) ____. There were four or five envelops, all with very exciting stamps. Even at the (5) ____ age of five I knew one doesn't open mail addressed to other people. However, tearing just the corners off the envelopes (6) ____ me as perfectly fair and allowable, and just what I did. I carefully tore as (7) ____ to the stamps as (8) ____, feeling that even the envelopes, which were addressed to my parents and not to be, should be treated with (9) ____. There was nothing furtive in what I did. I knew my parents would see what I'd done, and I didn't think there was any (10) ____ in it. They always let me (11) ____ the corners after they'd opened them. Why should I think there was any harm in doing it first, (12) ____ in mind that they weren't on hand to be (13) ____. Wouldn't they rather be left to doze in their summer deckchairs? (14) ____, though, my father solemnly showed me his letters. They looked distinctly moth-eaten, with bites taken out of the corners and sites. I began to (15) ____ what I've done. 1. A. stare B. watch C. look D. peer 2. A. glad B. pleased C. eager D. excited 3. A. came B. was C. lay D. arrived 4. A. investigate B. observe C. see D. notice 5. A. junior B. tender C. small D. little 6. A. struck B. seemed C. appeared D. felt 7. A. nearby B. close C. next D. round 8. A. able B. possibly C. possible D. could 9. A. gentleness B. caution C. honor D. respect 10. A. trouble B. wrong C. bad D. harm 11. A. take B. tear C. cut D. remove 12. A. having B. holding C. bearing D. keeping 13. A. consulted B. advised C. queried D. requested 14. A. After B. Then C. Later D. Soon 15. A. accept B. realize C. admit D. confess III. Read the following passage and answer the questions that follow. Write A, B, C or D to indicate your answers on the answer sheet. (10 pts) Every drop of water in the ocean, even in the deepest parts, responds to the forces that create the tides. No other force that affects the sea is so strong. Compared with the tides, the waves created by the wind are surface movements felt no more than a hundred fathoms below the surface. The currents also seldom involve more than the upper several hundred fathoms despite their impressive sweep. The tides are a response of the waters of the ocean to the pull of the Moon and the more distant Sun. In theory, there is a gravitational attraction between the water and even the outermost star of the universe. In reality, however, the BY ĐỖ BÌNH – THPT LIẾN SƠN, LẬP THẠCH, VĨNH PHÚC www.violet.vn/quocbinh72 pull of remote stars is so slight as to be obliterated by the control of the Moon and, to a lesser extent, the Sun. Just as the Moon rises later each day by fifty minutes, on the average, so, in most places, the time of high tide is correspondingly later each day. And as the Moon waxes and wanes in its monthly cycle, so the height of the tide varies. The tidal movements are strongest when the Moon is a sliver in the sky, and when it is full. These are the highest flood tides and the lowest ebb tides of the lunar month and are called the spring tides. At these times the Sun, Moon, and Earth are nearly in line and the pull of the two heavenly bodies is added together to bring the water high on the beaches, to send its surf upward against the sea cliffs, and to draw a high tide into the harbors. Twice each month, at the quarters of the Moon, when the Sun, Moon and Earth lie at the apexes of a triangular configuration and the pull of the Sun and Moon are opposed, the moderate tidal movements called neap tides occur. Then the difference between high and low water is less than at any other time during the month. 1. What is the main point of the first paragraph? A. The waves created by ocean currents are very large. B. Despite the strength of the wind, it only moves surface water. C. Deep ocean water is seldom affected by forces that move water. D. The tides are the most powerful force to affect the movement of ocean water. 2. The words "In reality" in the passage is closest in meaning to_____. A. surprisingly B. actually C. characteristically D. similarly 3. It can be inferred from the passage that the most important factor in determining how much gravitational effect one object in space has on the tides is_____. A. size B. distance C. temperature D. density 4. The word "configuration" in the passage is closest in meaning to_____. A. unit B. center C. surface D. arrangement 5. Neap tides occur when_____. A. the Sun counteracts the Moon's gravitational attraction B. the Moon is full C. the Moon is farthest from the Sun D. waves created by the wind combine with the Moon's gravitational attraction PART IV. WRITING I. Write the new sentences using the word give. Do not change the word given in any way. (10 pts) 1. We are afraid that the next train to Cardiff has been cancelled. REGRET We ________________________ the next train to Cardiff has been cancelled. 2. 'I was going to phone Sarah but I had too much homework to do', said John MEANT John said that ___________________ but he'd had too much homework to do. 3. Many people think that Gone with the Wind is one of the greatest films ever made. BE Gone with the Wind _________________ one of the greatest films ever made. 4. If you hold your breath for a few seconds it might get rid of your hiccups. HOLDING You might be able to get rid of your hiccups if you _____________ for a few seconds. 5. Anna didn't like it at all when one of her colleagues got the promotion instead of her. RESENTED Anna _____________________ the promotion instead of her. 6. I have a good relationship with my neighbors. ON My neighbors __________________well together. 7. It wasn't a good idea for me to spend all my money on beer and cigarettes, but I did. UP I wish ________________________ my money on beer and cigarettes. 8. The police left and then all the journalists arrived. ALREADY The police ____________________all the journalists arrived. 9. Although Sarah wasn't wearing a helmet, she wasn't injured in the accident. OF Sarah wasn't injured in the accident, ______________ a helmet. 10. She felt she had achieved a lot in life, despite her disadvantaged background. SENSE Despite her disadvantaged background, she felt _______________ about her life. II. Write a complete letter using the given word/phrases as cues. You should change the form of words if necessary. (15 pts) 1. Thank/ much/French/ cookery book/ you/ give/ me. 2. recipes/ look/ wonderful/, and/ I/ certainly/ enjoy/ use/ it. 3. I/ hope/ you/ and/ Auntie Susan/ have/ good/ Christmas. 4. we/ stay/ at/ home/, but/ have/ couple/ day/ trips/ out/ nearby. 5. our/ local/ theatre/ put/ on/ production/ "A Christmas Carol"/, which/ fun. 6. I/ go/ back/ University/ next/ week/, and/ this/ year/ I/have/ to/ study/ hard/ final/ exams. 7. I/ really/ enjoy/ the/ course/ so/ far/, but/ I/ not/ sure/ what/ do/ once/ I/ get/degree. BY ĐỖ BÌNH – THPT LIẾN SƠN, LẬP THẠCH, VĨNH PHÚC www.violet.vn/quocbinh72 8. university/ arrange/ career/ interviews/, but/ I/ not/really/ have/clear/ idea/what/ I/ want/ do. 9. hopefully/ it/ all/ become/ clearer/ during/ course/ the/ year. 10. Thank/ again/ lovely/ present/, and/ Happy/ New/ Year/ THE END SỞ GD&ĐT VĨNH PHÚC ———————— ĐỀ CHÍNH THỨC KỲ THI CHỌN HSG LỚP 1…. - THPT NĂM HỌC 201… - 201… (ĐỀ LUYỆN 05) ĐỀ THI MÔN: TIẾNG ANH ———————————— SECTION A: PHONETICS I. Pick out the word whose underlined part is pronounced differently from that of the other words. Write your answer in the space provided. (5 pts) B. sociable C. ancient D. ancestor 1. A. associate 2. A. walked B. threatened C. passed D. forced 3. A. too B. food C. soon D. good 4. A. legal B. legend C. generous D. manager 5. A. adventure B. future C. mature D. figure II. Pick out the word that differs from the other words in the position of the main stress. Write your answer in the space provided. (5 pts) 6. A. comedy B. collection C. comical D. calculate 7. A. ambitious B. memorial C. memory D. mechanic 8. A. remove B. cancel C. copy D. answer 9. A. Japanese B. engineer C. practical D. questionnaire 10. A. document B. develop C. opponent D. astonish SECTION B: VOCABULARY AND GRAMMAR I. Choose the word or phrase which best completes each sentence. Write your answer in the space provided. (15 pts) 11. Secondary schools offer a wide ______ of subjects. A. field B. scope C. list D. range 12. When he woke up, he realized that the things he had dreamt about could not ______ have happened. A. possibly B. likely C. certainly D. potentially 13. - “Do you think the book is expensive? - “Yes, it’s not ______ what we paid for it.” A. worthy B. worth of C. worth D. valuable 14. To ______ extent did she benefit from her uncle’s will? A. what B. how C. which D. whom 15. The new system didn’t ______ expectations. A. catch up with B. bring about C. come across D. come up to 16. Before the meeting finished, they had arranged when ______ next. A. they met B. they to meet C. to meet D. should they meet 17. ______ aren’t effective anymore because insects have become resistant to them. A. Fertilizers B. Pesticides C. Herbicides D. Composts 18. When he heard the joke, he burst into loud ______. A. smile B. laughter C. amusement D. enjoyment 19. The traffic lights ______ to green, and the car drove on. A. exchanged B. turned C. removed D. shone 20. It is a good idea to be ______ dressed when you go for an interview. A. finely B. boldly C. smartly D. clearly 21. We were so late that we ______ had time to catch the train. A. nearly B. almost C. hardly D. simply 22. They are going to make ______ excursion next month. A. a two-week B. two-weeks C. two weeks' D. a two-week's 23. I haven't had a very ______ week. I seem to have done nothing at all. A. extensive B. productive C. enthusiastic D. economic 24. The purpose of the survey was to ______ the inspectors with local conditions. A. inform B. acquaint C. instruct D. notify 25. Do you know the man ______ over there? A. interviewed B. interviewing C. to interview D. interviews II. There is one mistake in each of the following sentences. Find and correct it. (10 pts) BY ĐỖ BÌNH – THPT LIẾN SƠN, LẬP THẠCH, VĨNH PHÚC www.violet.vn/quocbinh72 26. Because the torrential rains that had devastated the area, the governor sent the National Guard to assist in the clean-up operation. 27. One of the most important things in life is a good health. 28. The city has spent a big amount of money on crime prevention. 29. Comparing with other countries, Libya spends a high percentage of income on education. 30. People are now enjoying a higher level of living. 31. In the United Kingdom women see their doctor on the average five times a year. 32. Although Mark has been cooking for many years, he still doesn’t know to prepare French foods in the traditional manner. 33. When we arrived at the store to purchase the dishwasher advertise in the newspaper, we learned that all the dishwashers had been sold. 34. After rising the flag to commemorate the holiday, the mayor gave a long speech. 35. This time tomorrow I will lie on the beach, enjoying the sunshine. III. Give the correct form of the words in the brackets in each of the following sentences. Write your answer in the space provided. (10 pts) 36. I was annoyed at his (REFUSE)______ to co-operate. 37. The book doesn’t say much about prices, but it is very (INFORM)______ about everything else. 38. The noise (LESS)______ as the plane got farther away. 39. He lost in the election because he was a weak and (DECIDE)______ leader. 40. I couldn’t help it. The accident was (AVOID)______. 41. She was (EXTREME)______ knowledgeable about the history of China. 42. He was very (SET)_______ when his cat was run over. 43. Jackson had another violent (AGREE)_______ with the referee. 44. Many people were buried (LIVE)______ after the earthquake. 45. She studied (ECONOMY)______ at university. SECTION C: READING COMPREHENSION I. Fill in each numbered blank with one suitable word. Write your answer in the space provided. (10 pts) Australia is a big country, but nearly all Australians live near the sea. On hot summer days, you can see thousands of people at the beach. Many beaches have waves (46)______ are very high. These large waves are known as surf and the people who ride them are called surfers. Surfing is a skill, and it needs learning. Don’t (47)______ to be able to surf properly the (48)______ time you try. However, by practising a few times you will learn (49)______ to do it. Surfing is not a new sport. Perhaps its origins need explaining. It started hundreds (50)______ years ago in Hawaii. Men swam (51)______ to sea to catch fish and found they could come back to land very quickly by riding the waves. These first surfers did not (52)______ a board. They were “body surfers”. Many people (53)______ do this type of surfing today. After a while people started to use boards and rode the waves by lying, kneeling or standing (54)______ them. These first surfboards were made of wood and the water made them rot after a while. Today, surfboards are made of plastic or fibreglass (55)______. II. Read the following passage and decide which answer (A, B, C, or D) best fits each gap. Write your answer in the space provided. (10 pts) What do you do well? What do you enjoy doing? Your answers to these two questions will help you identify your (56)_____. An employer will consider you seriously for a (57)_____ when you can show them that you know who you are, what you can offer and which you have studied. Sometimes it is difficult to know what your weaknesses are. Clearly not everyone is equally good (58)_____ everything. You may need to improve yourself and so (59)_____ courses in that field could turn a weakness into strength. You will need to (60)______ some time on your self-assessment. Your honesty and the desire for selfimprovement will lead to (61)______ in getting the right job. Explore the following seven areas to start to get to know yourself: your aptitude, your skills, your personality, the level of responsibility you feel comfortable with, your interests and your needs. Ask (62)_____ if you have any special talents and if you need to consider your physical health when choosing a job. Be as honest and realistic as you can, and ask for other people's (63)_____ if necessary. Make a list of these things. It is usually a good idea to talk about your aptitudes with teachers, family and friends. If you are considering a career that (64)_____ a special talent, such as art, acrobatics, mathematics or music, discuss your aptitudes with (65)_____ expert in that area and discover how they fit the needs of the occupation. 56. A. strong B. strength C. strengthen D. strengthened 57. A. position B. location C. spot D. room BY ĐỖ BÌNH – THPT LIẾN SƠN, LẬP THẠCH, VĨNH PHÚC www.violet.vn/quocbinh72 58. A. upon B. in C. at D. for 59. A. meeting B. taking C. making D. interviewing 60. A. use B. make C. lose D. spend 61. A. success B. successful C. successfully D. succeed 62. A. you B. your C. yours D. yourself 63. A. interests B. fields C. opinions D. attendances 64. A. requires B. asks C. tells D. urges 65. A. a B. an C. the D. this III. Read the following text and choose the best answer for the questions below. Write your answer in the space provided. (10 pts) May 7th 1840 was the birthday of one of the most famous Russian composers of the nineteenth century: Peter Ilyich Tchaikovsky, the son of a mining inspector. Tchaikovsky studied music as a child and later studied composition at the St. Petersburg Conservatory. His greatest period of productivity occurred between 1876 and 1890, during which time he enjoyed patronage of Madame von Meck, a woman he never met, who gave him a yearly living stipend. Madame von Meck later terminated her friendship with Tchaikovsky, as well as his living allowance, when she, herself, was facing financial difficulties. It was during the time of Madame von Meck’s patronage, however, that Tchaikovsky created the music for which he is most famous, including the music for the ballets of “Swan Lake” and “The Sleeping Beauty”. Tchaikovsky’s music, well-known for its rich melodic and sometimes melancholy passages, was one of the first that brought serious dramatic music to dance. Before this, little attention had been given to the music behind the dance. Tchaikovsky died ostensibly of cholera on November 6th 1893, though there are now some scholars who argue that he committed suicide. 66. The best title for this passage could be ______. A. “The Life and Music of Tchaikovsky” B. “Development of Tchaikovsky’s Music for Ballets” C. “Tchaikovsky’s Relationship with Madame von Meck” D. “The Cause of Tchaikovsky’s Death” 67. According to the passage, all of the following describe Madame von Meck EXCEPT ______. A. she had economic troubles B. she was generous C. she was never introduced to Tchaikovsky D. she enjoyed Tchaikovsky’s music 68. According to the passage, Tchaikovsky’s music is most well-known for ______. A. it’s repetitive and monotonous tones B. the ballet-like quality of music C. the richness and melodic drama of the music D. its lively, capricious melodies 69. Which of the following is NOT mentioned in the passage? A. Tchaikovsky’s influence on ballet music. B. Tchaikovsky’s unhappiness leading to suicide. C. The patronage of Madame von Meck. D. Tchaikovsky’s productivity in composing. 70. It can be inferred from the passage that ______. A. it was not the music behind the dance that made Tchaikovsky famous B. there is suspicion on the cause of Tchaikovsky’s death C. Madame von Meck was one of the most famous Russian composers D. Madame von Meck was one of Tchaikovsky’s girlfriends SECTION D: WRITING I. Finish the second sentence in such a way that it means the same as the sentence printed before it. (15 pts) 71. It is extremely difficult for us to make ends meet these days. We find .............................................................................................................................................................. 72. Alice and Charles did not decide to move to a bigger house until after the birth of their second child. Only when ........................................................................................................................................................... 73. While mending the road, they accidentally blocked our water pipes. They accidentally cut off ..................................................................................................................................... 74. He brought the umbrella along but it didn't rain. He needn't ......................................................................................................................................................... 75. While I strongly disapprove of your behaviour, I will help you this time. Despite ............................................................................................................................................................... II. Use the word given in brackets and make any necessary additions to write a new sentence in such a way that it is as similar as possible in meaning to the original sentence. Do NOT change the form of the given word(s). (10 pts) 76. When I was driving, I realized that the car wasn’t working properly. (WRONG) 77. Sandra said that she was willing to work late. (MIND) 78. They pretended to be enjoying themselves, but they weren’t really. (AS) 79. I can't lift this table on my own. (UNLESS) BY ĐỖ BÌNH – THPT LIẾN SƠN, LẬP THẠCH, VĨNH PHÚC www.violet.vn/quocbinh72 80. The coins are believed to have been buried for safe-keeping. THE END (IT) SỞ GD&ĐT VĨNH PHÚC ———————— ĐỀ CHÍNH THỨC KỲ THI CHỌN HSG LỚP 1…. - THPT NĂM HỌC 201… - 201… (ĐỀ LUYỆN 06) ĐỀ THI MÔN: TIẾNG ANH ———————————— B. PHONETICS Choose the word whose underlined part is pronounced differently from that of the other three(5 pts) B. contact C. background D. formality 1. A. socialize 2. A. prolong B. contact C. conscious D. common B. Japanese C. partner D. Canada 3. A. African 4. A. signal B. sign C. colleague D. regard 5. A. cloth B. trustworthy C. clothing D. brother C. LEXICAL - GRAMMAR I. Choose from the four options given marked A, B, C and D one best answer to complete each sentence. Identify your answer by writing the corresponding letter A, B, C or D on your answer sheet. (15 pts) 1. Please_______________ favour. A. do me a B. make me a C. do my D. make my 2. Mrs. Edwards…______________ history for 35 years, and is retiring soon. A. teaches B. is teaching C. taught D. has been teaching 3. There have been protests from animal right groups about ____________ on animals. A. experience B. experiments C. expiration D. trials 4. Faraday’s accomplishments seem more wonderful when we realize that he had very___________ education. A. a few B. few C. a little D. little 5. A: “What are the office hours where you work?” B: “_______., everyone comes in at eight and leaves at five.” A. General B. In generally C. In the general D. In general 6. Lots of people________________ Yoga to relax. A. practice B. take up C. give up D. take in 7. It isn’t quite________.. that he will turn up at the meeting. A. certain B. exact C. right D. sure 8. After waiting for an hour he realized that the bus was__________________ to come. A. improbable B. impossible C. uncertain D. unlikely 9. _______________ being tired, I went for a picnic with my family. A. That B. Though C. Since D. Despite 10. You_________________ have told that joke. Everybody has heard it too often. A. can’t B. mustn’t C. shouldn’t D. needn’t 11. At around two years of age, many children regularly produce sentences_____ three or four words. A . are containing B. containing C. contain D. contains 12. The first plant-like organisms probably ___________ in the sea, perhaps three billion years ago. A . life B. living C. lived D. it was living 13. These tennis courts don't ____________ very often. Not many people want to play. A . got used B. used C. get used D. get use 14. I would rather ___________ nothing than __________ that book. A . do - to read B. do - read C. doing - reading D. done - read 15. Do you want _______ with you or do you want to go alone? A . me coming B. me to come C. that I come D. that I will come II. The four underlined parts of the sentence are marked A, B, C or D. You are to identify the one underlined word or phrase that would not be acceptable standard written English. Write your answers on your answer sheet. (20 pts) 1. Draft horses are the tallest, most powerful and heavy group of horses. A B C D 2. Acting teacher Stella Adler played a vital role in the develop of the Method Schooling of Acting. A B C D 3. Before diamonds can be used as jewels, they must be cut and polish. A B C D 4. Of all mammals, dolphins are undoubtedly between the friendliest to humans. A B C D BY ĐỖ BÌNH – THPT LIẾN SƠN, LẬP THẠCH, VĨNH PHÚC www.violet.vn/quocbinh72 5. Working all night long, Jim felt extreme hungry and just wanted to have a sandwich. A B C D 6. Harmony, melody and rhythm are important elements in mostly forms of music. A B C D 7. Medical students must learn both the theory and the practice of medicines. A B C D 8. Archeological sites sometimes are revealed when the construction of roads and buildings. A B C D 9. Yolanda ran over to me and breathless informed the failure of our team. A B C D 10. All root vegetables grow underground, and not all vegetables that grow underground are roots. A B C D D. READING I. Read the following passage and fill an appropriate word in each blank. (10 pts) Fish live almost (1) ___ They are found in the near freezing waters of the Artic and in the steaming (2)____ in the tropic jungles. They live in roaring streams and in quiet (3)_____ rivers. Some fish make long journeys (4) ____ the ocean. Others spend most of their life buried in sand on the ocean (5) ______. Most fish can’t (6)_____ water; yet some survive for months in dried-up riverbeds. Fish have enormous importance (7)____ man. They provide food for millions of people. Fishermen catch them for sports, and many people (8)____ them as pets. Fish are also important in the balance of nature. They eat plants and animals and in turn, become (9)____ for plants and animals. Fish thus help keep in (10 ) _____ the total number of plants and animals on the earth. II. Choose the most suitable answer from the four options marked A, B, C and D to complete each of the numbered gaps in the passage below. Write your answer on the answer sheet. (10 pts) You've (1)________ heard someone say that the reason you caught a cold was that you were 'run down'. People generally accept that if you are (2)______ pressure or run down you are more likely to get ill. But is this really true (3)_______ is it just an old wives' tale? The problem with old sayings like this is that it's often difficult to (4)______ whether they are true or not. For example, if you ask people with colds whether they are feeling run down, they are (5)_______ certain to say yes. People without colds are more likely to say no. So, how do you (6)______ if it's the cold that is making them (7)_____ run down or the fact that they are run down that is making them more likely to catch a cold? Now it seems there is a scientific answer to this question. And answer is yes - if you are run down or under stress, you are more likely to catch a cold. Scientists at the Common Cold Research Centre ran a series of tests. They got volunteers without colds to attend the centre, where they were first given a questionnaire to complete. The questions measured the (8)_____ of stress each volunteer was under. The volunteers were then deliberately infected with a cold virus and left to see whether they developed a cold. After many people had been tested, the researchers looked for a (9)____ between the measured stress level and the chances of catching a cold. They found that the higher the stress, the more likely the person was to catch a cold. At the moment it is not known why stress makes someone more likely to become ill, but now (10)_____ the link has been found, researchers might be able to investigate further. They may even find different ways of combating disease. 1. A. perhaps B. maybe C. possibly D. probably 2. A. under B. behind C. at D. above 3. A. and B. or C. but D. so 4. A. show B. express C. state D. prove 5. A. nearly B. mostly C. almost D. surely 6. A. tell B. speak C. say D. suggest 7. A. be B. feel C. become D. get 8. A. amount B. number C. mass D. heap 9. A. combination B. joint C. link D. relation 10. A. that B. when C. because D. as III. Read the passage below and choose the best answer from the four options marked A, B, C or D. Identify your answer by writing the corresponding letter A, B, C or D on the answer sheet. (20 pts) As many as one thousand years ago in the Southwest, the Hopi and Zuni Indians of North America were building with adobe-sun-baked brick plastered with mud. Their homes looked remarkably like modern apartment houses. Some were four stories high and contained quarters for perhaps a thousand people, along with storerooms for grain and other goods. These buildings were usually put up against cliffs, both to make construction easier and for defense against enemies. They were really villages in themselves, as later Spanish explorers must have realized since they called them “pueblos”, which is Spanish for towns. The people of the pueblos raised what are called “the three sisters”- corn, beans, BY ĐỖ BÌNH – THPT LIẾN SƠN, LẬP THẠCH, VĨNH PHÚC www.violet.vn/quocbinh72 and squash. They made excellent pottery and wove marvelous baskets, some so fine that they could hold water. The Southwest has always been a dry country, where water is scarce. The Hopi and Zuni brought water from streams to their fields and gardens through irrigation ditches. Water was so important that it played a major role in their religion. They developed elaborate ceremonies and religious rituals to bring rain. The way of life of less – settled groups was simpler and more strongly influenced by nature. Small tribes such as the Shoshone and Ute wandered the dry and mountainous lands between the Rocky Mountains and the Pacific Ocean. They gathered seeds and hunted small animals such as rabbits and snakes. In the Far North the ancestors of today’s Inuit hunted seals, walruses, and the great whales. They lived right on the frozen seas in shelters called igloos built of blocks of packed snow. When summer came, they fished for salmon and hunted the lordly caribou. The Cheyenne, Pawnee and Sioux tribes, known as the Plains Indians, lived on the grasslands between the Rocky Mountains and the Mississippi River. They hunted bison, commonly called the buffalo. Its meat was the chief food of these tribes, and its hide was used to make their clothing and the covering of their tents and tips. 1. What does the passage mainly discuss? A. The architecture of early America Indian buildings B. The movement of American Indians across North America C. Ceremonies and rituals of American Indians D. The way of life of American Indian tribes in early North America 2. According to the passage, the Hopi and Zuni typically built their homes ________. A. in valleys B. next to streams C. on open plains D. against cliffs 3. The word “They” refers to ____________. A. goods B. buildings C. cliffs D. enemies 4. It can be inferred from the passage that the dwellings of the Hopi and Zuni were _______. A. very small B. highly advanced C. difficult to defend D. quickly constructed 5. The author uses the phrase “the three sisters” refers to_____________. A. Hopi women B. family members C. important crops D. rain ceremonies 6. Which of the following is true of the Shoshone and Ute? A. They were not as settled as the Hopi and Zuni B. They hunted caribou. C. They built their homes with adobe. D. They did not have many religious ceremonies. 7. According to the passage, which of the following tribes lived in the grasslands? A. The Shoshone and Ute B. The Cheyenne C. The Hopi and Zuni D. The Pawnee and Inuit 8. Which of the following animals was the most important to the Plains Indians? A. The salmon B. The caribou C. The seal D. The buffalo 9. The author gives an explanation for all of the following words EXCEPT______. A. adobe B. pueblos C. caribou D. bison 10. The author groups North American Indians according to their _________. A. tribes and geographical regions B. arts and crafts C. rituals and ceremonies D. date of appearance on the continent E. WRITING Finish each of the following sentences so that it means the same as the printed one. (20 pts) 1. I didn’t arrive in time to see her. → I wasn’t early______________________________________________________________. 2. We couldn’t drive because of the fog. → The fog prevented__________________________________________________________ 3. A train leaves at eight o’ clock every morning. → There is ______________________________________________________________. 4. John asked if it was the blue one or the green she wanted. → “Which _____________________________________ ?” 5. I haven’t eaten this kind of food before. → This is the first_________________________________________________________. 6. Only if you work hard now have you any chance of success. → Your chance____________________ 7. He tried very hard to give up smoking. → He made great_____________________________. 8. An aerial is not required with this radio. → You don’t_____________________________ 9. When he stops smoking, he’ll feel better. → The sooner__________________________. 10. The wedding was held despite the rain. BY ĐỖ BÌNH – THPT LIẾN SƠN, LẬP THẠCH, VĨNH PHÚC www.violet.vn/quocbinh72 → The wedding took____________ __________ THE END ___________ SỞ GD&ĐT VĨNH PHÚC ----------------ĐỀ CHÍNH THỨC KỲ THI CHỌN HSG LỚP 1…. THPT NĂM HỌC 201…-201…. (ĐỀ SỐ 07) ĐỀ THI MÔN: TIẾNG ANH PART II. GRAMMAR AND VOCABULARY I. Complete the following sentences by choosing the correct answer among four options (A, B, C or D). (15 pts) 1. He's really shy _______ girl. A. by B. at C. for D. with 2. The teacher _______ her to improve her drawing. A. insisted B. encouraged C. made D. persisted 3. I couldn't quite ______ what they were doing because they were so far away. A. bear out B. make out C. think out D. try out 4. The meal Mary cooked tastes_______. A. well B. nice C. good D. worse 5. ______ at the party, we saw Ruth standing alone. A. Arriving B. We arrived C. Arrived D. We were arriving 6. The people who______ the survey said that they had examined over 1,000 accidents. A. gave B. proceed C. set D. conducted 7. The judge found him ______ of stealing and sent him to prison. A. evil B. innocent C. guilty D. wicked 8. The house we have rented is______. So we will have to buy some beds, chairs, tables, etc. A. unrestored B. unrepaired C. unfurnished D. undecorated 9. He was turned down for the job because he is ________. A. qualified B. qualifying C. unqualified D. qualification 10. The trouble started only______ the other man came into the room. A. when B. until C. and then D. too soon 11. _______, the disaster would not have happened. A. Had you have obeyed the orders B. You had obeyed the orders C. You obeyed the orders D. Had you obeyed the orders 12. _______ had booked in advance were allowed in. A. Only who B. Only those who C. Only who were those D. Only were those who 13. Traveling alone to a jungle is adventurous, ________. A. if not impossible B. if it not impossible C. when not impossible D. when it not impossible 14. I ______ the hot weather in the south. A. use to B. used to C. am use to D. am used to 15. The meat looked very _______ to the dog. A. invited B. invite C. inviting D. invitingly II. Use the correct form of each word on the right to complete the numbered spaces provided in the passage. Write your answers on your answer sheet. (10 pts) The mysteries of the skies Three hundred and fifty years before the first men looked down on the amazingly beautiful surface of the moon from close quarters, Galileo’s newly built telescope (1) _____ him to look at 1. ABLE the edge of the hitherto mysterious sphere. He saw that the apparently (2) _____ surface was not 2. LIVE divinely smooth and round, but bumpy and imperfect. He realized that although the moon might 3. ACT appear (3) ___, resembling a still life painted by the hand of a cosmic (4) _____, it was a real 4. ART world, perhaps not very different from our own. This amounted to a great (5) ____ hardly to be 5. ACHIEVE expected in his day and age, although nowadays his (6) ___ may appear to some to be trivial and 6. (7) ______. CONCLUDE Not long after Galileo lunar’s observations, the skies which had previously been so (8) ______ 7. SIGNIFY revealed more of their extraordinary mysteries. Casting around for further wonders, Galileo 8. ELUDE focused his lens on the (9) _____ planet of Jupiter. Nestling next to it, he saw four little points of light circling the distant planet. Our moon it appeared, perhaps (10) _____ in the eyes of those 9. STRIKE fearful of what the discovery might mean, was not alone! 10. FORTUNE BY ĐỖ BÌNH – THPT LIẾN SƠN, LẬP THẠCH, VĨNH PHÚC www.violet.vn/quocbinh72 III. In the following passage, some numbered lines contain a word that shouldn’t be there. Tick (√) the sentences that are correct and write the words that shouldn’t be there in the numbered space. (10 pts) KEEPING YOUR DISTANCE Personal space is a term that refers to the distance we like to keep between 0 ___√___ ourselves and other people. When someone we do not know well gets too close 00 someone that we usually begin to feel uncomfortable. If such a business colleague comes 1________ closer than 1.2 meters, the most common response is to move away. Some 2 ________ interesting studies have been done in libraries. If strangers will come too close, 3 ________ many people get up and leave the building, others use to different methods such as 4 ________ turning their back on the intruder. Living in cities has made people to develop new 5 ________ skills for dealing with situations where they are very close to strangers. Most 6 ________ people on so crowded trains try not to look at strangers; they avoid skin contract, 7 ________ and apologize if hands touch by a mistake. People use newspapers as a barrier 8 ________ between themselves and other people, and if they do not have one, they stare into 9 ________ the distance, making sure they are not looking into anyone’s eyes. 10 _______ PART III. READING I. Complete the following passage by choosing A, B, C or D to fill in each blank. (10 pts) In recent years, there has been a remarkable increase into happiness. The researchers have come up a number of factors which contribute to a definition of happiness. First of all, there is, in some people, a moderate genetic predisposition to be happy, in other words, happiness (1)_______ in families. And happiness seems to correlate quite strongly with the main dimensions of personalities: extroverts are generally happier, neurotics are less so. Second, people often report good social relations as a reason for their happiness. In particular, friends are a great (2) ______ of joy, partly because of the agreeable things they do together, partly because of the way friends use positive non-verbal (3) ______ such as caressing and touching, to affirm their friendship. Marriage and similar (4) ______ relationships can also form the basis of lasting happiness. Third, job satisfaction undoubtedly (5) ______ overall satisfaction, and vice versa - perhaps this is why some people are happy in boring jobs: it (6) ______ both ways. Job satisfaction is caused not only by the essential nature of the work, but (7)_____ by social interactions with co-workers. Unemployment, on the contrary, can be a serious cause of unhappiness. Fourth, leisure is important because it is more under individual (8) ______ than most other causes of happiness. Activities (9) _____ sport and music, and participation in voluntary work and social clubs of various kinds, can give great joy. This is partly because of the (10) ______themselves, but also because of the social support of other group members – it is very strong in the case of religious groups. 1. A. runs B. arrives C. goes D. descends 2. A. source B. origin C. base D. meaning 3. A. movements B. signals C. slogans D. motions 4. A. near B. tight C. close D. heavy 5. A. consists of B. applies to C. counts on D. contributes to 6. A. works B. effects C. makes D. turns 7. A. too B. as well C. also D. plus 8. A. check B. power C. choice D. control 9. A. so B. such C. like D. thus 10. A. facilities B. activities C. exercises D. amenities II. Read the passage carefully, then choose the correct option (marked A, B, C or D) to answer the questions. (10 pts) Scientists have established that influenza viruses taken from man can cause disease in animals. In addition, man can catch the disease from animals. In fact, a greater numbers of wild birds seem to carry the virus without showing any evidences of illness. Some scientists conclude that a large family of influenza virus may have evolved in the bird kingdom, a group that has been on earth 100 million years and is able to carry the virus without contracting the disease. There is even convincing evidence to show that virus strain are transmitted from place to place and from continent to continent by migrating birds. It is known that two influenza viruses can recombine when both are present in an animal at the same time. The result of such recombination is a great variety of strains containing different H and N spikes. This raises the possibility that a human influenza virus can recombine with an influenza virus from a lower animal to produce an entirely new spike. Research is underway to determine if that is the way major new strains come into being. Another possibility is that two animal influenza strains may recombine in a pig, for example, to produce a new strain which is transmitted to man. 1. According to the passage, scientists have discovered that influenza viruses ______. BY ĐỖ BÌNH – THPT LIẾN SƠN, LẬP THẠCH, VĨNH PHÚC www.violet.vn/quocbinh72 A. cause ill health in wild animals B. do not always cause symptoms in birds C. are rarely present in wild birds D. change when transmitted from animals to man 2. What is known about the influenza virus? A. It was first found in a group of very old birds. B. All the different strains can be found in wild birds. C. It existed over 100 million years ago. D. It can survive in many different places. 3. According to the passage, a great variety of influenza strains can appear when______. A. H and N spikes are produced B. animal and bird viruses are combined C. dissimilar types of viruses recombine D. two viruses of the same type are contracted 4. New strains of viruses are transmitted to man by_______. A. a type of wild pig B. diseased lower animals C. a group of migrating birds D. a variety of means 5. It can be inferred from the passage that all of the following are ways of producing new strains of influenza EXCEPT___. A. two influenza viruses in the same animal recombining B. animal viruses recombining with human viruses C. two animal viruses recombining D. two animal viruses recombining in a human III. Read the passage and choose the best answer from the four options marked A, B, C or D in the following questions. Identify your answer by writing the corresponding letter A, B, C or D on your answer sheet. (10 pts) Several hundred million years ago, plants similar to modern ferns covered vast stretches of the land. Some were as large as trees, with giant fronds bunched at the top of trunks as straight as pillars. Others were the size of bushes and formed thickets of undergrowth. Still others lived in the shade of giant club mosses and horsetails along the edges of swampy lagoons where giant amphibians swam. A great number of these plants were true ferns, reproducing themselves without fruits or seeds. Others had only the appearance of ferns. Their leaves had organs of sexual reproduction and produced seeds. Although their “flowers” did not have corollas, these false ferns (today completely extinct) ushered in the era of flowering plants. Traces of these floras of the earliest times have been preserved in the form of fossils. Such traces are most commonly found in shale and sandstone rocks wedged between coal beds. Today only tropical forests bear living proof of the ancient greatness of ferns. The species that grow there are no longer those of the Carboniferous period, but their variety and vast numbers, and the great size of some, remind us of the time when ferns ruled the plant kingdom. 1. What does the passage mainly discuss? A. Plant reproduction B. How to locate fossils C. An ancient form of plant life D. Tropical plant life 2. The word “Others” refers to _________. A. plants B. pillars C. trees D. fronds 3. Which of the following is NOT mentioned as a characteristic of the plants described in the passage? A. They once spread over large areas of land. B. They varied greatly in size. C. They coexisted with amphibians, mosses, and horsetails. D. They clung to tree trunks and bushes for support. 4. The word “true” is closest in meaning to which of the following? A. accurate B. genuine C. straight D. dependable 5. The author states that fossils of early plant life are usually found in rocks located between deposits of _______. A. coal B. shale C. sandstone D. corollas IV. Read the passage carefully then fill in the blank a suitable word. (15 pts) As swimming became a popular recreation in England during the 1860s and 1870s, several (1) ______ sports developed, roughly patterned after land sports. (2) ______ them were water football (or soccer), water rugby, water handball, and water polo, in which players rode on floating barrels, painted to look (3) ______ horses, and struck the ball with a stick. Water rugby became most popular of these sports, but somehow the water polo name became attached to it, and it's been attached (4) ______ since. As played in England, the object of the sport was for a player to touch the ball, with both (5) ______, at the goal end of the pool. The goaltender stood on the pool deck, ready to dive on any opponent who was about to score. Water polo quickly became a very rough sport, filled (6) ______ underwater fights away from the ball, and it wasn't unusual for players to pass out for lack of air. In 1877, the sport was tamed in Scotland by the addiction of goalposts. The Scots also replaced (7) ______ original small, hard rubber ball with a soccer ball and adopted (8) ______ that prohibited taking the ball under the surface or, "tackling" a player unless he had the ball. BY ĐỖ BÌNH – THPT LIẾN SƠN, LẬP THẠCH, VĨNH PHÚC www.violet.vn/quocbinh72 The Scottish game, which emphasized swimming speed, passing, and (9) ______ work, spread to England during the early 1880s, to Hungary in 1889, to Austria and Germany in 1894, to France in 1895, and (10) ______ Belgium in 1900. Water polo was the first team sport added to the Olympic program, in 1900. PART IV: WRITING I. Write the new sentences using the given word. Do not change the word given in any way. (10 pts) 1. They have discovered some interesting new information. (LIGHT) 2. They suspended Jack for the next two matches. (BANNED) 3. I really want to see her again. (DYING) 4. She was so beautiful that I couldn't stop looking at her. (EYES) 5. We are looking forward to watching the program. (WAIT) II. Rewrite each of the following sentences so that it has a similar meaning to the original one. (10 pts) 1. If you changed your mind, you would be welcomed to join our class. → Were you______________________________________________________ 2. I'd rather not go out this afternoon. → I do not feel____________________________________________________ 3. Adeles tries hard, but she doesn't get anywhere. → However______________________________________________________ 4. It is thought that the boss is considering raising wages. → The boss______________________________________________________ 5. His disabilities did not prevent him from sailing around the world. → Despite the fact_________________________________________________ 6. I didn't arrive in time to see her. → I wasn't_______________________________________________________ 7. I'd prefer you not to smoke. → I'd rather______________________________________________________ 8. The mother smiled happily. She took the baby in her arms. → Smiling________________________________________________________ 9. The noise next door did not stop until after midnight. → It was not ______________________________________________________ 10. You can ring this number whenever there is any difficulty. → Should _________________________________________________________ THE END SỞ GD&ĐT VĨNH PHÚC ———————— ĐỀ CHÍNH THỨC KỲ THI CHỌN HSG LỚP 1…. - THPT NĂM HỌC 201… - 201… (ĐỀ LUYỆN 08) ĐỀ THI MÔN: TIẾNG ANH ———————————— Question 1: a- Choose the word whose underlined part is pronounced differently from that of the others: (5 pts) 1. A. health B. bread C. appear D. heavy B. since C. storm D. symptom 2. A. sugar 3. A. arrived B. experienced C. failed D. discovered 4. A. pronounce B. round C. sound D. pour 5. A. walls B. rewards C. sticks D. spectators b- Choose the words with the different stress pattern from the others: (5 pts) 1. A. basketball B. wonderful C. education D. trustworthy 2. A. surround B. restrict C. remove D. manual 3. A. planet B. affect C. annoy D. excuse 4. A. famous B. climate C. language D. attend 5. A. extensive B. decorate C. distinguish D. acquaintance Question 2: Put the verbs given in brackets into their appropriate tense or form; (10 pts) 1. It is imperative that the letter (send)_____ at once. 2. By this time tomorrow, we (travel)_____ to Nha Trang. 3. Up to now , we (complete)_____ four tests. BY ĐỖ BÌNH – THPT LIẾN SƠN, LẬP THẠCH, VĨNH PHÚC www.violet.vn/quocbinh72 4. The students (punish)_____ yesterday is my brother . 5. I think the play (perform)_____ now. 6. I didn’t do the test well. I (prepare)____ it very carefully at home. 7. It has been suggested that the government (assist) ____the poor in improving their living condition. 8. By the time he____ (become) heavy weight boxing champion, he …(win) over thirty fights. 9. He looked frightened as if he _____ (see) a ghost. Question 3: Supply the correct forms of the words in the capital letters (10 pts) 1. Their ____has lasted a life time. 2. Everyone has a number of ____but none has many true friends. 3. Cats are supposed to have nice ____. 4. The evening was ____spent playing and talking . 5. The cost of ____must be paid by the buyer . 6. After his illness , he started worrying that he was ____. 7. He lives in an attractive____ part of Sydney 8. We must learn about keeping the environment ____. 9. He finds it difficult to accept ……from others. 10. The film is entertaining but full of historical____. FRIEND ACCQUAINT LIVE ENJOY CARRY WEIGH RESIDE POLLUTE CRITICISE ACCURATE Question 4: fill in the blank with a suitable preposition (10 pts) 1. He has been ill _____ flu for a week . 2. I’m _____ any having more meetings. 3. She will remain here _____ the rest of the day. 4. A good friend always stand for you when you are _____ trouble. 5. Jim managed to climb into the house _____ means ____ a ladder he found. 6. I’d like to thank you, __________ behalf _____ everyone who was rescued . 7. Julia has nothing ____ common _____ Bill, they are quite different. Question 5 :Fill in each numbered blank one missing word. (10 pts) Speech is one of the most important __1__ of communicating. It consists of far more than just making noises. To talk and also to be __2__ by other people, we have to speak a language, that is , we have to use combinations of __3__ that everyone agrees stand for particular object or idea. Communication would be impossible if everyone made up their own language. Learning a language properly is very __4__ The basic __5__ of English is not very large, and only about 2000 words are needed to speak it quite __6__ But the more words you know, the more idea you can__7__ and the more precise you can be about their exact meaning. Words are the __8__ thing we use in communicating what we want to say. The way we __9__ the words is also very important. Our tone of voice can express many emotions and __10__ whether we are pleased or angry, for instance. Question 6: Choose the most suitable word given for each space in the text. (15 pts) Have you ever asked yourself what you are working for? If you have ever had the time to (1)____ this taboo question, or put it to others in moment of weakness or confidentiality, you (2)____ well have heard some or all of the (3)____ It’s the money of course, some say with a smile, as if explaining something to a small child. Or it’s the satisfaction of (4)____ well done, the sense of achievement behind the clinching of an important (5)____. I worked as a bus conductor once, and I can’t say I (6)____ the same as I staggered along the swaying gangway trying to (7)____ out tickets without falling over into someone’s lap. It’s the company of other people perhaps, but if that is the (8)____ , what about farmers? it is the conservation in the farmyard that keeps them captivated by the job? Work is power and a sense of status say those (9)____ have either attained these elusive goals, or feel aggrieved that nobody has yet recognized their leadership (10)____ Or we can blame it all on someone else, the family or the taxman. I suspect, and say this under my (11)_____., that most of us work rather as Mr. Micawber lived, hoping for something to (12)___ up. We’ll win the pools, and tell the boss what we really think. We’ll scrape together the (13)____ and open that little shop we always dreamed of, or go (14)____ the world, or spend more time in the garden. Once day we’ll get that (15)____ we deserve, but until then at least we have something to do. And we are so busy doing it that we won’t have time to wonder why. 1. A. propose B. meditate C. consider D. launch 2. A. might B. can C. will D. should 3. A. below B. rest C. following D. latter 4. A. a work B. a job C. a task D. an effort BY ĐỖ BÌNH – THPT LIẾN SƠN, LẬP THẠCH, VĨNH PHÚC www.violet.vn/quocbinh72 5. A. deal B. position C. job D. engagement 6. A. enjoyed B. wished C. hoped D. felt 7. A. make B. turn C. issue D. give 8. A. one B. case C. question D. former 9. A. people B. must C. who D. to 10. A. qualities B. status C. property D. requirements 11. A. oath B. suspicion C. breath D. pressure 12. A. move B. turn C. ease D. end 13. A. resources B. opportunities C. rest D. money 14. A. round B. over C. into D. to 15. A. ambition B. station C. vocation D. promotion Question 7: Fill each space with a suitable phrase from the list (a-k) below. (10 pts) (The first is done for you: 1-b) Linda: I want to go to a sports club. Black’s is good __1__ come too? __2__ sports? Julia: Oh yes, __3__ squash, for example. But really, __4__ sports which you can do outdoors. What about you, __5__ outdoor sports? Linda: I__6__ most sports, but __7__ tennis. Julia: What sports can we do at Black’s ? Linda: They offer a good range. And there is a swimming pool .But it’s rather far away. Julia: What about Forest’s? That’s nearer. And they have lots of tennis courts. Linda: __8__ go to a club with a swimming pool. Julia : But we’d have to catch a bus to Black’s. Kida: Well, __9__? Julia: (10)_____.You decide. Linda: O K, __10__ to go to Black’s . But _11_ to pay your bus fare! a. do you like b. would you like to c. what do you want to do d. Do you like e. I prefer f. I like g. my favorite is h. I’d rather i. I’m not sure j. enjoy k. I’d prefer Question 8: Read the passage carefully then choose the best answer (A, B, C or D). (5 pts) During the last 400 years, most scientists have relied on mathematics for the development of their inventions or discoveries. However, one great British scientist, Michael Faraday, did not make use of mathematics. Faraday, the son of a poor blacksmith, was born in London in 1791 and had no education beyond reading and writing. In 1812, Faraday was hired as a bottle washer by the great chemist Humphrey Davy. Later, Faraday became a greater scientist than Davy, making the last years of Davy’s life embittered with jealousy. Faraday made the first electric motor in 1821, a device that used electricity to produce movement. Then Faraday became interested in the relationship between electricity and magnetism. In 1831, he discovered that when a magnet is moved near a wire, electricity flows in the wire. With this discovery, he produced a machine for making electricity called dynamo. Faraday then went on to show how electricity affects chemical substances. Because Faraday believed that money should be given to the poor, when he grew old, he was very poor. However, Queen Victoria rewarded him for his discoveries by giving him a stipend and a house. He died in 1867. 1. Which of the following statements is not true about Faraday? A. He didn’t make use of mathematics to develop his inventions B. he just knew how to read and write. C. He was born in a rich family. D. His father was a blacksmith. 2. Humphrey Davy hired Faraday to work as a _________. A. chemist B. bottle washer C. scientist D. professor 3. All of the following are mentioned as Faraday’s achievements except _____. A. his invention of dynamo B. his discovery of the effects of electricity on chemical substances C. his invention of electric motor in 1821. D. his discovery of magnetism 4. Faraday got a stipend and a house as a reward from_____. A. Humphrey Davy B. his father C. Queen Victoria D. his friend 5. Faraday died in _________. D. 1821 A. 1867 B. 1831 C. 1812 Question 9: Sentence transformation a. Finish each of the following sentences in such away that it means exactly the same as the sentence printed before it. (5 pts) 1. I assumed that she would learn how to take shorthand after this course. I took it _____.. 2. When the Minister was asked about the strike, he decline to comment. On ..... 3. Someone rang the alarm as soon as the burglars left the building. No sooner _____. 4. We regret to inform you that your application has not been successful. Much to _____ BY ĐỖ BÌNH – THPT LIẾN SƠN, LẬP THẠCH, VĨNH PHÚC www.violet.vn/quocbinh72 5. I left without saying goodbye as I didn’t want to disturb the meeting. Rather _____. b. For each of the sentences below, write a new sentence as similar as possible in meaning to the original sentence, using the words given in block letters. The words must not be changed in any way. (5 pts) 1. Jenny didn’t feel like going to the party. MOOD 2. Did Pamela say why she was so late? REASON 3. The firm is going to raise everybody’s salary. GIVEN 4. This contract is not binding until we both sign it BOUND 5. As far as I know he is still working in Bristol. KNOWLEDGE Question 10: a. Change the direct speech in these sentences into reported speech. (5 pts) 1. “Why don’t you go and push a baby-carriage?” the taxi-driver said to the other driver angrily. “You’re not fit to drive a car.” 2. Harry said to his wife, “ I’m gong to see my mother this evening if I can get away from the office a little earlier. Have you any massages for her?” 3. “Let me help you with that suitcase,” Timothy said to the pretty girl at the station. “It looks a lot too heavy for you”. 4. “If you really think I said that about you,” said Charles, “I’m not surprised that you’re angry with me. But I assure you I did not.” 5. “We’d better bring some warmer clothes with us next time we come here,” said my father. “It gets a lot colder in winter than I realized.” b. Rearrange the order of words in these sentences in such a way that each of them becomes a correctly expressed question. (5 pts) 1. lottery/buy /if /what /you /would /a /you /won/? 2. light /please /off /won’t /turn /the /you /the hall /in/? 3. this /holiday /our /summer /for /we /go /shall /where/? 4. one /that /I /shall /dress /or /buy /this/? 5. milk /should /buy /how /tins /of /think /you /many /do /we/? THE END SỞ GD&ĐT VĨNH PHÚC ———————— ĐỀ CHÍNH THỨC KỲ THI CHỌN HSG LỚP 1…. - THPT NĂM HỌC 201… - 201… (ĐỀ LUYỆN 09) ĐỀ THI MÔN: TIẾNG ANH ———————————— PART B: VOCABULARY AND GRAMMAR I. Complete each of the following sentences with the correct answer (A, B, C or D). (15 pts) 1. The traffic problem has improved _____, out of the blue, really. A. gradually B. factually C. unexpectedly D. respectably 2. This picture book, the few pages _____ are missing, is my favorite. A. for which B. of that C. to which D. of which 3. It was felt that he lacked the _______ to pursue a difficult task to very end. A. persuasion B. commitment C. engagement D. obligation 4. Your decision will ______ a great strain on our relationship. A. impose B. propose C. expose D. suppose 5. We shouldn’t give the children everything they ask for; they will become completely _____. A. spoilt B. wounded C. damaged D. destroyed 6. The completion of the tunnel has been _______ owing to a strike. A. held up B. held off C. held on D. held over 7. It is with ________ regret that we have to inform you that your scholarship has been withdrawn. A. heavy B. deep C. somber D. high 8. _______ of all modern domestic poultry is the red jungle fowl is widely believed. A. The ancestor B. The ancestor is C. How the ancestor D. That the ancestor 9. Art critics do not all agree on what _______ a painting great. A. qualities to make B. are the qualities for making C. qualities make D. do the qualities that make 10. Farmers supply crops with phosphorus in areas ________ have removed it from the soil. A. because of long years of cultivation B. where long years of cultivation C. with long years of cultivation D. by long years of cultivation 11. Sarah congratulated _____ passing my driving test. BY ĐỖ BÌNH – THPT LIẾN SƠN, LẬP THẠCH, VĨNH PHÚC www.violet.vn/quocbinh72 A. me B. for C. me on D. on me 12. Had it not been for the intolerable heat in the hall, they _____ much longer. A. will stay B. would stay C. would be staying D. would have stayed 13. I can’t quite _____ out what the sign says. A. read B. get C. carry D. make 14. One _____ of the scheme is the very high cost. A. advantage B. shortage C. drawback D. shortcoming 15. In a new culture, many embarrassing situations occur _____ a misunderstanding. A. because of B. of C. for D. because II. Choose a word or phrase in each of the following sentences that needs correcting. (10 pts) 1. Paris has been well-known about its famous monuments, beautiful music, and wonderful restaurants for over 100 years. A B C D 2. In France people drive on the left, so making sure you go on the right side. A B C D 3. Up to now he wrote five novels and over sixteen short stories. A B C D 4. The new bridge makes it possibly to cross the river easily and quickly. A B C D 5. If you are working with young children in a primary school, you will find that teaching lively songs and rhymes are very popular. A B C D III. Use the correct form of the word in bracket to complete each of the following sentences. (10 pts) 1. He is unhappy because of his ________. (deaf) 2. The ________ of the swamps will destroy the mosquitoes’ breeding places. (drain) 3. He has made a great _________to the development of the country. (contribute) 4. We will hire new staff when the _____________arises. (necessary) 5. His repeated __________ from school is unacceptable. (absent) 6. The teacher’s words are a great ___________ to him. (encourage) 7. They are very __________ of one another. (support) 8. Her interests are very __________. (diversity) 9. I can’t stand his __________. (rude) 10. The holiday was beyond all _________. (expect) PART C: READING I. Read the text below and think of the word which best fits each space. Use only ONE word in each space. (15 pts) Around the age of eighteen, you must make one of the biggest decisions of your life. "Do I stay on at school and hopefully go on to university (1)_____? Do I leave and start work or begin (2) ____ training course?". The decision is yours, but it may be (3)_____ remembering two things: there is more unemployment among people (4)_____ haven't been to university, and people who have the right (5)_____ will have a big advantage in the competition for jobs. If you decide to go straight into a job, there are many opportunities (6)_____ training. Getting qualifications will (7)_____ you to get on more quickly in many careers, and evening classes allow you to learn (8)_____ you earn. Starting work and taking a break to study when you are older is (9)_____ possibility. This way, you can save up money for your student days, as well as (10)____ practical work experience. II. Choose the best answer from the four options marked A, B, C or D to complete each numbered gap in the passage below. (10 pts) If you (1)_____ to be walking in your local park tomorrow and you find an abandoned book with a label inside (2)_____ 'Read and Release me', don't just treat it as a joke. You've probably come (3)_____ an example of 'bookcrossing', a book-sharing movement started in 2001 by American software developer Ron Hornbaker, whose stated aim is to 'make the (4)_____ world a library'. Book-crossers 'release' books (5)_____ by passing them on to friends, or else by leaving them in public places for others to pick up, or 'catch', and then read, before they (6)_____ turn release them back 'into the wild'. (7)_____ a book has been 'caught', the person finding it is encouraged to record the event by logging on to BY ĐỖ BÌNH – THPT LIẾN SƠN, LẬP THẠCH, VĨNH PHÚC www.violet.vn/quocbinh72 the book-crossing website and entering the book's ID number written on the label. That (8)_____ , both the original owner and subsequent readers of the book can keep track of its progress. Over half a million people worldwide participate in book-crossing, 'releasing' books in a (9)_____ range of locations including cafés, airports, bus stations, telephone boxes and even underwater in public fountains. Often a book is left in a place (10)_____ is relevant to its title or content: Agatha Christie's Murder on the Orient Express on a train, for example, or an archeology book in a museum. 1. A. occur B. happen C. pass D. arise 2. A. telling B. saying C. talking D. answering 3. A. away B. along C. across D. aside 4. A. full B. absolute C. quite D. whole 5. A. either B. but C. or D. neither 6. A. at B. to C. in D. for 7. A. Soon B. Since C. Once D. While 8. A. way B. time C. place D. manner 9. A. deep B. long C. high D. wide 10. A. what B. which C. who D. where III. Read the text then answer the questions by choosing A, B, C or D. (20 pts) To date, Canada has produced only one classic children’s tale to rank with Alice’s Adventures in wonderland and the works of Mark Twain; this was Lucy Maud Montgomery’s Anne of Green Gables. Lucy Maud Montgomery was born in Clinton, Prince Edward Island. Her mother died soon after her birth, and when her father went to Saskatchewan to assume a business position, she moved in with her grandparents in Cavendish, Prince Edward Island. There she went to school and later qualified to be a teacher. Montgomery wrote the Anne books while living in Cavendish and helping her grandmother at the post office. The first of the books, Anne of Green Gabbles, was published in 1908, and in the next three years she wrote two sequels. Like Montgomery, the heroine of the book is taken in by an elderly couple who lives in the fictional town of Avonlea, and Montgomery incorporated many events from her life in Cavendish into the Anne books. In 1911, Montgomery married Evan Macdonald and the couple soon moved to Ontario, where she wrote many other books. However, it was her first efforts that secured her prominence, and the Anne books are still read all around the world. Her novels have helped create a warm picture of Prince Edward Island’s special character. Several movies, a television series, and a musical play have been based on her tales, and today visitors scour the Island for locations described in the book. 1. The main purpose of this passage is to ____. A. show the similarities between Montgomery’s life and that of her fictional character Anne. B. contrast Canadian children’s literature with that of other countries. C. provide a brief introduction to Prince Edward Island. D. introduce Montgomery and her Anne books. 2. According to the passage, Montgomery was raised primarily____. C. by her mother D. by her father A. by her grandparents B. in an orphanage 3. Approximately when did Lucy Maud Montgomery write the two sequels to her book Anne of Green Gables? C. From 1908 to 1911 D. From 1913 to 1918 A. From 1874 to 1908 B. From 1911 to 1913 4. The word “elderly” in line 8 is closest in meaning to____. A. kindly B. old C. friendly D. sly 5. In the Anne books, the main character lives in____. A. the town of Cavendish B. the town of Avonlea C. Saskatchewan D. Ontario 6. Which of the following can be concluded from the passage about Anne books? A. They were not as successful as Montgomery’s later works. B. They were influenced by the works of Mark Twain. C. They were at least partially autobiographical. D. They were not popular until after Montgomery had died. 7. Ontario was the place where ____. A. Montgomery wrote other books B. Montgomery got married C. Montgomery wrote two sequels D. Montgomery became famous 8. The word “prominence” in line 11 is closest in meaning to____. A. excellence B. reputation C. effort D. permanence 9. Which of the following is closest in meaning to the word “character” in line 12? A. a person in a novel B. nature C. a written symbol D. location 10. All of the following have been based on the Anne books EXCEPT____. A. a television series B. movies C. a play D. a ballet PART D: WRITING BY ĐỖ BÌNH – THPT LIẾN SƠN, LẬP THẠCH, VĨNH PHÚC www.violet.vn/quocbinh72 I. Complete the second sentence so that it has the similar meaning to the first sentence. Write your answer on your answer sheet. (10 pts) 1. “You’d better not lend her any more money, Paul,” said Tom. → Tom advised Paul 2. Although the traffic was bad, Peter arrived on time. → In spite of 3. The last time I saw Mary was in 2009. → I haven’t 4. The plumber repaired the leak this morning. → They had 5. They had a bad crop because the storm swept through the region. → If ___ II: Complete the second sentence so that it has the similar meaning to the first sentence. Use the word given. Do not change the word. Write your answer on your answer sheet. (10 pts) 1. It's possible that someone stole your purse when we were in the restaurant. (might) → __________ 2. ‘Me? No, I didn’t take Sue’s calculator,’ said Bob. (denied) → __________ 3. They said that an electrical failure was the cause of the fire. (blamed) → __________ 4. She apologized to the host for having to go so early, and left. (apologies) → __________ 5. Peter was the first person I asked for advice. (whose) → __________  THE END  SỞ GD&ĐT VĨNH PHÚC ———————— ĐỀ CHÍNH THỨC KỲ THI CHỌN HSG LỚP 1…. - THPT NĂM HỌC 201… - 201… (ĐỀ LUYỆN 10) ĐỀ THI MÔN: TIẾNG ANH ———————————— I. PHONETICS Part 1: Circle the word whose underlined part is pronounced differently from that of the others in each group. Circle A, B, C or D to indicate your answer. (5 pts) B. loud C. moldy D. poultry 1. A. arrow 2. A. basilisk B. bison C. basic D. basin B. indebtedness C. bombard D. combing 3. A. subtlety 4. A. benevolent B. content C. molecules D. technique 5. A. conscience B. bronchitis C. shuttle D. chauffeur Part 2: Choose the word whose syllable is stressed differently from that of the others in each line. Circle A, B, C or D to indicate your answer. (5 pts) 6. A. comic B. clementine C. climax D. thermonuclear 7. A. diligent B. dimension C. action D. innate 8. A. characterize B. absence C. datum D. charcoal 9. A. solicitor B. separately C. spacious D. sequence 10. A. parachute B. armchair C. accent D. accidentally II. LEXICO – GRAMMAR Part 1: Choose the word or phrase that best completes each sentence. Circle A, B, C or D to indicate your answer. (20 pts) 11. Assembly lines are useful for producing a large _____ of identical products. A. quality B. quantity C. quandary D. qualification 12. Only the _____ of the building is going to be remodeled. A. insides B. interior C. indoors D. inner 13. Whether the sports club survives is a matter of complete _____ to me. A. indifference B. disinterest C. importance D. interest 14. After years of neglect there was a huge _____ program to return the city to its former glory. A. restoration B. preservation C. conservation D. refurbishment 15. The assistant suggested _____ the next day when the manager would be there. A. we are coming back B. to come back C. we will come back D. we came back 16. I never get a _____ of sleep after watching a horror film. BY ĐỖ BÌNH – THPT LIẾN SƠN, LẬP THẠCH, VĨNH PHÚC www.violet.vn/quocbinh72 A. wink B. blink C. night D. ounce 17. As it was Christmas, the _____ at church was much larger than usual. A. audience B. convention C. congregation D. grouping 18. The sheep were huddled into a _____ to protect them from overnight frosts. A. cage B. kennel C. hutch D. pen 19. The jury _____ the defendant “not guilty”. A. gave B. returned C. subscribed D. found 20. Many _____ crafts such as weaving are now being revived. A. customary B. habitual C. traditional D. ordinary 21. He managed to finish his thesis under the _____ of his tutor. A. guidance B. help C. aid D. assistance 22. Mr. Henry was given a medal in _____ of his service to his country. A. gratitude B. knowledge C. recognition D. response 23. Everyone knows about pollution problems, but not many people have _____ any solutions. A. thought over B. come up with C. looked into D. got round to 24. You _____ as well seek for a fish in the tree as try to do that. A. must B. would C. should D. might 25. _____ calculations have shown that the earth’s resources may run out before the end of the next century. A. Raw B. Rude C. Crude D. Blunt 26. By the time you receive this letter, I _____ for China. A. will have left B. have left C. would have left D. will leave 27. Prizes are awarded _____ the number of points scored. A. resulting from B. adding up C. presented to D. according to 28. The needs of gifted children in schools have long been _____ neglected. A. dolefully B. woefully C. idly D. pathetically 29. I must take this watch to be repaired; it _____ over 20 minutes a day. A. increases B. gains C. accelerates D. progresses 30. It had been a trying afternoon, _____ at about six o’clock in the television breaking down. A. culminating B. leading C. arriving D. finalizing Part 2: Write the correct FORM of each bracketed word in the numbered space provided in the column on the right. (0) has been done as an example. (10 pts) According to some (0) _____ (SCIENCE), high-risk sports can be particularly (31) _____ scientists (VALUE) for certain types of people. Such activities help them to learn that being (32) 31. ________________ _____ (FRIGHT) doesn’t mean that they have to lose control. The recent fashion for 32. ________________ jumping from bridges attached to a (33) _____ (LONG) of elastic rope, known as “bungee 33. ________________ jumping”, has now been tried by over one million people (34) _____ (WORLD) and 34. ________________ interest in it is continuing to grow. 35. ________________ Before the special elastic rope (35) _____ (TIGHT) around them, jumpers reach speeds of 36. ________________ nearly 160kph. First-timers are usually too (36) _____ (TERROR) to open their mouths, 37. ________________ and when they are finally (37) _____ (LOW) safely to the ground, they walk around with 38. ________________ broad smiles on their faces, saying (38) _____ (REPEAT) how amazing it was. However, 39. ________________ for some people, it is only the (39) _____ (EMBARRASS) of refusing to jump at the last minute that finally persuades them to conquer their fear of (40) _____ (HIGH) and push 40. ________________ themselves off into space. Part 3: The passage below contains 10 mistakes. Underline the mistakes and write their correct forms in the space provided in the column on the right. (0) has been done as an example. (10 pts) Traditional, mental tests have been divided into two types. Achievement 0. traditional → traditionally tests are designed to measure acquiring skills and knowledge, particularly 41. _____________________ those that have been explicitness taught. The proficiency exams required by 42. _____________________ few states for high school graduation are achievement tests. Aptitude tests 43. _____________________ are designed and measure a person’s ability to acquire new skills but 44. _____________________ knowledge. For example, vocation aptitude tests can help you decide 45. _____________________ whether you would do better like a mechanic or musician. However, all 46. _____________________ mental tests are in some sense achievement tests because they assumption 47. _____________________ some sort of past learning or experience with certainly objects, words, or 48. _____________________ situations. The difference between achievement and aptitude tests is the 49. _____________________ degree and intention use. 50. _____________________ BY ĐỖ BÌNH – THPT LIẾN SƠN, LẬP THẠCH, VĨNH PHÚC www.violet.vn/quocbinh72 Part 4: Fill in each of the gaps with the correct preposition or particle. Write your answers in the numbered spaces provided below the passage. (10 pts) The show was fully booked (51) _____ for weeks, and when it opened last night, the public poured (52) _____ and very soon the London Arts Center was packed (53) _____. But why? What did they come to see? They came to see human beings take (54) _____ circus animals, men in cat suits who stood (55) _____ for real lions and tigers. The show was put (56) _____ by its creators to protest (57) _____ traditional circuses and to send a message about cruelty to animals. The show was timed to tie (58) _____ with the National Protection of Animals Week. It was a good idea, but the standard of the performances was third-rate and an embarrassing number of people simply walked (59) _____ before it ended. There were some amusing moments when the performers sent (60) _____ typical circus folks, but overall it was a dismal show. Despite the large turnout for the show’s first night, I doubt it will attract many people during the rest of its seven-day run. Part 5: Complete each sentence with the correct form of ONE of the phrasal verbs below. Write your answers in the numbered spaces provided below the passage. Each verb is used only once. (10 pts) try out slip up carry on get by put out take after get down look up go through turn down 61. If you’re finding it difficult to _____ on your salary, why don’t you ask for a raise? 62. I know what you’re _____ and I feel really sorry for you. 63. In many ways you _____ your father. 64. If you _____ you’ll get into trouble. 65. I proposed to her but she _____ me _____. 66. You’d better _____ your cigarette because smoking isn’t allowed in here. 67. If you _____ working so hard, you’ll make yourself ill. 68. Stop worrying about it. Don’t let this failure ____ you _____. 69. The car’s in quite good condition but you can _____ it _____ before you make any decision to buy, 70. When I was in New York, I was able to _____ several old friends I hadn’t seen for years. Part 6: Insert A, AN, THE or Ø (zero article) where necessary. Write your answers in the numbered spaces provided below the passage. (10 pts) Suddenly (71) _____ blackbird flew to (72) _____ top of (73) _____ beach. She perched way up on (74) _____ topmost twig that stuck up thin against (75) _____. Then she commenced to sing. Her little black body seemed only (76) _____ tiny dark speck at that distance. She looked like (77) _____ old dead leaf. But she poured out her song in (78) _____ great flood of rejoicing through (79) _____ whole forest. And (80) _____ things began to stir. III. READING COMPREHENSION Part 1: Read the following passage and decide which answer (A, B, C or D) best fits each gap. Circle A, B, C or D to indicate your answer. (10 pts) The ability to weep is a uniquely human form of emotional response. Some scientists have suggested that human tears are (81) _____ of an aquatic past – but this does not seem very likely. We cry from the moment we enter this world, for a number of reasons. Helpless babies cry to persuade their parents that they are ill, hungry or uncomfortable. As they (82) _____, they will also cry just to attract parental attention and will often stop when they get it. The idea that having a good cry do you (83) _____ is a very old one and now it has scientific validity since recent research into tears has shown that they (84) _____ a natural painkiller called enkaphalin. By fighting sorrow and pain this chemical helps you feel better. Weeping can increase the quantities of enkaphalin you (85) _____. Unfortunately, in our society we impose restrictions upon this naturally (86) _____ activity. Because some people still regard it as a (87) _____ of weakness in men, boys in particular are admonished when they cry. This kind of repression can only increase stress, both emotionally and physically. Tears of emotion also help the body (88) _____ itself of toxic chemical waste, for there is more protein in them than in tears resulting from cold winds or other irritants. Crying comforts, calms and can be very enjoyable – (89) _____ the popularity of highly emotional films which are commonly (90) _____ “weepies”. It seems that people enjoy crying together almost as much as laughing together. 81. A. witness B. evidence C. result D. display 82. A. evolve B. change C. develop D. alter 83. A. better B. fine C. good D. well 84. A. contain B. retain C. hold D. keep 85. A. construct B. achieve C. provide D. produce 86. A. curing B. treating C. healing D. improving 87. A. hint B. symbol C. feature D. sign 88. A. release B. rid C. loosen D. expel BY ĐỖ BÌNH – THPT LIẾN SƠN, LẬP THẠCH, VĨNH PHÚC www.violet.vn/quocbinh72 89. A. consider B. remark C. distinguish D. regard 90. A. named B. entitled C. subtitled D. called Part 2: Read the following passage and fill the blank with ONE suitable word. Write your answer in the space provided below the passage. (10 pts) New technologies, like all technologies, are morally neutral. (91) _____ their advent makes the world a better place or not depends on the uses to which they are (92) _____. And that, (93) _____ turn, depends upon the decisions of many people, especially of politicians, managers, trade (94) _____ leaders, engineers and scientists. The new technologies, cheap, flexible, dependent on knowledge and information as their main input, can (95) _____ human being from many of their current constraints for example constraints of resources and geography. (96) _____ the new technologies could also (97) _____ those with power to control their fellow citizens even more effectively than in the (98) _____ efficient dictatorships of the past. The new technological society will (99) _____ colossal demands on our imagination and ingenuity and on the capacity (100) _____ our institutions to respond to new challenges. Part 3: Read the following passage and answer the questions from 101 to 110. (10 pts) Legend has it that sometime toward the end of the Civil War (1861-1865) a government train carrying oxen traveling through the northern plains of eastern Wyoming was caught in a snowstorm and had to be abandoned. The driver returned the next spring to see what had become of his cargo. Instead of the skeletons he had expected to find, he saw his oxen, living, fat, and healthy. How had they survived? The answer lay in a resource that unknowing Americans lands trampled underfoot in their haste to cross the “Great American Desert” to reach lands that sometimes proved barren. In the eastern parts of the United States, the preferred grass for forage was a cultivated plant. It grew well with enough rain, then when cut and stored it would cure and become nourishing hay for winter feed. But in the dry grazing lands of the West that familiar bluejoint grass was often killed by drought. To raise cattle out there seemed risky or even hopeless. Who could imagine a fairy-tale grass that required no rain and somehow made it possible for cattle to feed themselves all winter? But the surprising western wild grasses did just that. They had wonderfully convenient features that made them superior to the cultivated eastern grasses. Variously known as buffalo grass, grama grass, or mesquite grass, not only were they immune to drought; but they were actually preserved by the lack of summer and autumn rains. They were not juicy like the cultivated eastern grasses, but had short, hard stems. And they did not need to be cured in a barn, but dried right where they grew on the ground. When they dried in this way, they remained naturally sweet and nourishing through the winter. Cattle left outdoors to fend for themselves thrived on this hay. And the cattle themselves helped plant the fresh grass year after year for they trampled the natural seeds firmly into the soil to be watered by the melting snows of winter and the occasional rains of spring. The dry summer air cured them much as storing in a barn cured the cultivated grasses. 101. What does the passage mainly discuss? A. A type of wild vegetation B. Western migration after Civil War C. The raising of cattle D. The climate of the Western United States 102. What can be inferred by the phrase “Legend has it” in line 1? A. Most history book include the story of the train. B. The story of the train is similar to other ones from that time period. C. The driver of the train invented the story. D. The story of the train may not be completed factual. 103. The word “they” in line 4 refers to _____. A. plains B. skeletons C. oxen D. Americans 104. What can be inferred about the “Great American Desert” mentioned in line 7? A. Many had settled there by the 1860’s. B. It was not originally assumed to be a fertile area. C. It was a popular place to raise cattle before the Civil War. D. It was not discovered until the late 1800’s. 105. The word “barren” in line 7 is closed in meaning to _____. A. lonely B. uncomfortable C. infertile D. dangerous 106. The word “preferred” in line 8 is closed in meaning to _____. A. favored B. available C. ordinary D. required 107. Which of the following can be inferred about the cultivated grass mentioned in the second paragraph? A. Cattle raised in the Western United States refused to eat it. B. It had to be imported into the United States. C. It would probably not grow in the western United States. D. It was difficult for cattle to digest. 108. Which of the following was NOT one of the names given to the western grasses? A. Mesquite grass B. Bluejoint grass C. Buffalo grass D. Grama grass 109. Which of the following was NOT mentioned as a characteristic of western grasses? A. They contain little moisture B. They have tough stems C. They can be grown indoors D. They are not affected by dry weather 110. According to the passage, the cattle help promote the growth of the wild grass by_____. A. eating only small quantities of grass. B. continually moving from one grazing area to another. BY ĐỖ BÌNH – THPT LIẾN SƠN, LẬP THẠCH, VĨNH PHÚC www.violet.vn/quocbinh72 C. naturally fertilizing the soil. D. stepping on and pressing the seeds into the ground. IV. WRITING (6/20 points) Part 1: Finish the second sentence in such a way that it means exactly the same as the sentence printed before it 111. Customs officials are stopping more travelers than usual this week. → An increased ________________ 112. I left without saying goodbye as I didn’t want to disturb the meeting. → Rather ________________ 113. My decision to get up and dance coincided with the band’s decision to stop playing. → The moment ________ 114. He never suspected that the bicycle had been stolen. → At no time ________________ 115. How could I help, except to offer to lend her some money? → Other ________________ Part 2: Use the word(s) given in the brackets and make any necessary additions to complete a new sentence in such a way that it is as similar as possible in meaning to the original sentence. Do NOT change the form of the given word(s). 116. To this day no one has equaled his achievements in the field of technology. (unsurpassed) → To this day ………………………………………………………… in the field of technology. 117. Peter grimaced as he swallowed the foul-tasting medicine. (pulled) → Peter ……………………………………………………………….the foul-tasting medicine. 118. It’s unfortunately that the construction of the building will not be finished as originally planned. (longer) → The construction of the building ………………………………………………………… unfortunate. 119. What he told me made me very curious to hear the rest of the history. (appetite) → What he told me ………………………………………………………………………….. the story. 120. They chose not to drive because they thought there would be too much snow. (fear) → They chose …………………………………………………………………….…. too much snow. _____THE END_____ SỞ GD&ĐT VĨNH PHÚC ———————— ĐỀ CHÍNH THỨC KỲ THI CHỌN HSG LỚP 1…. - THPT NĂM HỌC 201… - 201… (ĐỀ LUYỆN 11) ĐỀ THI MÔN: TIẾNG ANH ———————————— PART I: VOCABULARY AND GRAMMAR. (7, 0 POINTS) I. Choose the best option to complete each of the sentences. (2,0 points) 1. The strike was ___ owing to a last minute agreement with the management. A. called off B. broken up C. set back D. put down 2. Lindsay’s excuses for being late are beginning to ___ rather thin. A. get B. turn C. wear D. go 3. ___ the people who come to this club are in their twenties and thirties. A. By and large B. Altogether C. To a degree D. Virtually 4. My cousin was nervous about being interviewed on television, but she rose to the ___ wonderfully. A. event B. performance C. incident D. occasion 5. The train service has been a ___ since they introduced the new schedules. A. shambles B. rumpus C. chaos D. fracas 6. Is an inexperienced civil servant ___ to the task of running the company. A. capable B. skilled C. eligible D. suited 7. We ___ have been happier in those days. A. can’t B. couldn’t C. might not D. must not 8. You’ve lived in the city for most of your life, so ___ you’re used to the noise. A. apparently B. presumably C. allegedly D. predictably 9. The storm ripped our tent to ___ A. slices B. shreds C. strips D. specks 10. He ____ so much harm on the nation during his regime that it has never fully recovered. A. indicted B. inferred C. induced D. inflicted 11. Hotel rooms must be ____ by 10 a. m, but luggage may be left with porters. A. vacated B. evacuated C. abandoned D. left 12. I do not think there is so much as a ____ of truth in that rumor. A. crumb B. speck C. grain D. pebble 13. He’s not exactly rich but he certainly earns enough money to ____ BY ĐỖ BÌNH – THPT LIẾN SƠN, LẬP THẠCH, VĨNH PHÚC www.violet.vn/quocbinh72 A. get through B. get by C. get on D. get up 14. I have very ____ feelings about the plan – it might possibly work or it could be a disaster. A. certain B. mixed C. doubtful D. troubled 15. The noise of the typewriter really ____ me off. I just couldn’t concentrate. A. put B. pulled C. set D. took 16. The sixth time he called me at night was the ____ A. last cause B. last straw C. touch and go D. hot air 17. All three TV channels provide extensive ____ of sporting events. A. broadcast B. network C. coverage D. vision 18. They seemed to be ____ to the criticism and just carried on as before. A. disinterested B. sensitive C. uncaring D. indifferent 19. “Shall we go out tonight?” - “____” A. Yes, I can B. Yes, we are C. Yes, we go D. Yes, let’s 20. It's no use ___ over ___ milk. A. crying/ spilt B. to cry/ spilling C. crying/ spilling D. crying/ to spill Your answers: 1. 2. 3. 4. 5. 6. 7. 8. 9. 10. 11. 12. 13. 14. 15. 16. 17. 18. 19. 20. II. Read the passage below. Use the word given in bold to form a word that fits in the numbered gap. (2, 5 points) BLACK WIDOW SPIDER The black widow spider’s notoriety is not without foundation. However, an element of exaggeration has led to certain ___ (1. CONCEPT) regarding its evil nature. Firstly, this spider is not as dangerous as is often thought. While it is indeed one of the most ___ (2. VENOM) species of spider, its venom being fifteen times stronger than that of the prairie rattlesnake, its bite injects such as amount of venom by ___ (3. COMPARE) that it is unlikely to kill humans. In fact, ___ (4. FATAL) are rare. Black widows bite only if they are touched or their web is threatened. Furthermore, only the adult female is poisonous. Those most at risk from the female are the spider’s natural pray-insects-and male black widow spiders. The latter are vulnerable as the female is ___ (5. SOLITUDE) by nature, and has been known to kill and eat the male after mating. Such ___ (6. OCCUR) are rare, but they explain how the spider got its name – and its reputation. Nevertheless, the ___ (7. PLEASE) effects of this spider’s bite should not be ___ (8. ESTIMATE), and if you live in a temperate climate and have a fireplace in your home, it is advisable to take ___ (9. CAUTION). Black widow spiders often inhabit wood piles, so you should wear gloves when handling firewood. Furthermore, since black widow spiders are ___ (10. RESIST) to many insecticides, you should regularly clean out likely hiding places. Your answers: 1. 2. 3. 4. 5. 6. 7. 8. 9. 10. III. Complete each sentence with the correct form of ONE of the two-word verbs. (2, 0 point) bring out do up slow down save up hang over mix up grow up go with dress up run for 1. Silence ___the theatre as the audience awaited the opening curtain with expectation and excitement. 2. Having seen a sharp bend ahead, Tim pressed hard on the brake pedal to ___. 3. Charles Dickens was born near Portsmouth, Hampshire on 7 February 1812, but relocated to and ___ in Camden Town in London. 4. Barrack Obama has decided to ___ the American presidency in the election that will take place next year. 5. Because I hate ___ my shoes, I have bought a pair of shoes without any laces. 6. Sarah wanted to buy some curtains that would ___ his furniture, so she had brought a photo of her sofa with her to the store. 7. Kate had been ___ in order to buy a new laptop, but then she decided to use the money on the guitar lessons instead. BY ĐỖ BÌNH – THPT LIẾN SƠN, LẬP THẠCH, VĨNH PHÚC www.violet.vn/quocbinh72 8. Even though two of the bands are dead, a new ‘The Beatle’ album called Love was recently ___. 9. Brendan was worried about having to ___ for the boss’ retirement dinner as he didn’t own any formal clothes. 10. He ___ his grandmother’s phone number with his girlfriend’s, which led to some embarrassment for him. Your answers: 1. 2. 3. 4. 5. 6. 7. 8. 9. 10. IV. The passage below contains TEN mistakes. Underline them and write the correct forms in the numbered boxes. (2, 5 points) Large animals inhabit the desert have evolved adaptations for reducing the effects of extreme hot. One adaptation is to be light in color, and to reflect the Sun's rays. Desert mammals also depart from the normal mammalian practice of maintaining a constantly body temperature. Instead of try to keep down the body temperature inside the body, what would involve the expenditure of water and energy, desert mammals allow their temperatures rise to what would normally be fever height, and temperatures as high as 46 degree Celsius have been measured in Grant's gazelles. The overheated body cools down during the cold desert night, and indeed the temperature may fall unusual low by dawn, as low as 34 degrees Celsius in the camel. This is a advantage since the heat of the first few hours of daylight absorb in warming up the body. Your answers: 1. 2. 3. 4. 5. 6. 7. 8. 9. 10. PART II: READING COMPREHENSION. (6, 0 POINTS) I. Read the passage and fill in each of the blanks with ONE suitable word. (2,0 points) True relaxation is most certainly not a matter of flopping down in front of the television with a welcome drink. Nor is it about drifting (1) ___ an exhausted sleep. Useful though these responses to tension and over-tiredness (2) ___ be, we should distinguish between them and conscious relaxation in (3) ___ of quality and effect. (4) ___ of the level of tiredness, real relaxation is a state of alert yet at the same time passive awareness, in which our bodies are (5) ___ rest while our minds are awake. Moreover, it is as natural for a healthy person to be relaxed when moving as resting. (6) ___ relaxed in action means we bring the appropriate energy to everything we do, so as to have a feeling of healthy tiredness by the end of the day, (7) ___ than one of exhaustion. Unfortunately, as a result of living in today’s competitive world, we are under constant strain and have difficulty in coping, (8) ___ alone nurturing our body’s abilities. What needs to be rediscovered is conscious relaxation. With (9) ___ in mind we must apply ourselves to understanding stress and the nature of its causes (10) ___ deep-seated. Your answers: 1. 2. 3. 4. 5. 6. 7. 8. 9. 10. II. Read the text and do the tasks as follow. (2, 0 points) OUT OF THE ASHES Paragraph A On the afternoon of 30th August 1989, fire broke out at Uppark, a large eighteenth century house in Sussex. For a year builders had been replacing the lead on the roof, and by a stroke of irony, were due to finish the next day, on August 31st. Within fifteen minutes of the alarm being sounded, the fire brigade had arrived on the scene, though nothing was to survive of the priceless collection on the first floor apart from an oil painting of a dog which the firemen swept up as they finally retreated from the blaze. But due to the courage and swift action of the previous owners, the MeadeFeatherstonhaugh family, and the staff, stewards and visitors to the house, who formed human chains to pass the precious pieces of porcelain, furniture and paintings out on to the lawn, 95 percent of the contents from the ground floor and the basement were saved. As the fire continued to rage, the National Trust’s conservators were being mobilised, and that evening local stationers were especially opened to provide the bulk supplies of blotting paper so desperately needed in the salvage operation. Paragraph B The following morning, Uppark stood open to the sky. A sludge of wet charcoal covered the ground floor and basement, and in every room charred and fallen timbers lay amongst the smoke. It was a scene of utter devastation. Paragraph C After the initial sense of shock, the days which followed the fire were filled with discoveries. Helped by volunteers, the National Trust’s archaeologists and conservators swung into action, first of all marking the site out into a grid and then salvaging everything down to the last door handle. The position of each fragment was recorded, and all the BY ĐỖ BÌNH – THPT LIẾN SƠN, LẬP THẠCH, VĨNH PHÚC www.violet.vn/quocbinh72 debris was stored in countless dustbins before being sifted and categorised. Paragraph D There was great excitement as remnants of the lantern from the Staircase Hall were pulled out from the debris of two fallen floors, and also three weeks later when the Red Room carpet, thought to have been totally lost, was found wrapped around the remains of a piano. There was a lucky reprieve for the State Bed too. Staff who had left the scene at 3 a.m on the night of the fire had thought its loss was inevitable, but when they returned the next morning it had escaped largely undamaged. Firemen, directed by the National Trust’s conservators from outside the Tapestry Room window, dismantled the silk-hung bed and passed it out piece by piece. Twenty minutes later the ceiling fell in. Paragraph E The scale of the task to repair Uppark was unprecedented in the National Trust. The immediate question was whether it should be done at all. A decision had to be taken quickly, as the building was unsound and whatever had not been damaged by the fire was exposed to the elements. Within a month, after consulting many experts and with the agreement of the National Trust’s Executive Committee, the restoration programme began. It was undertaken for three main reasons. After the fire it had become apparent just how much remained of the structure with its splendidly decorated interiors; to have pulled the house down, as one commentator suggested, would have been vandalism. Also the property was covered by insurance, so the repairs would not call upon the National Trust’s own funds. Lastly, much had been saved of the fine collection acquired especially for Uppark from 1747 by Sir Matthew Featherstonhaugh and his son Harry. These objects belonged nowhere else, and complete restoration of the house would allow them to be seen and enjoyed again in their original setting. Paragraph F The search for craftsmen and women capable of doing the intricate restoration work was nation-wide. Once the quality and skill of the individual or company had been ascertained, they had to pass an economic test, as every job was competitively tendered. This has had enormous benefits because not only have a number of highly skilled people come to the fore - woodcarvers for example, following in the footsteps of Grinling Gibbons - but many of them, for example plasterers, have relearnt the skills of the seventeenth and eighteenth centuries which can now be of use to other country house owners when the need arises. Paragraph G In June 1994 the building programme was completed, on time and on budget. The total cost of the work to repair the house and its contents came to be nearly £20 million, largely met from insurance. In addition, it made economic sense for the National Trust to invest time and money in upgrading water and heating systems, installing modern environmental controls, and updating fire and security equipment. Paragraph H The final stages of restoration and the massive programme of reinstallation took eight months. The family and the room stewards were visibly moved when returning to their old haunts, perhaps the best testament that the spirit of Uppark had not died. But the debate will no doubt continue as to whether or not it was right to repair the house after the fire. The National Trust has done its best to remain true to Uppark; it is for others to judge the success of the project. Note: The National Trust is a charitable organisation in Britain set up over a hundred years ago to preserve the national heritage. Questions 1–6. The text has eight paragraphs, A–H. Which paragraphs contain the following information? Write the appropriate letters, A–H, in the boxes. 1. The procedure for sorting through the remains of the fire. 2. How Uppark looked after the fire. 3. Improvements made to the rebuilt Uppark. 4. The selection of people to carry out the repair work. 5. Why the National Trust chose to rebuild Uppark. 6. How people reacted to the rebuilt Uppark. Questions 8–10. Answer the questions below. Choose NO MORE THAN THREE WORDS from the text for each answer. Write your answers in the boxes. 7. On what date in 1989 should the original repairs to the roof have been completed? 8. By what method were things rescued immediately from the burning house? 9. After the fire, what did the conservators require large quantities of immediately? 10. Into what did the conservators put material recovered from the fire? Your answers: 1. 2. 3. 4. 5. BY ĐỖ BÌNH – THPT LIẾN SƠN, LẬP THẠCH, VĨNH PHÚC www.violet.vn/quocbinh72 6. 7. 8. 9. 10. III. Read the following passage and choose the best answer (A, B, C or D) to each question. Write your answer in the numbered box. (2, 0 points) A recent survey of crime statistics shows that we are all more likely to be burgled now than 20 years ago and the police advise everyone to take a few simple precautions to protect their homes. The first fact is that burglars and other intruders prefer easy opportunities, like a house which is very obviously empty. This is much less of a challenge than an occupied house, and one which is well-protected. A burglar will wonder if it is worth the bother. There are some general tips on how to avoid your home becoming another crime statistic. Avoid leaving signs that your house is empty. When you have to go out, leave at least one light on as well as a radio or television, and do not leave any curtains wide open. The sight of your latest music centre or computer is enough to tempt any burglar. Never leave a spare key in a convenient hiding place. The first place a burglar will look is under the doormat or in a flower pot and even somewhere more 'imaginative' could soon be uncovered by the intruder. It is much safer to leave a key with a neighbour you can trust. But if your house is in a quiet, desolate area be aware that this will be a burglar's dream, so deter any potential criminal from approaching your house by fitting security lights to the outside of your house. But what could happen if, in spite of the aforementioned precautions, a burglar or intruder has decided to target your home. Windows are usually the first point of entry for many intruders. Downstairs windows provide easy access while upstairs windows can be reached with a ladder or by climbing up the drainpipe. Before going to bed you should double-check that all windows and shutters are locked. No matter how small your windows may be, it is surprising what a narrow gap a determined burglar can manage to get through. For extra security, fit window locks to the inside of the window. What about entry via doors? Your back door and patio doors, which are easily forced open, should have top quality security locks fitted. Even though this is expensive it will be money well spent. Install a burglar alarm if you can afford it as another line of defence against intruders. A sobering fact is that not all intruders have to break and enter into a property. Why go to the trouble of breaking in if you can just knock and be invited in? Beware of bogus officials or workmen and, particularly if you are elderly, fit a chain and an eye hole so you can scrutinise callers at your leisure. When you do have callers never let anybody into your home unless you are absolutely sure they are genuine. Ask to see an identity card, for example. If you are in the frightening position of waking in the middle of the night and think you can hear an intruder, then on no account should you approach the intruder. It is far better to telephone the police and wait for help. 1. A well-protected house ………… A. is less likely to be burgled. B. is regarded as a challenge by most criminals. C. is a lot of bother to maintain. D. is very unlikely to be burgled. 2. According to the writer, we should ………… A. avoid leaving our house empty. B. only go out when we have to. C. always keep the curtains closed. D. give the impression that our house is occupied when we go out. 3. The writer thinks that hiding a key under a doormat or flower pot ………… A. is predictable. B. is useful. C. is imaginative. D. is where you always find a spare key. 4. What word best replaces “desolate” in paragraph 4? A. isolated B. populous C. dissatisfying D. depressing 5. The phrase “aforementioned precautions” in paragraph 5 refers to steps that ………… A. will tell a burglar if your house is empty or not. B. are the most important precautions to take to make your home safe. C. will stop a potential burglar. D. will not stop an intruder if he has decided to try and enter your home. 6. Gaining entry to a house through a small window ………… A. is surprisingly difficult. B. is not as difficult as people think. C. is less likely to happen than gaining entry through a door. D. is tried only by very determined burglars. 7. According to the writer, window locks, security locks and burglar alarms ………… A. cost a lot of money but are worth it. B. are good value for money. C. are luxury items. D. are absolutely essential items. 8. The writer argues that fitting a chain and an eye hole………… A. will prevent your home being burgled. B. avoids you having to invite people into your home. C. is only necessary for elderly people. D. gives you time to check if the visitor is genuine. 9. What word best replaces “scrutinise” in paragraph 7? BY ĐỖ BÌNH – THPT LIẾN SƠN, LẬP THẠCH, VĨNH PHÚC www.violet.vn/quocbinh72 A. glance B. gaze C. search D. examine 10. The best title for the text is ………… A. Increasing household crime. B. Protecting your home from intruders. C. Burglary statistics. D. What to do if a burglar breaks into your home. Your answers: 1. 2. 3. 4. 5. 6. 7. 8. 9. 10. PART III. WRITING. (4,0 POINTS) I. Complete each of the sentences so that it has similar meaning to the given one, using the word given in bold. Do NOT change the form of the word given. (1, 0 point) 1. Although Joe kept on attempting to contact his cousin, he didn't manage to speak to her until the next day. → Despite repeated ........................ TOUCH................... his cousin, Joe didn't manage to speak to her until the next day. 2. When it comes to computer games, Jack is a real expert. CONCERNED → As far ............................................................................., Jack is a real expert. 3. David said that the accident was his fault. TOOK → David .................................................................. the accident. 4. If he doesn’t get that job, who knows what he’ll do. KNOWING → If he doesn’t get that job, ........................................................... what he’ll do. 5. The stranded climber would never have been rescued if his brother hadn’t had an ingenious plan. INGENUITY → But .......................................................... plan, the stranded climber would never have been rescued. II. Use the word given and make any necessary additions to complete a new sentence in such a way that it is as similar as possible to the original one. Do NOT change the form of the word given. (1, 0 point) 1. They will try John for murder at the High Court next week. (trial) → John …………………………………………………………..……………………………………… 2. How do our sales compare with those of other firms? (relation) → How do …………………………………………………………………..…………………………… 3. The number of accidents has gone down steadily since the speed limit was imposed. (decline) → There has ……………………………………………………………..……………………………… 4. This new record is certain to sell a lot of copies. (doubt) → There is…………………………………………………………..…………………………………… 5. I certainly won’t change my mind about resigning. (question) → My changing …………………………………………………….…………………………………… III. The charts below give information on the ages of the populations of Yemen and Italy in 2000 and projections for 2050. Summarize the information by selecting and reporting the main features and make comparisons where relevant. (2, 0 points) 2000 2050 YEMEN 37,0% 46,3% 50,1% 57,3% 5,7% 3,6% 2000 24,1% 2050 ITALY 46,2% 42,3% BY ĐỖ BÌNH – THPT LIẾN SƠN, LẬP THẠCH, VĨNH PHÚC www.violet.vn/quocbinh72 61,6% 14,3% SỞ GD&ĐT VĨNH PHÚC ———————— ĐỀ CHÍNH THỨC KỲ THI CHỌN HSG LỚP 1…. - THPT NĂM HỌC 201… - 201… (ĐỀ LUYỆN 12) ĐỀ THI MÔN: TIẾNG ANH ———————————— A. LISTENING: (15 points) I. Listen to the conversation and then choose the correct answer among A, B, C, or D for each question: II. Listen to the conversation and fill in each blank with ONE word: B. PHONETICS: (5 points) Choose the word that has a different stress pattern from the others in the group: 16. A. responsibility B. originality C. accommodation D. mischievousness 17. A. appliance B. conscientious C. independent D. confidential 18. A. psychology B. environmental C. impossible D. photography 19. A. stimulate B. maximize C. interrupt D. register 20. A. appointment B. punishment C. publicity D. efficient C. GRAMMAR – VOCABULARY – LANGUAGE FUNCTIONS: I. Choose the word, phrase or expression which best completes each sentence: (15 points) 21. Tom: “I thought your performance last Sunday was wonderful.” Laura: “………………………..” A. Don’t tell a lie. I thought it was terrible. B. You must be kidding. It was not as good as I had expected. C. I completely agree with you. It was terrific. D. No doubt! 22. Boy: “What is your greatest phobia?” Girl: “………………………..” A. I'm afraid not. B. Worms, definitely! C. Probably people who smoke. D. I haven't made up my mind. 23. Jenny: “Thank you very much for your donation, Mr. Robinson.” Mr. Robinson: “………………………..” A. You can say that again. B. I see. C. You are right. D. Delighted I was able to help. 24. I don’t think you have been watering the plants near the gate. The soil is ……………………….. A. as dry as rice B. as dry as a tile C. as dry as a bone D. as dry as wood 25. Susan was sad because she wasn’t invited to any social events. She felt ……………………….. A. left out B. turned out C. omitted out D. gone out BY ĐỖ BÌNH – THPT LIẾN SƠN, LẬP THẠCH, VĨNH PHÚC www.violet.vn/quocbinh72 26. Most psychologists believe that the basic structure of an individual’s personality is……………………….. A. well established extremely by the age of five B. by the age of five it is extremely well established C. by the age of five and well established extremely D. extremely well established by the age of five 27. In most ………. developed countries, up to 50% of ……..population enters higher education at some time in their lives. A. Ø / Ø B. the / Ø C. Ø / the D. the / a 28. I wish you ………………….. me a new one instead of having it………………………..as you did. A. would give / to repair B. gave / to repair C. had given / to be repaired D. had given / repaired 29. Henry was really a silly boy when we were at high school. I still remember………………………..very stupid questions. A. him asking B. him to ask C. asking him D. his being asked 30. ………………………..the invention of the steam engine, most forms of transport were horse-drawn. A. With reference B. Akin C. Prior to D. In addition to 31. No sooner ………………………..to marry Jack ………………………..to have serious doubts. A. had Carol agreed / than she began B. Carol has agreed / than she began C. had Carol agreed / than she begins D. had Carol agreed / than she had begun 32. Having been served dinner, ……………………... A. the problem was discussed by the members of the committee. B. the committee members discussed the problem. C. it was discussed by the committee members the problem. D. a discussion of the problem was made by the members of the committee. 33. The marathon runner ……………………...for nearly one hour and a half when she ……………………...to the pavement. A. has been running / collapses B. were running / collapsed C. had been running / collapsed D. ran / had been collapsing 34. Son: “What is the process of ……………………..., Dad?” Father: “Well, it involves the heating of liquid such as milk in order to kill harmful bacteria.” A. industrialization B. pasteurization C. commercialization D. globalization 35. An artist ……………………...will do his best to express innocence and inexperience in the child’s face. A. portraying a child B. who portray a child C. he portrays a child D. portrayed a child II. Give the correct form of the words in brackets: (10 points) (POOR) 36. Deforestation and excessive farming have ……………………... the soil. 37. The damage caused by the terrible storm two days ago was……………………... by the (ESTIMATE) government. The real figures go up every minute. 38. Barack Obama is the first President of the United States with ……………….…... (RACE) (BREATH) background. (SUSPECT) 39. From the hotel there is a……………………...view across the canyon. 40. The policeman examined the parcel ……………………... as he had no idea what it could be. (OBSERVE) (FURIOUS) 41. It was very ……………………...of you to notice that. (VALID) 42. In his …………………….., Mike smashed all the breakable items in the kitchen. 43. Before enrolling on a course, you should first ensure that it has been ……………………... (ATTEND) by an officially recognized body. (PRONOUNCE) 44. Mr. John, who teaches us Latin, usually stresses the need for regular ……………………... 45. The government has yet to make an official ……………………...on the issue. III. Choose the underlined word or phrase in each sentence that needs correcting (5 points): 46. Dictionaries frequently explain the origin of the defined word, state its part of speech and indication its correct use. A B C D 47. The rings of Saturn are so distant to be seen from the Earth without a telescope. A B C D BY ĐỖ BÌNH – THPT LIẾN SƠN, LẬP THẠCH, VĨNH PHÚC www.violet.vn/quocbinh72 48. Children subjected to violence, exploitation , abuse and neglect are in risk of death, poor physical and mental health, HIV/AIDS infection, and educational problems. A B C D 49. It is important that cancer is diagnosed and treated as early as possible in order to assure a successful cure. A B C D 50. A ten-thousand-dollars reward was offered for the capture of the escaped prisoner. A B C D D. READING: I. Read the following passage and decide which option A, B, C or D best fits each space: (10 points) Everyone has got two personalities - the one that is shown to the world and the other that is (51)…………... and real. You don’t show your secret personality when you are (52)…………..., because you can control yourself. But when you are asleep, your feeling position (53)…………...the real you. In a normal (54)…………..., of course, people often change their position. The important position is the one you go to sleep in. If you go to sleep on your back, you are a very (55)…………...person. You usually believe people and you accept new things or new ideas easily. You don’t like to make people sad, so you never express your (56) …………... feeling. You are quite shy. If you sleep on your stomach, you are a rather secretive person. You (57)…………...a lot and you always easily become sad. You usually live for today not tomorrow. This means that you (58)…………...having a good time. If you sleep curled up, you are probably a very (59)…………...person. You have a low opinion of yourself. You are shy and don’t like meeting people. You (60)………….....to be on your own. You are easily hurt. 51. A. important B. serious C secret D. particular . 52. A. awake B. active C happy D. honest . 53. A. makes B. understand C changes D. shows . 54. A. room B. bed C night D. body . 55. A. independent B. open C talkative D. generous . 56. A. real B. lonely C cheerful D. gentle . 57. A. talk B. sleep C relax D. worry . 58. A. regret B. enjoy C mind D. deny . 59. A. strong B. healthy C nervous D. careful . 60. A. pretend B. oppose C refuse D. prefer . II. There are 5 blanks in the passage below. From the words given in the box, choose the most suitable for each blank. There are THREE extra words that you do not need to use: (5 points) A. legal C. responsibility E. serve G. organizations B. community D. give back F. long-term H. rare Volunteering has many intangible benefits. It can help you (61) ……………………...to society, break down barriers of misunderstanding or fear, explore personal issues and even have fun. It also has a meaningful, positive impact on your (62)…………………….... But did you know that it can have many benefits for you, too? You may have heard that volunteering helps you get into college, but keep in mind they are not just looking for a list of BY ĐỖ BÌNH – THPT LIẾN SƠN, LẬP THẠCH, VĨNH PHÚC www.violet.vn/quocbinh72 (63)……………………... and dates. Colleges want to see a complete picture of you and real examples of your commitment, dedication and interests. Volunteering brings together a variety of people. Both the recipients of your volunteer efforts and your coworkers can be rich sources of insight. For example, maybe you’ll learn about the (64)……………………...profession from a former lawyer you visit at a convalescent center. Colleges pay attention to your life inside and outside the classroom. Your extracurricular activities reveal a great deal about you, such as what your interests are, whether you can manage your priorities and maintain a (65)……………………... promise and how you’ve made an important contribution to something. III. Read the passage and choose the correct answer: (10 points) For many people who live in cities, parks are an important part of the landscape. They provide a place for people to relax and play sports, as well as a refuge from the often harsh environment of a city. What people often overlook is that parks also provide considerable environmental benefits. One benefit of parks is that plants absorb carbon dioxide—a key pollutant—and emit oxygen, which humans need to breathe. According to one study, an acre of trees can absorb the same amount of carbon dioxide that a typical car emits in 11,000 miles of driving. Parks also make cities cooler. Scientists have long noted what is called the Urban Heat Island Effect: building materials such as metal, concrete, and asphalt absorb much more of the sun’s heat and release it much more quickly than organic surfaces like trees and grass. Because city landscapes contain so much of these building materials, cities are usually warmer than surrounding rural areas. Parks and other green spaces help to mitigate the Urban Heat Island Effect. Unfortunately, many cities cannot easily create more parks because most land is already being used for buildings, roads, parking lots, and other essential parts of the urban environment. However, cities could benefit from many of the positive effects of parks by encouraging citizens to create another type of green space: rooftop gardens. While most people would not think of starting a garden on their roof, human beings have been planting gardens on rooftops for thousands of years. Some rooftop gardens are very complex and require complicated engineering, but others are simple container gardens that anyone can create with the investment of a few hundred dollars and a few hours of work. Rooftop gardens provide many of the same benefits as other urban park and garden spaces, but without taking up the much-needed land. Like parks, rooftop gardens help to replace carbon dioxide in the air with nourishing oxygen. They also help to lessen the Urban Heat Island Effect, which can save people money. In the summer, rooftop gardens prevent buildings from absorbing heat from the sun, which can significantly reduce cooling bills. In the winter, gardens help hold in the heat that materials like brick and concrete radiate so quickly, leading to savings on heating bills. Rooftop vegetable and herb gardens can also provide fresh food for city dwellers, saving them money and making their diets healthier. Rooftop gardens are not only something everyone can enjoy, they are also a smart environmental investment. 66. Based on its use in paragraph 2, it can be inferred that mitigate belongs to which of the following word groups? A. exacerbate, aggravate, intensify B. obliterate, destroy, annihilate C. allay, alleviate, reduce D. absorb, intake, consume 67. Using the information in paragraph 2 as a guide, it can be inferred that .................................. A. cities with rooftop gardens are cooler than those without rooftop gardens B. some plants are not suitable for growth in rooftop gardens C. most people prefer parks to rooftop gardens D. most people prefer life in the country over life in the city 68. According to the passage, the Urban Heat Island Effect is caused by the fact(s) that .................................. a. cities are warmer than nearby rural areas b. building materials absorb more of the sun’s heat than organic surfaces c. building materials release the sun’s heat more quickly than organic surfaces A. a. only B. a. and b. only C. b. and c. only D. a., b., and c. 69. Based on the information in paragraph 3, which of the following best describes the main difference between parks and rooftop gardens? A. Parks are expensive to create while rooftop gardens are not. B. Parks are public while rooftop gardens are private. C. Parks absorb heat while rooftop gardens do not. D. Parks require much space while rooftop gardens do not. 70. The author claims all of the following to be the benefits of rooftop gardens except .................................. BY ĐỖ BÌNH – THPT LIẾN SƠN, LẬP THẠCH, VĨNH PHÚC www.violet.vn/quocbinh72 A. increased space for private relaxation B. savings on heating and cooling costs C. better food for city dwellers D. improved air quality 71. According to the author, one advantage that rooftop gardens have over parks is that they .................................. A. decrease the Urban Heat Island Effect B. replenish the air with nourishing oxygen C. do not require the use of valuable urban land D. are less expensive than traditional park spaces 72. The author’s tone in the passage is best described as.................................. A. descriptive B. passionate C. informative D. argumentative 73. Which of the following pieces of information would, if true, most weaken the author's claim that rooftop gardens are good for the environment? A. Parks have many benefits that rooftop gardens do not share. B. More pollution is produced during rooftop garden construction than rooftop plants can remove from the air. C. Extremely high winds atop tall city buildings can severely damage some plants. D. The overall environmental benefits that result from driving less exceed those of planting a rooftop garden. 74. Which of the following best describes the organization of the passage? A. A hypothesis is stated and then analyzed. B. A proposal is evaluated and alternatives are explored. C. A viewpoint is established and then defended. D. A thesis is presented and then supported. 75. It can be inferred from the passage that the author would most likely endorse a program that .................................. A. permitted the construction of buildings in city park land provided they have rooftop gardens B. extended discounts on plants to customers who use them to create rooftop gardens C. offered free admission to schools willing to take their students on field trips to the city park D. promised vacation getaways to cooler destinations for those trapped in the city at the peak of summer E. WRITING: I. Finish each of the following sentences in such a way that it means exactly the same as the sentence printed before it: (5 points) 76. Its lack of irregular verbs makes Esperanto a unique language. Unlike………………………………….........................................………………………………………………………….. 77. Mr. Smith knew little about the Internet, so he didn’t invest into any computer companies. Had…......................................................................................................................................................................... 78. The Board of Directors discussed the business in length, but came to no decision. The Board of Directors had ...................................................................................................................................... 79. When she got to the party, everyone was dancing and singing. On.............................................................................................................................................................................. 80. Barbara runs a successful company and she also manages to look after her five children. Not only...................................................................................................................................................................... 81. It is my opinion that you should spend more time practicing English. If................................................................................................................................................................................. 82. Everyone was surprised that the famous actor had very little money when he died. The famous actor....................................................................................................................................................... 83. Success in the academic field depends on your ability to amass qualifications. The more.................................................................................................................................................................... 84. “Get out of my house or I will shoot you”, the woman shouted at the strange man. The woman threatened............................................................................................................................................. 85. What mainly distinguishes man from other animals is the power of speech. Man………….............................................................................................................................................................. II. After having attended a course on computing at ABC Information Technology Center for two weeks, you notice that many things are worse than what were advertised by the Center. Write a letter of complaint of about 150 words to the Director of the Center: (10 points) Your letter should include: BY ĐỖ BÌNH – THPT LIẾN SƠN, LẬP THẠCH, VĨNH PHÚC www.violet.vn/quocbinh72 - the reason for writing - the problems you have encountered - your suggestions and the actions you wish to be taken to resolve the problems SỞ GD&ĐT VĨNH PHÚC ———————— ĐỀ CHÍNH THỨC KỲ THI CHỌN HSG LỚP 1…. - THPT NĂM HỌC 201… - 201… (ĐỀ LUYỆN 13) ĐỀ THI MÔN: TIẾNG ANH ———————————— PART I: PHONETICS. (1, 0 POINT) Choose the word whose stress pattern is different from that of the other words in the same group. Write your answer in the numbered box. 1. A. exposure B. terminal C. utterance D. discipline 2. A. economy B. elaborate C. assassinate D. fascinate 3. A. mountain B. sustain C. contain D. retain 4. A. fulltime B. farmhand C. bookshop D. tradesman 5. A. metropolitan B. entrepreneurial C. hippopotamus D. curiosity 6. A. company B. comfortable C. together D. business 7. A. associate B. formal C. movement D. militant 8. A. Important B. cigarette C. protection D. informal 9. A. introduce B. conversation C. independent D. welcome 10. A. expect B. alone C. liquor D. invite Your answers: 1. 2. 3. 4. 5. 6. 7. 8. 9. 10. PART II: VOCABULARY AND GRAMMAR. (8, 0 POINTS) I. Choose the best answer to fill in each gap. Write your answer in the numbered box. (1, 5 point) 1. In the ___of security, personnel must wear their identity badges at all times. A. requirement B. interests C. demands D. assistance 2. ___ how angry he was he would never resort to violence. A. No matter B. No problem C. Although D. Because 3. We played the game ___ the rules. A. on account of B. ahead of C. according to D. apart from 4. ___ to get through to Jackie for days now. Either she’s away or her phone’s out of order. A. I’ve been trying B. I had tried C. I’m trying D. I tried 5. This is ___ the most difficult job I have ever tackled. A. by rights B. by all means C. by far D. by the way 6. She ___till the early hours listening to pop music. A. took me up B. kept me up C. caught me up D. held me up 7. Please don’t ___yourself out. A sandwich will do. A. let B. put C. leave D. take 8. The chairman requested that ___ A. the members studied more carefully the problem. B. the problem was more carefully studied. C. with more carefulness the problem could be studied. D. the members study the problem more carefully. 9. Smoking is ___ in many companies in our country. A. permitted B. taught C. banned D. stopped 10. After the battle, the ___ soldiers were helped by those who could walk. A. injured B. wounded C. broken D. killed 11. I don’t think this strange new fashion will ………… A. turn up B. care for C. show off D. catch on 12. Their house is ___ near the Cathedral. A. whereabouts B. anywhere C. somewhere D. any place 13. This young tree could not have been damaged by accident. I believe it was done ___ BY ĐỖ BÌNH – THPT LIẾN SƠN, LẬP THẠCH, VĨNH PHÚC www.violet.vn/quocbinh72 A. in fact B. on purpose C. by appointment D. by plan 14. There was hardly ___ money left in my bank account. A. more B. no C. some D. any 15. It is wrongly believed that natural resources will never be used ___. A. off B. out C. away D. up Your answers: 1. 2. 3. 4. 5. 6. 7. 8. 9. 10. 11. 12. 13. 14. 15. II. Give the correct tenses of the verbs in brackets. Write your answer in the box. (1, 5 point) 1. If he ____ (1. not drink) too much last night, he ____ (2. not - be) tired now. 2. I’m sure he must ____ (3. be) at home last night because the door was open when I ____ (4. come). 3. At this time next week, they ____ (5. sit) in the train on their way to Paris. 4. It’s raining. I would rather you ____ (6. close) the window. 5. I don’t know what the matter with him is. He ____ (7. act) funny since you ____ (8. be) away. 6. On the first of next month, he ____ (9. be) in prison for five years. 7. His personal problems seem ____ (10. distract) him from his work lately. 8. He suggested that a final decision ____ (11. make). 9. I don’t know why you ____ (12. always - make) noise in class, Tom. 10. ____ (13. you/visit) many museums when you were in Paris? 11. The car looks very clean. ____ (14. you/wash) it? 12. My best friend, James, was no longer there. He ____ (15. go) away. Your answers: 1. 2. 3. 4. 5. 6. 7. 8. 9. 10. 11. 12. 13. 14. 15. III. Put the correct form of the words in brackets. Write your answer in the box. (2, 0 points) AUTUMN COLORS A new term is rapidly gaining recognition in the American language - a 'leaf peeper' is someone who, in autumn, is on the ____ (1. LOOK) for areas where the leaves of deciduous trees have changed colour. In New England, in the USA, 'leaf peeping' is big business, generating millions of dollars annually. The first report that leaves are changing colour sets off an ____ (2. INVADE) of 'peepers' thus causing serious obstruction on some roads. Thousands of people log on to websites in their ____ (3. EAGER) to find the location of the trees that have foliage in the most ____ (4. GLORY) colours. The popularity of 'leaf tourism' is well established in New England, and the changing colours of autumn provide an annual ____ (5. TALK) point. The colours vary from year to year since the ____ (6. INTENSE) of the colour is ____ (7. DEPEND) on the chemical composition of the dying leaves. After a long dry summer, leaves often turn bright red, while cloudy autumn days will produce less spectacular yellows. If climate change leads to ____ (8. INCREASE) hot, dry summers in the northern hemisphere, then countries in Europe can expect summers not ____ (9. LIKE) those across the Atlantic. They will then enjoy a kaleidoscope of ____ (10. SEASON) colour to rival the deep reds and blazing oranges seen in New England. 2. 3. 4. 5. 1. 7. 8. 9. 10. 6. IV. Find out and correct the mistake in each sentence. Write your answer in the box. (2, 0 points) Example: Thirty hours a week are a heavy work schedule. Answer: C → is A B C D 1. A persimmon tastes best when it is such ripe that it looks wrinkled and almost spoiled. A B C D 2. American pioneers did water systems from logs with holes bored through their centers. A B C D 3. The pituitary gland is a small endocrine gland at the base of the brain that releases many hormones and regulates another endocrine glands. A B C D BY ĐỖ BÌNH – THPT LIẾN SƠN, LẬP THẠCH, VĨNH PHÚC www.violet.vn/quocbinh72 4. In America, the Indians used crude oil for fuel and medicine hundreds of years before the first white settlers arrive. A B C D 5. When radio programs became popular, approximately around 1925, many people stopped attending movies. A B C D 6. Musical comedies, as an American form of entertainment, often take its subjects from America’s present or past. A B C D 7. Of all seashore plants, seaweeds are best able to tolerate long periods out of water, followed by long periods covering by water. A B C D 8. The fruit of the plantain looks much like a banana, and it is not so sweet or so pleasing in flavor. A B C D 9. The viceroy butterfly, an insect that birds like to eat, has a color pattern similar to that of the monarch butterfly, whom birds do not like to eat. A B C D 10. Behavior therapy uses rewards and punishments to encourage patients to act in a way healthier. A B C D Your answers: 1. 2. 3. 4. 5. 6. 7. 8. 9. 10. V. Complete these sentences with proper prepositions or adverbs. Write your answers in the numbered box. (1, 0 point) 1. Don't use pencils. Please write the letter ___ ink. 2. My father made ____ his mind to settle in the South. 3. Minh couldn’t start his motor engine. I think it ran ____ fuel. 4. Time is off now. Please hand ____ your papers. 5. We invited 40 people to the party but only 23 turned ____ 6. That fish of yours has been in the fridge for weeks. It must have gone ____ by now. 7. He needed a model, someone to look ____ 8. Uncle Tom is an idiot, but I only have to put ____ him once a year. 9. I don’t like to make friends with the person who always runs ____ his old friends. 10. These two men’s farms are adjacent ____ each other. Your answers: 1. 2. 3. 4. 5. 6. 7. 8. 9. 10. PART III: READING COMPREHENSION. (6,0 POINTS) I. Read the passage below and decide which answer (A, B, C or D) best fits each gap. Write your answer in the numbered box. (2, 0 points) It is now extremely popular to take a gap year between school and university or university and work and to spend it traveling. There are plenty of reasons to recommend it - travel broadens the mind, you’re (1) ___ young once, life isn’t a rehearsal and so on. And if you don’t do it, you may always regret that you didn’t take the (2) ___. In the end, there’s only one response: well, why not? The idea may have its roots in the 18th century Grand tour once (3) ___ by the young, rich and noble, but it is the middle classes who have turned it (4) ___ something that 200,000 British youngsters do every year. (5) ___ has never been so easy and cheap, with more places open to tourists than ever. Also, the gap year is now (6) ___ by many employers and universities. The States, the Far East and Australia were among the original (7) ___ and although these remain in the top five, young explorers are now going even further. The most far-flung corners of the world are (8) ___ in popularity year by year. About $700 will buy a student ticket (9) ___ for six months that will take you from London to Calcutta, Singapore, Bangkok, Perth, Sydney, Auckland, Fiji, Tahiti, Los Angeles and (10) ___ again. 1. A. merely B. only C. slightly D. simply 2. A. occasion B. moment C. chance D. luck 3. A. undertaken B. gone C. done D. given BY ĐỖ BÌNH – THPT LIẾN SƠN, LẬP THẠCH, VĨNH PHÚC www.violet.vn/quocbinh72 4. A. up B. out C. over D. into 5. A. Voyage B. Travel C. Excursion D. Tour 6. A. received B. stood C. accepted D. admitted 7. A. destinations B. endings C. landings D. terminals 8. A. spreading B. expanding C. enlarging D. growing 9. A. genuine B. valid C. effective D. legal 10. A. now B. here C. back D. then Your answers: 1. 2. 3. 4. 5. 6. 7. 8. 9. 10. II. Read the passage and fill in each blank with ONE suitable word. Write your answer in the numbered box. (2, 0 points) THE LEGEND OF THE ROOT Ginseng is one of the great mysteries of the east. Often referred to as the “elixir of life”, its widespread use in oriental medicine has led to many myths and legends building up around this remarkable plant. Ginseng has featured (1) ____ an active ingredient in oriental medical literature for over 5,000 years. Its beneficial effects were, at one time, (2) ____ widely recognized and praised that the root was said to be worth its weight in gold. (3) ____ the long history of ginseng, no one fully knows how it works. The active part of the ____ (4) is the root. Its full name is Panax Ginseng – the word Panax, (5) ____ the word panacea, coming from the Greek for “all healing”. There is growing interest by western scientists in the study of ginseng. It is today believed that this remarkable plant may (6) ____ beneficial effects in the treatment of many diseases which are difficult to treat with synthetic drug. Today, ginseng is (7) ____ longer a myth or a legend. Throughout the world it is becoming widely recognized that this ancient herb holds the answer to relieving the stresses and ailments of modern living. It is widely used for the treatment of various ailments (8) ____ as arthritis, diabetes, insomnia, hepatitis and anemia. However, the truth behind (9) ____ ginseng works still remains a mystery. Yet its widespread effectiveness shows that the remarkable properties are (10) ____ than just a legend. 1. 2. 3. 4. 5. 6. 7. 8. 9. 10. III. Read the passage, then choose the correct options. Write your answer in the box. (2, 0 points) One of the 7 wonders of the ancient world, the great pyramid of Giza was a monument of wisdom and prophecy built as a tomb for Pharaoh Cheop in 1720 BC. Despite its antiquity, certain aspects of this construction makes it truly one of the truly great wonders of the world. The thirteen – acre structure near the Nile river is a solid mass of stone blocks covered with limestone. Inside are a number of hidden passageways and the burial chamber for the pharaoh. It is the largest single structure in the world. The 4 sides of the pyramid are aligned almost exactly on true north, south, east and west – an incredible engineering feat. The ancient Egyptians were sun worshipers and great astronomers, so computations for the great pyramid were based on astronomical observations. Explorations and detailed examinations of the base of the structure reveal many intersecting lines. Further scientific study indicates that these represent type of timeline of events – past, present and future. Many of the events have been interpreted and found to coincide with known facts of the past. Others are prophesied for future generations and currently under investigation. Many believe that pyramids have supernatural powers, and this one is no exception. Some researchers even associate with its extraterrestrial being of the ancient past. Was it superstructure made by ordinary beings, or one built by a race far superior to any known today. 1. What has research of the base reveal? A. There are racks in the foundation. B. Tomb robbers have stolen pharaoh’s body. C. The lines represent important events. D. A superior race of people built it. 2. Extraterrestrial beings are ____. A. very strong workers. B. astronomers in the ancient times. C. researchers in Egyptology. D. living beings from other planets. 3. What was the most probable reason for providing so many hidden passages? A. To allow the weight of the pyramid to settle evenly. B. To permit the high priests to pray at night. C. To enable the pharaoh’s family to bring food for his journey to the afterlife. D. To keep grave robbers from finding the tomb and the treasure buried with the pharaoh. 4. The word “intersecting” in line 9 is nearest in meaning to ____ A. crossing B. coming C. observing D. cutting BY ĐỖ BÌNH – THPT LIẾN SƠN, LẬP THẠCH, VĨNH PHÚC www.violet.vn/quocbinh72 5. What do the intersecting lines in the base symbolize? A. Architect’s plans for the hidden passage. B. Pathways of the great solar body. C. Astrological computation. D. Dates of important events taking place throughout time. 6. The word “prophesied” in line 12 is closest in meaning to ____ A. said B. armed C. terminated D. foretold 7. What is the best title for the passage? A. Symbolism of the Great pyramid. B. Problems with the construction of great pyramid. C. Wonders of the Great pyramid of Giza. D. Exploration of the burial chamber of Cheop. 8. On what did the ancient Egyptians based their calculation? A. Observation of the celestial bodies. B. Advanced technology. C. Advanced tools of measurements. D. Knowledge of the earth’s surface. 9. Why was the Great pyramid constructed? A. As a solar observatory. B. As a religious temple. C. As a tomb for the pharaoh. D. As an engineering feat. 10. Why is the Great pyramid of Giza considered one of the seven wonders of the world? A. It is perfectly aligned with the 4 cardinal points of the compass and contains many prophecies. B. It was selected as the tomb of the pharaoh Cheop. C. It was built a super race. D. It is very old. Your answers: 1. 2. 3. 4. 5. 6. 7. 8. 9. 10. PART IV: WRITING. (5, 0 POINTS) I. Rewrite each of the sentences without changing its meaning, using the cue given. (1, 0 point) 1. We couldn’t have managed without my father’s money. → If it hadn’t .............................................................................................................. 2. House prices have risen dramatically this year. → There has ............................................................................................................... 3. The only thing they didn’t steal was the television. → They stole ............................................................................................................... 4. He didn’t succeeded in searching for the stolen car. → He tried in vain ...................................................................................................... 5. She started to clean up the house just after the guests had left. → No sooner ............................................................................................................... 6. The boy was about to cry when he was reprimanded by his mother. → The boy was on ....................................................................................................... 7. People believe that the Chinese invented paper in 105 A.D. → Paper........................................................................................................................ 8. As people use a lot of wood-pulp, many trees are cut down. → The more ................................................................................................................. 9. It takes six hours to drive from here to London. → It is .......................................................................................................................... 10. He got so angry that no one dared to say anything. → So ............................................................................................................................ II. Rewrite each of the sentences without changing its meanings, using the word given. (1, 0 point) 1. If you want to see me, come here by six a.m. (PROVIDED) → .................................................................................................................................. 2. He won’t let anyone touch his records. (OBJECTS) → .................................................................................................................................. 3. He didn’t think much of the musical show yesterday. (OPINION) → .................................................................................................................................. 4. The concert was not as good as he had hoped. (EXPECTATIONS) → .................................................................................................................................. BY ĐỖ BÌNH – THPT LIẾN SƠN, LẬP THẠCH, VĨNH PHÚC www.violet.vn/quocbinh72 5. We missed the beginning of the concert because we had overslept. (CONSEQUENCE) → .................................................................................................................................. III. Write an essay (3, 0 points). Facebook dominates the free-time for too many people especially the students. It can have negative effect on their study and the physical development. Do you agree or disagree? Write an essay (250 - 300 words) to express your opinion. SỞ GD&ĐT VĨNH PHÚC ———————— ĐỀ CHÍNH THỨC KỲ THI CHỌN HSG LỚP 1…. - THPT NĂM HỌC 201… - 201… (ĐỀ LUYỆN 14) ĐỀ THI MÔN: TIẾNG ANH ———————————— SECTION I – PHONETICS I.Choose the word whose underlined part is pronounced differently from the rest in the same line. ( 5p) 1.A.several B. suppose C. decisive D. sister 2.A. muddle B. struggle C. bush D. interrupt B. measure C. steak D. stealth 3.A. ready 4.A. wind B. behind C. mind D. find B. challenge C. choice D. chemist 5.A.chalice II. Pick out the word that has the stress syllable differently from that of the other words.(5p) 1.A. application B. certificate C. biology D. security 2. A. university B. punctuality C. agricultural D.mathematician 3. A. identify B. secondary C. luxuriously D. majority 4. A. academic B. engineering C. availabe D.sympathetic 5.A. obligatory B. geographical C. international D. undergraduate SECTION II- VOCABULARY AND GRAMMAR I. Choose the best option ( A, B, C or D) to complete sentence below and write your answers in the answer sheet.( 10p) 1. She refused to eat meat under any ………………… . A. circumstances B. occasion C. opportunity D. reason 2. It was difficult to guess what he ……………. to the news would be. A. feelings B. reaction C. capital D. opinion 3. Though badly damaged by fire, the palace was eventually …… to its original splendour. A. repaired B. renewed C. restored D. renovated 4. He didn’t know anyone at the wedding ………….. than the bride and the groom. A. except B. other C. apart D. rather 5. When you are an old age pensioner, you have to learn to ………… a very small income. A. live on B. live up to C. live out D. live down 6. I felt so much better, my doctor ………….. me to take a holiday by the sea. A. suggested B. considered C. accepted D. advised 7. He said that the plane had already left and that I …………….. arrived an hour earlier. A. must have B. had to C. should have D. was supposed to 8. Alice didn’t expect ……………. to Bill’s party. A. to ask B. being asked C. to be asked D. asking 9.The restaurants on the island are expensive, so it’s worth ……………… a packed lunch. A. take B. to take C. taking D. taken 10. When the alarm went off, Mick just turned ………..…. and went back to sleep. A. down B. over C. up D. out II.Put the verb in brackets into the correct tense.( 10p) 1.When he ( not arrive) ……….. by 6.00, I ( know) ………. He ( miss) ………… the bus. 2. Something ( tell) ………….. me that you ( not listen) …………… to a single word I (say) ……….. in the past tense minutes. 3. Same ( not receive) ……………..…… the parcel the last time I ( speak) …………….. to him. 4. When you ( feel) ……….… hungry, room service ( bring) ………….. you whatever you want. III. Give the correct forms of the words in brackets to complete the blank space. ( 10p) 1.The judges describe Jones as a ….. criminal who was a danger to members of the public. HARD 2. Rescue team held out little hope of finding other …………. SURVIVE 3. He works for UNESCO in a purely ………….. role. ADVICE BY ĐỖ BÌNH – THPT LIẾN SƠN, LẬP THẠCH, VĨNH PHÚC www.violet.vn/quocbinh72 4. …………… of the new system will take several days. INSTALL 5. This type of behaviour is no longer …………. acceptable. SOCIETY 6. Watching television can be very …………….. EDUCATION 7. Teachers must keep a record of students’ ……………….. ATTEND 8. Our school set up a project to ……………. the library system. COMPUTER 9.The breakdown of the negotiations was not ……………….. EXPECT 10. Those countries are ……. on other countries for most of their food. DEPEND IV. Fill in each gap with a suitable preposition. (10 points) 1.……………… behalf of the department I would like to thank you all. 2. Many people expressed a strong preference …………. the original plan. 3. They were refused entrance ………… the exhibition . 4. He graduated …………….. York with a degree in Psychology. 5. We had an argument ………….. the waiter about the bill. 6. She complimented him ………….. his exellent German. 7. She sacrificed everything ……………. her children. 8. Sit down and make yourself ……………. home. 9. He works away …………… home during the week. 10. The searchers spread ………….. to over the area faster. SECTION III – READING I.Read the following passage carefully and then answer the questions below.(10p) Traditionally the United States has been described as a melting pot, a place where the previous identities of each immigrant group are melted down to create an intergrated, uniform society. Since the 1960s, many Americans have rejected the melting pot metaphor in favour of the image of the mosaic, a picture created by assembling many small stones or tiles. In a mosaic, each peace retains its own distinctive identity, while contributing to a larger design. Advocates of the mosaic metaphor assert that it better represents the diverse multicultural society of the United States. Today, may Americans value their immigrant heritage as an important part of their identity. More recent immigrant groups from Asia, such as Vietnamese Americans and Korean Americans, have established communities alongside those populated by the descendants of European immigrants, such as French Americans, German Americans, Irish Americans and Italian Americans. 1.Why has the United States often been described as a melting pot? 2. Why have people been in favour of the image of the “ mosaic”? 3. What does the word “mosaic” mean? 4. What have immigrant groups from Asia done to maintain their identity? 5. Name some recent immigrant groups from Asia in the United States? II.Read the following passage and then choose the most suitable word for each space. (10 p) CAR CATCHES FIRE Three people jumped ……………( 1) a car on a busy Oxford road after a fire started under the bonnet this morning. They were just able to rescue their possessions before the car burst into …………..(2). Mr. Peter Collins, 25, of Wey Road, Berinsfield …………….(3) his Avenger astate car home ………….(4) work with two friends when he noticed smoke coming into the car. He stopped, …………(5) was unable to open the bonnet. He poured a bottle of water over the radiator where the smoke was coming from, but could not put out the flames. He then ………… (6) to get fire extinguishers from a nearby…………(7) , but ………….(8) also failed to have any effect. ……….(9) he telephoned for a ……..(10) , but by the time it arrived, the car was totally burned out. 1. A. into B. on C. out of D. over 2. A. flames B. petrol C. smoke D. sparks 3. A.drove B. has driven C. was driving D. was driven 4. A. to B. from C. at D. in 5.A. but B. because C. although D. so 6. A. wandered B. jumped C. walked D. ran 7.A. hospital B. garage C.library D. swimming pool 8. A. this B. those C. that D. these 9. A. At length B. Finally C. At the end D. Terminally 10. A. police car B. ambulance C. taxi D. fire engine III.Read the passage below carefully. Then choose the correct answer: A, B, C or D that best answers the question about the passage. (5p) When we were in England last year, I went fishing with my friend, Peter. Early in the morning we were sitting quietly by the side of the lake when we had an unpleasant surprise. We saw a duck come along with three ducklings BY ĐỖ BÌNH – THPT LIẾN SƠN, LẬP THẠCH, VĨNH PHÚC www.violet.vn/quocbinh72 padding cheerfully behind her. As we watched them, there was a sudden swirl in the water. We caught a glimpse of the vicious jaws of a pike- a fish which is rather like a freshwater shark- and one of the ducklings was dragged below the surface. This incident made Peter furious. He vowed to catch the pike. On three successive mornings we returned to the vicinity and used several different kinds of bait. On the third day Peter was lucky. Using an artificial frog as bait, he managed to hook the monster. There was a desperate fight but Peter was determined to capture the pike and succeeded. When he had got it ashore and killed it, he wieghed the fish and found that it scaled nearly thirty pounds- a record for that district. 1.Why do you think Peter was sitting quietly by the lake ? A. He was watching the ducks. C. He was waiting for the pike to appear. B. He wasn’t very talkative. D. He was fishing 2. To what does surprise in line 3 probably refer? A. to the duck. B. to the ducklings. C. to the action of the pike. D. to the time of the day. 3. What were Peter’s feelings about the incident two days later? A. He caught and killed the pike. C. He vowed that he would catch the remaining ducklings B. He remained determined to catch the pike. D. He caught a frog and used it as bait for the pike. 4. How much was the pike worth? A. about thirty pounds. B. about two hundred and forty dollars. C. the passage contains no information on this point. D. the passage says that the fish scaled nearly thirty pound. 5. Which of the following titles best sums up the whole passage? A. Mysterious disappearance of ducklings. C. Record pike caught by an angry fisherman. B. Revenge on a duck. D. Huge pike caught by fisherman after desperate struggle at sea. Section IV – writing I. Finish each of the following sentences in such a way that it means exactly the same as the sentence printed before it (5 points). 1. But for your unsuspected talents as a mechanic, he would have been stranded there for hours. - If ………………………………………………………………………………………… 2. Let’s invite the Browns to the party on Sunday. - He suggested …………………………………………………………………………….. 3. The only thing that prevented the passing of the bill was the death of the Prime Minister. - Had it not ..………………………….………………………………………………………. 4. I’ve forgotten that commentator’s name but he’s very well-known. - That commentator, ………………………………………………………………………….. 5. I only recognised him when he came into the light. - Not until ……………………………….…………………………………………………….. II. Using the word given and other words, complete the sencond sentence so that it has a similar meaning to the first sentence. (5points) 1.There’s no point in asking George to help. (WORTH) - It ………………………………………………………….Gerge to help. 2. The last time I saw him was when I lived in London. ( SINCE) - I …………………………………….. I lived in London. 3. I don’t normally go into town by car. ( USED) - I ……………………………………………into town by car. 4. Nobody plays this piece as beautifully as he does. (MORE) - He plays this piece………………………………………………….else. 5. I couldn’t go to work because of the transport strike. ( PREVENTED) - I ……………………………………………to work by the transport strike. III. Using the cues below to complete each sentence of the following passage. (5p) 1.Teacher/ Day/ Vietnam/ fall/ twenty/ November/ each year. ........................................................................................................................................................................ 2. General/ it / be/ mean/ schoolchildren/ show/ appreciation/ teacher/ who/ guide/ in/ study. ……………………………………………………………………………………………………………… 3. In / a/ time/ forget/ bad/ experience/ student/ have/ teacher/ scold/ punish. ……………………………………………………………………………………………………………… 4. Students/ give/ teacher/ flower/ small/ gift. ……………………………………………………………………………………………………………… 5. Small/ party/ hold/ and/ there/ be/ atmosphere/ fun/ amusement/ school. BY ĐỖ BÌNH – THPT LIẾN SƠN, LẬP THẠCH, VĨNH PHÚC www.violet.vn/quocbinh72 ……………………………………………………………………………………………………………… IV. Writing an essay (10p) Living in a tranditional family of three or four generations has both advantages and disadvantages. What do you think of that matter? Write an essay of about 180- 200 words to support your points. SỞ GD&ĐT VĨNH PHÚC ———————— ĐỀ CHÍNH THỨC -----------THE END------------KỲ THI CHỌN HSG LỚP 1…. - THPT NĂM HỌC 201… - 201… (ĐỀ LUYỆN 15) ĐỀ THI MÔN: TIẾNG ANH ———————————— I/ READING: PART 1: Choose the word or phrase ( A, B, C or D). Write your answers in the spaces bellow. Number 0 is an example ( 2 ms ) 0. The … A …….-hour accident caused chaos for the local motorists yesterday evening. A. rush B. high C. traffic D. heavy 1. The unemployment rate of the area ……………..every year until 2003 then started to fall. A. raised B. rose C. dropped D. decreased 2. It is gravity……………………objects towards the earth. A. pulling B. to pull C. what pulls D. that pulls 3. …………problems in sailing in tropical seas is the coral reefs. A. One of the biggest B.The biggest one C. Of the biggest one D. There are the biggest 4. ……………foreign languages requires great effort. A. Learn B. Learning C. To learn it D. Learnt 5. "I am tired" “ ……… ” A. Me too B. Also me C. For me the same D. I also 6. I’m really not satisfied with the way you have …………………the situation. A. dealt B. handled C. done D. fingered 7. We may win, we may lose- It is just the ………………..of the draw. A. strike B. odds C. chance D. luck 8. War is ……………as open-armed conflict between countries or factions within countries. A. delineated B. declared C. defaulted D. defined 9. I hope you won’t take it…………………if I suggest an alternative remedy. A. offence B. amiss C. upset D. most 10. Claims for compensation could ………………… run into billions of pounds. A. far B. much C. well D. most YOUR ANSWERS: 0 ………….A…… 1…………………2……………………….3……………………….4……………………5……………….. . 6…………………7……………………….8……………………….9…………… ………10……………….. PART 2: Complete the spaces with A, AN, THE or (No article). Write your answers in the spaces bellow. Number 0 is an example (1 m) 0: Jason’s father bought him ………..bicycle that he had wanted for his birthday. Countries around (1) ……………….world have begun to deal with (2) …………………problem in various ways. Some countries, in (3) …………….effort to circumvent the problem, have allocated large amounts of land to animal reserves. They then charge admission prices to help defray the costs of maintaining (4) ……… parks, and they often must also depend on (5) ……………..world organizations for support. YOUR ANSWERS: 0 ………….the…… 1…………………2……………………….3……………………….4……………………5……………….. PART 3: Give the correct preposition. Write your answers in the spaces bellow. Number 0 is an example (1 m) 0. It is very good ……….you to volunteer to help the poor. 1. I’m sure you are capable …………passing the exam. 2. He is not aware ………….the dangers of smoking. 3. Many Vietnamese students have difficulty …………… learning English. 4. I get very annoyed ………………..people who don’t queue at bus stop. 5. If you have any complaints , tell me ………my face. I can’t stand people who do things behind my back. YOUR ANSWERS: 0 ………….of…… 1…………………2……………………….3……………………….4……………………5……………….. . PART 4: Give the correct forms and tenses of the verbs in brackets. Write your answers in the spaces bellow. Number 0 is an example (1 m) BY ĐỖ BÌNH – THPT LIẾN SƠN, LẬP THẠCH, VĨNH PHÚC www.violet.vn/quocbinh72 0. Her (get) ……………..sick was not expected. 1. She took a risk investing money with them. I’d rather she (not do) ………………..it. 2. He (not catch) …………………cold if he would not have walked in the rain so long. 3. I'll wait until he (finish)………………………….. his novel. 4. By next month I (leave) …………………………….for India. 5. He volunteered ( help) …………………… us. YOUR ANSWERS: 0 ………….getting…… 1…………………2……………………….3……………………….4……………………5……………….. . PART 5: Choose the word which best fits each gap of the passage. Write your answers in the spaces bellow. (2ms) Although all Americans do not (1) ....... the same way, their speech has enough in common that American English can be recognized as a (2) …… of English distinct from British English, Australian English and other national varieties. American English has grown up with the country. It began to (3) ........ from British English during its colonial beginnings and (4) ........ regional differences and ethnic flavor. Today it (5) …….other languages and other varieties of English because it is the medium by which the attractions of American culture - its literature, (6) ........pictures and television programs - are transmitted to the world. All speakers of English (7)........ a common linguistic system and a basic set of words. But American English (8) ......... from British English, Australian English and other national varieties in many of its pronunciations, words, spellings and grammatical (9) ........ Words or phrases of American (10) ....... and those used in America but not so much elsewhere, are called Americanisms. 1. A. say B. speak C. talk D. tell 2. A. alteration B. change C. variety D. variation 3. A. diverge B. divert C. digress D. depress 4. A. enquired B. inquired C. required D. acquired 5. A. affluence B. fluency C. influences D. nuances 6. A. motion B. moving C. movement D. removing 7. A. have B. bear C. tear D. share 8. A. differentiates B. differs C. changes D. alters 9. A. constructions B. building C. points D. syntax 10. A. birth B. source C. origin D. tradition YOUR ANSWERS: 1…………………2……………………….3……………………….4……………………5……………….. . 6…………………7……………………….8……………………….9……………………10…… ………….. . PART 6: Fill in each blank with one suitable word. Write your answers in the spaces bellow. (2ms) Is Photography Dead? For a long time in the past photography was not regarded as an art. It was simply a skill and it was criticized for being too mechanical and not creative enough. At last, however, photography is now accepted as a unique and very important (1)........of art. The photograph's claim to be an objective record of reality is now seriously challenged, and the important function of photography in modern-day society is consequently (2) ......threat. The threat has suddenly become all the more serious as more and more photographers are (3)....... to the new technology which computers offer. Moreover, a (n) (4) ........ number of colleges have now begun to offer (5) ....... in computer imaging. All these developments (6)........ a disturbing question. Is photography, as we know (7).......dead? In spite of its complete transformation by new technological developments, however, photography will continue to play a (8) ........role in our culture. Although it may no longer (9)........ to be realistic, modern photography can continue to provide us with fresh visral (10) .........about ourselves and the world in which we live. YOUR ANSWERS: 1…………………2……………………….3……………………….4……………………5……………….. . 6…………………7……………………….8……………………….9……………………10…… ………….. . PART 7: Read the passage below carefully and choose the best answer. Write your answers in the spaces bellow. (2ms) Langston Hughes was one of the greatest American writers of the twentieth century. He was born in Joplin, Missouri, and moved to Cleveland at the age of 14. Several years later he spent one year in Mexico before attending Columbia University in New York. For a few years after that, he roamed the world as a seaman, visiting ports around the world and writing some poetry. He returned to the United States and attended Lincoln University, where he won the Witter Bynner Prize for undergraduate poetry. After graduating in 1928, he traveled to Spain and to Russia with the help BY ĐỖ BÌNH – THPT LIẾN SƠN, LẬP THẠCH, VĨNH PHÚC www.violet.vn/quocbinh72 of a Guggenheim fellowship. His novels include Not without Laughter (1930) and The Big Sea (1940). He wrote an autobiography in 1956 and also published several collections of poetry. The collections include The Weary Blues (1926), The Dream Keeper (1932), Shakespeare in Harlem (1942), Fields of Wonder (1947), One Way Ticket (1947), and Selected Poems (1959) A man of many talents, Hughes was also a lyricist, librettist, and a journalist. As an older man in the 1960s, he spent much of his time collecting poems from Africa and from African-Americans to popularize black writers. Hughes is one of the most accomplished writers in American literary history, and he is seen as one of the artistic leaders of the Harlem Renaissance, the period when a neighborhood that was predominantly black produced a flood of great literature, music, and other art forms depicting daily city life for African-Americans. 1. What is the main topic of this passage? A. The life of Langston Hughes B. The Harlem Renaissance C. African-American writers D. American twentieth-century writers 2 .Where was Langston Hughes born? A. Spain B. New York C. Missouri D. North Carolina 3. As used in the passage, which of the following words could best replace the word "ports"? A. Islands B. Ships C. Friends. D. Harbors 4. To which of the following movements might Shakespeare in Harlem refer? A. The Civil War B. The Harlem Riots C. The Harlem Renaissance D. The Civil Rights Movement 5. What provided Hughes with assistance for his travel to Spain and Russia? A. His job as a reporter B. His career as a soldier C. A literary fellowship D. A college study program 6. The word "talents" in the passage could be replaced by which of the following? A Desires B. Abilities C. Strategies D. Careers 7. According to the author, what did Hughes do during the later years of his life? A. Write short stories B. Popularize African-American writers C. Advocate racial equality . D. Write about life in Harlem 8. Which of the following could best replace the word "accomplished" as used in the passage? A Successful B. Prolific C. Brilliant D. Imaginative 9. Which of the following can best substitute for the word "depicting" in the passage? A. Congratulating Blessing C. Screening D. Portraying 10. According to the passage, Langston Hughes was all of the following EXCEPT: A A novelist B. A poet C. A historian D. A journalism. YOUR ANSWERS: 1…………………2……………………….3……………………….4……………………5……………….. . 6…………………7……………………….8……………………….9……………………10…… ………….. . PART 8: Fill each blank with a suitable word or phrase. Write your answers in the spaces bellow. (2ms) 1. The car can't have broken down. I........................serviced last week. 2. If I..........................situation, I certainly wouldn't have had the courage to do what you did. 3. Although he had a difficult time during the war, it............................harm. 4. It's no..........................the cold and wet, you'll just have to put up with it. 5. I won't have .............................. their cars in the street outside the entrance to my house! 6. In times of hardship we have to learn to do.................................some basic necessities. 7. I'm................................death of spiders! 8. I heard the thunder...............................in the distance. 9. He's been...................................weather for ages and still isn't back at work. 10. We made the uniforms ourselves as we couldn't ………………..made. YOUR ANSWERS: 1…………………2……………………….3……………………….4……………………5……………….. . 6…………………7……………………….8……………………….9……………………10…… ………….. II. WRITING: PART 1: Finish each of the following sentences in such a way that it means exactly the same as sentence printed before it. (2ms) EXAMPLE: “They would be very disappointed if you did not come,” the man said to his daughter. The man told his daughter (that) they would be very disappointed if she did not come. 1. “I am happy you have passed the final exams. Congratulations!” Tom said to me. Tom congratulated ………………………………………………………………………………. 2. You ‘d better not swim too far from the shore,” the lifeguard said to us The lifeguard advised …………………………………………………………………………… BY ĐỖ BÌNH – THPT LIẾN SƠN, LẬP THẠCH, VĨNH PHÚC www.violet.vn/quocbinh72 3. Birth rates have fallen sharply recently. There ……………………………………………………………………………………………. 4. I didn’t hear the news until the next day. It was not ………………………………………………………………………………………… 5. Although it rain torrentially all day, we all enjoyed the excursion. Despite …………………………………………………………………………………………. PART 2:Write a new sentence using the word in brackets. Do not alter the word in any way (2ms) 1. He may choose to take early retirement. (option) …………………………………………………………………………… 2. He didn’t take any part in the conversation. (contribute) ……………………………………………………………………………. 3. We lose our way because the signposts were confusing. ( which) …………………………………………………………………………….. 4. He likes to be addressed as “Professor”. (call) …………………………………………………………………………….. 5. She always has a good relationship with the children. (gets) …………………………………………………………………………….. PART 3: Write an essay about 200 words on the following topic: What measures do you take in order to reduce air and water pollution in your city? (3ms) SỞ GD&ĐT VĨNH PHÚC ———————— ĐỀ CHÍNH THỨC KỲ THI CHỌN HSG LỚP 1…. - THPT NĂM HỌC 201… - 201… (ĐỀ LUYỆN 16) ĐỀ THI MÔN: TIẾNG ANH ———————————— B. PHONETICS (5 points). Choose the word that has a different stress pattern from the others. 16. A. community B. developing C. conditioner D. interested 17. A. continue B. importance C. different D. directed 18. A. medicines B. opposite C. pollution D. capable 19. A. preservation B. inspiration C. disposable D. popularity 20. A. exhausted B. atmosphere C. suspect D. computer C. GRAMMAR- VOCABULARY-LANGUAGE FUNCTIONS I. Choose the word, phrase or expression which best completes each sentence (15 points). 21. Hoa: “Are you going to buy a new computer or just continue using the old one?” Mary: “_________” A. Yes, I am. B. Yes, I’d like one. Thank you. C. That’s impossible. I can’t afford a new one. D. Neither. I’m going to lease one. 22. Mr. Black: “What a lovely house you have!” Mr John: “_________” A. No problem B. Thank you. Hope you will drop in. C. Of course not, it’s not costly D. I think so. 23. You have never been to Italy, ________? A. have you B. haven’t you C. did you D. had you 24. You can’t tell what someone is like just from their ________. A. character B. looking C. appearance D. personality 25. “How is it going?” - “________” A. By bike B. Not much C. It sounds better D. Mustn’t grumble 26. ________ a dentist, Mike is very concerned about having healthy teeth. A. Because B. He is C. As D. That he is 27. ________, you need to achieve a score of 60% or more. A. To pass this test B. For being passed this test C. In order pass this test D. So that to pass this test 28. As a famous person ________ many children admire, it is important for her to act responsibly. A. whose B. whom C. which D. when 29. The brochure says that the hotel has a great ________ of the sea. A. appearance B. look C. sight D. view 30. Our new coach is popular ________ the whole team. BY ĐỖ BÌNH – THPT LIẾN SƠN, LẬP THẠCH, VĨNH PHÚC www.violet.vn/quocbinh72 A. with B. to C. by D. for 31. As soon as you ________ that, I’d like you to go to bed. A. have done B. did C. will do D. will have done 32. Margaret was slow at school, but she went on ________ Prime Minister. A. being B. to be C. having been D. to have been 33. In 1870, ________, John D. Rockefeller and others created the Standard Oil Company. A. in spite of oil prices fluctuated B. despite fluctuating oil prices C. but the oil prices fluctuated D. oil prices were fluctuating 34. They’re staying in rented accommodation for the time ________. A. going B. making C. doing D. being 35. “I have an idea. Let’s go for a swim on Sunday afternoon”. - “________” A. OK, what time? B. You’re kidding C. I know D. I’m sure II. Give the correct form of the words in brackets (10 points). 36. We found it ____________ (thrill) to your wonderful news. 37. He left the room without any ____________ (explain). 38. He didn’t feel happy because he worked ____________ (success). 39. Many people expressed ____________ (disagree) with the whole idea. 40. There was a ____________ (wide) dissatisfaction with the government’s policies. 41. Her health has ____________ (bad) considerably since we last saw her. 42. A lot of plants and animals could be used as medicines against cancer, AIDS, heart diseases and other ____________ (sick). 43. He was ____________ (information) of the consequences in advance. 44. I was kept ____________ (wake) last night by the noise from a party in the flat above. 45. This road is so bad that it needs ____________ (surface). III. Choose the underlined word or phrase in each sentence that needs correcting (5 points). 46. Last week unless my mother had had enough money, she would have bought that toy for me. A B C D 47. It often takes me about fifteen minutes to go to work from here by foot. A B C D 48. Those people say that it is such polluted air that they can’t breath, don’t they? A B C D 49. It is noisy enough in this room, so I would rather you stop shouting like that. A B C D 50. Dr. Roberts, the first woman to be elected president of the university, is intelligent, capable and A B awareness of the problem to be solved. C D D. READING I. There are 5 blanks in the passage below. From the words given in the box, choose the most suitable for each blank. There are more words than blanks, so you don’t need all of them. (5 points). A. terrible B. size C. hope D. wrong E. loose F. problem G. want Choosing clothes can be difficult. Some people (51)________ to be fashionable, but they don’t want to look exactly like everybody else. Not all clothes are suitable for work or school, perhaps because they are not formal enough, or simply not comfortable. It is easy to buy the (52)_________ size, and find that your trousers are too tight, especially if you are a little bit overweight. Very (53)________ clothes make you feel slim, but when they have shrunk in the washing machine, then you have the same (54)________! If you buy light cotton clothes, then they might not be warm enough for winter. If your shoes are not tight, and if you aren’t dressed for the cold, you might look good, but feel (55)________! II. Read the following passage and decide which option A, B, C or D best fits each space. (10 points) Schools in the United States have not always had a large number of libraries. As (56)________as 1958 about half of the public schools in the United States had no libraries at all. The number of public school libraries increased dramatically (57)_______ the federal government passed the Elementary and Secondary Education Act of 1965, ( 58)_______ BY ĐỖ BÌNH – THPT LIẾN SƠN, LẬP THẠCH, VĨNH PHÚC www.violet.vn/quocbinh72 provided funds for school districts to improve their education programs and facilities, including their libraries. (59) ________, many educators claim that since the legislation was passed federal spending has not increased sufficiently to meet the rising (60)______ of new library technologies such as computer databases and Internet access. Because the federal government provides only limited funds to schools, individual school districts (61) _______ on funds from local property taxes to meet the vast majority of public school expenses. Therefore, the libraries of the public schools tend to reflect the (62) ______ capabilities of the communities in which they are located. Districts in wealthy suburbs often have fully staffed libraries (63) _______ abundant resources, spacious facilities, and curricular and instructional support. In (64) ______, school districts in many poor areas house their libraries in ordinary classrooms or in small rooms. The libraries in such areas are generally staffed by volunteers, who organize and (65) ______ books that are often out-of-date, irrelevant, or damaged. 56. A. freshly B. recently C. frequently D. newly 57. A. though B. with C. during D. when 58. A. that B. who C. which D. this 59. A. Nevertheless B. Therefore C. Consequently D. Otherwise 60. A. fine B. fee C. cost D. sum 61. A. go B. come C. rely D. stay 62. A. educational B. economical C. political D. financial 63. A. for B. with C. on D. by 64. A. country B. converse C. contrast D. conflict 65. A. attain B. obtain C. contain D. maintain III. Read the passage and choose the correct answer A, B, C or D(10 points). Before the mid-nineteenth century, people in the United States ate most foods only in season. Drying, smoking and salting could preserve meat for a short time, but the availability of fresh meat, like that of fresh milk, was very limited; there was no way to prevent spoilage. But in 1810, a French inventor named Nicolas Appert developed the cooking-andsealing process of canning. And in the 1850’s an American named Gail Borden developed a means of condensing and preserving milk. Canned goods and condensed milk became more common during the 1860’s, but supplies remained low because cans had to be made by hand. By 1880, however, inventors had fashioned stamping and soldering machines that mass-produced cans from tinplate. Suddenly all kinds of food could be preserved and bought at all times of the year. Other trends and inventions had also helped make it possible for Americans to vary their daily diets. Growing urban population created demand that encouraged fruit and vegetable farmers to raise more produce. Railroad refrigerator cars enabled growers and meat packers to ship perishables great distances and to preserve them for longer periods. Thus, by the 1890’s, northern city dwellers could enjoy southern and western strawberries, grapes, and tomatoes, previously available for a month at most, for up to six months of the year. In addition, increased use of iceboxes enabled families to store perishables. As easy means of producing ice commercially had been invented in the 1870’s, and by 1900 the nation had more than two thousand commercial ice plants, most of which made home deliveries. The icebox became a fixture in most homes and remained so until the mechanized refrigerator replaced it in the 1920’s and 1930’s. Almost everyone now had a more diversified diet. Some people continued to eat mainly foods that were heavily in starches or carbohydrates, and not everyone could afford meat. Nevertheless, many families could take advantage of previously unavailable fruits, vegetables, and dairy products to achieve more varied fare. 66. What does the passage mainly discuss? A. Causes of food spoilage B. Commercial production of ice C. Population movements in the nineteenth century D. Inventions that led to changes in the American diet 67. The phrase “in season” in line 1 refers to A. a particular time of year B. a kind of weather C. an official schedule D. a method of flavoring 68. During the 1860’s, canned food products were A. unavailable in rural areas B. available in limited quantities C. shipped in refrigerator cars D. a staple part of the American diet. 69. It can be inferred that railroad refrigerator cars came into use A. before 1860 B. before 1890 C. after 1900 D. after 1920 70. The word” them” in line 13 refers to A. refrigerator cars B.growers C. perishables D. distances. 71. The word” fixture” in line 18 is closest in meaning to BY ĐỖ BÌNH – THPT LIẾN SƠN, LẬP THẠCH, VĨNH PHÚC www.violet.vn/quocbinh72 A. commonplace object B. substance C. luxury item D. mechanical device 72.The author implies that in the 1920’s and 1930’s home deliveries of ice A. increased in cost B. occurred only in the summer C. decreased in number D. were on an irregular schedule 73. The word “ Nevertheless” in line 21 is closest meaning to A. occasionally B. however C. therefore D. because 74. Which of the following types of food preservation was NOT mentioned in the passage? A. Drying B. Chemical additives C. Canning D. Cold storage 75. Which of the following statements is supported by the passage? A. Most farmers in the United States raised only fruits and vegetables. B. People who lived in cities demanded home delivery of foods. C. Tin cans and iceboxes helped to make many foods more widely available. D. Commercial ice factories were developed by railroad owners E. Writing I. Finish each of the following sentences in such a way that it means exactly the same as the sentences printed before. (5 points) 76. Having nothing else to do, we decided to go for a walk. - Since............................................................................................................................. 77. She was not only bad-tempered but also very lazy. - As well ........................................................................................................................ 78. “ Nothing will persuade me to apply for that kind of job.” She said. _ She flatly....................................................................................................................... 79. The school I studied at last year was better than this one. - This school isn’t............................................................................................................. 80. You must leave now, or you’ll miss the bus. - You’ll miss...................................................................................................................... II. Use the suggested words and phrases to write complete sentences of a letter. (10 points) Dear Sir or Madam. 81. I/ write/ complain/ hair drier/ buy/ your shop/ last Saturday/ and/ treatment/ I receive/ when/ I try/ return/ a few days later. 82.I buy/ hair drier/ Wednesday, November 22nd. 83. first time/ try/ use/ handle become/ extremely hot/ and within a few minutes/ part/ plastic casing/ begin/ melt. 84. I turn/ off/ immediately/ return/ with/ receipt/ your shop/ Saturday. 85. I explain/ situation/ one/ assistants/ ask/ money back/ but / be told/ speak/ you. 86. Unfortunately you/ not available/ that day/ I/ write instead. 87. I enclose/ hair drier/ copy of/ original receipt. 88. Please send/ full fund/ soon/ possible. Yours faithfully, III. Write a paragraph of about 120 words on the following topic: ( 10 points) What club do you want to be founded in your school? Why? ---------------THE END--------------SỞ GD&ĐT VĨNH PHÚC ———————— ĐỀ CHÍNH THỨC KỲ THI CHỌN HSG LỚP 1…. - THPT NĂM HỌC 201… - 201… (ĐỀ LUYỆN 17) ĐỀ THI MÔN: TIẾNG ANH ———————————— Part I: Choose the best answer among A, B, C or D. 1. Our holiday was_________ by the weather. A. spoilt B. damaged C. overcome D. wasted 2. The_________ charged by the architect for the plans of the new building were unusually high. A. hire B. price C. fees D. sum 3. He_________ his son of the dangers of driving too fast in his new car A. warned B. remembered C. threatened D. concerned 4. The child was_________ by a lorry on the safety crossing in the main street. A. knocked out B. run across C. run out D. knocked down 5. When Ali arrived in London he spent all his time_________ and visited all the important museums and buildings. BY ĐỖ BÌNH – THPT LIẾN SƠN, LẬP THẠCH, VĨNH PHÚC www.violet.vn/quocbinh72 A. sight-seeing B. traveling C. looking D. touring 6. If you want a cheap air ticket you must_________ well in advance. A. book B. engage C. reserve D. buy 7. His sister was full of_________ for the way in which he had so quickly learned to drive a car. A. pride B. admiration C. surprise D. jealousy 8. He asked if we would_________ to share the room. A. accept B. consider C. agree D. approve 9. I wondered whether you would like to_________ to the theater tomorrow. A. visit B. go away C. go out D. walk out 10. _________ I would like to say how pleased I am to be here. A. Primarily B. Foremost C. Earliest D. First 11. The independent arbitrator managed to_________ the confrontation between the union and the employers. A. refuse B. confuse C. refute D. defuse 12. When I heard the footsteps behind me I was_________ that I would be attacked. A. horrified B. terror-struck C. terrorized D. terrified 13. His illness made him_________ of concentration. A. incompetent B. unable C. incapable D. powerless 14. Has the committee_________ a decision yet? A. done B. made C. arrived D. voted 15. I am a bit hungry. I think_____________ something to eat. A. I’ll have B. I’ll be having C. I’m going to have D. I’m having 16. What do you plan to do when you_____________ your course at college? A. finish B. will finish C. have finished D. is going to finish 17. Where_____________? Which hairdresser did you go to? A. did you cut your hair B. have you cut your hair C. did you have cut your hair D. did you have your hair cut 18. ‘Shall I stay here?’ ~ ‘I’d rather_____________ with us’. A. you come B. you to come C. you would come D. you came 19. I_____________ saying what I think. A. believe B. believe in C. believe for D. believe when 20. Somebody ran in front of the car as I was driving. Fortunately I_____________ just in time. A. could stop B. could have stopped C. managed to stop D. must be able to stop Part II: The passage below contains 10 mistakes. Underline the mistakes and write their correct forms in the space provided in the column on the right. (0) has been done as an example. Traditional, mental tests have been divided into two types. 0. traditional → traditionally Achievement tests are designed to measure acquiring skills and knowledge, 1. _____________________ particularly those that have been explicitness taught. The proficiency exams 2. _____________________ required by few states for high school graduation are achievement tests. 3. _____________________ Aptitude tests are designed and measure a person’s ability to acquire new 4. _____________________ skills but knowledge. For example, vocation aptitude tests can help you 5. _____________________ decide whether you would do better like a mechanic or musician. However, 6. _____________________ all mental tests are in some sense achievement tests because they 7. _____________________ assumption some sort of past learning or experience with certainly objects, 8. _____________________ words, or situations. The difference between achievement and aptitude tests 9. _____________________ is the degree and intention use. 10. _____________________ Part III: Give the correct form of the words in brackets to fill in the blanks. Dark black clouds in a dull sky meant one thing and one thing only: there was going to be a (1.thunder)….. Not one of us had brought an umbrella, or even a (2.rain)……. So when Jack suggested we should go to a museum, we all agreed immediately. As we had been (3. shop)………all morning we were now feeling very tired, it would be a (4. pleasant)………...to sit down. We took a bus and arrived just as large shops of rain were beginning to fall. The museum was quite (5.desert)……and very peaceful. We sat down in the main hall and listened to the rain (6. beat)…….against the windows. BY ĐỖ BÌNH – THPT LIẾN SƠN, LẬP THẠCH, VĨNH PHÚC www.violet.vn/quocbinh72 Suddenly, there was a great (7. disturb)……..at the (8. enter)……… a large party of schoolboys were (9. lead) ………in by a teacher. The poor man was trying to keep them quiet and threatening to punish them, but they did not pay the (10.slight)……. attention. Section B: Reading Part I: Read the following passage and choose the best answer to fill in the blanks. Fill each numbered blank with one suitable word from the list given below. The shark is a meat- eating fish and one of the most feared animals of the sea. Scientists (1)………… about 250 species of fish as sharks. These fish live in oceans (2)………..the world, but they are most common in warm seas. Sharks (3)………….greatly in size and habits. Whale sharks, the largest kind of shark, may grow 60 feet long. A whale shark weighs up to 15 tons, more than twice (4)…………….much as an African elephant. The smallest shark may (5)…………..only 4 inches long and weigh less than 1 ounce. Some kinds of sharks live in the depths of the ocean, but (6)……………are found near the surface. Some species live in coastal waters, but others (7)……………far out at sea. A few species can even live in (8)…………..water. All sharks are carnivores (meat- eaters). Most of them eat (9)…………fish, including other sharks. A shark’s only natural enemy is a large shark. Sharks eat their prey whole, or they tear off large chunks of flesh. They also (10)………………..on dead or dying animals. 1. A. classify B. divide C. organize D. arrange 2. A. all B. through C. throughout D. over 3. A. grow B. rise C. evolve D. vary 4. A. as B. so C. very D. exactly 5. A. stretch B. measure C. develop D. expand 6. A. some others B. others C. different kinds D. some sharks 7. A. dwell B. exist C. emigrate D. migrate 8. A. fresh B. sweet C. light D. clear 9. A. uncooked B. live C. lively D. alive 10. A. eat B. swallow C. exist D. feed Part II: Fill in each blank with a suitable word to fill in the blanks. Vitamins are substances required for the proper functioning of the body. In this century, thirteen vitamins have been (1)………… A lack of any vitamins in a person’s body can cause illness. In some cases, an excess of vitamins can also (2)…………to illness. For example, sailors in the past were prone to (3)…………from scurvy that is a disease resulting from the lack of vitamin C. It causes bleeding of the gum, loss of teeth and skin rashes. Sailors suffer from scurvy because they did not eat fruits and vegetables. Fruits and vegetables (4)…………vitamin C which is necessary for good (5)………… Vitamin B complex is composed of eight different vitamins. A lack of any of these vitamins will lead to different (6)…………For instance, a person who has too little vitamin B1 will suffer from beri-beri, a disease that causes heart problems and mental (7)…………A lack of vitamin B2 results in eye and skin problems while deficiency of vitamin B6 causes problems of the nervous system. Too little vitamin B12 will cause anemia. The (8)…………that vitamin deficiencies caused certain diseases led doctors to cure people suffering from these illnesses by giving them doses of the (9)……...vitamins. Today, vitamins are (10)…………in the form of pills and can easily be bought at any pharmacy. Part III: Read the following passage and answer the questions by choosing the best answer among A, B, C or D. Over the past 600 years, English has grown from a language of few speakers to become the dominant language of international communication. English as we know it today emerged around 1350, after having incorporated many elements of French that were introduced following the Norman invasion of 1066. Until the 1600s, English was, for the most part, spoken only in England and had not extended even as far as Wales, Scotland, or Ireland. However, during the course of the next two centuries, English began to spread around the globe as a result of exploration, trade (including slave trade), colonization, and missionary work. That small enslaves of English speakers became established and grew in various parts of the world. As these communities proliferated, English gradually became the primary language of BY ĐỖ BÌNH – THPT LIẾN SƠN, LẬP THẠCH, VĨNH PHÚC www.violet.vn/quocbinh72 international business, banking, and diplomacy. Currently, more than 80 percent of the information stored on computer systems worldwide is in English. Two thirds of the world’s science writing is in English, and English is the main language of technology, advertising, media, international airports, and air traffic controllers. Today there are 700 million English users in the world, and over half of these are nonnative speakers, constituting the largest number of nonnative users of any language in the world. 1. What is the main topic of the passage? A. The number of nonnative users of English. B. The French influence on the English language. C. The expansion of English as an internatonal language. D. The use of English for science and tecnology. 2. English began to be used beyond England approximately............................. A. in 1066 B. around 1350 C. before 1600 D. after 1600 3. According to the passage, all of the following contributed to the spread of English around the world EXCEPT ..................................... A. the slave trade B. the Norman invasion C. missionaries. D. colonization 4. Which of the following statements is NOT true? A. Most of the information stored on computer systems is in English. B. Only one thirds of the world’s science writing is in languages other than English. C. English is the only language used in technology, and advertising. D. International airports and air controllers use mostly English. 5. According to the passage, approximately how many nonnative users of English are there in the world today? A. A quarter million B. Half a million C. 350 million D. 700 million. Part IV: Read the passage then choose the best sentences A-K to fill in each gap. There is one extra sentence which you do not need to use: BITTER WATER HITS THE BIG TIME Chocolate, which has its origins in South America, is now part of a multi-million pound worldwide business. At Easter, British people spend over $230 million on chocolate. A massive eight per cent of all chocolate is bought at this time. (1)____. Although the large scale industrial production of chocolate began in the last century, the cacao plant was first cultivated by the Aztec, Toltec and Mayan civilizations of Central America over three thousand years ago. The cacao tree is an evergreen, tropical plant which is found in Africa, South and Central America, the West Indies and South East Asia. The fruit of this tree is melon-sized and contains 20-40 seeds. (2)____. In English – speaking countries, they are called cocoa beans. This is a misspelling from the 17th century when they were also called cacoa and cocao beans. The Aztecs used cocoa beans as money. (3)____. This is from the world in the Aztec language, Nahuatl, meaning “bitter water”. (4)____. The Spanish found the drink more palatable mixed with cinnamon and sugar, but the recipe did not spread to the rest of Europe for another century. In the late 17th century, chocolate houses were set up in Europe’s capital cities, where people gathered to drink chocolate. (5)____. But in 1826, CJ van Houten of the Netherlands invented chocolate powder. (6)____. The age of the chocolate bar as we know it began in 1847 when a Bristol company, Fry and Sons, combined cocoa butter with pure chocolate liquor and sugar to produce a solid block that you could eat. (7)____. At the turn of the century, the British chocolate market was dominated by French companies. In 1879 the English company Cadbury even named their Birmingham factory Bournville (ville is the French word for town) in the hope that a little glamour would rub off. But then came Cadbury’s famous Dairy Milk bar which began life as a Dairymaid in 1905. (8)____. It seems that, for the time being at least, chocolate intake in Britain has established at about four bars each week. (9)____. The latest market trick is the so-called “extended line”. This is when the humble chocolate bar becomes an ice cream, a soft drink or a dessert, to tempt chocoholics who have grown tired of conventional snacks. At the other end of the production process, cacao farmers are still feeling the effects of a crash in cocoa bean prices at the end of 1980s. (10)____. Perhaps you could spare a thought for them as you munch your next chocolate bars. A. This was made by extracting most of the cocoa butter from the crushed beans. BY ĐỖ BÌNH – THPT LIẾN SƠN, LẬP THẠCH, VĨNH PHÚC www.violet.vn/quocbinh72 B. A Swiss company then introduced milk solids to the process which gave us milk chocolate. C. They also used them to make a drink called xocoatl. D. Until the last century, the chocolate drink was made from solid blocks of chocolate which had to be melted down in hot water. E. When dried they become cacao beans, which can be used to make chocolate. F. Clever advertising which associated it with the healthy qualities of milk from the English countryside quickly established the bar as a rival to the more decadent French brands. G. British manufacturers include up to 5 per cent vegetable fat in their chocolate, something forbidden elsewhere. H. As most cacao farmers operate on a very small scale, many were forced out of business. I. This has forced manufacturers to look for new ways to attract customers. J. In Aztec times the chocolate drink was flavored with spices and used on ceremonial occasions and for welcoming visitors. K. Only at Christmas do people eat more of the cocoa-based foodstuffs. Section C: Writing Part I: Rewrite the following sentences so that they have a similar meaning with the first one. 1. “Don’t forget to phone the police”, she said She reminded him .................................................. .................................................. ......... 2. It is believed that the man escaped in a stolen car. The man............................................... .................................................. ................... 3. A small church lies at the foot of the hill. At the foot .................................................. .................................................. ..................... 4. If you changed your mind, you’ll be welcome to join our club . Were you .................................................. .................................................. ...................... 5. We don’t have to do so many things to please him. It is .................................................. .................................................. ................................. 6. I’m sure he didn’t do it by himself. He................................................ .................................................. ........................................ 7. He can’t afford to go to America this summer. He doesn’t .................................................. .................................................. ...................... 8. Timmy has become confident as a result of his success . Timmy’s success has turned............................................ ................................................ 9. I haven't seen my uncle for a long time. It's a long time .................................................. .................................................. .................... 10. She dances beautifully and she sings sweetly, too. Not only .................................................. .................................................. ............................. Part II: Finish the second sentence so that it means the same as the first one , using the word in capital letters which must not be altered in any way : 1. You should think about the price before you decide whether to buy it or not. (consideration) →…………………………………………………………………………………………. 2. People don’t want to buy cars with large engines any more. (call) →…………………………………………………………………………………………. 3. Twenty years ago this region produced twice as much coal as it does now.(halved) →…………………………………………………………………………………………. 4. The prime minister in unlikely to call an early general election. (likelihood) →…………………………………………………………………………………………. 5. The policeman acted quickly and averted an accident (prompt) →…………………………………………………………………………………………. Part III: Write a composition (300 words) about the following topic: How do movies and television influence people’s behavior? Use reasons and specific examples to support your answer. BY ĐỖ BÌNH – THPT LIẾN SƠN, LẬP THẠCH, VĨNH PHÚC www.violet.vn/quocbinh72 ĐÁP ÁN VÀ HƯỚNG DẪN CHẤM KỲ THI HỌC SINH GIỎI NĂM HỌC 2013- 2014 Môn thi: Tiếng Anh- lớp 12 Section A: Grammar & Vocabulary (40điểm) Part I (1x 20= 20 điểm) Câu 1 2 3 4 5 6 A C A D A A Câu 11 12 13 14 15 16 D D C B A A Part II (1 x 10 = 10 điểm) 1. acquiring → acquired 6. vocation → vocational 2. explicitness → explicitly 7. like → as 3. few → a few 8. assumption → assume 4. and → to 9. certainly → certain 5. but → to 10. intention → intended Part III (1 x 10 = 10 điểm) 7 B 17 D 8 A 18 D 9 C 19 B 10 D 20 C 1. thunderstorm 2. raincoat 3. shopping 4. pleasure 5. deserted 6. beating 7. disturbance 8. entrance 9. led 10. slightest Section B: Reading (35 điểm) Part I ( 1x 10 = 10 điểm) 1. A 2. C 3. D 4. A 5. B Part II (1x 10 = 10 điểm) 1.discovered 6. B 2. lead 6. diseases 7. disorders Part III (1x 5= 5 điểm) 1. C 2. D 3. B 4. C 7. A 8. A 9. B 10. D 3. suffer 4. contain 5. health 8. knowledge 9. necessary 10. available `5. C BY ĐỖ BÌNH – THPT LIẾN SƠN, LẬP THẠCH, VĨNH PHÚC www.violet.vn/quocbinh72 Part IV (1x 10= 10 điểm) 1.K 2.E 3.C 4.J 5.D 6.A 7.B 8.F 9.I 10H Section C: Writing (25 điểm) Part I (1x 10 = 10 điểm) 1. She reminded him to phone the police. 2. The man is believed to have escaped in a stolen car 3. At the foot of the hill lies a small church 4. Were you to change you mind, you’ll be welcome to join our club. 5. It is unnecessary to do so many things to please him. 6. He can’t have done it by himself. 7. He doesn’t have enough money to go to America this summer. 8. Timmy’s success has turned him into a confident person. 9. It’s a long time since I last saw my uncle/ I saw my uncle for the last time. 10. Not only does she dance beautifully but she also sings sweetly. Part II (1x 5= 5 điểm) 1. You should take the price into consi….. 2. there is no(longer any)/ there isn’t much call for cars…….. 3 .coal production/ the coal produced in this region has been halved in the 20 years. 4. there is little likelihood. The likelihood………is small of the PM calling an…/that the PM will call a……. 5. The policeman’s prompt action averted/ the prompt action of the policeman averted…….. Part III (10 điểm) – Giám khảo tự cho điểm, yêu cầu: - Thí sinh phải xác định được ảnh hưởng của phim và truyền hình đối với cách ứng xử của con người, cả về mặt tích cực lẫn tiêu cực - Nêu được 2- 3 ý nhỏ cho mỗi ý lớn - Văn viết trôi chảy, diễn đạt gãy gọn (dùng cấu trúc đơn giản nhưng diễn đạt rõ ý) - Sai không quá 5 lỗi nhỏ (về từ vựng, ngữ pháp, dùng từ…..) thì không trừ điểm BY ĐỖ BÌNH – THPT LIẾN SƠN, LẬP THẠCH, VĨNH PHÚC www.violet.vn/quocbinh72 SỞ GD&ĐT VĨNH PHÚC ----------------ĐỀ CHÍNH THỨC (Đề thi gồm 5 trang) KỲ THI CHỌN HSG LỚP 10 THPT NĂM HỌC 2011-2012 ĐỀ THI MÔN: TIẾNG ANH (Dành cho học sinh THP không chuyên) Thời gian thi: 180 phút, không kể thời gian giao đề PART I. LISTENING You are going to hear a talk about security in the UK. Listen to the talk and complete the statements below by writing no more than THREE words in the spaces provide. You will hear the talk TWICE. • Don’t carry more (1)________ than you need for daily expenses. • When you stay at a hotel, ask the (2)_______ to keep your valuables in hotel (3)________. • Don’t keep a note of the serial (4)________ together with your traveler’s cheques. • You should carry wallets and purses in an (5)________ pocket or a handbag. • Your passport, (6) ____________ and other important documents should be taken special care of. • You can leave your (7) _________ luggage in a luggage office at most large stations and pick it up later. • It’s necessary to keep the receipt and check the (8) _________ hours when you leave your luggage at the station. • The (9) _________ Property Office can be found at both (10) __________ and the station. PART II. GRAMMAR AND VOCABULARY I. Complete the following sentences by choosing the correct answer among four options (A, B, C or D). 1. He's really shy _______ girl. A. by B. at C. for D. with 2. The teacher _______ her to improve her drawing. A. insisted B. encouraged C. made D. persisted 3. I couldn't quite ______ what they were doing because they were so far away. A. bear out B. make out C. think out D. try out 4. The meal Mary cooked tastes_______. A. well B. nice C. good D. worse 5. ______ at the party, we saw Ruth standing alone. A. Arriving B. We arrived C. Arrived D. We were arriving 6. The people who______ the survey said that they had examined over 1,000 accidents. A. gave B. proceed C. set D. conducted 7. The judge found him ______ of stealing and sent him to prison. A. evil B. innocent C. guilty D. wicked 8. The house we have rented is______. So we will have to buy some beds, chairs, tables, etc. A. unrestored B. unrepaired C. unfurnished D. undecorated 9. He was turned down for the job because he is ________. A. qualified B. qualifying C. unqualified D. qualification 10. The trouble started only______ the other man came into the room. A. when B. until C. and then D. too soon 11. _______, the disaster would not have happened. A. Had you have obeyed the orders B. You had obeyed the orders C. You obeyed the orders D. Had you obeyed the orders 12. _______ had booked in advance were allowed in. A. Only who B. Only those who C. Only who were those D. Only were those who 13. Traveling alone to a jungle is adventurous, ________. A. if not impossible B. if it not impossible C. when not impossible D. when it not impossible BY ĐỖ BÌNH – THPT LIẾN SƠN, LẬP THẠCH, VĨNH PHÚC www.violet.vn/quocbinh72 14. I ______ the hot weather in the south. A. use to B. used to 15. The meat looked very _______ to the dog. A. invited B. invite C. am use to D. am used to C. inviting D. invitingly II. Use the correct form of each word on the right to complete the numbered spaces provided in the passage. Write your answers on your answer sheet. The mysteries of the skies Three hundred and fifty years before the first men looked down on the amazingly beautiful surface of the moon from close quarters, Galileo’s newly built telescope (1) _____________ him to look at the edge of the hitherto mysterious sphere. He saw that the apparently (2) _____________ surface was not divinely smooth and round, but bumpy and imperfect. He realized that although the moon might appear (3) _____________, resembling a still life painted by the hand of a cosmic (4) ____________, it was a real world, perhaps not very different from our own. This amounted to a great (5) _____________ hardly to be expected in his day and age, although nowadays his (6) _____________ may appear to some to be trivial and (7) _____________. Not long after Galileo lunar’s observations, the skies which had previously been so (8) _____________ revealed more of their extraordinary mysteries. Casting around for further wonders, Galileo focused his lens on the (9) _____________ planet of Jupiter. Nestling next to it, he saw four little points of light circling the distant planet. Our moon it appeared, perhaps (10) ____________ in the eyes of those fearful of what the discovery might mean, was not alone! 1. ABLE 2. LIVE 3. ACT 4. ART 5. ACHIEVE 6. CONCLUDE 7. SIGNIFY 8. ELUDE 9. STRIKE 10. FORTUNE III. In the following passage, some numbered lines contain a word that shouldn’t be there. Tick (√) the sentences that are correct and write the words that shouldn’t be there in the numbered space. KEEPING YOUR DISTANCE Personal space is a term that refers to the distance we like to keep between ourselves and other people. When someone we do not know well gets too close that we usually begin to feel uncomfortable. If such a business colleague comes closer than 1.2 meters, the most common response is to move away. Some interesting studies have been done in libraries. If strangers will come too close, many people get up and leave the building, others use to different methods such as turning their back on the intruder. Living in cities has made people to develop new skills for dealing with situations where they are very close to strangers. Most people on so crowded trains try not to look at strangers; they avoid skin contract, and apologize if hands touch by a mistake. People use newspapers as a barrier between themselves and other people, and if they do not have one, they stare into the distance, 0 ___√___ 00 someone 1________ 2 ________ 3 ________ 4 ________ 5 ________ 6 ________ 7 ________ 8 ________ 9 ________ 10 _______ BY ĐỖ BÌNH – THPT LIẾN SƠN, LẬP THẠCH, VĨNH PHÚC www.violet.vn/quocbinh72 making sure they are not looking into anyone’s eyes. PART III. READING I. Complete the following passage by choosing A, B, C or D to fill in each blank. In recent years, there has been a remarkable increase into happiness. The researchers have come up a number of factors which contribute to a definition of happiness. First of all, there is, in some people, a moderate genetic predisposition to be happy, in other words, happiness (1)_______ in families. And happiness seems to correlate quite strongly with the main dimensions of personalities: extroverts are generally happier, neurotics are less so. Second, people often report good social relations as a reason for their happiness. In particular, friends are a great (2) ______ of joy, partly because of the agreeable things they do together, partly because of the way friends use positive non-verbal (3) ______ such as caressing and touching, to affirm their friendship. Marriage and similar (4) ______ relationships can also form the basis of lasting happiness. Third, job satisfaction undoubtedly (5) ______ overall satisfaction, and vice versa - perhaps this is why some people are happy in boring jobs: it (6) ______ both ways. Job satisfaction is caused not only by the essential nature of the work, but (7)_____ by social interactions with co-workers. Unemployment, on the contrary, can be a serious cause of unhappiness. Fourth, leisure is important because it is more under individual (8) ______ than most other causes of happiness. Activities (9) _____ sport and music, and participation in voluntary work and social clubs of various kinds, can give great joy. This is partly because of the (10) ______themselves, but also because of the social support of other group members – it is very strong in the case of religious groups. 1. A. runs 2. A. source 3. A. movements 4. A. near 5. A. consists of 6. A. works 7. A. too 8. A. check 9. A. so 10. A. facilities B. arrives B. origin B. signals B. tight B. applies to B. effects B. as well B. power B. such B. activities C. goes C. base C. slogans C. close C. counts on C. makes C. also C. choice C. like C. exercises D. descends D. meaning D. motions D. heavy D. contributes to D. turns D. plus D. control D. thus D. amenities II. Read the passage carefully, then choose the correct option (marked A, B, C or D) to answer the questions. Scientists have established that influenza viruses taken from man can cause disease in animals. In addition, man can catch the disease from animals. In fact, a greater numbers of wild birds seem to carry the virus without showing any evidences of illness. Some scientists conclude that a large family of influenza virus may have evolved in the bird kingdom, a group that has been on earth 100 million years and is able to carry the virus without contracting the disease. There is even convincing evidence to show that virus strain are transmitted from place to place and from continent to continent by migrating birds. It is known that two influenza viruses can recombine when both are present in an animal at the same time. The result of such recombination is a great variety of strains containing different H and N spikes. This raises the possibility that a human influenza virus can recombine with an influenza virus from a lower animal to produce an entirely new spike. Research is underway to determine if that is the way major new strains come into being. Another possibility is that two animal influenza strains may recombine in a pig, for example, to produce a new strain which is transmitted to man. 1. According to the passage, scientists have discovered that influenza viruses ______. A. cause ill health in wild animals B. do not always cause symptoms in birds C. are rarely present in wild birds D. change when transmitted from animals to man 2. What is known about the influenza virus? BY ĐỖ BÌNH – THPT LIẾN SƠN, LẬP THẠCH, VĨNH PHÚC www.violet.vn/quocbinh72 A. It was first found in a group of very old birds. C. It existed over 100 million years ago. B. All the different strains can be found in wild birds. D. It can survive in many different places. 3. According to the passage, a great variety of influenza strains can appear when______. A. H and N spikes are produced B. animal and bird viruses are combined C. dissimilar types of viruses recombine D. two viruses of the same type are contracted 4. New strains of viruses are transmitted to man by_______. A. a type of wild pig B. diseased lower animals C. a group of migrating birds D. a variety of means 5. It can be inferred from the passage that all of the following are ways of producing new strains of influenza EXCEPT_______. A. two influenza viruses in the same animal recombining B. animal viruses recombining with human viruses C. two animal viruses recombining D. two animal viruses recombining in a human III. Read the passage and choose the best answer from the four options marked A, B, C or D in the following questions. Identify your answer by writing the corresponding letter A, B, C or D on your answer sheet. Several hundred million years ago, plants similar to modern ferns covered vast stretches of the land. Some were as large as trees, with giant fronds bunched at the top of trunks as straight as pillars. Others were the size of bushes and formed thickets of undergrowth. Still others lived in the shade of giant club mosses and horsetails along the edges of swampy lagoons where giant amphibians swam. A great number of these plants were true ferns, reproducing themselves without fruits or seeds. Others had only the appearance of ferns. Their leaves had organs of sexual reproduction and produced seeds. Although their “flowers” did not have corollas, these false ferns (today completely extinct) ushered in the era of flowering plants. Traces of these floras of the earliest times have been preserved in the form of fossils. Such traces are most commonly found in shale and sandstone rocks wedged between coal beds. Today only tropical forests bear living proof of the ancient greatness of ferns. The species that grow there are no longer those of the Carboniferous period, but their variety and vast numbers, and the great size of some, remind us of the time when ferns ruled the plant kingdom. 1. What does the passage mainly discuss? A. Plant reproduction B. How to locate fossils C. An ancient form of plant life D. Tropical plant life 2. The word “Others” refers to _________. A. plants B. pillars C. trees D. fronds 3. Which of the following is NOT mentioned as a characteristic of the plants described in the passage? A. They once spread over large areas of land. B. They varied greatly in size. C. They coexisted with amphibians, mosses, and horsetails. D. They clung to tree trunks and bushes for support. 4. The word “true” is closest in meaning to which of the following? A. accurate B. genuine C. straight D. dependable 5. The author states that fossils of early plant life are usually found in rocks located between deposits of _______. A. coal B. shale C. sandstone D. corollas IV. Read the passage carefully then fill in the blank a suitable word. As swimming became a popular recreation in England during the 1860s and 1870s, several (1) ______ sports developed, roughly patterned after land sports. (2) ______ them were water football (or soccer), water BY ĐỖ BÌNH – THPT LIẾN SƠN, LẬP THẠCH, VĨNH PHÚC www.violet.vn/quocbinh72 rugby, water handball, and water polo, in which players rode on floating barrels, painted to look (3) ______ horses, and struck the ball with a stick. Water rugby became most popular of these sports, but somehow the water polo name became attached to it, and it's been attached (4) ______ since. As played in England, the object of the sport was for a player to touch the ball, with both (5) ______, at the goal end of the pool. The goaltender stood on the pool deck, ready to dive on any opponent who was about to score. Water polo quickly became a very rough sport, filled (6) ______ underwater fights away from the ball, and it wasn't unusual for players to pass out for lack of air. In 1877, the sport was tamed in Scotland by the addiction of goalposts. The Scots also replaced (7) ______ original small, hard rubber ball with a soccer ball and adopted (8) ______ that prohibited taking the ball under the surface or, "tackling" a player unless he had the ball. The Scottish game, which emphasized swimming speed, passing, and (9) ______ work, spread to England during the early 1880s, to Hungary in 1889, to Austria and Germany in 1894, to France in 1895, and (10) ______ Belgium in 1900. Water polo was the first team sport added to the Olympic program, in 1900. PART IV: WRITING I. Write the new sentences using the given word. Do not change the word given in any way. 1. They have discovered some interesting new information. (LIGHT) 2. They suspended Jack for the next two matches. (BANNED) 3. I really want to see her again. (DYING) 4. She was so beautiful that I couldn't stop looking at her. (EYES) 5. We are looking forward to watching the program. (WAIT) II. Rewrite each of the following sentences so that it has a similar meaning to the original one. 1. If you changed your mind, you would be welcomed to join our class. → Were you______________________________________________________ 2. I'd rather not go out this afternoon. → I do not feel____________________________________________________ 3. Adeles tries hard, but she doesn't get anywhere. → However______________________________________________________ 4. It is thought that the boss is considering raising wages. → The boss______________________________________________________ 5. His disabilities did not prevent him from sailing around the world. → Despite the fact_________________________________________________ 6. I didn't arrive in time to see her. → I wasn't_______________________________________________________ 7. I'd prefer you not to smoke. → I'd rather______________________________________________________ 8. The mother smiled happily. She took the baby in her arms. → Smiling________________________________________________________ 9. The noise next door did not stop until after midnight. → It was not ______________________________________________________ 10. You can ring this number whenever there is any difficulty. → Should _________________________________________________________ BY ĐỖ BÌNH – THPT LIẾN SƠN, LẬP THẠCH, VĨNH PHÚC www.violet.vn/quocbinh72 -------------------------------The end--------------------------------- BY ĐỖ BÌNH – THPT LIẾN SƠN, LẬP THẠCH, VĨNH PHÚC www.violet.vn/quocbinh72 HƯỚNG DẪN CHẤM THI HSG TIẾNG ANH LỚP 10 THPT – NĂM HỌC 2011 - 2012 PART I. LISTENING (15 pts: 1,5pts/item) 1. cash 4. numbers 7. heavy 10. the airport 2. manager 5. inside 8. opening 3. safe 6. travel tickets 9. Lost PART II. GRAMMAR AND VOCABULARY (35pts) I. (15pts: 1pt/item) 1. D 2. B 3. B 4. C 5. A 6. D 7. C 8. C 9. C 10. A 11. D 12. B 13. A 14. D 15. C II. (10pts: 0.1pt/item) 1. enabled 4. artist 7. insignificant 10. unfortunately 2. lifeless 5. achievement 8. elusive 3. inactive 6. conclusion 9. striking III.(10pts: 1pt/item) 1. that 9. a 5. to 3. √ 7. √ 2. such 4. will 6. to 8. so 10. √ PART III . READING (30pts) I. (10pts: 1pt/item) 1. A 3. B 2. A 4. C II. (5pts: 1pt/item) 1. B 2. D III. (5pts: 1pt/item) 1. C 2. A IV. ( 10pts: 1pt/item) 1. water 2. Among 6. with 7. the 5. D 6. A 7. C 8. D 9. C 10. B 3. C 4. D 5. D 3. D 4. B 5. A 3. like 8. rules 4. ever 9. team 5. hands 10. to PART IV. WRITING (20pts) I. (10pt: 2 pts/item) 1. Some interesting new information has come to light. 2. Jack was banned from playing in the next two matches. 3. I'm dying to see her again. 4. She was so beautiful that I couldn't take my eyes off her. 5. We can't wait to watch the program. II. (10 pts: 1pt/item) 1. Were you to change your mind, you would be welcomed to join our class. 2. I do not feel like going out this afternoon. 3. However hard Adeles tries, she doesn't get anywhere/gets nowhere. 4. The boss is thought to be considering raising wages. 5. Despite the fact that he was disabled, he sailed/managed to sail around the world. 6. I wasn't early enough to see her. 7. I'd rather you didn't smoke. 8. Smiling happily, the mother took the baby in her arms. 9. It was not until after midnight that the noise next door stopped 10. Should there is any difficulty, you can ring this number. BY ĐỖ BÌNH – THPT LIẾN SƠN, LẬP THẠCH, VĨNH PHÚC www.violet.vn/quocbinh72 BY ĐỖ BÌNH – THPT LIẾN SƠN, LẬP THẠCH, VĨNH PHÚC www.violet.vn/quocbinh72 SỞ GD&ĐT -------ĐỀ CHÍNH THỨC --------------------------- KÌ THI CHỌN HSG VÒNG TỈNH – NĂM HỌC 2010 - 2011 ĐỀ THI MÔN: TIẾNG ANH LỚP 10 Dành cho học sinh các trường THPT không chuyên Thời gian làm bài: 180 phút (Đề thi gồm 05 trang) A.LISTENING I. Listen to the conversation between a female student and a male student then circle the appropriate letter. You will hear the conversation TWICE. 1. What are the students looking for? A. Main Hall B. Great Hall C. Old Hall D. Old building 2. Where is the administration building? A. On the left of the steps B. On the right of the steps C. Behind the steps D. In front of the steps 3. How many people are waiting in the queue? A. 300 B. 200 C. 100 D. 50 4. What does the woman order for lunch? A. fried rice B. pizza C. sandwich D. hot dog 5. What does the woman order to drink? A. orange juice B. coca cola C. tea D coffee 6. How much money does the woman give the man? A. $2.00 B. $ 3.00 C. $ 3.50 D. $ 5.00 7. What kind of fruits does the woman want the man to get for her? A. an orange B. an apple C. grapes D. water melon 8. What did the man buy for the woman to eat? A. a sandwich B. a slice of pizza C. fried rice D. hot dog 9. What did the man buy for the woman to drink? A. tea B. coffee C. coca cola D. orange juice 10. What must the students do as part of registration at the university? A. Check the notice board in the Law Faculty. B. Find out about lectures. C. Organize tutorial groups. D. Pay the union fees. II. Listen to the conversation between a woman and a clerk, then complete the registration form using NO MORE THAN 3 WORDS. You will hear the conversation TWICE. Name of student: (1) _______________________________________ Address : (2) _____________, (3) _______________________ Town : Brisbane Tel : (4) __________________________________ Course : (5) _____________________________________ BY ĐỖ BÌNH – THPT LIẾN SƠN, LẬP THẠCH, VĨNH PHÚC www.violet.vn/quocbinh72 B. PHONETICS Pick out the word whose underlined part is pronounced differently from that of the other words. Identify your answer by writing the corresponding letter A, B, C or D on your answer sheet. 1. A. of B. file C. off D. form 2. A. gestation B. jellyfish C. species D. ethnic 3. A. anthem B. abandon C. disaster D. ambassador 4. A. committee B. titanic C. citadel D. elimination 5. A. funeral B. document C. altitude D. volunteer C. LEXICAL – GRAMMAR I. Choose from the four options given (marked A, B, C and D) one best answer to complete each sentence. Identify your answer by writing the corresponding letter A, B, C or D on your answer sheet. 1. Anyone caught throwing ____________ here should be fined. A. rubbish B. bag C. money D. flowers 2. Let’s go for a picnic this week, ____________? A. do we B. don’t we C. shall we D. have we 3. I can’t stand the car ____________ Therefore, I hate traveling by car. A. illness B. sickness C. ailment D. disease 4. He decided to buy a computer ____________ he didn’t have much money. A. because B. after C. although D. Despite 5. A person who is more beautiful in photographs is said to be ____________. A. photogenic B. photography C. photograph D. photographic 6. When he __________, everyone ____________. A. arrived/ has left B. had arrived/ left C. arrived/ had left D. would arrive/ leave 7. I work as a ____________ tutor to earn money for a summer trip. A. general B. private C. secret D. single 8. I wouldn’t waste time ___________that book if I were you. A. to reading B. reading C. read D. to be read 9. She has got a bad ____________ for definition. A. memory B. mind C. brain D. nerve 10. “How does Ann like her new school?” “Fine, she is doing ____________ in her course.” A. extreme good B. extremely good C. extreme well D. extremely well. 11. We are planning to go on a ___________ holiday this summer. A. packing B. pack C. packed D. package 12. The series is watched by millions of ___________ A. viewing B. viewees C. viewers D. views 13. Be careful! The mountain road is ____________slippery. A. terrify B. terrifyingly C. terrifying D. terrified 14. Teacher will never be ___________ by computers in the classroom. A. replaced B. exchanged C. worked D. preferred 15. If he ___________ late, he ____________sleepy now . BY ĐỖ BÌNH – THPT LIẾN SƠN, LẬP THẠCH, VĨNH PHÚC www.violet.vn/quocbinh72 A. hadn’t stayed up/ wouldn’t have been C. didn’t stay up/ wouldn’t have been B. didn’t stay up/ wouldn’t be D. hadn’t stayed up/ wouldn’t be II. Supply the correct form of the words in brackets. 1. The students are already _____________ with the work of Shakespeare. (ACQUAINT) 2. The children all have very different ____________(PERSON) 3. An _______________ is a person who is concerned about the natural environment and wants to improve and protect it. (ENVIRONMENT) 4. She had never been greatly concerned about her ___________(APPEAR) 5. The _____________ of people interviewed prefer TV to radio. (MAJOR) III. Each sentence below has four underlined words or phrases marked A, B, C, D. Circle one underlined word or phrase that must be changed in order for the sentence to be correct. 1. People think computers never do any mistakes, but in fact, they do A B C D 2. Going to the cinema is an enjoyable way relaxing at weekend. A B C D 3. Since Mai looks a bit pale these days, I think she should have taken up some exercises. A B C D 4. He appears being a perfect man but in fact, he is very rude to his wife. A B C D 5. The youth nowadays prefer pop music than traditional music. A B C D D. READING I. Read the following passage then choose the best answer from the four options (marked A, B, C and D) to complete the numbered blanks in the passage. Identify your answers by writing the corresponding letter A, B, C or D on your answer sheet. Christmas is a Christian holiday. It commemorates the __________ (1) of Jesus Christ more than 2,000 years ago. Christian churches hold ___________(2) services to celebrate Christmas ______(3) midnight on Christmas ____________(4), most churches hold special candlelight services. But Christmas is also a social and family____________(5). It is a festival of goodwill, a time____________ (6) family, friends, food, and gift-giving. Many Americans share Christmas cookies, decorate their home, and place presents ____________(7) the family Christmas tree. Children often hang up stockings for Santa Claus to fill _________(8) small gifts. __________ (9) tradition, Santa comes on a sleigh pulled by reindeer. Presents are ____________(10) opened on Christmas Eve or Christmas Day. 1.A. day B. birth C. God D. father 2. A. religious B. religion C. region D. regional 3. A. on B. in C. under D. at 4. A. morning B. afternoon C. evening D. Eve 5. A. Christmas B. holiday C. weekend D. day 6. A. of B. to C. for D. on BY ĐỖ BÌNH – THPT LIẾN SƠN, LẬP THẠCH, VĨNH PHÚC www.violet.vn/quocbinh72 7. A. on B. under C. beside D. in 8. A. with B. in C. of D. on 9. A. Despite B. Because of C. According to D. But 10. A. never B. rarely C. sometimes D. usually II. Read the following passage and then choose the best answer by writing the letter A, B, C or D on the answer sheet. Line A rather surprisingly geographical feature of Antarctica is that a huge freshwater lake, one of the world’s largest and deepest, lies hidden there under four kilometers of ice. Now known as Lake Vostok, this huge body of water is located under the ice block that comprises Antarctica. The lake is able to exist in its unfrozen state beneath this block of ice because its waters are warmed by geothermal heat from the earth’s core. The thick glacier above Lake Vostok (5) actually insulates it from the frigid temperature (the lowest ever recorded on Earth) on the surface. The lake was first discovered in the 1970s while a research team was conducting an aerial survey of the area. Radio waves from the survey equipment penetrated the ice and revealed a (10) body of water of indeterminate size. It was not until much more recently that data collected by satellite made scientist aware of the tremendous size of the lake; satellite –borne radar detected an extremely flat region where the ice remains level because it is floating on the water of the lake. The discovery of such a huge freshwater lake trapped under Antarctic is of interest to the (15) scientific community because of potential that the lake contains ancient microbes that have survived for thousands upon thousands of years, unaffected by factors such as nuclear fallout and elevated ultraviolet light that have affected organisms in more exposed areas. The downside of the discovery, however, lies in the difficulty of conducting research on the lake in such a harsh climate and in the problems associated with obtaining uncontaminated (20) samples from the lake without actually exposing the lake to contamination. Scientists are looking for possible ways to accomplish this. 1. The purpose of the passage is to A. explain how Lake Vostok was discovered B. provide satellite data concerning Antarctica C. discuss future plans for Lake Vostok D. present an expected aspect of Antarctica’s geography. 2. The word “lies” in line 2 could best be replaced by A. sleeps B. sits C. tells falsehoods D. inclines. 3. What is true of Lake Vostok? A. It is completely frozen B. It is not a saltwater lake C. It is beneath a thick slab of ice D. It is heated by the sun 4. Which of the following is closet in meaning to “frigid” in line 6? A. Extremely cold B. Never changing C. Quite harsh D. Rarely recorded 5. All of the following are true about the 1970 survey of Antarctica EXCEPT that it A. was conducted by air B. made use of radio waves C. did not measure the exact size of the lake D. was controlled by a satellite 6. It can be inferred from the passage that the ice would not be flat if A. There were no lake B. the lake was not so big C. Antarctica were not so cold D. radio waves were not used 7. The word “microbes” in line 15 could be best be replaced by which of the following? A. Pieces of dust B. Trapped bubbles C. Tiny organisms D. Rays of light BY ĐỖ BÌNH – THPT LIẾN SƠN, LẬP THẠCH, VĨNH PHÚC www.violet.vn/quocbinh72 8. The passage mentions which of the following as a reason for the importance of Lake Vostok? A. It can be studied using radio waves B. It may contain uncontaminated microbes C. It may have elevated levels of ultraviolet light. D. It has already been contaminated. 9. The word “downside” in line 18 is closest in meaning to A. bottom level B. negative aspect C. underside D. buried section 10. The paragraph following the passage most probably discusses A. further discoveries on the surface of Antarctica B. Problems with satellite- borne radar equipment C. ways to study Lake Vostok without contaminating it D. the harsh climate of Antarctica. III. Read the following passage then choose the best answer from the four options (marked A, B, C and D) to complete the numbered blanks in the passage. Identify your answers by writing the corresponding letter A, B, C or D on your answer sheet. If asked who ____________(1) the game of baseball, most ___________(2) would probably reply that it was Abner Doubleday. At the beginning of this century, there was some disagreement over ___________(3) the game of baseball had actually originated, so sporting-goods manufacturer Spaulding inaugurated a commission to research the ____________(4). In 1908 a report was published by the commission _________(5) which Abner Doubleday, a U.S. Army officer from Cooperstown, New York, was ____________(6) credit for the intervention of the game. The National Baseball Hall of Fame was established in Cooperstown in __________(7) of Doubleday. Today, most sports historians are in agreement that Doubleday ____________(8) did not have much to do with the development of baseball. __________(9), baseball seems to be a close ____________(10) to the English game of rounders and probably has English rather than American roots. 1. A. discovered B. founded C. invented D. found 2. A. Americans B. American C. The US D. America 3. A. what B. how C. when D. that 4. A. problem B. question C. matter D. issue 5. A. at B. to C. of D. in 6. A. provided B. passed C. given D. delivered 7. A. honor B. reward C. award D. ceremony 8. A. truly B. really C. as well D. probably 9. A. But B. Moreover C. Instead D. Additionally 10. A. relate B. relationship C. relative D. relation E. WRITING I. Finish each of the following sentences in such a way that it means exactly the same as the sentence printed before it. 1. Tom is the most industrious pupil. No other pupil _____________________________________________________________ 2. The last time I played football was in 1971. I ________________________________________________________________________ 3. I only bought the dog because my children wanted a pet. If _______________________________________________________________________ 4. The weather was so beautiful that we went swimming. It was ____________________________________________________________________ 5. I am really sorry I didn’t invite her to the party. I really wish _______________________________________________________________ II. Complete the second sentence so that it has a similar meaning to the first sentence, using the given word. 1. She owns all this land now. (belongs) All _____________________________________________________________________ BY ĐỖ BÌNH – THPT LIẾN SƠN, LẬP THẠCH, VĨNH PHÚC www.violet.vn/quocbinh72 2. Organized activities don’t interest Eva very much. ( interested) Eva_____________________________________________________________________ 3. If you run everyday, your breathing improves quickly. (running ) Daily____________________________________________________________________ 4. Do you like meat more than fish? (prefer) Do______________________________________________________________________? 5. He hasn’t got the intelligence to be a programmer ( intelligent) He ______________________________________________________________________ ------- THE END ------SỞ GD&ĐT VĨNH PHÚC --------------------------ĐỀ CHÍNH THỨC HƯỚNG DẪN CHẤM ĐỀ THI CHỌN HSG NĂM HỌC 2010 - 2011 ĐỀ THI MÔN: TIẾNG ANH 10 Dành cho học sinh các trường THPT không chuyên (Hướng dẫn chấm gồm có 03 trang) Tổng điểm: 100 điểm A. LISTENING I. (10pts:.1 pts/item) 1. A 2. B 6. D 7. B 3. D 8. B 4. C 9. D 5. A 10. D II. (10 pts: 2pt/item) 1. JULIA PERKINS 2. flat 5 3. 15 Waratah Road 4. no phone/ none/ not connected/ to be advised 5. First Year Law B. PHONETICS I. ( 5 pts: 01pt/ item) 1. A 2. C 3. C 4.B 5.D 3.B 8. B 13. B 4.C 9. A 14. A 5.A 10. D 15. D C. LEXICAL – GRAMMAR I. ( 15pts: 1pt/ item) 1.A 2.C 6.C 7.B 11. D 12. C II. (5 pts: 1pt/ item) 1. acquainted 2. personalities 3.environmentalist 4. appearance 5. majority III.(5 pts: 1pt/item) 1. B 2. D D. READING I. ( 10pts: 1pt/ item) 3.C 1.B 3.D 2.A 4.A 4.D 5. C 5.B BY ĐỖ BÌNH – THPT LIẾN SƠN, LẬP THẠCH, VĨNH PHÚC www.violet.vn/quocbinh72 6.C 7.B II. (10 pts: 1pt /item) 8. A 9. C 10.D 1.D 6.A 3.C 8.B 4.A 9.B 5.D 10.C 3.B 8.B 4.B 9.C 5.D 10.D 2.B 7.C III. (10 pts: 1 pt /item) 1. C 6. C 2.A 7. A E. WRITING I. ( 10 pts: 2pt/ item) 1. No other pupil is more industrious than Tom/ as industrious as Tom. 2. I haven’t played football since 1971. 3. If my children hadn’t wanted a pet, I wouldn’t have bought the dog. 4. It was such beautiful weather that we went swimming. 5. I really wish I had invited her to the party. II. ( 10 pts: 2pt/ item) 1. All this land belongs to her. 2. Eva isn’t interested in organized activities very much. 3. Daily running (quickly) improves your breathing (quickly). 4. Do you prefer meat to fish? 5. He isn’t intelligent enough to be a programmer. LISTENING TAPESCRIPT I. F : Female student M: male student. F: Excuse me. Can you help me? I was looking for the Main Hall. M: Maybe I can, actually. I am looking for the Main Hall, too. I think it’s in the Administration building. Are you anew student? F: Yes, I am Repeat M: I thought you looked as lost as me. I’m trying to find the admin building, too, so that I can register for my course. But I don’t seem to be having much luck. F: Well, look, according to this map of the campus here, you go straight up the steps, turn left and the building is on the right. OK, let’s see if we can find it. M: Oh, this looks right. Oh, yeah, it must be. Look, there are hundreds of other people here! F: There must be at least 50 people in the queue- we’ll be here till gone 2 o’clock at this rate. BY ĐỖ BÌNH – THPT LIẾN SƠN, LẬP THẠCH, VĨNH PHÚC www.violet.vn/quocbinh72 M: And I’m starving! F: So am I M: Actually, I was on my way to the canteen to get something for lunch. Why don’t we go to the canteen and buy something and you stay here and wait? F: Good idea. M: What would you like? Pizza, sandwich, hot dog, fried rice? They do everything…….. F: Oh, something easy. Take away fried rice sounds good. M: Ok, fried…… F: No, on second thoughts, I’ll have a cheese and tomato sandwich. M: Right- one cheese and tomato- anything to drink? F: Yeah, get me a coffee, would you? M: Oh, hot coffee is a bit hard to carry. What about a coke or an orange juice? F: Oh, um….. get me an orange juice, then. Look, here’s five dollars. M: Oh, take two dollars back, it shouldn’t cost me more than three dollars. F: Well, keep the five and we’ll sort it out later. Oh, and could you get me an apple as well? M: Ok. Back in a minute. M: Oh, there you are F: I thought you were never going to come back. M: Sorry! The canteen was absolutely packed and I had to wait for ages. Then when I got to the front of the queue they had hardly any food left. So I had to get you a slice of pizza. I’m sorry. F: Oh, that’s ok. I could eat anything. I’m so hungry. M: And there’s your bottle of orange juice and your apple. At least I managed that. F: Great. Thanks a lot. M: Oh and here’s your $2 back. F: Don’t worry about it. Buy me a cup of coffee later! M: Oh, alright then! So how’d you go? F: Well in order to register we’ve got to go to the Law Faculty and get this card stamped and then go back to the Admin building and pay union fees. That means we’re registered. After that we have to the notice board to find out about lectures and then we have to put our names down for tutorial groups and go to the library to …….. M: Great. Well first let’s sit down and have our lunch. II. F: female student C: clerk F: Oh, hello. I’m here to register for the First Year Law course. C: I’ll just have to fill out this form for our records. What’s your name? F: Julia Perkins. C: Can you spell that for me? F: Yeah, that’s J-U-L-I-A P-E-R-K-I-N-S. C: Address? F: Flat 5, 15 Waratah Road, that’s W-A-R-A-T-A-H, Brisbane. C: Brisbane…… and your telephone number? F: We haven’t got the phone on yet. We’ve just only moved in. C: OK, well can you let us have the number once the phone’s connected and I’ll make a note here to be advised. And the course ? F: I beg your pardon? BY ĐỖ BÌNH – THPT LIẾN SƠN, LẬP THẠCH, VĨNH PHÚC www.violet.vn/quocbinh72 C: What course are you doing? F: First Year Law. C: Right. Well, you’ll have to go across to the Law Faculty and get this card stamped and then you come back here with it and pay your union fee. F: Thank you very much BY ĐỖ BÌNH – THPT LIẾN SƠN, LẬP THẠCH, VĨNH PHÚC www.violet.vn/quocbinh72 SỞ GD&ĐT ------————— ĐỀ CHÍNH THỨC (Đề thi có 4 trang) KỲ THI CHỌN HSG LỚP 10 THPT NĂM HỌC 2011-2012 ĐỀ THI MÔN: TIẾNG ANH Dành cho học sinh trường THPT Chuyên Vĩnh Phúc Thời gian làm bài: 180 phút, không kể thời gian giao đề ———————— PART I. LISTENING You are going to listen to a conversation between Janet and her friend. Janet is telling her friend about her holiday. As you listen, write down brief notes in the boxes below about her holiday. You should write NO MORE THAN THREE WORDS in each blank. You will hear the recording TWICE. Day What Janet did Saturday Arrived at the hotel at (1) ______. Sunday Hired a (2) ______. Went to Safari Park and saw monkeys and (3) ______. Monday Went to (4) _______ and Stonehenge. Took a lot of (5) ______. Tuesday Joined a (6) _______. Visited Trafalgar (7)______, Westminster Abbey, the Houses of Parliament, and saw the (8) _______ of the guard at Buckingham Palace. Also went to (9) ______ and the Tower of London. Wednesday Went to Greenwich by (10) ______. Thursday Went shopping for (11) _______. Went to see a film called (12) _______ in the evening. Friday (13) _______ all day. Stayed in hotel. Played (14) ______. Saturday Left hotel at (15) _______. PART II. GRAMMAR AND VOCABULARY (3.5 PT) I. Complete the following sentences by choosing the correct answer among four options (A, B, C or D) 1. Jack _____________ a fortune when his great uncle Jack passed on. A. made into B. went into C. came into D. bought 2. Telephone service to that remote village can't be ______ this year. A. provided B. supplied C. improved D. made 3. His company had to close because of_______. A. redundancy B. economic difficulties C. subtitles D. a high rate of unemployment 4. I couldn't ______over how well the team play! A. make B. get C. turn D. put 5. Even if you are good at a game, you shouldn't be______. A. overconfident B. unconfident C. confidential D. confidentable 6. “Is it true that you fell asleep in class yesterday?” “Unfortunately, yes. ________ is unbelievable. I’m very embarrassed.” A. That I could do such a thing it B. That I could do such a thing C. I could do such a thing it D. I could do such a thing 7. ________ is the biggest city in Michigan, it is not the capital. A. Detroit B. If Detroit C. Although Detroit D. Detroit, which 8. An almost _______ line of traffic was moving at a snail’s pace through the town. A. continuous B. constant C. continual D. stopping 9. They were walking on tiptoe ______ the Director's room. A. pass B. passed C. past D. passing 10. I haven’t been feeling very well ________. A. of late B. not long ago C. currently D. by now BY ĐỖ BÌNH – THPT LIẾN SƠN, LẬP THẠCH, VĨNH PHÚC www.violet.vn/quocbinh72 II. Each line of the following passage has one mistake related to either grammar or vocabulary usage. Find and correct them. Air pollution is a cause for ill – health in human beings. It a lot of countries, there are laws limited the amount of smoke which factories can produce. Because there isn't enough information on the amount of smoke in the atmosphere, doctors have proved that air pollution makes lung cancer. The gases from the exhausts of cars have also risen air pollution in most cities. The lead in petrol produces a poisoned gas which often collects in busy streets surrounding by high buildings. Children who live in areas where there is a lot of lead in the atmosphere cannot think as quick as other children and they are clumsy where they use their hands. There are long-term effects of pollution. If the gases in the atmosphere continues to increase, the earth's climate will become warmer. A lot of ice near the Poles may water and may cause serious floods. 0. for --> of 1. _________ 2. _________ 3. _________ 4. _________ 5. _________ 6. _________ 7. _________ 8. _________ 9. _________ 10. ________ III. Use the given phrasal verbs to replace the underlined words/phrases in the sentences. Then put the verbs in the correct form in the sentences. come into go round fall through do without turn in make out draw up take after let on turn down 1. Before we do anything else, we ought to prepare a plan of action. 2. It was getting late so I decided to go to bed. 3. I believe that Diana has recently inherited a lot of money. 4. Do you think there is enough food to feed everybody? 5. If we can’t get any bread, we’ll just have to manage. 6. He speaks very badly. I can’t understand what he’s saying. 7. Don’t say anything to the children about the party. I want it to be a promise. 8. I was rejected for the army on the health ground. 9. We’ve made all the arrangements. Let’s hope our plans don’t fail. 10. I resemble my mother. She was small with blond hair and had a terrible memory too. IV. Use the word given in capitals to form a word that fits in the gap in the sentences. Write your answers on your answer sheet. 1. I’ll never forget the _________ I felt in the situation. (HUMILIATE) 2. She’s so ________ that she won’t let anything stand in the way of her ambition. (MIND) 3. That was a very ________ thing to say. (HURT) 4. It’s ______ of him to lose his temper like that – he’s usually very calm. (CHARACTER) 5. He stood at the door to make sure that no one _______ the party. (GATE) PART III. READING I. Read the passage carefully , then fill in the blank a suitable word. The majority of lottery winners change their lives (1) __________ little, and continue on their settled way happy ever after. A couple of years ago, a Mr. David Horabin won a million. He had been struggling to (2) _________ a success of his dry cleaning shop for the past 12 months. He accepted his cheque in a small ceremony (3) ________ the premises at 2.30, and by three o'clock he had reopened for business. The reaction of Mr. Pasquale Consalvo who won $30 million in the New York state lottery was very (4) _________. He was unhappy not to be able to fulfill his desire to go to work as (5) ___________ on the day he won. He also said that if the money made him (6) ____________ he would give it back. In fact, the chances of his life being made a misery by his new-found wealth are almost (7) _________ slim though not quite as the sixty million-to-one odds he beat to take a jackpot (8)________ had remained unclaimed through six previous draws. Gambling small amounts (9) __________ the lottery is a harmless if BY ĐỖ BÌNH – THPT LIẾN SƠN, LẬP THẠCH, VĨNH PHÚC www.violet.vn/quocbinh72 futile hobby. (10) __________, gambling can become an addiction, increasingly so as the activity becomes socially acceptable. II. Read the passage carefully, then choose the correct option (marked A, B, C or D) to answer the questions. A recent survey of crime statistics shows that we are all more likely to be burgled now than 20 years ago and the police advise everyone to take a few simple precautions to protect their homes. The first fact is that burglars and other intruders prefer easy opportunities, like a house which is very obviously empty. This is much less of a challenge than an occupied house, and one which is well-protected. A burglar will wonder if it is worth the bother. There are some general tips on how to avoid your home becoming another crime statistic. Avoid leaving signs that your house is empty. When you have to go out, leave at least one light on as well as a radio or television, and do not leave any curtains wide open. The sight of your latest music centre or computer is enough to tempt any burglar. Never leave a spare key in a convenient hiding place. The first place a burglar will look is under the doormat or in a flower pot and even somewhere more 'imaginative' could soon be uncovered by the intruder. It is much safer to leave a key with a neighbor you can trust. But if your house is in a quiet, desolate area be aware that this will be a burglar's dream, so deter any potential criminal from approaching your house by fitting security lights to the outside of your house. But what could happen if, in spite of the aforementioned precautions, a burglar or intruder has decided to target your home? Windows are usually the first point of entry for many intruders. Downstairs windows provide easy access while upstairs windows can be reached with a ladder or by climbing up the drainpipe. Before going to bed you should double-check that all windows and shutters are locked. No matter how small your windows may be, it is surprising what a narrow gap a determined burglar can manage to get through. For extra security, fit window locks to the inside of the window. What about entry via doors? Your back door and patio doors, which are easily forced open, should have top quality security locks fitted. Even though this is expensive it will be money well spent. Install a burglar alarm if you can afford it as another line of defence against intruders. A sobering fact is that not all intruders have to break and enter into a property. Why go to the trouble of breaking in if you can just knock and be invited in? Beware of bogus officials or workmen and, particularly if you are elderly, fit a chain and an eye hole so you can scrutinize callers at your leisure. When you do have callers never let anybody into your home unless you are absolutely sure they are genuine. Ask to see an identity card, for example. If you are in the frightening position of waking in the middle of the night and think you can hear an intruder, then on no account should you approach the intruder. Far better to telephone the police and wait for help. 1. According to the writer, we should _______. A. avoid leaving our house empty B. only go out when we have to C. always keep the curtains closed D. give the impression that our house is occupied when we go out 2. The “aforementioned precautions” refer to steps that _______. A. will tell a burglar if your house is empty or not B. are the most important precautions to take to make your home safe C. will stop a potential burglar D. will not stop an intruder if he has decided to try and enter your home 3. Gaining entry to a house through a small window _______. A. is surprisingly difficult B. is not as difficult as people think C. is less likely to happen than gaining entry through a door D. is tried only by very determined burglars 4 . According to the writer, window locks, security locks and burglar alarms _______. A. cost a lot of money but are worth it B. are good value for money C. are luxury items D. are absolutely essential items 5. The writer argues that fitting a chain and an eye hole _______. A. will prevent your home being burgled BY ĐỖ BÌNH – THPT LIẾN SƠN, LẬP THẠCH, VĨNH PHÚC www.violet.vn/quocbinh72 B. avoids you having to invite people into your home C. is only necessary for elderly people D. gives you time to check if the visitor is genuine III. Choose the word that best fits each of the blanks in the following passage. Circle A, B, C or D to indicate your answers. Viewed from the outside (1) ________, the Houses of Parliament look impressive. The architecture gives the place a traditional look, and the buildings are sandwiched between a busy square and the river, making them a (2) ________between the country house of an eccentric duke and a Victorian railway station. You have only to learn that the members (3) ______ to each other as ‘The Honorable Member to (4) ______ the picture of a dignified gentlemen’s club, with of course a few ladies to (5) _______ the numbers. Sadly, over the past few years first radio, and now television, have shown the general public, who are (6) ______ the electorate, what in fact goes on when bills are discussed and questions are asked. The first obvious fact is that the chamber is very rarely full, and there may be only a handful of members present, some of whom are quite clearly asleep, telling jokes to their neighbor, or shouting like badlybehaved schoolchildren. There is not enough room for them all in the chamber in any (7) _______, which is a second worrying point. Of course, television does not follow the work of committees, which are the small discussions groups that do most of the real work of the House. But the (8) ______ impression that voters receive of the workings of government is not a good one. To put it (9) _______, parliament looks disorganized, is clearly behind the time and seems to be filled with bores and comedians. This is presumably why members (10) _______ for so long the efforts of the BBC to broadcast parliamentary matters on television. 1. A. likewise B. at least C. nevertheless D. as well 2. A. mixture B. combination C. cross D. match 3. A. call B. refer C. speak D. submit 4. A. finalize B. end C. conclude D. complete 5. A. take away B. bring about C. make up D. set in 6. A. after all B. anyway C. even D. furthermore 7. A. point B. way C. matter D. case 8. A. total B. broad C. overall D. comprehensive 9. A. bluntly B. shortly C. directly D. basically 10. A. prevented B. checked C. defied D. resisted PART IV: WRITING I. Finish each of the following sentences in such a way that it means exactly the same as the sentence printed before it. 1. The phone stopped ringing the moment I got down stairs. No sooner _______________________________________________ 2. He is determined to carry on working when he is 65. He has no ________________________________________________ 3. He was very sorry that he didn’t see Audrey on her trip to London. He greatly ______________________________________________ 4. She agreed to go out to dinner with him because she assumed he was not married. Had she __________________________________________________ 5. Everyone was surprised that the singer had very little money when he died. The singer had _____________________________________________ II. Write about the following topic: ‘Some people feel that certain workers like nurses, doctors and teachers are undervalued and should be paid more’ How far do you agree? Give reasons for your answer and include any relevant examples from your own knowledge or experience. You should write at least 150 words. BY ĐỖ BÌNH – THPT LIẾN SƠN, LẬP THẠCH, VĨNH PHÚC www.violet.vn/quocbinh72 -------The end------- KỲ THI CHỌN HSG LỚP 11 THPT NĂM HỌC 2011-2012 ĐỀ THI MÔN: TIẾNG ANH Dành cho học sinh trường THPT Chuyên Vĩnh Phúc ———————— PART 1. LISTENING (15 pts: 1/item) SỞ GD&ĐT VĨNH PHÚC ————— 1. 5 pm. 2. small family car 3. lions 4. Oxford 5. photographs 6. sightseeing tour 7. Square 8. changing 9. Tower Bridge 10. boat PART II. GRAMMAR AND VOCABULARY (35pt) I. (10pts: 1pt/item) 1. C 2. A 3. B 6. B 7. C 8. A II. (10pts: 1pt/item) 11. presents and souvenirs 12. Star Wars 13. Rained 14. table tennis 15. 10 am 4. B 9. C 5. A 10. A 1. limited --> limiting 6. surrounding --> surrounded 2. Because --> Although 7. quick --> quickly 3. makes --> causes 8. where--> when 4. risen --> increased 9. continues --> continue 5. poisoned --> poisonous 10. water --> melt III. (10pts: 1 pt/item) 1. draw up 2. turn in 6. make out 7. let on IV. ( 5pts: 1pt/ item) 1. humiliation 2. single-minded/ strong-minded 3. came into 8. turned down 3. hurtful 4. go round 9. fall though 4. uncharacteristic 5. do without 10. take after 5. gate-crashed PART III. READING I. (10pts: 1pt/item) 1. but 2. make 3. at 4. similar 5. usual 6. unhappy 7. as 8. that 9. on 10. However II. (5 pts: 1 pt/item) 1. D 2. D 3. B 4. A 5. D III. (10pts: 1pt/item) 1. B 2. C 3. B 4. D 5. C 6. A 7. D 8.C 9. A 10. D PART IV. WRITING I. (5 pts: 1pt/item) 1. No sooner had I got downstairs than the phone stopped ringing. 2. He has no intention of giving up working/ retiring when he is 65. 3. He greatly regretted not seeing Audrey on her trip to London. 4. Had she known that he was married, she would not have agreed to go out to dinner with him. 5. The singer had very little money (left) when he died, which surprised everybody. II. Write about the following topic: 20 pts The impression mark is based on the following scheme: 1. Content (10 pts): a provision of all main ideas and details as appropriate BY ĐỖ BÌNH – THPT LIẾN SƠN, LẬP THẠCH, VĨNH PHÚC www.violet.vn/quocbinh72 2. Language (5pts): a variety of vocabulary and structures appropriate to the level of English language gifted upper-secondary school students 3. Presentation (5pts): coherence, cohesion, and style appropriate to the level of English language gifted upper-secondary school students. BY ĐỖ BÌNH – THPT LIẾN SƠN, LẬP THẠCH, VĨNH PHÚC www.violet.vn/quocbinh72 SỞ GD&ĐT .......... ĐỀ CHÍNH THỨC KỲ THI CHỌN HSG LỚP 10 THPT NĂM HỌC 2014-2015 ĐỀ THI MÔN: TIẾNG ANH (Dành cho học sinh THPT không chuyên) Thời gian làm bài: 180 phút, không kể thời gian phát đề I. Reading comprehension 1. After the water workers went on strike, there was a___ of water. A. drain B. shortage C. loss D. decrease 2. As the streets of our city become busier, people are turning more and more to the___ bicycle. A. historical B. old - fashioned C. old- aged D. elderly 3. ___ it was raining heavily, we went out without a raincoat. A. In spite B. In spite of C. However D. Although 4. As far as I’m___ it’s quite all right for you to leave early. A. concerned B. regarded C. consulted D. bothered 5. I expect it will rain again when we are on holiday this year, but at last we are properly prepared___ it this time A. about B. at C. with D. for 6. If only he___ told us the truth in the first place, things wouldn’t have gone so wrong. A. had B. has C. would have D. should have 7. I know him by___ but I have no idea what his name is. A. sight B. myself C. heart D. chance 8. No child___ the age of sixteen will be admitted to this film. A. before B. lacking C. except D. below 9. Mr. Smith was___ in a road accident. A. wrong B. wounded C. injured D. damaged 10. Is it worth waiting for a table in this restaurant or shall we go___ else? A. anywhere B. otherwise C. somewhere D. everywhere 11. The picture is___ the thief will be most disappointed when he tries to sell it. A. priceless B. invalid C. unprofitable D. worthless 12. I’m feeling really___ I’m going to bed. A. sleepy B. sleeping C. tiring D. asleep 13. I can___ what he’s doing; it’s so dark down there. A. see through B. make out C. look into D. show up 14. Do you know the time the train___ to Birmingham? A. reaches B. gets C. arrives D. comes 15. Having looked the place___ the gang went away to make their plans A. through B. over C. down D. out 16. When we came back from holiday our suitcase were___ by the customs officer. A. guarded B. tested C. corrected D. examined 17. The child was so noisy that his mother told him not to be in such a___. A. nuisance B. trouble C. bother D. worried 18. Could you be more specific about what is___ in this particular job? A. enclosed B. concentrated C. presented D. involved 19. They haven’t beaten me yet, I still have one or two___ up my sleeve. A. traps B. tricks C. jokes D. defenses 20. The brothers are so alike that I can’t ___ one from the other? A. say B. notice C. mark D. tell II. Sentence transformation Finish each of the following sentence in such a way that it means exactly the same as the sentence printed before it. BY ĐỖ BÌNH – THPT LIẾN SƠN, LẬP THẠCH, VĨNH PHÚC www.violet.vn/quocbinh72 1. John only understood very little of what the teacher said. John could hardly……………………………………………………………….. 2. Unless someone has a key, we can not get into the house We can only……..……………………………………………………………….. 3. I’m sure you didn’t lock the front door. Here’s the key. You can’t……..……………………………………………………………….. 4. He prefers golf to tennis He’d rather…….. ……………………………………………………………….. 5. He is sorry now that he didn’t invite Mary to the party. He wishes……………………………………………………………………….. III. Use of English Section 1: Fill in the blank with one appropriate word Computers are helpful in many ways: First they are fast. They can work with information much more quickly then a person. (1)___, computer can work with lots of information at the (2)___ time. Third, they can keep (3)………for a long time. They do not forget things the way (4)___ do. Also, computers are almost always correct. They are not (5)___, of course, but they usually don’t make mistakes. These days, (6)___ is important to know something about computers. There are a number of ways to (7)___. Some companies have class at work. Also, most universities offer day and (8)___ courses in computer science. Another way to learn is (9)___ a book. There are many books about computers in bookstores and libraries. Or you can learn from a friend. After a few hours of practice you can work with (10)___ You may (11)___ be an expert, but you can have fun! Section 2: Finish each of the following sentences in such a way that it means exactly the same as the sentences printed before it. 32. I only made that terrible mistake because I wasn’t thinking If I………………..…………………………………………………………….. 33. We had planned to visit our grandmother, so we left early in the morning. We were………………………………………………………………………….. 34. Someone rang the alarm as soon as the burglar left the building. No sooner………….…………………………………………………………….. 35. As television programmes become more popular they seem to get worse The more………….…………………………………………………………….. 36. The authorities will prosecute anyone they find trespassing on this land. Anyone found……..…………………………………………………………….. 37. I prefer going out for a meal to staying at home I’d rather………………………………………………………………………….. 38. It would have been a super weekend if it had not been for the weather. But………………………………………………………………………………….. 39. It’s possible that he did not get my letter He might…………….…………………………………………………………….. 40. The last time it snowed here was six years ago It……………………..…………………………………………………………….. Section 3: Guided sentence building 41. You/ interested/ listen/ pop music/ classical music? 42. He/ tall/ than/ his sister/ a head. 43. Jane/ promise/ keep in touch/ us/ Australia. 44. He/ used to/ go/ hunting/ forest/ younger. 45. He/ learn/ English/ before/ he/ go/ England. 46. She / wait/ him/ ten hours. 47. Not allowed/ enter/ museum before 9. BY ĐỖ BÌNH – THPT LIẾN SƠN, LẬP THẠCH, VĨNH PHÚC www.violet.vn/quocbinh72 48. I/ be used/ get up/ early/ don’t mind/ leave / 5 o’clock/ morning. 49. Asked/ mother/ 5 pounds/ buy/ books 50. I/ not feel/ have breakfast/ this morning/ because/ headache. I. Keys: Reading comprehension 12. A 23. information 1. B 13. C 24. to 2. B 14. B 25. perfect 3. D 15. B 26. It 4. A 16. D 27. learn 5. D 17. A 28. night 6. A 18. D 29. from 7. D 19. D 30. computers 8. D 20. C 31. not 9. C 21. second 10. C 11. D 22. same III. Use of English Section II 32. If I had been thinking, I would not have made that mistake 33. We were to visit our grandmother, so we left early in the morning/ We were planning/ going/ intending to visit our grandmother, so we left early in the morning. 34. No sooner had the burglar left the building than the alarm rang. 35. The more popular television programmes become, the worse they seem to get 36. Anyone found trespassing on this land will be prosecuted (by the authorities.) 37. I’d rather go out for a meal than stay at home. 38. But for the weather, it would have been a super weekend. 39. He might not have got my letter. 40. Is hasn’t snowed here for six years. Section III 41. Are you interested in listening to pop music or classical music? BY ĐỖ BÌNH – THPT LIẾN SƠN, LẬP THẠCH, VĨNH PHÚC www.violet.vn/quocbinh72 42. He’s taller than his sister by a head. 43. Jane promised to keep in touch with us in Australia. 44. He used to go hunting in the forest when he was younger. 45. He had learnt English before he went to England. 46. She has been waiting for him for ten hours. 47. We were not allowed to enter the museum before 9 o’clock. 48. I am used to getting up early so I don’t mind leaving at 5 o’clock in the morning. 49. He asked his mother for 5 pounds to buy some books. 50. I did not feel like having breakfast this morning because I had a headache. II. Jgj\kjg III. Use of English Section 1: Section 2: 41. Section 3: Kjg\ BY ĐỖ BÌNH – THPT LIẾN SƠN, LẬP THẠCH, VĨNH PHÚC www.violet.vn/quocbinh72 SỞ GD&ĐT VĨNH PHÚC --------------------------- ĐỀ CHÍNH THỨC KÌ THI CHỌN HSG VÒNG TỈNH – NĂM HỌC 2010 - 2011 ĐỀ THI MÔN: TIẾNG ANH LỚP 10 Dành cho học sinh các trường THPT không chuyên Thời gian làm bài: 180 phút (Đề thi gồm 05 trang) A.LISTENING I. Listen to the conversation between Mary and John then decide the given statements are true or false by writing T ( for true), F (for false) in the box provided. You will hear the conversation TWICE. 1. John has just published his latest book. 2. The topic of the book is about the Loch Ness Monster. 3. He has some problems with his computer. 4. He is going to Scotland to visit some of his friends. 5. He expects that when he gets back home he will find his word processor working properly. II. You will hear a man called Mike telling you about his plans for the evening. For the question 110, complete the notes which summarize what he says. You should write NO MORE THAN 3 WORDS into each of the blanks. You will hear the conversation TWICE. 1. Mike knows that Jill is coming to see him because she ____________________ 2. She should turn up just ____________________ 3. Mike has just bought himself ____________________ 4. Jill used to be mad on music and dancing but Mike liked watching ____________________ 5. Two things prevent Mike from going to a restaurant: one is the poor ____________________ 6. and the other is he ____________________ 7. Mike wants to take Jill to a special room at ____________________ 8. The furniture in the room is the wrong ____________________ 9. Mike wants Jill to try the room out but the museum closes ____________________ 10. In the end, Mike wonders about taking Jill to see Herbert and Muriet, who are ____________________ B. PHONETICS Pick out the word whose underlined part is pronounced differently from that of the other words. Identify your answer by writing the corresponding letter A, B, C or D on your answer sheet. 1. A. advance B. agrarian C. apartment D. attitude 2. A. minority B. performance C. helicopter D. floppy 3. A. eradicate B. represent C. longevity D. telecommunication 4. A. cultivation B. consumption C. subscriber D. shuttle 5. A. games B. partners C. fields D. reports C. LEXICAL – GRAMMAR I. Choose from the four options given (marked A, B, C and D) one best answer to complete each sentence. Identify your answer by writing the corresponding letter A, B, C or D on your answer sheet. 1. At the end of the meal, we had delicious __________ fruit juices. A. ending B. closing C. dessert D. last 2. Friendship is like a game. If you want to join it, you must learn the __________. A. laws B. facts C. customs D. rules BY ĐỖ BÌNH – THPT LIẾN SƠN, LẬP THẠCH, VĨNH PHÚC www.violet.vn/quocbinh72 3. Mr. Ralph _________Mathematics before he ________ to literature. A. had taught/ had changed B. was teaching/ changed C. taught/ was changing D. had taught/ changed 4. __________mad men are responsible for their actions. A. Fewer B. Few C. Less D. Little 5. The last World Cup was __________ in South Africa. A. broken B. happened C. held D. taken palce 6. I object _________ to the sea in this cold weather. A. to going B. going C. about going D. of going 7. What an excellent student she is! She almost has no _________ the exercise. A. difficult to finish B. difficulty to finish C. difficulty in finishing D. difficult finishing 8. I want to congratulate you __________ the contest. A. to win B. on winning C. at winning D. about winning. 9. You gave me precious help __________ I am extremely grateful. A. to which B. for that C. to that D. for which 10. I bought 20 roses, three __________ were destroyed before I came home. A. of them B. among them C. among which D. of which 11. John, __________ that his girlfriend went out with another guy, decided to say goodbye to her. A. believing B. believed C. to believe D. believe 12. Unfortunately, the university reacted unfavorably _________ the proposals. A. against B. by C. to D. with 13. Either I or they __________enjoying the party now. A. are B. is C. was D. were 14. Unlike my brother, who always fails to finish English tests, I am quite good _________ this subject. A. in B. about C. of D. at 15. I really must __________my car repaired soon. A. need B. have needed my car C. have D. let my car. II. Supply the correct form of the words in brackets. 1. The surgeon tried their best to save his life, but unfortunately the operation was _______.(SUCCESS) 2. He was very _______ of the work he had done. (PRIDE) 3. There were 50 _______ in the talent contest. (COMPETE) 4. Our _______ has lasted a lifetime. (FRIEND) 5. It was a complete _________ due to their poor planning. (FAIL) III. Fill in the blanks suitable particles or prepositions. 1. The bomb went _______ in a crowded street last night. 2. Peter arranged to meet Helen after work last night but she didn’t turn _______. 3. I know how wonderful you are. There no need to show _______. 4. There used to be a shop at the end of this street but it closed _______ a month ago. 5. Things will get easier as time goes _______. D. READING I. Read the following passage then choose the best answer from the four options (marked A, B, C and D) to complete the numbered blanks in the passage. Identify your answers by writing the corresponding letter A, B, C or D on your answer sheet. Many of the world’s people are concerned __________(1) the dwindling number of whales in the oceans and seas. They are worried because the number of whales is getting so __________(2). Whales are very large, aquatic animals. People have hunted whales __________(3) about the eleventh century. Certain types of whales have been hunted too much. Recently, their numbers have decreased so BY ĐỖ BÌNH – THPT LIẾN SƠN, LẬP THẠCH, VĨNH PHÚC www.violet.vn/quocbinh72 much that they are in _________(4) of becoming extinct. These concerned people are working to save the whales. Why do people want to save the whales? There are two important reasons. One reason is that whales help to keep a balance between plants and animals. People have __________(5) this balance. People get rid of their wastes by throwing them into the oceans and seas. People’s sewage and garbage increase the _________(6) of salt in ocean and sea water. The increased salt helps some plants and some very small animals _________(7). These plants and animals can be _________(8) to fish. Whales eat enormous amounts of plankton, the plants and animals that thrive in very salty water. __________(9), whales are very important because they keep the ocean environment clean enough for fish. In addition, because fish __________(10) necessary food for many people, people need whales, and many people want to save them. 1.A. of B. about C. to D. with 2. A. little B. great C. large D. small 3. A. in B. since C. at D. of 4. A. dangerous B. endangered C. danger D. dangered 5. A. distributed B. destroyed C. disturbed D. followed 6. A. number B. amount C. variety D. much 7. A. grow B. develop C. appear D. increase 8. A. good B. useful C. harmful D. bad 9. A. Therefore B. However C. But D. Moreover 10. A. bring B. provide C. give D. take II. Read the following passage and then choose the best answer by writing the letter A, B, C or D on the answer sheet. Line Just two months after the flight of Apollo 10, the Apollo 11 astronauts made their historic landing on the surface of the Moon. This momentous trip for humanity also provided scientists with an abundance of material for study; from rock and soil samples brought back from the Moon, scientists have been able to determine much about the composition of the Moon as well as to draw interferences about the development of the Moon from its (5) composition. The Moon soil that came back on Apollo 11 contains small bits of rock and glass which were probably ground from the large rocks when meteors impacted with the surface of the Moon. The bits of glass are spherical in shape and constitute approximately half of the Moon soil. (10) Scientists found no trace of animal or plant life in this soil. In addition to the Moon soil, astronauts gathered two basic types of rocks from the surface of the Moon: basalt and breccia. Basalt is cooled and hardened volcanic lava common to the Earth. Since basalt is formed under extremely high temperatures, the presence of this type of rock is an indication that the temperature of the Moon was once extremely hot. Breccia, the (15) other kind of rock brought back by the astronauts, was formed during the impact of falling objects on the surface of the Moon. This second type of rock consists of small pieces of rock compressed together by the force of impact. Gases such as hydrogen and helium were found in some of the rocks, and scientists believe that these gases were carried to the Moon by solar wind, the streams of gases that are constantly emitted by the Sun. BY ĐỖ BÌNH – THPT LIẾN SƠN, LẬP THẠCH, VĨNH PHÚC www.violet.vn/quocbinh72 1. The paragraph preceding the passage most likely discusses A. astronaut training B. the inception of the Apollo space program C. a different space trip D. previous Moon landings. 2. What is the subject of this passage? A. The Apollo astronauts B. Soil on the Moon. C. What the Moon is made of D. Basalt and breccia. 3. An “ abundance” in line 3 is A. a disorderly pile B. a wealthy bunch C. an insignificant proportion D. a large amount 4. According to the passage, what does Moon soil consist of? A. Hydrogen and helium B. Large chunks of volcanic lava C. Tiny pieces of stones and glass D. Stream of gases 5. Which of the following was NOT brought back to the Earth by the astronauts? A. Basalt B. Soil C. Breccia D. Plant life 6. An “ indication” in line 14 is A. an exhibition B. a clue C. a denial D. a dictate 7. According to the passage, breccia was formed A. when objects struck the Moon B. from volcanic lava C. when streams of gases hit the surface of the Moon D. from the interaction of helium and hydrogen. 8. It is implied in the passage that scientists believe that the gases found in the Moon rocks A. were not originally from the Moon B. were created inside the rocks C. traveled from the Moon to the Sun D. caused the Moon’s temperature to rise 9. The author’s purpose in this passage is to A. describe some rock and soil samples B. explain some of the things learned from space flights C. propose a new theory about the creation of the Moon D. demonstrate the difference between basalt and breccia 10. It can be inferred from the passage that A. the only items of importance that astronauts brought back from the Moon were rock and soil samples B. scientists learned relatively little from the Moon rock and soil samples C. scientists do not believe that it is necessary to return to the Moon D. rock and soil samples were only some of a myriad of significant items from the Moon. III. Read the following passage then choose the best answer from the four options (marked A, B, C and D) to complete the numbered blanks in the passage. Identify your answers by writing the corresponding letter A, B, C or D on your answer sheet. Stamp collecting! What a wonderful hobby! I began when I was only five. I used to _________(1) for the postman’s arrival, always eager to seize unwanted envelopes and tear off the corner with the stamp stuck __________(2) it. Once- I remember it all too clear- my mother and father were sunning themselves in the garden when the post __________(3) on the doormat. I heard the clatter of the letter flap and __________(4) went to investigate. There are four or five envelopes, all with very enticing stamps. __________(5) at the tender age of five I knew one doesn’t open mail addressed to other people. __________(6), tearing just the corners off the envelopes struck me as perfectly fair and allowable, and that’s what I did. I carefully tore as __________(7) to the stamps as possible, feeling that even the envelopes, which were addressed to my parents and not to me, should be treated with __________(8). There was nothing furtive in what I did. I knew my parents would see what I’d done, and I didn’t think there was any__________(9) in it. They always let me remove the corners after they’d opened them. Later, though, my father solemnly showed me his letters. They looked distinctly moth-eaten, with bites taken out of the corners and sides. I began to __________(10) what I’d done! BY ĐỖ BÌNH – THPT LIẾN SƠN, LẬP THẠCH, VĨNH PHÚC www.violet.vn/quocbinh72 1. A. stare B. watch C. look D. peer 2. A. over B. in C. on D. of 3. A. came B. was C. lay D. arrived 4. A. hurriedly B. hurried C. hurry D. hurrily 5. A. Because of B. Even C. Due to D.To 6. A. However B. But C. So D. Although 7. A. nearby B. next C. close D. round 8. A. gentleness B. caution C. honour D. respect 9. A. trouble B. harm C. wrong D. bad 10. A. accept B. admit C. realize D. confess. E. WRITING I. Finish each of the following sentences in such a way that it means exactly the same as the sentence printed before it. 1. If I were you, I’d look for another job I suggest ____________________________________________________________________ 2. He lost his money simply because he wasn’t careful. If __________________________________________________________________________ 3. Jack’s parents made him study for his exams. Jack was ____________________________________________________________________ 4. They believe he is mad. He _________________________________________________________________________ 5. I tried as hard as I could, but I just couldn’t get the money. No matter___________________________________________________________________ II. Complete the second sentence so that it has a similar meaning to the first sentence, using the given word. 1. She owns all this land now. (belongs) All _______________________________________________________________________ 2. Organized activities don’t interest Eva very much. ( interested) Eva_______________________________________________________________________ 3. If you run everyday, your breathing improves quickly. (running ) Daily______________________________________________________________________ 4. There are too many people for so few resources.. (enough) There______________________________________________________________________ 5. If it doesn’t stop snowing, we won’t get home ( unless) We _______________________________________________________________________ -------- THE END --------BY ĐỖ BÌNH – THPT LIẾN SƠN, LẬP THẠCH, VĨNH PHÚC www.violet.vn/quocbinh72 SỞ GD&ĐT VĨNH PHÚC --------------------------ĐỀ CHÍNH THỨC HƯỚNG DẪN CHẤM ĐỀ THI CHỌN HSG VÒNG TỈNH – NĂM HỌC 2010 - 2011 ĐỀ THI MÔN: TIẾNG ANH LỚP 11 Dành cho học sinh các trường THPT không chuyên Thời gian làm bài: 180 phút (Hướng dẫn chấm gồm 03 trang) Tổng điểm: 100 điểm A. LISTENING I. (10ts:.2 pts/item) 1. F 2. T II. (10 pts: 1,pt/item) 1. has (just) phoned 2. after 4 o’clock 3. a local paper 4. sport (on television/ TV) 5. food B. PHONETICS ( 5 pts: 01pt/ item) 1. D 2. B 3. T 4. F 5. T 6. can’t afford it 7. the Science Museum 8. shape(s) and size(s) 9. at 5 o’clock 10. (professional) musicians 3. A 4.C 5.D 1.C 2.D 6.A 7. C 11. A 12. C II. (5 pts: 1pt/ item) 1. unsuccessful 2. proud III. (5 pts : 1pt/item) 1. off 2. up 3. off D. READING I. ( 10pts: 1pt/ item) 3.D 8. B 13. A 4.B 9. D 14. D 5.C 10. D 15. C 3. competitors 4. friendship 5. failure 1.B 2.D 6.B 7. A II. (10 pts: 1pt /item) 3.B 8. C 4.C 9.A 5. A 10. B 1.C 2.C 6.B 7.A III. (10 pts: 1 pt /item) 1. B 2.C 6. A 7. C 3.D 8.A 4.C 9.B 5.D 10.D 3.D 8.D 4.A 9.B 5.B 10.C C. LEXICAL – GRAMMAR I. ( 15pts: 1pt/ item) 4. down 5. by E. WRITING I. ( 10 pts: 2pt/ item) 1. I suggest (that) you (should) look for another job. 2. If he had been careful, he wouldn’t have lost his money. 3. Jack was made to study for his exam (by his parents). BY ĐỖ BÌNH – THPT LIẾN SƠN, LẬP THẠCH, VĨNH PHÚC www.violet.vn/quocbinh72 4. He is believed to be mad. 5. No matter how hard I tried, I couldn’t get the money. II. ( 10 pts: 2pt/ item) 1. All this land belongs to her. 2. Eva isn’t interested in organized activities very much. 3. Daily running (quickly) improves your breathing (quickly). 4. There are not enough resources for so many people. 5. We won’t get home unless it stops snowing. LISTENING TAPESCRIPT Section I. Mary: Hello, John John: Hello, Mary. Mary: How is your latest book going? John: Oh, you mean the one I am writing on the Loch Ness monster? Well, it is going rather slowly at present. Mary: Oh, dear. Why’s that? John: Well, the inspiration’s there but everything is wrong. One day I tire myself out with the writing but the next day I can hardly keep myself awake. On top of that my work processor keeps breaking down. Mary: Perhaps you need to take a week off. John: Well, I am going up to Scotland to do some research for the book next week. Mary: That should be nice, especially if you see the monster! But you’ll still have your mind on your work the whole time. John: That’s true. In fact at this stage seeing the monster would be a mixed blessing as I’d have to rewrite and revise large chunk that I’ve already done. But going away gives the engineer a good chance to see where the trouble is. By the time I get back, my machine should be free of all its gremlins, even if I’m still under the weather. Section II. Mike: Hello, I’m Mike. Jill has just phoned to say that she is coming to visit me. She should arrive just after four o’clock. So here I am at the Tourist Information Office. I wanted to call at lunchtime but they were closed and I’ve had to wait until two o’clock. They’ve given me some leaflets and I’ve bought myself a local paper. I want to find somewhere good for us to go tonight. Jill used to be mad on music and dancing. And me, I was always keen on sport, well watching it on television, anyway. Jill was the one who went in for all the exciting things like skiing down mountains. There are plenty of restaurants, but the food isn’t always all that good and I probably can’t afford it anyway. Oh dear, I don’t know what to choose. Ah, this looks good. There is a guided tour of the local television studios. If you’re lucky you can actually see some of the stars filming new episodes of soap operas and things. What does it say here in the small print? “Small group only, who have to watch from behind the screen.” That sounds OK to me as long as there aren’t loads of other people, in which case they probably won’t let anyone watch at all. Oh, yes, there’s the Science Museum. Jill says she doesn’t like science but I’ve been on about taking her there for years and she is quite keen on the hands- on exhibit. There’s a room where all the walls are distorted and painted with misleading lines, and the furniture is all the wrong shapes and sizes. It deceives your eyes and the funny thing is that you stumble about and bump into things. Jill says it BY ĐỖ BÌNH – THPT LIẾN SƠN, LẬP THẠCH, VĨNH PHÚC www.violet.vn/quocbinh72 wouldn’t happen to her. She thinks she’s got a superior sense of balance or something. But she just doesn’t know what it is like in there. I’d certainly love to prove her wrong. Oh, dear, the Science Museum closes at five each evening. That’s not very good. Oh well, we can go tomorrow but that still leaves tonight to plan. I wonder whether she’d like to go and see Herbert and Muriel. She likes music, so why not spend the evening with two really top-class professionals? BY ĐỖ BÌNH – THPT LIẾN SƠN, LẬP THẠCH, VĨNH PHÚC www.violet.vn/quocbinh72 SỞ GD&ĐT ....... ———————— ĐỀ ĐỀ XUẤT KỲ THI CHỌN HSG LỚP 10 - THPT NĂM HỌC 2013- 2014 ĐỀ THI MÔN: TIẾNG ANH ———————————— PART I: VOCABULARY AND GRAMMAR. (7, 0 POINTS) I. Choose the best option to complete each of the sentences. (2,0 points) 1. The strike was ___ owing to a last minute agreement with the management. A. called off B. broken up C. set back D. put down 2. Lindsay’s excuses for being late are beginning to ___ rather thin. A. get B. turn C. wear D. go 3. ___ the people who come to this club are in their twenties and thirties. A. By and large B. Altogether C. To a degree D. Virtually 4. My cousin was nervous about being interviewed on television, but she rose to the ___ wonderfully. A. event B. performance C. incident D. occasion 5. The train service has been a ___ since they introduced the new schedules. A. shambles B. rumpus C. chaos D. fracas 6. Is an inexperienced civil servant ___ to the task of running the company. A. capable B. skilled C. eligible D. suited 7. We ___ have been happier in those days. A. can’t B. couldn’t C. might not D. must not 8. You’ve lived in the city for most of your life, so ___ you’re used to the noise. A. apparently B. presumably C. allegedly D. predictably 9. The storm ripped our tent to ___ A. slices B. shreds C. strips D. specks 10. He ____ so much harm on the nation during his regime that it has never fully recovered. A. indicted B. inferred C. induced D. inflicted 11. Hotel rooms must be ____ by 10 a. m, but luggage may be left with porters. A. vacated B. evacuated C. abandoned D. left 12. I do not think there is so much as a ____ of truth in that rumor. A. crumb B. speck C. grain D. pebble 13. He’s not exactly rich but he certainly earns enough money to ____ A. get through B. get by C. get on D. get up 14. I have very ____ feelings about the plan – it might possibly work or it could be a disaster. A. certain B. mixed C. doubtful D. troubled 15. The noise of the typewriter really ____ me off. I just couldn’t concentrate. A. put B. pulled C. set D. took 16. The sixth time he called me at night was the ____ A. last cause B. last straw C. touch and go D. hot air 17. All three TV channels provide extensive ____ of sporting events. A. broadcast B. network C. coverage D. vision 18. They seemed to be ____ to the criticism and just carried on as before. A. disinterested B. sensitive C. uncaring D. indifferent 19. “Shall we go out tonight?” - “____” A. Yes, I can B. Yes, we are C. Yes, we go D. Yes, let’s 20. It's no use ___ over ___ milk. A. crying/ spilt B. to cry/ spilling C. crying/ spilling D. crying/ to spill Your answers: 1. 2. 3. 4. 5. 6. 7. 8. 9. 10. 11. 12. 13. 14. 15. 16. 17. 18. 19. 20. II. Read the passage below. Use the word given in bold to form a word that fits in the numbered gap. (2, 5 points) BLACK WIDOW SPIDER The black widow spider’s notoriety is not without foundation. However, an element of exaggeration has led to certain ___ (1. CONCEPT) regarding its evil nature. BY ĐỖ BÌNH – THPT LIẾN SƠN, LẬP THẠCH, VĨNH PHÚC www.violet.vn/quocbinh72 Firstly, this spider is not as dangerous as is often thought. While it is indeed one of the most ___ (2. VENOM) species of spider, its venom being fifteen times stronger than that of the prairie rattlesnake, its bite injects such as amount of venom by ___ (3. COMPARE) that it is unlikely to kill humans. In fact, ___ (4. FATAL) are rare. Black widows bite only if they are touched or their web is threatened. Furthermore, only the adult female is poisonous. Those most at risk from the female are the spider’s natural pray-insects-and male black widow spiders. The latter are vulnerable as the female is ___ (5. SOLITUDE) by nature, and has been known to kill and eat the male after mating. Such ___ (6. OCCUR) are rare, but they explain how the spider got its name – and its reputation. Nevertheless, the ___ (7. PLEASE) effects of this spider’s bite should not be ___ (8. ESTIMATE), and if you live in a temperate climate and have a fireplace in your home, it is advisable to take ___ (9. CAUTION). Black widow spiders often inhabit wood piles, so you should wear gloves when handling firewood. Furthermore, since black widow spiders are ___ (10. RESIST) to many insecticides, you should regularly clean out likely hiding places. Your answers: 1. 2. 3. 4. 5. 6. 7. 8. 9. 10. III. Complete each sentence with the correct form of ONE of the two-word verbs. (2, 0 point) bring out do up slow down save up hang over mix up grow up go with dress up run for 1. Silence ___the theatre as the audience awaited the opening curtain with expectation and excitement. 2. Having seen a sharp bend ahead, Tim pressed hard on the brake pedal to ___. 3. Charles Dickens was born near Portsmouth, Hampshire on 7 February 1812, but relocated to and ___ in Camden Town in London. 4. Barrack Obama has decided to ___ the American presidency in the election that will take place next year. 5. Because I hate ___ my shoes, I have bought a pair of shoes without any laces. 6. Sarah wanted to buy some curtains that would ___ his furniture, so she had brought a photo of her sofa with her to the store. 7. Kate had been ___ in order to buy a new laptop, but then she decided to use the money on the guitar lessons instead. 8. Even though two of the bands are dead, a new ‘The Beatle’ album called Love was recently ___. 9. Brendan was worried about having to ___ for the boss’ retirement dinner as he didn’t own any formal clothes. 10. He ___ his grandmother’s phone number with his girlfriend’s, which led to some embarrassment for him. Your answers: 1. 2. 3. 4. 5. 6. 7. 8. 9. 10. IV. The passage below contains TEN mistakes. Underline them and write the correct forms in the numbered boxes. (2, 5 points) Large animals inhabit the desert have evolved adaptations for reducing the effects of extreme hot. One adaptation is to be light in color, and to reflect the Sun's rays. Desert mammals also depart from the normal mammalian practice of maintaining a constantly body temperature. Instead of try to keep down the body temperature inside the body, what would involve the expenditure of water and energy, desert mammals allow their temperatures rise to what would normally be fever height, and temperatures as high as 46 degree Celsius have been measured in Grant's gazelles. The overheated body cools down during the cold desert night, and indeed the temperature may fall unusual low by dawn, as low as 34 degrees Celsius in the camel. This is a advantage since the heat of the first few hours of daylight absorb in warming up the body. Your answers: 1. 2. 3. 4. 5. 6. 7. 8. 9. 10. PART II: READING COMPREHENSION. (6, 0 POINTS) I. Read the passage and fill in each of the blanks with ONE suitable word. (2,0 points) True relaxation is most certainly not a matter of flopping down in front of the television with a welcome drink. Nor is it about drifting (1) ___ an exhausted sleep. Useful though these responses to tension and over-tiredness (2) ___ be, we should distinguish between them and conscious relaxation in (3) ___ of quality and effect. (4) ___ of the level of tiredness, real relaxation is a state of alert yet at the same time passive awareness, in which our bodies are (5) ___ rest while our minds are awake. Moreover, it is as natural for a healthy person to be relaxed when moving as resting. (6) ___ relaxed in action means we bring the appropriate energy to everything we do, so as to have a feeling of healthy tiredness by the end of the BY ĐỖ BÌNH – THPT LIẾN SƠN, LẬP THẠCH, VĨNH PHÚC www.violet.vn/quocbinh72 day, (7) ___ than one of exhaustion. Unfortunately, as a result of living in today’s competitive world, we are under constant strain and have difficulty in coping, (8) ___ alone nurturing our body’s abilities. What needs to be rediscovered is conscious relaxation. With (9) ___ in mind we must apply ourselves to understanding stress and the nature of its causes (10) ___ deep-seated. Your answers: 1. 2. 3. 4. 5. 6. 7. 8. 9. 10. II. Read the text and do the tasks as follow. (2, 0 points) OUT OF THE ASHES Paragraph A On the afternoon of 30th August 1989, fire broke out at Uppark, a large eighteenth century house in Sussex. For a year builders had been replacing the lead on the roof, and by a stroke of irony, were due to finish the next day, on August 31st. Within fifteen minutes of the alarm being sounded, the fire brigade had arrived on the scene, though nothing was to survive of the priceless collection on the first floor apart from an oil painting of a dog which the firemen swept up as they finally retreated from the blaze. But due to the courage and swift action of the previous owners, the Meade-Featherstonhaugh family, and the staff, stewards and visitors to the house, who formed human chains to pass the precious pieces of porcelain, furniture and paintings out on to the lawn, 95 percent of the contents from the ground floor and the basement were saved. As the fire continued to rage, the National Trust’s conservators were being mobilised, and that evening local stationers were especially opened to provide the bulk supplies of blotting paper so desperately needed in the salvage operation. Paragraph B The following morning, Uppark stood open to the sky. A sludge of wet charcoal covered the ground floor and basement, and in every room charred and fallen timbers lay amongst the smoke. It was a scene of utter devastation. Paragraph C After the initial sense of shock, the days which followed the fire were filled with discoveries. Helped by volunteers, the National Trust’s archaeologists and conservators swung into action, first of all marking the site out into a grid and then salvaging everything down to the last door handle. The position of each fragment was recorded, and all the debris was stored in countless dustbins before being sifted and categorised. Paragraph D There was great excitement as remnants of the lantern from the Staircase Hall were pulled out from the debris of two fallen floors, and also three weeks later when the Red Room carpet, thought to have been totally lost, was found wrapped around the remains of a piano. There was a lucky reprieve for the State Bed too. Staff who had left the scene at 3 a.m on the night of the fire had thought its loss was inevitable, but when they returned the next morning it had escaped largely undamaged. Firemen, directed by the National Trust’s conservators from outside the Tapestry Room window, dismantled the silk-hung bed and passed it out piece by piece. Twenty minutes later the ceiling fell in. Paragraph E The scale of the task to repair Uppark was unprecedented in the National Trust. The immediate question was whether it should be done at all. A decision had to be taken quickly, as the building was unsound and whatever had not been damaged by the fire was exposed to the elements. Within a month, after consulting many experts and with the agreement of the National Trust’s Executive Committee, the restoration programme began. It was undertaken for three main reasons. After the fire it had become apparent just how much remained of the structure with its splendidly decorated interiors; to have pulled the house down, as one commentator suggested, would have been vandalism. Also the property was covered by insurance, so the repairs would not call upon the National Trust’s own funds. Lastly, much had been saved of the fine collection acquired especially for Uppark from 1747 by Sir Matthew Featherstonhaugh and his son Harry. These objects belonged nowhere else, and complete restoration of the house would allow them to be seen and enjoyed again in their original setting. Paragraph F The search for craftsmen and women capable of doing the intricate restoration work was nation-wide. Once the quality and skill of the individual or company had been ascertained, they had to pass an economic test, as every job was competitively tendered. This has had enormous benefits because not only have a number of highly skilled people come to the fore - woodcarvers for example, following in the footsteps of Grinling Gibbons - but many of them, for example plasterers, have relearnt the skills of the seventeenth and eighteenth centuries which can now be of use to other country house owners when the need arises. Paragraph G In June 1994 the building programme was completed, on time and on budget. The total cost of the work to repair the house and its contents came to be nearly £20 million, largely met from insurance. In addition, it made economic sense for the National Trust to invest time and money in upgrading water and heating systems, installing modern environmental controls, and updating fire and security equipment. Paragraph H BY ĐỖ BÌNH – THPT LIẾN SƠN, LẬP THẠCH, VĨNH PHÚC www.violet.vn/quocbinh72 The final stages of restoration and the massive programme of reinstallation took eight months. The family and the room stewards were visibly moved when returning to their old haunts, perhaps the best testament that the spirit of Uppark had not died. But the debate will no doubt continue as to whether or not it was right to repair the house after the fire. The National Trust has done its best to remain true to Uppark; it is for others to judge the success of the project. Note: The National Trust is a charitable organisation in Britain set up over a hundred years ago to preserve the national heritage. Questions 1–6. The text has eight paragraphs, A–H. Which paragraphs contain the following information? Write the appropriate letters, A–H, in the boxes. 1. The procedure for sorting through the remains of the fire. 2. How Uppark looked after the fire. 3. Improvements made to the rebuilt Uppark. 4. The selection of people to carry out the repair work. 5. Why the National Trust chose to rebuild Uppark. 6. How people reacted to the rebuilt Uppark. Questions 8–10. Answer the questions below. Choose NO MORE THAN THREE WORDS from the text for each answer. Write your answers in the boxes. 7. On what date in 1989 should the original repairs to the roof have been completed? 8. By what method were things rescued immediately from the burning house? 9. After the fire, what did the conservators require large quantities of immediately? 10. Into what did the conservators put material recovered from the fire? Your answers: 1. 2. 3. 4. 5. 6. 7. 8. 9. 10. III. Read the following passage and choose the best answer (A, B, C or D) to each question. Write your answer in the numbered box. (2, 0 points) A recent survey of crime statistics shows that we are all more likely to be burgled now than 20 years ago and the police advise everyone to take a few simple precautions to protect their homes. The first fact is that burglars and other intruders prefer easy opportunities, like a house which is very obviously empty. This is much less of a challenge than an occupied house, and one which is well-protected. A burglar will wonder if it is worth the bother. There are some general tips on how to avoid your home becoming another crime statistic. Avoid leaving signs that your house is empty. When you have to go out, leave at least one light on as well as a radio or television, and do not leave any curtains wide open. The sight of your latest music centre or computer is enough to tempt any burglar. Never leave a spare key in a convenient hiding place. The first place a burglar will look is under the doormat or in a flower pot and even somewhere more 'imaginative' could soon be uncovered by the intruder. It is much safer to leave a key with a neighbour you can trust. But if your house is in a quiet, desolate area be aware that this will be a burglar's dream, so deter any potential criminal from approaching your house by fitting security lights to the outside of your house. But what could happen if, in spite of the aforementioned precautions, a burglar or intruder has decided to target your home. Windows are usually the first point of entry for many intruders. Downstairs windows provide easy access while upstairs windows can be reached with a ladder or by climbing up the drainpipe. Before going to bed you should double-check that all windows and shutters are locked. No matter how small your windows may be, it is surprising what a narrow gap a determined burglar can manage to get through. For extra security, fit window locks to the inside of the window. What about entry via doors? Your back door and patio doors, which are easily forced open, should have top quality security locks fitted. Even though this is expensive it will be money well spent. Install a burglar alarm if you can afford it as another line of defence against intruders. A sobering fact is that not all intruders have to break and enter into a property. Why go to the trouble of breaking in if you can just knock and be invited in? Beware of bogus officials or workmen and, particularly if you are elderly, fit a chain and an eye hole so you can scrutinise callers at your leisure. When you do have callers never let anybody into your home unless you are absolutely sure they are genuine. Ask to see an identity card, for example. If you are in the frightening position of waking in the middle of the night and think you can hear an intruder, then on no account should you approach the intruder. It is far better to telephone the police and wait for help. 1. A well-protected house ………… A. is less likely to be burgled. B. is regarded as a challenge by most criminals. C. is a lot of bother to maintain. D. is very unlikely to be burgled. 2. According to the writer, we should ………… A. avoid leaving our house empty. B. only go out when we have to. BY ĐỖ BÌNH – THPT LIẾN SƠN, LẬP THẠCH, VĨNH PHÚC www.violet.vn/quocbinh72 C. always keep the curtains closed. D. give the impression that our house is occupied when we go out. 3. The writer thinks that hiding a key under a doormat or flower pot ………… A. is predictable. B. is useful. C. is imaginative. D. is where you always find a spare key. 4. What word best replaces “desolate” in paragraph 4? A. isolated B. populous C. dissatisfying D. depressing 5. The phrase “aforementioned precautions” in paragraph 5 refers to steps that ………… A. will tell a burglar if your house is empty or not. B. are the most important precautions to take to make your home safe. C. will stop a potential burglar. D. will not stop an intruder if he has decided to try and enter your home. 6. Gaining entry to a house through a small window ………… A. is surprisingly difficult. B. is not as difficult as people think. C. is less likely to happen than gaining entry through a door. D. is tried only by very determined burglars. 7. According to the writer, window locks, security locks and burglar alarms ………… A. cost a lot of money but are worth it. B. are good value for money. C. are luxury items. D. are absolutely essential items. 8. The writer argues that fitting a chain and an eye hole………… A. will prevent your home being burgled. B. avoids you having to invite people into your home. C. is only necessary for elderly people. D. gives you time to check if the visitor is genuine. 9. What word best replaces “scrutinise” in paragraph 7? A. glance B. gaze C. search D. examine 10. The best title for the text is ………… A. Increasing household crime. B. Protecting your home from intruders. C. Burglary statistics. D. What to do if a burglar breaks into your home. Your answers: 1. 2. 3. 4. 5. 6. 7. 8. 9. 10. PART III. WRITING. (4,0 POINTS) I. Complete each of the sentences so that it has similar meaning to the given one, using the word given in bold. Do NOT change the form of the word given. (1, 0 point) 1. Although Joe kept on attempting to contact his cousin, he didn't manage to speak to her until the next day. → Despite repeated ........................ TOUCH................... his cousin, Joe didn't manage to speak to her until the next day. 2. When it comes to computer games, Jack is a real expert. CONCERNED → As far ............................................................................., Jack is a real expert. 3. David said that the accident was his fault. TOOK → David .................................................................. the accident. 4. If he doesn’t get that job, who knows what he’ll do. KNOWING → If he doesn’t get that job, ........................................................... what he’ll do. 5. The stranded climber would never have been rescued if his brother hadn’t had an ingenious plan. INGENUITY → But .......................................................... plan, the stranded climber would never have been rescued. II. Use the word given and make any necessary additions to complete a new sentence in such a way that it is as similar as possible to the original one. Do NOT change the form of the word given. (1, 0 point) 1. They will try John for murder at the High Court next week. (trial) → John …………………………………………………………..……………………………………… 2. How do our sales compare with those of other firms? (relation) → How do …………………………………………………………………..…………………………… 3. The number of accidents has gone down steadily since the speed limit was imposed. (decline) → There has ……………………………………………………………..……………………………… 4. This new record is certain to sell a lot of copies. (doubt) → There is…………………………………………………………..…………………………………… 5. I certainly won’t change my mind about resigning. (question) → My changing …………………………………………………….…………………………………… III. The charts below give information on the ages of the populations of Yemen and Italy in 2000 and projections for 2050. Summarize the information by selecting and reporting the main features and make comparisons where relevant. (2, 0 points) BY ĐỖ BÌNH – THPT LIẾN SƠN, LẬP THẠCH, VĨNH PHÚC www.violet.vn/quocbinh72 2000 2050 YEMEN 37,0% 46,3% 50,1% 57,3% 5,7% 3,6% 2000 24,1% 2050 ITALY 46,2% 42,3% 14,3% 61,6% 11,5% 0-14 years 15-59 years 60+ years ……………………………………………………………………………………………………………………………… ……………………………………………………………………………………………………………………………… ……………………………………………………………………………………………………………………………… ……………………………………………………………………………………………………………………………… ……………………………………………………………………………………………………………………………… ……………………………………………………………………………………………………………………………… ……………………………………………………………………………………………………………………………… ……………………………………………………………………………………………………………………………… ……………………………………………………………………………………………………………………………… ……………………………………………………………………………………………………………………………… ……………………………………………………………………………………………………………………………… ……………………………………………………………………………………………………………………………… ……………………………………………………………………………………………………………………………… ……………………………………………………………………………………………………………………………… ……………………………………………………………………………………………………………………………… ……………………………………………………………………………………………………………………………… ……………………………………………………………………………………………………………………………… ……………………………………………………………………………………………………………………………… ……………………………………………………………………………………………………………………………… ……………………………………………………………………………………………………………………………… ……………………………………………………………………………………………………………………………… ……………………………………………………………………………………………………………………………… ……………………………………………………………………………………………………………………………… ……………………………………………………………………………………………………………………………… ……………………………………………………………………………………………………………………………… BY ĐỖ BÌNH – THPT LIẾN SƠN, LẬP THẠCH, VĨNH PHÚC www.violet.vn/quocbinh72 ……………………………………………………………………………………………………………………………… ……………………………………………………………………………………………………………………………… ……………………………………………………………………………………………………………………………… ……………………………………………………………………………………………………………………………… ……………………………………………………………………………………………………………………………… ……………………………………………………………………………………………………………………………… ……………………………………………………………………………………………………………………………… ……………………………………………………………………………………………………………………………… ……………………………………… ……………………………………………………………………………………………………………………………… ……………………………………………………………………………………………………………………………… ……………………………………………………………………… THE END PART PART I: LEXICOGRAMMAR I. Choose the best option to complete each of the sentences. (2,0 points) (7,0/20 points) II. Read the passage below. Use the word given in bold to form a word that fits in the numbered gap. (2,5 points) III. Complete each sentence with the correct form of ONE ANSWERS MARK 1. A 0.1 p 2. C 3. A 4. D 5. A 6. D 7. B 8. B 9. B 10. C 11. A 12. C 13. B 14. B 15. A 16. B 17. C 18. D 19. D 20. A 1. misconception 2. venomous 3. comparison 4. fatalities 5. solitary 6. occurences 7. unpleasant 8. underestimated 0.1 p 0.1 p 0.1 p 0.1 p 0.1 p 0.1 p 0.1 p 0.1 p 0.1 p 0.1 p 0.1 p 0.1 p 0.1 p 0.1 p 0.1 p 0.1 p 0.1 p 0.1 p 0.1 p 0.25 p 0.25 p 0.25 p 0.2 5p 0.2 5p 0.2 5p 0.25 p 0.25 p 9. precautions 10. resistant 1. hung over 2. slow down 3. grew up 4. run for 0.25 p 0.2 5p 0.2 p 0.2 p 0.2 p 0.2 p BY ĐỖ BÌNH – THPT LIẾN SƠN, LẬP THẠCH, VĨNH PHÚC www.violet.vn/quocbinh72 of the two-word verbs. (2,0 point) IV. The passage below contains TEN mistakes. Underline them and write the correct forms in the numbered boxes. (2,5 points) PART II: READING: (6,0/20 points) I. Read the passage and fill in each of the blanks with ONE suitable word. (2,0 points) II. Read the text and do the tasks as follow. (2,0 points) III. Read the following passage and choose the best answer (A, B, C or D) to each question. (2,0 points) PART III: WRITING: (4,0/20 points) I. Complete each of the sentences. (1,0 point) 5. doing up 6. go with 7. saving up 8. brought out 9. dress up 10. mixed up 1. inhabit inhabiting/which(that) inhabit 2. hot heat 3. constantly constant 4. try trying 5. what which 6. rise to rise 7. degree degrees 8. unusual unusually 9. a an 10. absorb absorbed 1. into 2. might/may/can 3. terms 4. regardless/irrespective 5. at 6. Being 7. rather 8. let 9. that/this 10. how 1. C 2. B 3. G 4. F 5. E 6. H 7. August 31st 8. human chain 9. blotting paper 10. dustbins 1. A 2. D 3. A 4. A 5. C 6. B 7. A 8. D 9. D 10. B 0.2 p 0.2 p 0.2 p 0.2 p 0.2 p 0.2 p 0.5 p 0.5 p 0.5 p 0.5 p 0.5 p 0.5 p 0.5 p 0.5 p 0.5 p 0.5 p 0.2 p 0.2 p 0.2 p 0.2 p 0.2 p 0.2 p 0.2 p 0.2 p 0.2 p 0.2 p 0.2 p 0.2 p 0.2 p 0.2 p 0.2 p 0.2 p 0.2 p 0.2 p 0.2 p 0.2 p 0.2 p 0.2 p 0.2 p 0.2 p 0.2 p 0.2 p 0.2 p 0.2 p 0.2 p 0.2 p 1. Despite repeated [attempts/efforts to get in touch with] his cousin, Joe didn't manage to speak to her until the next day. 2. As far [as computer games are concerned], Jack is a real expert. 3. David took [the blame/responsibility for] the accident. 4. If he doesn’t get that job, [there is no knowing] 0.2 p 0.2 p 0.2 p 0.2 p BY ĐỖ BÌNH – THPT LIẾN SƠN, LẬP THẠCH, VĨNH PHÚC www.violet.vn/quocbinh72 II. Use the word given and make any necessary additions to complete a new sentence . (1,0 point) III. Summarize the information by selecting and reporting the main features and make comparisons where relevant. (2,0 points) what he’ll do. 5. But [for the ingenuity of his brother’s] plan, the stranded climber would never have been rescued. 1. John will stand trial murder at the High Court next week. 2. How do our sales stand in relation to those of other firms? 3. There has been a steady decline in the number of accidents since the speed limit was imposed. 4. There is no doubt that this new record will sell a lot of copies. 5. My changing my mind about resigning is out of question. 1. Content: a. Providing main ideas and details. (summarizing the information, reporting the main features, and make comparisons where relevant) b. Communicating intentions sufficiently and effectively. 2. Organization and presentation: a. Ideas are well-organized and presented with coherence, cohesion, and clarity. b. The paragraph is well-structured. 3. Language: a. Variety of appropriate vocabulary and structures. b. Good use of grammatical structures. 4. Handwriting, punctuation, and spelling: a. Intelligible handwriting. b. Good punctuation and no spelling mistakes. 0.2 p 0.2 p 0.2 p 0.2 p 0.2 p 0.2 p 1.0 0.25 0.5 0.25 BY ĐỖ BÌNH – THPT LIẾN SƠN, LẬP THẠCH, VĨNH PHÚC www.violet.vn/quocbinh72 SỞ GD&ĐT ....... ———————— ĐỀ ĐỀ XUẤT KỲ THI CHỌN HSG LỚP 10 - THPT NĂM HỌC 2014- 2015 ĐỀ THI MÔN: TIẾNG ANH ———————————— B. PHONETICS Choose the word whose underlined part is pronounced differently from that of the other three(5 pts) B. contact C. background D. formality 1. A. socialize B. contact C. conscious D. common 2. A. prolong 3. A. African B. Japanese C. partner D. Canada 4. A. signal B. sign C. colleague D. regard B. trustworthy C. clothing D. brother 5. A. cloth C. LEXICAL - GRAMMAR I. Choose from the four options given marked A, B, C and D one best answer to complete each sentence. Identify your answer by writing the corresponding letter A, B, C or D on your answer sheet. (15 pts) 1. Please_______________ favour. A. do me a B. make me a C. do my D. make my 2. Mrs. Edwards…______________ history for 35 years, and is retiring soon. A. teaches B. is teaching C. taught D. has been teaching 3. There have been protests from animal right groups about ____________ on animals. A. experience B. experiments C. expiration D. trials 4. Faraday’s accomplishments seem more wonderful when we realize that he had very___________ education. A. a few B. few C. a little D. little 5. A: “What are the office hours where you work?” B: “_______., everyone comes in at eight and leaves at five.” A. General B. In generally C. In the general D. In general 6. Lots of people________________ Yoga to relax. A. practice B. take up C. give up D. take in 7. It isn’t quite________.. that he will turn up at the meeting. A. certain B. exact C. right D. sure 8. After waiting for an hour he realized that the bus was__________________ to come. A. improbable B. impossible C. uncertain D. unlikely 9. _______________ being tired, I went for a picnic with my family. A. That B. Though C. Since D. Despite 10. You_________________ have told that joke. Everybody has heard it too often. A. can’t B. mustn’t C. shouldn’t D. needn’t 11. At around two years of age, many children regularly produce sentences_____ three or four words. A . are containing B. containing C. contain D. contains 12. The first plant-like organisms probably ___________ in the sea, perhaps three billion years ago. A . life B. living C. lived D. it was living 13. These tennis courts don't ____________ very often. Not many people want to play. A . got used B. used C. get used D. get use 14. I would rather ___________ nothing than __________ that book. A . do - to read B. do - read C. doing - reading D. done - read 15. Do you want _______ with you or do you want to go alone? A . me coming B. me to come C. that I come D. that I will come II. The four underlined parts of the sentence are marked A, B, C or D. You are to identify the one underlined word or phrase that would not be acceptable standard written English. Write your answers on your answer sheet. (20 pts) 1. Draft horses are the tallest, most powerful and heavy group of horses. A B C D 2. Acting teacher Stella Adler played a vital role in the develop of the Method Schooling of Acting. A B C D 3. Before diamonds can be used as jewels, they must be cut and polish. A B C D 4. Of all mammals, dolphins are undoubtedly between the friendliest to humans. A B C D 5. Working all night long, Jim felt extreme hungry and just wanted to have a sandwich. A B C D 6. Harmony, melody and rhythm are important elements in mostly forms of music. A B C D 7. Medical students must learn both the theory and the practice of medicines. A B C D BY ĐỖ BÌNH – THPT LIẾN SƠN, LẬP THẠCH, VĨNH PHÚC www.violet.vn/quocbinh72 8. Archeological sites sometimes are revealed when the construction of roads and buildings. A B C D 9. Yolanda ran over to me and breathless informed the failure of our team. A B C D 10. All root vegetables grow underground, and not all vegetables that grow underground are roots. A B C D D. READING I. Read the following passage and fill an appropriate word in each blank. (10 pts) Fish live almost (1) ___ They are found in the near freezing waters of the Artic and in the steaming (2)____ in the tropic jungles. They live in roaring streams and in quiet (3)_____ rivers. Some fish make long journeys (4) ____ the ocean. Others spend most of their life buried in sand on the ocean (5) ______. Most fish can’t (6)_____ water; yet some survive for months in dried-up riverbeds. Fish have enormous importance (7)____ man. They provide food for millions of people. Fishermen catch them for sports, and many people (8)____ them as pets. Fish are also important in the balance of nature. They eat plants and animals and in turn, become (9)____ for plants and animals. Fish thus help keep in (10 ) _____ the total number of plants and animals on the earth. II. Choose the most suitable answer from the four options marked A, B, C and D to complete each of the numbered gaps in the passage below. Write your answer on the answer sheet. (10 pts) You've (1)________ heard someone say that the reason you caught a cold was that you were 'run down'. People generally accept that if you are (2)______ pressure or run down you are more likely to get ill. But is this really true (3)_______ is it just an old wives' tale? The problem with old sayings like this is that it's often difficult to (4)______ whether they are true or not. For example, if you ask people with colds whether they are feeling run down, they are (5)_______ certain to say yes. People without colds are more likely to say no. So, how do you (6)______ if it's the cold that is making them (7)_____ run down or the fact that they are run down that is making them more likely to catch a cold? Now it seems there is a scientific answer to this question. And answer is yes - if you are run down or under stress, you are more likely to catch a cold. Scientists at the Common Cold Research Centre ran a series of tests. They got volunteers without colds to attend the centre, where they were first given a questionnaire to complete. The questions measured the (8)_____ of stress each volunteer was under. The volunteers were then deliberately infected with a cold virus and left to see whether they developed a cold. After many people had been tested, the researchers looked for a (9)____ between the measured stress level and the chances of catching a cold. They found that the higher the stress, the more likely the person was to catch a cold. At the moment it is not known why stress makes someone more likely to become ill, but now (10)_____ the link has been found, researchers might be able to investigate further. They may even find different ways of combating disease. 1. A. perhaps B. maybe C. possibly D. probably 2. A. under B. behind C. at D. above 3. A. and B. or C. but D. so 4. A. show B. express C. state D. prove 5. A. nearly B. mostly C. almost D. surely 6. A. tell B. speak C. say D. suggest 7. A. be B. feel C. become D. get 8. A. amount B. number C. mass D. heap 9. A. combination B. joint C. link D. relation 10. A. that B. when C. because D. as III. Read the passage below and choose the best answer from the four options marked A, B, C or D. Identify your answer by writing the corresponding letter A, B, C or D on the answer sheet. (20 pts) As many as one thousand years ago in the Southwest, the Hopi and Zuni Indians of North America were building with adobe-sun-baked brick plastered with mud. Their homes looked remarkably like modern apartment houses. Some were four stories high and contained quarters for perhaps a thousand people, along with storerooms for grain and other goods. These buildings were usually put up against cliffs, both to make construction easier and for defense against enemies. They were really villages in themselves, as later Spanish explorers must have realized since they called them “pueblos”, which is Spanish for towns. The people of the pueblos raised what are called “the three sisters”- corn, beans, and squash. They made excellent pottery and wove marvelous baskets, some so fine that they could hold water. The Southwest has always been a dry country, where water is scarce. The Hopi and Zuni brought water from streams to their fields and gardens through irrigation ditches. Water was so important that it played a major role in their religion. They developed elaborate ceremonies and religious rituals to bring rain. The way of life of less – settled groups was simpler and more strongly influenced by nature. Small tribes such as the Shoshone and Ute wandered the dry and mountainous lands between the Rocky Mountains and the Pacific Ocean. They gathered seeds and hunted small animals such as rabbits and snakes. In the Far North the ancestors of today’s Inuit hunted seals, walruses, and the great whales. They lived right on the frozen seas in shelters called igloos built of blocks of packed snow. When summer came, they fished for salmon and hunted the lordly caribou. The Cheyenne, Pawnee and Sioux tribes, known as the Plains Indians, lived on the grasslands between the Rocky Mountains and the Mississippi River. BY ĐỖ BÌNH – THPT LIẾN SƠN, LẬP THẠCH, VĨNH PHÚC www.violet.vn/quocbinh72 They hunted bison, commonly called the buffalo. Its meat was the chief food of these tribes, and its hide was used to make their clothing and the covering of their tents and tips. 1. What does the passage mainly discuss? A. The architecture of early America Indian buildings B. The movement of American Indians across North America C. Ceremonies and rituals of American Indians D. The way of life of American Indian tribes in early North America 2. According to the passage, the Hopi and Zuni typically built their homes ________. A. in valleys B. next to streams C. on open plains D. against cliffs 3. The word “They” refers to ____________. A. goods B. buildings C. cliffs D. enemies 4. It can be inferred from the passage that the dwellings of the Hopi and Zuni were _______. A. very small B. highly advanced C. difficult to defend D. quickly constructed 5. The author uses the phrase “the three sisters” refers to_____________. A. Hopi women B. family members C. important crops D. rain ceremonies 6. Which of the following is true of the Shoshone and Ute? A. They were not as settled as the Hopi and Zuni B. They hunted caribou. C. They built their homes with adobe. D. They did not have many religious ceremonies. 7. According to the passage, which of the following tribes lived in the grasslands? A. The Shoshone and Ute B. The Cheyenne C. The Hopi and Zuni D. The Pawnee and Inuit 8. Which of the following animals was the most important to the Plains Indians? A. The salmon B. The caribou C. The seal D. The buffalo 9. The author gives an explanation for all of the following words EXCEPT______. A. adobe B. pueblos C. caribou D. bison 10. The author groups North American Indians according to their _________. A. tribes and geographical regions B. arts and crafts C. rituals and ceremonies D. date of appearance on the continent E. WRITING Finish each of the following sentences so that it means the same as the printed one. (20 pts) 1. I didn’t arrive in time to see her. → I wasn’t early______________________________________________________________. 2. We couldn’t drive because of the fog. → The fog prevented__________________________________________________________ 3. A train leaves at eight o’ clock every morning. → There is ______________________________________________________________. 4. John asked if it was the blue one or the green she wanted. → “Which _____________________________________ ?” 5. I haven’t eaten this kind of food before. → This is the first_________________________________________________________. 6. Only if you work hard now have you any chance of success. → Your chance____________________ 7. He tried very hard to give up smoking. → He made great_____________________________. 8. An aerial is not required with this radio. → You don’t_____________________________ 9. When he stops smoking, he’ll feel better. → The sooner__________________________. 10. The wedding was held despite the rain. → The wedding took____________ __________ THE END ___________ SỞ GD&ĐT VĨNH PHÚC ———————— ĐỀ CHÍNH THỨC KỲ THI CHỌN HSG LỚP 1…. - THPT NĂM HỌC 201… - 201… (ĐỀ LUYỆN 06) ĐỀ THI MÔN: TIẾNG ANH ———————————— BY ĐỖ BÌNH – THPT LIẾN SƠN, LẬP THẠCH, VĨNH PHÚC www.violet.vn/quocbinh72 B. PHONETICS Choose the word whose underlined part is pronounced differently from that of the other three(5 pts) B. contact C. background D. formality 1. A. socialize 2. A. prolong B. contact C. conscious D. common 3. A. African B. Japanese C. partner D. Canada B. sign C. colleague D. regard 4. A. signal 5. A. cloth B. trustworthy C. clothing D. brother C. LEXICAL - GRAMMAR I. Choose from the four options given marked A, B, C and D one best answer to complete each sentence. Identify your answer by writing the corresponding letter A, B, C or D on your answer sheet. (15 pts) 1. Please_______________ favour. A. do me a B. make me a C. do my D. make my 2. Mrs. Edwards…______________ history for 35 years, and is retiring soon. A. teaches B. is teaching C. taught D. has been teaching 3. There have been protests from animal right groups about ____________ on animals. A. experience B. experiments C. expiration D. trials 4. Faraday’s accomplishments seem more wonderful when we realize that he had very___________ education. A. a few B. few C. a little D. little 5. A: “What are the office hours where you work?” B: “_______., everyone comes in at eight and leaves at five.” A. General B. In generally C. In the general D. In general 6. Lots of people________________ Yoga to relax. A. practice B. take up C. give up D. take in 7. It isn’t quite________.. that he will turn up at the meeting. B. exact C. right D. sure A. certain 8. After waiting for an hour he realized that the bus was__________________ to come. A. improbable B. impossible C. uncertain D. unlikely 9. _______________ being tired, I went for a picnic with my family. A. That B. Though C. Since D. Despite 10. You_________________ have told that joke. Everybody has heard it too often. A. can’t B. mustn’t C. shouldn’t D. needn’t 11. At around two years of age, many children regularly produce sentences_____ three or four words. A . are containing B. containing C. contain D. contains 12. The first plant-like organisms probably ___________ in the sea, perhaps three billion years ago. A . life B. living C. lived D. it was living 13. These tennis courts don't ____________ very often. Not many people want to play. A . got used B. used C. get used D. get use 14. I would rather ___________ nothing than __________ that book. A . do - to read B. do - read C. doing - reading D. done - read 15. Do you want _______ with you or do you want to go alone? A . me coming B. me to come C. that I come D. that I will come II. The four underlined parts of the sentence are marked A, B, C or D. You are to identify the one underlined word or phrase that would not be acceptable standard written English. Write your answers on your answer sheet. (20 pts) 1. Draft horses are the tallest, most powerful and heavy group of horses. A B C D 2. Acting teacher Stella Adler played a vital role in the develop of the Method Schooling of Acting. A B C D 3. Before diamonds can be used as jewels, they must be cut and polish. A B C D 4. Of all mammals, dolphins are undoubtedly between the friendliest to humans. A B C D 5. Working all night long, Jim felt extreme hungry and just wanted to have a sandwich. A B C D 6. Harmony, melody and rhythm are important elements in mostly forms of music. A B C D 7. Medical students must learn both the theory and the practice of medicines. A B C D 8. Archeological sites sometimes are revealed when the construction of roads and buildings. A B C D 9. Yolanda ran over to me and breathless informed the failure of our team. A B C D 10. All root vegetables grow underground, and not all vegetables that grow underground are roots. BY ĐỖ BÌNH – THPT LIẾN SƠN, LẬP THẠCH, VĨNH PHÚC www.violet.vn/quocbinh72 C D D. READING I. Read the following passage and fill an appropriate word in each blank. (10 pts) Fish live almost (1) _ everywhere _ They are found in the near freezing waters of the Artic and in the steaming (2)_waters_ in the tropic jungles. They live in roaring streams and in quiet (3)_ underground __ rivers. Some fish make long journeys (4) _across_ the ocean. Others spend most of their life buried in sand on the ocean (5) _bottom__. Most fish can’t (6)_ leave __ water; yet some survive for months in dried-up riverbeds. Fish have enormous importance (7)_to_ man. They provide food for millions of people. Fishermen catch them for sports, and many people (8)_keep_ them as pets. Fish are also important in the balance of nature. They eat plants and animals and in turn, become (9)_ food __ for plants and animals. Fish thus help keep in (10 ) __balance__ the total number of plants and animals on the earth. II. Choose the most suitable answer from the four options marked A, B, C and D to complete each of the numbered gaps in the passage below. Write your answer on the answer sheet. (10 pts) You've (1)________ heard someone say that the reason you caught a cold was that you were 'run down'. People generally accept that if you are (2)______ pressure or run down you are more likely to get ill. But is this really true (3)_______ is it just an old wives' tale? The problem with old sayings like this is that it's often difficult to (4)______ whether they are true or not. For example, if you ask people with colds whether they are feeling run down, they are (5)_______ certain to say yes. People without colds are more likely to say no. So, how do you (6)______ if it's the cold that is making them (7)_____ run down or the fact that they are run down that is making them more likely to catch a cold? Now it seems there is a scientific answer to this question. And answer is yes - if you are run down or under stress, you are more likely to catch a cold. Scientists at the Common Cold Research Centre ran a series of tests. They got volunteers without colds to attend the centre, where they were first given a questionnaire to complete. The questions measured the (8)_____ of stress each volunteer was under. The volunteers were then deliberately infected with a cold virus and left to see whether they developed a cold. After many people had been tested, the researchers looked for a (9)____ between the measured stress level and the chances of catching a cold. They found that the higher the stress, the more likely the person was to catch a cold. At the moment it is not known why stress makes someone more likely to become ill, but now (10)_____ the link has been found, researchers might be able to investigate further. They may even find different ways of combating disease. 1. A. perhaps B. maybe C. possibly D. probably 2. A. under B. behind C. at D. above C. but D. so 3. A. and B. or 4. A. show B. express C. state D. prove 5. A. nearly B. mostly C. almost D. surely B. speak C. say D. suggest 6. A. tell 7. A. be B. feel C. become D. get B. number C. mass D. heap 8. A. amount 9. A. combination B. joint C. link D. relation 10. A. that B. when C. because D. as III. Read the passage below and choose the best answer from the four options marked A, B, C or D. Identify your answer by writing the corresponding letter A, B, C or D on the answer sheet. (20 pts) As many as one thousand years ago in the Southwest, the Hopi and Zuni Indians of North America were building with adobe-sun-baked brick plastered with mud. Their homes looked remarkably like modern apartment houses. Some were four stories high and contained quarters for perhaps a thousand people, along with storerooms for grain and other goods. These buildings were usually put up against cliffs, both to make construction easier and for defense against enemies. They were really villages in themselves, as later Spanish explorers must have realized since they called them “pueblos”, which is Spanish for towns. The people of the pueblos raised what are called “the three sisters”- corn, beans, and squash. They made excellent pottery and wove marvelous baskets, some so fine that they could hold water. The Southwest has always been a dry country, where water is scarce. The Hopi and Zuni brought water from streams to their fields and gardens through irrigation ditches. Water was so important that it played a major role in their religion. They developed elaborate ceremonies and religious rituals to bring rain. The way of life of less – settled groups was simpler and more strongly influenced by nature. Small tribes such as the Shoshone and Ute wandered the dry and mountainous lands between the Rocky Mountains and the Pacific Ocean. They gathered seeds and hunted small animals such as rabbits and snakes. In the Far North the ancestors of today’s Inuit hunted seals, walruses, and the great whales. They lived right on the frozen seas in shelters called igloos built of blocks of packed snow. When summer came, they fished for salmon and hunted the lordly caribou. The Cheyenne, Pawnee and Sioux tribes, known as the Plains Indians, lived on the grasslands between the Rocky Mountains and the Mississippi River. They hunted bison, commonly called the buffalo. Its meat was the chief food of these tribes, and its hide was used to make their clothing and the covering of their tents and tips. 1. What does the passage mainly discuss? A. The architecture of early America Indian buildings B. The movement of American Indians across North America A B BY ĐỖ BÌNH – THPT LIẾN SƠN, LẬP THẠCH, VĨNH PHÚC www.violet.vn/quocbinh72 C. Ceremonies and rituals of American Indians D. The way of life of American Indian tribes in early North America 2. According to the passage, the Hopi and Zuni typically built their homes ________. A. in valleys B. next to streams C. on open plains D. against cliffs 3. The word “They” refers to ____________. C. cliffs D. enemies A. goods B. buildings 4. It can be inferred from the passage that the dwellings of the Hopi and Zuni were _______. A. very small B. highly advanced C. difficult to defend D. quickly constructed 5. The author uses the phrase “the three sisters” refers to_____________. D. rain ceremonies A. Hopi women B. family members C. important crops 6. Which of the following is true of the Shoshone and Ute? A. They were not as settled as the Hopi and Zuni B. They hunted caribou. C. They built their homes with adobe. D. They did not have many religious ceremonies. 7. According to the passage, which of the following tribes lived in the grasslands? A. The Shoshone and Ute B. The Cheyenne C. The Hopi and Zuni D. The Pawnee and Inuit 8. Which of the following animals was the most important to the Plains Indians? A. The salmon B. The caribou C. The seal D. The buffalo 9. The author gives an explanation for all of the following words EXCEPT______. A. adobe B. pueblos C. caribou D. bison 10. The author groups North American Indians according to their _________. B. arts and crafts A. tribes and geographical regions C. rituals and ceremonies D. date of appearance on the continent E. WRITING Finish each of the following sentences so that it means the same as the printed one. (20 pts) 1. I didn’t arrive in time to see her. → I wasn’t early enough to see her. 2. We couldn’t drive because of the fog. → The fog prevented us from driving 3. A train leaves at eight o’ clock every morning.→ There is an eight o’ clock train every morning 4. John asked if it was the blue one or the green she wanted. → “Which one do you want, the blue or the green?” John said to her. 5. I haven’t eaten this kind of food before. → This is the first time I have eaten this kind of food 6. Only if you work hard now have you any chance of success. → Your chance of success depends on your working hard. 7. He tried very hard to give up smoking. → He made great effort to give up smoking. 8. An aerial is not required with this radio. → You don’t need to use an aerial with the radio. 9. When he stops smoking, he’ll feel better. → The sooner he stops smoking, the better he’ll feel 10. The wedding was held despite the rain. → The wedding took place despite /(in spite of) the rain __________ THE END ___________ BY ĐỖ BÌNH – THPT LIẾN SƠN, LẬP THẠCH, VĨNH PHÚC www.violet.vn/quocbinh72 SỞ GD&ĐT ........ --------------------ĐỀ CHÍNH THỨC (Đề thi gồm 4 trang) KỲ THI CHỌN HSG LỚP 10 THPT NĂM HỌC 2014-2015 ĐỀ THI MÔN: TIẾNG ANH Dành cho học sinh trường THPT chuyên Thời gian thi: 180 phút, không kể thời gian giao đề PART I. LISTENING Listen to a conversation between a foreign student and the accommodation secretary of a college, and complete the accommodation table. Write NO MORE THAN THREE words or numbers for each answer. You will hear the recording TWICE Types of room a single room (1) a _______________ Shared rooms (2)_________________ (3)_________________ (4)____________ facilities What is not provided (5)_________________ Kind of washing machines in the launderettes (6)_________ - operated How often the room are cleaned (7)_________________ Place for students to watch TV (8)_________________ Kind of accommodation that provides breakfast and dinner (9)_________________ Kind of accommodation that the student signs up for (10)________________ PART II. GRAMMAR AND VOCABULARY I. Complete the following sentences by choosing the correct answer among four options (A, B, C or D). 1. I'm addicted ______ these sweets. I loved them! A. on B. at C. to D. for 2. You are being thoroughly _________in refusing to allow this ceremony to take place. A. unrequited B. unrepresentative C. unreliable D. unreasonable 3. Nylon, a synthetic _______ from a combination of water, air, and a by-product of coal, was first introduced in 1938. A. to make B. make C. made D. making 4. He ______ book the tickets, but he had no time to call at the Cinema. A. would B. might C. could D. was going to 5. It is necessary that an employee _________ his work on time. A. finish B. finished C. finishing D. finishes 6. This dress is eye-catching, and many people________ me on it. A. complain B. complement C. compliment D. complementary 7. Bill is jealous ________ your promotion. A. with B. of C. at D. about 8. _______ are sought by an elementary school. A. Mentally retarded teachers B. Teachers retarded for mental cases C. Teachers for the mentally retarded D. Mental retarded case teachers 9. Since I came in half an hour late this morning, I have to stay until 5:30 to _______ for it. A. make up B. save up C. keep up D. hold up 10. Earthquake can damage a tree ______violently, and it can take several years for the tree to heal. A. to cause to shake it B. when shaking it causes BY ĐỖ BÌNH – THPT LIẾN SƠN, LẬP THẠCH, VĨNH PHÚC www.violet.vn/quocbinh72 C. to cause shaking it D. by causing it to shake II. In the following text, some of the lines are correct, and some have a word which should not be there. If a line is correct, put a tick (√). If a line has a word which should not be there, write the word. There are two examples at the beginning. We can tend to think of poverty as the main cause of violent crime. However, research from both sides of the Atlantic shows that fathers may be the most important factor in preventing to children from turning over to crime. In the United States, children from better-off family were compared with ones who from families with lower incomes. Children from both groups that lived with their fathers also committed the same number of crimes. In the United Kingdom, a study was carried out of comparing a group of boys who had never been accused of not any crimes such as assault and stealing vehicles. All the boys had difficulties at the school and came from large families that didn't earn a lot of money. The biggest difference between from the groups was that fifty-five percent of the "good boys" lived with their fathers, while only four per cent of the "bad boys" did it. Eighty per cent of the well – behaved boys said how they felt close to their fathers even if they didn't live in the same house. It seems that by having a father who takes an interest in his children encourages youngsters not to break the law. 0____ can ____ 00___√_______ 1. ___________ 2. ___________ 3. ___________ 4. ___________ 5. ___________ 6. ___________ 7. ___________ 8. ___________ 9. ___________ 10. __________ 11. __________ 12. __________ 13. __________ 14. __________ 15. __________ III. Complete the following passage by supplying the correct form of the word to fill in each blank. There is a saying that first impressions are generally correct and I would say that (1) _______ throughout my entire life, I have found this to be true apart from one notable exception of a good friend and neighbor with whom my first (2) ______ was decidedly unfriendly. At the time of our first meeting I was living in a (3) __________ area of London, not far from the exit of a dual carriageway and this meant that although it was a (4) ____________ street, even with the (5) _______ will in the world, drivers would often travel dangerously fast within inches of my front door. This used to really annoy me and I have to admit I did often go a bit (6) ________ in expressing my anger even though (or perhaps because) I knew I didn't stand a (7) ________ chance of being taken notice of or even noticed at all. So when one day, I shouted my usual string of swear words at a rapidly passing car and it immediately screamed to a halt, my first reaction was an enormous sense of (8) ______. But then the driver of the car opened his window and shouted a stream of swear words back at me. So the next morning, I was more than a little surprised to find an apology note from the same motorist in my mail box explaining that in an attempt to (9) _______ his career as an up and coming IT (10) ______, he had been driving too fast the previous evening and inviting me to have dinner with him and his wife. ALL CHANGE DEPRIVE RESIDENCE GOOD BOARD REASON ACHIEVE FAR CONSULT PART III. READING BY ĐỖ BÌNH – THPT LIẾN SƠN, LẬP THẠCH, VĨNH PHÚC www.violet.vn/quocbinh72 I. Read the passage carefully, then choose the correct option (marked A, B, C or D) to answer the questions. The food we eat seems to have profound effects on our health. Although science have made enormous steps in making food more fit to eat, it has, at the same time, made any foods unfit to eat. Some research has shown that perhaps 80 percent of all human illness are related to diet and 40 percent of cancer is related to diet as well, especially cancer of colon. People of different cultures are more prone to contact certain illnesses because of the characteristics food they consume. That food is related to illness is not a new discovery. In 1945, government researchers showed that nitrates and nitrites (commonly used to preserve color in meats) as well as other food additives caused cancer. Yet, these carcinogenic additives remain in our food, and it becomes more difficult all the time to know which ingredients on the packaging labels of processed food are helpful or harmful. The additives we eat are not all so direct. Farmers often give penicillin to cattle and poultry, and because of this, penicillin has been found in the milk of treated cows. Sometimes similar drugs are administered to animals not for medical purposes, but for financial reasons. The farmers are simply trying to fatten the animals in order to obtain a higher price on the market. Although the Food and Drug Administration (FDA) has tried repeatedly to control these procedures, the practices continue. A healthy diet is directly related to good health. Often we are unaware of detrimental substances we ingest. Sometimes, well – meaning farmers or others who do not realize the consequences add these substances to food without our knowledge. 1. How has science done a disservice to people? A. Because of science, disease caused by contaminated food has been virtually eradicated. B. It has caused a lack of information concerning the value of food. C. As a result of scientific intervention, some potentially harmful substances have been added to our food. D. The scientists have preserved the color of meats, but not of vegetables. 2. The word "prone" is nearest in meaning to ______. A. supine B. unlikely C. healthy D. predisposed 3. What are nitrates used for? A. They preserve flavor in packaged foods. B. They preserve the color of meats. C. They are the objects of research. D. They caused to animals to become fatter. 4. The word "these" refers to_______. A. meats B. colors C. researchers D. nitrates and nitrites 5. All of the following statements are true EXCEPT_______. A. drugs are always given to animals for medical reasons. B. some of the additives in our food are added to the food itself and some are given to the living animals. C. researchers have known about the potential hazards of food additives for more than 60 years. D. food may cause 40 percent of cancer in the world II. Read the passage and fill in the blank a suitable word. PEER PRESSURE One of the strongest influences on children today (1) ______ that of their peers. What their classmates think, how they dress and how they act in class and out of it (2) ________ the behavior of nearly every child at school. In their efforts not to be different, some children go so far as to hide their intelligence and ability in case they are made ( 3)_________ of. Generally, children do not want to stand (4) _______ from the crowd. They want to fit in, to be accepted. In psychological terms, the importance of (5)_________ pressure cannot be over emphasized. There is a lot of evidence that it has great bearing on all aspects of children's lives, (6) ________ the clothes they wear, the music they listen to and their attitude to study to their ambitions in life, their relationships and their sense. (7) _________, as children grow up into adolescents, individuality becomes more acceptable, desirable even, and in their search for their (8) _______ personal style, the teenager and young adult will begin to experiment and be more willing to run the (9) _______ of rejection by the group. Concern about intellectual prowess and achieving good exam results can dominate as the atmosphere of competition develops and worries (10) ______ the future override any fears of appearing too brainy. BY ĐỖ BÌNH – THPT LIẾN SƠN, LẬP THẠCH, VĨNH PHÚC www.violet.vn/quocbinh72 III. Complete the following passage by choosing A, B, C or D to fill in each blank. Last year Gladys Kalema became the Ugandan Wildlife Service's chief and only vet after (1) _____ for the Royal Veterinary College in London. She was the first person to fill the post for 30 years and at the age of 26, easily the youngest. If Gladys did nothing else, caring for the world's (2) ______ population of 650 gorillas would alone justify her wages. Since the 1970s, gorillas have (3) ______ severely from war and poaching. Now for $150 each, tourists can be led through the forest and come within five meters of gorilla – no closer, for (4) ______ of transmitting diseases such as measles and flu. The gorillas here make a small but viable population. (5) ______ in the national parts the usual animals, elephants, rhinos, giraffes, are either not there or present in insignificant numbers which are dangerously out of (6) ______ with the creatures around them. If Uganda stays calm, wildlife may, in (7) _______, return by itself. But Gladys believes the country cannot wait. Animals must be brought in to swell tourism and provide (8) ______to expand her work. Despite her difficulties, Gladys feels more useful and fulfilled than she would be anywhere else. "At this moment, my friends from vet school are reading the best way to (9) ______ a cat or dog, and here am I planning to translocate elephants. In my small (10) _______ I am part of the reconstruction and rehabilitation of my country." 1. A. leaving B. qualifying C. graduating D. passing 2. A. living B. surviving C. continuing D. lasting 3. A. endured B. harmed C. died D. suffered 4. A. risk B. fear C. fright D. danger 5. A. Somewhere B. Anywhere C. Elsewhere D. Nowhere 6. A. balance B. relation C. comparison D. equality 7. A. terms B. years C. ages D. time 8. A. figures B. funds C. accounts D. savings 9. A. treat B. prescribe C. heal D. operate 10. A. means B. manner C. method D. way PART IV. WRITING I. Write the new sentences using the word give. Do not change the word given in any way. 1. She felt she had achieved a lot in life, despite her disadvantaged background (SENSE) 2. I think you should try to be optimistic as you can. (SIDE) 3. In the end, it was quite a good sunny day after all. (OUT) 4. I hate watching late night films on TV. (STAND) 5. The principal was the first person who arrived at the meeting. (ARRIVE) II. Writing a topic. Computers can translate all kinds of languages well, so our children needn’t learn more languages in the future. What is your opinion about this idea? Within 200-250 words, use evidence and examples to write a passage to clarify your idea. -------------The end--------------- BY ĐỖ BÌNH – THPT LIẾN SƠN, LẬP THẠCH, VĨNH PHÚC www.violet.vn/quocbinh72 HƯỚNG DẪN CHẤM MÔN TIẾNG ANH LỚP 10 THPT CHUYÊN NĂM HỌC: 20110 - 2012 PART I. LISTENING (15pts: 1,5pt/item) 1. a twin study bedroom 2. a bathroom 3. a toilet 4. shower facilities 5. towels 6. coin 7. once a week 8. a common room 9. Half board 10. With a family/ homestay PART II. GRAMMAR AND VOCABULARY I. (10 pts: 1pt/item) 1. C 2. D 3. C 4. D 6. C 7. B 8. C 9. A II. (15 pts: 1/item) 1. to 2. over 3. who 4. √ 6. of 7. not 8. the 9. √ 12. it 13. how 14. by 11. √ III. (10pts: 1pt/item) 1. overall 6. overboard 2. interchange/ exchange 7. reasonable 3. deprived 8. achievement 4. residential 9. further 5. best 10. consultant 5. A 10. D 5. also 10. from 15. √ PART III. READING I. (5 pts: 1/item) 1. C 2. D II. (10pts: 1pt/item) 1. is 2. affect 6. from 7. However III. (10 pts: 1pt/item) 1. C 2. B 6. A 7. D 3. B 4. D 5. A 3. fun 8. own 4. out 9. risk 5. peer 10. about 3. D 8. C 4. B 9. A 5. C 10. D PART IV. WRITING I. (5pts: 1pt/item) 1. Despite her disadvantaged background, she felt a sense of achievement about her life 2. I think you should look on the bright side. 3. The day turned out to be quite sunny after all/ It turned out to be quite a good sunny day after all. 4. I can't stand watching late night films on TV. 5. The principal was the first person to arrive at the meeting II. 20 pts - The student’s paper has logical structure, with opening, development and closing parts, using right linking words: 5 pts - The paper has less than 5 mistakes (both grammar and spelling mistakes): 5 pts - The paper has interesting, creative content: 10 pts BY ĐỖ BÌNH – THPT LIẾN SƠN, LẬP THẠCH, VĨNH PHÚC www.violet.vn/quocbinh72 SỞ GD&ĐT ......... ----------------ĐỀ ĐỀ XUẤT (Đề thi gồm 06 trang) KỲ THI CHỌN HSG LỚP 10 THPT NĂM HỌC 2013-2014 ĐỀ THI MÔN: TIẾNG ANH (Dành cho học sinh THP không chuyên) Thời gian thi: 180 phút, không kể thời gian giao đề PART I: LISTENING SECTION 1. Questions 1 – 5 Choose the correct letter, A, B, C or D 1. What does her bag look like? 2. 3. A C 4. A C 5. What time was it when the bag was stolen? Where was she when the bag was stolen? at the shopping centre by the supermarket What was in the bag? wallet, passport and camera passport, traveler’s cheques The woman is ____________ B at the cinema D at the movies B passport, traveler’s cheques and pens D traveler’s cheques, wallet and pens BY ĐỖ BÌNH – THPT LIẾN SƠN, LẬP THẠCH, VĨNH PHÚC www.violet.vn/quocbinh72 A at a conference B on holiday C working D visiting family Section 2. Complete the form below. Write ONE WORD AND/ OR A NUMBER for each answer. CAR INSURANCE Example: Answer: Name: Patrick Jones Address: 27 ____________ (1) Road, Greendale Contact number: 730453 Occupation: _________________ (2) Size of car engine: 1200cc Type of car: Manufacturer: Hewton Model: ____________________ (3) Year: 1997 Previous insurance company: ________________ (4) Star Any insurance in the last five years? Yes If yes, give brief details: Car was _____________ (5) in 1999 Name(s) of other driver(s): Simon ________________ (6) Relationship to main driver: Brother-in-law Uses of car: No - social - Traveling to ________________ (7) Start date: 31 __________________(8) Recommended insurance arrangement Name of company: Red __________________ (9) Annual cost: $ ___________________ (10) PART II. PHONETICS I. Pick out the word whose underlined part is pronounced differently from those of the other words. Write your answers in the numbered blanks provided below 1. A. random B. canal C. many D. anaesthetic 2. A. provide B. product C. production D. procedure 3. A. death B. ease C. breath D. leather 4. A. ambitious B. background C. humanitarian D. journalist 5. A. turn B. suburb C. curry D. resurface II. Choose the word whose stress pattern is differently from those of the other words. Write your answers in the numbered blanks provided below 1. A. powerful B. composer C. musician D. traditional 2. A. lyrical B. emotion C. favorite D. classical 3. A. entertain B. atmosphere C. serious D. joyfulness 4. A. understand B. appearance C. remember D. employer 5. A. nation B. nature C. nationality D. national BY ĐỖ BÌNH – THPT LIẾN SƠN, LẬP THẠCH, VĨNH PHÚC www.violet.vn/quocbinh72 PART III. LEXICO-GRAMMAR I. Read the sentences carefully and choose the best answers to make complete ones. 1. Those interested in the post of Assistant Manager are requested to apply _______. A. as a person B. in person C. as persons D. by person 2. I don’t care _______ you go or not. A. where B. when C. why D. whether 3. If it wasn’t you who took my pen, it must have been _______. A. somebody B. somebody else C. another one D. anybody 4. Did the man _______ by the committee accept the job? A. appointing B. appointed C. was appointed D. to appoint 5. Don’t leave your clothes _______ about on the bedroom floor. A. lie B. lay C. lying D. laying 6. She raced by in a car, with her hair _______ behind. A. streamed B. stream C. streams D. streaming 7. I had my younger brother _______ the newspaper to me while I was abroad. A. to send B. sent C. send D. sending 8. Joe, I am inclined _______ Appiah’s complicity in the big fraud. A. about believing B. to believe C. for believing D. in believing 9. Now listen to me John. As far as your examination results are concerned, you needn’t worry so much, _______? A. isn’t it B. need you C. do you D. aren’t they 10. His arrogance makes him _______ whenever he goes. A. hate B. hated C. to hate D. hating II. The passage below contains 10 errors. IDENTIFY and CORRECT them. (0) has been done as an example. Write your answers in the numbered blanks below After an absence in thirty years, I decided visiting my old school again. I had expected to find changes, but no a completely different building. As I walked up the school drive, I wondered for a moment if I had come to right address. The grimy, red brick fortress with their tall windows that had looked up grimly on the playground and playing fields had swept away. In its place stood a bright, modern block risen from the ground on great concrete stilts. A huge expanse of glass extending across the face of the building, and in front, there was a well-kept lawn where previous there had been untidy gravel yard. Answers: 0. in of III. Fill in each blank with a suitable PREPOSITION or PARTICLE. Write your answers in the numbered blanks provided below the passage. 1. Without a fridge, fresh food will go _________ very quickly. 2. I haven’t heard_________ Mandy since she wrote in July. 3. This service is free ___________ charge. 4. They went ahead contrary ____________ my advice. 5. The monk told me that the old woman had not really dies. He said that she simply passed _______ her next life 6. BBC stands _______ British Broadcasting Corporation 7. When I was looking through my papers, I came ______ this picture 8. The taxi driver decided to go ___________ duty as he was beginning to feel sleepy. BY ĐỖ BÌNH – THPT LIẾN SƠN, LẬP THẠCH, VĨNH PHÚC www.violet.vn/quocbinh72 9. The band was __________ the verge of stardom. 10. Jim won’t be coming tonight because he’s feeling a bit ________________ the weather. IV. Write the correct FORM of each bracketed word. Write your answers in the numbered spaces provided below 1. The situation is so ___________ (CHAOS) in some countries now that it is difficult to see any solution. 2. Good friends can enrich the quality of our lives _____________ (MEASURE). 3. Computers are now considered ________________ (DISPENSE) in the business world. 4. Due to ______________ (ELECTRIC) the difference between urban life and rural life is more and more reduced. 5. I will resign if you continue (REGARD) ________________ what I say 6. She was late as she (ESTIMATE)_______________ how much time she’d need 7. Kenneth gave her child a (HAND)___________ of sweets 8. Politeness is one thing. Real kindness is another. You must learn to (DIFFERENT)_______________ the two 9. He’s the most unpleasant, _______________ (MANNER) person I’ve ever met. 10. I can’t stand out little boy. He’s really ______ (ANNOY). PART IV. READING I. Read the following text carefully and choose the answer which best fits each space Stress-related hair loss When stress levels become so high that they affect the immune system it can have a profound effect on the body and the hair. Hair loss itself causes immense stress and then it can become a vicious (1)_______. Bonnie Kinnear, 50s, housewife: “Last year my life was quite stressful due to my husband’s high-profile career being constantly in the (2) ________.At this time I began to develop an (3) ________ scalp which was wrongly diagnosed as psoriasis, for which I was prescribed steroids. Over the next two months my scalp become worse, and hair began to fall out, leaving (4) _______ patches, which was devastating. Having to look good and be a support to my famous husband was stressful because wherever we went people would look at us. I had lost at (5) ________ one third of my hair and, in (6) ________, visited a high-profile Mayfair trichologist. Without any diagnosis I was ordered to buy £300 worth of products and my head was massaged by a girl who proceeded to turn my hair into what is (7) _______ as ‘bird’s-nest syndrome’. It took four hours to (8) ________. My GP convinced me to seek help from Andrew Bernie. I was diagnosed with a stress-triggered (9) __________ where the skin cells multiply fast, grow down the hair shaft and bind with sebaceous matter to strangle the hair. I visit Andrew regularly for treatments. I am also trying to (10) ________ my stress levels, which is easier now my hair is returning.’ 1. A 2. A 3. A 4. A 5. A 6. A 7. A 8. A cycle daylight intense empty once reply supposed replace B B B B B B B B effect spotlight enlarged bald all fun known untangle C C C C C C C C closure searchlight inflamed dark least hope seen return D D D D D D D D circle starlight inflicted hairy first desperation referred reset BY ĐỖ BÌNH – THPT LIẾN SƠN, LẬP THẠCH, VĨNH PHÚC www.violet.vn/quocbinh72 9. A 10.A state destroy B B symptom assess C C outcome manage D D condition measure II. Fill in each blank space with an appropriate word. A Success Story At 19, Ben Way is already a millionaire and one of a growing number of teenagers _________ (1) have made their fortune through the Internet. __________ (2) makes Ben's story all the more remarkable is that he is dyslexic, and was told by teachers ___________ (3) his junior school that he would never be able to read or write properly. "I wanted to prove them wrong", says Ben, creator and director of Way search, a net search engine which can be used to find goods __________ (4) online shopping malls. When he was eight, his local authorities provided him ___________ (5) a PC to help with schoolwork. Although he was unable to read the manuals, he had a natural ___________ (6) with the computer, and encouraged by his father, he soon began charging people E10 an hour for his knowledge and skills. At the age of 15 he set up his own computer consultancy, Quad Computer, which he ran from his bedroom, two years later he left school __________ (7) devote all his time to business. "By this time the company had grown and I needed to take __________ (8) a couple of employees to help me," says Ben. “That enabled me to start doing business with bigger companies.” It was his ability to consistently ___________ (9) difficult challenges that led him to win the You Entrepreneur of the Year award in the same year that he formed Way search, and he has recently signed a deal worth £25 million with a private investment company, which will finance _________ (10) search engine. III. Read the following passage and choose the best answer. Write your answers in the numbered blanks provided below the passage. As Christmas evolved in the United States, new customs were adopted and many old ones were reworked. The legend of Santa Claus, for example, had origins in Europe and was brought by Dutch settlers to New York in the early 18th century. Traditionally, Santa Claus - from the Dutch Sinter Klaas was depicted as a tall, dignified, religious figure riding a white horse through the air. Known as Saint Nicholas in Germany, he was usually accompanied by Black Peter, an elf who punished disobedient children. In North America he eventually developed into a fat, jolly old gentleman who had neither the religious attributes of Saint Nicholas nor the strict disciplinarian character of Black Peter. Santa’s transformation began in 1823, when a New York newspaper published the poem A Visit from Saint Nicholas, which Clement Clark Moore had written to amuse his daughter. The poem introduced many Americans to the story of a kindly saint who flew over housetops in a reindeer-drawn sleigh. Portraits and drawings of Santa Claus by American illustrator Thomas Nast further strengthened the legend during the second half of the 19th century. Living at the North Pole and assisted by elves, the modern Santa produced and delivered toys to all good children. By the late 19th century he had become such a prominent figure of American folklore that in 1897, when Virginia O’Hanlon wrote to the New York Sun newspaper asking if Santa was real, she received a direct answer: “Yes, Virginia, there is a Santa Claus”. 1. Who brought the legend of Santa Claus to the USA according to the passage? A. Sinter Klaas B. Saint Nicholas C. A German D. Dutch settlers 2. Santa Claus was traditionally described as a __________. A. tall man who could walk through the air B. fat, jolly, old man C. religious figure D. fat man riding a white horse 3. Santa Claus in North America was depicted as ___________. A. a man with the strict disciplinarian character of Black Peter B. a good old man with less religious character C. one with religious attributes of Saint Nicholas. BY ĐỖ BÌNH – THPT LIẾN SƠN, LẬP THẠCH, VĨNH PHÚC www.violet.vn/quocbinh72 D. a jolly man on horseback. 4. Who was Black Peter? A. an elf accompanying Saint Nicholas C. one of the disobedient children B. an elf who rode a white horse D. a popular traditional figure 5. What word is closest in meaning to “attributes”? A. symbols of a person B. natural qualities C. effects D. outer appearance 6. Where did the legend of Santa Claus come from? A. the North Pole B. Europe C. North America D. the City of New York 7. 1823 was mentioned as a year when ____________. A. Clement Clark Moore wrote his first poem. B. Clement Clark Moore’s poem made him popular. C. Saint Nicholas visited New York. D. the image of Santa Claus was transformed 8. According to Clement Clark Moore’s poem, A. Santa Claus had nothing different in appearance from the traditional one. B. Santa Claus had wings and could fly. C. Santa Claus liked poetry. D. Santa Claus was a kindly saint who flew over housetops in a sleigh. 9. The answer “Yes, Virginia, there is a Santa Claus” is an illustration for the fact that________ A. the New York Sun was popular with children. B. Santa Claus was a prominent figure at that time. C. newspapers are unreliable. D. Virginia O’Hanlon was a reader of the New York Sun. 10. Which of the following statements is TRUE? A. Santa Claus was an imaginary old man created by artists based on traditional figures. B. Living in the North Pole, Santa Claus visited children at Christmas. C. Santa Claus was a real figure living in northern America. D. Santa Claus was a story based on Saint Nicholas and Black Peter. PART V. WRITING I. Finish each of the following sentences in such a way that it means the same as the sentence printed before it 1. Someone stole the old lady’s handbag. The old lady was ___________________________. 2. A true story forms the basis of Mary’s new novel. Mary’s new novel _______________________________. 3.If you have completed your test, you can go home. Get ___________________________. 4. The employee insisted on being given a full bonus. Nothing but __________________________________ 5. ‘I didn’t harm anybody!’ cried the accused. The accused _____________________________. BY ĐỖ BÌNH – THPT LIẾN SƠN, LẬP THẠCH, VĨNH PHÚC www.violet.vn/quocbinh72 II. There is a fact that a number of parents now increasingly desire to send their children to study overseas. What are the reasons for this trend? You should write a paragraph of about 150 words on this topic. SỞ GD&ĐT VĨNH PHÚC ----------------ĐÁP ÁN (Đề thi gồm 5 trang) KỲ THI CHỌN HSG LỚP 10 THPT NĂM HỌC 2013-2014 ĐỀ THI MÔN: TIẾNG ANH (Dành cho học sinh THP không chuyên) Thời gian thi: 180 phút, không kể thời gian giao đề Total: 130 pts PART I: LISTENING Section 1. (10pts) 1. D 2. A 3. C 4. B 5.B 1. Bank 2. Dentist 3. Sable 4. Northern 5. stolen 6. Paynter 7. work 8. January 9. Flag 10. 450 Section 2 (10pts) PART II. PHONETICS I. Pick out the word whose underlined part is pronounced differently from those of the other words. Write your answers in the numbered blanks provided below (5pts) 1. C 2. B 3. B 4. D 5. C II. Choose the word whose stress pattern is differently from those of the other words. Write your answers in the numbered blanks provided below (5pts) 1.A 2.B 3.A 4.A 5.C PART III. LEXICO-GRAMMAR I. Read the sentences carefully and choose the best answers to make complete ones. (10pts) 1. B 6.D 2.D 7.C 3.B 8.B 4.B 9.B 5.C 10.B II. The passage below contains 10 errors. IDENTIFY and CORRECT them. (0) has been done as an example. Write your answers in the numbered blanks below (10pts) 1. visiting -->to visit 2. no --> not 3. right address -->the right address BY ĐỖ BÌNH – THPT LIẾN SƠN, LẬP THẠCH, VĨNH PHÚC www.violet.vn/quocbinh72 4. their --> its 7. risen --> raised 9. previous --> previously 5. up --> down 6. had swept --> had been swept 8.extending --> extended 10. untidy gravel yard --> an untidy gravel yard III. Fill in each blank with a suitable PREPOSITION or PARTICLE. Write your answers in the numbered blanks provided below the passage.(10pts) 1. off 6. for 2. from 7. across 3. of 8. off 4. to 9. on 5. into 10. under IV. Write the correct FORM of each bracketed word. Write your answers in the numbered spaces provided below (10pts) 1. chaotic 6.underestimated 2. immeasurably 7. handful 3. indispensable 8. differentiate 4. electrification 9. ill/badmannered 5. disregarding 10. annoying PART IV. READING I. Read the following text carefully and choose the answer which best fits each space (10pts) 1.B 2.B 3.A 4.D 5.B 6.C 7.C 8.D 9.D 10.A II. Fill in each blank space with an appropriate word. (10pts) 1. who 6. ability 2. what 7. to 3. at 8.on 4. in 9. overcome 5.with 10. his III. Read the following passage and choose the best answer. Write your answers in the numbered blanks provided below the passage. (10pts) 1.D 6.B 2.C 7.D 3.B 8.D 4.A 9.B 5.A 10.A PART V. WRITING I. Finish each of the following sentences in such a way that it means the same as the sentence printed before it (10pts) 1. was robbed of her hand bag. 2. is based on a true story. 3. your test completed/ finished and you can go home. 1. a full bonus would satisfy the employee. 4. denied causing/having caused harm to anybody. II. There is a fact that a number of parents now increasingly desire to send their children to study overseas. What are the reasons for this trend? You should write a paragraph of about 150 words on this topic. (20pt) Marking scheme The impression mark given is based on the following scheme: 1. Content: 50% of total mark: a provision of all main ideas and details as appropriate BY ĐỖ BÌNH – THPT LIẾN SƠN, LẬP THẠCH, VĨNH PHÚC www.violet.vn/quocbinh72 2. Language: 30% of total mark: a variety of vocabulary and structures appropriate to the level of English language gifted upper-secondary school students 3. Presentation: 20% of total mark: coherence, cohesion, and style appropriate to the level of English language gifted upper-secondary school students. ------The end ------ BY ĐỖ BÌNH – THPT LIẾN SƠN, LẬP THẠCH, VĨNH PHÚC www.violet.vn/quocbinh72 SỞ GD&ĐT VĨNH PHÚC ĐỀ CHÍNH THỨC (Đề thi gồm 05 trang) KỲ THI CHỌN HSG LỚP 10 THPT NĂM HỌC 2013-2014 ĐỀ THI MÔN: TIẾNG ANH (Dành cho học sinh THPT chuyên) Thời gian thi: 180 phút, không kể thời gian giao đề PART A. LISTENING I. You’ll hear two students, Ann and Chris talking about post graduate courses in music. Listen and give short answer to the questions from 1 to 10. You will listen to the conversation TWICE. 1. Which college does Chris suggest would be best? 2. What entry requirements are common to all colleges? 3. How much does the course at Leeds Conservatory of Contemporary Music cost? 4. What other expenses are payable to the colleges? 5. When is the deadline of The Academy in London? Which facilities do the colleges have? Choose five answers from the box and write the correct letter, A – G, next to questions 6-10. Colleges 6. Northdown College Facilities 7. The Academy in London A. large garden 8. Leeds Conservatory and Contemporary Music B. multiple sites 9. The Henry Music Institute C. practice rooms 10. The James Academy of Music D. recording studio E. research facility F. student carteen G. technology suite II. You will hear a conversation about technology. Write NO MORE THAN THREE WORDS AND/OR A NUMBER for each answer. You will hear the conversation TWICE. The Antarctic Polar View project maps Antarctic sea ice by using (1) _________. Problems to navigate through the water: - the safety of the ship - the speed of the ship - the (2)________ of the ship NVSAT Satellite: - Collect data - Identify difference between open water and (3) ________. Scientists can see surface of sea clearly by using (4) ________. Helicopter: - Advantage: - map the ice in the air - look for (5) ________. - Disadvantages: - much more difficulty - (6) __________. The colour of the map is (7) ____________. Problems of sending pictures in Antarctic ship: (8) __________. Measure to the problem: (9) ________ images into JPEG 2000 format. The equipment scientists need for mapping is a (10) _________ on ship. BY ĐỖ BÌNH – THPT LIẾN SƠN, LẬP THẠCH, VĨNH PHÚC www.violet.vn/quocbinh72 PART B. LEXICO - GRAMMAR I. Complete each of the following sentences with the correct answer (A, B, C or D). Identify your answer by writing the corresponding letter A, B, C or D on your answer sheet. 1. The audience couldn’t listen to the lengthy and pointless speech _________. A. far any more B. any longer C. any more far D. any lengthier 2. He missed two most important lectures. He _________very ill. A. had to be B. must be C. was to be D. must have been 3. Allan: “Do you mind if I use your dictionary?” Nick: “_______.” A. I’m afraid not B. Without doubt C. No, feel free D. Straight ahead 4. Oh, I’m always forgetting _______ these medicines. Is that before or after meal, Ron? A. when to take B. what I will take with C. on which I should take D. when I take 5. We rang the doorbell again _______ they hadn’t heard it the first time. A. because B. although C. for fear of D. in case 6. How boring the lecture was! I _______ just in the first half of it. A. dropped off B. fell out C. called off D. came out 7. Will it make any _______ to them if we deliver their equipment tomorrow? A. displeasure B. alteration C. difference D. conflict 8. What time are you _______ duty? Let’s have a coffee after that. A. over B. on C. off D. out of 9. Don’t give up your harboring dream. _______ working harder and harder. A. Try out B. Keep on C. Speed up D. Go off 10. It’s been a good year. I’ve written two books and a couple of articles, and _______ are now in print. A. both of which B. both of them C. all of which D. all of them 11. A: Why didn't you go to work yesterday? B: I was feeling a bit ____________. A. under the weather B. full of hot air C. breeze D. under pressure 12. The closets are filled with things we don’t need. We should have a _______soon. A. garage sale B. rush hour C. housekeeper D. doggy bag 13. If you want to return the suit, you must bring the __________with you as proof of purchase. A. note B. recipe C. prescription D. receipt 14. Nancy and I had both bought exactly the same kind of school bag without knowing it. It was a complete ____________. A. luck B. chance C. fortune D. coincidence 15. I think some modern designs are more like ________ of art. A. works B. makes C. objects D. jobs 16. Time will _________ whether I made a right choice or not. A. see B. say C. know D. tell 17. That kind of dress is the _________ of fashion these days. A. height B. top C. head D. lead 18. You are having problems now but I am sure things will change _________ the better soon. A. on B. to C. by D. for 19. Despite being a very good student, she didn’t fulfill her ________ later in life. A. makings B. potential C. capacity D. aptitude 20. If you _______ any problems when you arrive at the airport, give me a ring. A. come about B. catch on C. run into D. face up II. There are 10 mistakes in the following passage. Find and then correct them. (0) have been done for you. BY ĐỖ BÌNH – THPT LIẾN SƠN, LẬP THẠCH, VĨNH PHÚC www.violet.vn/quocbinh72 Anthony Masters was a writer in exceptional gifts and prodigious energy. He began his eventful and versatile career like a teenager, when he was expelled from school for organizing a revolt against the school uniform. In order to earn a living, he fulfilled his childhood ambition and took on writing. In 1964, at the age of 23, he published A Pocketful of Rye, a collection of short stories where freshness of style earned him a distinction of being runner – up in the John Llewellyn Rhys Memorial Prize, an established and prestigious British – based literacy award. He made the award two years later with his novel The Seahorse, after which he continued to display his considerable talent by writing both fiction or non – fiction. The inspiration for many of his novels came from his experience helping the social excluded: he ran soup kitchens for drug addicts and campaigned for the civic rights of gypsies and another ethnic minorities. His non – fiction output was typically eclectic, ranged from biographies to social histories, but it was a writer of children fiction that Masters outshone his contemporaries. His work contains a sensitivity which remains unequalled by some other writer of the genre. Answers: 0. in of III. Supply the correct form of the word in capital letter. Write your answers on your answer sheet. With the development of fast motorway systems in many countries, once remote parts of the countryside are (1. INCREASE) _________ becoming (2. ACCESS) _________ to people who live in towns and cities. In many parts of the world, rising (3. PERSON) _________ incomes have resulted in mass (4. OWN) _________ of motor cars at a time when people have more leisure time to fill. This means that there is a lot of (5. PRESS) _________ on the countryside to accept a growing number of visitors. In Britain, for example, the (6. NATION) _________ parks are used by more than one hundred million people annually. In (7. ADD) _________, similar numbers visit areas that do not enjoy the same level of (8. PROTECT) _________. Some people regard this invasion of the countryside as (9. DESIRE) _________ and they have begun campaigning to try and save some aspects of (10. TRADITION) _________ rural culture. IV. Complete the following sentences with one preposition/particle for each blank. 1. That factory turns ________ hundreds of small appliances everyday. 2. The boss asked me to make _______ the hours I missed last week. 3. The suspect was released from prison _______ bail. 4. Once again poor Colin has been passed _______for promotion. 5. Everyone approved of the scheme, but when we asked for volunteers they all hung _______. PART C. READING I. Read the following passage and choose the best answer for each question. Identify your answer by writing the corresponding letter A, B, C or D on your answer sheet. Have you ever thought about the names of the months? Why are “January” and “February” not called “Primo” or “Secondo”? Is it because the original names were created in ancient times? Or is it because the originators preferred odd words? Take February, for example. Say it aloud a few minutes and you start to wonder. Most people don’t know who developed these names. However, a little research reveals that the names of the months came mostly from a combination of the names of Roman gods and goddesses, important festivals, and the original numbers of the months. Julius Caesar and Pope Gregory XIII change the calendar to make it more exact. Caesar developed a new calendar of 364 and a quarter days, the time it takes the earth to orbit the sun from one spring season to the next. The Pope’s astronomers refined the calendar regarding leap years; they determined that there should be no leap year in years ending in 00- unless they were divisible by 400; the years 1700, 1800, 1900 and 2100 would not be considered leap years, while the years 1600 and 2000 BY ĐỖ BÌNH – THPT LIẾN SƠN, LẬP THẠCH, VĨNH PHÚC www.violet.vn/quocbinh72 would be. This new Gregorian calendar was so accurate that today, scientists need only add leap seconds every few years to the clock in order to keep the calendar matching the Earth’s cycles. 1. What is the topic of the passage? A. how the modern calendar was named and developed B. how the months were named C. how the leap year system was developed D. how accurate the modern day is calendar 2. It can be inferred from paragraph 1 that the author think the names of the months are __________. A. odd B. difficult to pronounce C. inappropriate D. none of the answers 3. The word “they” in paragraph 3 refers to ______________. A. calendars B. days C. astronomers D. years 4. The word “accurate” in paragraph 3 is closest in meaning to ____________. A. interesting B. informative C. correct D. simple 5. Which of the following will be a leap year? A. 2300 B. 2400 C. 2200 D. 2500 6. Which of the following is true of the Gregorian calendar? A. It needs major improvements. B. It was so well designed, it needs little adjusting today. C. It copied the Roman calendar’s formula of leap years. D. none of the answers 7. Why is Caesar important in calendar making? A. He changed the length of the year . B. He extended summer. C. He has a month named for him. D. He altered the number of days in the year. 8. In what order is the information in the passage presented? A. Caesar’s calendar, the Gregorian calendar, the modern calendar B. Roman Gods, important festivals, original numbers of months C. names of months, Caesar’s calendar, the Gregorian calendar D. none of the answers 9. The word “refined” in paragraph 3 is closest in meaning to ____________. A. studied B. invented C. observed D. improved 10. Why is the number of 364 and a quarter important? A. It is the length of time from the beginning of spring to the end of winter. B. It is the length of a planetary year. C. It is the most accurate number for calendars. D. It was a number randomly chosen by Caesar for his calendar. II. Complete the following passage by choosing the correct option (A, B, C or D) to fill in blanks. The Alexandra Palace in north London was built with private funds as a “People’s Palace”. Serviced by its own station, it was opened in 1873 and was extremely well (1)_______ until, two weeks after its opening, it burnt down. It was replaced by a slightly larger building which opened in 1875 and featured, (2)________other things, a splendid organ an Great Hall, which was the size of a football pitch. Despite the extraordinarily wide range of events (3)_______ there – from dog shows to great concerts and banquets, from elephant displays to bicycle matches – it always operated at a loss and by 1877 much of the park around it had been sold to speculative builders, leaving only about half of the original land. In 1900, a committee was appointed, whose principal duty was to run the palace and park “for the free use of the people forever.” There were, however, (4) ________ to charge for entry so that the substantial costs could be (5) _______ . The Palace continued, with (6) ________ degrees of success, as BY ĐỖ BÌNH – THPT LIẾN SƠN, LẬP THẠCH, VĨNH PHÚC www.violet.vn/quocbinh72 an entertainment centre. In the 1930s, it was probably most (7) ________ for being the home of the world’s first high definition television broadcasts. In 1980 the building was once more devastated by fire and (8) _______ to a ruin. It was then decided to (9) _________ it and to create a major exhibition centre with community (10)_______, such as a restaurant and a health club. 1. A. inhabited B. attended C. crowded D. visited 2. A. among B. between C. from D. around 3. A. performed B. set C. staged D. laid 4. A. powers B. terms C. allowances D. authorities 5. A. fulfilled B. covered C. matched D. made 6. A. unsteady B. varying C. altering D. unsettled 7. A. distinct B. marked C. considerable D. notable 8. A. turned B. converted C. reduced D. wrecked 9. A. recover B. revise C. restore D. reform 10. A. facilities B. conveniences C. supplies D. appliances III. Read the text below and think of the word which best fits each space. Use only ONE word in each space. All tournament chess games are played with a chess clock – that is two clocks joined together. When one player makes his move, he presses the button which stops his clock and starts his opponent’s clock. (1)_________fails to keep the time limit, no (2)________ what the position on the board, loses the game. Weekend tournaments with a fast time limit and long sessions of play of (3)________ to twelve hours a day are very strenuous and result in fatigue and time troubles. The play is quite sharp. Active, attacking chess is the (4)________ of the day and it is difficult to maintain (5)_________ sustained, precise defense against such play. A score of the game must be (6)_________ as play goes on. Each move is written down on a score sheet, which has to be handed to the tournament officials at the end of each round. The only thought in everybody’s head is to win. Talent and youth – that’s (7)_________ is needed for success at chess, with the (8)________ on youth. Some approach the board with a slow, purposeful manner (9)________ giving you a second glance – you simply don’t count. The seem to imply that the outcome is a foregone conclusion for them; you (10)_______ need to accept it with good grace. PART D. WRITING I. Complete the second sentence so that it has the similar meaning to the first sentence. Write your answers on your answer sheet. 1. Mr. Smith knew little about the Internet, so he didn’t invest into any computer companies. Had _______________________________________________. 2. When she got to the party, everyone was dancing and singing. On _______________________________________________. 3. Barbara runs a successful company and she also manages to look after her five children. Not only __________________________________________. 4. Because of the traffic jam, Andrew couldn't get to the interview on time. The traffic jam ______________________________________. 5. This old car probably won’t last for more than three years. It’s unlikely ________________________________________. II. Write a passage of about 250 words to show your opinion on the following statement: “Some people believe that children’s leisure activities must be educational, otherwise they are a complete waste of time” ---------------THE END--------------BY ĐỖ BÌNH – THPT LIẾN SƠN, LẬP THẠCH, VĨNH PHÚC www.violet.vn/quocbinh72 Thí sinh không được sử dụng tài liệu. Cán bộ coi thi không giải thích gì thêm. Họ và tên thí sinh:…………………….………..…….…….….….; Số báo danh…………………… SỞ GD&ĐT VĨNH PHÚC HƯỚNG DẪN CH ẤM (Đề thi gồm 02 trang) KỲ THI CHỌN HSG LỚP 10 THPT NĂM HỌC 2013-2014 ĐỀ THI MÔN: TIẾNG ANH (Dành cho học sinh THPT chuyên) Thời gian thi: 180 phút, không kể thời gian giao đề Total: 100pts PART A: LISTENING I. You’ll hear two students, Ann and Chris talking about post graduate courses in music. Listen and give short answer to the questions from 1 to 10. (5 pts) 1. The Henry Music Institute 2. a face-to-face interview/ an interview 3. 6,000 pound a year 4. the charges for applying/ application fee 5. January 30th Which facilities do the colleges have? Choose five answers from the box and write the correct letter, A – G, next to questions 6-10. (5pts) 6.E 7.B 8.C 9.G 10. D II. You will hear a conversation about technology. Write NO MORE THAN THREE WORDS AND/OR A NUMBER for each answer. (10pts) 1. satellites 6. time consumption 2. efficiency 7. (shades of) grey 3. (the) sea ice 8. (poor) Internet connection 4. radar 9. compress 5. leads 10. laptop PART B. LEXICO - GRAMMAR I. Complete each of the following sentences with the correct answer (A, B, C or D). (10pts) 1. B 2. D 3. C 4. A 5. D 6. A 7. C 8. C 9. B 11. A 12. A 13. D 14. D 15. A 16. D 17. A 18. D 19. B 10. D 20. C II. There are 10 mistakes in the following passage. Find and then correct them. You should also write the line of the mistakes. (10pts) Qs Mistake Correction 1 like as 2 on up 3 where whose 4 made won 5 or and 6 social socially 7 another other 8 ranged ranging 9 children children’s 10 some any BY ĐỖ BÌNH – THPT LIẾN SƠN, LẬP THẠCH, VĨNH PHÚC www.violet.vn/quocbinh72 III. Supply the correct form of the word in capital letter. Write your answers on your answer sheet. (5 pts) 1. increasingly 2. accessible 3. personal 4. ownership 5. pressure 6. national 7. addition 8. protection 9. undesirable 10. traditional IV. Complete the following sentences with one preposition/particle for each blank. (5 pts) 1. out 2. up 3. on 4. over 5. back PART C. READING I. Read the following passage and choose the best answer for each question. Identify your answer by writing the corresponding letter A, B, C or D on your answer sheet. (10pts) 1. A 2. A 3. C 4. C 5. B 6. B 7. A 8. C 9. D 10. B II. Complete the following passage by choosing the correct option (A, B, C or D) to fill in blanks. (10 pts) 1. B 2. A 3, C 4, A 5. B 6. B 7. D 8. C 9. C 10. A III. Read the text below and think of the word which best fits each space. Use only ONE word in each space. Write your answers on your answer sheet. (10pts) 1. whoever/who 2. matter 3. up 4. order 5. a/any 6. kept 7. what 8. emphasis 9. without 10. only PART D: WRITING I. Complete the second sentence so that it has the similar meaning to the first sentence. (5pts) 1. Had Mr. Smith known something / more / a little more / some more about the Internet, he would have invested into some computer companies. 2. On her arrival at the party, everyone was dancing and singing. 3. Not only does Barbara run a successful company, but she also manages to look after her five children /but she manages to look after her five children as well. 4. The traffic jam prevented Andrew from getting to the interview on time. 5. It is unlikely that this old car will last for more than three years. II. Write a passage of about 250 words to show your opinion on the following statement: “Some people believe that children’s leisure activities must be educational, otherwise they are a complete waste of time” (15pts) Marking scheme The impression mark given is based on the following scheme: 1. Content: 50% of total mark: a provision of all main ideas and details as appropriate 2. Language: 30% of total mark: a variety of vocabulary and structures appropriate to the level of English language gifted upper-secondary school students 3. Presentation: 20% of total mark: coherence, cohesion, and style appropriate to the level of English language gifted upper-secondary school students. BY ĐỖ BÌNH – THPT LIẾN SƠN, LẬP THẠCH, VĨNH PHÚC www.violet.vn/quocbinh72 ---------------THE END------------- BY ĐỖ BÌNH – THPT LIẾN SƠN, LẬP THẠCH, VĨNH PHÚC www.violet.vn/quocbinh72 SỞ GD&ĐT ....... ———————— ĐỀ ĐỀ XUẤT KỲ THI CHỌN HSG LỚP 10 - THPT NĂM HỌC 2014- 2015 ĐỀ THI MÔN: TIẾNG ANH ———————————— PART II. GRAMMAR AND VOCABULARY I. Complete the following sentences by choosing the correct answer among four options (A, B, C or D). (15 pts) 1. He's really shy _______ girl. A. by B. at C. for D. with 2. The teacher _______ her to improve her drawing. A. insisted B. encouraged C. made D. persisted 3. I couldn't quite ______ what they were doing because they were so far away. A. bear out B. make out C. think out D. try out 4. The meal Mary cooked tastes_______. A. well B. nice C. good D. worse 5. ______ at the party, we saw Ruth standing alone. A. Arriving B. We arrived C. Arrived D. We were arriving 6. The people who______ the survey said that they had examined over 1,000 accidents. A. gave B. proceed C. set D. conducted 7. The judge found him ______ of stealing and sent him to prison. A. evil B. innocent C. guilty D. wicked 8. The house we have rented is______. So we will have to buy some beds, chairs, tables, etc. A. unrestored B. unrepaired C. unfurnished D. undecorated 9. He was turned down for the job because he is ________. A. qualified B. qualifying C. unqualified D. qualification 10. The trouble started only______ the other man came into the room. A. when B. until C. and then D. too soon 11. _______, the disaster would not have happened. A. Had you have obeyed the orders B. You had obeyed the orders C. You obeyed the orders D. Had you obeyed the orders 12. _______ had booked in advance were allowed in. A. Only who B. Only those who C. Only who were those D. Only were those who 13. Traveling alone to a jungle is adventurous, ________. A. if not impossible B. if it not impossible C. when not impossible D. when it not impossible 14. I ______ the hot weather in the south. A. use to B. used to C. am use to D. am used to 15. The meat looked very _______ to the dog. A. invited B. invite C. inviting D. invitingly II. Use the correct form of each word on the right to complete the numbered spaces provided in the passage. Write your answers on your answer sheet. (10 pts) The mysteries of the skies Three hundred and fifty years before the first men looked down on the amazingly beautiful 1. ABLE surface of the moon from close quarters, Galileo’s newly built telescope (1) _____ him to look at 2. LIVE the edge of the hitherto mysterious sphere. He saw that the apparently (2) _____ surface was not 3. ACT divinely smooth and round, but bumpy and imperfect. He realized that although the moon might 4. ART appear (3) ___, resembling a still life painted by the hand of a cosmic (4) _____, it was a real 5. ACHIEVE world, perhaps not very different from our own. This amounted to a great (5) ____ hardly to be 6. expected in his day and age, although nowadays his (6) ___ may appear to some to be trivial and CONCLUDE (7) ______. 7. SIGNIFY Not long after Galileo lunar’s observations, the skies which had previously been so (8) ______ 8. ELUDE revealed more of their extraordinary mysteries. Casting around for further wonders, Galileo focused his lens on the (9) _____ planet of Jupiter. Nestling next to it, he saw four little points of 9. STRIKE light circling the distant planet. Our moon it appeared, perhaps (10) _____ in the eyes of those 10. fearful of what the discovery might mean, was not alone! BY ĐỖ BÌNH – THPT LIẾN SƠN, LẬP THẠCH, VĨNH PHÚC www.violet.vn/quocbinh72 FORTUNE III. In the following passage, some numbered lines contain a word that shouldn’t be there. Tick (√) the sentences that are correct and write the words that shouldn’t be there in the numbered space. (10 pts) KEEPING YOUR DISTANCE Personal space is a term that refers to the distance we like to keep between 0 ___√___ ourselves and other people. When someone we do not know well gets too close 00 someone that we usually begin to feel uncomfortable. If such a business colleague comes 1________ closer than 1.2 meters, the most common response is to move away. Some 2 ________ interesting studies have been done in libraries. If strangers will come too close, 3 ________ many people get up and leave the building, others use to different methods such as 4 ________ turning their back on the intruder. Living in cities has made people to develop new 5 ________ skills for dealing with situations where they are very close to strangers. Most 6 ________ people on so crowded trains try not to look at strangers; they avoid skin contract, 7 ________ and apologize if hands touch by a mistake. People use newspapers as a barrier 8 ________ between themselves and other people, and if they do not have one, they stare into 9 ________ 10 _______ the distance, making sure they are not looking into anyone’s eyes. PART III. READING I. Complete the following passage by choosing A, B, C or D to fill in each blank. (10 pts) In recent years, there has been a remarkable increase into happiness. The researchers have come up a number of factors which contribute to a definition of happiness. First of all, there is, in some people, a moderate genetic predisposition to be happy, in other words, happiness (1)_______ in families. And happiness seems to correlate quite strongly with the main dimensions of personalities: extroverts are generally happier, neurotics are less so. Second, people often report good social relations as a reason for their happiness. In particular, friends are a great (2) ______ of joy, partly because of the agreeable things they do together, partly because of the way friends use positive nonverbal (3) ______ such as caressing and touching, to affirm their friendship. Marriage and similar (4) ______ relationships can also form the basis of lasting happiness. Third, job satisfaction undoubtedly (5) ______ overall satisfaction, and vice versa - perhaps this is why some people are happy in boring jobs: it (6) ______ both ways. Job satisfaction is caused not only by the essential nature of the work, but (7)_____ by social interactions with co-workers. Unemployment, on the contrary, can be a serious cause of unhappiness. Fourth, leisure is important because it is more under individual (8) ______ than most other causes of happiness. Activities (9) _____ sport and music, and participation in voluntary work and social clubs of various kinds, can give great joy. This is partly because of the (10) ______themselves, but also because of the social support of other group members – it is very strong in the case of religious groups. 1. A. runs B. arrives C. goes D. descends 2. A. source B. origin C. base D. meaning 3. A. movements B. signals C. slogans D. motions 4. A. near B. tight C. close D. heavy 5. A. consists of B. applies to C. counts on D. contributes to 6. A. works B. effects C. makes D. turns 7. A. too B. as well C. also D. plus 8. A. check B. power C. choice D. control 9. A. so B. such C. like D. thus 10. A. facilities B. activities C. exercises D. amenities II. Read the passage carefully, then choose the correct option (marked A, B, C or D) to answer the questions. (10 pts) Scientists have established that influenza viruses taken from man can cause disease in animals. In addition, man can catch the disease from animals. In fact, a greater numbers of wild birds seem to carry the virus without showing any evidences of illness. Some scientists conclude that a large family of influenza virus may have evolved in the bird kingdom, a group that has been on earth 100 million years and is able to carry the virus without contracting the disease. There is even convincing evidence to show that virus strain are transmitted from place to place and from continent to continent by migrating birds. BY ĐỖ BÌNH – THPT LIẾN SƠN, LẬP THẠCH, VĨNH PHÚC www.violet.vn/quocbinh72 It is known that two influenza viruses can recombine when both are present in an animal at the same time. The result of such recombination is a great variety of strains containing different H and N spikes. This raises the possibility that a human influenza virus can recombine with an influenza virus from a lower animal to produce an entirely new spike. Research is underway to determine if that is the way major new strains come into being. Another possibility is that two animal influenza strains may recombine in a pig, for example, to produce a new strain which is transmitted to man. 1. According to the passage, scientists have discovered that influenza viruses ______. A. cause ill health in wild animals B. do not always cause symptoms in birds C. are rarely present in wild birds D. change when transmitted from animals to man 2. What is known about the influenza virus? A. It was first found in a group of very old birds. B. All the different strains can be found in wild birds. C. It existed over 100 million years ago. D. It can survive in many different places. 3. According to the passage, a great variety of influenza strains can appear when______. A. H and N spikes are produced B. animal and bird viruses are combined C. dissimilar types of viruses recombine D. two viruses of the same type are contracted 4. New strains of viruses are transmitted to man by_______. A. a type of wild pig B. diseased lower animals C. a group of migrating birds D. a variety of means 5. It can be inferred from the passage that all of the following are ways of producing new strains of influenza EXCEPT___. A. two influenza viruses in the same animal recombining B. animal viruses recombining with human viruses C. two animal viruses recombining D. two animal viruses recombining in a human III. Read the passage and choose the best answer from the four options marked A, B, C or D in the following questions. Identify your answer by writing the corresponding letter A, B, C or D on your answer sheet. (10 pts) Several hundred million years ago, plants similar to modern ferns covered vast stretches of the land. Some were as large as trees, with giant fronds bunched at the top of trunks as straight as pillars. Others were the size of bushes and formed thickets of undergrowth. Still others lived in the shade of giant club mosses and horsetails along the edges of swampy lagoons where giant amphibians swam. A great number of these plants were true ferns, reproducing themselves without fruits or seeds. Others had only the appearance of ferns. Their leaves had organs of sexual reproduction and produced seeds. Although their “flowers” did not have corollas, these false ferns (today completely extinct) ushered in the era of flowering plants. Traces of these floras of the earliest times have been preserved in the form of fossils. Such traces are most commonly found in shale and sandstone rocks wedged between coal beds. Today only tropical forests bear living proof of the ancient greatness of ferns. The species that grow there are no longer those of the Carboniferous period, but their variety and vast numbers, and the great size of some, remind us of the time when ferns ruled the plant kingdom. 1. What does the passage mainly discuss? A. Plant reproduction B. How to locate fossils C. An ancient form of plant life D. Tropical plant life 2. The word “Others” refers to _________. A. plants B. pillars C. trees D. fronds 3. Which of the following is NOT mentioned as a characteristic of the plants described in the passage? A. They once spread over large areas of land. B. They varied greatly in size. C. They coexisted with amphibians, mosses, and horsetails. D. They clung to tree trunks and bushes for support. 4. The word “true” is closest in meaning to which of the following? A. accurate B. genuine C. straight D. dependable 5. The author states that fossils of early plant life are usually found in rocks located between deposits of _______. A. coal B. shale C. sandstone D. corollas IV. Read the passage carefully then fill in the blank a suitable word. (15 pts) As swimming became a popular recreation in England during the 1860s and 1870s, several (1) ______ sports developed, roughly patterned after land sports. (2) ______ them were water football (or soccer), water rugby, water handball, and water polo, in which players rode on floating barrels, painted to look (3) ______ horses, and struck the ball with a stick. Water rugby became most popular of these sports, but somehow the water polo name became attached to it, and it's been attached (4) ______ since. BY ĐỖ BÌNH – THPT LIẾN SƠN, LẬP THẠCH, VĨNH PHÚC www.violet.vn/quocbinh72 As played in England, the object of the sport was for a player to touch the ball, with both (5) ______, at the goal end of the pool. The goaltender stood on the pool deck, ready to dive on any opponent who was about to score. Water polo quickly became a very rough sport, filled (6) ______ underwater fights away from the ball, and it wasn't unusual for players to pass out for lack of air. In 1877, the sport was tamed in Scotland by the addiction of goalposts. The Scots also replaced (7) ______ original small, hard rubber ball with a soccer ball and adopted (8) ______ that prohibited taking the ball under the surface or, "tackling" a player unless he had the ball. The Scottish game, which emphasized swimming speed, passing, and (9) ______ work, spread to England during the early 1880s, to Hungary in 1889, to Austria and Germany in 1894, to France in 1895, and (10) ______ Belgium in 1900. Water polo was the first team sport added to the Olympic program, in 1900. PART IV: WRITING I. Write the new sentences using the given word. Do not change the word given in any way. (10 pts) 1. They have discovered some interesting new information. (LIGHT) 2. They suspended Jack for the next two matches. (BANNED) 3. I really want to see her again. (DYING) 4. She was so beautiful that I couldn't stop looking at her. (EYES) 5. We are looking forward to watching the program. (WAIT) II. Rewrite each of the following sentences so that it has a similar meaning to the original one. (10 pts) 1. If you changed your mind, you would be welcomed to join our class. → Were you______________________________________________________ 2. I'd rather not go out this afternoon. → I do not feel____________________________________________________ 3. Adeles tries hard, but she doesn't get anywhere. → However______________________________________________________ 4. It is thought that the boss is considering raising wages. → The boss______________________________________________________ 5. His disabilities did not prevent him from sailing around the world. → Despite the fact_________________________________________________ 6. I didn't arrive in time to see her. → I wasn't_______________________________________________________ 7. I'd prefer you not to smoke. → I'd rather______________________________________________________ 8. The mother smiled happily. She took the baby in her arms. → Smiling________________________________________________________ 9. The noise next door did not stop until after midnight. → It was not ______________________________________________________ 10. You can ring this number whenever there is any difficulty. → Should _________________________________________________________ THE END BY ĐỖ BÌNH – THPT LIẾN SƠN, LẬP THẠCH, VĨNH PHÚC www.violet.vn/quocbinh72 SỞ GD&ĐT VĨNH PHÚC ----------------ĐỀ CHÍNH THỨC KỲ THI CHỌN HSG LỚP 1…. THPT NĂM HỌC 201…-201…. (ĐỀ SỐ 07) ĐỀ THI MÔN: TIẾNG ANH PART II. GRAMMAR AND VOCABULARY I. Complete the following sentences by choosing the correct answer among four options (A, B, C or D). (15 pts) 1. He's really shy _______ girl. A. by B. at C. for D. with 2. The teacher _______ her to improve her drawing. C. made D. persisted A. insisted B. encouraged 3. I couldn't quite ______ what they were doing because they were so far away. A. bear out B. make out C. think out D. try out 4. The meal Mary cooked tastes_______. A. well B. nice C. good D. worse 5. ______ at the party, we saw Ruth standing alone. B. We arrived C. Arrived D. We were arriving A. Arriving 6. The people who______ the survey said that they had examined over 1,000 accidents. A. gave B. proceed C. set D. conducted 7. The judge found him ______ of stealing and sent him to prison. D. wicked A. evil B. innocent C. guilty 8. The house we have rented is______. So we will have to buy some beds, chairs, tables, etc. A. unrestored B. unrepaired C. unfurnished D. undecorated 9. He was turned down for the job because he is ________. D. qualification A. qualified B. qualifying C. unqualified 10. The trouble started only______ the other man came into the room. A. when B. until C. and then D. too soon 11. _______, the disaster would not have happened. A. Had you have obeyed the orders B. You had obeyed the orders C. You obeyed the orders D. Had you obeyed the orders 12. _______ had booked in advance were allowed in. A. Only who B. Only those who C. Only who were those D. Only were those who 13. Traveling alone to a jungle is adventurous, ________. B. if it not impossible C. when not impossible D. when it not impossible A. if not impossible 14. I ______ the hot weather in the south. A. use to B. used to C. am use to D. am used to 15. The meat looked very _______ to the dog. A. invited B. invite C. inviting D. invitingly II. Use the correct form of each word on the right to complete the numbered spaces provided in the passage. Write your answers on your answer sheet. (10 pts) The mysteries of the skies Three hundred and fifty years before the first men looked down on the amazingly beautiful surface of the moon from close quarters, Galileo’s newly built telescope (1) _ enabled _ him to 1. ABLE look at the edge of the hitherto mysterious sphere. He saw that the apparently (2) _lifeless_ surface 2. LIVE was not divinely smooth and round, but bumpy and imperfect. He realized that although the moon 3. ACT might appear (3) inactive _, resembling a still life painted by the hand of a cosmic (4) artist__, it 4. ART was a real world, perhaps not very different from our own. This amounted to a great (5) _ 5. ACHIEVE BY ĐỖ BÌNH – THPT LIẾN SƠN, LẬP THẠCH, VĨNH PHÚC www.violet.vn/quocbinh72 achievement _ hardly to be expected in his day and age, although nowadays his (6) conclusion_ may appear to some to be trivial and (7) _ insignificant _. Not long after Galileo lunar’s observations, the skies which had previously been so (8) elusive__ revealed more of their extraordinary mysteries. Casting around for further wonders, Galileo focused his lens on the (9) _striking_ planet of Jupiter. Nestling next to it, he saw four little points of light circling the distant planet. Our moon it appeared, perhaps (10) _ unfortunately _ in the eyes of those fearful of what the discovery might mean, was not alone! 6. CONCLUDE 7. SIGNIFY 8. ELUDE 9. STRIKE 10. FORTUNE III. In the following passage, some numbered lines contain a word that shouldn’t be there. Tick (√) the sentences that are correct and write the words that shouldn’t be there in the numbered space. (10 pts) KEEPING YOUR DISTANCE Personal space is a term that refers to the distance we like to keep between 0 ___√___ ourselves and other people. When someone we do not know well gets too close 00 someone that we usually begin to feel uncomfortable. If such a business colleague comes 1__ that ___ closer than 1.2 meters, the most common response is to move away. Some 2 __such__ interesting studies have been done in libraries. If strangers will come too close, 3 __√___ many people get up and leave the building, others use to different methods such as 4 __will____ turning their back on the intruder. Living in cities has made people to develop new 5 ___to___ skills for dealing with situations where they are very close to strangers. Most 6 __to___ people on so crowded trains try not to look at strangers; they avoid skin contract, 7 __√___ and apologize if hands touch by a mistake. People use newspapers as a barrier 8 __so_____ between themselves and other people, and if they do not have one, they stare into 9 ___a___ 10 __√___ the distance, making sure they are not looking into anyone’s eyes. PART III. READING I. Complete the following passage by choosing A, B, C or D to fill in each blank. (10 pts) In recent years, there has been a remarkable increase into happiness. The researchers have come up a number of factors which contribute to a definition of happiness. First of all, there is, in some people, a moderate genetic predisposition to be happy, in other words, happiness (1)_______ in families. And happiness seems to correlate quite strongly with the main dimensions of personalities: extroverts are generally happier, neurotics are less so. Second, people often report good social relations as a reason for their happiness. In particular, friends are a great (2) ______ of joy, partly because of the agreeable things they do together, partly because of the way friends use positive nonverbal (3) ______ such as caressing and touching, to affirm their friendship. Marriage and similar (4) ______ relationships can also form the basis of lasting happiness. Third, job satisfaction undoubtedly (5) ______ overall satisfaction, and vice versa - perhaps this is why some people are happy in boring jobs: it (6) ______ both ways. Job satisfaction is caused not only by the essential nature of the work, but (7)_____ by social interactions with co-workers. Unemployment, on the contrary, can be a serious cause of unhappiness. Fourth, leisure is important because it is more under individual (8) ______ than most other causes of happiness. Activities (9) _____ sport and music, and participation in voluntary work and social clubs of various kinds, can give great joy. This is partly because of the (10) ______themselves, but also because of the social support of other group members – it is very strong in the case of religious groups. B. arrives C. goes D. descends 1. A. runs 2. A. source B. origin C. base D. meaning C. slogans D. motions 3. A. movements B. signals 4. A. near B. tight C. close D. heavy 5. A. consists of B. applies to C. counts on D. contributes to 6. A. works B. effects C. makes D. turns 7. A. too B. as well C. also D. plus 8. A. check B. power C. choice D. control 9. A. so B. such C. like D. thus 10. A. facilities B. activities C. exercises D. amenities BY ĐỖ BÌNH – THPT LIẾN SƠN, LẬP THẠCH, VĨNH PHÚC www.violet.vn/quocbinh72 II. Read the passage carefully, then choose the correct option (marked A, B, C or D) to answer the questions. (10 pts) Scientists have established that influenza viruses taken from man can cause disease in animals. In addition, man can catch the disease from animals. In fact, a greater numbers of wild birds seem to carry the virus without showing any evidences of illness. Some scientists conclude that a large family of influenza virus may have evolved in the bird kingdom, a group that has been on earth 100 million years and is able to carry the virus without contracting the disease. There is even convincing evidence to show that virus strain are transmitted from place to place and from continent to continent by migrating birds. It is known that two influenza viruses can recombine when both are present in an animal at the same time. The result of such recombination is a great variety of strains containing different H and N spikes. This raises the possibility that a human influenza virus can recombine with an influenza virus from a lower animal to produce an entirely new spike. Research is underway to determine if that is the way major new strains come into being. Another possibility is that two animal influenza strains may recombine in a pig, for example, to produce a new strain which is transmitted to man. 1. According to the passage, scientists have discovered that influenza viruses ______. A. cause ill health in wild animals B. do not always cause symptoms in birds C. are rarely present in wild birds D. change when transmitted from animals to man 2. What is known about the influenza virus? A. It was first found in a group of very old birds. B. All the different strains can be found in wild birds. C. It existed over 100 million years ago. D. It can survive in many different places. 3. According to the passage, a great variety of influenza strains can appear when______. A. H and N spikes are produced B. animal and bird viruses are combined C. dissimilar types of viruses recombine D. two viruses of the same type are contracted 4. New strains of viruses are transmitted to man by_______. A. a type of wild pig B. diseased lower animals C. a group of migrating birds D. a variety of means 5. It can be inferred from the passage that all of the following are ways of producing new strains of influenza EXCEPT___. A. two influenza viruses in the same animal recombining B. animal viruses recombining with human viruses C. two animal viruses recombining D. two animal viruses recombining in a human III. Read the passage and choose the best answer from the four options marked A, B, C or D in the following questions. Identify your answer by writing the corresponding letter A, B, C or D on your answer sheet. (10 pts) Several hundred million years ago, plants similar to modern ferns covered vast stretches of the land. Some were as large as trees, with giant fronds bunched at the top of trunks as straight as pillars. Others were the size of bushes and formed thickets of undergrowth. Still others lived in the shade of giant club mosses and horsetails along the edges of swampy lagoons where giant amphibians swam. A great number of these plants were true ferns, reproducing themselves without fruits or seeds. Others had only the appearance of ferns. Their leaves had organs of sexual reproduction and produced seeds. Although their “flowers” did not have corollas, these false ferns (today completely extinct) ushered in the era of flowering plants. Traces of these floras of the earliest times have been preserved in the form of fossils. Such traces are most commonly found in shale and sandstone rocks wedged between coal beds. Today only tropical forests bear living proof of the ancient greatness of ferns. The species that grow there are no longer those of the Carboniferous period, but their variety and vast numbers, and the great size of some, remind us of the time when ferns ruled the plant kingdom. 1. What does the passage mainly discuss? A. Plant reproduction B. How to locate fossils C. An ancient form of plant life D. Tropical plant life 2. The word “Others” refers to _________. A. plants B. pillars C. trees D. fronds 3. Which of the following is NOT mentioned as a characteristic of the plants described in the passage? A. They once spread over large areas of land. B. They varied greatly in size. C. They coexisted with amphibians, mosses, and horsetails. D. They clung to tree trunks and bushes for support. 4. The word “true” is closest in meaning to which of the following? A. accurate B. genuine C. straight D. dependable 5. The author states that fossils of early plant life are usually found in rocks located between deposits of _______. A. coal B. shale C. sandstone D. corollas IV. Read the passage carefully then fill in the blank a suitable word. (15 pts) BY ĐỖ BÌNH – THPT LIẾN SƠN, LẬP THẠCH, VĨNH PHÚC www.violet.vn/quocbinh72 As swimming became a popular recreation in England during the 1860s and 1870s, several (1) _ water _ sports developed, roughly patterned after land sports. (2) _among__ them were water football (or soccer), water rugby, water handball, and water polo, in which players rode on floating barrels, painted to look (3) __like__ horses, and struck the ball with a stick. Water rugby became most popular of these sports, but somehow the water polo name became attached to it, and it's been attached (4) __ever_ since. As played in England, the object of the sport was for a player to touch the ball, with both (5) _ hands ___, at the goal end of the pool. The goaltender stood on the pool deck, ready to dive on any opponent who was about to score. Water polo quickly became a very rough sport, filled (6) __with_ underwater fights away from the ball, and it wasn't unusual for players to pass out for lack of air. In 1877, the sport was tamed in Scotland by the addiction of goalposts. The Scots also replaced (7) __the_ original small, hard rubber ball with a soccer ball and adopted (8) __rules_ that prohibited taking the ball under the surface or, "tackling" a player unless he had the ball. The Scottish game, which emphasized swimming speed, passing, and (9) __team_ work, spread to England during the early 1880s, to Hungary in 1889, to Austria and Germany in 1894, to France in 1895, and (10) __to_ Belgium in 1900. Water polo was the first team sport added to the Olympic program, in 1900. PART IV: WRITING I. Write the new sentences using the given word. Do not change the word given in any way. (10 pts) 1. They have discovered some interesting new information. (LIGHT) →Some interesting new information has come to light. 2. They suspended Jack for the next two matches. (BANNED) →Jack was banned from playing in the next two matches. 3. I really want to see her again. →I'm dying to see her again. (DYING) 4. She was so beautiful that I couldn't stop looking at her. (EYES) →She was so beautiful that I couldn't take my eyes off her. 5. We are looking forward to watching the program. →We can't wait to watch the program. (WAIT) II. Rewrite each of the following sentences so that it has a similar meaning to the original one. (10 pts) 1. If you changed your mind, you would be welcomed to join our class. → Were you to change your mind, you would be welcomed to join our class. 2. I'd rather not go out this afternoon. → I do not feel like going out this afternoon. 3. Adeles tries hard, but she doesn't get anywhere. → However hard Adeles tries, she doesn't get anywhere/gets nowhere. 4. It is thought that the boss is considering raising wages. → The boss is thought to be considering raising wages. 5. His disabilities did not prevent him from sailing around the world. → Despite the fact that he was disabled, he sailed/managed to sail around the world. 6. I didn't arrive in time to see her. → I wasn't early enough to see her. 7. I'd prefer you not to smoke. → I'd rather you didn't smoke. 8. The mother smiled happily. She took the baby in her arms. → Smiling happily, the mother took the baby in her arms. 9. The noise next door did not stop until after midnight. → It was not until after midnight that the noise next door stopped. 10. You can ring this number whenever there is any difficulty. → Should there is any difficulty, you can ring this number. THE END BY ĐỖ BÌNH – THPT LIẾN SƠN, LẬP THẠCH, VĨNH PHÚC www.violet.vn/quocbinh72 [...]... hảI dơng Kì thi chọn học sinh giỏi tỉnh Lớp 10 thpt năm học 2012 - 2013 -Môn thi : Tiếng Anh Thời gian làm bài: 180 phút (Đề thi gồm 05 trang) đề chính thức Học sinh làm bài vào tờ giấy thi Phần trắc nghiệm: Chỉ cần viết chữ cái A hoặc B, C, D Phần tự luận : Viết đầy đủ theo yêu cầu của bài (Thí sinh không đợc sử dụng bất cứ tài liệu gì.) A LISTENING (15 points) Hớng dẫn phần thi Nghe hiểu:... 0.5 im - Tớnh mch lc v trụi chy (coherence and cohesion) + di (length): 0.5 im 10 S GDT BC LIấU CHNH THC (Gm 06 trang) K THI CHN HSG LP 10, 11 VềNG TNH NM HC 2011 - 2012 * Mụn thi: TING ANH * Lp: 10 (Bng A) * Thi gian: 180 phỳt (Khụng k thi gian giao ) PART 1 LISTENING (2 points) A Listen and circle the best answer 1 This is _ A a survey B an interview C a face-to-face conversation D a chat... Incoherent.Errors showing lack of basic knowledge of English - HT - 2 S GDT BC LIấU CHNH THC (Gm 06 trang) K THI CHN HSG LP 10, 11 VềNG TNH NM HC 2011 - 2012 * Mụn thi: TING ANH * Lp: 10 (Bng B) * Thi gian: 180 phỳt (Khụng k thi gian giao ) PART 1 LISTENING (2 points) A Listen and circle the best answer 1 This is _ A a survey B an interview C a face-to-face conversation D a chat 2 The people talking... - THE END 6 S GDT BC LIấU CHNH THC (Gm 02 trang) K THI CHN HSG LP 10, 11 VềNG TNH NM HC 2011 - 2012 * Mụn thi: TING ANH * Lp: 10 (Bng B) * Thi gian: 180 phỳt (Khụng k thi gian giao ) HNG DN CHM PART 1 LISTENING 2 points Each correct answer gets 0.2p 1 C 2 D 3 B 4 UA675 5 Tokyo 6 Johnstone 7 Bill 8 2 9 555-432 10 3 PART 2 PHONETICS I 1 point Each correct answer gets 0.2p 1 A 2 B 3... Incoherent.Errors showing lack of basic knowledge of English - HT - 2 S GDT BC LIấU CHNH THC (Gm 06 trang) K THI CHN HSG LP 10, 11 VềNG TNH NM HC 2011 - 2012 * Mụn thi: TING ANH * Lp: 10 (Bng B) * Thi gian: 180 phỳt (Khụng k thi gian giao ) PART 1 LISTENING (2 points) A Listen and circle the best answer 1 This is _ A a survey B an interview C a face-to-face conversation D a chat 2 The people talking... - THE END 6 S GDT BC LIấU CHNH THC (Gm 02 trang) K THI CHN HSG LP 10, 11 VềNG TNH NM HC 2011 - 2012 * Mụn thi: TING ANH * Lp: 10 (Bng B) * Thi gian: 180 phỳt (Khụng k thi gian giao ) HNG DN CHM PART 1 LISTENING 2 points Each correct answer gets 0.2p 1 C 2 D 3 B 4 UA675 5 Tokyo 6 Johnstone 7 Bill 8 2 9 555-432 10 3 PART 2 PHONETICS I 1 point Each correct answer gets 0.2p 1 A 2 B 3... - HT 6 S GDT BC LIấU CHNH THC (Gm 02 trang) K THI CHN HSG LP 10, 11 VềNG TNH NM HC 2011 - 2012 * Mụn thi: TING ANH * Lp: 10 (bng A) * Thi gian: 180 phỳt (Khụng k thi gian giao ) HNG DN CHM PART 1 LISTENING Each correct answer gets 0.2p 1 C 2 D 3 B 4 UA675 5 Tokyo 6 Johnstone 7 Bill 8 2 9 555-432 10 3 PART 2 PHONETICS I 1 point Each correct answer gets 0.2p 1 A 2 B 3 C 4...1 B 2 B 3 C 4 C 5 A 6 A 7 A 8 D 9 B 10 D PART IX: 2 im = 0,2 / 1 cõu ỳng 1 D 2 B 3 C 4 C 5 B 6 B 7 D 8 B 9 D 10 A D WRITING 5 IM Part IX: 2 im = 0,2 / 1 cõu ỳng my student has performed in Canada 1 This will be the first time 2 In six months time we will have completed this course 3 More people understand him than he expected./ has expected/ expects 4... yêu cầu của bài (Thí sinh không đợc sử dụng bất cứ tài liệu gì.) A LISTENING (15 points) Hớng dẫn phần thi Nghe hiểu: Bài nghe gồm 2 phần, mỗi phần thí sinh đợc nghe 2 lần Mở đầu và kết thúc mỗi phần nghe có tín hiệu Mọi hớng dẫn cho thí sinh (bằng tiếng Anh) có trong bài nghe Part 1 Listen to a conversation between an optometrist and a patient and fill in the form Write no more than 3 words or numbers... the sentences below (1 point) 1.There are not thing special about his clothes from his flowery tie A but B except C other D apart 2 Id .you explained to her why we cant go A better B rather C want D need 3 I we meet outside the cinema tomorrow at 8.30 A think B suggest C consider D introduce 4 I walked away as calmly as I could they thought I was the thief A or else B to avoid C owing to D in case

Ngày đăng: 28/09/2015, 22:06

Từ khóa liên quan

Tài liệu cùng người dùng

Tài liệu liên quan